ORDINARY AND PARTIAL DIFFERENTIAL EQUATIONS [18 ed.]
 9899107446, 9911310888, 8121908925

Table of contents :
Cover
Title Page
Contents
PART-I ELEMENTARY DIFFERENTIAL EQUATIONS
PART-II ADVANCED ORDINARY DIFFERNTIAL EQUATIONSAND SPECIAL FUNCTIONS
PART-III PARTIAL DIFFERENTIAL EQUATIONS

Citation preview

ORDINARY AND PARTIAL DIFFERENTIAL EQUATIONS [For BA, B.Sc. and Honours (Mathematics and Physics), M.A., M.Sc. (Mathematics and Physics), B.E. Students of Various Universities and for I.A.S., P.C.S., A.M.I.E. GATE, C.S.I.R. U.G.C. NET and Various Competitive Examinations]

Dr. M.D. RAISINGHANIA M.Sc., Ph.D. Formerly Reader and Head, Department of Mathematics S.D. College, Muzaffarnagar, U.P.

S.CHAND & COMPANY LTD. (AN ISO 9001 : 2008 COMPANY) RAMNAGAR, NEW DELHI - 110 055

Created with Print2PDF. To remove this line, buy a license at: http://www.software602.com/

S. CHAND & COMPANY LTD. (An ISO 9001 : 2008 Company)

Head Office: 7361, RAM NAGAR, NEW DELHI - 110 055 Phone: 23672080-81-82, 9899107446, 9911310888 Fax: 91-11-23677446

Shop at: schandgroup.com; e-mail: [email protected] Branches : AHMEDABAD

: 1st Floor, Heritage, Near Gujarat Vidhyapeeth, Ashram Road, Ahmedabad - 380 014, Ph: 27541965, 27542369, [email protected] BENGALURU : No. 6, Ahuja Chambers, 1st Cross, Kumara Krupa Road, Bengaluru - 560 001, Ph: 22268048, 22354008, [email protected] BHOPAL : Bajaj Tower, Plot No. 243, Lala Lajpat Rai Colony, Raisen Road, Bhopal - 462 011, Ph: 4274723. [email protected] CHANDIGARH : S.C.O. 2419-20, First Floor, Sector - 22-C (Near Aroma Hotel), Chandigarh -160 022, Ph: 2725443, 2725446, [email protected] CHENNAI : 152, Anna Salai, Chennai - 600 002, Ph: 28460026, 28460027, [email protected] COIMBATORE : 1790, Trichy Road, LGB Colony, Ramanathapuram, Coimbatore -6410045, Ph: 0422-2323620, 4217136 [email protected] (Marketing Office) CUTTACK : 1st Floor, Bhartia Tower, Badambadi, Cuttack - 753 009, Ph: 2332580; 2332581, [email protected] DEHRADUN : 1st Floor, 20, New Road, Near Dwarka Store, Dehradun - 248 001, Ph: 2711101, 2710861, [email protected] GUWAHATI : Pan Bazar, Guwahati - 781 001, Ph: 2738811, 2735640 [email protected] HYDERABAD : Padma Plaza, H.No. 3-4-630, Opp. Ratna College, Narayanaguda, Hyderabad - 500 029, Ph: 24651135, 24744815, [email protected] JAIPUR : 1st Floor, Nand Plaza, Hawa Sadak, Ajmer Road, Jaipur - 302 006, Ph: 2219175, 2219176, [email protected] JALANDHAR : Mai Hiran Gate, Jalandhar - 144 008, Ph: 2401630, 5000630, [email protected] JAMMU : 67/B, B-Block, Gandhi Nagar, Jammu - 180 004, (M) 09878651464 (Marketing Office) KOCHI : Kachapilly Square, Mullassery Canal Road, Ernakulam, Kochi - 682 011, Ph: 2378207, [email protected] KOLKATA : 285/J, Bipin Bihari Ganguli Street, Kolkata - 700 012, Ph: 22367459, 22373914, [email protected] LUCKNOW : Mahabeer Market, 25 Gwynne Road, Aminabad, Lucknow - 226 018, Ph: 2626801, 2284815, [email protected] MUMBAI : Blackie House, 103/5, Walchand Hirachand Marg, Opp. G.P.O., Mumbai - 400 001, Ph: 22690881, 22610885, [email protected] NAGPUR : Karnal Bag, Model Mill Chowk, Umrer Road, Nagpur - 440 032, Ph: 2723901, 2777666 [email protected] PATNA : 104, Citicentre Ashok, Govind Mitra Road, Patna - 800 004, Ph: 2300489, 2302100, [email protected] PUNE : 291/1, Ganesh Gayatri Complex, 1st Floor, Somwarpeth, Near Jain Mandir, Pune - 411 011, Ph: 64017298, [email protected] (Marketing Office) RAIPUR : Kailash Residency, Plot No. 4B, Bottle House Road, Shankar Nagar, Raipur - 492 007, Ph: 09981200834, [email protected] (Marketing Office) RANCHI : Flat No. 104, Sri Draupadi Smriti Apartments, East of Jaipal Singh Stadium, Neel Ratan Street, Upper Bazar, Ranchi - 834 001, Ph: 2208761, [email protected] (Marketing Office) SILIGURI : 122, Raja Ram Mohan Roy Road, East Vivekanandapally, P.O., Siliguri-734001, Dist., Jalpaiguri, (W.B.) Ph. 0353-2520750 (Marketing Office) VISAKHAPATNAM: Plot No. 7, 1st Floor, Allipuram Extension, Opp. Radhakrishna Towers, Seethammadhara North Extn., Visakhapatnam - 530 013, (M) 09347580841, [email protected] (Marketing Office)

© 1976, M.D. Raisinghania All rights reserved. No part of this publication may be reproduced or copied in any material form (including photo copying or storing it in any medium in form of graphics, electronic or mechanical means and whether or not transient or incidental to some other use of this publication) without written permission of the copyright owner. Any breach of this will entail legal action and prosecution without further notice. Jurisdiction : All disputes with respect to this publication shall be subject to the jurisdiction of the Courts, tribunals and forums of New Delhi, India only. First Edition 1976 Subsequent Editions and Reprints 1991, 95, 97, 98, 2000, 2001, 2002, 2003, 2004, 2005, 2006, 2007, 2008, 2009, 2010, 2011 2012 Fifteenth Revised Edition 2013

ISBN : 81-219-0892-5

Code : 14C 282

PRINTED IN INDIA

By Rajendra Ravindra Printers Pvt. Ltd., 7361, Ram Nagar, New Delhi-110 055 and published by S. Chand & Company Ltd., 7361, Ram Nagar, New Delhi -110 055.

Created with Print2PDF. To remove this line, buy a license at: http://www.software602.com/

PREFACE TO THE FIFTEENTH EDITION Questions asked in recent papers of GATE and various university examinations have been inserted at appropriate places. This enriched inclusion of solved examples and variety of new exercises at the end of each article and chapter makes this book more useful to the reader. While revising this book I have been guided by following simple teaching philosophy : “An ideal text book should teach the students to solve all types of problems”. Any suggestion, remarks and constructive comments for the improvement of this book are always welcome.

PREFACE TO THE SIXTH EDITION

AUTHOR

It gives me great pleasure to inform the reader that the present edition of the book has been improved, well-organised, enlarged and made up-to-date in the light of latest syllabi. The following major changes have been made in the present edition: Almost all the chapters have been rewritten so that in the present form, the reader will not find any difficulty in understanding the subject matter. The matter of the previous edition has been re-organised so that now each topic gets its proper place in the book. More solved examples have been added so that the reader may gain confidence in the techniques of solving problems. References to the latest papers of various universities and I.A.S. examination have been made at proper places. Errors and omissions of the previous edition have been corrected. In view of the above mentioned features it is expected that this new edition will prove more useful to the reader. I am extremely thankful to the Managing Director, Shri Rajendra Kumar Gupta and the Director, Shri Ravindra Kumar Gupta for showing keen interest throughout the publication of the book. Suggestions for further improvement of the book will be gratefully received. AUTHOR

PREFACE TO THE FIRST EDITION This book has been designed for the use of honours and postgraduate students of various Indian universities. It will also be found useful by the students preparing for various competitive examinations. During my long teaching experience I have fully understood the need of the students and hence I have taken great care to present the subject matter in the most clear, interesting and complete form from the student’s point of view. Do not start this book with an unreasonable fear. There are no mysteries in Mathematics. It is all simple and honest reasoning explained step by step which anybody can follow with a little effort and concentration. Often a student has difficulty in following a mathematical explanation only because the author skips steps which he assumes the students to be familiar with. If the student fails to recount the missing steps, he may be faced with a gap in the reasoning and the author’s conclusion may become mysterious to him. I have avoided such gaps by giving necessary references throughout the book. I have been influenced by the following wise-saying. ‘‘My passion is for lucidity. I don’t mean simple mindedness. If people can’t understand it, why write it.’’ AUTHOR

Created with Print2PDF. To remove this line, buy a license at: http://www.software602.com/

Dedicated to the momory of my Parents

Created with Print2PDF. To remove this line, buy a license at: http://www.software602.com/

PART-I ELEMENTARY DIFFERENTIAL EQUATIONS CHAPTERS 1. Differential equations. Their formation and solutions 1.1 Differential equation. Definition 1.2 Ordinary differential equation 1.3 Partial differential equation 1.4 Order of a differential equation 1.5 Degree of a differential equation 1.6 Linear and non-linear differential equations 1.7 Solution of a differential equation. Definition 1.8 Family of curves 1.9 Complete primitive (or general solution). Particular solution and singular solution. Definitions 1.10 Formation of differential equations 1.11 Solved examples based on Art. 1.10 1.12 The Wronskian. Definition 1.13 Linearly dependent and independent set of functions 1.14 Existence and uniqueness theorem 1.14A Some theorems related to Art. 1.14 1.15 Solved examples based on Art. 1.14 and 1.14A 1.16 Some important theorems 1.17 Solved examples based on Art. 1.16 1.18 Linear differetial equation and its general solution Objective problems on chapter 1 2. Equations of first order and first degree 2.1 Introduction 2.2 Separation of variables 2.3 Examples of type-1 based on Art. 2.2 2.4 Transformation of some equations in the form in which variables are separable 2.5 Examples of type-2 based on Art. 2.4 2.6 Homogeneous equations 2.7. Working rule for solving homogeneous equations 2.8 Examples of type-3 based on Art. 2.7 2.9 Equations reducible to homogeneous form 2.10 Examples of type-4 based on Art. 2.9 2.11 Pfaffian differential equation. Definition 2.12 Exact differential equation 2.13 Necessary and sufficient conditions for a differential equation of frst order and first degree to be exact 2.14 Working rule for solving an exact differential equation 2.15 Solved examples of type-5 based on Art. 2.14 (v)

PAGES 1.3–1.35 1.3 1.3 1.3 1.3 1.4 1.4 1.4 1.5 1.5 1.6 1.6 1.10 1.10 1.11 1.12 1.13 1.14 1.22 1.28 1.31 2.1–2.76 2.1 2.1 2.1 2.4 2.5 2.7 2.7 2.8 2.11 2.12 2.16 2.16 2.16 2.17 2.17

Created with Print2PDF. To remove this line, buy a license at: http://www.software602.com/

(vi) 2.16 2.17 2.18 2.19 2.20 2.21 2.22 2.23

Integrating factor. Definition 2.22 Solved examples of type-6 based on rule I 2.23 Solved examples of type-7 based on rule II 2.25 Solved examples of type-8 based on rule III 2.26 Solved examples of type-9 based on rule IV 2.28 Solved examples of type-10 based on rule V 2.29 Solved examples of type-11 based on rule VI 2.30 Linear differential equation 2.32 Working rule for solving linear equations 2.33 2.24 Examples of type-12 based on Art. 2.23 2.33 2.25 Equations reducible to linear form 2.38 2.25A Bernoulli’s equation 2.39 2.26 Examples of type-13 based on Art. 2.25 2.39 2.27 Examples of type-14 based on Art. 2.25A 2.43 2.28 Geometrical meaning of a differential equation of the first order and first degree 2.46 2.29 Applications of equations of first order and first degree 2.46 2.30 List of important results for direct applications 2.46 2.31 Solved examples of type-15 based on Art. 2.30 2.48 2.32 Some typical examples on chapter 2 2.61 Objective problems on chapter 2 2.66 3. Trajectories 3.1–3.16 3.1 Trajectory. Definition 3.1 3.2 Determination of orthogonal trajectories in cartesian co-ordinates 3.1 3.3 Self orthogonal family of curves. Definition 3.2 3.4 Working rule for finding orthogonal trajectories of the given family of cuves in cartesian co-ordinates 3.2 3.5 Solved examples of type-1 based on Art. 3.4 3.2 3.6 Determination of orthogonal trajecories in polar co-ordinates 3.8 3.7 Working rule for getting orthogonal trajectories in polar co-ordinates 3.9 3.8 Solved examples of type-2 based on Art. 3.7 3.9 3.9 Determination of oblique trajectories in cartesian co-ordinates 3.12 3.10 Working rule for finding the oblique trajectories 3.13 3.11 Solved examples of type-3 based on Art. 3.10 3.13 Objective problems on chapter 3 3.14 4. Equations of the first order but not of the first degree singular solutions and extraneous loci 4.1–4.47 PART 4.1 4.2 4.3 4.4 4.5

I: Different methods of finding general solutions Equations of the first order but not of the first degree Method I: Equations solvable for p Solved examples based on Art. 4.2 Method II: Equations solvable for x Solved examples based on Art. 4.4

4.1–4.26 4.1 4.1 4.2 4.6 4.7

Created with Print2PDF. To remove this line, buy a license at: http://www.software602.com/

(vii) 4.6 4.7 4.8 4.9 4.10 4.11 PART 4.12 4.13

Method III: Equations solvable for y Solved examples based on Art. 4.6 Method IV: Equations in Clairaut’s form Solved examples based on Art. 4.8 Method V: Equations reducible to Clairaut’s form Solved examples based on Art. 4.10 II: Singular solutions Introduction

4.11 4.12 4.18 4.19 4.20 4.21 4.26–4.39 4.26

Relation between the singular solution of a differential equation and the envelope of the family of curves represented by that differential equation

4.26

4.14

c-discriminant and p-discriminant relations

4.27

4.15

Determination of singular solutions

4.27

4.16

Working rule for finding the singular solution

4.28

4.17

Solved examples based on singular solutions

4.29

PART 4.18 4.19 4.20 4.21 4.22 4.23

III: Extraneous loci Extraneous loci. Definition The tac locus Node locus Cusp locus4.40 Working rule for finding singular solutions and extraneous loci Solved examples based on Art. 4.22 Objective problems on chapter 4 5. Linear differential equations with constant coefficients PART I: Usual methods of solving linear differential equations with constant coefficients 5.1 Some useful results 5.2 Linear differential equations with constant coefficients 5.3 Determination of complementary function (C.F.) of the given equation 5.4 Working rule for finding C.F. of the given equation 5.5 Solved examples based on Art. 5.4 5.6 The symbolic function 1/f(D). Definition 5.7 Determination of the particular integral (P.I.) of the given equation 5.8 General method of getting P.I.

1

eax !

x n ax e n!

4.39–4.44 4.39 4.39 4.39 4.40 4.41 4.44 5.1–5.70 5.1–5.52 5.1 5.1 5.2 5.4 5.5 5.9 5.9 5.9

5.9

Corollary. If n is a positive integer, then

5.10 5.11 5.12 5.13 5.14 5.15 5.16 5.17

Working rule for finding P.I. 5.11 Solved examples based on Art. 5.10 5.11 Short methods for finding P.I. of f(D)y = X, when X is of certain special form 5.14 Short method of finding P.I. of f (D) y = X, when X = eax 5.14 Working rule for finding P.I. of f (D) y = X, when X = eax 5.14 Solved examples based on Art. 5.14 5.15 Short method of finding P.I. of f (D) y = X, when X = sin ax or cos ax 5. 20 Solved examples based on Art. 5.16 5. 22

( D #) n

5.10

Created with Print2PDF. To remove this line, buy a license at: http://www.software602.com/

(viii) Short method of finding P.I. of f (D) y = X, when X = xm, m being a positive integer 5.19 Solved examples based on Art. 5.18 5.20 Short method of finding P.I. of f (D) y = X, when X = eaxV, where V is any function of x 5.21 Solved examples based on Art. 5.20 5.22 Short method of finding P.I. of f (D) y = X,when X = xV, where V is any function of x. 5.23 Solved examples based on Art. 5.22 5.24 More about particular integral 5.25 Solved examples based on Art. 5.25 and miscellaneous examples on part I of this chapter PART II: Method of undetermined coefficients 5.26 Method of undetermined coefficients for solving linear equations with constant coefficients 5.27 Solved examples based on Art. 5.26 Objective problems on chapter 5 6. Homogeneous linear equations or Cauchy-Euler equations 6.1 Homogeneous linear equation (or Cauchy-Euler equation) 6.2 Method of solution of homogeneous linear differential equations 6.3 Working rule for solving linear homogeneous differential equations 6.4 Solved examples based on Art. 6.3 5.18

5.28 5.28 5.32 5.32 5.40 5.42 5.46 5.46 5.52–5.64 5.52 5.53 5.64 6.1–6.24 6.1 6.1 6.2 6.2

Definition of {1/f (D1)} X, where D1 ∃ d / dz , x = ez and X is any function of x 6.13 6.6A. An alternative method of getting P.I. of homogeneous equations 6.14 6.6B. Particular cases 6.14 6.7 Solved examples based on Art. 6.5 and 6.6A 6.15 6.8 Solved examples based on Art. 6.5 and 6.6B 6.16 6.9 Equations reducible to homogeneous linear form. Legendre’s linear equations 6.18 6.10 Working rule for solving Legendre’s linear equations 6.19 6.11 Solved examples based on Art. 6.10 6.19 Objective problems on chapter 6 6.23 7. Method of variation of parameters 7.1–7.26 7.1 Method of variation of parameters for solving dy/dx + P(x)y = Q(x) 7.1 7.2 Working rule for solving dy/dx + Py = Q by variation of parameters, where P and Q are functions of x or constants. 7.1 7.3 Method of variation of parameters for solving d2y/dx2 + P(x) (dy/dx) + Q(x) = R(x) 7.2 7.4A. Working rule for solving d2y/dx2 + P(dy/dx) + Qy = R by variation of parameters, where P, Q and R are functions of x or constants 7.3 7.5A. Solved examples based on Art. 7.4A 7.3 7.4B. Alternative working rule for solving d2y/dx2 + P(dy/dx) + Qy = R by variation of parameters, where P, Q and R are functions of x or constants. 7.17 6.5

Created with Print2PDF. To remove this line, buy a license at: http://www.software602.com/

(ix) 7.5B. 7.6

Solved examples based on Art. 7.4B 7.17 Working rule for solving d3y/dx3+ P(d2y/dx2) + Q(dy/dx) + Ry = S by variation of parameters, where P, Q, R and S are functions of x or constants 7.23 7.7 Solved examples based on Art. 7.6 7.23 8. Ordinary simultaneous differential equations 8.1–8.25 8.1 Introduction 8.1 8.2 Methods for solving ordinary simultaneous differential equations with constant coefficients 8.1 8.3 Solved examples based on Art. 8.2 8.3 8.4 Solution of simultaneous differential equations involving operators x(d/dx) or t(d/dt) etc 8.21 8.5 Solved examples based on Art. 8.4 8.21 8.6 Miscellaneous examples on chapter 8 8.22 Objective problems on chapter 8 8.24 9. Exact differential equations and equations of special forms 9.1–9.18 9.1 Exact differential equation. Definition 9.1 9.2 Condition of exactness of a linear differential equation of order n 9.1 9.3 Working rule for solving exact equations 9.2 9.4 Examples (Type-1) based on working rule of Art. 9.3 9.2 9.5 Integrating factor 9.7 9.6 Examples (type-2) based on Art. 9.5 9.7 9.7 Exactness of non-linear equations. Solutions by trial 9.9 9.8 Exactness (type-3) based on Art. 9.7 9.9 9.9 Equations of the form dny/dxn = f(x) 9.11 9.10 Examples (type-4) based on Art. 9.9 9.11 9.11 Equations of the form d2y/dx2 = f(y) 9.12 9.12 Examles (Type-5) based on Art. 9.11 9.12 9.13 Reduction of order. Equations that do not contain y directly 9.13 9.14 Examples (Type-6) based on Art. 9.13 9.13 9.15 Equations that do not contain x directly 9.15 9.16 Examples (type-7) based on Art. 9.15 9.15 Objective problems on chapter 9 9.17 10. Linear differential equations of second order 10.1–10.58 10.1 The general (standard) form of the linear differential equation of the second order 10.1 10.2 Complete solution of y%% & Py % & Qy ! R is terms of one known integral belonging to the complementary function (C.F.) Solution of y %% & Py % & Qy ! R by reduction of its order 10.1 Rule for getting an integral belonging to C.F. of y%% & Py % & Qy ! R Working rule for finding complete primitive (solution) when an integral of C.F. is known or can be obtained 10.4A. Theorem related to Art. 10.2 10.4B. Solved examples based on Art. 10.4A 10.5 Solved examples based on Art. 10.4 10.5A. Some typical solved examples 10.3 10.4

10.2 10.2 10.3 10.4 10.6 10.24

Created with Print2PDF. To remove this line, buy a license at: http://www.software602.com/

(x) 10.6 Removal of first derivative. Reduction to normal form. 10.28 10.7 Working rule for solving problems by using normal form 10.29 10.8 Solved examples based on Art. 10.7 10.29 10.9 Transformation of the equation by changing the independent variable 10.39 10.10 Working rule for solving equations by changing the independent variable 10.39 10.11 Solved examples based on Art. 10.10 10.40 10.12 An important theorem 10.47 10.13 Method of variation of parameters 10.48 10.14 Solved examles based on Art. 10.13 10.48 10.15 Solutions by operators 10.55 10.16 Solved examles based on Art. 10.15 10.56 11. Applications of differential equations 11.1–11.27 PART I : Applications of first order differential equations 11.1–11.4 11.1 Introduction 11.1 11.2 Mixture problems 11.1 11.3 Solved examples based on Art. 11.2 11.2 PART II: Applications of second order linear differential equations 11.4–11.25 11.4 Introduction 11.4 11.5 Newton’s second law and Hooke’s law 11.5 116 The differential equation of the vibrations of a mass on a spring 11.5 11.7 Free, undamped motion 11.6 11.8 Free, damped motion 11.8 11.9 Solved examlpes based on Art. 11.8 11.9 11.10 Forced motion 11.12 11.11 Resonance phenomena 11.15 11.12 Elecric circuit problems 11.20 11.13 Solved examples based on Art. 11.12 11.21 PART III: Applications to simultaneous differential equations 11.25–11.27 11.14 Applications to mechanics 11.25 11.15 Solved examles based on Art 11.4 11.25 Miscellaneous problems based on this part of the book M.1-M.8

PART-II ADVANCED ORDINARY DIFFERNTIAL EQUATIONS AND SPECIAL FUNCTIONS CHAPTERS

PAGES

1. Picard’s iterative method. Uniqueness and existence theorem 1.3–1.25 1.1 Introduction 1.3 1.2A. Picard’s method of successive approximation (or Picard’s iteration method) 1.3 1.2B. Solved examples based on Art. 1.2A 1.4 1.3A. Working rule for Picard’s method of solving simulataneous differential equations with initial conditions 1.10 1.3B. Solved examples based on Art. 1.3A 1.10 1.4 Problems of existence and uniqueness 1.14 1.5 Lipschitz condition 1.14

Created with Print2PDF. To remove this line, buy a license at: http://www.software602.com/

(xi) 1.6 Picard’s theorem. Existence and uniqueness theorem 1.15 1.7 An important theorem 1.18 1.8 Solved examples based on Articles 1.4 to 1.7 1.18 2. Simultaneous differential equations of the form (dx)/P = (dy)/Q = (dz)/R 2.1–2.24 2.1 Introduction 2.1 2.2 The nature of solution of (dx)/P = (dy)/Q = (dz)/R 2.1 2.3 Geometrical interpretation of (dx)/P = (dy)/Q = (dz)/R 2.1 2.4 Rule I for solving (dx)/P = (dy)/Q = (dz)/R 2.1 2.5 Solved examples based on Art. 2.4 2.1 2.6 Rule II for solving (dx)/P = (dy)/Q = (dz)/R 2.3 2.7 Solved examples based on Art. 2.6 2.3 2.8 Rule III for solving (dx)/P = (dy)/Q = (dz)/R 2.5 2.9 Solved examples based on Art. 2.8 2.5 2.10 Rule IV for solving (dx)/P = (dy)/Q = (dz)/R 2.12 2.11 Solved examples based on Art. 2.10 2.13 2.12 Orthogonal trajectories of a system of curves on a surface 2.23 2.12A. Solved examples based on Art. 2.12 2.23 3. Total (or Pfaffian) differential equations 3.1–3.32 3.1 Introduction 3.1 3.2 Total differential equation or Pfaffian differential equation 3.1 3.3 Necessary and sufficient conditions for integability of a single differential equation Pdx + Qdy + Rdz = 0 3.1 3.4 The conditions for exactness of Pdx + Qdy + Rdz = 0 3.3 3.5 Method of solving Pdx + Qdy + Rdz = 0 3.4 3.6 Special method I. Solution by inspection 3.4 3.7 Solved examples based on Art. 3.6 3.4 3.8 Special method II. Solution of homogeneous equation 3.12 3.9 Solved examples based on Art. 3.8 3.13 3.10 Special method III. Use of auxiliary equations 3.17 3.11 Solved examples based on Art. 3.10 3.17 3.12 General method of solving Pdx + Qdy + Rdz = 0 by taking one variable as constant 3.19 3.13 Solved examples based on Art. 3.12 3.20 3.14 Solution of Pdx + Qdy + Rdz = 0 when it is exact and homogeneous of degree n ∋ 1. 2.24 3.15 3.16 3.17 3.18 3.19 3.20 3.21 3.22

The non-integrable single equation Working rule for finding the curves represented by the solution of non-integrable total differential equation Solved examples bsed on working rule 3.16 Geometrical interpretation of Pdx + Qdy + Rdz = 0 To show that the locus of Pdx + Qdy + Rdz = 0 is orthogonal to the locus of (dx)/P = (dy)/Q = (dz)/R Total differential equation containing more than three variables Solved examples based on Art. 3.20 Working rule (based on Art. 3.3) for solving Pdx + Qdy + Rdz = 0

3.25 3.25 2.25 3.27 3.27 3.27 3.28 3.31

Created with Print2PDF. To remove this line, buy a license at: http://www.software602.com/

(xii) 4. Riccati’s equation 4.1–4.5 4.1 Introduction 4.1 4.2 General solution of Riccati’s equation 4.1 4.3 The cross-ratio of any four particular integrals of a Riccati’s equation is independent of x 4.2 4.4 Method sof solving Riccati’s equation when three particular integrals are known 4.2 4.5 Method of solving Riccati’s equation when two particular integrals are known 4.3 4.6 Method of solving Riccati’s equation when one particular integral is known 4.4 4.7 Solved examples 4.4 5. Chebyshev polynomials 5.1–5.9 5.1 Chebyshev polynomials 5.1 5.2 Tn(x) and Un(x) are independent solutions of Chebyshev equation 5.1 5.3 Orthogonal properties of Chebyshev polynomials 5.2 5.4 Recurrence relations (formulas) 5.3 5.5 Some theorems on Chebyshev polynomials 5.3 5.6 First few Chebyshev polynomials 5.5 5.7 Generating functions for Chebyshev polynomials 5.6 5.8 Specal values of Chebyshev polynomials 5.7 5.9 Illustrative solved examples 5.8 6. Beta and Gamma functions 6.1–6.22 6.1 Introduction 6.1 6.2 Euler’s integrals. Beta and Gamma functions 6.1 6.3 Properties of Gamma function 6.1 6.4 Extension of definition of Gamma function 6.2 6.5 6.6 6.7 6.8 6.9

To show that ((1/ 2) ! )

Transformation of Gamma function Solved examles based on Gamma function Symmetrical property of Beta function Evaluation of Beta function B(m, n) in an explicit form when m or n is a positive integer 6.10 Transformation of Beta function 6.11 Relation between Beta and Gamma functions 6.12 Solved examples 6.13 Legendre duplication formula 6.14 Solved examlpes 7. Power series 7.1 Introduction 7.2 Summary of useful results 7.3 Power series 7.4 Some important facts about the power series 7.5 Radius of convergence and interval of convergence 7.6 Formulas for determining the radius of convergence 7.7 Solved examples based on Art. 7.6 7.8 Some theorems about power series

6.3 6.3 6.4 6.8 6.8 6.9 6.12 6.15 6.20 6.21 7.1–7.7 7.1 7.1 7.2 7.2 7.2 7.3 7.4 7.6

Created with Print2PDF. To remove this line, buy a license at: http://www.software602.com/

(xiii) 8. Integration in series 8.1 Introduction 8.2 Some basic definitions 8.3 Ordinary and singular points 8.4 Solved examples based on Art. 8.3 8.5 Power series solution in powers of (x – x0) 8.6 Solved examples based on Art. 8.5 8.7 Series solution about regular singular point x = 0. Frobenius method 8.8 Working rule for solution by Frobenius method 8.9 Examples of type-1 based on Frobenius method 8.10 Examples of type-2 based on Frobenius method 8.11 Examples of type-3 based on Frobenius method 8.12 Examples of type-4 based on Fronenius method 8.13 Series solution about regular singular point at infinity 8.14 Solved examples based on Art. 8.13 8.15 Series solution in descending powers of independent variable 8.16 Solved examples based on Art. 8.15 8.17 Method of differentiation Objective problems on chapter 8 9. Legendre polynomials PART I: Legendre function of the first kind 9.1 Legendre’s equation and its solution 9.2 Legendre function of the first kind or Legendre polynomial of degree n 9.3 Generating function for Legendre polynomials 9.4 Solved examples based on Art. 9.2 and Art. 9.3 9.5 Trigonometric series for Pn(x) 9.6 Laplace’s definite integrals for Pn(x) 9.7 Some bounds on Pn(x) 9.8 Orthogonal properties of Legendre’s polynomials 9.9 Recurrence relations (formulas) 9.10 Beltrami’s result 9.11 Christoffel’s summation formula 9.12 Christoffel’s expansion 9.13 Solved examples based on Art. 9.8 and Art. 9.9 9.14 Rodigue’s formula 9.15 Solved examples based on Art. 9.14 9.16 Legendre’s series for f(x), where f(x) is a polynomial 9.17 Solved examples based on Art. 9.16 9.18 Expansion of function f(x) in a series of Legendre polynomials 9.19 Even and odd functions 9.20 Expansion of xn is Legendre polynomials 9.21 Solved examples based on Art. 9.20 Objective problems

8.1–8.60 8.1 8.1 8.2 8.2 8.4 8.4 8.15 8.17 8.18 8.29 8.35 8.44 8.51 8.51 8.55 8.56 8.57 8.58 9.1–9.50 9.1–9.43 9.1 9.3 9.4 9.5 9.10 9.12 9.13 9.14 9.15 9.17 9.17 9.18 9.18 9.26 9.27 9.34 9.35 9.36 9.37 9.38 9.41 9.43

Created with Print2PDF. To remove this line, buy a license at: http://www.software602.com/

(xiv) PART II: Associated Legendre functions of the first kind 9.22 Associated Legendre functions 9.23 Properties of associated Legendre functions 9.24 Orthogonality relations for associated Legendre functions 9.25 Recurrence relations for associated Legendre functions 10. Legendre functions of the second kind 10.1 Some useful results 10.2 Recurrence relations 10.3 Theorem 10.4 Complete solution of Legendre’s equation 10.5 Christoffel’s second summation formula 10.6 A relation connecting Pn(x) and Qn(x) 10.8 Solved examples on chapter 8 11. Bessel functions 11.1 Bessel’s equations and its solution 11.2 Bessel’s function of the first kind of order n 11.3 List of important results of Gamma and Beta functions 11.4 Relation between Jn(x) and J–n(x), n being an integer 11.5 Bessel’s function of the second kind of order n 11.6 Integration of Bessel equation in series for n = 0 Bessel’s function of zeroth order, i.e., J0(x) 11.6A. Solved examples based on Articles 11.1 to 11.6 11.7 Recurrence relations for Jn(x) 11.7A. Solved examples based on Art. 11.7 11.7B. Solved examles involving integration and recurrence relations 11.8 Generating function for the Bessel’s function Jn(x) 11.9 Trigonometric expansions involving Bessel functions 11.9A. Solved examples based on Art. 11.8 and Art. 11.9 11.10 Orthogonality of Bessel functions 11.11 Bessel-series or Fourier-Bessel expansion for f(x) 11.11A. Solved examples based on Art. 11.11 Objective problems on chapter 11 12 Hermite polynomials 12.1 Hermite equation and its solution 12.2 Hermite polynomial of order n 12.3 Generating function for Hermite polynomials 12.4 Alternative expressions for the Hermite polynomials Rodrigues formula for Hermite polynomials 12.5 Hermite polynomials for some special values of n 12.6 Evaluation of values of H2n(0) and H2n+1(0) 12.7 Orthogonality properties of the Hermite polynomials 12.8 Recurrence relations (or formulas) 12.9 Solved examples

9.43–9.50 9.43 9.45 9.46 9.48 10.1–10.12 10.1 10.2 10.5 10.5 10.6 10.7 10.8 11.1–11.45 11.1 11.2 11.3 11.3 11.5 11.5 11.5 11.6 11.7 11.19 11.27 11.31 11.33 11.33 11.40 11.42 11.42 11.44 12.1–12.12 12.1 12.3 12.3 12.3 12.3 12.4 12.5 12.5 12.6. 12.7

Created with Print2PDF. To remove this line, buy a license at: http://www.software602.com/

(xv) 13. Laguerre polynomial 13.1–13.11 13.1 Laguerre equation and its solution 13.1 13.2A. Laguerre polynomial of order (or degree) n 13.2 13.2B. Alternative definition of Leguerre polynomial of order (or degree) n 13.2 13.3 Generating function for Laguerre polynomials 13.2 13.4 Alternating expression for the Laguerre polynomials 13.3 13.5 First few Laguerre polynomials 13.3 13.6 Orthogonal properties of Laguerres polynomials 13.4 13.7 Expansion of a polynomial in a series of Laguerre polynomials 13.5 13.8 Relation between Laguerre polynomial and their derivatives 13.6 13.9 Solved examples 13.7 14. Hypergeometric function 14.1–14.18 14.1 Pochhammer symbol 14.1 14.2 General hypergeometric function 14.1 14.3 Confluent hypergeometric (or Kummer) function 14.1 14.4 Hypergeometric function 14.1 14.5 Gauss’s hypergeometric equation 14.2 14.6 Solution of hypergeometric equation 14.2 14.7 Symmetric property of hypergeometric function 14.3 14.8 Differentiation of hypergeometric function 14.3 14.9 Integral representation for hypergeometric function 14.4 14.10 Gauss theorem 14.5 14.11 Vandermonde’s theorem 14.5 14.12 Kummer’s theorem 14.6 14.13 More about confluent hypergeometric function 14.6 14.14 Differentiation of confluent hypergeometric function 14.8 14.15 Integral representation for confluent hypergeometric function 14.8 14.16 Kummer’s relation 14.9 14.17 Contiguous hypergeometric functions 14.9 14.18 Contiguous relationship 14.9 14.19 Contiguous relationship for confluent hypergeometric function 14.9 14.20 Solved examples 14.10 15. Orthogonal set of functions and Strum-Liouville problem 15.1–15.25 15.1 Orthogonality 15.1 15.2 Orthogonal set of function 15.1 15.3 Orthonormal set of functions 15.1 15.4 Orthogonality with respect to a weight function 15.1 15.5 Orthogonal set of functions with respect to a weight function 15.1 15.6 Orthogonal set of functions with respect to a weight function 15.1 15.7 Working rule for getting orthonormal set 15 . 2 15.8 Gram-Schmidth process of orthonormalization 15 .2 15.9 Illustrative solved examples 15.3 15.10 Strum-Liouville problem 15.10 Eigen (or characteristic) functions and eigen (or characteristic) values 15.10

Created with Print2PDF. To remove this line, buy a license at: http://www.software602.com/

(xvi) 15.11 15.12 15.13 15.14 15.15 15.16 15.17

Orthogonality of eigenfunctions Reality of eigenvalues Solved examples Orthogonality of Legendre polynomials Orthogonality of Bessel functions Orthogonality on an infinite interval Orthogonal expansion or generalised Fourier series Objective problems on chapter 15 Miscellaneous problems based on this part of the book

15.10 15.12 15.13 15. 21 15. 21 15. 22 15. 22 15. 24 M.1-M.4

PART-III PARTIAL DIFFERENTIAL EQUATIONS CHAPTERS PAGES 1. Origin of partial differential equations 1.1 Introduction

1.3–1.19 1.3

1.2

Partial differential equation. Definition

1.3

1.3

Order of a partial differential equation

1.3

1.4

Degree of a partial differential equation

1.3

1.5

Linear and non-linear partial differential equation

1.3

1.6

Notations

1.3

1.7

Classification of first order partial differential equations

1.4

1.8

Origin of partial differential equations

1.4

1.9

Rule I. Derivation of partial differential equations by the elimination of arbitrary constants

1.4

1.10

Solved examples based on rule I of Art. 1.9

1.5

1.11

Rule II. Derivation of partial differential equations by elimination of arbitrary functions ∗ from the equation ∗(u, v) = 0, where u and v are functions of x, y and z

1.12 1.13

Solved examples based on rule II of Art. 1.11 Cauchy’s problem for first order equations Objective problems on chapter 1 2. Linear partial differential equations of order one 2.1 Lagrange’s equations 2.2 Lagrange’s method of solving Pp + Qq = R 2.3 Working rule for solving Pp + Qq = R by Lagrange’s method 2.4 Examples based on working rule of Art. 2.3 2.5 Type 1 based on rule I for solving (dx)/P = (dy)/Q = (dz)/R 2.6 Solved examples based on Art. 2.5 2.7 Type 2 based on rule II for solving (dx)/P = (dy)/Q = (dz)/R 2.8 Solved examples based on Art. 2.7 2.9 Type 3 based on rule III for solving (dx)/P = (dy)/Q = (dz)/R 2.10 Solved examples based on Art. 2.9

1.11 1.11 1.17 1.18 2.1–2. 40 2.1 2.1 2.2 2.2 2.2 2.2 2.4 2.5 2.8 2.8

Created with Print2PDF. To remove this line, buy a license at: http://www.software602.com/

(xvii) 2.11 2.12 2.13 2.14 2.15 2.16 2.17 2.18(a)

Type 4 based on rule IV for solving (dx)/P = (dy)/Q = (dz)/R 2.16 Solved examples based on Art. 2.11 2.16 Miscellaneous examples based on Pp + Qq = R 2.26 Integral surfaces passing through a given curve 2.27 Solved examples based on Art. 2.14 2.27 Surfaces orthogonal to a gvien system of surfaces 2.31 Solved examples based on Art. 2.16 2.31 Geometrical description of the solutions of Pp + Qq = R and the system of equations (dx)/P = (dy)/Q = (dz)/R and relationship between the two 2.33 2.18(b) Another geometrical interpretation of Lagrange’s equation 2.34 2.19 Solved examples based on Art. 2.18(a) and 2.18(b) 2.34 2.20 The linear partial differentil equation with n independent variables and its solutions 2.35 2.21 Solved examples based on Art. 2.20 2.35 3. Non-linear partial differential equations of order one 3.1–3.83 3.1 Complete integral (or complete solution), particular integral, singular integral (or singular solution) and general integral (or general solution) 3.1 3.2 Geometrical interpretation of integrals of f(x, y, z, p, q) = 0 3.2 3.3 Method of getting singular integral directly from the partial differential equation of first order 3.3 3.4 Compatible system of first-order equations 3.3 3.5 A particular case of Art. 3.4 3.4 3.6 Solved examples based on Art. 3.4 and Art. 3.5 3.5 3.7 Charpit’s method 3.11 3.8A. Working rule while using Charpit’s method 3.12 3.8B. Solved examples based on Art. 3.8A 3.12 3.9 Special methods of solutions applicable to certain standard forms 3.33 3.10 Standard form I. Only p and q present 3.33 3.11 Solved examples based on Art. 3.10 3.34 3.12 Standard form II. Clairaut’s equations 3.44 3.13 Solved examles based on Art. 3.12 3.44 3.14 Standard form III. Only p, q and z present 3.49 3.15 Working rule for solving equations of the form f (p, q, z) = 0 3.49 3.16 Solved examples based on Art. 3.15 3.50 3.17 Standard form IV. Equation of the form f1(x, p) = f2(y, q) 3.57 3.18 Solved examples based on Art. 3.17 3.57 3.19 Jacobi’s method 3.63 3.20 Working rule for solving partial differential equations with three or more independent variables. Jacobi’s method 3.64 3.21 Solved examples based on Art. 3.20 3.65 3.22 Jacobi’s method for solving a non-linear first order partial differential equation in two independent variables 3.74 3.23 Cauchy’s method of characteristics for solving non-linear partial differential equations 3.76 3.24 Some theorems 3.79 3.25 Solved examples based on Art. 3.23 and Art. 3.24 3.79

Created with Print2PDF. To remove this line, buy a license at: http://www.software602.com/

(xviii) 4. Homogeneous linear partial differential equations with constant coefficients 4.1–4.34 4.1 Homogeneous and non-homogeneous linear partial differential equations with constant coefficients 4.1 4.2 Solution of a homogeneous linar partial differential equation with constant coefficients 4.1 4.3 Method of finding the complementary function (C.F.) of linear homogeneous partial differential equation with constant coefficients 4.2 4.4A.

Working rule for finding C.F. of linear partial differential equations with constant coefficients 4.4 4.4B. Alternative working rule for finding C.F. 4.4 4.5 Solved examples based on Art. 4.4A and 4.4B 4.5 4.6 Particular integral (P.I.) of homogeneous partial differential equations 4.6 4.7 Short methods of finding P.I. in certain cases 4.7 4.8 Short method I. When f(x, y) is of the form ∗(ax + by) 4.7 4.9 Solved examples based on Art. 4.8 4.9 4.10 Short method II. When f (x, y) is of the form xm yn or a rational integral algebraic function of x and y 4.18 4.11 Solved examples based on Art. 4.10 4.19 4.12 A general method of finding the P.I. of linear homogeneous parial differential equation with constant coefficients 4.25 4.13 Solved examples based on Art. 4.12 4.26 4.14 Solutions under given geometrical conditions 4.32 4.15 Solved examples based on Art. 4.14 4.32 Objective problems on chapter 4 4.34 5. Non-homogeneous linear partial differential equations with constant coefficients 1–30 5.1 Non-homogeneous linear partial differential equation with constant coefficients 5.1 5.2 Reducible and irreducible linear differential operators 5.1 5.3 Reducible and irreducible linear partial differential equations with constant coefficients 5.1 5.4 Theorem. If the operator F ( D, D%) is reducible, then the order in which the linear factors occur is unimportant 5.1 5.5 Determination of complementary function (C.F) of a reducible nonhomogeneous linear partial differential equation with constant coefficients 5.2 5.6 Working rule for finding C.F. of reducible non-homogeneous linear partial differential equations with constant coefficients 5.3 5.7 Solved examples based on Art. 5.6 5.4 5.8 Method of finding C.F. of irreducible linear partial differential equation with constant coefficients 5.5 5.9 Solved examples based on Art. 5.8 5.6 5.10 General solution of non-homogeneous linear partial differential equations with constant coefficients 5.9 5.11 Particulars integral (P.I.) of non-homogeneous linear partical differetial equations 5.9 5.12 Determination of P.I. of non-homogeneous linear partial differential equations (reducible or irreducible) 5.9

Created with Print2PDF. To remove this line, buy a license at: http://www.software602.com/

(xix) 5.13 5.14

Solved examples based on Art. 5.6, Art. 5.8 and Art. 5.12. 5.10 General method of finding P.I. for only reducible non-homogeneous linear partial differential equations with constant coefficients 5.26 5.15 Working rule for finding P.I. of any reducible linear partial differential equations (homogeneous or non-homogeneous) 5.27 5.16 Solved examples based on Art. 5.15 5.28 5.17 Solutions under given geometrical conditions 5.30 6. Partial differential equations reducible to equations with constant coefficients 6.1–6.11 6.1 Introduction 6.1 6.2 Method of reducible Euler-Cauchy type equation to linear partial differential equation with constant coefficients 6.1 6.3 Working rule for solving Euler-Cauchy type equations 6.2 6.4 Solved examples based on Art. 6.3 6.2 6.5 Solutions under given geometical conditions 6.10 7. Partial differential equations of order two with variables coefficients 7.1–7.14 7.1 Introduction 7.1 7.2 Type I 7.1 7.3 Solved examles based on Art 7.2 7.2 7.4 Type II 7.4 7.5 Solved examples based on Art. 7.4 7.4 7.6 Type III 7.7 7.7 Solved examples based on Art. 7.6 7.7 7.8 Type IV 7.11 7.9 Solved examples based on Art. 7.8 7.12 7.10 Solutions of equations under given geometrical conditions 7.12 7.11 Solved examles based on Art. 7.10 7.13 8. Classifiation of partial differential equations Reduction to cononial or normal form. Riemann method 8.1–8.50 8.1 Classification of partial differential equation of second order 8.1 8.2 Classificationof partial differential equations in three independent variables 8.2 8.2A. Solved examples based on Art. 8.2 8.2 8.3 Cauchy’s problem of second order partial differential equations 8.3 8.4 Solved examples based on Art. 8.3 8.4 8.5 Laplace transformation. Reduction to canonical (or normal) form. 8.4 8.6 Working rule for reducing a hyperbolic equation to its canonical form 8.7 8.7 Solved examples based on Art. 8.6 8.7 8.8 Working rule for reducing a parabolic equations to its canonical form 8.23 8.9 Solved examples based on Art. 8.8 8.24 8.10 Working rule for reducing an elliptic equation to its canonical form 8.31 8.11 Solved examples based on Art. 8.10 8.31 8.12 The solution of linear hyperbolic equations 8.36 8.13 Riemann method of solution of general linear hyperbolic equation of second order 8.36 8.14 Solved examples based on Art. 8.13 8.39 8.15 Riemann-Volterra method of solving Cauchy problem for the onedimensional wave-equations 8.44

Created with Print2PDF. To remove this line, buy a license at: http://www.software602.com/

(xx) 9. Monge’s method 9.1–9.51 9.1 Introduction 9.1 9.2 Monge’s method of integrating Rr + Ss + Tt = V 9.1 9.3 Type-I: When the given equation Rr + Ss + Tt = V leads to two distinct intermediate intergrals and both of them are used to get the desired solution 9.3 9.4 Solved examples based on Art. 9.3 9.3 9.5 Type 2: When the given equation Rr + Ss + Tt = V leads to two distinct intermediate integrals and only one is employed to get the desired solution 8.11 9.6 Solved examples based on Art. 9.5 9.12 9.7 Type 3: When the given equation Rr + Ss + Tt = V leads to two identitcal intermediate integrals 9.22 9.8 Solved examples based on Art. 9.7 9.22 9.9 Type 4: When the given equation Rr + Ss + Tt = V fails to yield an intermediate integral as in types 1, 2 and 3 9.30 9.10 Solved examples based on Art. 9.9 9.30 9.11 Monge’s method of integrating Rr + Ss + t + U(rt – s2) = V 9.33 9.12 Type 1: When the roots of +-quadratic are identical 9.34 9.13 Type 2: When the roots of +-quadratic are distinct 9.40 9.14 Miscellaneous examples on Rr + Ss + Tt + U(rt – s2) = V 9.47 10. Transport equation 10.1–10.5 10.1 Introduction 10.1 10.2 An important theorem 10.1 10.3 Generalised or weak solution 10.2 10.4 Transport equation for a linear-hyperbolic system 10.3 Miscellaneous problems based on this part of the book M.1-M.10

Created with Print2PDF. To remove this line, buy a license at: http://www.software602.com/

(xxi)

LIST OF SOME USEFUL RESULTS FOR DIRECT APPLICATIONS I Table of elementary integrals

# sin x dx !

# sinh x dx ! cosh x

cos x

# cos x dx ! sin x

# cosh x dx ! sinh x

# sec

# sech x dx ! tanh x

2

2

x dx ! tan x

# cosec x dx ! 2

# cosech x dx ! 2

cot x

coth x

# sec x tan x dx ! sec x

# sech x tanh x dx !

# cosec x cot x dx !

# cosech x coth x dx !

# e dx ! e x

#

cosec x

cosech x

# a dx ! (a ) / log a

x

x

xn ∃1 ,n% 1 n ∃1

x n dx !

sech x

x

n # & f ( x )∋

1

e

f (( x) dx !

n ∃1

1

f (( x ) dx ! log x f ( x)

# x dx ! log x

#

# tan x dx ! logsec x = – log cos x

# cot x dx ! log sin x

# cosec x dx ! log tan( x / 2) ! log(cosec x

& f ( x)∋ n ∃1 , n %

cot x)

# sec x dx ! log tan () / 4 ∃ x / 2) ! log(sec x ∃ tan x) #x

dx 2

∃a

# x( x

2

!

1 tan a

dx 2

2 1/2

a ) dx

# (a

2

x )

# (a

2

∃ x 2 )1/2

# (x

2

2 1/ 2

dx

dx 2 1/ 2

a )

!

1

x ; a

#x

1 sec a

! sin

1

1

x a

! sinh

1

! cosh

1

x a

dx 2

a

2

!

1 x a log , x ∗ a; 2a x∃a

#a

dx 2

x

2

!

1 a∃ x log ,x+a 2a a x

or

1 cosec a

or

cos

1

1

x a

x a

x a

or

log{x + (x2 + a2)1/2}

x a

or

log{x ∃ ( x 2

a 2 )1/ 2 }

Created with Print2PDF. To remove this line, buy a license at: http://www.software602.com/

(xxii)

# (a # (a # (x

2

2

x 2 )1/ 2 dx ! ( x / 2) , (a 2

x 2 )1/ 2 ∃ (a 2 / 2) , sin 1 ( x / a)

∃ x 2 )1/ 2 dx ! ( x / 2) , (a 2 ∃ x 2 )1/ 2 ∃ (a 2 / 2) , log {x ∃ (a 2 ∃ x 2 )1/ 2 }

2

a 2 )1/ 2 dx ! ( x / 2) , ( x 2

a 2 )1/ 2

(a 2 / 2) , log { x ∃ ( x 2

a 2 )1/ 2 }

Note. If x is replaced by ax + b (a and b being constants) on both sides of any formula of the above table, then the standard form remains true, provided the result on R.H.S. is divided by a, the coefficient of x. For examples,

#

#e

II.

#e #e #

sin( ax ∃ b) ; a eax ( a sin bx b cos bx)

#e

cos( ax ∃ b)dx !

ax

ax

ax

sin bx dx ! cos bx dx !

2

2

2

2

a ∃b e ( a cos bx ∃ b sin bx)

sin (bx ∃ c) dx !

;

#



;

#



0

ax

a ∃b e

ax

2

a ∃b III. Integration by parts

ax

e

ax

e ax ∃b a

dx !

sin bx dx !

cos bx dx !

b 2

a ∃ b2

a 2

a ∃ b2

ax

. sin 0 bx ∃ c tan 2 a ∃b e

&a sin(bx ∃ c)

2

&a cos(bx ∃ c) ∃ b cos(bx ∃ c)∋ !

eax 2

e

2

a ∃b

eax cos (bx ∃ c)dx !

0

ax ∃ b

b cos(bx ∃ c)∋ !

2

eax

. cos 0 bx ∃ c tan 2 a ∃b 2

1

2

2

1

b/ 1 a3 b/ 1 a3

# f ( x) f ( x)dx ! f ( x) &# f ( x) dx∋ # :>8< dx f ( x)9= , &# f ( x) dx∋;? dx 1

2

1

46 d

2

1

5

7

2

In words, this formula states The integral of the product of two functions = Ist function × integral of 2nd – integral of (diff. coeff. of 1st × integral of 2nd) The success of this method depends upon the choosing the first and second functions in such a way that the second term on the R.H.S. is easily integrable. Note. While choosing the first and second function, note carefully the following facts: (i) The second function must be chosen in such a way that its integral is known (ii) If the integrals of both the functions in the product to be integrated are known, then the second function must be chosen in much a way that the new integral on the R.H.S. should be integrable directly or it should the simpler than the original integral. (iii) If the integrals of both the functions are known and if one them be of the form xn or a0xn n–1 + a1x + ... + an–1 x + an (where n is a positive integers and a0, a1, ..., an are constants), then that function must be chosen as the first function. For example, in

# x e dx, x 3 x

3

must be chosen as the

first function. (iv) If in the product of two functions the integral of one of the functions is not known, then that function must be taken as the first function. For example, in –1

# x tan

1

x dx and

# x log x dx

–1

etc we do not know the integrals of tan x and log x and hence we must choose tan x and log x etc as first function. (v) Sometimes we are to evaluate the integral of a single function by the method of integration by parts. In such cases, unity (i.e., 1) must be taken as the second function. For example, to find

Created with Print2PDF. To remove this line, buy a license at: http://www.software602.com/

(xxiii)

# tan

1

# log x dx

x dx,

# tan

1

etc, we always take 1 as the function. Thus, we write

#

x dx ! (tan

1

# log x dx ! # (log x) ≅1 dx

x) ≅1 dx,

etc.

(vi) The formula of integration by parts can be applied more than once, if necessary. BERNOULLI’S FORMULA OR GENERALISED RULE OF INTEGRATION BY PARTS OR CHAIN RULE OF INTEGRATION BY PARTS. Let u and v be two functions of x. Let dashes denote differentiation and suffixes integration with respect to x. Thus, we have

du , dx

u( !

du (

u (( !

dx 2

!

d 2u dx 2

#

# u v dx ! u v

Then

#

v1 ! v dx,

,...,

v2 ! v1 dx !

## v(dx) , 2

and so on.

u (v2 ∃ u ((v3 u (((v4 ∃ ...

1

The above rule is applied when u is of the form xn or a0xn + a1 xn–1 + .... + an–1 x + an (where n is a positive integer) and v is a function of the forms eax, ax, sin ax or cos ax. While applying the above rule, simplification should be done only when the whole process of integration is over. Study the solutions of the following problems carefully. Example 1.

# x e dx ! ( x )(e ) 4 x

4

x

(4 x3 ) (e x ) ∃ (12 x2 )(e x ) (24 x) (e x ) ∃ (24)(e x )

= ex (x4 – 4x3 + 12x2 – 24x + 24) Example 2.

#

)

0

x5 sin x dx ! 4> ( x5 )( cos x) (5 x 4 ) ( sin x) ∃ (20 x3 ) (cos x) (60 x2 ) (sin x) ∃ (120 x) ( cos x) (120) ( sin x) 5?

! 4> ( x5 ∃ 20 x3 120 x) cos x ∃ (5 x4

60 x 2 ∃ 120) sin x 5?

) 0

) 0

! ( )5 ∃ 20)3 120)) cos ) ∃ ( )5 ∃ 20)3 120)) , ( 1) ! )5 20)3 ∃ 120) Some useful direct results based on integration by parts

#e

ax

{a f ( x) ∃ f (( x)}dx ! e ax f ( x). Its particulars case are

# e { f ( x) ∃ f (( x)}dx ! e x

x

#e

f ( x);

x

{ f ( x) ∃ f (( x)}dx ! e

x

f ( x)

IVProperties of definite integrals (i)

#

b

f ( x)dx !

a

(iii)

#

b

(iv)

#

a

a

a

#

b

f (t ) dt

a

f ( x ) dx !

#

f ( x)dx ! 2

b

a

#

a

0

f ( x) dx ∃

(ii)

#

b

c

#

b

a

f ( x) dx !

#

a

b

f ( x) dx

f ( x) dx, where a < c < b

f ( x) dx, if f (–x) = f (x);

#

a a

f ( x) dx ! 0, if f (–x) = – f(x)

Created with Print2PDF. To remove this line, buy a license at: http://www.software602.com/

(xxiv)

(v)

#

2a

0

#

f ( x) dx ! 2

a

f ( x) dx, if f (2a – x) = f (x);

0

#

2a

0

f ( x) dx ! 0, if f (2a – x) = –f (x)

V Walli’s formulas (i) If n is an even positive integer, then

#

)/2

0

#

sin n x dx !

)/2

0

cos n x dx !

n 1 n 3 3 1 ) ≅ ≅≅≅ ≅ ≅ n n 2 4 2 2

9 7 5 3 1 ) ≅ ≅ ≅ ≅ ≅ 10 8 6 4 2 2 Note carefully that answer is written down very easily by beginning with the denominator. We then have the ordinary sequence of natural numbers written down backwards. Thus, in the above example, we write (10 under 9) × (8 under 7) × (6 under 5) .... etc. stopping at (2 under 1), and writing a factor )/2 in the end. (ii) If n is an odd positive integer, then

#

For example

)/2

0

#

)/2

0

#

sin n x dx !

)/ 2

0

sin10 x dx !

cosn x dx !

n 1 n 3 4 2 ≅ ≅≅≅≅ ≅ n n 2 5 3

)/2

8 6 4 2 sin 9 x dx ! ≅ ≅ ≅ 9 7 5 3 Thus, as above, we begin with the denominator. We then have the ordinary sequence of natural numbers written down backwards. Thus, in the above example, we write (9 under 8) × (7 under 6) × .... etc stopping at (3 under 2) and additional factor ) / 2 is not written in the end.

#

For example.

0

(iii) If m and n are positive integers, then

#

)/ 2

0

sin m x cosn x dx !

( m 1) ( m 3) ( m 5)...( n 1) ( n 3)( n 5)... , k, ( m ∃ n) ( m ∃ n 2) ( m ∃ n 4)...

where k is ) / 2 if m and n are both both positive even integers otherwise k = l. The last factor in each of the three products (namely, (m – 1) (m – 3) (m – 5) ..., (n – 1) (n – 3) (n – 5) .... and (m + n) (m + n – 2) (m + n – 1) ...) is either 1 or 2. In case any of m or n is 1, we simply write 1 as the only factor to replace its product. Example 1

#

)/2

Example 2

#

)/2

Example 3

#

)/ 2

0

0

0

sin 4 x cos 2 x dx !

3 ≅1 ≅1 ) ) , ! 6 ≅ 4 ≅ 2 2 32

sin 4 x cos3 x dx !

3 ≅ 1≅ 2 2 ,1 ! 7 ≅ 5 ≅ 3 ≅1 35

sin 4 x cos x dx !

3 ≅1 ≅1 1 ! 5 ≅ 3 ≅1 5

Created with Print2PDF. To remove this line, buy a license at: http://www.software602.com/

ELEMENTARY DIFFERENTIAL EQUATIONS Where is What Chapters

1. Differential equations. Their formation and solutions

Pages

1.3 – 1.36

2. Equations of first order and first degree

2.1 – 2.76

3. Trajectories

3.1 – 3.16

4. Equations of the first order but not of the first degree, singular solutions and extraneous loci

4.1 – 4.48

5. Linear differential equations with constant coefficients

5.1 – 5.70

6. Homogeneous Linear Equations or Cauchy-Euler Equations

6.1 – 6.24

7. Method of variation of parameters

7.1 – 7.25

8. Ordinary simultaneous differential equations

8.1 – 8.24

9. Exact differential equations and equations of special forms

9.1 – 9.18

10. Linear equations of second order

10.1 – 10.56

11. Applications of differenital equations

11.1 – 11.28

Created with Print2PDF. To remove this line, buy a license at: http://www.software602.com/

1 Differential Equations Their Formation And Solutions 1.1 Differential equation Definition. An equation involving derivatives or differentials of one or more dependent variables with respect to one or more independent variables is called a differential equation. For examples of differential equations we list the following: dy ! (x " sin x) dx, ... (1) 5

d 4 x d 2 x ! dx # 2 # % & ∃ et , dt 4 dt ∋ dt (

y!

x

... (2)

dy k # , dx dy / dx

... (3)

k (d2y/dx2) ! {1 " (dy/dx)2}3/2 )2v/)t2 ! k ()3v/)x3)2 and )2u/)x2 " )2u/)y2 " )2u/)z2 ! 0 Note. Unless otherwise stated, y∗ (or y1 ), y+ (or y 2), ..., y (n) (or yn ) will dy d 2 y dn y , 2 , ..., n respectively. Thus, for example equation (3) may be re-written as dx dx dx y!

x y∗ # k / y∗

or

y!

... (4) ... (5) ... (6) denote

x y1 # k / y1.

1.2 Ordinary differential equation Definition. A differential equation involving derivatives with respect to a single independent variable is called an ordinary differential equation. In Art. 1.1 equations (1), (2), (3) and (4) are all ordinary differential equations. 1.3 Partial differential equation Definition. A differential equation involving partial derivatives with respect to more than one independent variables is called a partial differential equation. In Art. 1.1 equations (5) and (6) are both partial differential equations. 1.4 Order of a differential equation Definition. The order of the highest order derivative involved in a differential equation is called the order of the differential equation. 1.3

Created with Print2PDF. To remove this line, buy a license at: http://www.software602.com/

1.4

Differential Equations their Formation and Solutions

In Art. 1.1 equation (2) is of the fourth order, equations (1) and (3) are of the first order, equations (4) and (6) are of the second order and equation (5) is of the third order. 1.5 Degree of a differential equation Definition. The degree of a differential equation is the degree of the highest derivative which occurs in it, after the differential equation has been made free from radicals and fractions as far as the derivatives are concerned. Note that the above definition of degree does not require variables x, t, u etc. to be free from radicals and fractions. In Art 1.1 equations (1), (2) and (6) are of first degree. Making equation (3) free from fractions, we obtain y (dy/dx) ! x ( dy / dx ) 2 # k , which is of second degree. Again we must square both sides of (4) to make it free from radicals. Then by definition, the degree of (4) and (5) is two. 1.6 Linear and non-linear differential equations Definition. A differential equation is called linear if (i) every dependent variable and every derivative involved occurs in the first degree only, and (ii) no products of dependent variables and/or derivatives occur. A differential equation which is not linear is called a non-linear differential equation. In Art 1.1 equations (1) and (6) are linear and equations (2), (3), (4) and (5) are all non-linear. SOLVED EXAMPLES Ex. 1. Find the order and degree of the following differential equations. Also classify them as linear and non-linear. (a) y ! x (dy/dx) " k/(dy/dx) (b) y ! x (dy/dx) " a {1 " (dy/dx)2}1/2 (c) dy ! (y " sin x) dx (d) (d2y/dx2)3 " x (dy/dx)5 " y ! x2 (e) {y + x (dy/dx)2}4/3 ! x(d2y/dx2) [Rajsthan 2010] (f) (d 2y/dx2)1/3 = (y + dy/dx)1/2 [Pune 2010] Sol. (a) Multiplying both sides of the given equation by dy/dx, we get y (dy/dx) ! x ( dy / dx ) 2 # k , ... (1) which is of the first order and second degree because the order of the highest differential coefficient dy/dx is one and the highest degree of dy/dx is 2. Here (1) is non-linear differential equation because degree of dy/dx is 2 and product y (dy/dx) of dependent variable y and its derivative (dy/dx) occurs. (b) Re-writing the given equation, y – x (dy/dx) ! a {1 " (dy/dx)2}1/2 To get rid of radicals, square both sides to obtain y2 " x2 (dy/dx)2 – 2xy (dy/dx) ! a2 {1 " (dy/dx)2}, which is of the first order and second degree because the order of the highest differential coefficient dy/dx is one and the highest degree of dy/dx is 2. Since degree of dy/dx is 2, the given equation is non-linear. (c) Ans. It is of first order, first degree and linear. (d) Ans. It is of second order, third degree and non-linear. (e) Order 2, degree 3, non-linear (f) order 2, degree 2, non-linear 1.7 Solution of a differential equation Definition. Any relation between the dependent and independent variables, when substituted in the differential equation, reduces it to an identity is called a solution or integral of the differential equation. It should be noted that a solution of a differential equation does not involve the derivatives of the dependent variable with respect to the independent variable or variables.

Created with Print2PDF. To remove this line, buy a license at: http://www.software602.com/

Differential Equations their Formation and Solutions

1.5

For example, y ! ce2x is a solution of dy/dx ! 2y because by putting y ! ce2x and dy/dx ! 2ce2x, the given differential equation reduces to the identity 2ce 2x ! 2ce2x. Observe that y ! ce2x is a solution of the given differential equation for any real constant c which is called an arbitrary constant. SOLVED EXAMPLES Ex. 1. Show that y ! (A/x) " B is a solution of (d2y/dx2) + (2/x) × (dy/dx) ! 0. Sol. Given that (d2y/dx2) " (2/x) × (dy/dx) ! 0. ... (1) Also given that y ! (A/x) " B. ... (2) Differentiating (2) w.r.t. ‘x’, dy/dx ! – (A/x2) ... (3) 2 2 3 Differentiating (3) w.r.t. ‘x’, d y/dx ! 2A/x ... (4) 2 2 Substituting for dy/dx and d y/dx from (3) and (4) in (1), we get (2A/x3) " (2/x) × (– A/x2) ! 0 or 0 ! 0, which is true. Hence (2) is a solution of (1). Ex. 2. Show that y ! a cos (mx " b) is a solution of the differential equation d2y/dx2 " m2y ! 0. Sol. Try yourself. 1.8 Family of curves Definition. An n-parameter family of curves is a set of relations of the form {(x, y) : f (x, y, c1, c2, ..., cn) ! 0}, where ‘f ’ is a real valued function of x, y, c1, c2, ..., cn and each ci (i ! 1, 2, ..., n) ranges over an interval of real values. For example, the set of concentric circles defined by x2 " y2 ! c is one parameter family if c takes all non-negative real values. Again, the set of circles, defined by (x – c1)2 " (y – c2)2 ! c3 is a three-parameter family if c1, c2 take all real values and c3 takes all non-negative real values. 1.9 Complete primitive (or general solution). Particular solution and singular solution. Definitions Let F (x, y, y1, y2, ..., yn) ! 0 ... (1) be an nth order ordinary differential equation. (i) A solution of (1) containing n independent arbitrary constants is called a general solution. (ii) A solution of (1) obtained from a general solution of (1) by giving particular values to one or more of the n independent arbitrary constants is called a particular solution of (1). (iii) A solution of (1) which cannot be obtained from any general solution of (1) by any choice of the n independent arbitrary constants is called a singular solution of (1). The student can easily verify that y ! c1ex " c2e2x ... (2) is the general solution of y+ – 3y∗ " 2y ! 0. ... (3) Since c1 and c2 are independent arbitrary constants and the order of (3) is two, (2) is a general solution of (3). Some particular solutions of (3) are given by y ! ex " e2x, y ! ex – 2e2x etc. Again, the reader can verify that y ! (x " c)2 ... (4) is the general solution of (dy/dx)2 – 4y ! 0. ... (5) The reader can also verify that y ! 0 is also solution of (5). Moreover, y ! 0 cannot be obtained by any choice of c in (4). Hence, y ! 0 is a singular solution of (5).

Created with Print2PDF. To remove this line, buy a license at: http://www.software602.com/

1.6

Differential Equations their Formation and Solutions

1.10 Formation of differential equations Suppose we are given a family of curves containing n arbitrary constants. Then we can obtain an nth order differential equation whose solution is the given family as follows. Working rule to form the differential equation from the given equation in x and y, containing n arbitrary constants. Step I. Write the equation of the given family of curves. Step II. Differentiate the equation of step I, n times so as to get n additional equations containing the n arbitrary constants and derivatives. Step III. Eliminate n arbitrary constants from the (n " 1) equations obtained in steps I and II. Thus, we obtain the required differential equation involving a derivative of nth order. 1.11 Solved examples based on Art. 1.10 Ex. 1. Find the differential equation of the family of curves y ! emx, where m is an arbitrary constant. Sol. Given that y ! emx. ... (1) Differentiating (1) w.r.t. ‘x’, we get dy/dx ! memx. ... (2) Now, (1) and (2) , dy/dx ! my , m ! (1/y) × (dy/dx). ... (3) Again, from (1), mx ! loge y so that m ! (loge y)/x. ... (4) Eliminating m from (3) and (4), we get (1/y) × (dy/dx) ! (1/x) × loge y. Ex. 2. (a) Find the differential equation of all straight lines passing through the origin. (b) Find the differential equation of all the straight lines in the xy-plane. Sol. (a) Equation of any straight line passing through the origin is y ! mx, m being arbitrary constant. ... (1) Differentiating (1) w.r.t. ‘x’, dy/dx ! m. ... (2) Eliminating m from (1) and (2), we get y ! x (dy/dx). (b) We know that equation of any straight line in the xy-plane is given by y ! mx " c, m and c being arbitrary constants. ... (1) Differentiating (1) w.r.t. ‘x’, we get dy/dx ! m. ... (2) Differentiating (2) w.r.t. ‘x’, we get d2y/dx2 ! 0, ... (3) which is the required differential equation. Note. Equation (3) is free from m and c and so it is not necessary to eliminate m and c from (1), (2) and (3) as usual. Ex. 3. (a) Obtain a differential equation satisfied by family of circles x2 " y2 ! a2, a being an arbitrary constant. (b) Obtain a differential equation satisfied by the family of concentric circles. Sol. (a) Given x2 " y2 ! a2. ... (1) Differentiating (1) w.r.t. ‘x’, we get 2x " 2y (dy/dx) ! 0 or x " y (dy/dx) ! 0, which is the required differential equation. (b) Let the centre of the given family of concentric circles be (0, 0). Then we know that the equation of the family of concentric circles is given by x2 " y2 ! a2, a being arbitrary constant. Now proceed as in part (a). Ans. x " y (dy/dx) ! 0. Ex. 4. (a) Find the differential equation of all circles which pass through the origin and whose centres are on the x-axis. [I.A.S. (Prel.) 2002]

Created with Print2PDF. To remove this line, buy a license at: http://www.software602.com/

Differential Equations their Formation and Solutions

1.7

(b) Find the differential equation of the system of circles touching the y-axis at the origin. [I.A.S. (Prel.) 1997] (c) Find the differential equation of all circles touching a given straight line at a given point. Sol. (a) We know that the equation of any circle passing through the origin and whose centre is on the x-axis is given by x2 " y2 " 2gx ! 0, g being an arbitrary constant. ... (1) Differentiating (1) w.r.t. ‘x’, we get 2x " 2y (dy/dx) " 2g ! 0. ... (2) 2 2 2 2 From (1), 2gx ! – (x " y ) so that 2g ! – (x " y )/x ... (3) Substituting for 2g from (3) in (2), we have x2 # y − ! dy 2x " 2y % & . !0 x ∋ dx (

or

2xy

dy " x2 – y2 ! 0. dx

(b) We note that any circle which touches the y-axis at the origin must have its centre on the x-axis and so equation of any such circle is given by x2 " y2 " 2gx ! 0, g being an arbitrary constant. ... (1) Now proceed as in part (a) and obtain same answer. (c) For the sake of simplification, without loss of any generality, take the given point as the origin and the given straight line as the y-axis. Now proceed as in part (b) and get the same answer. Ex. 5. (a) Find the differential equation of all circles which pass through the origin and whose centres are on the y-axis. (b) Find the differential equation of the system of circles touching the x-axis at the origin. [I.A.S. (Prel.) 1999] Sol. Parts (a) and (b). Here in both parts the equation of any circle is x2 " y2 " 2fy ! 0, f being an arbitrary constant. ... (1) Proceed as in Ex. 4(a). Ans. (x2 – y2) (dy/dx) – 2xy ! 0. Ex. 6. Find the differential equation which has y ! a cos (mx " b) for its integral, a and b being arbitrary constants and m being a fixed constant. Sol. Given that y ! a cos (mx " b). ... (1) Differentiating (1) w.r.t. ‘x’, we get dy/dx ! – am sin (mx " b). ... (2) Differentiating (2) w.r.t. ‘x’, we get d2y/dx2 ! – am2 cos (mx " b). ... (3) 2 2 2 or d y/dx ! – m y, using (1) Thus, the required differential equation is d2y/dx2 + m2y ! 0. Ex. 7. Find the differential equation from the relation y ! a sin x " b cos x " x sin x, where a and b are arbitrary constants. Sol. Given y ! a sin x " b cos x " x sin x. ... (1) Differentiating (1) w.r.t. ‘x’, dy/dx ! a cos x – b sin x " sin x " x cos x. ... (2) Differentiating (2) w.r.t. ‘x’, d2y/dx2 ! – a sin x – b cos x " 2 cos x – x sin x 2 2 or d y/dx ! 2 cos x – (a sin x + b cos x + x sin x) ! 2 cos x – y, by (1). 2 / (d y/dx2) " y ! 2 cos x, which is the required differential equation. Ex. 8. (a) Find the differential equation of the family of curves y ! ex (A cos x " B sin x), where A and B are arbitrary constants. [G.N.D.U. Amritsar 2010] x (b) Form a differential equation of which y ! e (A cos 2x " B sin 2x) is a solution, A and B being arbitrary constants. Sol. (a) Given that y ! ex (A cos x + B sin x). ... (1)

Created with Print2PDF. To remove this line, buy a license at: http://www.software602.com/

Differential Equations their Formation and Solutions

1.8

y∗ ! ex (– A sin x " B cos x) " ex (A cos x " B sin x) or y∗ ! e (– A sin x " B cos x) " y, using (1). ... (2) Differentiating (2) again with respect to x, we get y+ ! – ex (A cos x " B sin x) " ex (– A sin x " B cos x) " y∗. ... (3) x Now from (2), we get e (– A sin x " B sin x) ! y∗ – y. ... (4) Hence, eliminating A and B from (1), (3) and (4), we get y+ ! – y " y∗ – y " y∗ or y+ – 2y∗ " 2y ! 0. (b) Proceed as in Ex. 8(a). Ans. y+ – 2y∗ " 5y ! 0 Ex. 9. By eliminating the constants a and b obtain the differential equation for which xy ! aex " be–x " x2 is a solution. [I.A.S. 1992] Sol. Given that xy ! aex " be–x " x2. ... (1) x –x Diff. (1) w.r.t. ‘x’, we get xy∗ " y ! ae – be " 2x. ... (2) x –x Diff. (2) w.r.t. ‘x’, we get xy+ " y∗ " y∗ ! ae " be " 2 or xy+ " 2y∗ ! (xy – x2) " 2, using (1) or xy+ " 2y∗ – xy " x2 – 2 ! 0. Ex. 10. Find the differential equation corresponding to the family of curves y ! c (x – c)2, where c is an arbitrary constant. [I.A.S. (Prel.) 2009; Karnatak 1995] Sol. Given that y ! c (x – c)2. ... (1) Diff. (1) w.r.t. ‘x’, we get y∗ ! 2c (x – c). ... (2) From (1) and (2), y∗/y ! 2/(x – c) so that c ! x – (2y/y∗). ... (3) Putting this value of c in (2), the required equation is y∗ ! 2 {x – (2y/y∗)} × (2y/y∗) or (y∗)3 ! 4y (xy∗ – 2y). Ex. 11. Find the differential equation of all circles of radius a. [Nagarjuna 2003] Sol. The equation of all circles of radius a is given by (x – h)2 " (y – k)2 ! a2, ... (1) where h and k, are to be taken as arbitrary constants. Diff. (1) w.r.t. ‘x’, we get (x – h) " (y – k) y∗ ! 0. ... (2) 2 2 Diff. (2), 1 " (y∗) " (y – k) y+ ! 0 or y – k ! – {1 " (y∗) }/y+. ... (3) Putting this value of y – k in (2), we get x – h ! – (y – k) y∗ ! {1 " (y∗)2} × (y∗/y+). ... (4) Using (3) and (4), (1) gives the required equation as Differentiating (1),

x

{1 # ( y ∗) 2 }2 ( y ∗) 2

or

{1 # ( y ∗) 2 }2

! a2 or {1 " (y∗)2}3 ! a2 (y+)2. ( y +) 2 ( y +) 2 Ex. 12. Show that Ax2 " By2 ! 1 is the solution of x [y (d2y/dx2) " (dy/dx)2] ! y (dy/dx). [Gauhati 1996, Indore 1997] Sol. Given that Ax2 " By2 ! 1. ... (1) Diff. (1), 2Ax " 2By (dy/dx) ! 0 or Ax " By (dy/dx) ! 0. ... (2) Diff. (2), A " B {y (d2y/dx2) " (dy/dx) × (dy/dx)} ! 0. ... (3) 2 2 2 Multiplying (3) by x, we get Ax " Bx {y (d y/dx ) " (dy/dx) } ! 0. ... (4) Subtracting (2) from (4), we get Bx {y (d2y/dx2) " (dy/dx)2} – By (dy/dx) ! 0 x [y (d2y/dx2) " (dy/dx)2] ! y (dy/dx), as required. #

Created with Print2PDF. To remove this line, buy a license at: http://www.software602.com/

Differential Equations their Formation and Solutions

1.9

Ex. 13. Find the third order differential equation whose solution is the 3-parameter family of curves defined by x2 " y2 " 2ax " 2by " c ! 0, where a, b, c are parameters. Sol. Given x2 " y2 " 2ax " 2by " c ! 0. ... (1) Differentiating (1) w.r.t. ‘x’ three times in succession, we have x " y y(1) " a " b y(1) ! 0, where y(1) ! dy/dx ... (2) (1) 2 (2) (2) (2) 2 2 1 " [y ] " y y " b y ! 0, where y ! d y/dx ... (3) and 3y(1) y(2) " y y(3) " b y(3) ! 0., where y(3) ! d3y/dx3 ... (4) We now eliminate a, b, c from (1), (2), (3) and (4). To do so, we simply eliminate b from (3) and (4). Multiplying both sides of (3) by y(3) and (4) by y(2) and subtracting, we have y(3) " y(3) [y(1)]2 " y y(2) y(3) – 3y(1) [y(2)]2 – y y(2) y(3) ! 0 or [1 " (y(1))2] y(3) – 3y(1) (y(2))2 ! 0. EXERCISE 1(A) 1. Form the differential equations for the following: (a) y ! Ae2x " Be–2x, A, B being arbitrary constants.

Ans. y+ ! 4y 2 Ans. y ! y∗ (1 . x ) sin–1x

(b) y ! k sin–1 x, k parameter

(c) y ! 0x " 1x2, 0, 1 parameters [Rajasthan 2010] Ans. x2y+ – 2xy∗ " 2y ! 0 (d) y ! A cos nt " B sin nt, (A, B parameters) Ans. (d 2x/dt2) " n2x ! 0 (e) xy ! aex " be–x, (a, b parameters) [Behrampur 2010] Ans. xy+ + 2y∗ – xy ! 0 3x 5x 2. Find the differential equation of the family of curves y ! Ae " Be ; for different values of A and B. Ans. y+ – 8y∗ " 15y ! 0 3. Find the differential equation of all circles passing through origin and having their centres on the xaxis. Ans. 2xy∗ ! y2 – x2 2 2 4. Show that v ! B " A/r is a solution of (d v/dr ) " (2/r) × (dv/dr) ! 0. 5. Find a differential equation with the following solution: y ! aex " be–x " c cos x " d sin x, where a, b, c and d are parameters. Ans. d 4y/dx4 – y ! 0 6. Classify the following equations as linear and non-linear equations and write down their orders

d4y d2y dy # ! ex. (c) " y2 ! x2. dx 4 dx 2 dx Ans. (a) Non-linear; 3 (b) Linear, 4 (c) Non-linear, 1 Write down the order and degree of x2 (d 2y/dx2)3 " y (dy/dx)4 " y4 ! 0. How many constants does the general solution of the differential equation must contain. Ans. 2, 3, 2 Find the differential equation of the family of parabolas y2 ! 4ax. Ans. y ! 2x (dy/dx) Show that the differential equation of the family of circles of fixed radius r with centre on y-axis is (x2 – r2) (dy/dx)2 " x2 ! 0. Find the differential equation of all (a) parabolas of latusrectum 4a and axis parallel to y-axis. (b) tangent lines to the parabola y ! x2. (c) ellipses centered at the origin. (d) circles through the origin. (e) circles tangent to y-axis. (f) parabolas with axis parallel to the axis of y. (g) parabolas with foci at the origin and axis along x-axis. (h) all conics whose axes coincide with axes of co-ordinates. Ans. (a) 2ay2 – 1 ! 0 (b) 4 (y – xy1) " (y1)2 ! 0 2 (c) xyy2 " x (y1) – yy1 ! 0 (d) (x2 " y2) y2 ! 2 (xy1 – y) (1 " y12) (a)

7. 8. 9. 10.

a 3 y d 2 y dy # 2 " y ! x. dx 3 dx 2 dx

(b) x

Created with Print2PDF. To remove this line, buy a license at: http://www.software602.com/

1.10

Differential Equations their Formation and Solutions

(e) x2 y12 – 2xy1y2 (1 " y12) – (1 " y12) ! 0 (f) y3 ! 0 2 (g) yy1 " 2xy1 – y ! 0 (h) xyy2 " xy12 ! xy1. 11. Form the differential equation of family of curves y ! cx " c2, c being a comitent

[Pune 2010] Ans. y ! xy∗ ! y∗∗2

1.12 The Wronskian [Delhi Maths (Hons) 2000] Definition. The Wronskian of n functions y1 (x), y2 (x), ..., yn (x) is denoted by W (x) or W (y1, y2, ..., yn) (x) and is defined to be the determinant y1 y2 yn y1∗ y2∗ yn∗ W (y1, y2, ..., yn) (x) ! W (x) ! . y1( n .1)

y2( n .1)

yn( n .1)

1.13 Linearly dependent and independent set of functions Definitions. n functions y1 (x), y2 (x), ..., yn (x) are linearly dependent if there exist constants c1, c2, ..., cn (not all zero), such that c1 y1 " c2 y2 " ... " cn yn ! 0. ... (1) If, however, identity (1) implies that c1 ! c2 ! ... ! cn ! 0, then y1, y2, ..., yn are said to be linearly independent. SOLVED EXAMPLES Ex. 1. Consider the two functions f1 (x) ! x3 and f2 (x) ! x2 | x | on the interval [– 1, 1]. (i) Show that their Wronskian W (f1, f2) vanishes identically. (ii) Show that f1 and f2 are not linearly dependent. (iii) Do (i) and (ii) contradict theorem III, Art. 1.16 If not, why not. Sol. Left as an exercise. Ex. 2. Define the concept of linear dependence and independence. Hence, show that (i) sin x and cos x, – 3 < x < 3 are linearly independent. (ii) ei4x, sin 4 x, cos 4 x, – 3 < x < 3, 4 being a real number, are linearly dependent. (iii) 1, x, x2, ......, xn, – 3 < x < 3 are linearly independent. (iv) x2 and x | x | are linearly independent on – 3 < x < 3. (v) sin x, sin 2x, sin 3x are linearly independent on [0, 25]. (vi) ex, cos x, sin x are linearly independent on a real line. (vii) sin x, sin (x + 5/8), sin (x – 5/8) are linearly dependent on ]– 3, 3[. (viii) x4 and x3 | x | are linearly independent on [– 1, 1] but are linearly dependent on [– 1, 0] and [0, 1] (ix) f1 + f2 and f1 – f2 are linearly independent on an interval I whenever f1 and f2 are linearly independent on the interval I. (x) f and g are linearly independent on [– 1, 1], if functions f and g are defined on [– 1, 1] as follows: f ( x) ∃ 0 6 f ( x ) ∃ sin x 6 7 if x 9 [– 1, 0]; 7 if x 9 [0, 1] g ( x) ∃ 1 8 g (x) ∃ 1 . x 8

Created with Print2PDF. To remove this line, buy a license at: http://www.software602.com/

Differential Equations their Formation and Solutions

1.11

Sol. A set of n real or complex valued functions y1 (x), y2 (x), ......, yn (x), n : 2 defined on an interval I are linearly dependent on I if there exist n constants, real or complex, c1, c2, ......, cn, not all of them simultaneously zero such that c1 y1 (x) + c2 y2 (x) + ...... + cn yn (x) ! 0 for each x in I. The functions y1 (x), y2 (x), ......, yn (x) are linearly independent on I if they are not linearly dependent on I. (i) Consider c1 sin x + c2 cos x ! 0, – 3 < x < 3 ... (1) Differentiating (1) w.r.t. ‘x’, we get c1 cos x – c2 sin x ! 0 ... (2) Solving (1) and (2), c 1 ! c2 ! 0. Hence, sin x and cos x are linearly independent on – 3 < x < 3. (ii) We have, ei4x ! cos 4x + i sin 4x, by Euler’s theorem Hence, ei4x – cos 4x – i sin 4x ! 0, i4x showing that e , cos 4x, sin 4x are linearly dependent. (iii) Consider, c0 + c1x + c2 x2 + ...... + cn xn ! 0, – 3 < x < 3 ... (A1) Differentiating (A1) w.r.t. ‘x’ successively on – 3 < x < 3 yields c1 + 2c2x + 3c3x2 + ...... + n cnxn–1 ! 0 ... (A2) 2c2 + 6c3x + ...... + n (n – 1) cn xn–2 ! 0 ... (A3) ....................................................................... n ! cn ! 0 ... (A4) Solving (A1), (A 2), ......, (A n) yields c 1 ! c 2 ! c3 ! ...... ! cn–1 ! cn ! 0 and so the given functions 1, x, x2, ......, xn are linearly independent (iv) Here, c1 x2 + c2 x | x | ! 0 ... (1) 2 2 , c1 x + c2 x ! 0 for x : 0 ... (2) and c1 x2 – c2 x2 ! 0 for x < 0 ... (3) In order that (1) may hold, (2) and (3) should hold simultaneously. This is possible only when c1 + c2 ! 0 and c1 – c2 ! 0, i.e., c1 ! c2 ! 0. Hence, x 2 and x | x | are linearly independent on – 3 < x < 3. 1.14 Existence and uniqueness theorem [Delhi B.Sc. (Hons) II 2011] Consider a second order linear differential equation of the form a0 (x) y+ " a1 (x) y∗ " a2 (x) y ! r (x), ... (1) where a0 (x), a1 (x), a2 (x) and r (x) are continuous functions on an interval (a, b) and a0 (x) ; 0 for each x 9 (a, b). Let c1 and c2 be arbitrary real numbers and x0 9 (a, b). Then there exists a unique solution y (x) of (1) satisfying y (x0) ! c1 and y∗ (x0) ! c2. Moreover, this solution y (x) is defined over the interval (a, b). Note 1. The above theorem is an existence theorem because it says that the initial value problem does have a solution. It is also a uniqueness theorem, because it says that there is only one solution. Clearly, this theorem also applies to an associated homogenous equation a0 (x) y+ " a1 (x) y∗ " a2 (x) y ! 0. ... (2) Note 2. In this chapter, we shall assume without proof, the above basic theorem for initial value problems associated with linear differential equations. Note 3. The conditions of existence and uniqueness theorem cannot be further relaxed. For example, if a0 (x) ! 0 for some x 9 (a, b), then the solution of (1) may not be unique or may not exist at all. For an example, refer solved example 1 of Art. 1.15.

Created with Print2PDF. To remove this line, buy a license at: http://www.software602.com/

Differential Equations their Formation and Solutions

1.12

Note 4. Existence and uniqueness theorem can be extended to an nth order linear differential equation. Corollary. If y (x) be a solution of a0 (x) y+ " a1 (x) y∗ " a2 (x) y ! 0 satisfying y (x0) ! 0 and y∗ (x0) ! 0 for some x0 9 (a, b), then y (x) is identically zero on (a, b). Proof. By definition, here y (x) is a solution of the given equation which satisfies y (x0) ! 0 and y∗ (x0) ! 0. Again, by existence and uniqueness theorem, y (x) is the unique solution satisfying y (x0) ! 0 and y∗ (x0) ! 0. It follows that y (x) < 0 on (a, b), i.e., y (x) is identically zero on (a, b). Note 1. A real valued function y (x) is said to be identically zero on an interval (a, b) written as y (x) < 0, if y (x) ! 0 for each x 9 (a, b). Note 2. A function u (x) is called a solution of the equation a0 (x) y+ (x) " a1 (x) y∗ (x) " a2 (x) y (x) ! 0 if a0 (x) u+ (x) " a1 (x) u∗ (x) " a2 (x) u (x) ! 0, for each x 9 (a, b). 1.14A (a) State the existence and uniqueness theorem for the nth order differential equation L (y) (x) ! y(n) (x) + p1 (x) y(n–1) (x) + ...... + pn (x) y (x) ! 0, x 9 I, which is a linear homogeneous equation. (b) Show that there are three linearly independent solutions of the third order equation y+∗ + p1 (x) y+ + p2 (x) y∗ + p3 (x) y ! 0, x 9 I where p1, p2 and p3 are functions, defined and continuous on an interval I. (c) Let = be any solution of y+∗ + p1 (x) y+ + p2 (x) y∗ + p3 (x) y ! 0, x 9 I. Here p1, p2 and p3 are functions defined and continuous on an interval I. Further, let =1, =2 and =3 be three linearly independent solutions of the given equation. Prove that constants c1, c2 and c3 exist such that = ! c1 =1 + c2 =2 + c3 =3, x 9 I. Sol. (a) Statement of the existence and uniqueness theorem for L (y) (x) ! y(n) (x) + p1 (x) y(n–1) (x) + ...... + pn (x) y (x) ! 0, x 9 I ... (1) Let p1, p2, ......, pn be defined and continous on an interval I which contains a point x0. Let a0, a1, ......, an–1 be n constants. Then, there exists a unique solution = on I of the nth order equation (1) satisfying the initial conditions. = (x0) ! a0, =∗ (x0) ! a1, ...... , =(n–1) (x0) ! an–1 Note. Suppose that =1 (x), ......, =n (x) are n solutions of L (y) (x) ! 0 given in (1) and suppose that c1, c2, ......, cn are n arbitrary constants. Since L (=1) ! L (=2) ! ...... ! L (=n) ! 0, and L is a linear operator, hence we have L (c1 =1 + c2 =2 + ...... + cn =n) ! c1 L (=1) + ...... + cn L (=n) ! 0 In case n solutions =1, ......, =n are linerly independent and c1, c2, .... , cn are constants, then c1=1 + c2=2 + ..... + cn=n ! 0, x 9 I , c1 ! c2 ! .... cn ! 0 (b) Given y+∗ + p1 (x) y+ + p2 (x) y∗ + p3 (x) y ! 0, x 9 I ... (1) Using the existence and uniqueness theorem stated in part (a), we conclude that there exist solutions =1 (x), =2 (x) and =3 (x) of (1) such that for x0 9 I.

and

=1 (x0) ! 0,

=1∗ ( x0 ) ∃ 0 ,

=1∗∗ ( x0 ) ∃ 0

=2 (x0) ! 0,

=∗2 ( x0 ) ∃ 1,

=∗∗2 ( x0 ) ∃ 0 7 > =∗∗3 ( x0 ) ∃ 1 8

6 >

=3 (x0) ! 0, =∗3 ( x0 ) ∃ 0, We now proceed to prove that =1, =2 and =3 are linearly independent. Assume that c1 =1 (x) + c2 =2 (x) + c3 =3 (x) ! 0, x 9 I

... (2)

... (3)

Created with Print2PDF. To remove this line, buy a license at: http://www.software602.com/

Differential Equations their Formation and Solutions

1.13

for some constants c1, c2 and c3. At x ! x0, from (3), we obtain c1 =1 (x0) + c2 =2 (x0) + c3 =3 (x0) ! 0 Differentiating (3) w.r.t. ‘x’ and then replacing x by x0 yields

... (4)

c1 =1∗ ( x0 ) # c2 =∗2 ( x0 ) # c3 =∗3 ( x0 ) ∃ 0

... (5)

Differentiating (3) twice w.r.t. ‘x’ and then replacing x by x0 yields

c1 =1∗∗ ( x0 ) # c2 =∗∗2 ( x0 ) # c3 =∗∗3 ( x0 ) ∃ 0

... (6)

Using (2) in (4), (5) and (6), we have c1 ! c2 ! c3 ! 0. Hence, =1, =2 and =3 are linearly independent. (c) Given y+ + p1 (x) y+ + p2 (x) y∗ + p3 (x) y ! 0, x 9 I ... (1) Given that = is a solution of (1). Using the existence and uniqueness theorem stated in part (a), at x ! x0 9 I, there exist constants a1, a2 and a3 such that =(x0) ! a1, =∗(x0) ! a2 and =∗∗(x0) ! a3 The solutions =1, =2 and =3 are as given by part (b). We now define a function ? on I such that ?(x) ! a1=1(x) + a2 =2(x) + a3=3(x), x 9 I. Clearly ? satisfies (1) and ?(x0) ! a1, ?∗(x0) ! a2 and ?∗∗(x0) ! a3 Observe that two solutions = and ? of (1) have the same initial conditions. Hence by the existence and uniqueness theorem, it follows that =(x) ! ?(x) for x 9 I. Remark. From the parts (b) and (c), it follows that for a third order equation (1) of part (a) and (b), there are three linearly independent solutions and that any other solution of that equation is a linear combination of these solutions. 1.15 Solved examples based on Art 1.14 and 1.14A Ex. 1. Show that the function y ! cx2 " x " 3 is a solution, though not unique, of the initial value problem x2y+ – 2xy∗ " 2y ! 6 with y (0) ! 3, y∗ (0) ! 1 on (– 3, 3). [Delhi Maths (Hons.) 1994, 2007] Sol. Given equation is x2y+ – 2xy∗ " 2y ! 6 ... (1) 2 and given function is y (x) ! cx " x " 3. ... (2) Differentiating (2), we get y∗ ! 2cx " 1 and y+ ! 2c. ... (3) 2 2 / L.H.S. of (1) ! x (2c) – 2x (2cx " 1) " 2 (cx " x " 3), by (2) and (3) ! 6 ! R.H.S. of (1), showing that (2) is a solution of (1). Again, from (2) and (3), we get y (0) ! (c × 0) " 0 " 3 ! 3 and y∗ (0) ! (2c) × (0) " 1 ! 1. Comparing (1) with a0 (x) y+ " a1 (x) y∗ " a2 (x) y ! r (x), here a0 (x) ! x2, a1 (x) ! – 2x, a2 (x) ! 2 and r (x) ! 6, which are continuous functions on (– 3, 3). Since a0 (x) ! x2 ! 0 for x ! 0 9 (– 3, 3), therefore, the solution y ! cx2 " x " 3 is not unique (refer note 3 of Art 1.14). We see that y ! cx2 " x " 3 is solution for any real value of c. For example, y ! 2x2 " x " 3 and y ! 3x2 " x " 3 are both solutions of (1) with y (0) ! 3 and y∗ (0) ! 1. Ex. 2. Show that y ! 3e2x " e–2x – 3x is the unique solution of the initial value problem y+ – 4y ! 12x, where y (0) ! 4. y∗ (0) ! 1. [Delhi Maths (Hons.) 1996] Sol. Given equation is y+ – 4y ! 12x ... (1) and the given function is y ! 3e2x " e–2x – 3x. ... (2)

Created with Print2PDF. To remove this line, buy a license at: http://www.software602.com/

Differential Equations their Formation and Solutions

1.14

Differentiating (2), we get y∗ ! 6e2x – 2e–2x – 3 and y+ ! 12e2x " 4e–2x. ... (3) 2x –2x 2x –2x / L.H.S. of (1) ! 12e " 4e – 4 (3e " e – 3x), using (2) and (3) ! 12x ! R.H.S. of (1), showing that (2) is a solution of (1). Again, from (2) and (3), we get y (0) ! 3 " 1 – (3 × 0) ! 4 and y∗ (0) ! 6 – 2 – 3 ! 1. Comparing (1) with a0 (x) y+ " a1 (x) y∗ " a2 (x) y ! r (x), here a0 (x) ! 1, a1 (x) ! 0, a2 (x) ! – 4 and r (x) ! 12x, which are all continuous functions in (– 3, 3) and a0 (x) ! 1 ; 0 for each x 9 (– 3, 3). Therefore, by existence and uniqueness theorem, it follows that (2) is the unique solution of (1), satisfying y (0) ! 4 and y∗ (0) ! 1. Ex. 3. Find the unique solution of y+ ! 1 satisfying y (0) ! 1 and y∗ (0) ! 2. Sol. Given equation is y+ ! d2y/dx2 ! 1 ... (1) Integrating (1), y∗ ! dy/dx ! x " c1 ... (2) 2 Integrating (2), y ! x /2 " c1x " c2 ... (3) Putting x ! 0 in (2) and (3) and using y (0) ! 1 and y∗ (0) ! 2, we get c1 ! 2 and c2 ! 1. Hence (3) becomes y ! x2/2 " 2x " 1 ... (4) Now, comparing (1) with a0 (x) y+ " a1 (x) y∗ " a2 (x) y ! r (x), we have a0 (x) ! 1, a1 (x) ! 0, a2 (x) ! 0 and r (x) ! 1. These are all continuous in (– 3, 3) and a0 (x) ; 0 for each x 9 (– 3, 3). Hence, by existence and uniqueness theorem, the solution (4) is unique. Exercise 1(B) 1. Show that y ! x " x log x – 1 is the unique solution of xy+ – 1 ! 0 satisfying y (1) ! 0 and y∗ (1) ! 2. 2. Show that y ! (1/4) × sin 4x is a unique solution of the initial value problem y+ " 16y ! 0 with y (0) ! 0 and y∗ (0) ! 1. 3. Show that y≅ < x and y2 < x3 are two different solutions of x2 y∗∗ – 3xy∗ + 3y ! 0 satisfying the initial conditions y (0) ! y’ (0) ! 0. Explain why these facts do not contradict the existence and uniquenen theorem. [Delhi B.Sc (Hons) II 2011] 2 4. Given that y ! c1 " c2x is a two parameter family of solutions of xy+ – y∗ ! 0 on the interval – 3 < x < 3. Show that constants c1 and c2 cannot be found so that a member of the family satisfies the initial conditions y (0) ! 1, y∗ (0) ! 0. Explain why this does not violate existence and uniqueness theorem.

1.16 Some important theorems Theorem 1. If y1 (x) and y2 (x) are any two solutions of a0 (x) y+ (x) " a1 (x) y∗ (x) " a2 (x) y (x) ! 0, then the linear combination c1 y1 (x) " c2 y2 (x), where c1 and c2 are constants, is also a solution of the given equation. [Delhi Maths (G) 2001, 02] Proof. Given a0 (x) y+ (x) " a1 (x) y∗ (x) " a2 (x) y (x) ! 0. ... (1) Since y1 (x) and y2 (x) are solutions of (1), we have a0 ( x ) y1∗∗ ( x ) # a1 ( x ) y1∗ ( x ) # a2 ( x ) y1 ( x ) ∃ 0 ... (2) and a0 ( x ) y 2∗∗ ( x ) # a1 ( x ) y2∗ ( x ) # a2 ( x ) y2 ( x ) ∃ 0. ... (3) Let u (x) ! c1 y1 (x) " c2 y2 (x). ... (4) Differentiating (4) twice w.r.t. ‘x’, we have u∗ ( x ) ∃ c1 y1∗ ( x ) # c2 y2∗ ( x )

and

u ∗∗ ( x ) ∃ c1 y1∗∗ ( x ) # c2 y2∗∗ ( x ). ... (5)

Created with Print2PDF. To remove this line, buy a license at: http://www.software602.com/

Differential Equations their Formation and Solutions

1.15

Then, a0 (x) u+ (x) " a1 (x) u∗ (x) " a2 (x) u (x) ! a0 ( x ) [c1 y1∗∗ ( x ) # c2 y2∗∗ ( x )] # a1 ( x ) [c1 y1∗ ( x ) # c2 y2∗ ( x )] " a2 (x) [c1 y1 (x) " c2 y2 (x)], using (4) and (5) ! c1 [a0 ( x ) y1∗∗ ( x ) # a1 ( x ) y1∗ ( x) # a2 ( x ) y1 ( x )] # c2 [a0 ( x ) y2∗∗ ( x ) # a1 ( x ) y2∗ ( x ) # a2 ( x ) y2 ( x )] ! c1 2 0 " c2 2 0 using (2) and (3) Thus, a0 (x) u+ (x) " a1 (x) u∗ (x) " a2 (x) u (x) ! 0, showing that u (x), i.e., c1 y1 (x) " c2 y2 (x) is also solution of (1). Note. The result of the above theorem 1 can be generalised as follows: If y1 (x), y2 (x), ..., yn (x) be n solutions of a0 (x) y+ (x) " a1 (x) y∗ (x) " a2 (x) y (x) ! 0, then their linear combination c1 y 1 (x) " c 2 y 2 (x) " ... " c n yn (x) is also solution of the given equation, c 1, c 2, ..., c n being constants. Theorem II. There exist two linearly independent solutions y1 (x) and y2 (x) of the equation a0 (x) y+ (x) " a1 (x) y∗ (x) " a2 (x) y (x) ! 0 such that its every solution y (x) may be written as y (x) ! c1 y1 (x) " c2 y2 (x), x 9 (a, b) where c1 and c2 are suitable chosen constants. [Delhi Maths (Hons) 1996] Proof. Given a0 (x) y+ (x) " a1 (x) y∗ (x) " a2 (x) y (x) ! 0. ... (1) Let x0 9 (a, b) and y1 (x) and y2 (x) be two solutions of (1) satisfying y1 (x0) ! 1 and y1∗ ( x0 ) ∃ 0 ... (2) y2 (x0) ! 0 and y2∗ ( x0 ) ∃ 1. ... (3) To prove that y1 (x) and y2 (x) are linearly independent. Let, if possible y1 (x) and y2 (x) be linearly dependent. Then, by definition, there must exist constants c1 and c2, not both zero, such that c1 y1(x) " c2 y2(x) ! 0 for each x 9 (a, b). ... (4) Now (4) , c1 y1∗ ( x ) # c2 y2∗ ( x ) ∃ 0 for each x 9 (a, b). ... (5) By assumption, x0 9 (a, b). Hence (4) and (5) give c1 y1 (x0) " c2 y2 (x0) ! 0 ... (6) and c1 y1∗ ( x0 ) # c2 y2∗ ( x0 ) ∃ 0. ... (7) Using (2) and (3), (6) , c1 ! 0 and (7) , c2 ! 0. This is a contradiction of the fact that c1 and c2 are not both zero. Hence, our assumption that y1 (x) and y2 (x) are linearly dependent is not possible and so by definition, y1 (x) and y2 (x) must be linearly independent. We now prove the last part of the theorem. Let y (x) be an any solution of (1) satisfying y (x0) ! c1 and y∗ (x0) ! c2. ... (8) Let u (x) ! y (x) – c1 y1 (x) – c2 y2 (x). ... (9) (9) shows that u (x) is a linear combination of solutions y (x), y1 (x) and y2 (x) of (1) and hence u (x) is also a solution of (1). [Refer note for generalisation of theorem I] From (9), u∗ ( x ) ∃ y ∗ ( x ) . c1 y1∗ ( x ) . c2 y2∗ ( x ). ... (10) Now, (9) , u (x0) ! y (x0) – c1 y1 (x0) – c2 y2 (x0) ! 0, by (2), (3) and (8). and (10) , u∗ ( x0 ) ∃ y ∗ ( x0 ) . c1 y1∗ ( x0 ) . c2 y 2∗ ( x0 ) ∃ 0, by (2), (3) and (8). and

Thus, we find that u (x) is a solution of (1) satisfying u (x0) ! 0 and u∗ (x0) ! 0. Hence, u (x) < 0 on (a, b) [Refer corollary of Art. 1.14] and so by (9), we have

Created with Print2PDF. To remove this line, buy a license at: http://www.software602.com/

Differential Equations their Formation and Solutions

1.16

y (x) – c1 y1 (x) – c2 y2 (x) ! 0 or y (x) ! c1 y1 (x) " c2 y2 (x), where c1 and c2 are suitable chosen constants and are given by (8). Theorem III. Two solutions y1 (x) and y2 (x) of the equation, a0 (x) y+ " a1 (x) y∗ " a2 (x) y ! 0, a0 (x) ; 0, x 9 (a, b) are linearly dependent if and only if their Wronskian is identically zero. [Mumbai 2010; Delhi B.Sc. (Prog.) II 2007, 09; Delhi Maths Prog II 2007, 08] [Delhi Maths (G) 2000; Delhi Maths (H) 2000, 01, 02, 06, 08; Lucknow 1995] Proof. Condition is necessary. Let y1 (x) and y2 (x) be linearly dependent. Then, there must exist two constants c1 and c2, not both zero, such that c1 y1 (x) " c2 y2 (x) ! 0 for each x 9 (a, b). ... (1) From (1), c1 y1∗ ( x ) # c2 y2∗ ( x ) ∃ 0 for each x 9 (a, b). ... (2) Since c1 and c2 cannot be zero simultaneously, the system of simultaneous equation (1) and (2) possess non-zero solutions for which the condition is y1 ( x ) y2 ( x ) W (x) ! ! 0 for each x 9 (a, b) y1∗ ( x ) y2∗ ( x ) , W (x) < 0 on (a, b), i.e., Wronskian is identically zero. Condition is sufficient. Suppose that Wronskian of y1 (x) and y2 (x) is identically zero on (a, b), i.e., let y1 ( x ) y2 ( x ) W (x) ! < 0 on (a, b). ... (3) y1∗ ( x ) y2∗ ( x ) Let x ! x0 9 (a, b). Then from (3), we have y1 ( x0 ) y2 ( x0 ) ! 0. y1∗ ( x0 ) y2∗ ( x0 )

... (4)

Now, (4) is the condition for existence of two constants k1 and k2, both not zero, such that k1 y1 (x0) " k2 y2 (x0) ! 0 ... (5) and k1 y1∗ ( x0 ) # k2 y2∗ ( x0 ) ∃ 0. ... (6) Let y (x) ! k1 y1 (x) " k2 y2 (x). ... (7) Then y (x) being a linear combination of solutions y1 (x) and y2 (x) is also a solution of the given equation. [Refer theorem 1 of Art. 1.16] Again, from (8) y∗ (x) ! k1 y1∗ ( x ) # k 2 y2∗ ( x ). ... (8) y (x0) ! k1 y1 (x0) " k2 y2 (x0) ! 0, using (5) and (8) , y∗ (x0) ! k1 y1∗ ( x0 ) # k2 y2∗ ( x0 ) ∃ 0, using (6). Thus, we find that y (x) is a solution of the given equation such that y (x0) ! 0 and y∗ (x0) ! 0. Hence, y (x) # 0 on (a, b). [Refer corollary of Art. 1.14] and so by (7), we have k1 y1 (x) " k2 y2 (x) ! 0 for each x 9 (a, b), where k1 and k2 are constants, both not zero. Hence, by definition, y1 (x) and y2 (x) are linearly dependent. Corollary to theorem III. Two solutions y 1 (x) and y 2 (x) of the equation a0 (x) y+ " a1 (x) y∗ " a2 (x) y ! 0, a0 (x) ; 0, x 9 (a, b) are linearly independent if and only if their Wronskian is not zero at some point x0 9 (a, b). [Delhi B.A. (Prog) II 2010; Delhi B.Sc. (Hons) II 2011] Proof. Condition is necessary. Let y1 (x) and y2 (x) be linearly independent. Then, by definition, y1 (x) and y2 (x) are not linearly dependent. Hence, by theorem III, we cannot have W (x) < 0 on Now, (7) ,

Created with Print2PDF. To remove this line, buy a license at: http://www.software602.com/

Differential Equations their Formation and Solutions

1.17

(a, b), for otherwise y1 (x) and y2 (x) would become linearly dependent. It follows that there must exist some x0 9 (a, b), such that W (x0) ; 0. Hence the result. Condition is sufficient. Suppose there exist some x0 9 (a, b), such that W (x0) ; 0. Then, it follows that W (x) ; 0 on (a, b) and hence y1 (x) and y2 (x) cannot be linearly dependent by theorem III. So by definition, y1 (x) and y2 (x) must be linearly independent. Theorem IV. The Wronskian of two solutions of the equation, a0 (x) y+ " a1 (x) y∗ " a2 (x) y ! 0, a0 (x) ; 0, x 9 (a, b) is either identically zero or never zero on (a, b). [Delhi B.Sc. (Prog.) II 2009; 2010 Delhi Maths (Hons.) 2005, 07; Lucknow 2001;Nagpur 1997; Delhi Maths (G) 2005, 05] Proof. Given differential equation is a0 (x) y+ " a1 (x) y∗ " a2 (x) y ! 0, a0 (x) ; 0, x 9 (a, b). ... (1) Let y1 (x) and y2 (x) be two solutions of (1). Then their Wronskian W (x) is given by y1 ( x ) y 2 ( x ) ∃ y1 ( x ) y 2∗ ( x ) . y2 ( x ) y1∗ ( x ). W (x) ! ... (2) y1∗ ( x ) y 2∗ ( x )

or or

and

Differentiating both sides of (2) with respect to ‘x’, we get d d [ y1 ( x ) y2∗ ( x )] . [ y2 ( x ) y1∗ ( x )] W ∗ (x) ! dx dx W ∗ (x) ! [ y1∗ ( x ) y2∗ ( x ) # y1 ( x ) y2∗∗ ( x )] . [ y 2∗ ( x ) y1∗ ( x ) # y2 ( x ) y1∗∗ ( x )] W ∗ (x) ! y1 ( x ) y2∗∗ ( x ) . y2 ( x ) y1∗∗ ( x ). Since a0 (x) ; 0, dividing by a0 (x) and re-writing, (1) becomes y+ (x) ! – (a1/a0) y∗ (x) – (a2/a0) y (x). Since y1 (x) and y2 (x) are solutions of (4), we have

... (3) ... (4)

y1∗∗ ( x ) ! . (a1 / a0 ) y1∗ ( x ) . (a2 / a0 ) y1 ( x )

... (5)

y2∗∗ ( x ) ! . (a1 / a0 ) y2∗ ( x ) . (a2 / a0 ) y2 ( x ).

... (6)

Substituting the values of y1∗∗ ( x ) and y2∗∗ ( x ) given by (5) and (6) in (3), we get W ∗ (x) ! y1 ( x ) [. ( a1 / a0 ) y2∗ ( x ) . (a2 / a0 ) y2 ( x )] – y2 ( x ) [. (a1 / a0 ) y1∗ ( x ) . (a2 / a0 ) y1 ( x )] W ∗ (x) ! . (a1 / a0 ) [ y1 ( x ) y2∗ ( x ) . y2 ( x ) y1∗ ( x )] W ∗ (x) ! – (a1/a0) W (x), using (2) ... (7) a0 (x) W ∗ (x) " a1 (x) W (x) ! 0. ... (8) From (8), it follows that W (x) is a solution (8). Now, the following two cases arise: Case I. Let W (x) ; 0 on (a, b). Then the second part of the theorem is proved. Case II. If possible, let W (x0) ! 0 for some x0 9 (a, b). Then, (7) , W ∗ (x0) ! – (a1/a0) W (x0) ! 0. Thus, W (x) is a solution of (8), such that W (x0) ! 0 and W ∗ (x0) ! 0. Hence, W (x) < 0 on (a, b), i.e., Wronskian is identically zero on (a, b). [Refer corollary of Art. 1.14] This proves the first part of the theorem. Theorem V. The nth order homogeneous linear equation, a0 (x) (dny/dxn) " a1 (x) (dn–1y/dxn–1) " ... " an–1 (x) (dy/dx) " an (x) y ! 0 always possesses n independent solutions y1 (x), y2 (x), ..., yn (x) and its general solution is given by y ! c1 y1 (x) " c2 y2 (x) " ... " cn yn (x), where c1, c2, ..., cn are n arbitrary constants. Proof. Left for the reader. or or or

Created with Print2PDF. To remove this line, buy a license at: http://www.software602.com/

Differential Equations their Formation and Solutions

1.18

Theorem VI. Let p1 (x), p2 (x), ......, pn (x) be real or complex valued functions defined and continuous on an internal I and =1, =2, ......, =n are n solutions of the equation L (y) (x) = y(n) (x) + p1 (x) y(n–1) (x) + ...... + pn (x) y (x) = 0, x 9 I existing on I. Then n solutions are linearly independent on I if and only if W (x) ; 0 for every x 9 I. [Himachal 2008] Sol. The condition is necessary. Let there be a point x0 9 I such that

W (=1, =2, ......, =n) (x0) ! W (x0) !

=1 ( x0 ) =1∗ ( x0 )

=2 ( x0 )...... =n ( x0 ) =2∗ ( x0 )...... =∗n ( x0 )

................................................... =1( n .1) ( x0 )

; 0 ... (1)

=n2 .1 ( x0 )...... =(nn .1) ( x0 )

Let there exist constants c1, c2, ......, cn such that c1 =1 (x) + c2=2(x) + ...... + cn =n (x) ! 0; for each x 9 I In order to prove that =1, =2, ......, =n are linearly independent, we must show that c1 ! c2 ! ...... ! cn ! 0. Differentiating (2) successively w.r.t. ‘x’, we have c1 =1∗ ( x) # c2 =∗2 ( x) # ...... # cn =∗n ( x) ∃ 0; for each x 9 I c1=1∗∗( x ) # c2 =∗∗2 ( x) # ...... # cn =∗∗n ( x) ∃ 0; for each x 9 I

6 > > 7 ....................................................................... > c1 =1(n .1) ( x ) # c2 =(2n .1) ( x) # ...... # cn =(nn .1) ( x) ∃ 0; for each x 9 I >8 At x ! x0 9 I, (2) and (4) yield

... (2) ... (3)

... (4)

c1 =1 ( x0 ) # c2 =2 ( x0 ) # ...... # cn =n ( x0 ) ∃ 0 c1 =1∗ ( x0 ) # c2 =∗2 ( x0 ) # ...... # cn =∗n ( x0 ) ∃ 0

6 > > ... (5) 7 ............................................................... > c1 =1(n .1) ( x0 ) # c2 =(2n .1) ( x0 ) # ...... # cn =(nn .1) ( x0 ) ∃ 0>8 (5) represents a system of n simultaneous homogeneous equations in c1 , c2, ......, c n as n unknown constants. The determinant of the coefficients of the above n equations (5) is clearly W (=1, =2, ......, =n) (x0) or W (x0), which is non-zero by (1). Hence, there is only one solution of the system (5), namely c1 ! c2 ! ...... ! cn ! 0. Hence, =1, =2, ......, =n are linearly independent on I.

The condition is sufficient. Suppose, that solutions =1, =2, ......, =n of L (y) (x) ! y(n) (x) + p1 (x) y(n–1) (x) + ...... + pn (x) y (x) ! 0, x 9 I

... (6)

are linearly independent. Suppose, if possible, there is an x0 9 I such that W (=1, =2, ......, =n) (x0) ! W (x0) ! 0

... (7)

Then (7) implies that the system (5) has a solution c1, c2, ......, cn where not all the constants c1, ......, cn are zero. Let c1, c2, ......, cn be such a non-trivial solution of (5) and consider the function ? (x) such that ? (x) ! c1 =1 (x) + c2=2(x) + ...... + cn =n (x), for each x 9 I

... (8)

Since, =1, =2, ......, =n are solutions of (6), we have L (=1) ! L (=2) ! ...... ! L (=n) ! 0. Then, from (8) and (9),

L (?) ! 0,

... (9) ... (10)

Created with Print2PDF. To remove this line, buy a license at: http://www.software602.com/

Differential Equations their Formation and Solutions

1.19

showing that ? is a solution (6). From (5) and (8), we have ? (x0) ! 0, ?∗ (x0) ! 0, ......, ?(n–1) (x0) ! 0 ... (11) Thus, ? (x) is a solution of (6) satisfying the initial conditions (11). From the existence and uniqueness theorem (refer part (a) of Art. 1.14A), it follows that ? (x) ! 0 for all x in I, and hence from (8), c1 =1 (x) + ...... + cn =n (x) ! 0 for all x 9 I. where all c1, c2, ......, cn are not simultaneously zero, leading to conclusion that =1, =2, ......, =n are linearly dependent which contradicts the fact that =1, ......, =n are linearly independent on I. Thus, the supposition that there was a point x0 9 I such that (1) holds must be false. Consequently, we must have W (=1, =2, ......, =n) (x) ; 0 for all x in I Theorem VII. Abel’s formula Let functions p1 (x) and p2 (x) in L (y) (x) ! y+ (x) + p1 (x) y∗ (x) + p2 (x) y (x) ! 0, x 9 I ... (i) be defined and continuous on an interval I. Let =1, any =2 be two linearly independent solutions of (1) existing on I containing a point x0. Then,

Α

Β

x

W (=1, =2) (x) ! exp . Χ p1 ( x ) dx W (=1, =2) (x0) x0

... (ii)

[Note that here exp A stands for eA] [Delhi B.Sc. (Hons) II 2011] Proof. Given y+ (x) + p1 (x) y∗ (x) + p2 (x) y (x) ! 0, x 9 I ... (iii)

W ∗ ( =1 , =2 ) ∃ =1∗ =2∗ # =1 =∗∗2 . ( =1∗∗ =2 # =1∗ =∗2 )

From (iii), or

=1 =2 ∃ =1 =∗2 . =1∗ =2 =1∗ =∗2

W (=1 , =2 ) ∃

Here,

W ∗ ( =1 , =2 ) ∃ =1 =∗∗2 . =1∗∗ =2 Since =1 and =2 satisfy (i), we have =1∗∗ # p1 =1∗ # p2 =1 ∃ 0 , =1∗∗ ∃ . p1 =1∗ . p2 =1

... (iv)

=∗∗2 # p1 =∗2 # p2 =2 ∃ 0 , =∗∗2 ∃ . p1 =∗2 . p2 =2

and

Substituting the above values of =1∗∗ and =∗∗2 in (iv), we get or

W∗ (=1, =2 ) ! =1 ( . p1 =1∗ . p2 =1 ) . =2 ( . p1 =2∗ . p2 =2 ) W ∗ (=1 , =2 ) ∃ . p1 (=1 =∗2 . =2 =1∗ ) ∃ . p1 W (=1 , =2 ) , by (iii) Thus, W (=1, =2) satisfies a first order linear homogeneous equation W∗ + p1 W ! 0, x 9 I

or so that

dW ∃ . p1 W dx

or

dW ∃ . p1 dx W

Α

W (=1 , =2 ) ( x ) ∃ c exp . Χ

x

or x x0

p1 ( x ) dx

Β

log W . log c ∃ . Χ p1 dx x0

... (v)

where c is a constant. By putting x ! x0 in (v), we get c ! W (=1, =2) (x0). Substituting this value of c in (v), we get the required result. We now state and prove Abel’s formula for general case: Statement. Let the functions p1 (x), p2(x),......, pn (x) in L (y) (x) ! y(n) (x) + p1 (x) y(n–1) (x) + p2(x)y(n–2) (x) + ...... + pn (x) y (x) ! 0, x 9 I ... (1) be defined and continous on an interval I. Let =1, =2 ,......, =n be n linearly independent solutions of (1) existing on I containing a point x0. Then, we have

Α

W (=1 , ......, =n ) ( x ) ∃ exp . Χ

x x0

Β

p1 ( x ) dx W ( =1 , ......, =n ) ( x0 )

... (2)

Created with Print2PDF. To remove this line, buy a license at: http://www.software602.com/

Differential Equations their Formation and Solutions

1.20

Proof. We let W (x) ! W (=1, =2, ......, =n) (x) for brevity. We have,

W ( x ) ∃ W (=1............., =n ) ( x ) ∃

=1 =1∗

=2 =∗2

...... ......

=n =∗n

=1∗∗ ......

=∗∗2 ......

...... ......

=∗∗n ......

=1( n .1)

=2( n .1)

... (3)

...... =(nn .1)

Differentiating W (x) w.r.t. ‘x’, we see that W∗ is a sum of n determinants. W∗ (x) ! V1 + V2 + ...... + Vi + ...... + Vn, ... (4) where Vi differs from W (x) only in the i th row, and the i th row of Vi is obtained by differentiating the i th row of W (x). Thus, we arrive at

W ∗ ( x) ∃

=1∗ =1∗ =1∗∗

=∗2 =∗2 =2∗∗

......

......

=1( n .1)

=(2n .1)

=n∗ =n∗ =n∗∗

=1 =1∗∗ =1∗∗

=2 =2∗∗ =2∗∗

......

......

......

...... =(nn .1)

=1( n .1)

=(2n .1)

...... ...... ...... ......

#

...... ...... ...... ......

=n = ∗∗n = ∗∗n

=1 =1∗ # . # =1∗∗ ...... ......

...... =(nn .1)

=1( n )

=2 =2∗ =2∗∗

......

=n =∗n =n∗∗

=(2n)

...... =(nn)

...... ...... ...... ...... ......

The first n – 1 determinants V1, ......, Vn–1 are all zero, since they each have two identical rows. Thus, we obtain

W ∗ ( x) ∃

=1 =1∗

=2 =∗2

......

......

=1( n .2) =1( n )

=2( n .2) =(2n )

...... ...... ......

=n =∗n ......

... (5)

...... =(nn .2) ......

=(nn )

Now, since =1, =2, ......, =n are solutions of (1), we have

=i( n ) # p1 =i( n .1) # p2 =i( n . 2) # ...... # pn =i ∃ 0 for i ∃ 1, 2, ......, n =(i n ) ∃ . p1 =i( n .1) . p2 =i( n .2) . ...... . pn =i for i ∃ 1, 2, ......, n , ... (6) (n) (n) (n) Putting i ! 1, 2, ......, n in (6), get values of =1 , =2 , ......, =n and substitute these values in (5) and obtain

W∗ ∃

. p1=1( n .1) .

=1 =1∗ =1∗∗

=2 =∗2 =∗∗2

=n =∗n =∗∗n

......

......

......

=1( n . 2) p2 =1( n . 2)

=(2n .2) p2 =(2n .2)

=(nn .2)

...... . pn =1

. p1=(2n .1) .

...... . pn =2

. p1=(nn .1) . p2 =(nn .2) ...... . pn =n

From the properties of a determinant, we know that the value of the above determinant is unchanged, if we multiply any row by a number and add to the last row. Multiplying the first row by pn, the second row by pn–1, ......, the (n – 1) – st row by p2 and adding these to the last row, we obtain

Created with Print2PDF. To remove this line, buy a license at: http://www.software602.com/

Differential Equations their Formation and Solutions =1 =1∗

=2 =2∗

.

.

=n =∗n

...... ...... ......

W ∗ ( x) ∃ =1( n .2) p1 =1( n .1)

1.21

=2( n .2) p1 =(2n .1)

∃ . p1 W ( x ), by (3)

=(nn . 2) p1 =(nn .1)

...... ...... .

Thus, W satisfies a first order linear differential equation dW/dx ! – pW or log W – log c ! . Χ

Integrating,

Α

W ( x) ∃ c exp . Χ

so that

x x0

x

x0

(1/W) dW ! – p1 dx

p1 dx

Β

p1 ( x ) dx ,

... (7)

where c is an arbitrary constant. By putting x ! x0 in (4), we get c ! W (x0). Substituting this value of c in (4), we get

Α W (= , = , ....., = ) ( x ) ∃ exp Α . Χ

x

Β p ( x ) dx Β W ( = , = , ......, = ) ( x )

W ( x ) ∃ exp . Χ p1 ( x) dx W ( x0 )

i.e.,

1

2

n

x0

x

x0

1

1

2

n

0

... (8)

Corollary. If the coefficients p1 (x), p2 (x), ......, pn (x) of equation (1) are constants, then W (=1, =2, ......, =n) (x) ! e . p1 ( x . x0 ) W (=1, =2, ......, =n)(x0) x

Χx

Proof. If p1 is constant, then we have / (8) yields,

0

p1 dx ∃ p1 Χ

x x0

dx ∃ p1 ( x . x0 )

W (=1, =2,......, =n) (x) ! exp [– p1 (x – x0)] W (=1, =2, ......, =n) (x0)

W (=1, =2, ......, =n) (x) ! e . p1 ( x . x0 ) W (=1, =2 , ......, =n) (x0) An important application of Abel’s formula. A consequence of Abel’s formula is that n solutions =1, =2, ......, =n of equation (1) on an interval I are linearly independent there if and only if W (=1, =2, ......, =n) (x0) ; 0 for any particular x0 in interval I. From (5), it is clear that if W (=1, =2, ......, =n) (x0) ; 0, then W (=1, =2, ......, =n) (x) ; 0 for x 9 I. Hence, it is enough to show that W (=1, =2, ......, =n) (x) ; 0 only at just one point of I. This criterion yields the linear independence of n solutions of (1). We now given a simple illustration of the use of Abel’s formula. Example. To show that solutions =1 (x) ! e2x, =2 (x) ! xe2x and =3 (x) ! x2 e2x are linearly independent solutions of y+∗ – 6y+ + 12y∗ – 8y ! 0 on an interval 0 ∆ x ∆ 1. Solution. We have i.e.,

e2 x

xe2 x

x 2 e2 x

d (e 2 x ) / dx

d ( x e 2 x ) / dx

d ( x 2 e 2 x ) / dx

d 2 (e 2 x ) / dx 2

d 2 ( x e 2 x ) / dx 2

d 2 ( x 2 e 2 x ) / dx 2

xe 2 x

x 2 e2 x

W (=1 , =2 , =3 ) ( x ) ∃ 2e 2 x

(1 # 2 x ) e 2 x

(2 x # 2 x 2 ) e 2 x

2x

2x

W (=1 , =2 , =3 ) ( x ) ∃

e2 x

or

4e

(4 # 4 x ) e

2

(2 # 8 x # 4 x ) e

... (1)

2x

Created with Print2PDF. To remove this line, buy a license at: http://www.software602.com/

Differential Equations their Formation and Solutions

1.22

Clearly, it is not very easy to evaluate R.H.S. of (1). We chose 0 9 [0, 1]. Then, from (1), 1 0 0 W (=1 , =2 , =3 ) (0) ∃ 2 1 0 ∃ 2 4 4 2

... (2)

By corollary of Abel’s formula, we have W (=1, =2, =3) (x) ! e . p1 ( x . x0 ) W (=1, =2, =3) (x0) Here p1 ! – 6 and x0 ! 0. Hence, (3) reduces to W (=1, =2, =3) (x) ! 2e6x, using (2).

... (3)

1.17 Solved examples based on Art. 1.16 Ex. 1. If y1 (x) ! sin 3x and y2 (x) ! cos 3x are two solutions of y+ " 9y ! 0, show that y1 (x) and y2 (x) are linearly independent solutions. [Delhi Maths (Hons) 1996] Sol. The Wronskian of y1 (x) and y2 (x) is given by y1 ( x ) y2 ( x ) sin 3x cos 3x ∃ W (x) ! y1∗ ( x ) y2∗ ( x ) 3 cos 3x . 3 sin 3x ! – 3 sin2 3x – 3 cos2 3x ! – 3 (sin2 3x " cos2 3x) ! – 3 ; 0. Since W (x) ; 0, y1 (x) and y2 (x) are linearly independent solutions of y+ " 9y ! 0. Ex. 2. Prove that sin 2x and cos 2x are solutions of y+ " 4y ! 0 and these solutions are linearly independent. [Delhi Maths (G) 1998] Sol. Given equation is y+ " 4y ! 0. ... (1) Let y1 (x) ! sin 2x and y2 (x) ! cos 2x. ... (2) Now,

y∗1 (x) ! 2 cos 2x

y∗∗1 (x) ! – 4 sin 2x.

and

... (3)

/ y1∗∗ ( x ) # 4 y1 ( x ) ! – 4 sin 2x " 4 sin 2x ! 0, by (2) and (3) Hence, y1 (x) ! sin 2x is a solution of (1). Similarly, we can prove that y2 (x) is a solution of (1). Now, the Wronskian W (x) of y1 (x) and y2 (x) is given by y1 ( x ) y2 ( x ) sin 2 x cos 2 x ∃ W (x) ! y1∗ ( x ) y2∗ ( x ) 2 cos 2 x . 2 sin 2 x ! – 2 sin2 2x – 2 cos2 2x ! – 2 (sin2 2x " cos2 2x) ! – 2 ; 0. Since W (x) ; 0, sin 2x and cos 2x are linearly independent solutions of (1). Ex. 3. Show that linearly independent solutions of y + – 2y∗ " 2y ! 0 are ex sin x and ex cos x. What is the general solution? Find the solution y (x) with the property y (0) ! 2, y∗ (0) ! 3. [Delhi B.A. (Prog.) 2009; Delhi Maths (Hons) 2002; Delhi Maths (G) 2006] Sol. Given equation is y + – 2y∗ " 2y ! 0. ... (1) x x Let y1 (x) ! e sin x and y2 (x) ! e cos x. ... (2) x x x From (2), y1∗ ( x ) ! e sin x " e cos x ! e (sin x " cos x) ... (3) From (3),

y1∗∗ ( x ) ! ex (sin x " cos x) " ex (cos x – sin x) ! 2ex cos x.

... (4)

/ y1∗∗ ( x ) . 2 y1∗ ( x ) # 2 y1 ( x ) ! 2e cos x – 2e (sin x " cos x) " 2e sin x ! 0, showing that y1 (x) ! ex sin x is a solution of (1). Similarly, we can show that y2 (x) ! ex cos x is a solution of (1). Now, the Wronskian W (x) of y1 (x) and y2 (x) is given by x

x

x

Created with Print2PDF. To remove this line, buy a license at: http://www.software602.com/

Differential Equations their Formation and Solutions W (x) !

y1 ( x ) y1∗ ( x )

y2 ( x ) e x sin x ∃ y2∗ ( x ) e x (sin x # cos x )

1.23

e x cos x e x (cos x . sin x )

! e2x (sin x cos x – sin2 x) – e2x (sin x cos x " cos2 x) ! – e2x ; 0, showing that W (x) ; 0, and hence y1 (x) and y2 (x) are linearly independent solutions of (1). The general solution of (1) is [Refer theorem V, Art. 1.16] y (x) ! c1 y1 (x) " c2 y2 (x) ! ex (c1 sin x " c2 cos x), ... (5) where c1 and c2 are arbitrary constants. From (5), y∗(x) ! ex (c1 sin x " c2 cos x) " ex (c1 cos x – c2 sin x). ... (6) Putting x ! 0 in (5) and using the given result y (0) ! 2, we get y (0) ! c2 or c2 ! 2 Putting x ! 0 in (6) and using the given result y∗ (0) ! – 3, we get y∗ (0) ! c2 " c1 or – 3 ! 2 " c2 or c1 ! – 5, as c2 ! 2 / From (5), solution of (1)satisfying the given properties is y ! ex (2 cos x – 5 sin x). Ex. 4. Show that e2x and e3x are linearly independent solutions of y+ – 5y∗ " 6y ! 0. Find the solution y (x) with the property that y (0) ! 0 and y∗ (0) ! 1. [Delhi Maths (G) 98, 2006] Sol. Given equation is y+ – 5y∗ " 6y ! 0. ... (1) Let y1 (x) ! e2x and y2 (x) ! e3x. ... (2) 2x 2x From (2), y1∗ ( x ) ! 2e and y1∗∗ ( x ) ! 4e . ... (3) / y1∗∗ ( x ) . 5 y1∗ ( x ) # 6 y1 ( x ) ! 4e2x – 5 (2e2x) " 6e2x ! 0, showing that y1 (x) is a solution of (1). Similarly, y2 (x) ! e3x is a solution of (1). Now, the Wronskian W (x) of y1 (x) and y2 (x) is given by y2 ( x ) e2 x e3 x ∃ ! 3e5x – 2e5x ! e5x ; 0, y2∗ ( x ) 2 e2 x 3e 3 x showing that e2x and e3x are linearly independent solutions of (1). The general solution of (1) is given by y (x) ! c1e2x " c2e3x, c1 and c2 being arbitrary constants. ... (4) 2x 3x From (4), y∗ (x) ! 2c1e " 3c2e . ... (5) Putting x ! 0 in (4) and using y (0) ! 0, c1 " c2 ! 0. ... (6) Putting x ! 0 in (5) and using y∗ (0) ! 1, 2c1 " 3c2 ! 1. ... (7) Solving (6) and (7), c1 ! – 1 and c2 ! 1 and so from (4), we have y (x) ! e3x – e2x as the required solution. Ex. 5. (a) Show that y 1 (x) ! sin x and y 2 (x) ! sin x – cos x are linearly independent solutions of y+ " y ! 0. Determine the constants c 1 and c 2 , so that the solution sin x " 3 cos x < c1 y1 (x) " c2 y2 (x). [Delhi Maths (P) 2002] Sol. Given equation is y+ " y ! 0. ... (1)

W (x) !

Here

y1 ( x ) y1∗ ( x )

y1 (x) ! sin x,

so that

y1∗ ( x ) ! cos x

and

y1∗∗ ( x ) ! – sin x.

... (2)

Hence, y1∗∗ ( x ) # y1 ( x ) ! – sin x " sin x ! 0, showing that y1 (x) is a solution of (1). Similarly,, we can show that y2 (x) is also a solution of (1). Now, the Wronskian of y1 (x) and y2 (x) is given by y1 ( x ) y 2 ( x ) sin x sin x . cos x ∃ W (x) ! y1∗ ( x ) y 2∗ ( x ) cos x cos x # sin x ! sin x (cos x " sin x) – cos x (sin x – cos x) ! 1 ; 0,

Created with Print2PDF. To remove this line, buy a license at: http://www.software602.com/

Differential Equations their Formation and Solutions

1.24

showing that y1 (x) and y2 (x) are linearly independent solutions of (1). Given that sin x " 3 cos x < c1 y1 (x) " c2 y2 (x) or sin x " 3 cos x < c1 sin x " c2 (sin x – cos x). ... (3) Comparing the coefficients of sin x and cos x on both sides of (3), we have c1 " c2 ! 1 and – c2 ! 3 so that c1 ! 4 and c2 ! – 3. Ex. 5. (b) Define Wronskian. Evaluate Wronskian of the functions y1(x) ! sin x and y2(x) ! sin x – cos x and hence conclude whether or not they are linearly independent. Also, form the differential equation. [Meerut 2003] Sol. For definition of Wronskian, refer Art. 1.12 For second part, refer Ex. 5(a) For the last part, proceed as follows: Since sin x and sin x – cos x are linearly independent functions, these will form solution of a differential equation of the form y = A sin x + B (sin x – cos x), A, B being parameters. ...(1) Differentiating (1) w.r.t. ‘x’, we have y' ! A cos x + B (cos x + sin x) ...(2) From (2), y'' ! – A sin x + B (–sin x + cos x) ...(3) Adding (1) and (3), y + y'' ! 0, which in the desired differential equation. Ex. 6. Show that x and xex are linearly independent on the x-axis. Sol. The Wronskian W (x) of x and xex is given by x xe x x xe x ∃ W (x) ! ! x (ex " xex) – xex ! x2ex. dx / dx d ( xe x ) / dx 1 e x # xe x Thus, W (x) ; 0 for x ; 0 on the x-axis. Hence, x and xex are linearly independent on the x-axis [Refer corollary to theorem III of Art. 1.16] Ex. 7. Show that the Wronskian of the functions x 2 and x 2 log x is non-zero. Can these functions be independent solutions of an ordinary differential equation. If so, determine this differential equation. [Meerut 1998] 2 2 Sol. Let y1 (x) ! x and y2 (x) ! x log x. The Wronskian W (x) of y1 (x) and y2 (x) is given by, y2 x2 x 2 log x ∃ ! x2 (2x log x " x) – 2x3 log x. y 2∗ 2 x 2 x log x # x / W (x) ! x3, which is not identically equal to zero on (– 3, 3). Hence, functions y1 (x) and y2 (x), i.e., x2 and x log x can be linearly independent solutions of an ordinary differential equation. To form the required differential equation. The general solution of the required differential equation may be written as, y ! A y1 (x)" B y2 (x) ! Ax2 " Bx2 log x, ... (1) where A and B are arbitrary constants. Differentiating (1), we get y∗ ! 2Ax " B (2x log x " x). ... (2) Differentiating (2), we get y+ ! 2A " B (2 log x " 2 " 1). ... (3) We now eliminate A and B from (1), (2) and (3). To this end, we first solve (2) and (3) for A and B. Multiplying both sides of (3) by x, we get xy+ ! 2Ax " B (3x " 2x log x). ... (4) Subtracting (2) from (4), xy+ – y∗ ! 2Bx or B ! (xy+ – y∗)/2x. Substituting this value of B in (3), we have 2A ! y+ – (1/2x) × (xy+ – y∗) (3 " 2 log x) or A ! (1/4x) × [2xy+ – (xy+ – y∗) (3 " 2 log x)].

W (x) !

y1 y1∗

Created with Print2PDF. To remove this line, buy a license at: http://www.software602.com/

Differential Equations their Formation and Solutions

1.25

Substituting the above values A and B in (1), we have y ! (x/4) × [2xy+ – 3xy+ " 3y∗ – 2xy+ log x " 2y∗ log x] " (x/2) × (xy+ – y∗) log x or 4y ! x (– xy+ " 3y∗ – 2xy+ log x " 2y∗ log x) " 2x (xy+ – y∗) log x or x2y+ – 3xy∗ " 4y ! 0, which is the required equation. Ex. 8. Find the Wronskian of x and xex. Hence, conclude whether or not these are linearly independent. If they are independent, set up the differential equation having them as its independent solutions. [Meerut 1997] Sol. Let y1 ! x and y2 ! xex. Then their Wronskian W (x) is given by y1 y 2 x xe x ∃ W (x) ! ! xex " x2ex – xex ! x2ex, y1∗ y 2∗ 1 e x # xe x which is not identically equal to zero on (– 3, 3). Hence, y1 and y2 are linearly independent. To form the required differential equation. The general solution of the required differential equation may be written as y ! Ay1 " By2 ! Ax " Bx ex, ... (1) where A and B are arbitrary constants. Differentiating (1), we get y∗ ! A " B (ex " xex) ! A " B (1 " x) ex. ... (2) x x x Differentiating (2), we get y+ ! B [e " (1 " x) e ] ! Be (2 " x). ... (3) We now eliminate A and B from (1), (2) and (3). From (3), B ! y+/[ex (2 " x)]. Substituting this value of B in (2), we have 1# x (2 # x ) y ∗ . (1 # x ) y + A ! y∗ – B (1 " x) ex ! y∗ – y+ ! . 2#x 2#x Substituting the above values of A and B in (1), we get Ε Φ x Ε (2 # x ) y ∗ . (1 # x ) y + Φ y+ x#Γ x y! Γ Η xe Η 2# x Ι ϑ Ι e (2 # x ) ϑ

(2 " x) y ! x (2 " x) y∗ – x (1 " x) y+ " xy+ x2y+ – x (2 " x) y∗ " (2 " x) y ! 0, which is required equation. Ex. 9. (a) Show that the solutions ex, e–x, e2x of (d3y/dx3) – 2 (d2y/dx2) – (dy/dx) " 2y ! 0 are linearly independent and hence or otherwise solve the given equation. [Delhi Maths (G) 1993, 98; Meerut 1998] Sol. Given equation is y+∗ – 2y+ – y∗ " 2y ! 0. ... (1) x –x 2x Let y1 ! e , y2 ! e and y3 ! e ... (2) or or

Here

y∗1 ! ex,

y∗∗1 ! ex

and

x y∗∗∗ 1 ! e .

... (3)

/ y1∗∗∗ . 2 y1∗∗ . y1∗ " 2y1 ! e – 2e – e " 2e ! 0, by (2) and (3) x Hence, y1 ! e is a solution of (1). Similarly, show that e–x and e2x are also solutions of (1). Now, the Wronskian W (x) of y1, y2, y3 is given by x

W (x) !

y1 y 2 y1∗ y 2∗ y1∗∗ y 2∗∗

ex y3 y3∗ ∃ e x y3∗∗ ex

x

x

e. x

e2 x

. e. x

2e 2 x

e. x

4e 2 x

x

1 1 1 1 0 0 [using operations 2x ! (e 2 e 2 e ) 1 . 1 2 ∃ e 1 . 2 1 C2 Κ C2 . C1 1 1 4 1 0 3 C3 Κ C3 . C1 ] ! – 6e2x, which is not identically zero on (– 3, 3) x

.x

2x

Created with Print2PDF. To remove this line, buy a license at: http://www.software602.com/

3

1.26

Differential Equations their Formation and Solutions

Hence, y1, y2, y3 are linearly independent solutions of (1) [Refer corollary of theorem III of Art. 1.16]. Since the order of the given equation (1) is three, it follows that the general solution of (1) will contain three arbitrary constants c1, c2, c3 and is given by [Refer Theorem V of Art. 1.16] y ! c1 y1 " c2 y2 " c3 y3 , i.e., y ! c1ex " c2e–x " c3e2x. Ex. 9. (b) Show that the e –x, e3x, e4x are linearly independent solutions of d3y/dx 3 – 6(d2y/dx2) + 5 (dy/dx) + 12y = 0 on the interval – 3 < x < 3 are write the general solution. [Delhi B.A (Prog) II 2011] Hint. Try yourself as in Ex. 9. (a) Ans. y ! c1e–x " c2e3x " c4e4x. Ex. 10. Prove that the functions 1, x, x2 are linearly independent. Hence, form the differential equation whose solutions are 1, x, x2. [Meerut 1997] Sol. Let y1 (x) ! 1, y2 (x) ! x and y3 (x) ! x2. ... (1) Then the Wronskian W (x) of y1, y2, y3 is given by y1 y2 W ( x ) ∃ y1∗ y 2∗ y1∗∗ y2∗∗

1 x x2 y3 y3∗ ∃ 0 1 2 x , using (1) y3∗∗ 0 0 2

W (x) ! 2 ; 0 for any x 9 (– 3, 3). Hence, y1, y2 and y3 are linearly independent. To form the required differential equation. The general solution of the required differential equation may be written as y ! Ay1 " By2 " Cy3 ! A " Bx " Cx2, ... (1) where A, B, C are arbitrary constants. Differentiating (1) w.r.t. ‘x’, we get y∗ ! B " 2Cx. ... (2) Differentiating (2) w.r.t. ‘x’, we get y+ ! 2C. ... (3) Differentiating (3) w.r.t. ‘x’, we get y+∗ ! 0, i.e., d3y/dx3 ! 0. ... (4) Since (4) is free from arbitrary constants hence (4) is the required differential equation. Ex. 11. Use Wronskian to show that x, x2, x3 are independent. Determine the differential equation with these as independent solutions. [Meerut 1995, 2001, 2002] 2 Sol. Let y1 (x) ! x, y2 (x) ! x and y3 ! x3 . ... (1) The Wronskian W (x) of y1, y2 and y3 is given by x x 2 x3 y1 y2 y3 or

W ( x ) ∃ y1∗ y 2∗ y1∗∗ y2∗∗

y3∗ ∃ 1 2 x 2 x 2 , using (1) y3∗∗ 0 2 6x

or W (x) ! x (12x2 – 6x2) – (1) × (6x3 – 2x3) ! 2x3, which is not identically equal to zero. Hence, the functions y1, y2 and y3 are linearly independent. To form the differential equation. The general solution of the required differential equation may be written as y ! Ay1 " By2 " Cy3 ! Ax " Bx2 " Cx3. ... (2) Differentiating (2) w.r.t. ‘x’, we get y∗ ! A " 2Bx " 3Cx2. ... (3) Differentiating (3) w.r.t. ‘x’, we get y+ ! 2B " 6Cx. ... (4) Differentiating (4) w.r.t. ‘x’, we get y+∗ ! 6C. ... (5) From (5), C ! y+∗/6. Then, from (4), B ! (y+ – xy+∗)/2. ... (6) 2 3 Multiplying both sides of (3) by x, xy∗ ! Ax " 2Bx " 3Cx . ... (7) Subtracting (7) from (2), we get y – xy∗ ! – Bx2 – 2Cx3 or y – xy∗ ! – (1/2) × x2 (y+ – xy+∗) – (2x3) × (y+∗/6), using (5) and (6)

Created with Print2PDF. To remove this line, buy a license at: http://www.software602.com/

Differential Equations their Formation and Solutions

1.27

or 6y – 6xy∗ ! – 3x2y+ " 3x3y+∗ – 2x3y+∗ or x3y+∗ – 3x2y+ " 6xy∗ – 6y ! 0, which is the required differential equation. Ex. 12. If y1 (x) and y2 (x) are linearly independent solutions of the differential equation a0 (x) y+ " a1 (x) y∗ " a2 (x) y ! 0, then show that any other solution of the equation can be written in the form y (x) ! C1 y1 (x) " C2 y (x), where C1 and C2 are suitably chosen constants. Sol. Refer theorem I of Art. 1.16. [Delhi Maths (G) 2004] Ex. 13. If y1 (x) and y2 (x) are linearly independent solutions of a0 (x) y+ " a1 (x) 2 y∗ (x) " a2 (x) y ! 0, then prove that every other solution of the equation is a linear combination of two solutions y1 (x) and y2 (x). Hence, show that every solution of d2y/dx2 " y ! 0 is a linear combination of cos x " sin x and cos x – sin x. [Delhi Maths (H) 2004] Sol. Ist part. See theorem I of Art. 1.16. Second part. Given d2y/dx2 " y ! 0 or (D2 " 1) y ! 0, D < d / dx ... (1) / Solution of (1) is y ! A cos x " B sin x, A, B being constants. ... (2) Let y1 (x) ! cos x " sin x and y2 (x) ! cos x – sin x ... (3) From (3), dy1/dx ! – sin x " cos x and d2y1/dx2 ! – cos x – sin x ... (4) 2 2 From (3) and (4), d y1/dx " y1 (x) ! – cos x – sin x " cos x " sin x ! 0, showing that y1 (x) is a solution of (1). Similarly, y2 (x) is also a solution of (1). Now, the Wronskian W (x) of y1 (x) and y2 (x) is given y1 ( x ) y2 ( x ) cos x # sin x cos x . sin x ∃ W (x) ! y1∗ ( x ) y2∗ ( x ) . sin x # cos x . sin x . cos x ! – (cos x " sin x)2 – (cos x – sin x)2 ! – 2 ; 0. Since the Wronskian of y1 (x) and y2 (x) is non-zero, it follows that y1 (x) and y2 (x) are linearly independent solutions of (1). Hence by the first part, every solution of (1) will be of the form y ! C1 y1 (x) " C2 y2 (x), i.e., y ! C1 (cos x " sin x) " C2 (cos x – sin x) ! (C1 " C2) cos x " (C1 – C2) sin x or y ! A cos x " B sin x, taking A ! C1 " C2 and B ! C1 – C2 which is the same as (2). Hence, the required result follows. Ex. 14. Show that sin x, cos x and sin x – cos x are solutions of the differential equation y+ + y ! 0, where y∗ ! dy/dx. Prove that these solutions are linearly dependent. (Use the idea of Wronskian) [Delhi Maths (Prog) 2007 ] Sol. Given differential equation is y+ + y ! 0 ... (1) Let y1 ! sin x, y2 ! cos x and y3 ! sin x – cos x ... (2) Then,

y1' ∃ cos x

and

y1" ∃ . sin x

and so

y1" # y1 ∃ 0,

showing that y1 is a solution of (1). Similary, we find that y2 and y3 are also solutions of (1). y1 y 2 y3 sin x cos x sin x . cos x Now, W (y1, y2, y3) ! y1∗ y 2∗ y3∗ ∃ cos x . sin x cos x # sin x y1∗∗ y 2∗∗ y3∗∗ . sin x . cos x . sin x # cos x

Hence,

sin x cos x sin x . cos x cos x . sin x cos x # sin x , by operating R3 Κ R3 + R1 ! 0 0 0 W (y1, y2, y3) ! 0 and therefore y1, y2, y3 are linearly dependent as desired.

Created with Print2PDF. To remove this line, buy a license at: http://www.software602.com/

1.28

Differential Equations their Formation and Solutions Exercise 1(C)

mx m x m x 1. Prove that the Wronskian of the functions e 1 , e 2 , e 3 , is equal to (m1 – m 2) (m2 – m3) (m3 – m1) e ( m1 # m2 # m3 ) x . Are these functions linearly independent.

Ans. Given functions are linearly independent if m 1 ; m 2 ; m3. 2. Test the linear independence of the following sets of functions: (i) sin x, cos x. Ans. Linearly independent (ii) 1 " x, 1 " 2x, x2. Ans. Linearly independent (iii) x2 – 1, x2 – x " 1, 3x2 – x – 1. Ans. Linearly dependent (iv) sin x, cos x, sin 2x. [Meerut 2010] Ans. Linearly independent (v) ex, e–x, sin ax. Ans. Linearly independent (vi) ex, xex, sinh x. Ans. Linearly independent (vii) sin 3x, sin x, sin 3 x. Ans. Linearly dependent x x 3. Show that the functions e cos x and e sin x are linearly independent. Form the differential equation of second order having these two functions as independent solutions. Ans. y+ – 2y∗ " 2y ! 0 x x 4. Evaluate the Wronskian of the functions e and xe . Hence, conclude whether or not they are linearly independent. If they are independent set up the differential equation having them as its independent solutions. Ans. y+ – 2y∗ " y ! 0 5. Show that linearly independent solutions of y+ – 3y∗ " 2y ! 0 are ex and e2x. Find the solution y (x) with the property that y (0) ! 0, y∗ (0) ! 1. [Delhi Maths (G) 2000] Ans. y (x) ! e2x – ex 6. Show that the y1 (x) ! x and y2 (x) ! | x | are linearly independent on the real line, even though the Wronskian cannot be computed. 7. Show graphically that y1 (x) ! x2 and y2 (x) ! x | x | are linearly independent on – 3 < x < 3, however, Wronskian vanishes for every real value of x. 8. Show that ex and e–x are linearly independent solutions of y+ – y ! 0 on any interval. [Lucknow 2001; Nagpur 1996] 9. Show that y1 (x) ! e–x/2 sin ( x 3 / 2) and y2 (x) ! e–x/2 cos ( x 3 / 2) are linearly independent solutions of the differential equation y+ " y∗ " y ! 0. [Delhi Maths (G) 1999, 2000] 10. Using the idea of Wronskian, show that ex cos x and ex sin x are linearly independent solution of y+ – 2y∗ " 2y ! 0. Find the solution with the property that y (0) ! 1 and y∗ (0) ! 2. [Delhi Maths (H) 2004] Hint. Proceed like Ex. 3 of Art. 1.17. Ans. y ! ex (cos x " sin x) 11. Define the Wronskian of two solutions y1 (x) and y2 (x) of the equation a0 (x) y+ + a1 (x) y∗ + a2 (x) y ! 0. [Delhi Maths (G) 2006] Ans. Refer Art. 1.12. Accordingly, the Wronskian W (y1, y2) of y1 and y2 is given by

W ( y1 , y2 ) ∃

y1 ( x ) y1∗ ( x )

y2 ( x ) y2∗ ( x )

1.18 Linear differential equation and its general solution Linear differential equation contains dependent variable and its derivative in their first degree. The most general form of linear differential equation of order n is y(n) " P1 y(n–1) " ... " Pn y ! Q, ... (1) where P1, P2, ..., Pn, Q are functions of x and are assumed to be continuous on interval I. The differential equation y(n) " P1 y(n–1) " ... " Pn y ! 0 ... (2) is said to be associated homogeneous equation of (1). We now state an important theorem without proof. (Proof follows from the well known existence uniqueness theorem of differential equation of nth order, refer Art. 1.14A) In what follows we shall use the following notations y(1) ! dy/dx, y(2) ! d2y/dx2, y1(1) ! dy1/dx,

Created with Print2PDF. To remove this line, buy a license at: http://www.software602.com/

Differential Equations their Formation and Solutions y1(2) ∃ d 2 y1 / dx 2 ,

y2(1) ∃ dy2 / dx

1.29

y2(2) ∃ d 2 y2 / dx 2 , W (1) ∃ dW / dx and so on

and

Theorem I. A solution y (x) of (2), satisfying the initial conditions y (x0) ! y(1) (x0) ! ...... !y (x0) ! 0, is identically zero. As a particular case (n ! 1), we have Theorem II. If a solution of a first-order equation y(1) " Py ! 0 ... (3) vanishes at a single point x0, the solution is identically zero. Theorem III. The Wronskian of two solutions of differential equation y(2) " P y(1) " Q y ! 0, ... (4) where P, Q are either constants or functions of x alone, is either identically zero or never zero. [Delhi Maths (Hons.) 1997, 2000] Proof. Let y1 (x) and y2 (x) be two solutions of (4). Then, we have (n–1)

y1(2) # P y1(1) # Q y1 ∃ 0

y2(2) # P y2(1) # Q y2 ∃ 0

and

, y1(2) ∃ . ( P y1(1) # Q y1 ) and Now, the Wronskian W of y1 and y2 is given by W! (6) ,

y1

y2

y1(1)

y2(1)

y2(2) ∃ . ( P y2(1) # Q y2 ).

∃ y1 y2(1) . y2 y1(1) .

... (5)

... (6)

W (1) ! y1(1) y2(1) # y1 y2(2) . ΕΙ y2(1) y1(1) # y2 y1(2) Φϑ ∃ y1 y2(2) . y1(2) y2

Λ

Μ

Λ

Μ

or

W (1) ! . y1 Py2(1) # Qy2 # y2 Py1(1) # Qy1 , using (5)

or

W (1) ! . P y1 y2(1) . y2 y1(1) ∃ . PW , using (6)

Λ

Μ

, W " PW ! 0, showing that W is identically zero or never zero (refer theorem II). Theorem IV. Consider the linear differential equation y(2) " P y(1) " Q y ! 0, ... (7) where P, Q are either constants or functions of x alone. Then two solutions of (7) are linearly dependent if and only if their Wronskian vanishes identically. Proof. Let y1 (x) and y2 (x) be solutions of (7). Let W be the Wronskian of y1, y2, so that (1)

W (x) !

y1 ( x ) y1(1)

( x)

y2 ( x ) y2(1) ( x )

Assume that W (x) < 0. If x0 be any point, then we have W (x0) < 0,

so that

y1 ( x0 )

y2 ( x0 )

y1(1) ( x0 )

y2(1) ( x0 )

!0

, There exist constants c1 and c2, not both zero, such that

c1 y1 ( x0 ) # c2 y2 ( x0 ) ∃ 0

and

c1 y1(1) ( x0 ) # c2 y2(1) ( x0 ) ∃ 0.

Let y (x) ! c1 y1 (x) " c2 y2 (x). ... (8) (1) Then (8) shows that y (x) is a solution of (7) satisfying the conditions y (x0) ! 0, y (x0) ! 0. / y (x) ! 0 for all x (using Theorem I) , There exist constants c1 and c2 not both zero, such that c1 y1 (x) " c2 y2 (x) ! 0, for all x. , y1 and y2 are linearly dependent, by definition.

Created with Print2PDF. To remove this line, buy a license at: http://www.software602.com/

Differential Equations their Formation and Solutions

1.30

Conversely, let y1, y2 be linearly dependent. Then, there exist constants c1, c2 not both zero, such that c1 y1 (x) " c2 y2 (x) ! 0, for all x. ... (9A) c1 y1(1) ( x ) # c2 y2(1) ( x ) ∃ 0, for all x. Differentiating (9A) w.r.t. ‘x’, we get Eliminating c1, c2 from (9A) and (9B), we have

y1 ( x ) y1(1)

( x)

y2 ( x ) y2(1) ( x )

... (9B)

! 0, for all x.

, W (x) ! 0, for all x , Wronskian of y1, y2 vanishes identically. Corollary. Two solutions of (7) are linearly independent if their Wronskian does not vanish identically. Proof. Left as an exercise for the reader. Theorem V. The general solution of differential equation y(2) " P y(1) " Q y ! 0, ... (10) where P, Q are either constants or functions of x alone, can be put in the form c1 y1 (x) " c2 y2 (x), ... (11) where c1, c2 are constants and y1, y2 are any pair of linearly independent solutions of (10). Proof. Clearly c1 y1 " c2 y2 is a solution of (10). To prove the required result, it is sufficient to prove that every solution of (10) can be put in the form (11). To this end, we assume that y ! c1 y1 " c2 y2 , ... (12) where y is any solution of (1) and c1, c2 are constants. y (1) ∃ c1 y1(1) # c2 y2(1) . (12) , Solving (12) and (13) for c1, c2, we have

Λ

Μ

c1 ∃ (1 / W ) Ν yy2(1) . y (1) y2 ,

where

W ! Wronskian of y1 and y2 !

... (13)

Λ

Μ

c2 ∃ (1 / W ) Ν y (1) y1 . yy1(1) , ... (14) y1

y2

y1(1)

y2(1)

! { y1 y2(1) . y2 y1(1) } ; 0, for all x (! y1, y2 are linearly independent) For c1, c2 given by (14), y and c1 y1 " c2 y2 have the same value at a point x and the same result applies to their derivatives. The required result now follows by the existence theorem of solution of differential equation. Theorem VI. If y1 (x), y2 (x) be any two linearly independent solutions of the homogeneous differential equation y(2) " P y(1) " Q y ! 0 ... (15) and y0 is any particular solution of the non-homogeneous differential equation y(2) " P y(1) " Q y ! R, ... (16) the general solution of (16) is y0 " c1 y1 " c2 y2 , ... (17) where c1, c2 are arbitrary constants. Proof. Since y0 is a solution of (16), we have

y0(2) # P y0(1) # Q y0 ∃ R.

Let y be any arbitrary solution of (16). Then, we have y(2) " P y(1) " Q y ! R. Let u ! y – y0 . (20) ,

u(1) ∃ y(1) . y0(1)

Subtracting (18) from (19), we have

and

... (18) ... (19) ... (20)

u(2) ∃ y(2) . y0(2) .

... (21)

y (2) . y0(2) # P { y (1) . y0(1) } # Q ( y . y0 ) ∃ 0

Created with Print2PDF. To remove this line, buy a license at: http://www.software602.com/

Differential Equations their Formation and Solutions

1.31

or u(2) " P u(1) " Q u ! 0, using (20) and (21) showing that u is a solution of (15) and so we have u ! c1 y1 " c2 y2, (c1, c2 are some constants) or y – y0 ! c1 y1 " c2 y2, using (20) so that y ! y0 " c1 y1 " c2 y2 , showing that (17) is the general solution of (16). All theorems of the present article have natural generalization to equation of higher order. We close this article by giving extension of Theorem VI without proof. Theorem VII. If y1, y2, ..., yn be any n linearly independent solutions of the homogeneous linear differential equation of the nth order y(n) " P1 y(n–1) " ... " Pn y ! 0 ... (22) and y0 is any particular solution of the non-homogeneous differential equation y(n) " P1 y(n–1) " ... " Pn (y) ! Q, ... (23) the general solution of (23) is y0 " c1 y1 " c2 y2 " ... " cn yn, ... (24) where c’s are arbitrary constants. Remark. The general solution c1 y 1 " c2 y 2 " ... " cn y n of (22) is called complementary function (C.F.) and the particular solution y0 of (23) is called particular integral (P.I.). Example. Show that y ! C1e2x " C2xe2x is the general solution of y+ – 4y∗ " 4y ! 0 on any interval. [Nagpur 2002] Sol. Given y+ – 4y∗ " 4y ! 0 ... (1) Let y ! e2x so that y∗ ! 2e2x and y+ ! 4e2x Then, L.H.S. of (1) ! 4e2x – 8e2x " 4e2x ! 0 ... (2) 2x Again, let y ! xe , so that y∗ ! e2x " 2xe2x, y+ ! 2e2x " 2e2x " 4xe2x Then, L.H.S. of (1) ! 4e2x " 4xe2x – 4 (e2x " 2xe2x) " 4xe2x ! 0 ... (3) From (2) and (3), it follows that e2x and xe2x are solutions of (1). Again, Wronskian of e2x, xe2x !

e2 x

xe 2 x

2e 2 x

e 2 x # 2 xe 2 x

! e4x ; 0 for all x.

Hence by theorem V, Art. 1.18, it follows that y ! C1e2x " C2xe2x is the general solution of (1) in any interval. OBJECTIVE PROBLEMS ON CHAPTER 1 Ex. 1. The differential equation of the family of circles of radius ‘r’ whose centre lie on the x-axis, (a) y (dy/dx) " y2 ! r2 (b) y {(dy/dx) " 1} ! r2 2 2 (c) y {(dy/dx) " 1} ! r (d) y2 {(dy/dx)2 " 1} ! r2 [I.A.S. (Prel.) 1993] Sol. Ans. (d). Equation of a family of circles of radius r whose centre lie on the x-axis is given by (x – Ο)2 " y2 ! r2, where Ο is a parameter ... (1) Differentiating w.r.t. ‘x’, (1) gives 2 (x – Ο) " 2yy∗ ! 0, so that x – Ο ! – yy∗ ... (2) Then, (1) and (2) , y2y∗2 " y2 ! r2 or y2 {(dy/dx)2 " 1} ! r2. Ex. 2. The equation of the curve, for which the angle between the tangent and the radius vector is twice the vectorial angle is r2 ! A sin 24. This satisfies the differential equation (a) r (dr/d4) ! tan 24 (b) r (d4/dr) ! tan 24 (c) r (dr/d4) ! cos 24 (d) r (d4/dr) ! cos 24. [I.A.S. (Prel.) 1993] Sol. Ans. (b). Given that r2 ! A sin 24 ... (1) Differentiating w.r.t. ‘4’, (1) gives is

Created with Print2PDF. To remove this line, buy a license at: http://www.software602.com/

Differential Equations their Formation and Solutions

1.32

2r (dr/d4) ! 2A cos 24 or r (dr/d4) ! A cos 24 ... (2) Dividing (1) by (2), r (d4/dr) ! tan 24. Ex. 3. The maximum number of linearly independent solutions of the differential equation d4y/dx4 = 0 with the condition y (0) = 1 is (a) 4 (b) 3 (c) 2 (d) 1 [GATE 2010] Ans. (a) 2 3/2 2 2 Ex. 4. The order and degree of differential equation {1 + (dy/dx) } = k (d y/dx ) is (a) 3c1 (b) 3c2 (c) 3, 3 (d) 2, 2 [Garhwal 2010] Sol. Ans. (d). See Art. 1.4, 1.5 and 1.6 Ex. 5. Consider the following differential equations: 1/ 2

.2 / 3 6 Π ! d3y 5 6 d 2 Π> ! d 2 y > > > 1 # # Θ % 3& 7 7 ∃ 0 2 Θ% 2 & dx > ∋ dx ( >Ρ ∋ dx ( >8 >8 Ρ –3/2 2. dy/dx – 6x ! {ay " bx (dy/dx)} , b ; 0. The sum of the order of the first differential equation and degree of the second differential equation is (a) 6 (b) 7 (c) 8 (d) 9. [I.A.S. (Prel.) 2002] Sol. Ans. (d). Re-writing the last term of the first equation 6

! d2y 1. x % 2 & # y .2 / 3 ∋ dx ( 2

.2/3 6 .5/3 3 6 .5/3 3 6 Π Π Π d 2 >! d 2 y d >! 2 ! d 2 y d y> 2 d >! d 2 y d y> > . . ! ! % & % & % & Θ 7 Θ 7 Θ 7 % &% 2 & 2 % 2 & 3 2 % & dx > ∋ 3 ( ∋ dx ( 3 dx >∋ dx ( dx >∋ dx ( dx > dx 3 > Ρ 8> Ρ 8 Ρ 8 . 8/3 2 . 5/3 Π ! d3y ! d2y ! d 4 y 6> 2 > ! 5 ! d3y ! . Θ . % & %% 2 && %% 3 && # %% 2 && %% 4 && 7 , 3 > ∋ 3 ( ∋ dx ( ∋ dx ( ∋ dx ( ∋ dx ( >8 Ρ

which involves fourth order derivative d4y/dx4. So, by definition of order of a differential equation (see Art 1.4), the order of the first equation is 4. Next, re-writing the second differential equation, we get (dy/dx – 6x) {ay " bx (dy/dx)}3/2 ! 1 Squaring both sides, we get (dy/dx – 6x)2 {ay " bx (dy/dx)}3 ! 1 or {(dy/dx)2 – 12x (dy/dx) " 36x2} × {a3y3 " 3a2y2bx (dy/dx) " 3ayb2x2 (dy/dx)2 " b3x3 (dy/dx)3} ! 1 ... (1) 5 On multiplying the two factors on the L.H.S. of (1), we find that (dy/dx) occurs in the resulting equation. Hence by definition of degree of a differential equation (see Art. 1.5), the degree of the given second equation is 5. Hence, the sum of the order of the first equation and the degree of the second equation is (4 " 5), i.e., 9. Ex. 6. The degree of the equation (d3y/dx3)2/3 " (d3y/dx3)3/2 ! 0 is (a) 3 (b) 5 (c) 4 (d) 9. [I.A.S. (Prel.) 2004] Sol. Ans. (d). Re-writing the given equation, we have (d3y/dx3)2/3 ! – (d3y/dx3)3/2 or (d3y/dx3)4 ! (d3y/dx3)9 So, by the definition, the degree of the given equation is 9. Ex. 7. Linear combinations of solutions of an ordinary differential equation are solutions if the differential equation is (a) Linear non-homogeneous (b) Linear homogeneous (c) Non-linear homogeneous (d) Non-linear non-homogeneous [GATE 2002] Sol. Ans. (b). Refer theorem V of Art. 1.18.

Created with Print2PDF. To remove this line, buy a license at: http://www.software602.com/

Differential Equations their Formation and Solutions

1.33

Ex. 8. Which of the following pair of functions is not a linearly independent solutions of y+ " 9y ! 0? (a) sin 3x, sin 3x – cos 3x (b) sin 3x " cos 3x, 3 sin x – 4 sin3 x (c) sin 3x, sin 3x cos 3x (d) sin 3x " cos 3x, 4 cos3 x – 3 cos x. [GATE 2001] Sol. Ans. (c). Use theorem IV of Art. 1.18. Ex. 9. Let y ! = (x) and y ! ? (x) be solutions of y+ – 2xy∗ " (sin x 2) y ! 0, such that = (0) ! 1, =∗ (0) ! 1 and ? (0) ! 1, ?∗ (0) ! 2. The value of Wromhian W (=, ?) at x ! 0 is (a) 0 (b) 1 (c) e (d) e2 [GATE 2004] Sol. Ans. (b). We know that = ( x ) =∗ ( x ) and hence its value at x ! 0 is given by ? ( x ) ?∗ ( x )

W (=, ?) !

1 1 = (0) =∗ (0) , i.e., , i.e., 2 – 1, i.e., 1. 1 2 ? (0) ?∗ (0)

Ex. 10. What are the order and degree respectively of the differential equation d2 dx 2

.3 / 2

! d2y ∃ 0 (a) 1, 4 (b) 4, 1 (c) 4, 4 (d) 1, 1. % 2& ∋ dx ( Sol. Ans. (b) Re-writing the given differential equation, we have d dx

Π d ! d 2 y .3 / 2 6 > > Θ % 2& 7 !0 dx dx ( >Ρ ∋ >8 3 . 2

or .7/2

2

2

! d3y ! d2y % 3 & #% 2 & ∋ dx ( ∋ dx (

.5/ 2

d 4 y 6> 7 !0 dx 4 > 8

.5/2

2

! d 3y !d2y ! d3y d4y d2y d4y 5% 3 & ∃ 2 2 or % 3 & ∃% 2 & 4 dx dx dx 4 ∋ dx ( ∋ dx ( ∋ dx ( By definitions, its order is 4 and degree is 1. Ex. 11. What is the degree of the differential equation for a given curve in which (subtangent)m ! (subnormal)n in cartesian form, where 0 < n < m, m, n, m/n are integers? (a) m " n (b) m – n (c) mn (d) m/n. [I.A.S. (Prel.) 2006] Sol. Ans. (a) From calculus, we know that subtangent ! y/(dy/dx) ! y (dy/dx)–1, and subnormal ! y (dy/dx) Hence, the relation (subtangent)m ! (subnormal)n

or

5! d2y % & 2 ∋ dx 2 (

.5 / 2 3 6 3 d Π> ! d 2 y d y> . Θ% 2 & 7 !0 2 dx > ∋ dx ( dx 3 > Ρ 8

or

Π 5 ! d 2 y .7 / 2 > Θ. % 2 & 2 ∋ dx ( Ρ>

[I.A.S. (Prel.) 2006]

m

n m #n Π> ! dy .1 6> ! dy ! dy or y m. n ∃ % & , , Θy % & 7 ∃ % y & ∋ dx ( ∋ dx ( Ρ> ∋ dx ( 8> which is a differential equation of order m " n. Ex. 12. What are the order and degree respectivels of the differental equation of the family

of curves y2 ! 2c ( x # c ) , (a) 1, 1

(b) 1, 2

(c) 1, 3

[I.A.S. Prel. 2007]

y2 ! 2 c ( x # c )

Sol. Ans. (c) Given Differentiating (1) w.r.t. ‘x’

(d) 2, 1

2yy∗ ! 2c

so that

... (1) c ! yy∗

... (2)

Created with Print2PDF. To remove this line, buy a license at: http://www.software602.com/

Differential Equations their Formation and Solutions

1.34

Substituting the value of c given by (2) in (1), we have y 2 ∃ 2 yy ∗{x # ( yy ∗)1/ 2 }

y 2 . 2 xyy∗ ∃ 2 yy∗( yy∗)1/ 2 ...(3)

or

y 4 . 4 xy 3 y ∗ # 4 x 2 y 2 y ∗2 ∃ 4 y 3 y ∗3

Squaring both sies of (3), we get

which is a differential equation whose order is one and degree is three. Ex. 13. Which one of the following statement is correct ? The differential equation ( dx / dy )2 # 5 y1/ 3 ∃ x is (a) linear equation of order 2 and degree 1 (b) nonlinear equation of order

1 and degree 2 (c) non-linear equation of order 1 and degree 6 (d) linear equation of order 1 and degree 6. [I.A.S. (Prel.) 2008] Sol. Ans. (b). Refer Art. 1.4 and Art. 1.5

Miscellaneous problems on chapter 1 Ex.1. Show that sin 3x, cos 3x and sin 3x + cos 3x are solutions of differential equation [Delhi 2008]

y ∗∗ # 9 y ∃ 0 . Are these solutions linearly dependent ? Use the idea of Wronskian.

Hint. Proceed like solved Ex. 14, page 1.27. Ex. 2. Show that e 2x and e 3x are linearly independent solutions of the equation y ∗∗ . 5y + 6 = 0 on . 3 < x Σ 3. What is the general solution ? Find the solution y(x) that satisfies the conditions : y (0) = 2, y∗(0) = 3.

[Delhi B.A. (Prog) II 2010]

Hint: Proceed as in Ex. 4, page 1.23. General solution is y (x) ! c1e2x + c2 e3x. The solution satisfying the given initial conditions is y(x) ! 3e2x – e3x. Ex. 3. If y1 (x) ! 1 + x and y2 (x)! ex be two solution of y ∗∗ ( x)+ P( x ) y ∗ ( x ) + Q ( x) y ( x ) = 0 , then P (x) ! (a) 1 + x (b) –1 – x (c) (1 + x) /x (d ) (–1 – x)/ x [GATE 2009] Ex. 4. Consider the differential equation y ∗∗ ( x ) + P( x ) y ∗( x) + Q( x ) y ( x ) = 0 . The set of inital

conditions for which the above differential equation has no solution is (a) y (0) ! 2; y ∗(0) ∃ 1

(c) y(1) ! 0, y ∗(1) ∃ 1

(c) y (1) ! 1, y ∗(1) ∃ 0

(d) y (2) ! 1, y ∗(2) ∃ 2

[GATE 2009]

Ex. 5. If y1 (x) and y2 (x) are linearly independent solutions of the homogeneous differential equations y∗∗ + P (x) y∗ + Q (x) y = 0, then show that P (x) ! {(y1 y2∗∗ – y2 y1∗∗ )/W (y1, y2)} and Q(x) ! (y1∗ y2∗∗ – y2∗ y1∗∗ )/W (y1, y2). Hence construct the differential equation having two linearly independent solution e2x and xe2x. [Mumbai 2010] Ans. y∗∗ – 4 y∗ + 4 ! 0 Ex. 6. Let y1 and y2 be two solution of the differential equation y∗∗ + p(x) y∗ + Q (x) y ! 0. on [a, b]. If y1 and y2 have a maxima at x0 9 (a, b), then show that y2 is a constant multiple of y1 or y1 is a constant multiple of y2 on [a, b]. [Mumbai 2010] Ex. 7. Let y1 and y2 be two linearly independent solutions of the second order differential equation y∗∗ + p (x) y∗ + q (x) y ! 0. and W [y1, y2] be their Wronkian. Show that dW/dx = – p (x) W. Hence deduce that W ! k exp ( .Χ p( x )dx), where k in constant.[Delhi B.Sc. (Hons) II 2011]

Hint. Proceed as in theorem vii, page 1.19. Here note that, we have =1 ! y1, =2 ! y2, p1 (x) ! p (x), p2 (x) ! q (x), W (=1 , =2 ) = W ( y1 , y2 ) and c ! k. Thus, we get dW/dx = – p (x) W so that (1/W) dW = – p (x) dx. Integrating, log W – log K = –

Χ p( x) dx

Α

Β

W = K esp . Χ p ( x ) dx .

Created with Print2PDF. To remove this line, buy a license at: http://www.software602.com/

Differential Equations their Formation and Solutions

1.35

Ex. 8. Which one of the following equations has the same order and degree? (a) d4y/dx4 " 8 (dy/dx)4 " 5y ! ex (b) 5 (d3y/dx3)4 " 8 (dy/dx " 1)2 " 5y ! x3 (c) {1 " (dy/dx)3}2/3 ! 4 (d3y/dx3) (d) y ! x2 (dy/dx) " {(dy/dx)2 " 1}1/2.[I.A.S. (Prel.) 2005] Sol. Ans. (c). Refer Art. 1.4 and Art. 1.5. Ex. 9. Let y 1 and y 2 be any two solutions of a second order linear non-homogeneous ordinary differential equation and c be any arbitrary constant. Then, in general (a) y1 " y2 is its solution, but cy1 is not (b) cy1 is its solution, but y1 " y2 is not (c) both y1 " y2 and cy1 are its solutions (d) neither y1 " y2 nor cy1 is its solution. [I.A.S. (Prel.) 2005] Sol. Ans. (d). Refer Art. 1.18. Ex. 10. Consider the following statements regarding the differential equation | dy/dx | " | y | ! 0, 0 < x < 1 satisfying y (0) ! 1: 1. It is a linear differential equation 2. It has a unique solution. Which of the statements given above is/are correct? (a) 1 only (b) 2 only (c) both 1 and 2 (d) neither 1 nor 2. [I.A.S. (Prel.) 2005]

Created with Print2PDF. To remove this line, buy a license at: http://www.software602.com/

2 Equations of First Order And First Degree 2.1 Introduction There are two standard forms of differential equations of first order and first degree, namely, (i) dy/dx ! f (x, y) (ii) M (x, y) dx " N (x, y) dy ! 0. In what follows we shall see that an equation in one of these forms may readily be written in the other form. It will be assumed that the necessary conditions for the existence of solutions are satisfied. We now discuss various methods to solve such equations. 2.2 Separation of variables If in an equation, it is possible to get all the functions of x and dx to one side and all the functions of y and dy to the other, the variables are said to be separable. Working rule to solve an equation in which variables are separable. Step 1: Let dy/dx ! f1 (x) f2 (y), ... (1) be given equation. f1 (x) is a function of x alone and f2 (y) is a function of y alone. Step 2: From (1), separating variables, [1/f2 (y)] dy ! f1 (x) dx. ... (2) Step 3: Integrating both sides of (2), we have

# [1/ f2 ( y )] dy ! #

f1 ( x ) dx

c,

... (3)

where c is constant of integration, is the required solution. Note 1. In all solutions (3), an arbitrary constant c must be added in any one side only. If c is not added, then the solution obtained will not be a general solution of (1). Note 2. To simplify the solution (3), the constant of integration can be chosen in any suitable form so as to get the final solution in a form as simple as possible. Accordingly, we write log c, tan–1 c, sin c, ec, (1/2) × c, (– 1/3) × c etc. in place of c in some solutions. Note 3. The students are advised to remember by heart the following formulas. These will help them to write solution (3) in compact form (i) log x " log y ! log xy. (ii) log x – log y ! log (x/y). n (iii) n log x ! log x . (iv) tan–1 x " tan–1 y ! tan–1 [(x " y)/(1 – xy)] –1 –1 –1 (v) tan x – tan y ! tan [(x – y)/(1 " xy)] (vi) elog f (x) ! f (x). 2.3 Examples of Type 1 based on Art 2.2 Ex. 1. (a) Solve dy/dx ! ex–y " x2e–y [Agra 1995; Lucknow 1998; Mysore 2004;Punjab 1994; Meerut 2009; Agra 2005] (b) Solve dy/dx ! ex"y ! x2ey Sol. (a) For separating variables, we re-write the given equation as dy/dx ! e–y (ex " x2) or eydy ! (x2 " ex) dx. 2.1

Created with Print2PDF. To remove this line, buy a license at: http://www.software602.com/

Equations of First Order and First Degree

2.2

Integrating, ey ! x3/3 " ex " c, c being an arbitrary constant. (b) Do like part (a). Ans. – e–y ! x3/3 " ex " c. Ex. 2. Find the curves passing through (0, 1) and satisfying sin (dy/dx) ! c. [I.A.S. (Prel.) 2005] Sol. Re-writing the given equation, we have dy/dx ! sin–1 c or dy ! (sin–1 c) dx. Integrating, y ! x sin–1 c " c∃, c∃ being arbitrary constant. ... (1) Since, (1) must pass through (0, 1), we put x ! 0 and y ! 1 in (1) and obtain c∃ ! 1. Hence, (1) reduces to y ! x sin–1 c " 1 or (y – 1)/x ! sin–1 c or sin {(y – 1)/x} ! c, which gives the desired curves. Ex. 3. Solve (dy/dx) tan y ! sin (x " y) " sin (x – y). Sol. Using formula sin C " sin D ! 2 sin {(C " D)/2} cos {(C – D)/2}, the given equation can be rewritten as (tan y) (dy/dx) ! 2 sin x cos y or sec y tan y dy ! 2 sin x dx. Integrating, sec y ! – 2 cos x " c, c being an arbitrary constant. Ex. 4. Solve the following differential equations: dy sin x x cos x dy x (2 log x 1) ! ! (i) (ii) . [I.A.S. (Prel.) 2009] dx y (2 log y 1) dx sin y y cos y Sol. (i) Re-writing the given equation, (sin x " x cos x) dx ! (2y log y " y) dy. % cos x

Integrating, Now,

# x cos x dx

!

# x cos x dx ! 2 # y log y dy ( y / 2) c. x sin x % # sin x dx, integrating by parts 2

# x cos x dx

or

! x sin x " cos x.

... (1)

... (2)

Also, # y log y dy ! (log y ) & ( y 2 / 2) % # {(1/ y ) & ( y 2 / 2)} dy , integrating by parts

# y log y dy ! ( y

or

or

or

2

/ 2) & log y % y 2 / 4

... (3)

Using (2) and (3), (1) reduces to – cos x " x sin x " cos x ! 2 {(y2/2) & log y – y2/4 } " y2/2 + c. x sin x ! y2 log y " c, c being an arbitrary constant. (ii) Proceed exactly as in part (i). Ans. x2 log x ! y sin y " c. Ex. 5. Solve log (dy/dx) ! ax " by. Sol. Re-writing the given equation, we get dy/dx ! eax"by ! eaxeby or e–by dy ! eax dx. Integrating, – (1/b) e–by ! (1/a) eax " c, c being an arbitrary constant. Ex. 6. Solve y – x (dy/dx) ! a (y2 + dy/dx). [Meerut 1993; Delhi Maths (G) 1994; Purvanchal 2006, Rajasthan 1995; Agra 1993; Indore 1993] Sol. The given equation can be re-written as dy dx dy ! (a " x) ! y – ay or dx x a y (1 % ay ) dx x

∋ a ! ) a + 1 % ay

1( dy, on resolving into partial fractions. y ∗,

Created with Print2PDF. To remove this line, buy a license at: http://www.software602.com/

Equations of First Order and First Degree log (x " a) ! – log (1 – ay) " log y " log c,

Integrating, or or

2.3

∋ cy ( log (x " a) ! log ) ∗ +1 % ay ,

x"a!

or

cy 1 % ay

(x " a) (1 – ay) ! cy, which is the required solution. Ex. 7. Solve 3e tan y dx " (1 – ex) sec2 y dy ! 0. [Meerut 2008; Kanpur 1997] x

3e x

Sol. Separating the variables, we get

or

1 % ex

dx

sec 2 y dy ! 0. tan y

Integrating, – 3 log (1 – ex) " log (tan y) ! log c, c being an arbitrary constant. log (tan y) ! log (1 – ex)3 " log c or tan y ! c (1 – ex)3. Ex. 8. Solve

x2

(1

y2

x 2 y2 )

xy ( dy / dx ) ! 0.

Sol. Re-writing the given differential equation, we have x 2 ) (1

[(1

(1

or

y 2 )]

x 2 ) dx

y dy

x

# #

or

!0

2

(1

Integrating, Now,

xy ( dy / dx ) ! 0

(1

x 2 ) (1

y )

x (1

dx

#

2 1/ 2

x(1 x ) dx 2 1/ 2

x(1 x )

!

#

x dx 2 1/ 2

(1 x )

#

(% 1/ t 2 ) dt (1/ t ) 1 (1/ t )

dt

! %#

(t

2

1)

−/ 1 ! % log 2 8/ x

2

y dy (1 y 2 )1\ 2

t2

x )

(1

#

x dx 2 1/ 2

(1 x )

!

#

t dt , putting 1 " x2 ! t 2 t

!

1 2

#t

%1/ 2

dt ! t1/ 2 ! (1

1} (1 x 2 ) ./ 3 x 9/

... (2)

x 2 )1/ 2 .

... (3)

y dy ! (1 " y2)1/2. (1 y 2 )1/ 2 Using (2), (3) and (4), (1) gives the required solution as log x – log {1 + (1 + x2)1/2} + (1 + x2)1/2 + (1 + y2)1/2 ! C.

#

Similarly,

Ex. 9. Solve dy/dx ! e x

y

x 2e x

3

y

... (4)

. dy/dx ! e y ( e x

Sol. From given equation, we get or

! 0.

... (1)

! log x – log {1 + (1 + x2)1/2} Again,

y2 )

1 t

−/ 1 1 ./ 15 3 ! % log 2 7 9/ 8/

0 1 4 2 6x

y dy

2

! C.

, putting x !

! % log {t

xy (dy / dx ) ! 0

x 2 ) dx

(1

or

y2)

e–y dy ! (e x

3

x 2e x ) dx .

3

x 2 e x ). ... (1)

Created with Print2PDF. To remove this line, buy a license at: http://www.software602.com/

Equations of First Order and First Degree

2.4

#

Integrating (1),

– e–y ! e x

or

# e dx # (1/ 3) # e t dt c,

e % y dy !

x

3

x2 e x dx putting x3 ! t 3

– e–y ! ex " (1/3) et " c ! ex " (1/3) e x " c. Ex. 10. If dy/dx ! ex"y and it is given that for x ! 1, y ! 1; find y when x ! – 1. Sol. Rewriting the given equation, we get e–y dy ! ex dx. –y x Integrating it, – e ! e " c. ... (1) –1 –1 Putting x ! 1, y ! 1 in (1), –e !e"c so that c ! – e – e. Hence (1) becomes – e–y ! ex – e–1 – e. ... (2) –y –1 –1 Putting x ! – 1 in (2), we obtain –e !e –e –e so that y ! – 1.

or

Exercise 2(A) 1. (ex " 1) y dy ! (y " 1) ex dx. [Agra 1996] 2. (dy/dx) – y tan x ! – y sec2 x. 3.

y 2 ) dx

x (1

Ans. (ex " 1) (y " 1) ! c ey Ans. y cos x ! c e– tan x

y (1 x 2 ) dy ! 0. [Bangalore 1996]

Ans.

4. (2ax " x2) (dy/dx) ! a2 " 2ax. [Kanpur 1996] 5. dr ! a (r sin : d: – cos : dr). 6. (ey " 1) cos x dx " ey sin x dy ! 0. [Lucknow 1992]

(a

7. (a)

x) ( dy / dx )

x ! 0.

(1 x 2 )

(1

y2 ) ! C

Ans. x (x " 2a)3 ! Ce (2y/a) Ans. r (1 " a cos :) ! c Ans. (sin x) (ey " 1) ! c [Rohilkhand 1995; Bundelkhand 1998] Ans. y " (2/3) (x – 2a) (a " x)1/2 ! c

(b) dy/dx " 8. 9. 10. 11. 12.

(1 % y 2 ) /(1 % x 2 ) !0

[Pune 2010; Bangalore 1996]

Ans. sin–1 x " sin–1 y ! c

– dy " " dx ! 0. Ans. log (x/y) – (x " y)/(xy) ! c (xy2 " x) dx " (yx2 " y) d y ! 0. [Agra 2005, Rajsthan 2010] Ans. (x2 " 1) (y2 " 1) ! c sec2 x tan y dx " sec2 y tan x dy ! 0. [Agra 2006] Ans. tan x tan y ! c (1 " x) y dx " (1 " y) x dy ! 0. Ans. x " y " log (xy) ! c Find the function ‘f’ which satisfies the equation df/dx ! 2f, given that f (0) ! e3. Ans. f ! e2x"3 (x2

yx2)

(y2

xy2)

13. (1 – x2) (1 – y) dx ! xy (1 " y) dx. Ans. log [x (1 – y)2] ! 12 (x2 – y2) – 2y " c [Jabalpur 1993; Guwahati 1996; Vikram 1992; Nagpur 2005; Agra 1992; Meerut 1995] 14. x2 (y " 1) dx " y2 (x – 1) dy ! 0. Ans. x2 " y2 " 2 (x – y) " 2 log {(x – 1) (y " 1)} ! c 15. (dy/dx) tan y ! sin (x " y) " sin (x – y). Ans. 2 cos x " sec y ! c 16. x dy – y dx ! (a2 " y2)1/2 dx. Ans. 2a2 log (xc) ! y (a2 " y2)1/2 " a2 log {y " (a2 " y2)1/2} – y2

dy dy 1 0 ! 3 41 x 2 5. dx dx 7 6 18. cos y log (sec x " tan x) dx ! cos x log (sec y " tan y) dy. 17.

y%x

sec x Ans. log sec y

Ans. (y – 3) (1 " 3x) ! cx [Kanpur 1994]

tan x log {(sec x " tan x) (sec y " tan y)} ! c tan y

2.4 Transformation of some equations in the form in which variables are separable Equations of the form [Nagpur 2003] dy/dx ! f (ax " by " c) or dy/dx ! f (ax " by) can be reduced to an equation in which variables can be separated. For this purpose, we use the substitution ax " by " c ! v or ax " by ! v.

Created with Print2PDF. To remove this line, buy a license at: http://www.software602.com/

Equations of First Order and First Degree

2.5

2.5 Examples of Type 2 based on Art 2.4 Ex. 1. (a) Solve dy/dx ! (4x " y " 1)2. [I.A.S. (Prel.) 2006] 2 (b) dy/dx ! (4x " y " 1) if y (0) ! 1. [Delhi Maths (G) 2006] Sol. Let 4x " y " 1 ! v. ... (1) Differentiating (1) with respect to x, we get 4 + (dy/dx) ! dv/dx or dy/dx ! (dv/dx) – 4... (2) Using (1) and (2), the given equation becomes (dv/dx) –4 ! v2 or dv/dx ! 4 + v2 Now, separating variables x and v, dx ! (dv) / (4 + v2) –1 Integrating, x " c∃ ! (1/2) & tan (v/2), where c∃ is an arbitrary constant. or 2x " c ! tan–1 (v/2) or v ! 2 tan (2x " c), where c ! 2c∃ or 4x " y " 1 ! 2 tan (2x " c), using (1) ... (2) (b) Putting x ! 0, y ! 1 in (2), we get tan c ! 1, so that c ! ;/4. < Required solution is 4x " y " 1 ! 2 tan (2x " ;/4). 2 2 Ex. 2. Solve (x " y) (dy/dx) ! a . [Meerut 1997; Indore 1998; I.A.S. (Prel.) 1994; Delhi Maths (G) 1997; Ravishankar 1992] Sol. Let x " y ! v. ... (1) Differentiating, 1 " (dy/dx) ! dv/dx or dy/dx ! dv/dx– 1... (2) Using (1) and (2), the given equation becomes

0 dv 1 v 2 4 % 15 ! a 2 6 dx 7 or

or or

dx !

v2 2

v a Integrating,

2

dv

v2

or

∋ a2 ( dx ! )1 % 2 ∗ dv . v 2 ,∗ +) a x " c ! v – a2 × (1/a) & tan–1 (v/a), where c is arbitrary constant

0x y1 x " c ! x " y – a tan–1 4 5 6 a 7 Ex. 3. Solve dy/dx ! sec (x " y) cos (x " y) dy ! dx. Sol. Let x"y!v Using (1), the given equation becomes

0x y1 y – a tan–1 4 5 ! c. 6 a 7 [Delhi Maths (P) 2005] [Kanpur 1992] dy/dx ! (dv/dx) – 1.... (1)

or

so that

dv 1 !1" dx cos v

or 2

dx !

v2

or

dv – 1 ! sec v dx

or

dv ! a2 dx

2 cos 12 v % 1 cos v dv ! dv 1 cos v 1 2 cos2 12 v % 1

Integrating, x " c ! v – tan 12 v or Ex. 4. Solve dy/dx ! sin (x " y) " cos (x " y). Sol. Let x"y!v Differentiating (1) w.r.t ‘x’,

1"

dx ! (1 % 12 sec 2 12 v ) dv .

or

dy dv ! dx dx

y – tan 12 (x " y) ! c, by (1). [Guwahati 2007; Garhwal 1994] ... (1) or

dy dv ! – 1... (2) dx dx

Created with Print2PDF. To remove this line, buy a license at: http://www.software602.com/

Equations of First Order and First Degree

2.6

Using (1) and (2), the given equation becomes dv dv – 1 ! sin v " cos v or ! 1 " sin v " cos v.... (3) dx dx But 1 " sin v " cos v!1 " 2 sin (v/2) cos (v/2) " 2 cos2 (v/2) – 1! 2 cos2 (v/2) [1 " tan v/2].

< (3) reduces to

or

dv

dx !

!

1 2

sec 2 (v / 2) dv

. 1 tan (v / 2) 2 cos 2 (v / 2) [1 tan (v / 2)] Integrating, x " c ! log [1 " tan (v/2)], c being an arbitrary constant x " c ! log [1 + tan {(x " y)/2}], on using (1). Ex. 5. Solve (x " y) (dx – dy) ! dx " dy. [Calcutta 1995] Sol. Re-writing the given equation, we get (x " y – 1) dx ! (x " y " 1) dy

dy x ! dx x

or

y %1 . y 1

... (1)

Let x " y ! v. ... (2) (2) = 1+ dy/dx ! dv/dx so that dy/dx ! (dv/dx) – 1. ... (3) Using (2) and (3), (1) becomes v %1 dv dv 2v 11 0 %1 ! or ! or 2dx ! 41 5 dv . dx v 1 dx v 1 > 6 7 < Integrating, 2x " c ! v " log v or x – y " c ! log (x " y), by (2) Ex. 6. Solve dy/dx ! (4x + 6y + 5) / (3y + 2x + 4) [Delhi Maths (G) 2005; Calcutta 1995; Delhi Maths (H) 2002; Karnataka 1995 Rajasthan 2010] dy 2 (2 x 3 y ) 5 ! . dx (2 x 3 y ) 4

Sol. The given equation may be re-written as < We take Differentiating, (2) w.r.t. ‘x’

2x " 3y ! v. dy dv 2"3 ! dx dx

... (1) ... (2)

or

dy 1 0 dv 1 ! 4 % 25 . dx 3 6 dx 7

... (3)

Using (2) and (3), (1) gives 2v 5 1 0 dv 1 % 25 ! 4 3 6 dx v 4 7

or

dx v ! dv 8v

or

3 (2v 5) dv ! dx v 4

(1/ 8) & (8v 23) 4 % ? 23 / 8 ≅ ∋1 4 ! ! ) 23 8v 23 +8

2 !

8v v

23 4

( 9 ∗ 8 (8v 23) ,

∋1 ( 9 dx ! ) ∗ dv . + 8 8 (8v 23) , Integrating, x " c ! (v/8) " (9/64) log (8v " 23), c being an arbitrary constant or 8x " 8c ! 2x " 3y " (9/8) log (16x " 24y " 23), using (2) and multiplying by 8 or 3y – 6x " (9/8) log (16x " 24y " 23) ! 8c or y – 2x " (3/8) log (16x " 24y " 23) ! c∃, where c∃ (! 8c/3) is an arbitrary constant. Ex.7 Solve (x + 2y – 1) dx ! (x + 2y + 1) dy [Delhi Maths (H) (2007) Sol. Rewritting the given equation, dy/dx ! (x + 2y – 1) / (x + 2y + 1) ...(1) Let x + 2y ! v so that 1+ 2 (dy/dx) ! dv/dx or dy/dx ! (dv/dx–1)/2

Separating variables,

Created with Print2PDF. To remove this line, buy a license at: http://www.software602.com/

Equations of First Order and First Degree
From (i), we have

?

Hence, required second solution

/ (1/ x )dx / x

?x 2

y

4

c1 x 2 ! c2∃ / x

14

dx

1(1/ 3 x 3 )

1 x 2 ∆ (1/ 3x 3 ) 1 (1/ 3 x ) . y

Also, the general solution is i.e.,

...(i)

c1 x 2 ! c2 ∆ ( 11/ 3) ∆ (1/ x ) .

where

c2∃

1 (c2 / 3)

Hence the second linearly independent solution can be taken as 1/x. Ex. 3. Given that the equation x (1 – x) y'' + (3/2 – 2x) y' – y/4 " 0 has a particular integral of the form xn, prove that n " – (1/2) and that the primitive of the equation is y " x12 (A + B sin–1 x1/2) where A and B are arbitrary constants. [Gwaliar 2004, Pune 2001] x (1 1 x ) y ∃∃ ! (3/ 2 1 2 x ) y ∃ 1 y / 4 0

Sol. Given

Let f (x) " x n–1 ∃ From (2), f (x) " nx and Since f (x) is a particular integral of (1), we must have

...(1)

n

f # (x) " n(n–1)x

x (1 1 x ) f ∃∃( x ) ! (3/ 2 1 2 x ) f ∃ ( x ) 1 (1/ 4) ∆ f ( x ) or or

x (1 1 x )n( n 1 1) x n 1 2 ! ≅ 3/ 2 1 2 x Α nx n 11 1 (1/ 4 ) ∆ x n

n–2

...(2) ...(3)

0

0 using (2) and (3)

x n 61 n ≅ n 1 1Α 1 2 n 1 1 / 47 ! x n 11 6n ≅ n 1 1Α 1 ≅ 3 / 2 Α ∆ n7 0 ,

which must be an identity in x. Hence, we must have – 6n ≅ n 1 1Α ! 2 n ! 1/ 47 0 , and

n ≅ n 1 1Α 1 (3 / 2) ∆ n

i.e.,

≅ 2n ! 1Α2

0,

0, which is also satisfied by n

giving n "## $ (1/2) 1(1/ 2)

Created with Print2PDF. To remove this line, buy a license at: http://www.software602.com/

Linear Equations of Second Order

10.6

xn

f ( x)

Hence, we have

Comparing (1) with p( x ) y ∃∃ ! q( x) y ∃ ! r ( x) y

1/

> exp 8< 1 / 6q( x ) / p( x )7 dx 9= elog[ x > From (i), ?

/

...(i)

3/ 2 1 2 x 23 1 + dx 1 / 3 1 − dx , on resolving into partial fractions. x ≅1 1 x Α 2 x 2(1 1 x) . 4

1 ≅ 3 / 2 Α ∆ log x ! ≅1 / 2 Α ∆ log ≅1 1 x Α 3

x 11{ x(1 1 x )}11/ 2 (x

11/ 2 2

)

6

(11 x )] 11/ 2

{ x3 (1 1 x )}11/ 2

dx 2 /

> General solution of (1) is

y

x 11 / 2 ( A ! B sin 11 x1/ 2 ),

7

1 ≅1 / 2 Α ∆ log x3 (1 1 x)

?x 11/ 2

Hence, the second solution of (1)

y

0, here p (x) " x (1 – x) and q (x) " 3/2 – 2x

exp 8 1 / 6q( x ) / p( x )7 dx 9 < = dx, using formula (6) of Art.10.4.A / 2 6 f ( x)7

2 q( x ) + Now, / 31 − dx 4 p( x ) .

or

...(4)

x11/ 2 and hence the second solution of (1) is ? f ( x ) , i.e., ? x11/ 2 , where

Also, here f ( x) ?

x 11/ 2

1 1/ 2 1/ 2

2(1 1 x )

x

dx

log {x 3 (1 1 x )}11/ 2 11/ 2

x 11 6 x(1 1 x )7 2 sin 11 x1/ 2

2 x11/ 2 sin 11 x1/ 2 c1 f ( x) ! c2 ? f ( x) by taking

c1 x 11/ 2 ! 2c2 x 11/ 2 sin 11 x1/ 2 c1

A

and

c2

B.

10.5 Solved examples based on Art. 10.4 Ex. 1. Prove that y " sin x is a part of C.F. of the equation (sin x – x cos x)y# – x sin x y∃ ! y sin x " 0. [I.A.S. 2005; Bangalore 1994] Sol. Given (sin x – x cos x)y# – x sin x y∃ ! y sin x " 0. ... (1) Given that y " sin x so that y∃ " cos x and y# " – sin x. With these values of y, y∃ and y#Γ we have L.H.S. of (1) " (sin x 1 x cos x ) ( 1 sin x ) 1 x sin x cos x ! sin 2 x " 1 sin 2 x ! x sin x cos x 1 x sin x cos x ! sin 2 x 0, showing that y " sin x is a part of C.F. of (1). Ex. 2(a). Solve xy# – (2x – 1)y∃ ! (x – 1)y " 0. [Patna 2003; Delhi Maths (G) 2006; Bangalore 2002, 05. Osmania 2001, 04, 07 Kanpur 1994; Meerut 2010] or Solve by reducing the order. x y(2) – (2x–1) y(1) + (x – 11y = 0, given that ex is one integral part. where y(1) " dy/dx and y(2) " d2y/dx2. [Delhi B.Sc/ B.A. Maths (Prog) 2007] Sol. Putting the given equation in standard form, we get d2 y

1 & dy % 1 & % 1 ∋2 1 ( ! ∋ 1 1 ( 0. x ∗ dx ) x ∗ dx ) Comparing (1) with y# ! Py∃ ! Qy = R, we have P " –(2 – 1/x), Q " 1 – (1/x),

... (1)

2

R " 0.

... (2)

Created with Print2PDF. To remove this line, buy a license at: http://www.software602.com/

Linear Equations of Second Order

10.7

Here, 1 ! P ! Q " 1 – 2 ! (1/x) ! 1 – (1/x) " 0, showing that u " ex is a part of C.F. of the solution of (1). Let the complete solution (1) be y = uv 2 du & dv % !∋P ! ( u dx ∗ dx dx )

% 1 2 de x & dv ! 1 2 ! ! ∋ ( x e x dx (∗ dx dx 2 ∋)

d 2v

1 % & dv ! ∋ 12 ! ! 2 ( 2 x dx ) ∗ dx dv/dx " q

Let

Then (5) becomes Integrating, or

R u

2

d 2v

or

... (4)

d 2v

Then v is given by or

... (3)

0,

using (2) and (3)

0

d 2v 2

so that

dq q ! dx x

log q " log c1 – log x

1

dx . x

q " c1/x

or

dv 9 dv c1 c dx 8 or q dv 1 : dx ;= dx x x < Integrating, v " c1 log x ! c2, c1, c2 being arbitrary constants ... (7) From (3), (4) and (7), the required complete solution is y " e xv or y " c1 ex log x ! c2 ex. Ex. 2(b). Solve (3 – x)y# – (9 – 4x)y∃ ! (6 – 3x)y " 0. [Delhi Maths (G) 1998; Allahabad 2003; Garhwal 1997; Kurukshetra 2000, 05]] Sol. Re-writing the given equation in standard form, we get d2 y

9 1 4 x dy 6 1 3 x 1 ! 0. 3 1 x dx 3 1 x dx 2 Comparing (1) with y# ! Py∃ ! Qy " R, we have P " –(9 – 4x) / (3 – x), Q " (6 – 3x) / (3 – x),

9 1 4 x 6 1 3x ! 31 x 31 x showing that u " ex is a part of C.F. of the solution of (1). Let the complete solution of (1) be

1 ! P ! Q " 11

Here

% 9 1 4 x 2 de x & dv ! ! ∋1 ( dx 2 ∋) 3 1 x e x dx (∗ dx

d 2v

or d 2v 2

dx Let

!

2(3 1 x ) 1 (9 1 4 x ) dv 31 x dx dv/dx " q

... (1) R " 0.... (2)

3 1 x 1 (9 1 4 x ) ! 6 1 3 x 31 x

0

y " uv

0, or

so that

0,

... (3)

d 2v % 2 du & dv P! ( 2 ∋ u dx ∗ dx dx )

Then v is given by

or

1 dv x dx

dq q

or

0

!

0. ...(5) dx d2v/dx2 " dq/dx... (6)

or

... (4) R u

using (2) and (3) d 2v

2 x 1 3 dv 0 ... (5) 3 1 x dx dx d2v / dx2 " dq/dx. ... (6) 2

!

Created with Print2PDF. To remove this line, buy a license at: http://www.software602.com/

Linear Equations of Second Order

10.8

dq 2 x 1 3 ! q 0 dx 3 1 x

Then (5) becomes dq 2 x 1 3 dx q x13 Integrating,

or

or or or

dq dx

or

dq q

or

1

2x 1 3 q 31 x

3 & % ∋2 ! ( dx. x 13∗ )

log q " 2x ! 3 log (x – 3) ! log c1

log q – log(x – 3) – log c1 " 2x or 2x 3 q " c1e (x–3) or dv " c1e2x (x – 3)3 dx.

q/[c1(x – 3)3] " e2x dv/dx " c1e2x (x – 3)3, by (6)

3

Integrating, v c1 / ( x 1 3)3 e 2 x dx ! c2 , c1, c2 being arbitrary constants. or

8 e2 x e2 x 9 v c2 ! c1 :( x 1 3)3 1 / 3( x 1 3)2 0 dx ; , integrating by parts 2 2 < = c2 ! c2 !

c1 3 8 e2 x e2 x 9 ( x 1 3)3 e 2 x 1 c1 :( x 1 3)2 0 1 / 2( x 1 3) 0 dx ; , integrating by parts again 2 2 < 2 2 = c1 3 3 8 e2 x e2 x 9 ( x 1 3)3 e3 x 1 c1 ( x 1 3)2 e 2 x ! c1 :( x 1 3) 0 1 / 10 dx ; 2 4 2 < 2 2 = (Integrating by parts again)

" c2 ! (1/ 2) ∆ c1 ( x 1 3) 3 e 2 x 1 (3 / 4) ∆ c1 ( x 1 3) 2 e 2 x ! (3 / 4) ∆ c1 ( x 1 3)e 2 x 1 (3 / 8) ∆ c1e 2 x " c2 ! (1/ 8) ∆ c1e 2 x [4( x 1 3) 3 1 6( x 1 3) 2 ! 6( x 1 3) 1 3] or

v

c2 ! (1/ 8) ∆ c1e2 x (4 x3 1 42 x 2 ! 150 x 1 183).

... (7)

From (3), (4) and (7), the required complete solution is y " ex v or y " c2ex ! (1/8) × c1e3x (4x3 – 42x2 ! 150x – 183). Ex. 3. Solve (x ! 2)y# – (4x ! 9)y∃ ! (3x ! 7)y " 0. [Delhi Maths (G) 1994] Hint. Do as in Ex. 2(b). Ans. y " c1(2x ! 3)e3x ! c2ex Ex.4(a) Find general solution of (1 – x2)y# – 2xy∃ ! 2y " 0, if y " x is a solution of it. (b) If y " x is a solution of x2y# ! xy∃ – y " 0, find the solution. [Mumbai 2010] Sol. (a) Re-writing the given equation in standard form, we get d2 y dx

2

1

2x

dy 2 ! y 1 1 x dx 1 1 x 2 2

Comparing (1) with y# ! Py∃ ! Qy " R, we get P " – (2x)/(1 – x2), Q " 2/(1 – x2), Here u"x is given to be part a of C.F. of the solution of (1). Let the complete solution of (1) be 2

Then v is given by

d y dx 2

0.

... (1)

R " 0.

y " uv.

2 du & dv % !∋P ! ( u dx ∗ dx )

... (2) ... (3) ... (4)

R u

Created with Print2PDF. To remove this line, buy a license at: http://www.software602.com/

Linear Equations of Second Order

10.9

d2 y

2 dx & dv % 2x ! ∋1 ! ( 2 x dx ∗ dx dx ) 11 x

or

2

dv/dx " q

Then (5) becomes Integrating, or

qx 2 (1 1 x 2 ) c1

or

dv

using (2) and (3)

d 2v % 2 2 x & dv !∋ 1 0. ( 2 dx ) x 1 1 x 2 ∗ dx so that

or Let

0,

... (5) d2v/dx2 " dq/dx.... (6)

dq % 2 2x & dq % 2 2x & !∋ 1 or !∋ 1 (q 0 ( dx 0 dx ) x 1 1 x 2 ∗ q ) x 1 1 x2 ∗ log q ! 2 log x ! log (1 – x2) " log c1

dv / dx c1 / x 2 (1 1 x2 ) by (6)

or c1

1 dx x (1 1 x 2 ) 2

1 & % 1 c1 ∋ 2 ! ( dx , on resolving into partial fractions 1 1 x2 ∗ )x

8 x 11 1 1! x 9 c1 : ! log ... (7) ; ! c2 , c1, c2 beng arbitrary constant 11 x = < 11 2 From (3), (4) and (7), the required general solution is x 1! x & % y uv xv y c2 x ! c1 ∋ 11 ! log or (. 2 11 x ∗ ) (b) Hint : Proceed as in part (a). Ans. y " x ! x–1. 2 Ex. 5. Solve x y# ! xy∃ – y " 0, given that x ! (1/x) is one integral by using the method of reduction of order. [Delhi Maths (G) 2001] Sol. Re-writing the given equation in standard form, we get y# ! (1/x)y∃ – (1/x2)y " 0; ... (1) 2 Comparing (1) with y# ! Py∃ ! Q y " R, we get P " 1/x, Q " – (1/x ). R " 0 ... (2) Here given that u " x ! 1/x ... (3) is part of C.F. of the solution of (1). Let the complete solution of (1) be y " uv. ... (4) v

Integrating,

d 2v

2 du & dv % !∋P ! ( u dx ∗ dx dx )

Then v is given by d 2v

81 2 d % 1 &9 dv !: ! ∋ x ! (; x x ! (1 / x ) dx x ∗= dx dx )
(5) Β Integrating,

0

... (2) ... (3) ... (4)

R u

2

d 2v

or

R

u " x,

showing that

2

Then v is given by

... (1)

0.

... (5) d2v / dx2 " dq/dx. ... (6)

dq % x cos x 2& dq or ! ∋1 ! (q 0 q dx ) x sin x ! cos x x ∗ log q " log (x sin x ! cos x) – 2 log x

% x cos x 2& 1 ( dx. ∋ ) x sin x ! cos x x ∗ ! log C1

Created with Print2PDF. To remove this line, buy a license at: http://www.software602.com/

Linear Equations of Second Order q " C1 x sin x ! cos x x2

or > or

or

10.11

% sin x cos x & C1 ∋ ! 2 ( , by (6) ) x x ∗

dv dx

or

1 1 C1 / sin x dx ! C1 / 2 cos x dx x x 1 81 9 % 1 & v " C1 : (1 cos x ) 1 / ∋ 1 2 ( ( 1 cos x )dx ; ! C1 / 2 cos x dx ! C2 ) x ∗ x The required solution is y " uv " (sin x) [C1(x / sin x) ! C2]. or y " C1x ! C2 sin x, where C1 and C2 are arbitrary constants. (iv) Rewriting the given equation in standard form, we have y# ! 0 0 y∃ – 2 cosec2x 0 y " 0. Comparing (1) with y# ! Py∃ ! Q y " R, P " 0, Q " – 2 cosec2 x, Given that u " cot x, is a part of C.F. of (1). Let the general solution of (1) be y " uv.

... (1) R " 0.... (2) ... (3) ... (4)

Created with Print2PDF. To remove this line, buy a license at: http://www.software602.com/

Linear Equations of Second Order

10.12

d 2v

2 du & dv % !∋P ! ( u dx ∗ dx dx )

Then v is given by or

R u

2

d 2v

2 d cot x & dv % ! ∋0 ! ( 2 cot x dx ∗ dx dx ) Let dv/dx " q

d 2v

or

0

dx 2

2cosec 2 x dv 0. ... (5) cot x dx d2v / dx2 " dq/dx. ... (6)

1

so that

dq 4 dq 1 q 0 or 4 cosec 2 x dx. dx 2sin x cos x dx Integrating, log q " 4 ∆ (1/2) ∆ log tan x ! log C1 or q " C1 tan2 x dv/dx " C1 tan2 x or dv " C1 (sec2 x – 1) dx. Integrating, v " C1 (tan x – x) ! C2, C1, C2 being arbitrary constants. ... (7) From (3), (4) and (7), the required general solution is given by y " uv or y " cot x [C1(tan x – x) ! C2] " C1 (1 – x cot x) ! C2 cot x. (v) Rewriting the given equation in standard form, we have

Then (5) becomes or

d2 y

dy a2 1 y 0. dx 2 1 1 x 2 dx 1 1 x 2 y# ! Py∃ ! Q y " R, we have Q " – a2 / (1 – x2),

Comparing (1) with P " – x / (1 – x2),

1

x

11

Since ea sin x is a solution of (1), therefore is a part of C.F. of (1) Let the general solution of (1) be d 2v

Then v is given by

dx 2

2 du & dv % !∋P ! ( u dx ∗ dx )

11 % x 2 de a sin x ∋ ! 1 ! 11 dx dx 2 ∋) 1 1 x 2 e a sin x dv/dx " q so that

Let

8 x 2a 9 : ;q 1 :

or

or

... (2) ... (3) ... (4)

d 2v

or

dx

2

12

dv 1. ... (5) dx

so that d v / dx2 " dq/dx. ... (6) (dq/dx) – 2q " 1, which is linear in q and x. 2

( 12) dx Its integrating factor " I.F. " e /

q e 12 x

R " x.

R u

2

d 2v

... (1)

/ 10 e

12 x

e 12 x and solution is dx ! C1

1 (1 / 2) ∆ e 12 x ! C1

q 1 (1/ 2) ! C1e2 x

or dv / dx 1 (1/ 2) ! C1e2 x dv " [– (1/2) ! C1e2x] dx. Integrating, v " – (x/2) ! (C1/2) e2x ! C2. ... (7) 2x From (3), (4) and (7), the required general solution is y " uv " x [–(x/2) ! (C1/2) e ! C2] y " C1∃ xe2x ! C2x – (x2/2), where C1∃ " C1/2; C1' and C2 being arbitrary constants. Ex. 10. Solve x2 y# – (x2 ! 2x)y∃ ! (x ! 2)y " x3 ex. [Himanchal 2008; Delhi Maths (G) 1997; Meerut 2005, 09] Sol. Dividing by x2, the given equation in standard form is d2 y

% 2 & dy % 1 2 & 1 ∋1 ! ( ! ∋ ! 2 ( y x ∗ dx ) x x ∗ dx ) Comparing (1) with y# ! Py∃ ! Q y " R, we have P " – (1 ! 2/x), Q " 1/x ! 2/x2, Here P ! Qx " 0, showing that is a part of C.F. of (1). 2

x ex.

R " x e x. u"x

... (1) ... (2) ... (3)

Created with Print2PDF. To remove this line, buy a license at: http://www.software602.com/

Linear Equations of Second Order

10.14

y " uv.

Let the general solution of (1) be 2

2 du & dv % !∋P ! ( u dx ∗ dx dx )

d v

Then v is given by

... (4)

R u

2

2 2 dx & dv xe x d 2 v dv % ! ∋ 11 1 ! 1 e x . ... (5) or ( 2 x x dx ∗ dx x dx dx ) dx Let dv/dx " q so that d2v / dx2 " dq/dx.... (6) Then (5) becomes (dq/dx) –q " ex, which is linear in q and x.

or

d 2v 2

( 11)dx Its integraging factor I.F. " e /

qe 1 x

/ (e

x

x ! C1

dv/dx " (x + C1)ex

or

Integrating, or

0 e 1 x ) dx ! C1

e 1 x and so solution is or

q ( x ! C1 ) e x

or

dv " (x + C1)exdx

? " (x + C1)ex – / (1 0 e x ) dx ! C2 , C1, C2 being arbitrary constants.

v " (x ! C1) ex – ex ! C2 " (x ! C1 – 1) ex ! C2. ... (7) From (3), (4) and (7), the required general solution is y " uv " x [(x ! C1 – 1) ex ! C2] or y " C1xex ! C2ex ! (x – 1) xex. 2 2 Ex. 11.(a) Solve (x ! 1) (d y / dx ) – 2(x ! 3) (dy/dx) ! (x ! 5)y " ex. [Garhwal 1993] Sol. Dividing by (x ! 1), the given equation in standard form is d2 y

1

2( x ! 3) dy x ! 5 ! y x ! 1 dx x ! 1

ex . x !1

... (1)

dx 2 Comparing (1) with y# ! Py∃ ! Q y " R, we get P " – 2(x ! 3)/(x ! 1), Q " (x ! 5)/(x ! 1),

Here

1 ! P ! Q " 11

2 x ! 6 x ! 5 x ! 1 1 (2 x ! 6) ! x ! 5 ! 0, x ! 1 x !1 x !1 u " ex

showing that is a part of C.F. of (1). Let the general solution of (1) be

y " uv. d 2v

2 du & dv % !∋P ! ( u dx ∗ dx dx )

Then v is given by

or

d 2v

2 x ! 6 & dv 1 % ! ∋21 ( 2 x ! 1 ∗ dx x ! 1 dx ) Let dv/dx " q

1 [4/( x !1)]dx Its integrating factor I.F. " e /

q( x ! 1)14

e x ( x ! 1) d 2v

4 dv 1 . ... (5) x ! 1 dx x !1 dx d2v / dx2 " dq/dx. ... (6) 2

so that

dq 4 1 q dx x ! 1 1

/ x ! 1 0 ( x ! 1)

1

1 , which is linear in q and x. x !1

e 14log ( x !1) 14

... (4)

ex

or

Then (5) becomes

... (3)

R u

2

d 2 v % 2 x ! 6 2 de x & dv !∋1 ! ( dx 2 ∋) x ! 1 e x dx (∗ dx

or

R " ex/(x ! 1).... (2)

dx ! C1

( x ! 1)14 . and solution is

/ ( x ! 1)

15

dx ! C1

Created with Print2PDF. To remove this line, buy a license at: http://www.software602.com/

Linear Equations of Second Order

10.15

dv/dx " –(1/4) ! C1 (x ! 1)4 or dv " [–(1/4) ! C1(x ! 1)4]dx. 5 Integrating, v " –(1/4)x ! (C1/5) (x ! 1) ! C2. ... (7) From (3), (4) and (7), the required general solution is y " uv " ex[–(1/4)x ! (C1/5) (x ! 1)5 ! C2] or y " C1∃ ex (x ! 1)5 ! C2 ex – (1/4)x ex, where C1∃ " C1/5. Ex. 11(b) Solve xy# – 2(x ! 1)y∃ ! (x ! 2)y " (x – 2)ex. [Bangalore 2001, 04] 3 x Sol. Do as in Ex. 11(a). Ans. y " (1/3) ∆ C1x e ! C2ex ! (x – x2/2)ex. Ex. 11(c) Solve d2y / dx2 – cot x (dy/dx) – (1 – cot x)y " ex sin x. [Delhi Maths (G) 2005; Meerut 1996; S.V. University (A.P.) 1997, Kanpur 2006] Sol. Comparing the given equation with y# ! Py∃ ! Q y " R, we get P " –cot x, Q " –1 ! cot x, R " ex sin x. ... (1) x Here 1 ! P ! Q " 0, showing that u"e ... (2) is a part of C.F. of the given equation. Let the required general solution be y " uv ... (3) or

d 2v

2 du & dv % !∋P ! ( u dx ∗ dx dx )

Then v is given by or

% 2 de x ! 1 cot x ! ∋ dx 2 ∋) e x dx Let dv/dx = q Hence we get d 2v

Its I.F. " e / q

or or

2

e2 x " sin x

(2 1 cot x ) dx

& dv (( ∗ dx

e x sin x e

or

x

R u d 2v dx

2

! (2 1 cot x )

so that d2v/dx2 " dq/dx dq/dx + (2 – cot x)q " sin x, which linear in q and x

e2 x 1 log sin x

% e2 x & sin x 0 ∋ ( dx ! c1 / ∋) sin x (∗

e 2 x 0 e log (sin x )

11

dv dx

sin x

... (4)

e 2 x (sin x ) 11 and solution is

1 2x e ! c1 , c1 being an arbitrary constant 2

q " (1/2) ∆ sin x ! c1e–2x sin x or dv/dx " (1/2) ∆ sin x ! c1e–2x sin x. –2x dv " [(1/2) ∆ sin x ! c1e sin x] dx. Integrating,

v " ( 11/ 2) ∆ cos x ! c1 / e 12 x sin x dx ! c2 , c2 being an arbitrary constant

cos x c cos x c1 ! 2 1 2 (12 sin x 1 cos x ) ! c2 or ? " 1 1 (2 sin x ! cos x ) ! c2 . ...(5) 2 2 5 1 ! (12)

or

? "1

or

8 9 e ax ax e sin bx dx (a sin bx 1 b cos bx ); : / 2 2 a !b < = Hence form (2), (3) and (5), the required general solution is y " uv " ex [–(1/2) ∆ cos x – (c1/5) ∆ e–2x (2 sin x ! cos x) ! c2] y " c1∃ e–2x (2 sin x ! cos x) ! c2ex – (1/2) ∆ ex cos x, where c1∃ "## – (c1/5). Ex. 11(d) Solve y# ! (1 – cot x)y∃ – y cot x " sin2x. [Meerut 1999,Delhi Maths (G) 2000; Rohilkhand 1997] Hint. Do as in Ex.11(c). General solution is given by y " c1 (sin x – cos x) ! c2e–x – (1/10) ∆ (sin 2x – 2 cos 2x).

Created with Print2PDF. To remove this line, buy a license at: http://www.software602.com/

Linear Equations of Second Order

10.16

Ex. 11(e) For the equation y2 ! (1 – cot x)y1 – y cot x " sin2x, find an integral of C.F. [Bangalore 1995] Sol. Given y2 ! (1 – cot x)y1 – y cot x " sin2 x. ... (1) Comparing (1) with y2 + Py1 ! Q y " R, here P " 1 – cot x, Q " – cot x. Hence 1 – P ! Q " 1 – (1 – cot x) – cot x " 0 and so e–x is an integral of C.F. Ex. 12. Solve x(d2y / dx2) – (dy/dx) ! (1 – x)y " x2e–x. [Delhi Maths (G) 1996] Sol. Re-writing, (d2y/dx2) – (1/x)(dy/dx) ! (1/x – 1)y " xe–x. ... (1) –x Comparing (1) with y# ! Py∃ ! Q y " R, here P " –1/x, Q " (1/x) –1, R " xe ... (2) Hence 1 ! P ! Q " 0, showing that u " ex ... (3) is a part of C.F. of the given equation. Let the required general solution be y " uv. ... (4) d 2v

2 du & dv % !∋P ! ( u dx ∗ dx dx )

Then v is given by or

2

% 1 2 de x & dv xe 1 x ! ∋1 ! ( dx 2 ∋) x e x dx (∗ dx ex Let dv/dx " q d 2v

(2 11/ x ) dx Its I.F. " e /

q " xe

or > >

e2 x 1log x

q 0 e 2 x x 11

> –2x

d 2v

or

dx 2

so that

dq % 1& ! ∋21 ( q dx ) x∗

Then (5) becomes

e 2 x 0 e 1 log x

/ ( xe

R u

12 x

(x ! c1)

xe 12 x , which is linear in q and x.

e 2 x 0 x 11 and solution is

) 0 ( e2 x x 11 ) dx ! c1

2

x ! c1 dv/dx " (x2 ! c1x)e–2x,

or

/ dv / ( x

1 & dv % ! ∋21 ( xe12 x . ... (5) x ∗ dx ) d2v/dx2 " dq/dx.... (6)

by (6)

! c1 x )e 12 x dx ! c2

v c2 ! ( x 2 ! c1 x ) ∆ (11/ 2) ∆ e 12 x 1 / {(2 x ! c1 ) ∆ ( 11/ 2) ∆ e12 x }dx , integrating by parts " c2 1 (1/ 2) ∆ ( x 2 ! c1 x) e 12 x ! (1/ 2) ∆ / (2 x ! c1 )e 12 x dx % e 12 x 1 18 " c2 1 ( x 2 ! c1 x)e12 x ! :(2 x ! c1 ) ∋∋ 2 2 :< ) 12

& % e 12 x (( 1 / (2) ∋∋ ∗ ) 12

& 9 ((dx ; ∗ ;=

" c2 1 (1/ 2) ∆ ( x 2 ! c1 x)e12 x 1 (1/ 4) ∆ (2 x ! c1 )e12 x 1 (1/ 4) ∆ e12 x . " c2 1 (1/ 4) ∆ e12 x (2 x2 ! 2c1 x ! 2 x ! c1 ! 1). or

or

12 x 2 12 x v " c2 1 (1/ 4) ∆ e (2 x ! 2 x ! 1) 1 (1/ 4) ∆ e c1 (2 x ! 1). ... (7) From (3), (4) and (7), the required general solution is y " uv " ex[c2–(1/4) ∆ e–2x (2x2 ! 2x ! 1) – (1/4) ∆ e–2x c1(2x ! 1)] y " c1∃ (2x ! 1) e–x ! c2ex –(1/4) ∆ (2x2 ! 2x ! 1) e–x, where c1∃ " –(1/4) ∆ c1. Ex. 13. Solve the following differential equations : (a) (d2y / dx2) – (1 ! x) (dy/dx) ! xy " x. [Delhi Maths (G) 1993, 94, Gujrat 2003, 05; Kurukshetra 2004] (b) x(d2y / dx2) – (dy/dx) ! (1 – x)y " xe–x. [Delhi Maths (G) 1995] 2 2 2 (c) (d y / dx ) – x (dy/dx) ! xy " x. [Delhi Maths (G) 1994, 99]

Created with Print2PDF. To remove this line, buy a license at: http://www.software602.com/

Linear Equations of Second Order

10.17

(d) x(d2y / dx2) ! (1 – x) (dy/dx) – y " ex. [Lucknow 2006; Srivenkateshwara 2003] (e) x(d2y / dx2) – (x – 2) (dy/dx) –2y " x3. [Luknow 2002, Delhi Maths (G) 1996;Kanpur 2006] (f) (2x – 1) (d2y / dx2) – 2(dy/dx) ! (3 – 2x)y " 2ex. (g) (d2y / dx2 ) ! [1 ! (2/x) cot x – (2/x2 )]y " x cos x, given that (sin x)/x is an integral included in C.F. [Bhopal 2002, 05, Indore 2001, 02, Purvanchal 2000, 04, 07] (h) x2y2 ! xy1 – y " x2ex. [Rohilkhand 1994] Sol. (a) Given y# – (1 ! x)y∃ ! xy " x. ... (1) Comparing (1) with y# ! Py∃ ! Q y " R, here P " – 1 – x, Q " x, R " x. ... (2) x Here 1 ! P ! Q " 0, showing that u"e, ... (3) is a part of C.F. of (1). Let the required general solution of (1) be y " uv. ... (4) d 2v

2 du & dv % !∋P ! ( u dx ∗ dx dx )

Then v is given by or

% 2 de x ! 1 1 1 x ! ∋ dx 2 ∋) e x dx Let dv/dx " q Then (5) becomes d 2v

(11 x ) dx Its I.F. " e /

qe x 1 x

Putting >

2

/2

ex1x

–x /2 " t qe x 1 x

2

/2

2

/2

/ ( xe

"

2

1x

& dv (( dx ∗

"

d 2v

or

ex

dx 2

! ≅1 1 x Α

dv dx

xe 1 x .

... (5)

d2v / dx2 " dq/dx. ... (6)

and so its solution is 0 ex1x

2

/2

) dx ! c1

/ xe

1 ( x 2 / 2)

dx ! c1 .

–xdx " dt, we have

/ e (1dt ) ! c1 t

dv/dx " c1 e 1 x ! x Integrating,

x

R u

so that (dq/dx) ! (1 – x)q " xe–x.

so that

2

/ 2)

2

/2

q " e 1( x 1 x

> or

2

1 et ! c1

1 e1 ( x

2

/ 2)

! c1 .

8 c 1 e 1 x 2 / 2 9 c e 1 x ! x 2 / 2 1 e1 x < 1 = 1

1 e1 x

v " c1 / e 1 x ! x

or 2

/2

dv [ c1e1 x ! x

2

/2

1 e1 x ] dx.

dx ! e 1 x ! c2 , c1, c2 being arbitrary constants ... (7)

From (3), (4) and (7), the required general solution is given by 2 2 y " uv " e x 8c1 / e 1 x ! x / 2 dx ! e 1 x ! c2 9 c1e x / e 1 x 1 x / 2 dx ! c2 e x ! 1. < = Remark: Some solutions are left without evaluating the integral which are not integrable by well known standard methods.

(b) Do as in part (a).

Ans. y " c1 (2x ! 1) e–x ! c2ex ! e x / x log x e 12 x dx.

(c) Given y# – x2 y∃ ! xy " x Comparing (1) with y# ! Py∃ ! Q y " R, P " –x2, Q " x, Here P + Qx " 0, showing that a part of C.F. of (1) is given by Let the required general solution of (1) be y " uv.

... (1) R " x. ... (2) u " x. ... (3) ... (4)

Created with Print2PDF. To remove this line, buy a license at: http://www.software602.com/

Linear Equations of Second Order

10.18

d 2v

2 du & dv % !∋P ! ( u dx ∗ dx dx )

Then v is given by d 2v

or Let

2 dx & dv x % ! ∋ 1 x2 ! ( 2 x dx ∗ dx x dx ) dv/dx " q

>

qx 2 e 1 x

3

/3

d 2v

%2 & dv ! ∋ 1 x2 ( 1. ... (5) dx )x ∗ dx d2v / dx2 " dq/dx. ... (6)

or

2

so that

dq % 2 & ! ∋ 1 x 2 ( q 1 , which is linear in q and x. dx ) x ∗

Then (5) becomes [(2 / x ) 1 x Its I.F. " e /

R u

2

2

]dx

"

" e 2 log x 1( x

/ (1 0 x

3

3

2 1x /3

e

2

e log x 0 e1 x

/ 3)

3

x 2 e1 x

/3

c1 1 / et dt

) dx ! c1

3

/3

c1 1 et

and solution is c1 1 e 1 x

3

/3

[Putting –x3/3 " t so that –x2 dx " dt] or

q " (1/ x 2 )e x

3

/3

( c1 1 e 1 x

3

/3

dv / dx ( c1 / x 2 )e x

or

)

dv " [( c1 / x 2 )e x

or Integrating,

v " c1 / (1/ x 2 ) e x

3

/3

3

/3

3

/3

1 x 12

1 x 12 ] dx.

dx ! x 11 ! c2 , c1, c2 being arbitrary constants

... (7)

From (3), (4) and (7), the required general solution is y " uv " x 8c1 / (1/ x 2 ) e x < (d)

3

/3

dx ! x 11 ! c2 9 =

c1 x / (1/ x 2 ) e x

3

/3

dx ! c2 x ! 1

Ans. y " c1e x / x 11e x dx ! c2 e x ! e x log x.

Try yourself.

(e)

Dividing by x, the given equation in standard form is y# – (1 – 2/x)y∃ – (2/x)y " x2. Comparing (1) with y# ! Py∃ ! Q y " R, here P " –1 ! 2/x, Q " –2/x, Here 1 ! P ! Q " 0, showing that a part of C.F. of (1) is given by u " ex. Let the reqired general solution of (1) be y " uv. d 2v

2 du & dv % !∋P ! ( u dx ∗ dx dx )

Then v is given by or

2

d 2v % 2 2 de x ! ∋∋ 11 ! ! x 2 x e dx dx ) Let dv/dx " q Then (5) becomes Its

& dv (( ∗ dx

e

d 2v

or

x

dx 2

/ [( x

e x ! 2 log x 2 1x

e x 0 e log x

2

or

% 2 & dv ! ∋1 ! ( ) x ∗ dx

x 2 e 1 x . ... (5)

e ) 0 ( x 2 e x )] dx ! c1

x 2 e x and solution is

x 2 / 5 ! c1

or q " dv/dx " x 12 e1 x [ x 2 / 5 ! c1 ] ( x 3 / 5) ∆ e1 x ! c1 x 12 e1 x Integrating,

R u

so that d2v / dx2 " dq/dx. ... (6) 2 –x (dq/dx) ! (1 ! 2/x)q " x e , which is a linear equation.

(1! 2 / x ) dx I.F. " e /

q( x 2 e x ) "

x2

... (1) R " x .. (2) ... (3) ... (4) 2

or

dv " {(x3/5) × e–x + c1x–2e–x}dx

3 1x 12 1 x v " (1/ 5) ∆ / x e dx ! c1 / x e dx ! c2 , c1, c2 being arbitrary constants

3 1x 2 1x 12 1 x v " (1/ 5) ∆ 8< x (1e ) 1 / (3 x )(1e ) dx 9= ! c1 / x e dx ! c2

[Integrating by parts only the first integral]

Created with Print2PDF. To remove this line, buy a license at: http://www.software602.com/

Linear Equations of Second Order

10.19

" 1(1/ 5) ∆ x 3e 1 x ! (3 / 5) ∆ / x 2 e 1 x dx ! c1 / x 12 e1 x dx ! c2 " 1(1/ 5) ∆ x 3e 1 x ! (3 / 5) ∆ 8< x 2 (1e 1 x ) 1 / (2 x )(1e 1 x ) dx 9= ! c1 / x 12 e 1 x dx ! c2 " 1(1/ 5) ∆ x3 e 1 x 1 (3 / 5) ∆ x 2 e 1 x ! (6 / 5) ∆ / xe 1 x dx ! c1 / x 12 e1 x dx ! c2 " 1(1/ 5) ∆ x 3e 1 x 1 (3 / 5) ∆ x 2 e 1 x ! (6 / 5) ∆ 8< x (1e 1 x ) 1 / {10 ( 1e 1 x )}dx 9= ! c1 / x 12 e 1 x dx ! c2 " 1(1/ 5) ∆ x 3e 1 x 1 (3 / 5) ∆ x 2 e 1 x 1 (6 / 5) ∆ xe 1 x 1 (6 / 5) ∆ e 1 x ! c1 / x 12 e1 x dx ! c2 ? " 1(1/ 5) ∆ e 1 x ( x 3 ! 3 x 2 ! 6 x ! 6) ! c1 / x 12 e 1 x dx ! c2 .

or

... (7)

Hence from (3), (4) and (7), the required solution is x 1x 3 2 12 1 x y " uv " e 8< 1 ≅1/ 5 Α ∆ e ( x ! 3 x ! 6 x ! 6) ! c1 / x e dx ! c2 9=

y " c1e x / x 12 e1 x dx ! c2 e x 1 ≅1/ 5 Α ∆ ( x 3 ! 3 x 2 ! 6 x ! 6).

or

Ans. y " – c1 x e–x ! c2 ex – x ex

(f) Try yourself.

(2 x 1 1) / e2 x log (2 x 1 1) dx 9 dx. = (g) Comparing the given equation with y# ! Py∃ ! Q y " R, we have P " 0, Q " 1 ! (2/x) cot x – (2/x2), R " x cos x.... (1)

+e

x

/ (2 x 1 1) log(2 x 1 1)dx 1 2e / 8

% sin x & q∋ ( " ) x ∗

q"

or or or

/ dv

x 2 cot x.

2

... (4) d2v / dx2 " dq/dx.

x 2 cot x , which is a linear equation

e 2(log sin x 1log x)

2 8 2 % sin x & 9 x cot x 0 : ; dx ! c1 ∋ ( /: ) x ∗ ;=
by (1), or

and

d 2u dx 2

u

v!2

d2y dx 2

d 2u du dv d 2v v ! 2 ! u . dx dx dx 2 dx 2

du dv d 2v dv & % du ! u 2 ! P ∋ v ! u ( ! Quv dx dx dx dx ∗ dx )

R

% d 2u & d 2v % du & dv du ! Pu ! 2 ! v ! Qu (( R ∋∋ 2 ! P ∋ ( 2 dx ∗ dx dx dx ) ) dx ∗

Created with Print2PDF. To remove this line, buy a license at: http://www.software602.com/

Linear Equations of Second Order or

10.29

& 2 du & dv 1 % d 2u du R % !∋P! ! ∋∋ 2 ! P ! Qu (( v . ( u dx dx u dx u dx ) ∗ ) dx ∗ In order to remove the first derivative dv/dx from (2), we take du 1 2 du 1 Pdx. P! 0 or u 2 u dx d 2v

... (2)

2

... (3) 1

1 / Pdx Integrating, log u " 1 ≅1/ 2 Α ∆ / Pdx or u" e 2 ... (4) Thus, the required suitable subsitiution for the dependent variable is y " uv where u is given by (4)., Now, from (3), we have

du 1 " 1 Pu dx 2 2

d u

or

dx 2

d 2u

so that

dx

1 % 1 & 1 dP " 1 P ∋ 1 Pu ( 1 u, 2 ) 2 ∗ 2 dx

2

1

1 du 1 dP P 1 u 2 dx 2 dx

... (5)

du putting value of dx

& 1 % d 2u du 1 %1 1 dP 1 & ! Qu (( " ∋ P 2 u 1 u 1 P 2 u ! Qu ( , by (5) ∋∋ 2 ! P u )4 2 dx 2 u ) dx dx ∗ ∗

>

& 1 % d 2u du ! Qu (( ∋∋ 2 ! P u ) dx dx ∗

Thus,

1 1 dP Q 1 P2 1 4 2 dx

I , say

... (6)

Also take S " R/u Using (3), (6) and (7), (2) becomes d2v / dx2 ! Iv " S, where I and S are given by (6) and (7). (7) is known as normal form of (1).

... (7)

10.7 Working rule for solving problems by using normal form Step 1. Put the equation in the standard form y# ! Py∃ ! Qy " R, 2 2 in which the coefficient of d y / dx must be unity. u e Step 2. To remove the first derivative, we choose Step 3. We now asume that the complete solution of given equation is Then the given equation reduces to normal form d 2v

1

1 P dx 2/

y " uv.

1 1 dP R Q 1 P2 1 and S . 4 2 dx u dx Important Note. The success in solving the given equation depends on the success in solving d2v / dx 2 ! Iv " S. Now this latter equation can be solved easily if I takes two special forms (i) when I " constant, then resulting equation being with constant coefficients can be solved by usual methods of chapter 5 (ii) when I " (constant) / x2, then the resulting equation reduces to homogeneous form and hence it can be solved by using usual methods of chapter 6. Step 4. After getting v, the complete solution is given by y " uv. 2

! Iv

S,

where

I

10.8 Solved examples based on working rule 10.7 Ex. 1. Solve the following differential equations : (i) y# – 2 tan x 0 y∃ ! 5y " 0. [Agra 2006, 07; Delhi Maths (G) 1993] (ii) y# – 2 tan x 0 y∃ ! y " 0. [Delhi Maths (G) 1996, 98] (iii) y# – 2 tan x 0 y∃ – 5y " 0. [Delhi Maths (G) 1995]

Created with Print2PDF. To remove this line, buy a license at: http://www.software602.com/

Linear Equations of Second Order

10.30

Sol. (i) Given y# – 2 tan x 0 y∃ ! 5y " 0. Comparing (1) with y# ! Py∃ ! Q y " R, P " –2 tan x, Q " 5, To remove the first derivative from (1), we choose 1

R " 0.

... (1) ... (2)

1

1 / P dx 1 / ( 12 tan x ) dx e 2 u" e 2 Let the required general solution be Then v is given by normal form

where, I " Q – ( P 2 / 4) – (1/2) ∆ (dP/dx)

e logsec x

sec x.

... (3) ... (4) ... (5)

y " uv. (d v / dx2) ! Iv " S, 2

" 5 – (1/4) ∆ (4 tan2 x) – (1/2) ∆ (–2 sec2 x), by (2)

" 5 – tan2 x ! sec2 x " 5 – tan2 x ! (tan2 x ! 1) " 6 and S " R/u " 0, since R " 0 by (2). 2 Then (5) becomes (d v / dx2) ! 6v " 0 or (D2 ! 6)v " 0,where D Η d/dx. Here the auxiliary equation of (6) is

D2 + 6 " 0

so that

D " Ιi 6 .

So solution of (6) is v " C.F. " c1 cos ( x 6) ! c2 sin ( x 6). From (3), (4) and (7), the required general solution is y " uv (ii)

...(6)

... (7)

y " sec x[c1 cos ( x 6 ) ! c2 sin ( x 6 )]

or

Ans. y " sec x[c1 cos ( x 2) ! c2 sin ( x 2)]

Proceed as in part (i).

2x 12 x (iii) Proceed as in part (i). Ans. y " sec x[c1 e ! c2 e ] Ex. 2. Make use of the transformation y(x) " v(x) sec x to obtain the solution of

y# – 2y∃ tan x ! 5y " 0, y(0) " 0, y∃(0) " 6 . [I.A.S. 1997] Sol. Given, y# – 2y∃ tan x ! 5y " 0. ... (1) Also given y " v(x) sec x. ... (2) From (2), y∃ " v∃ sec x ! v sec x tan x. ... (3) From (3), y# " v# sec x ! 2v∃ sec x tan x ! v[sec x tan2 x ! sec3 x]. Substituting the above values of y, y∃ and y# in (1), we get v# sec x ! 2v∃ sec x tan x ! v(sec x tan2 x ! sec3 x) – 2 tan x (v∃ sec x ! v sec x tan x) ! 5v sec x " 0 3 or v# sec x ! v(sec x – sec x tan2 x ! 5 sec x) " 0 or v# ! v(sec2 x – tan2 x ! 5) " 0 or (D2 ! 6)v " 0, D Η d/dx. Its auxiliary equation is

D2 ! 6 " 0

so that

D"±i 6.

> v " C.F. " c1 cos ( x 6) ! c2 sin ( x 6), c1 and c2 being arbitrary constants. Hence from (2), the general solution of (1) is y(x) " sec x[c1 cos ( x 6 ) ! c2 sin ( x 6 )] .. (4) Putting x " 0 in (4) and using the given fact y(0) " 0, we get 0 " c1. Hence (4) reduces to y(x) " c2 sec x sin ( x 6) . From (5),

... (5)

y∃(x) " c2 sin x tan x sin ( x 6) ! c2 6 sec x cos ( x 6).

Putting x " 0 and using the given fact y∃(0) " from (5), the required solution is

6 , we get

6 " c2 6 so that c2 1 . Then,

y sec x sin ( x 6).

Created with Print2PDF. To remove this line, buy a license at: http://www.software602.com/

Linear Equations of Second Order

10.31

Ex. 3(a). Solve y# – 2 tan x 0 y∃ ! 5y " sec x 0 ex. [Agra 2006; Garhwal 2010; Kanpur 2009; Meerut 1998; Rhilkhand 2001; S.V.University (A.P.) 1997; (b) Solve v# – 2 tan x 0 y∃ – (a2 ! 1)y " ex 0 sec x. Gulbarga 2005] x Sol. (a). Given y# – 2 tan x 0 y∃ ! 5y " sec x 0 e . ... (1) Comparing (1) with y# ! Py∃ ! Q y " R, P " –2 tan x, Q " 5, R " sec x 0 ex. ... (2) To remove the first derivative from (1), we choose u e

1

1 Pdx 2/

e

1

1 ( 12 tan x ) dx 2/

e log sec x

sec x

... (3) ... (4) ... (5)

y " uv. (d v / dx2) ! Iv " S,

Let the required general solution be Then v is given by normal form

2

I " Q – P 2 / 4 – (1/2) ∆ (dP/dx) " 5 – (1/4) ∆ (4 tan2 x) – (1/2) ∆ (–2 sec2 x), by (2) " 5 – tan2 x ! sec2 x " 5 – tan2 x ! (tan2 x ! 1) " 6 and S " R/u " (sec x 0 ex) / sec x " ex, using (2) Then (5) becomes (d2v / dx2) ! 6v " ex or (D2 ! 6)v " ex, ... (6) where,

D2 ! 6 " 0

Its auxiliary equation is

D"Ιi 6.

so that

> C.F. of (6) " c1 cos ( x 6) ! c2 sin ( x 6) , c1and c2 being arbitrary constants P.I. "

and

1 ex D !6 2

1 ex 1 !6 2

1 x e . 7

Hence solution of (6) is v " c1 cos ( x 6) ! c2 sin ( x 6) ! (1/7) ∆ ex From (3), (4) and (7), the required general solution is y " uv

... (7)

y " sec x[c1 cos ( x 6 ) ! c2 sin ( x 6 )] ! (1/7) ∆ ex].

or

Ans. y " sec x [c1eax ! c2 e1ax ! e x / (1 1 a 2 )

(b) Do as in part (a).

Ex. 4(a). Solve (d2y / dx2) – (2/x) ∆ dy/dx) ! (n2 ! 2/x2)y " 0.

[Delhi Maths (G) 1997]

d % dy dy & 2 ∋ x 1 y ( 1 2 x ! 2 y ! x y 0. [Agra 2000, 03; Rohilkhand 2002, 04] dx ) dx dx ∗ Sol. (a) Comparing the given equation with y# ! Py∃ ! Q y " R, we get P " –(2/x), Q " (n2 ! 2/x2), and R " 0. ... (1) To reduce the given equation into normal form, we choose (b) Solve x

1

1 / P dx u" e 2 Let the required general solution be Then v is given by normal form

e

1

1 ( 12 / x ) dx 2/

e log x

x.

y " uv. (d2v / dx2) ! Iv " S,

... (2) ... (3) ... (4)

1 1 dP 2 1% 4 & 1% 2 & I " Q 1 P2 1 n 2 ! 2 1 ∋ 2 ( 1 ∋ 2 ( n 2 , by (1). 4 2 dx 4) x ∗ 2) x ∗ x and S " R/u " 0, as R " 0. 2 2 2 Then (4) becomes (d v / dx ) ! n v " 0 or (D2 ! n2) v " 0. ... (5) Its auxiliary equation is D2 ! n2 " 0 so that D " Ι in. > Solution of (5) is v " C.F. " c1 cos nx ! c2 sin nx. ... (6) From (2), (3) and (6), the required solution is y " uv or y " x(c1 cos nx ! c2 sin nx). where

Created with Print2PDF. To remove this line, buy a license at: http://www.software602.com/

Linear Equations of Second Order

10.32

% d 2 y dy dy & dy x ∋∋ x 2 ! 1 (( 1 2 x ! 2 y ! x 2 y "0 dx dx dx dx ) ∗

(b) Re-writing the given equation,

or (d2y / dx2) – (2/x) (dy/dx) ! (1 ! 2/x2)y " 0, which is just the same as part (a) taking n " 1. Ans. y " x(c1 cos x ! c2 sin x)

d % 2 dy & 2 ∋ cos x ( ! y cos x 0. dx ) dx ∗ Sol. Re-writing the given equation, we have Ex. 5. Solve

cos2 x

or

d2 y dx

2

1 2 cos x sin x

[Agra 2004, Rajasthan 2003, 06]

dy ! y cos2 x 0 dx

d2 y

or

dx 2

1 2 tan x

dy !y dx

0.

Now proceed as in Ex. 1(a). Ans. y " sec x [c1 cos ( x 2) ! c2 sin ( x 2)] Ex. 6(a). Solve (y# ! y) cot x ! 2(y∃ ! y tan x) " sec x. (b) Solve (y# ! y) cot x ! 2(y∃ ! y tan x) " 0. [Delhi Maths (H) 1999] Sol. (a) Given cot x 0 y# ! 2y∃ ! (cot x ! 2 tan x)y " sec x. y# ! 2 tan x ! (1 ! 2 tan2 x)y " sec x tan x. ... (1) Comparing (1) with y# ! Py∃ ! Q y " R, we have P " 2 tan x, Q " 1 ! 2 tan2 x and R " sec x tan x, ... (2) In order to remove the first derivative from (1), we choose 1

1

1 / P dx 1 / ( 2 tan x ) dx u" e 2 e 2 Let the required general solution be Then v is given by normal form

e log cos x

... (3) ... (4) ... (5)

cos x. y " uv. (d2v / dx2) ! Iv " S,

1 1 dP 1 1 I " Q 1 P2 1 1 ! 2 tan 2 x 1 (4 tan 2 x ) 1 (2 sec2 x ) 4 2 dx 4 2 " 1 ! tan2 x – sec2 x " sec2 x – sec2 x " 0 and S " R/u " (sec x tan x)/cos x " sec2 x tan x. Then (5) becomes (d2v / dx2) " sec2 x tan x " (sec x) (sec x tan x) Integrating it, dv/dx " (1/2) × sec2 x ! c1. ... (6) Integrating (6), v " (1/2) × tan x ! c1x ! c2, c1, c2 being arbitrary constants. ... (7) From (3), (4) and (7), the required general solution is y " uv or y " cos x [(1/2) ∆ tan x ! c1x ! c2]. (b) Hint : Proceed as in part (a). Note that R " 0 in this case and so S " R/u " 0. Hence (5) reduces to d2v / dx2 " 0. Integrating, dv/dx " c1 so that v " c1 x ! c2 . Hence the required solution is y " uv " cos x (c1x ! c2). Ex. 7. Solve y# – (2/x)y∃ ! (1 ! 2/x2)y " xex by changing the dependent variable. [Kanpur 2009; Patna 2003; Bangalore 2005] Sol. Comparing the given equation with y# ! Py∃ ! Q y " R, we get P " –2/x, Q " 1 ! (2/x2) and R " xex. ... (1) where

We choose

u" e

1

1 P dx 2/

e

1

1 ( 12 / x ) dx 2/

e/

(1/ x ) dx

e log x

x.

... (2)

Created with Print2PDF. To remove this line, buy a license at: http://www.software602.com/

Linear Equations of Second Order

10.33

Let the required general solution be y " uv. ... (3) Then v is given by the normal form (d2v / dx2) ! Iv " S, ... (4) 1 1 dP 2 1% 4 & 1% 2 & where I " Q 1 P2 1 1! 2 1 ∋ 2 ( 1 ∋ 2 ( 1 4 2 dx 4) x ∗ 2) x ∗ x x and S " R/u " (xe )/x " ex. 2 Then (4) becomes (d v / dx2) ! v " ex or (D2 ! 1)v " ex. ... (5) Its auxiliary equation is D2 ! 1 " 0 so that D"Ιi > C.F. of (5) " c1 cos x + c2 sin x, c1 and c2 being arbitrary constants. P.I. "

and

1 ex D !1

1 ex 1 !1

2

2

1 x e 2

v " C.F. ! P.I. " c1 cos x ! c2 sin x ! (1/2) × ex y " uv " x[c1 cos x ! c2 sin x ! (1/2) × ex].

Hence the solution of (5) is and so the required solution is

2

Ex. 8(a). Solve y# – 4xy∃ ! (4x2 – 1)y " 13e x sin 2 x. [Guwahati 2007; Meerut 2004; Delhi Maths (G) 2004, 05; I.A.S. 2000] Sol. Comparing the given equation with y# ! Py∃ ! Q y " R, we get P " –4x, Q " 4x2 –1 1

1 / P dx u" e 2 Let the required general solution be Then v is given by the normal form

We choose

1 1 dP I " Q 1 P2 1 4 2 dx

where

2

R 13e x sin 2 x.

and e

1

1 ( 14 x ) dx 2/

ex .

... (2)

y " uv. (d2v / dx2) ! Iv " S,

... (3) ... (4)

1 1 4 x 2 1 1 1 (16 x 2 ) 1 (14) 1 4 2 2

S " R/u " ( 13e x sin 2 x ) / e x

and

2

... (1)

2

1 3sin 2 x.

Then (4) becomes (d v / dx ) ! v " –3 sin 2x or (D2 ! 1)v " –3 sin 2x. Its auxiliary equation is D2 ! 1 " 0 so that D"Ιi > Its C.F. " c1 cos x ! c2 sin x, c1 and c2 being arbitrary constants 2

P.I. "

and

2

1 2

D !1

( 13sin 2 x ) 13

1 2

12 ! 1

sin 2 x sin 2 x.

> v " C.F. ! P.I. " c1 cos x ! c2 sin x ! sin 2x. From (2), (3) and (5), the required general solution is y " uv

... (5)

2

y " e x ( c1 cos x ! c2 sin x ! sin 2 x ).

or

Ex. 8(b). Solve y# – 4xy∃ ! (4x2 – 3)y " e x

2

[Delhi Maths (G) 2006, Bangalore 2005] 2

Ans. y e x ( c1e x ! c2 e 1 x 1 1)

Hint : Do as in part (a) 2

Ex. 8(c). Solve y# – 4xy∃ ! (4x2 – 1)y " 13e x (sin 2 x ! 5e12 x ! 6). Hint : As in part (a), here As before,

u ex

2

and

P " –4x, I " 1.

Q " 4x2 –1, Also

2

R " 13e x (sin 2 x ! 5e12 x ! 6). S " R/u " –3 sin 2x – 15e–2x –18.

Created with Print2PDF. To remove this line, buy a license at: http://www.software602.com/

Linear Equations of Second Order

10.34

Hence normal form is

(d2v / dx2) ! v " –3 sin 2x – 15e–2x – 18 (D ! 1)v " –3 sin 2x – 15e–2x – 18. C.F. " c1 cos x ! c2 sin x. 2

or As before, we get

1 1 1 1 ( 13sin 2 x 1 15e 12 x 1 18) " 13 2 sin 2 x 1 15 2 e 12 x 1 18 2 e0. x D2 !1 D !1 D !1 D !1 1 1 1 " 13 2 sin 2 x 1 15 e 12 x 1 18 2 e0. x " sin 2x – 3e–2x – 18. 2 12 ! 1 (12) ! 1 0 !1 > v " C.F. ! P.I " c1 cos x ! c2 sin x ! sin 2x – 3e–2x – 18.

P.I. "

2

y " uv " e x ( c1 cos x ! c2 sin x ! sin 2 x 1 3e 12 x 1 18).

and required solution is

Ex. 9(a). Solve y# – 2bxy∃ ! b2x2y " x. (b) Solve y# – 2bxy∃ ! b2x2y " 0. Sol. (a) Comparing the given equation with P " –2bx, Q " b2x2 1 1 / P dx e 2

I " Q1

R " x. ... (1)

2

... (2) ... (3) ... (4)

1 b 2 x 2 1 b2 x 2 1 (12b) b 2

xe1(bx

Then (4) becomes (d2v / dx2) ! bv

we get

e( bx / 2) y " uv. (d2v / dx2) ! Iv " S,

P 2 1 dP 1 4 2 dx

S " R/u

and

y# ! Py∃ ! Q y " R, and

1 1 / ( 12 bx ) dx e 2

We choose u" Let the required general solution be Then v is given by the normal form where

[Sagar 2002]

2

/ 2)

, using (1) and (2)

xe1(bx

2

/ 2)

(D2 ! b)v

or

Now the auxiliary equation of (5) is D2 ! b " 0

xe1(bx

2

/ 2)

... (5)

D"Ιi b.

so that

> C.F. of (5) " c1 cos( x b ) ! c2 sin( x b ) , c1, c2 being arbitrary constants and

2 1 xe 1( bx / 2) , which cannot be evaluated by well known methods. D !b 2 1 xe1 (bx / 2) > v " C.F. ! P.I. " c1 cos ( x b ) ! c2 sin ( x b ) ! 2 D !b From (2), (3) and (6), the required general solution is

P.I. "

2

... (6)

1 8 1 ( bx 2 / 2) 9 : (8) reduces to

/2

dx " / et dt , putting x 2 / 2 t and x dx dt

Thus, we have

/x

/2

v " / x2 e x

Integrating (7),

2

dv/dx" 2(t et 1 / et dt ) ! 3et ! c1 , integrating by parts.

or

or

Then

( x 3 ! 3 x)e x

2 2 dv " / x 3 e x / 2 dx ! 3/ x e x / 2 dx ! c1 . dx so that xdx " dt, (6) becomes

dv / dx " / (2t )et dt ! 3/ et dt ! c1

Now,

R u

2

/2

dx ! / e x

2

/2

dx ! c1 x ! c2

2

xe x

2

/2

! c1 x ! c2

2

y " uv " e 1 x / 2 [ xe x / 2 ! c1 x ! c2 ] x ! e 1 x / 2 ( c1 x ! c2 ) Ex. 10(b). Solve the equation xy# – 2(x ! 1)y∃ ! (x ! 2)y " (x – 2)ex, (x > 0) by changing into normal form. [Bangalore 1995] 2 x x Sol. Try yourself. Ans. y " –(1/2) ∆ x e ! xe ! (1/3) ∆ c1x3ex ! c2ex Ex. 11. Solve the following differential equations : (a)

d2 y dx 2 d2 y

!

1 dy % 1 1 6 & !∋ 1 1 ( 0. x1/ 3 dx ) 4 x 2 / 3 6 x 4 / 3 x 2 ∗

[Punjab 2003; Vikram 2001, 03]

1 dy 1 ! 2 ( x ! x1/ 2 1 8) y 0. [Agra 2005; Pune 2006, Vikram 2001, 03\ dx x dx 4 x (c) 4x2(d2y / dx2) ! 4x5(dy/dx) ! (x8 ! 6x4 ! 4)y " 0. [Vikram 2001,03] 3 2 2 2 2 (d) (x – 2x ) (d y / dx ) ! 2x (dy/dx) ! 12(x– 2)y " 0. (e) x2 (log x)2 (d2y / dx2) – 2x log x (dy/dx) ! [2 ! log x – 2 (log x)2]y " x2(log x)3 [Vikram 2001] (f) x2 (d2y / dx2) – 2x(3x – 2)(dy/dx) ! 3x(3x – 4)y " e2x. [Agra 1997] Sol. (a) Comparing the given equation with y# ! Py∃ ! Q y " R, we get

(b)

2

P

1

1/ 2

x1 x / 3,

We choose

Q

(1/ 4 x 2 / 3 ) 1 (1/ 6 x 4 / 3 ) 1 (6 / x 2 )

u"

1 1 / P dx e 2

1 1 / x 11/3 dx e 2

and 2/3

e 1(3/ 4) x .

R

0

... (1) ... (2)

Created with Print2PDF. To remove this line, buy a license at: http://www.software602.com/

Linear Equations of Second Order

10.36

Let the required general solution be y " uv. ... (3) Then v is given by the normal form (d2v / dx2) ! Iv " S, ... (4) 1 1 dP 1 1 6 1 1% 1 & 6 where I " Q 1 P2 1 1 4/3 1 2 1 2/3 1 ∋ 1 4/3 ( 1 2 2/3 4 2 dx 2 4x 6x x 4x ) 3x ∗ x and S " R/u " 0, as R " 0. Then (4) becomes (d2v / dx2) – (6/x2)v " 0 or (x2D2 – 6)v " 0... (5) which is a homogeneous linear equation. Here D Η d/dx. To solve it, let x " ex (or z " log x) and D1 Η d/dz.... (6) 2 2 We have x D " D1(D1 – 1). Then (6) reduces to [D1(D1 – 1) – 6]v " 0 or (D21 – D1 – 6)v " 0.... (7) 2 Here the auxiliary equation of (7), is D1 – D1 – 6 " 0. so that D1 " 3, –2. 3z 12 z C.F. " c1e ! c2 e

>

c1 ( e z )3 ! c2 (e z )12

c1 x 3 ! c2 x 12

v " C.F. " c1x3 ! c1x–2.

Hence the solution of (7) is

From (2), (3) and (8), the required solution is

y " uv

or

... (8)

y" e

1( 3 / 4 ) x 2 / 3

( c1 x ! c2 x 12 ) 3

1/ 2

(b) Try as in part (a). Ans. y " e x ( c1 x 2 ! c2 x 11 ) (c) Dividing by 4x2, the given equation in standard form is y# ! x3y∃ ! [(x8 ! 6x4 ! 4)/4x2]y " 0. ... (1) Comparing (1) with y# ! Py∃ ! Q y " R, we have P " x3, Q " x6/4 + 3x2/2 + 1/x2 and R " 0. ... (2) 1

1 / P dx u" e 2 Let the required general solution be Then v is given by the normal form

We choose

e

1

1 3 x dx 2/

e1 x

4

/8

y " uv. (d v / dx2) ! Iv " S,

Then (5) becomes (d2v / dx2) ! (1/x2)v " 0 or z Let x " e (or z " log x), D Η d/dx so that x2D2 " D1(D1 – 1). Then (6) reduces to {D1(D1–1) ! 1}v " 0 or Its auxiliary equation is D12 – D1 ! 1 " 0 >

C.F. of (7) " e "

so that

1 x

2

,S

R u

0.

(x2D2 ! 1)v " 0.... (6) and D1 Η d/dz (D21 – D1 ! 1)v " 0. ... (7)

D1 " (1Ι 1 1 4 )/2 " (1/2) Ι i ( 3 / 2 )

z/2

[c1 cos {( 3 / 2) z} ! c2 sin {( 3 / 2) z}] z (1/ 2) (e ) [c1 cos {( 3 / 2) z} ! c2 sin {( 3 / 2) z}] x1/ 2 [c1 cos {( 3 / 2) log x} ! c2 sin {( 3 / 2) log

" Hence the solution of (7) is given by or

... (4) ... (5)

2

1 2 1 dP x 6 3x 2 1 x 6 1 ! ! 2 1 1 ∆ (3x 2 ) I "Q1 P 1 " 4 2 dx 4 2 4 2 x

where

... (3)

.

x}] , as x = ez

v " C.F.

v " x1/ 2 [c1 cos {( 3 / 2) log x} ! c2 sin {( 3 / 2) log x}] From (3), (4) and (8), the required general solution is y uv

or

...(8)

4 y e1 x / 8 x1/ 2 8:c1 cos {( 3 / 2) log x} ! c2 sin {( 3 / 2) log x}9; < =

Created with Print2PDF. To remove this line, buy a license at: http://www.software602.com/

Linear Equations of Second Order

10.37

11 4 13 (d) Try yourself. Ans. y " ( x 1 2) (c1 x ! c2 x ) (e) Dividing by x2 (log x)2, the given equation in standard form is

d2 y

2 dy 2 ! log x 1 2(log x )2 ! y x log x dx x 2 (log x )2

1

dx 2 Comparing (1) with y# ! Py∃ ! Q y " R,

2 , P" 1 x log x

2

Q

2

x (log x )

2

!

1 2

x log x

2

and

x2

R log x.

... (2)

e log(log x )

log x.

... (3) ... (4) ... (5)

y " uv. (d v / dx2) ! Iv " S, 2

I " Q 1 (1/ 4) ∆ P 2 1 (1/ 2) ∆ (dP / dx)

8 9 1 1 ! :1 2 1 2 2; x (cos x ) x cos x x x (cos x ) < x cos x x (cos x ) = 2 I " –2/x and S " R/u " (log x)/(log x) " 1. 2 2 2 Then (5) reduce to (d v / dx ) – (2/x )v " 1 or (x2D2 – 2)v " x2. ... (6) x Let x " e (or z " log x), D Η d/dx and D1 " d/dz so that x2D2 " D1(D1 – 1). Then (6) reduces to [D1(D1 – 1) – 2]v " e2z or (D21 – D1 – 2)v " e2z. ... (7) Its auxiliary equation is D12 – D1 – 2 " 0 so that D1 " 2, –1 "

> and

1

(1/ x )

1

or

... (1)

we have

dx 1 / P dx / e log x We choose u" e 2 Let the required general solution be Then v is given by the normal form

where

log x.

2

2

2

!

1

2

2z 1z C.F. " c1e ! c2 e

P.I. " "

1 D12

1 D1 1 2

e2 z

1 1 e2 z 3 ( D1 1 2)1

1

2

2

1

1

2

c1 ( e z )2 ! c2 ( e z )11

2

c1 x 2 ! c2 x 11

1 1 e2 z ( D1 1 2) ( D1 ! 1) 1 z 2z e 3 1!

1 1 2z e D1 1 2 2 ! 1

8 :
Solution of (7) is v " C.F. ! P.I. " c1x2 ! c2x–1 ! (1/3) ∆ x2 log x ... (8) From (3), (4) and (8), the required general solution is y " uv or y " log x [c1x2 ! c2x–1 ! (1/3)x2 log x] (f)

Try yourself.

Ans. y " x12 e3x [c1x 2 ! c2 x 11 ! (1/ 3) ∆ x2 log x

Ex. 12(a). Reduce the equation x2y# – 2x(1 ! x)y∃ ! 2(1 ! x)y " x3, (x > 0) into the normal form and hence solve it. [Bangalore 1995]

2x 2 Ans. y " c1 xe ! c2 x 1 ( x / 2).

Ex. 12(b). Solve y# ! (4 cosec 2x)y∃ ! (2 tan 2x)y " ex cot x by changing the dependent variable. [Bangalore 2005] Sol. Comparing the given equation with y# ! Py∃ ! Q y " R, here P " 4 cosec 2x, Q " 2 tan2x and R " ex cot x... (1) 1 1 dP 1 1 Hence I " Q 1 P 2 1 " 2 tan2 x 1 ∆ (16 cosec 2 2 x ) 1 ∆ ( 18 cosec 2 x cot 2 x ) 4 2 dx 4 2

Created with Print2PDF. To remove this line, buy a license at: http://www.software602.com/

Linear Equations of Second Order

10.38

cos 2 x & % 1 " 2 tan 2 x 1 4 cosec 2 x ∋ 1 ( ) sin 2 x sin 2 x ∗ 8cosec 2 x sin 2 x % 1 1 cos 2 x & 2 " 2 tan 2 x 1 4 cosec 2 x ∋ ( 2 tan x 1 sin 2 x ) sin 2 x ∗

I"

Then,

2sin 2 x cos2 x

1

8sin 2 x

12(1 1 sin 2 x )

(2sin x cos x) 2

cos 2 x

12

... (2)

which is a constant. Hence to solve the given equation, we choose ( 1 P / 2) dx ( 12cosec 2 x ) dx e/ e1 log tan x cot x u " e/ ... (3) Let the required complete solution be y " uv ... (4) Then v is given by the normal form d2v / dx2 ! Iv " S ... (5) x x where S " R/4 " (e cot x)/cot x " e ... (6) Then (2), (6) and (5) yield d2v / dx2 – 2v " ex or (D2 – 2)v " ex ... (7)

D2 – 2 " 0

Here auxiliary equation is > C.F. of (7) " C1e x

2

! C2 e 1 x

2

P.I. of (7) "

and

2

, C1, C2 being arbitrary constants 1 ex D 12 2

1 ex (1 1 2)

1e x

v " C1e x

> v " C.F. ! P.I or From (3), (4) and (8), the required solution is y " uv

Ι

D

giving

2

! C2 e 1 x

y " cot x (C1e x

or

2

2

1 e x ... (8)

! C2 e1 x

2

1 ex )

EXERCISE 10(B) Solve the following differential equations by reducing to normal form: 1. x2 y2 – 2(x2 ! x)y1 ! (x2 ! 2x ! 2)y " 0. [Rohilkhand 2001; Mumbai 1997, Pune 1998; Nagpur 2000; Delhi Maths (G) 2001; Madurai Kamraj 2008; Kanpur 2008] Ans. y " xex (c1x ! c2) Ans. y " e

2. y2 ! 4xy1 ! 4x2y " 0 [Karnataka 2001, Vikram 1999]

11

1 x2 / 2

2

4. y2 ! 2 xy1 ! ( x ! 5) y xe 5. (1 – x2)y2 – 4xy1 – (1 ! x2)y " x. [I.A.S. 2004] x2

6. y2 – 4xy1 ! (4x2 – 1)y " e (5 1 3cos 2 x ).

nx

2

)

! c2 e1 nx )

1 x2 / 2

( c1 cos x ! c2 sin x ! x / 4) Ans. y " e Ans. y " (1 – x2)–1 (c1 sin x ! c2 cos x ! x) x2

Ans. y " e ( c1 cos x ! c2 sin x ! 5 ! cos 2 x ) Ans. y " sin x. (c1 ! c2x) x

2

Ans. y " e [ c1 cos( x 2 ) ! c2 sin( x 2 )]

.

9. y# ! 2xy∃ ! (x2 – 8)y " xe

! c2 e1 x

11

7. y# – 2 cot x 0 y∃ ! (1 ! 2 cot2x)y " 0. x

2

Ans. y " x (c1 cos nx ! sin nx )

(b) y2 ! (2/x)y1 ! n2y " 0. [Delhi Maths (G) 2000]

8. y# – 4xy∃ ! 4x2y " e

( c1e x

Ans. y " x (c1e

3. (a) y2 ! (2/x)y1 – n2y " 0.

2

1 x2

1 x2 / 2

10. y# ! (2/x)y∃ ! y " (sin 2x)/x.

.

Ans. y " e 1 x

2

/2

(c1e3 x ! c2 e12 x 1 x 2 / 9 1 2 / 81)

Ans. y " x 11 [c1 cos x ! c2 sin x – (1/3) ∆ sin 2x]

Created with Print2PDF. To remove this line, buy a license at: http://www.software602.com/

Linear Equations of Second Order 11. y# – 2xy∃ ! (x2 ! 2)y " e

( x2 ! x) / 2

10.39

.

Ans. y " e

x2 / 2

[c1 cos x 3 ! c2 sin x 3 ! (1 / 4) ∆ e( x

12. – 2xy∃ ! ! 2)y " 13. y# ! (2/x)y∃ – y " 0. [Nagpur 1996] x2 y#

(x2

2

! 2x) / 2

]

Ans. y " x(c1 cos x ! c2 sin x ! 2 Ans. y " x – 1 (c1ex ! c2e–x)

x3ex.

ex /

10.9. Transformation of the equation by changing the independent variable. Consider d2y / dx2 ! P(dy/dx) ! Q y " R, ... (1) where P, Q and R are functions of x and let the independent variable be changed from x to z, where z " f(x), say dy dy dz df df dz Using the formula, we have 0 0 , dx dz dx dx dz dx d2 y

and

dx

d2 y

or

dx

d % dy & d % dy dz & ∋ ( " ∋ ( dx ) dx ∗ dx ) dz dx ∗

"

2

2

d % dy & dz dy d 2 z ! ∋ ( dx ) dz ∗ dx dz dx 2 2

d % dy & dz dz dy d 2 z ∋ (∆ ∆ ! dz ) dz ∗ dx dx dz dx 2

d 2 y % dz & dy d 2 z ∋ ( ! dz 2 ) dx ∗ dz dx 2

Putting the above values of dy/dx and d2y / dx2 in (1), we obtain 2 d 2 y % dz & % d 2 z dz & dy ! Qy ( ! ∋∋ 2 ! P (( 2 ∋ dx ∗ dz dz ) dx ∗ ) dx

2

d 2 y % dz & dy d 2 z dy dz !P ! Qy ( ! 2 ∋ 2 dx dz dz dx dz ) ∗ dx

or

R

2

d2 y

% dz & Dividing by ∋ ( , we have ) dx ∗

where

P1 "

(d 2 z / dx 2 ) ! P (dz / dx ) (dz / dx)

2

dz 2

,

Q1

! P1

Q (dz / dx)

2

dy Q1 y dz

... (2)

R1 ,

and

R

R1

R (dz / dx )2

(3)

Here P1, Q1 and R1 are functions of x but these can be converted to functions of z by using the relation z " f(x). If by equating Q1 to a constant quantity we see that P1 also becomes constant then (2) can be solved (since it will be linear equation with constant co-efficient) to obtain the required solution. 10.10. Working Rule for solving equation by changing the independent variable: Step 1. Put the given equation in standard form by keeping the coefficient of d2y/dx2 as unity, i.e. y# ! Py∃ ! Q y " R. ... (1) Step 2. Suppose Q " Ι k f(x), then we assume a relation between the new independent variable z and the old independent variable x given by (dz/dx)2 " k f(x). Note carefully that we omit –ve sign of Q while writing this step. This is extremely important to find real values. Sometimes we assume that (dz/dx)2 " f(x) whenever we anticipate complicated relation between z and x. Step 3. We now solve (dz/dx)2 " k f(x). Rejecting negative sign, we get dz/dx " Now separating variables, (2) gives dz "

... (2)

[k f ( x) ]. [k f ( x )] dx

so that

z"

/

[ kf ( x )] dx, ... (3)

where we have omitted constant of integration since we are interested in finding just a relation between z and x.

Created with Print2PDF. To remove this line, buy a license at: http://www.software602.com/

Linear Equations of Second Order

10.40

form

Step 4. With the relationship (3) between z and x, we transform (1) to get an equation of the (d2y / dz2) ! P1(dy/dz) ! Q1y " R1, ... (4)

where P1 "

(d 2 z / dx 2 ) ! P (dz / dx) (dz / dx )

2

,

Q1

Q (dz / dx)

2

,

and ,

R1

R (dz / dx)2

... (5)

Ιk f ( x ) Ι k , a constant. Then we calculate P1. If P1 is also d f ( x) constant, then (4) can be solved because it will be a linear equation with constant coefficients. If, however, P1 does not become constant, then this rule will not be useful. The students must, therefore, be sure that P1 comes out to be constant before proceeding further. The value of P1, Q1 and R1 must be remembered for direct use in problems. R1 can be converted to a function of z by using (3). Step 5. After solving equation (4) by usual methods the variable z is replaced by x by using (3).

Now by virtue of (2), Q1 "

10.11. Solved examples based on Art. 10.10. Ex. 1. Solve sin2x y# ! sin x cos x 0 y∃ ! 4y " 0. or y# ! cot x 0 y∃ ! 4 cosec2 x 0 y " 0. [Agra 2006; Kanput 2006; Delhi Maths (G) 1997; Meerut 2001; Rohilknand 2001] Sol. Dividing by sin2x, the given equation becomes y# ! cot x 0 y ! 4 cosec2x 0 y " 0. ... (1) Comparing (1) with y# ! Py∃ ! Q y " R, we have P " cot x, Q " 4 cosec2 x and R " 0. ... (2) 2 2 We choose z such that (dz/dx) " 4 cosec x ... (3) *so that dz/dx " 2 cosec x giving z " 2 log tan (x/2) ... (4) Now changing the independent variable from x to z by using relation (4), (1) becomes (d2y / dz2) ! P1(dy/dz) ! Q1y " R1, ... (5) where

P1 "

( d 2 z / dx 2 ) ! P(dz / dx )

12cosec x cot x ! cot x 0 (2 cosec x )

( dz / dx )2

4 cosec 2 x

" 0,

[Using relations (2), (3) and (4)] Q1 "

4 cosec 2 x

Q

"1

2

and

R

0 , by (2) and (3). ( dz / dx ) 4 cosec x (dz / dx)2 > From (5), (d2y / dz2) ! y " 0 or (D12 ! 1)y " 0, where D1 Η d/dz 2 Its auxiliary equation is D1 ! 1 " 0 so that D1 " Ι i. Hence the required solution is y " C.F. " c1 cos z ! c2 sin z or y " c1 cos {2 log tan (x/2)} ! c2 sin {2 log tan (x/2)}. Ex. 2. Solve (a) y# ! (2/x)y∃ ! (a2/x4) y " 0. [Agra 2000;Kanpur 2005; Sagar 2004;] (b) x4y# ! 2x3y ! n2y " 0. [Kurukshetra 2000] Sol. (a) Comparing the given equation with y# ! Py∃ ! Q y " R, we get P " 2/x, Q " a2/x4 and R " 0. ... (1) We choose z such that (dz/dx)2 " a2/x4 ... (2) 2 *so that dz/dx " a/x giving z " –a/x. ... (3) 2

R1

* In practice, while extracting square root on both sides of (3), we shall take positive sign.

Created with Print2PDF. To remove this line, buy a license at: http://www.software602.com/

Linear Equations of Second Order

10.41

Now changing the independent variable from x to z by using relation (3), the given equation is transformed into (d2y / dz2) ! P1(dy/dz) ! Q1y " R1, ... (4) where

P1 "

Q1 "

(d 2 z / dx 2 ) ! P (dz / dx )

12a / x 3 ! (2 / x ) ∆ (a / x 2 )

(dz / dx) 2

(a 2 / x 4 )

a2 / x4

Q

0, using (1), (2) and (3) R

R1 "

0, by (1) and (2) (dz / dx )2 ( dz / dx ) a /x > From (4), (d2y / dz2) ! y " 0 or (D12 ! 1)y " 0, where D1 Η d/dz Its auxiliary equation is D12 ! 1 " 0 so that D1 " Ι i. Hence the required solution is y " c1 cos z ! c2 sin z or y " c1 cos (–a/x) ! c2 sin (–a/x) " c1 cos (a/x) – c2 sin (a/x), by (3) (b) Hint. Divide by x4, we have y# ! (2/x)y∃ ! (n2/x4)y " 0, which is the same as in part (a) with a " n. Ans. y " c1 cos (n/x) – c2 sin (n/x). 2 2 2 Ex. 3. Solve (1 ! x ) y# ! 2x(1 ! x )y∃ ! 4y " 0. [Meerut 2004; Vikram 2005] Sol. Dividing by (1 ! x2)2 the given equation in standard form is 2

2

and

1

4

d2 y

2 x dy 4 ! y 0. ... (1) 2 dx 1 ! x dx (1 ! x 2 )2 Comparing (1) with y# ! Py∃ ! Q y " R, we have P " (2x)/(1 ! x2), Q " 4/(1 ! x2)2 and R " 0 ... (2) 2 2 2 Choose z such that (dz/dx) " 4/(1 ! x ) so that dz/dx " 2/(1 ! x2). ... (3) dx Integrating, z " 2/ or z " 2 tan–1x... (4) 1 ! x2 Now changing the independent variable from x to z by using relation (4), (1) becomes (d2y / dz2) ! P1(dy/dz) ! Q1y " R1, ... (5) 2

d 2z dz !P 2 dx P1 " dx (dz / dx )2

where Q1

or

Q

≅ dz / dx Α

2

!

1

4 /(1 ! x 2 )2 4 /(1 ! x 2 )2

4x 2x 2 ! ∆ (1 ! x 2 )2 1 ! x 2 1 ! x 2 4 /(1 ! x 2 )2 1 and

R1

0, by (2) and (3)

R

0,

≅ dz / dx Α2

by

(2)

and

(3)

> From (5), (d2y/dz2) ! y " 0 or (D12 ! 1)y " 0, where D1 Η d/dz. Its auxiliary equation is D12 ! 1 " 0 so that D1 " Ι i. Hence the required solution is y " c1 cos z ! c2 sin z, c1, c2 being arbitrary constants. y " c1 cos (2 tan–1x) ! c2 sin (2 tan–1x). ... (6) –1 Let tan x " ϑ so that x " tan ϑ. Then, we have 1 1 tan 2 ϑ

1 1 x2

2 tan ϑ 2x . 2 1 ! tan ϑ 1 ! x 2 1 ! tan ϑ 1 ! x > (6) becomes y " c1[(1 – x2)/(1 ! x2)] ! c2[(2x)/(1 ! x2)] or (1 ! x2)y " c1(1 – x2) ! 2c2x. Ex. 4. Solve x6y# ! 3x5y∃ ! a2y " 1/x2. [Delhi Maths (G) 2006; Rajasthan 2010]

cos (2 tan–1x) " cos 2ϑ "

2

2

,

Sol. Dividing by x6, given equation becomes

sin (2 tan–1x) " sin 2ϑ "

d2 y dx

2

!

3 dy a 2 ! y x dx x 6

1 x8

... (1)

Created with Print2PDF. To remove this line, buy a license at: http://www.software602.com/

Linear Equations of Second Order

10.42

y# ! Py∃ ! Q y " R,

Comparing (1) with

2

a2

% dz & ∋ ( ) dx ∗ With this change, (1) becomes

Choose z such that

d 2z dz !P 2 dx P1 " dx (dz / dx )2

where and

x

Q " a2/x6,

R " 1/x8... (2)

a a so that z " 1 2 . ... (3) 3 x 2x 2 2 (d y / dz ) ! P1(dy/dz) ! Q1y " R1. ... (4) dz dx

3a 3 a ! ∆ x 4 x x3 (dz / dx )2 1

1

Q

Q1

0,

(dz / dx )2

1

2z

, by (2) and (3). ( dz / dx ) a /x a x a3 > (4) gives (D21 ! 1)y " –2z/a3. Its auxiliary equation is D21 ! 1 " 0, where D1 Η d/dz > D1 " Ι i and hence C.F. " c1 cos z ! c2 sin z. Now,

P.I. "

2

2

6

2 2

2 % 2z & 2 11 ∋ 1 3 ( 1 3 (1 ! D1 ) z !1 ) a ∗ a

1 D12

1

2 a

3

(1 1 ...) 0 z

1

2z a3

y " CF. ! P.I. " c1 cos z ! c2 sin z – 2z/a3

> Required solution is or

1

1/ x8

R

R1 "

or

6

P " 3/x,

1 1 % a & % a & % a & % a & y " c1 cos ∋ 1 2 ( ! c2 sin ∋ 1 2 ( ! 2 2 " c1 cos ∋ 2 ( 1 c2 sin ∋ 1 2 ( ! 2 2 . ) 2x ∗ ) 2x ∗ a x ) 2x ∗ ) 2x ∗ a x Ex. 5. Solve xy# – y∃ ! 4x3y " x5 or y# – (1/x) 0 y∃ ! 4x2y " x4. [Pune 2001, 05; Andhra 2003; Osmania 2003; Nagpur 1996; Garhwal 2005] Sol. Given y# – (1/x) 0 y∃ ! 4x2y " x4 ... (1) 2 4 Comparing (1) with y# ! Py∃ ! Q y " R, P " –1/x, Q " 4x , R " x . ... (2) 2

% dz & 2 ∋ ( " 4x ) dx ∗

Choose z such that

P1 "

so that

2x

d 2 y / dz 2 ! P1 ≅ dy / dz Α ! Q1 y

Then, (1) reduces to where

dz dx

or

d 2 z / dx 2 ! P ≅ dz / dx Α

2 ! ≅ 11/ x Α ∆ 2 x

≅ dz / dx Α2

4x2 x4

Q1

... (4)

R1 , Q

4 x2

( dz / dx )2

4 x2

>

R1 "

1

x2 4

z , by (2) and (3). 4 (dz / dx )2 4 x 2 > (4) gives (D21 ! 1)y " z/4, where D1 Η d/dz 2 Auxiliary equation is D1 + 1 " 0 giving D1 " ± i > C.F. of (5) " c1 cos z + c2 sin z, c1 and c2 being arbitrary constants.) 1 z 1 1 z 0 (1 ! D12 ) 11 z (1 1 D12 ! ....) z . and P.I. " 2 4 4 D1 ! 4 4 4

and

R

0,

z " x2 ... (3)

... (5)

So the required solution is y c1 cos z ! c2 sin z ! z / 4 c1 cos x 2 ! c2 sin x 2 ! x 2 / 4 Ex. 6(a) Solve cos x y# ! y∃ sin x – 2y cos3x " 2 cos5x. [Guwahati 2007; Bangalore 1995; Meerut 1997; Purvanchal 2007] Sol. Dividing by cos x, given equation in standard form is y# ! tan x 0 y' – (2 cos2x)y " 2 cos4x. ... (1)

Created with Print2PDF. To remove this line, buy a license at: http://www.software602.com/

Linear Equations of Second Order

10.43

Comparing (1) with P " tan x,

y# ! Py∃ ! Q y " R, Q " –2 cos2x

Choose z such that

(dz/dx)2 " 2 cos2x

> dz " 2 cos x dx With this z, (1) transforms to P1 "

where Q1 "

Q

we have and

R " 2 cos4x. ... (2) (dz/dx) "

or

2 cos x

so that z " 2 sin x. 2 (d y / dz ) ! P1 (dy/dz) ! Q1y " R1,

... (3) ... (4)

2

( d 2 z / dx 2 ) ! P (dz / dx )

1 2 sin x ! tan x 0 2 cos x

( dz / dx )2

2 cos2 x 2 cos 4 x

R

" cos2x " 1 – sin2x " 1 – z2/2,by (2) and (3). ( dz / dx )2 2 cos 2 x ( dz / dx )2 > (4) gives (D21 – 1)y " 1 – z2/2, where D1 " d/dz Auxiliary equation of (5) is D12 – 1 " 0 giving D1 " ± 1. > C.F. of (5) " c1ez + c2e–z, where c1 and c2 are arbitrary constants

and P.I. "

and R1 "

0,

11

1 D12

8 1 29 :1 1 z ; 11 < 2 =

1 D12

11

e0.z !

1 1 z2 2 2 (1 1 D1 )

1

1 eΚ0 z ! (1 1 D12 )11 z 2 2 0 11 2

" 11 ! (1/ 2) ∆ (1 ! D12 ! ....) z 2 " – 1 ! (1/2) ∆ (z2 ! 2) " z2/2. y " c1e z ! c2 e 1 z ! z 2 / 2

y " C.F. ! P.I., i.e.,

Hence the required solution is

c1 e 2 sin x ! c2e 1 2 sin x ! sin 2 x, as z " 2 sin x. Remarks. Sometimes a relation between new independent variable z and given independent variable is given in some problems and we are required to transform the given differential equation and hence solve it. We adopt the method explained in the following examples 6(b) and 6(d). Ex. 6(b). Transform the differential equation cos x 0 y# ! sin x 0 y∃ – 2y cos3x " 2 cos5x into the one having z as indpendent variable, where z = sin x and solve it. [Himachal 2003] Sol. Given that z " sin x so that dz/dx " cos x. ... (1)

or

y

dy dy dz " 0 dx dx dx

Now and

d2y dx

2

d % dy & ∋ ( dx ) dz ∗

cos x

dy , by (1) dz

... (2)

dy d % dy & d % dy & ∋ cos x ( " 1 sin x dz ! cos x dx ∋ dz ( dx ) dz ∗ ) ∗

dy d % dy & dz dy d2y ! cos x ∋ ( . 1 sin x ! cos2 x 2 , by (1) dz dz ) dz ∗ dx dz dz Using (2) and (3), the given equation becomes

" 1 sin x

or or

... (3)

% dy d2y & dy cos x ∋∋ 1 sin x ! cos2 x 2 (( ! sin x cos x 1 2cos3 x 0 y 2 cos5 x dz dz d z ) ∗ d2y / dz2 – 2y " 2 cos2x or d2y / dz2 – 2y " 2(1– sin2x) (D21 – 2)y " 2(1 – z2), where D1 Η d/dz ... (4)

D21 – 2 " 0

The auxiliary equation of (4) is Here,

C.F. of (4) " c1e z

2

! c2 e 1 z

2

so that

D1 " Ι

2.

, c1 and c2 being arbitrary constants

Created with Print2PDF. To remove this line, buy a license at: http://www.software602.com/

Linear Equations of Second Order

10.44

Also P.I. " 2

1 D12

12

1

(1 1 z 2 ) 2

12(1 1

D12

/ 2)

(1 1 z 2 )

" –(1 – D1 /2) (1 – z ) " –(1 ! D12/2 ! ...) (1 – z2) " –(1 – z2 – 1) " z2. 2

–1

2

y " c1e z

Hence complete solution of (4) is

! c2 e1 z

2

2

! z2.

2 sin x ! c2 e 1 2 sin x ! sin 2 x , as z " sin x y " c1e Ex. 6(c). Solve xy# ! (2x2 – 1)y∃ – 24x3y " 4x3 sin x2 using the transformation z " x2, x > 0. Sol. Try yourself as in Ex. 6(b). [Bangalore 1996]

or

–1 0 using the transformation z " tan x. [Bangalore 2005] z " tan–1x ...(1) 2 dz/dx " 1/(1 ! x ) ... (2)

Ex. 6(d). Solve (1 ! x2 )2 y ∃∃ ! 2 x(1 ! x 2 ) y ∃ ! y Sol. Given From (1) dy dx

Now, d2 y

and

dx d2 y

2

d % dy & dx ∋) dx (∗

dy dz 0 dz dx

d % 1 dy & dx ∋) 1 ! x 2 dz (∗

1

dy 1 ! x dz

... (3)

2

1

dy 1 d % dy & ! ∋ ( (1 ! x ) dz 1 ! x 2 dx ) dz ∗ 2x

2 2

dy 1 d % dy & dz 2x dy 1 d2 y ! 1 ! " , by (2) ∋ ( dx 2 (1 ! x 2 )2 dz 1 ! x 2 dz ) dz ∗ dx (1 ! x 2 )2 dz (1 ! x 2 )2 dz 2 Substituting values of dy/dx and d2y/dx2 as given by (3) and (4) in the given equation, we get 2x

1

2, d2y dy +, 1 2x 1 dy (1 ! x 2 ) 2 3 1 ! 2 x (1 ! x 2 ) ∆ !y 0 2 2 2 2 2 dz − (1 ! x ) 1 ! x 2 dz 4, (1 ! x ) dz ., or d2y/dz2 ! y " 0 or (D12 ! 1)y " 0, where D1 Η d/dz whose general solution is y " C1 cos z ! C2 sin z,C1,C2 being arbitrary constants. y " C1 cos ( tan 11 x ) ! C2 sin ( tan 11 x ),

or

From Trigonomery,

6

11 2 1/ 2 tan 11 x " cos 1 /(1 ! x )

by (1)

7

... (5)

6

sin 11 x /(1 ! x 2 )1 / 2

7

Hence, from (5), the required solution takes the form

6

7

6

11 2 1/ 2 ! C2 sin sin 11 x /(1 ! x 2 )1 / 2 y " C1 cos cos 1 /(1 ! x )

7

y " C1/ (1 + x2)1/2 + (C2x) / (1 + x2)1/2 or y(1 ! x2 )1/ 2 C1 ! C2 x Ex. 7(a) Solve the equation d2y / dx 2 ! (2 cos x ! tan x) × (dy/dx) ! y cos2x " cos4x by changing the independent variable. [Gulbarga 2005] Sol. Comparing the given equation with y# ! Py∃ ! Q y " R, we have P " 2 cos x ! tan x, Q " cos2x and R " cos4x ... (1) Choose z such that (dz/dx)2 " cos2x or dz/dx " cos x ... (2) From (2), dz " cos x dx so that z " sin x ... (3) With this value of z, the given equation transforms to d2y / dz2 ! P1(dy/dz) ! Q1y " R1 ... (4) or

where

P1 "

d 2 z / dx 2 ! P( dz / dx ) ( dz / dx )

2

1 sin x ! (2 cos x ! tan x ) ∆ cos x cos 2 x

2

Created with Print2PDF. To remove this line, buy a license at: http://www.software602.com/

Linear Equations of Second Order Q

10.45

cos 4 x

R

cos2 x 1 1 sin 2 x 1 1 z 2 (dz / dx) 2 ( dz / dx )2 cos 2 x Hence (4) yields d2y/dz2 ! 2(dy/dz) ! y " 1– z2 or (D21 ! 2D1 ! 1)y " 1 – z2 2 2 or (D1 ! 1) y " 1 – z , where D1 Η d/dz. ... (5) 2 The auxiliary equation of (5) is (D1 ! 1) " 0 giving D1 " –1, –1 > C.F. " (C1 ! C2 z)e–z, C1 and C2 being arbitrary constants 1 and P.I. " (1 1 z 2 ) (1 ! D1 ) 12 (1 1 z 2 ) " (1 1 2 D1 ! 3 D12 ! ...)(1 1 z 2 ) ( D1 ! 1) 2 " 1 – z2 – 2D1(1– z2) ! 3D12(1 – z2) ! ... " 1 – z2 – 2 × (– 2z) ! 3 × (–2) " – z2 ! 4z – 5 Hence the required solution is y " C.F. ! P.I or y " (C1 ! C2z)e–z – z2 ! 4z – 5 –sin x 2 or y " (C1 ! C2 sin x)e – sin x ! 4sin x – 5, as z " sin x 2 2 Ex. 7(b) Solve x(d y / dx ) – (dy/dx) – 4x2y " 8x3 sin x2. [Kanpur 2002, Rohilkhand 2001; Bangalore 2005; Garhwal 1994; Vikram 2001, 05] [Gurukul Kangri U. 2004; Agra 2005] 2 2 Sol. Dividing by x, (d y / dx ) – (1/x) (dy/dx) – 4x2y " 8x2 sin x2. ... (1) Comparing (1) with y# ! Py∃ ! Q y " R, we have P " –1/x, Q " –4x2, and R " 8x2 sin x2. ... (2) 2 2 Choose z such that (dz/dx) " 4x or (dz/dx) " 2x so that z " x2. ... (3)

Q1 "

"1

and

R1 "

P1 " 0, Q1 " –1, R1 " (8 x 2 sin x 2 ) / 4 x 2 2sin x 2 2sin z. (D12 – 1)y " 2 sin z. whose C.F. " c1ez ! c2e–z 1 1 2 sin z 2 2 sin z 1 sin z. and P.I. " 2 D1 1 1 11 1 1 1z z x2 1 x2 2 > The required solution is y " c1e ! c2 1 sin z c1e ! c2 e 1 sin x . Ex. 8(a). Solve y# – y∃ cot x – y sin2 x " cos x – cos3 x. (b) Solve y# – y∃ cot x – y sin2 x " 0. [Rohilkhand 1996] Sol. (a) Comparing the given equation with y# ! Py∃ ! Q y " R, we have P " –cot x, Q " –sin2x and R " cos x – cos3 x " cos x sin2 x. ... (1) 2 2 Choose z such that (dz/dx) " sin x and dz/dx " sin x... (2) As usual, > You will get

Integrating, z " / sin x dx or z " –cos x.... (3) Now changing the independent variable form x to z by using relation (3), the given equaiton is transformed into (d2y / dz2) ! P1(dy/dz) ! Q1y " R1, ... (4) 2 2 d z / dx ! P( dz / dx ) cos x ! ( 1 cot x ) (sin x ) 0, by (1) and (2) where P1 " ( dz / dx )2 sin 2 x Q1 "

(1 sin 2 x)

Q

2

1 1,

cos x 0 sin 2 x

R

cos x 1 z. (dz / dx ) sin x (dz / dx ) sin 2 x > From (5), (d2y / dz2) – y " –z or (D21 – 1)y " –z, where D1 Η d/dz, ... (5) Its auxiliary equation is D12 – 1 " 0 so that D1 " Ι 1 z 1z 1 cos x cos x c1e ! c2 e > C.F. of (5) is " c1e ! c2 e , by (3) 1 1 (1 z ) z (1 1 D12 ) 11 z (1 ! D12 ! ...) z z 1 cos x. P.I. " 2 D1 1 1 1 1 D12 Hence the required solution is y " c1e1 cos x ! c2 ecos x 1 cos x. 1 cos x ! c2 ecos x (b) Try yourself as in part (a). Ans. y " c1e 2

R1

2

Created with Print2PDF. To remove this line, buy a license at: http://www.software602.com/

Linear Equations of Second Order

10.46

Ex. 9. Solve (a) (1 ! x)2 (d2y / dx2) ! (1 ! x)(dy/dx) ! y " 4 cos log (1 ! x). [Delhi Maths (Hons.) 1993]

(b)(1 ! x )2 ( d 2 y / dx 2 ) ! (1 ! x)( dy / dx) ! y 4 sin log (1 ! x ) . Sol. (a) Dividing by (1 ! x)2, the given equation in standard form is d2 y dx 2

Comparing (1) with P " (1 ! x)–1, Choose z such that

1 dy 1 4 cos log(1 ! x ) ! y . ... (1) 1 ! x dx (1 ! x )2 (1 ! x )2 y# ! Py∃ ! Q y " R, we have Q " (1 ! x)–2 and R " 4(1 ! x)–2 cos log(1 ! x). ... (2) 2 2 (dz/dx) " 1/(1 ! x) so that dz/dx " 1/(1 ! x) ... (3) !

1

/ 1 ! x dx

z"

Integrating it

With this z, (1) reduces to P1 "

where Q1 " > >

2

( d z / dx ) ! P( dz / dx )

(1 ! x )12

Q

(d2y / dz2) ! P1(dy/dz) ! Q1y " R1,

2

( dz / dx )

z " log (1 ! x). ... (4)

or

1(1 ! x )

12

11

! (1 ! x ) (1 ! x )

0,

(1 ! x )12

2

4(1 ! x )12 cos log(1 ! x )

R

R1 "

1,

... (5)

11

4 cos z ( dz / dx )2 (1 ! x )12 ( dz / dx )2 (1 ! x )12 From (5), (d2y / dz2) ! y " 4 cos z or (D12 ! 1)y " 4 cos z, where D1 Η d/dz. ... (6) Its auxiliary equation is D12 ! 1 " 0 so that D1 " Ι i C.F. of (6) " c1 cos z ! c2 sin z " c1 cos log (1 ! x) ! c2 sin log (1 ! x), by (4) 1

1

z sin z, (2 ∆ 1)

1 z cos az sin az as 2 2 2 a !1 D1 ! a " 2 log (1 ! x) sin log (1 ! x), using (4) Hence the required general solution is y " C.F. ! P.I., i.e. y " c1 cos log (1 ! x) ! c2 sin log (1 ! x) ! 2 log (1 ! x) sin log (1 ! x).

P.I. "

(b)

or

!1

4 cos z

4

D12

2

cos z

4

Try yourself as in part (a) Use the result

1 D12

!a

2

sin az

1

d z cos az , D1 Η dz 2a

Ans. y " c1 cos log (1 ! x) ! c2 sin log (1 ! x) – 2 log (1 ! x) cos log (1 ! x) Ex. 10. Solve (d2y / dx2) ! (tan x – 1)2 (dy/dx) – n(n – 1)y sec4x " 0. Sol. Comparing the given equation with y# ! Py∃ ! Q y " R, we get P " (tan x – 1)2, Q " –n(n – 1) sec4 x and R " 0. ... (1) 2 4 Choose z such that (dz/dx) " sec x ... (2) dz/dx " sec2x so that z " tan x ... (3) With this z, the given equation becomes (d2y / dz2) ! P1(dy/dz) ! Q1y " R1, ... (4)

where Q1 " >

D12

P1 "

( d 2 z / dx 2 ) ! P (dz / dx)

2 sec 2 x tan x ! (tan x 1 1)2 sec 2 x

( dz / dx )2

sec 4 x

1 n( n 1 1) sec 4 x

Q

1 n ( n 1 1)

4

R1

R

0. ( dz / dx ) sec x ( dz / dx ) 2 From (4), (d2y / dz2) ! (dy/dz) – n(n – 1)y " 0 or [D12 ! D1 – n(n ! 1)]y " 0, D1 Η d/dz 2

and

1,

Created with Print2PDF. To remove this line, buy a license at: http://www.software602.com/

Linear Equations of Second Order

10.47

or

Its auxiliary eqaution is D12 ! D1 – n(n – 1) " 0 or (D12 – n2) ! (D1 ! n) " 0 (D1 ! n) (D1 – n) ! (D1 ! n) " 0 or (D1 ! n) (D1 – n ! 1) " 0 so that D1 " –n, n –1

>

The required solution is

y " c1e1 nz ! c2e( n11) z

c1e1 n tan x ! c2 e( n11) tan x , as z " tan x.

EXERCISE 10(C) Solve the following differential equations : 1. y# ! y∃ tan x ! y cos2x " 0.

[Delhi Maths (G) 2004; Meerut 1996; Mysore 2004] Ans. y " c1 cos(sin x) ! c2 sin (sin x) Ans. y " e

2. xy# ! (4x2 – 1)y∃ ! 4x3y " 2x3.

1 x2

( c1 ! c2 x 2 ) ! (1/ 2)

2 1/ 2 2 1/ 2 Ans. y " c1 cos{n ( x 1 1) } ! c2 sin {n ( x 1 1) }.

3. (x3 – x)y# ! y∃ ! n2x3y " 0. 4. y# ! (tan x – 3 cos x)y∃ ! 2y cos2x " cos4x. Ans. y " c1e

sin x

5. (a) (a2 – x2)y# – (a2/x)y∃ ! (x2/a)y " 0.

! c2 e1 sin x 1 (5 / 4) 1 (3/ 2) ∆ sin x 1 (1/ 2) ∆ sin 2 x.

[BundelKhand 2001] Ans. y " c1 cos {(a2 – x2)/a}1/2 ! c2sin{(a2 – x2)/a}1/2 Ans. y " c1 cos{(81 – x2)1/2/9} ! c2 sin{(81 – x2)1/2/9}

(b) (81 – x2)y# – (81/x)y∃ ! (x2/9)y " 0.

Ans. y " c1e cos x ! c2 e 2cos x ! (1/ 6) ∆ e 1 cos x

6. y# ! (3 sin x – cot x)y∃ ! 2y sin2x " e–cos x sin2x.

x 7. y# – (1 ! 4ex)y∃ ! 3e2xy " e 2( x ! e ) . [Agra 2006;Kanpur 1997]

Ans. y " c1e

3e x

x

! c2 e e 1 e2 e

x

8. y# ! (1 – 1/x)y∃ ! 4x2e–2xy " 4(x2 ! x3)e–3x. Ans. y " c1 cos {2e–x(1 ! x)} ! c2sin{2e–x (1 ! x)} ! ex(1 ! x). Ans. y " e 2 x [c1 cos( x 3) ! c2 sin( x 3)] ! (1/ 4) ∆ e 2 x ! 1

9. y# – (8e2x ! 2)y∃ ! 4e4xy " e6x.

10. 12 An important theorem. If y = y 1 (x) and y = y 2 (x) are two solutions of the equation (d 2 y / dx 2 ) + P(x) (dy/dx) + Q(x)y = 0, where P(x), Q(x) are continuous function of x, prove that dy2 dy 1 P dx , c being an arbitrary constant. 1 y2 1 c e / dx dx [Himachal 2000; Kalkata 2001, 03 05, 07; Kurukshetra 200, 03; Allahabad 2002, 04, 07; Lucknow 2001, 04] Proof. Since y1 and y3 are solutions of the given equation, we have y1

d 2 y1 dx

and

2

d 2 y2

! P( x )

dy1 ! Q( x ) y1 dx

0.

dy2 ! Q ( x ) y 2 0. dx dx Multiplying (1) by y2 and (2) by y1 and then substracting, we get 2

y1

Let

d 2 y2 dx

2

1 y2

! P( x )

d 2 y1 dx y1

2

dy & % dy ! P ∋ y1 2 1 y2 1 ( 0. dx ∗ ) dx

dy2 dx

1 y2

dy1 dx

v.

... (1) ... (2)

... (3) ... (4)

Created with Print2PDF. To remove this line, buy a license at: http://www.software602.com/

Linear Equations of Second Order

10.48

Differentiating both sides of (4) w.r.t. x, we get % d 2 y 2 dy1 dy2 & % d 2 y1 dy2 dy1 & ! ! ∋∋ y1 ( 1 ∋ y2 ( dx dx (∗ ∋) dx dx dx (∗ dx 2 ) dv > (3) becomes ! Pv 0 dx

Integrating, or

dv dx

dy2 dy 1 y2 1 dx dx

d 2 y2 dx 2

dv v

or

log v – log c " 1/ P dx y1

y1

or

1 y2

d 2 y1 dx 2

dv . dx

1 Pdx. 1 P dx v " ce /

or

1 P dx , using (4) ce /

10.13 Method of variation of parameters We have already explained the method of variation of parameters for solving dy 2 /dx 2 ! P(dy/dx) ! Qy " R, where P,Q and R are functions of x in Art 7.3, Art. 7.4A and Art. 7.4B in chapter 7. So far we have used the method of variation of parameters to solve linear differential equations with constant coefficients or Cauchy-Euler equations (refer chapter 7). In this article, we purpose to solve differential equations whose complementary function can be obtained by methods of the present chapter However, it should be carefully noted that the method of variation of parameters is used when (i) The solution of y2 ! Py1 ! Qy " R coannot be obtained by mehtods explained in Art. 10.3, Art. 10.7 and Art. 10.10. (ii) You are asked in solve a given equation by using variation of parameters. 10.14 Solved examples based on Art. 10.13 Ex. 1. Verify that e x and x are solutions of the homogeneous equation corresponding to (1 – x)y2 ! xy1 – y " 2(x – 1)2e–x, 0 < x < 1. Thus find its general solution. Sol. The given equation in standard form is y2 ! [x/(1 – x)]y1 – y/(1 – x) " 2(1 – x)e–x. ... (1) Consider y2 ! [x/(1 – x)]y1 – y/(1 – x) " 0 ... (2) x which is said to be the homogeneous equaiton corresponding to (1). Take y " e so that y1 " ex, y2 " ex. With these values, x x ex 1 1 x ! x 11 e 1 ex 0. 11 x 11 x 11 x So ex is a solution of (2). Next, take y " x so that y1 " 1, y2 " 0. with these values,

the L.H.S. of (2) " e x !

the L.H.S. of (2) " 0 !

1 x x1 1! x 11 x

0.

So x is also a solution of (2). Now, the Wronskian W(ex, x) of ex and x is given by W(ex, x) "

ex x e

x

1

e x 1 xe x 5 0

Hence ex and x are linearly independent solutions of (2) [refer chapter 1]. Hence the general solution of (2) is y " aex ! bx. So the C.F. of (1) is aex ! bx, a and b being arbitrary constants. We shall now use the method discussed in Art. 7.4B of chapter 7 Let y " Aex ! Bx ... (3)

Created with Print2PDF. To remove this line, buy a license at: http://www.software602.com/

Linear Equations of Second Order

10.49

be the complete solution of (1). Then A and B are functions of x which are so chosen that (1) will be satisfied. Differentiating (3), we get y1 " A1ex ! Aex ! B1x ! B. ... (4) x Choose A and B such that A1e ! B1x " 0. ... (5) Then (4) reduces to y1 " Aex ! B. ... (6) x x Differentiating (6), y2 " A1e ! Ae ! B1, where A1 " dA/dx, B1 " dB/dx ... (7) Using (3), (6) and (7), (1) reduces to A1ex ! B1 " 2(1 – x)e–x. ... (8) Subtracting (5) from (8), we have (1 – x)B1 " 2(1 – x)e–x or B1 " dB/dx " 2e–x so that B " c1 – 2e–x. –x x –2x Then (5) gives A1 " dA/dx " %##$ 2xe )/e " –2xe . Integrating and using the chain rule of integration by parts, we have 1& 8 % 1 & %1 &9 % c2 1 2 :( x ) ∋ 1 e 12 x ( 1 (1) ∋ e12 x (; c2 ! e12 x ∋ x ! ( 2 4 2∗ ∗ ) ∗= ) < ) Putting the value of A and B in (3), the required solution is y " [c2 ! e–2x(x ! 1/2)]ex ! (c1 – 2e–x)x " c1x ! c2ex ! e–x[(1/2) – x] Ex. 2. (i) Using the method variation of parameters, solve the differential equation (x – 1)D2y – xDy + y = (x – 1)2, where D Η d/dx. [Guwahati 2007] (ii) Apply the method of variation of parameters to solve (x – 1)y2 – xy1 + y = (x – 1)2, given that the integrals in the complementary function are x and ex Sol. (i) The given equation in the standard form y2 + Py1 + Q y = R is

A " 12/ xe12 x dx ! c2

x 1 y1 ! y x 1 1. x 11 x 11 x 1 y2 1 y1 ! y 0. x 11 x 11 y2 ! Py1 ! Q y " R, we have Q " 1/(x – 1). We easily verify that y2 1

Consider

... (1) ... (2)

Comparing (2) with P " x/(x – 1), x x x 1 P ! Qx " 1 and 1 ! P ! Q " 11 ! 0 ! 1 0. x 11 x 11 x 1 1 x 11 So x and ex are integrals of C.F. (1) or solutions of (2). [See Art. 10.3].Again, we have Wronskian of e and x " W(e , x) " x

x

ex x

x

e x 1 xe x 5 0.

e 1 Hence, e and x are linearly independent solutions of (2) [See chapter 1]. Hence, the general solution of (2) is y " aex ! bx. So the C.F. of (1) is aex ! bx, a and b being arbitrary constants. We now use Art. 7.4B of chapter 7. Let y " Aex ! Bx ... (3) be the complete solution of (1). Then A and B are functions of x which are so chosen that (1) will be satisfied. Differentiatin (3), we get y1 " A1ex ! Aex ! B1x ! B. ... (4) Choose A and B such that A1ex ! B1x " 0. ... (5) x Then (4) reduces to y1 " Ae ! B. ... (6) x x Differentiating (6), y2 " A1e ! Ae ! B1, where A1 " dA/dx, B1 " dB/dx ... (7) Using (3), (6) and (7), (1) reduces to A1ex ! B1 " x – 1. ... (8) x

Created with Print2PDF. To remove this line, buy a license at: http://www.software602.com/

Linear Equations of Second Order

10.50

Subtracting (5) from (8), we have (1 – x)B1 " x – 1 Hence B1 " dB/dx " –1 so that B " c1 – x. x Then (5) gives A1 " dA/dx " x/e " xe–x. Integrating,

A " c2 !

/ xe

1x

c2 ! ( x )( 1e 1 x ) 1 / {1 0 ( 1e 1 x )} dx

dx

c2 1 e1 x ( x ! 1)

Putting the above values of A and B in (3), the required solution is y " [c2 – e–x(x ! 1)]ex ! (c1 – x)x " c1x ! c2ex – (x2 ! x ! 1). Part (ii). Since x and ex are integrals in C.F. So aex ! bx is C.F. of (1). So now start from equation (3) onwards as in part (i). Ex. 3. Apply the method of variation of parameters to solve the equation (x ! 2)y2 – (2x ! 5)y1 ! 2y " (x ! 1)ex. [Kanpur 2009] x Or Solve (x ! 2)y2 – (2x ! 5)y1 ! 2y " (x ! 1)e by method of variation of parameters when C.F. is a (2x ! 5) ! be2x. [Kakitya 1997] Sol. Putting the given equation in standard form y2 ! Py1 ! Q y " R, we get 2x ! 5 2 x !1 x y1 ! y e . x!2 x!2 x!2 2x ! 5 2 y2 1 y1 ! y 0. x!2 x!2 y2 ! Py1 ! Q y " R, we have Q " 2/(x ! 2)

y2 1

Consider Comparing (2) with P " –(2x ! 5)/(x ! 2),

... (1) ... (2) R " 0.

and

2(2 x ! 5) 2 Here 22 ! 2P ! Q " 4 1 ! 0. x!2 x!2 Hence u " e2x [See Art. 10.3] is an integral of (2). We now use method of Art. 10.4 to find solution of (2). Let the complete solution of (2) be y " uv. Then (2) reduces to d 2v

2 du 9 dv 8 ! :P ! 2 u dx ;= dx dx
xD2 ! (1 – x)D – 1 " (xD ! 1)(D – 1). ... (2) If we take the other order i.e. (D – 1)(xD ! 1), then we get (D – 1)(xD ! 1) " D(xD ! 1) – (xD ! 1) " 10 D ! x 0 D2 ! D – xD –1, which is different from L.H.S. of (2). So the order in (2) is correct. Using (2), (1) gives (xD ! 1)(D – 1)y " ex. ... (3) Let (D – 1)y " v. ... (4) x Then (3) gives (Dx ! 1)v " e . ... (5) We first slove (5), i.e.

x

(1/ x )0dx which is linear. Its I.F. " e /

vx "

dv ! v ex dx

e log x

/ x 0 (1/ x )e

x

dv 1 ! v dx x

or

1 x e , x

x and solution is

dx ! c1

e x ! c1 ; c1 being on arbitary constant.

> v " (1/x)ex ! (1/x)c1. Putting this value of ? in (4), we get (D – 1)y " (1/x)ex ! (1/x)c1 or(dy/dx) – y " (1/x)ex ! (1/x)c1, which is again a linear equation of the first order. 1 dx Its I.F. " e /

e1 x . Hence its solution is given by 1 & e1 x %1 dx. ye–x " c2 ! / ∋ e x ! c1 ( e 1 x dx c2 ! log x ! c1 / x ∗ x )x

y " c1e x / ( e1 x / x ) dx ! c2 e x ! e x log x.

> The solution of the given equation is

Ex. 2. Factorise the operator on the L.H.S of [(x ! 2)D2 – (2x ! 5)D ! 2]y " (x ! 1)ex and hence solve it. [Guwahat 1997, Kanpur 1998] Sol. L.H.S. of the given equation " (x ! 2)D2 – [2(x ! 2)+1] ! 2 " (x ! 2)D2 – 2(x ! 2)D – (D – 2) " (x ! 2)D (D – 2) – (D – 2) " [(x ! 2)D – 1](D – 2). ... (1) We cannot reverse the order, for then (D – 2)[(x ! 2)D – 1] " D ! (x ! 2)D2 – D – 2(x ! 2)D ! 2 " (x ! 2)D2 – (2x ! 4)D ! 2. which is clearly different from L.H.S. of the given equation. Thus the order (1) is correct. Hence the given equation gives [(x ! 2)D – 1](D – 2)y " (x ! 1)ex. ... (2) Put (D – 2)y " v. ... (3) Then, (2) gives [(x ! 2)D – 1]v " (x ! 1)ex. ... (4) We first solve (4), i.e., 1 dx /( x ! 2) Its I.F. " e /

( x ! 2)

dv 1v dx

( x ! 1)e x

or

dv 1 1 v dx x ! 1

x !1 x e , x!2

e 1 log ( x ! 2) 1/( x ! 2) and its solution is given by

Created with Print2PDF. To remove this line, buy a license at: http://www.software602.com/

Linear Equations of Second Order v0

" or

1 " x!2

x !1

/ ( x ! 2)2 e

x

10.57

dx ! c1

/

( x ! 2) 1 1 ( x ! 2)

2

e x dx

e x dx ! c1 "

e x dx

/ x ! 2 1 / ( x ! 2)2 ! c1

e x dx 8 ( x ! 2)11 x ( x ! 2)11 x 9 1 e 1 : / x ! 2 < ( 11) / ( 11) e ;= ! c1 , integrating by parts only the second integral

c1 ! e x / ≅ x ! 2 Α Putting this in (3), (D – 2)y " c1(x ! 2) ! ex ? / ≅ x ! 2Α

12 dx Its I.F. " e /

v " c1(x ! 2) ! ex.

so that

dy/dx – 2y " c1(x ! 2) ! ex

or

e 12 x and its solution is

ye–2x " c2 ! / [c1 ( x ! 2) ! e x ] e 12 x dx " c2 ! c1 / ( x ! 2) e 12 x dx ! / e 1 x dx 1 1 8 % 1 & %1 & 9 " c2 ! c1 :( x ! 2) ∋ 1 e 12 x ( ! / ∋ e 12 x ( dx ; 1 e 1 x " c2 1 c1 ( x ! 2) e 12 x 1 c1e 12 x 1 e1 x 2 4 2 4 ) ∗ ) ∗ =
o, the displacements of the mass become negligible over a long period of time. x Case I: Motion of an over damped system: Here we consider the 2 2 case in which b – µ > 0. In this situation the system is said to be over damped because the damping coefficent a is large when compared to the spring constant k. The corresponding solution of (1) is t O

x (t ) , e

−≅t

{C1 e

t ( b2 −3 2 )1/ 2

+ C2 e

− t ( b2 −3 2 )1/ 2

},

...(4)

Fig (i). Motion of an over damped system

which represents a smooth and nonoscillatory motion. Figure (i) shows two graphs of x(t). Here C1 and C2 are arbitrary constants. Case II: Motion of a critically damped system: Here we consider the case in which b2 – µ2 ! 0. In this situation the system is said to be critically damped system because slight

Created with Print2PDF. To remove this line, buy a license at: http://www.software602.com/

Applications of differential equations

11.9

decrease in the damping force would result in oscillatory motion. When b2 – µ2 ! 0, (3) 2 D ! – b, – b. The corresponding solution of (1) is x (t) ! e–bt (C1 + C2 t), C1, C2 being arbitrary constants ...(5) Some graphs of typical motion are given in figure (ii). Observe that the motion is quite similar to that of an overdamped system. It is also apparent from (5) that the mass can pass through the equilibrium position at most one time. Case III. Motion of an underdamped system: Here we consider the case in which b2 – µ2 < 0 so that µ2 – b2 > 0. Re-writting (3) we have D ! –b ± {–(µ2 – b2)}1/2 x

D ! – b ± i (µ2 – b2)½, where i ! –1 . In this situation the system is said to be underdamped because the damping coefficient a is small compared to the spring constant k. The corresponding solution of (1) is x (t) !e –bt {C1 cos t (µ2 – b2)½ + C2 sin t (µ2 – b2)½} ...(6) As shown in figure (iii), the motion described by (6) is oscillatory, but because of the coefficient e–bt, the amplitudes of vibration # 0 as t # ∃. or

x

O

t

Fig. (ii). Motion of a critically damped system

undamped

underdamped

t

O

Fig. (iii). Motion of an under damped system

11.9 Solved examples based on Art. 11.8 Example 1. An 8-pound weight stretches a spring 2 feet. Assuming that a damping force numerically equal to 2 times the instantaneous velocity acts on the system, determine the equation of motion if the weight is released from the equilibrium position with an upward velocity of 3 ft/sec. Sol. Using Hooke’s law we have 8 ! k × 2 so that k ! 4 lb/ft. Again W ! mg 2 8 ! m ×32 so that m ! 1/4 slug. Also, here damping factor ! 2. Using the above facts, the basic differential equation of the vibrations of the given mass on the spring for free damped motion (refer Art 11.8) namely m (d2x/dt2) + a (dx/dt) + kx ! 0 reduces to (1/4) × (d2x/dt2) + 2x(dx/dt) + 4x ! 0 or (D2 + 8D + 16)x ! 0 where D / d/dt ...(1) The initial condition are x(0) ! 0 and x' (0) ! –3 ...(2) 2 2 Its auxiliary equation is D + 8D + 16 ! 0 or (D + 4) ! 0 so that D ! – 4, – 4 (equal roots). Hence the system is critically damped (refer case II of Art. 11.8) and hence, we have x(t) ! (C1 + C2 t) e– 4t, C1, C2 being arbitrary constants....(3) x t=1/4 Differentiating (3) w.r.. ‘t’, we have t – 4t –4t x' (t) ! C2 e – 4 (C1 + C2t) e ...(4) –0.276 Applying the initial conditions (2) in (3) and (4), we find, in turn, that C1 ! 0 and C2 ! – 3. Hence from (3), we get maximum height above x (t) ! – 3t e– 4t ...(5) equilibrium position From (5), x' (t) ! – 3e – 4t (1 – 4t), showing that x' (t) ! 0 when Critically damped system t ! 1/4. The corresponding extreme displacement is given by x (1/4) ! – 3 × (1/4) × e – 1 ! – 0.276. As shown in the adjoining figure, we interpret this value to mean that the weight reaches the maximum height of 0.276 foot above the equilibrium position. Example 2. A 32–lb weight is attached to the lower end of a coil spring suspended from the ceiling. The weight comes to rest in its equilibrium position, there by stretching the spring 2ft. The weight in then pulled down 6 inches below its equilibrium position and released at t = 0. No external forces are present; but the resistance of the medium in pounds is numerically equal to

Created with Print2PDF. To remove this line, buy a license at: http://www.software602.com/

Applications of differential equations

11.10

8 (dx/dt), where dx/dt is the instantaneous velocity in feet per second. Determine the resulting motion of the weight on the spring. Sol. Here e ! the elongation of the spring after the weight is attached ! 2 feet. Using Hooke’s law, we have 32 ! k × 2 so that k ! 16 lb/ft. Again, W ! mg 2 32 ! m × 32 2 m ! 1 slug. Here damping factor ! a ! 8. Using these facts, the basic differential equation of the vibrations of the given mass on the spring for free damped motion (refer Art 11.8) namely m (d2x/dt2) + a (dx/dt) + k x ! 0 reduces to d2x/dt + 8 (dx/dt) + 16x ! 0 or (D2 + 8D + 16)x ! 0, D / d/dt ...(1) The initial conditions are : x(0) ! 6/12 ! 1/2 and x' (0) ! 0 ...(2) The auxiliary equation for (1) is D2 + 8D + 16 ! 0 or (D + 4)2 ! 0 giving D ! – 4, – 4: Hence the system is critically damped (refer case II or Art 11.8) and x(t) ! (C1 + C2t)e– 4t, C1 and C2 being arbitrary constants ...(3) From (3), x' (t) ! C2 e– 4t – 4(C1 + C2 t) e– 4t ! (C2 – 4C1 – 4C2 t)e – 4t ...(4) Applying the initial conditions (2) in (3) and (4), we find, in turn, C1 ! 1/2 and C2 –4 C1 ! 0 so that C1 ! 1/4 and C2 ! 2. Hence, from (3), we get x(t)!(1/2) × (1 + 4t)e– 4t ...(5) Interpretation: The motion is critically damped. Using (5), we have x ! 0 Α t ! – (1/4). It follows that x Β 0 for t > 0 and the weight does not pass through its equilibrium position. Also, from (5), x'(t) ! (2–2–8t) e–4t ! –8te–4t < 0 for all t > 0. Thus, the displacement of the weight from its equilibrum position is decreasing function of t for all t > 0. In other words the weight starts to move back towards its equilibrium position at once and x # 0 monotonically as t # ∃. The graph of the solution (5) is shown in the following figure. x

0.5

O

t 0.5

1.0

1.5

Example 3. A 16-pound weight is attached to a 5-foot long spring. At equilibrium the spring measures 8.2 feet. If the weight is pushed up and released from rest at a point 2 feet above the equilibrium position, find the displacement x(t) if it is further known that surrounding medium offers a resistance numerically equal to the instantaneous velocity. Sol. Here e ! the elongation of the spring after the weight is attached ! 8.2 – 5 ! 3.2 ft. Using Hooke’s law, we have 16 ! k × 3.2 so that k ! 5 lb/ft. Again, W ! mg 2 16 ! m × 32 so that m ! 1/2 slug. Also here damping factor ! a ! 2. Using the above facts the basic differential equation of the vibrations of the given mass on the spring for free damped motion (refer Art 11.8), namely m (d2x/dt) + a (dx/dt) + kx ! 0 reduces to (1/2) × (d2x/dt) + dx/dt + 5x ! 0 2 or (D + 2D + 10)x ! 0 ...(1) The initial conditions are: x(0) ! – 2 and x' (0) ! 0 ...(2) The auxiliary equation for (1) is D2 + 2D + 10 ! 0 so that D ! – 1 ± 3i, which then implies that the system is underdamped and x (t) ! e–t (C1 cos 3t + C2 sin 3t), C1 and C2 being arbitrary constants ...(3)

Created with Print2PDF. To remove this line, buy a license at: http://www.software602.com/

Applications of differential equations

11.11

From (3), x' (t) ! – e– t (C1 cos 3t + C2 sin 3t) + 3e– t (–C1 sin 3t + C2 cos 3t) ...(4) Applying the initial conditions x(0) ! – 2 and x' (0) ! 0 in (3) and (4), yield, in turn, C1 ! – 2 and C2 ! – (2/3). Hence from (3), we obtain x (t) ! – (2/3) × e– t (3 cos 3t + sin 3t) Example 4. A 32–lb weight is attached to the lower end of a coil spring suspended from the ceiling. The weight comes to rest in its equilibrium position, thereby stretching the spring 2 ft. The weight is then pulled down 6 inches below its equilibrium position and released at t = 0. No external forces are present; but the resistance of the medium is numerically equal to 4(dx/dt), where dx/dt is the instantaneous velocity in feet per second. Determine the resulting motion of the weight on the spring. Sol. Here e ! the elongation of the spring after the weight is attached ! 2 feet. Using Hooke’s law,we have 32 ! k × 2 so that k ! 16 lb/ft. Again, W ! mg 2 32 ! m × 32 so that m ! 1 slug. Here damping factor ! a ! 4. Using these facts, the basic differential equation of the vibrations of the given mass on the spring for free damped motion (refer Art 11.8), namely, m(d2x/dt2) + a (dx/dt) + kx ! 0 reduces to d2x/dt + 4 (dx/dt) + 16 ! 0 or (D2 + 4D + 16) x ! 0, D / d/dt ...(1) The initial conditions are: x(0) ! 6/12 ! 1/2, and x1 (0) ! 0 ...(2) The auxiliary equation for (1) is D + 4D + 6 ! 0 giving D!–2± 2 3 which then implies that the system is underdamped and x(t) ! e– 2t (C1 sin 2 3 t + C2 cos 2 3 t), C1 and C2 being arbitrary constants From (3), x'(t) ! – 2e

– 2t

or

x' (t) ! e

– 2t

...(3)

–2t

(C1 sin 2 3 t + C2 cos 2 3 t )+ 2 3 e (C1 cos 2 3 t – C2 sin 2 3 t)

{(– 2 C1 – 2 3 C2) sin 2 3 t + ( 2 3 C1 – C2) cos 2 3 t}

...(4)

Applying the initial conditions (2) to equation (3) and (4), we have C2 ! 1/2 and 2 3 C1 – 2C2 ! 0 so that C1 ! 3 / 6 and C2 ! 1/2. Hence from (3), the solution of (1) is given by x ! e– 2t {( 3 / 6 ) × sin 2 3t + (1/2) × cos 2 3t } ...(5) > 1 3 3 1 3 3 But sin 2 3t + cos 2 3t ! ( sin 2 3t + cos 2 3t ) ! cos( 2 3t – ) 6 2 2 6 3 3 2 – 2t (5) takes the form x ! 3 / 3 × e cos ( 2 3t – >/6) ...(6)

Χ



Interpretation: (6) represents a damped oscillatory motion. The damping factor is ( 3 / 3 ) e– 2t, the perod is (2>)/(2 3 ) ! ( 3 >)/3. The graph of the solution (6) is shown in the following figure, where the dashed curves represent the curves given by x ! ± ( 3 /3) × e– 2t. x

0.5 x = ( 3 /3) × e O

0.5

t

1.0 x = – ( 3/3) × e

–2t

1.5

2.0

–2t

–0.5

Created with Print2PDF. To remove this line, buy a license at: http://www.software602.com/

Applications of differential equations

11.12

Example 5. A 32-pound weight is attached to the lower end of a coil spring suspended from the ceiling. The weight comes to rest in its equilibrium position, thereby stretching the spring 2 feet. The weight is then pulled down 6 inches below its equilibrium position and released at t = 0. No external forces are present; but the resistance of the medium is numerically equal to 10 (dx/dt), where dx/dt is the instantaneous velocity in feet per second. Determine the resulting motion of the weight on the spring. Sol. Here e ! the elongation of the spring after the weight is attached ! 2 feet. Using Hooke’s law, we have 32 ! k × 2 so that k ! 16 lb/ft. Again, W ! mg 2 32 ! m ×32 2 m ! 1 slug. Here damping factor ! a ! 10. Using these facts, the basic differential equation of the vibrations of the given mass on the spring for free damped motion (refer Art 11.8) namely, m(d2x/dt2) + a (dx/dt) + k x ! 0 reduces to d2x/dt + 10(dx/dt) + 16x ! 0 or (D2 + 10 D + 16) x ! 0, D / d/dt...(1) The initial conditions are: x(0) ! 6/12 ! 1/2 and x' (0) ! 0...(2) 2 The auxiliary equaion for (1) as D + 10D + 16 ! 0 giving D ! – 2, – 8. Hence the system is over-damped (refer case I of Art 11.8). The general solution of (1) is x(t) ! C1 e– 2t + C2 e– 8t, C1 and C2 being arbitrary constants. From (3), x' (t) ! – 2 C1 e– 2t – 8 C2 e– 8t ...(4) Applying the initial conditions (2) to (3) and (4), we get C1 + C2 ! 1/2 and – 2C1 – 8C2 ! 0 so that C1 ! 2/3 and C2 ! – (1/6) Hence, from (3), the solution of the given problem is x ! (2/3) ! e– 2t – (1/6) ! e– 8t ...(5) Interpretation: Qualitatively the motion is the same as that of the solution (5) of Ex. 2. Here, however, due to the increased damping, the weight returns to its equilibrium position at a slower rate. The graph of (5) is shown in the following figure. x

0.5

O

0.5

1.0

t 1.5

11.10 Forced Motion In the present article, we propose to discuss an important special case of forced motion. That is, we not only consider the effect of damping upon the mass on the spring but also the effect upon it of a periodic external impressed force F defined by F(t) ! p cos Εt for all t Φ 0, where p and Ε are constants. Then the basic differential equation of forced motion is given by (refer equation (2) of Art 11.6). m(d2x/dt2) + a (dx/dt) + kx ! p cos Εt ...(1) 2 2 2 2 2 or d x/dt + 2b(dx/dt) + ≅ x ! E cos Εt. or (D + 2bD + ≅ )x ! E cos Εt. ...(2) where, 2b ! a/m, k/m ! ≅2, p/m ! E and D / d/dt ...(3)

Created with Print2PDF. To remove this line, buy a license at: http://www.software602.com/

Applications of differential equations

11.13

We shall assume that the positive damping constant a is small enough so that the damping is less than critical. In other words we assume that b < ≅. Now, the auxiliary equation for (2) is D2 + 2b D + ≅2 ! O so that D ! – b ± i (≅2 – b2)1/2 Complementary function of (2) ! C e –bt cos ((≅2 – b2)½ t + 4), where C and 4, are arbitrary constants. Again, as usual, P.I., (i.e.,) particular integral of (2) is given by 1 1 E cos Εt = E 2 cos Εt 2 D + 2b D+ ≅ −Ε + 2b D+ ≅ 2

P.I. !

2

;


/4 sec.

4. An 8–lb weight is attached to the lower end of a coil spring suspended from a fixed support. The weight comes to rest in its equilibrium position, thereby stretching the spring 6 inches. The weight is then pulled down 9 inches below its equilibrium position and released at t ! 0. The medium offers a resistance in pounds equal to 4(dx/dt), where dx/dt is the instantaneous velocity in feet per second. Determine the displacement of the weight as a function of the time. [Delhi B.Sc. II (Prog) 2009, 10] Ans. x ! (6t + 3/4)e– 8t 5. A 6–lb weight is attached to the lower end of a coil spring suspended from the ceiling, the spring constant being 27 lb/ft. The weight comes to rest in its equilibrium position, and beginning at t ! 0 an external force given by F(t) ! 12 cos 20 t is applied to the system. Determine the resulting displacement as a function of the time, assuming damping is negligible. [Delhi B.Sc. II (Prog) 2011] Ans. x ! (cos 12t – cos 20t)/4 6. A 10–lb weight is hung on the lower end of a coil spring suspended from the ceiling, the spring constant of the spring being 20 lb/ft. The weight comes to rest in its equlibrium position, and beginning at t ! 0 an external force given by F(t) ! 10 cos 8t is applied to the system. The medium offers a resistance in pounds numerically equal to 5(dx/dt), where dx/dt is the instantaneous velocity in feet per second. Find the displacement of the weight of the weight as a function of the time. Ans. x(t) ! – 2t e– 8t + (1/4) × sin 8t 7. A 6–lb weight is hung on the lower end of a coil spring suspended from the ceiling. The weight comes to rest in its equilibrium position, thereby stretching the spring 4 inches. Then beginning at t ! 0 an external force given by F(t) ! 27 sin 4t – 3cos 4t is applied to the system. If the medium offers a resistance in pounds numerically equal to three times the instantaneous velocity, measured in feet per second, find the displacement as a function of the time. Ans. x(t) ! (1/2) × e– 8t ( 2 sin 4 2 t + 2 cos 4 2 t) + sin4t – cos 4t 8. A 12–lb weight is attached to the lower end of a coil spring suspended from the ceiling. The weight comes to rest in its equilibrium position thereby stretching the spring 6 inches. Begninning at t ! 0 an external force gives by F(t) ! 2cos Εt is applied to the system. (i) If the damping force in pounds is numerically equal to 3(dx/dt), where dx/dt is the instantaneous velocity in feet per second, determine the resonance frequency of the resulting motion and find the displacement as a function of the time when the forcing function is in resonance with the system.(ii) Assuming there is no damping, determine the value of Ε which gives rise to undamped resonance and find the displacement as a function of the time in this case. Ans. (i) (2 2 )/>; x(t) ! – (1/18) × e– 4t ( 3 sin 4 3 t + cos 4 3 t) + (1/18) × ( 2 sin 4 2 t + cos 4 2 t) (ii) 8; x(t) ! (t/3) × sin 8t 9. The differential equation for the motion of a unit mass on a certain coil spring under the action of an external force of the form F(t) ! 30 cos Εt is (d2x/dt2) + a (dx/dt) + 24x ! 30 cos Εt, where a Φ 0 is the damping coefficient (i) If a ! 4, find the resonance frequency and determine the amplitude of the steady–state vibration when the forcing function is in resonance with the system. (b) Proceed as in part (i) if a ! 2.

Ans. (i) 2/>; 3 5 /4 (ii)

22 / 2> ; 15 23 / 23

Created with Print2PDF. To remove this line, buy a license at: http://www.software602.com/

Applications of differential equations

11.20

11.12 Electric Circuit Problems In this article we propose to study the application of differential equations to series circuits containing (1) an electromotive force, and (2) resistors,inductors, and capacitors. In what follows, the following conventional symbols will be used. L

E R

R

E

L

C

C Figure showing LRC-series circuit

Some useful results related to series circuits: Electromotive force (for example, a battery or generator) produces a flow of current in a closed circuit and that this current produces a so called voltage drop across each resistor, inductor and capacitor. See the following table for symbols and unis. Quantity and symbol emf or voltage E current i charge q resistance R inductance L capacitance C

Unit volt (V) ampere coulomb ohm (Ο) henry (H) farad

Recall the following three laws concerning the voltage drops across resistor, inductor and capacitor: Law I: The voltage drop ER acorss a resisor is given by ER ! R i, ...(1) where R is a constant of proportionally called the resistance, and i the current. Law II: The voltage drop EL across an inductor is given by EL ! L(di/dt), ...(2) where L is a constant of proportionality called the inductane. Law III: The voltage drop EC across a capacitor is given EC ! q/C, ...(3) where C is a constant of proportionality called the capacitance and q is instantaneous charge on the capacitor. The fundamental law in the study of electric circuits is the following: Kirchhoff’ Voltage Law: The sum of the voltage drops across resistor, inductors, and capacitors is equal to the total electromotive force in a closed circuit. Let us apply Kirchhoff’s law to the circuit of figure. Let E denote the electromotive force. Then, using the above mentioned laws 1, 2 and 3 for voltage drops, we obtain L (di/dt) + R i + q/C ! E ...(4) containing two dependent variables i and q. But, we also have i ! dq/dt so that di/dt ! d2q/dt2 ...(5) 2 2 Using (5), (4) takes the form L (d q/dt ) + R (dq/dt) + q/C ! E, ...(6) which is a second – order linear differential equation in the single dependent variable q. So we can obtain q from (6).

Created with Print2PDF. To remove this line, buy a license at: http://www.software602.com/

Applications of differential equations

11.21

Now, differentiating (4) w.r.t. ‘t’, gives L (d2i/dt2) + R (di/dt) + (1/C) × (dq/dt) ! dE/dt or L (d2i/dt2) + R (di/dt) + (1/C) × i ! dE/dt, using (5) ...(7) which is a second order linear differential equation in the single dependent variable i. So we can obtain i from (7). Particular cases: We now consider two very simple cases in which the problem reduces to a first order linear differential equation. Case I: If the circuit contains no capacitor (so that C ! 0), then (4) reduces to L (di/dt) + R i ! E ...(8) Case II: If the circuit contains no inductor (so that L ! 0), then (6) reduces to R (dq/dt) + q/C ! E ...(9) Electro-mechanical analogy: Observe that the differential equation (6) for the charge is exactly the same as the differential equation (2) of Art. 11.6 for the vibrations of a mass on a coil spring, except for the notations used. That is, the electrical system described by (6) is analogous to the mechanical system described by equation (2) of Art. 11.6. Since electrical circuits are easy to assemble and the currents and voltages are accurately measured very easily, this affords a practical method of studying the oscillations of complicated mechanical systems which are expensive to make and unwieldy to handle by cosidering an equivalent electric circuit. While making an electric equivalent of a mechanical system, the correspondence between the elements shown in the following table should be kept in mind. Mechanical system

Electric system

mass m

inductance L

damping constant a

Resistance R

spring constant k

reciprocal of capacitance ! 1/C

impressed force F(t)

impressed voltage or emf E

displacement x

charge q

velocity ! v ! dx/dt

current ! i ! dq/dt

11.13 Solved examples based on Art. 11.2 Ex.1. A circuit has in series an electromotive force given by E ! 100 sin 40t V, a resistor of 10 Ο and an inductor of 0.5 H. If the initial current in 0, find the current at time t > 0. Sol. For the given problem the circuit diagram is R shown in the adjoining figure. Let i denote the current in amperes at time t. The total electromotive force is E 100 sin 40 t. Then, as usual (refer laws 1 and 2 of Art L 11.12), we have The voltage drop across the resistor ! ER ! Ri ! 10i and the voltage drop across the inductor ! EL ! L (di/dt) ! (1/2) × (di/dt) Applying Kirchhoff’s law, we have (1/2) × (di/dt) + 10i ! 100 sin 40t or di/dt + 20i ! 200 sin 40t, which is first order linear equation ...(1) Since the initial current is 0, the initial condition is i (0) ! 0 ...(2)

Created with Print2PDF. To remove this line, buy a license at: http://www.software602.com/

Applications of differential equations

11.22

20dt Integrating factor of (1) ! e . ! e20t and hence its solution is

i e20t !

.

{(200 sin 40 t) × e20t } dt + C ! 200 . e20t sin 40t dt + C

.e

Since, from Integral Calculus, hence,

20t . e sin 40t dt !

ax

...(3)

sin bx dx ! {eax (a sin bx – b cos bx)}/(a2 + b2),

e 20t Χ 20sin 40t – 40cos 40t ∆

Χ 20 ∆2 + Χ 40 ∆2

!

e 20t Χ sin 40t – 2 cos 40t ∆ 100

i e20t ! 2 e20t (sin 40t – 2cos 40t) + C i ! 2 (sin 40t – 2cos 40t) + C e–20t Applying the condition (2), i ! 0 when t ! 0, (4) gives C ! 4. Hence (4) becames i ! 2 (sin 40 – 2 cos 40t) + 4 e– 20t We transform (5) in a “phase – angle” form as follows: (3) reduces to

or

sin 40t – 2 cos 40t ! 5

;Χ1/ 5 ∆ sin 40t – Χ 2 / 5 ∆ cos 40t< !

cos 4 ! 1/ 5

where

sin 4 ! 2 / 5

and

From (7), 4 Ν – 1.11 radians Hence,

5 sin (40t + 4)

sin 40t – 2 cos40t !

...(4) ...(5) ...(6) ...(7)

5 sin (40t – 1.11)

i ! 2 5 sin (40t – 1.11) + 4e– 20t or i ! 4.47 sin (40t – 1.11) + 4e– 20t ...(8) Interpretation. The current is presented as the sum of a sinusodial term and an exponential. The exponential becomes so very small in a short time that its effect is soon practically negligible; it is the transient term. Thus, after a short time, essentially all that remains is the sinusodial term; it is the steady current. Observe that its period >/20 is the same as that of the electromotive force. However, the phase angle 4 Ν – 1.11 radians indicates that the electromotive force leads the steady–state current by approximately (1/46) × 1.11. The grah of the current as a function of time is shown in the following figure. (5) transforms to

i 5

0

0.1

0.2

0.3

0.4

0.5

0.6

0.7

t

–5

Ex. 2. A circuit has in series an electromotive force given by E = 100 sin 60 t V, a resistor of 2Ο, an inductor of 0.1H, and a capacior of 1/260 farads. If the initial current and the initial charge on the capacitor are both zero, find the charge on the capacitor at any time t > 0. Sol. The circuit diagram is shown in R = 20 hns the adjoining figure. We have L ! 1/10 heneries, C ! 1/260 farads, R ! 2 ohms and E ! 100 sin 60t V. Let q denote the 1 C= farads E instantaneous charge on the capacitor. 260 1 Then q is given in terms of L, C, R and E L= heneries 10 by the following second–order linear differential equation:

Created with Print2PDF. To remove this line, buy a license at: http://www.software602.com/

Applications of differential equations

or

11.23

L (d2q/dt2) + R (dq/dt) + q/C ! E or (1/10) × (d2q/dt2) + 2 (dq/dt) + 260 q!100 sin 60 t (D2 + 20 D + 2600)q ! 1000 sin 60t, where D / d/dt ...(1) Since the charge q is initially zero, we have first initial condition: q(0) ! 0 ...(2) Since the current i is also initially zero and i ! dq/dt ! q1 (t), we have Second initial condition: q1(0) ! 0 ...(3) 2 The auxiliary equation for (1) is D + 20 D + 2600 ! 0 giving D ! – 10 ± 50 i. qc ! C.F. of (1) ! e–10t (C1 sin 50 t + C2 cos 50 t), C1,C2 being arbitrary constants qp ! P.I. !

and

1 2

D + 20D + 2600

! 50

1000sin 60t ! 1000

1 2

–(60) + 20 D + 2600

sin 60t

1 1 sin 60t sin 60t ! 50 (D + 50) 2 D – 50 D – (50)2

! 50 (D + 50)

1 2

–(60) – (50)

2

sin 60 t ! –

1 (60 cos 60t + 50 sin 60t ) (2 ! 61)

! – (25/61) × sin 60t – (30/61) × cos 60t Hence the general solution of (1) is q ! qc + qp, that is, q(t) ! e– 10t (C1 sin 50t + C2 cos 50t) – (25/61) × sin 60t – (30/61) × cos 60t ...(4) Differentiating (4) w.r.t. ‘t’ and simplifying, we obtain q1 (t) ! e– 10t {(–10C1 – 50C2} sin 50t + (50C1 – 10C2) cos 50t} – (500/61) × sin 60t + (1800/61) × sin 60t ...(5) Applying condition (2) to equation (4) and condition (3) to equation (5), we get C2 – (30/61) ! 0 and 50 C1 – 10 C2 – (1500/61) ! 0 giving C1! 36/61, C2!30/61 Substituting these values in (4), the required solution is q ! (6/61) × e– 10t (6 sin 50t + 5cos 50t) – (5/61) × (5 sin 60t + 6 cos 60t) ...(6) We shall now re-write (6) in a “phase-angle” form. We have 6 sin 50t + 5 cos 50t !

;


0. Ans. q ! (1 – e– 500t)/50; i ! 10 e– 500t 3. A circuit has in series an electromotive force given by E(t) ! 100 sin 200t V, a resistor of 40Ο, an inductor of 0.25 H, and a capacitor of 4 × 10– 4 farads. If the inital current is zero, and he initial charge on the capacitor is 0.01 coulombs, find the current at any time t > 0. Ans. i ! e– 80t (– 4.588 sin 60t + 1.247 cos 60t) – 1.247 cos 200t + 1.331 sin 200t 4. A circuit has in series a resistor R Ο, an inductor L H, and a capacitor of C farads. The initial current is zero and the initial charge on the capacitor in Q0 coulombs. (a) Show that the charge and the current are damped oscillatory functions of time if and only if R < 2 (L/C)1/2, and find the expressions for the charge and the current in this case. (b) If R Φ 2 (L/C)1/2, discuss the nature of the charge and current as functions of time.

Created with Print2PDF. To remove this line, buy a license at: http://www.software602.com/

Applications of differential equations Ans. (a) i ! –

11.25

% (4 L – R 2 C )1/ 2 e – Rt / 2 L sin ∋ ∋ (4LC – R 2 C 2 )1/ 2 2L C ) 2Q0

& t( ; ( ∗

Γ Q R C % (4 L – R 2 C )1/ 2 & % (4 L – R 2 C )1/ 2 & Η – Rt / 2 L 0 sin ∋ t ( + Q0 cos ∋ t (ϑ Ι q! e 2 1/ 2 ∋ ( ∋ (ϑ 2L C 2L C ) ∗ ) ∗Λ ΚΙ (4 L – R C )

Part III. Applications to Simultaneous Differential Equations 11.14 Applications to Mechanics System of linear differential equations originate in the mathematical formulation of various problems in mechanics. In the next article we shall discuss such problems in details. For details of method of solution refer Chapter 8. 11.15 Solved example based on Art. 11.14 Ex.1. On a smooth horizontal plane BC an object A1 is connected to a fixed point P by a massless spring S1 of natural length L1. An object A2 is then connected to A1 by a massless spring S2 of natural length L2 in such a way that the fixed point P and the centres of gravity A1 and A2 all lie in a straight line (refer fig. (i)) The object A1 is then displaced a distance a1 to the right or left of its equilibrium position O1, the object A2 is displaced a distance a2 to the right or left of its equilibrium position O2 and at time t = 0 the two objects are released (see fig. (ii)). What are the positions of the two objects at any time t > 0. Sol. Let m1 and m2 be masses of objects A 1 and A2 respectively Also, assume that spring constants of springs S1 and S2 be k1 and k2 respectively. P

A1

S1

A2

S2

B

C L1

P

L2

O1

A1

S1

O2

Figure (i)

A2

S2

B

C a1 O1

P

a2 O2

Figure (ii) A1

S1

A2

S2

B

C x1 O1

Figure (iii)

x2 O2

Let x1 denote the displacement of A1 from its equilibrium position at time t Φ 0 and assume that x1 is positive when A1 is to the right of O1. Similarly, let x2 denote the displacement of A2 from its equilibrium position O2 at time t Φ 0 and assume that x2 is positive when A2 is to the right of O2 (see figure (iii)).

Created with Print2PDF. To remove this line, buy a license at: http://www.software602.com/

Applications of differential equations

11.26

The forces acting on A1 at time t > 0 are: (i) Force F1 exerted by the spring S1 (ii) Force F2 exerted by spring S2. By Hooke’s law, the force F1 is of magnitude k 1|x 1|. Since this force is exterted toward the left when A1 is to the right of O1 and toward the right when A1 is to the left of O1, we have F1 ! – k1x1. Again using Hooke’s law, the force F2 is of magnitude k2e, where e is the elongation of S2 at time t. But e ! |x2 – x1| and hence magnitude of F2 is k2 |x2 – x1|. Again, since this force is exerted toward the left when x2 – x1 < 0 and toward the right when x2 – x1 > 0, we have F2 ! k2 (x2 – x1). Applying Newton’s second law to the object A1, we have 2 m1(d x/dt2) ! – kx1 + k2 (x2 – x1) or (m1D2 + k1 + k2) x1 – k2 x2! 0, ...(1) where D / d/dt. The object A2 is acted upon by only one force F3 which is exerted by spring S2. By Hooke’s law, the magnitude of F3 is k 2|x2 – x 1|. Since F3 is exerted toward the left when x2 – x1 > 0 and toward the right when x2 – x1 < 0, we have F3 ! – k2(x2 – x1). Applying Newton’s second law to the object A2, we have m2 (d2x2/dt2) ! – k2(x2 – x1) or (m2D2 + k2) x2 – k2 x1!0 ...(2) From the statement of the problem, the inditial conditions are: x1(0) ! a1, x' (0) ! 0, x2(0) ! a2, and x12 (0) ! 0 ...(3) Solution of a specific case: Suppose the two objects A1 and A2 are each of unit mass, so that m1 ! m2 ! 1. Also, suppose that the springs S1 and S2 have spring constants k1 ! 3 an k 2 ! 2, repectively. Further, we take a1 ! –1 and a2 ! 2. Then, (1), (2) and (3) take the following forms: (D2 + 5)x1 – 2x2 ! 0 ...(4) – 2x1 + (D2 + 2) x2 ! 0 ...(5) x1(0) ! – 1,

x11 (0) ! 0,

x2(0) ! 2

x'2(0) ! 0

and

...(6)

2

Operating both sides of (4) by (D + 2) and multiplying (5) by 2 and then adding the resulting equations, we find {(D2 + 2) (D2 + 5) – 4}x1 ! 0 or (D4 + 7D2 + 6)x1 !0 ...(7) 4 2 The auxiliary equation for (7) is D + 7D + 6 ! 0 or (D2 + 6) (D2 + 1) ! 0 so that D ! ± i 6 , ± i and hence the general solution of (7) is x1 ! C1 sin t + C2 cos t + C3 sin t 6 + C4 cos t 6 , C1, C2, C3, C4 being arbitrary contants ...(8) (8) 2 Dx1 ! C1 cost – C2 sin t + C3 6 cos t 6 – C4 6 sin t 6

...(9)

(9) 2 D2x1 ! – C1 sin t – C2 cos t – 6 C3 sin t 6 – 6C4 cos t 6

...(10)

Now, (4) 2 2x2 ! D2x1 + 5x1 ! – C1 sin t – C2 cos t – 6 C3 sin t 6 – 6 C4 cos t 6 + 5(C1 sin t + C2 cos t + C3 sin t 6 + C4 cos t 6 ), using (8) and (10) or

x2 ! 2C1 sin t + 2C2 cos t – (1/2) × C3 sin t 6 – (1/2) × C4 cos t 6

...(11)

From (6), we have x1 ! –1 and x11 ! dx1/dt ! Dx1 ! 0 when t ! 0. Hence (8) and (9) give – 1 ! C2 + C4 From (11), x'2 ! dx2/dt ! 2C1 cos t – 2C2 sin t –

and

Χ



0 ! C1 + C3 6

6 / 2 × C3 cos t 6 +

Χ

...(12)



6 / 2 × C4 sin t 6 ...(13)

From (6), we have x2 ! 2 and x12 ! dx2/dt ! 0 when t ! 0. Hence (11) and (13) give 2 ! 2C2 – (1/2) + C4

and

0 ! 2C1 –

Χ



6 / 2 ! C3

...(14)

Created with Print2PDF. To remove this line, buy a license at: http://www.software602.com/

Applications of differential equations

11.27

Solving (12) and (14), C1 ! 0, C2 ! 3/5, C3 ! 0 and C4 ! –(8/5). Substituting the above values in (8) and (11), the particular solution of the specific problem consisting of the system of equations (4) and (5) and initial conditions (6) is given by x1(t) ! (3/5) × cos t – (8/5) ! cos t 6 and x2 (t) ! (6/5) × cos t + (4/5) × cos t 6 Ex 2. Solve the problem of Ex. 1. for the case in which the objects A 1 has mass m1 = 2, the object A2 has mass m2 = 1, the spring S1, has spring constant k1 = 4, the spring S2 has s2 has spring constant k2 = 2, and the initial conditions are x1(0) = 1, x11 (0) = 0, x2(0) = 5 and x12 (0) = 0.

Ans. x1 ! 2 cos t – cos 2t, x2 ! 4 cos t + cos 2t

Ex. 3. A projectile of mass m is fired into the air from a gun which is inclimed at an angle Μ with the horizontal, and suppose the initial velocity of the projectile is v0 feet per second. Neglect all forces except that of gravity and the air resistance, and assume that this latter force (in pounds) is numerically equal to k times the velocity (in feet/second). (i) Taking the origin at the position of the gun, with x-axis horizontal and the y-axis vertical, show that the differential equations of the resulting motion are m (d2x/dt2) + k (dx/dt) ! 0 and m(d2y/dt2) + k (dx/dt) + mg ! 0. (ii) Find the solution of the system of differential equation of part (i). MISCELLANEOUS EXAMPLES ON CHATPER 11 Ex. 1. When a switch is closed in circuit containing a battery E, a resistor R and an inductance L, the current i builds up at a rate given by L (di/dt) + Ri = E. Find i as a function of t. [M.S. Univ. T.N. 2007] Sol. Re-writing the given equation, di/dt + (R/L) × i ! E/L ... (1) ( R / L ) dt , e Rt / L Integrating factor of linear equation (1) ! e .

ieRt/L !

Solution of (1) is

. ( E / L) e

Rt / L

dt + c , c being an arbitrary constant

i e ! (E/L) × (L/R) e + c or i ! E/R + ce–(Rt/L) ... (2) Initially, at t ! 0, i ! 0. So (2) givens 0 ! E/R + c so that c ! – E/R –Rt/L Hence, (2) reduces to i ! (E/R) × (1 – e ) ... (3) Ex. 2. A 12 volt battery is connected to a simple series circuit in which the inductance is (1/2) H and the resistance is 10 Ο. Determine the current i if i(0) = 0. [M.S.Univ. T.N. 2007] Sol. If a circuit has in series an electromotive force E volt, a resistor R ohm and an inductor L heneries, then current i in amperes at time t is given by L (di/dt) + Ri ! E ... (1) Here L ! (1/2) H, R ! 10Ο and E ! 12 volt, So (1) reduces to (1/2) × (di/dt) + 10i ! 12 or di/dt + 20i ! 24 ... (2) 20 dt 20 t . , e and solution is given by Its I.F. ! e or

Rt/L

i e20i !

Rt/L

. (24 e

20 t

) dt + c

or

i e20t ! (6/5) × e20t + c

i ! (6/5) + c e–20t, c being an arbitrary constant ... (3) Since i (0) ! 0, putting i ! 0 and t ! 0 in (3), we get c ! – 6/5 Hence, (3) reduces to i ! (6/5) × (1 – e–20t) Ex. 3. 16 lb weight is placed upon the lower end of a coil spring suspended from the ceiling and comes to rest in the equilibrium position, thereby stretching the spring 8 in At time t = 0 the weight is then struck so as to set it into motion with initial velocity of 2ft/sec directed downward. The mediam ofters a resistance in pounds numerically equal to 6 (dx/dt), where dx/dt is the or

Created with Print2PDF. To remove this line, buy a license at: http://www.software602.com/

11.28

Applications of differential equations

instantaneous velocity in feet per second. Determine the resulting displacement of the weight as a function of time. [Delhi B.Sc. II (Prog.) 2008] 4. For an electric circuit with circuit constants L, R, C the charge q on a plate of condenser is given by L (d 2 q / dt 2 ) + R (dq / dt ) + q / c , 100 . Given L = 1, R = 1200, C = 10–6, q = dq/dt = 0 for t > 0, find the charge q. [Madurai Kamraj 2008] 5. A current of electricity on a circuit of resistance R ohms commences at time t = 0. The self induction of the circuit is L and when t = 0, the electromotive force is E. The current satisfies the equation L (di / dt ) + Ri , E . Solve the equation and show that i , ( E / R) ! (1 − e− Rt / 2 ) [Madurai Kamraj 2008]

Created with Print2PDF. To remove this line, buy a license at: http://www.software602.com/

MISCELLANEOUS PROBLEMS BASED ON THIS PART OF THE BOOK Ex. 1. What is the general solution of 2x (dy/dx) = 10x3y5 + y ? (a) y4 ! cx2 – 4x3 (b) y – 4 = c/x2 + 4x3 4 2 3 (c) y = c/x + 4x (d) y – 4 = c/x2 – 4x3 Sol. Ans. (d). Re-writing, the given equation reduces to

[I.A.S. Prel. 2008]

dy 1 !4 dy y !4 y !5 ! y 5 x 2 …(1) 5x2 # or dx 2 x dx 2x Let y – 4 ! v so that – (4y–5) × (dy/dx) ! d v /dx. Then, (1) reduces to – (1/4) × (d v /dx) – (1/2x) × v ! 5x2 or d v /dx + (2/x) × v ! – 20x2, …(2)

y !5

whose I.F. ! e

vx 2 or

or

∃ (2/x)dx

e 2log x

x 2 and hence its solution is

∃ %(!20x ) ∋ x & dx # c 2

2

– (2/y)dy whose I.F. ! e ∃

v y–2 !

or or

or

!4 x5 # c

y–4 ! c/x2 – 4x3, c being an arbitrary constant. Ex. 2. Solve the differential equation ydx + (x + x3y2)dy = 0 [I.A.S. 2008] Sol. Re-writing the given equation, y (dx/dy) + x + x3 y2 ! 0 dx/dy + (1/y) × x ! – x3y or x–3 (dx/dy) + (1/y) × x–2 ! – y ...(1) –2 –3 Let x ! v so that – (2x ) × (dx/dy) ! d v /dy.. Then, (1) reduces to

– (1/ 2) ∋ (d v / dy ) # v /y = – y

or

y !4 x2

or

∃ (2y × y

-2

e!2log y

or

dv/dy – (2/y) × v ! 2y, …(2)

y !2 and hence its solution is

)dy # c

or

x!2 y !2

2log y ! 2log c 2 2

log (y/c) ! 1/(2x2y2) or y = c e(1/ 2 x y ) 2 3 Ex. 3. Solve (1 + x + xy )dy + (y + y )dx = 0 [Delhi 2008] 2 2 Sol. Re-writing the given equation, y(1 + y )dx + {1 + x (1 + y )}dy ! 0 (1 + y2) (ydx + xdy) + dy ! 0 or d(xy) + {1/(1+y2)}dy ! 0 2 Integrating, xy + log (1 + y ) – log c ! 0 or log {(1+y2)/c} ! – xy 2 –xy 1 + y ! c e , c being an arbitrary constant. Ex. 4. What is the solution of the equation x(dy/dx) + y2/x = y? Here ln x ! logex (a) ln(y/x) – (1/x) = C (b) ln x – (x/y) = C (c) ln(x/y) – (1/x) = C (d) ln x + (x/y) = C [I.A.S. (Prel.) 2009] 2 Sol. Ans. (b). Given x(dy/dx) + y /x ! y or dy/dx – (1/x)y ! – (y2/x2) –2 –1 2 y (dy/dx) – (1/x)y ! – (1/x ) …(1) Putting y–1 ! v so that – y–2 (dy/dx) ! d v /dx, (1) reduces to – (d v /dx) – (1/x) v ! – (1/x2) or d v /dx + (1/x) v ! 1/x2, …(2)

which is a linear differential equation whose I.F. ! e ∃(1/x)dx ! e logx ! x and hence solution of (2) is (1/ x 2 ) ∋ x dx # c ( log e x # c ( ln x # c( , c( being an arbitrary constant. vx !

∃%

&

x/y ! ln x # c ( where c ! ( ! c() is an arbitrary constant. or

or

ln x

( x / y)

c,

M.1

Created with Print2PDF. To remove this line, buy a license at: http://www.software602.com/

M.2

Miscellaneous problems based on this part of the book

Ex. 5. Which one of the following differential equations represents the orthogonal trajectories of the family of curves xy = k2? Sol. Ans. (c). Given xy ! k2, where k is a parameter … (1) Differentiating (1) w.r.t. ‘x’, y + x (dy/dx) ! 0, … (2) which is the differential equation of the given family of curves (1). Replacing dy/dx by – (dx/dy), the differential of the required orthogonal trajectories is given by y – x (dx/dy) ! 0 or xdx – ydy ! 0 Ex. 6. Solve (D3 + D)y = 2x2 + 4 sin x [Delhi 2008] Sol. The auxiliary equation of the given equation is given by D3 + D ! 0,

D(D2 + 1) ! 0

i.e.,

D ! 0,

giving

)i

Hence C.F. ! C1eo. x # C2 cos x # C3 sin x , C1, C2 and C3 being arbitrary constants Here, P.I. corresponding to 2x2 !

1 2

D ( D # 1)

2 x2

2 2 2 x3 ( x ! 2) ! ! 4x D 3

2 2 (1 # D 2 )!1 x 2 ! (1 ! D 2 # D 4 ∗) x 2 D D

and P.I. corresponding to 4 sin x !

1 3

D #D

4 sin x

4

1

1 sin x D #1 D

4

2

x sin x !2 x sin x, 2 ∋1 Hence the required solution is

! ! 4∋

as

1 2

D #1

( ! cos x)

1 2

2

!4

1 2

D #1

cos ax

cos x

x sin ax 2a

D #a y ! C.F. + total P.I., i.e.,

y ! C1 + C2 cos x + C3 sin x + (2 x3 / 3) – 4x – 2x sin x Ex. 7. Solve the differential equation (d 2 y / dx 2 ) ! 9 y undetermined coefficients.

e2 x # x by the method of [Delhi 2008]

Ans. y ! c1e3x # c2 e!3x ! (e2 x / 5) ! ( x / 9)

Hint. Do like Ex 2, page.5.54.

Ex. 8. One particular solution of y111 – y11 – y1 + y = – ex is a constant multiple of (a) xe–x (b) xex (c) x2e–x (d) x2ex [GATE 2008] Sol. Ans. (d). Let D ! d/dx. Then, the given equation reduces to (D3 – D2 – D + 1)y ! – ex or (D – 1)2 (D + 1) y ! – ex P.I. !

1

1 (! e x ) D # 1 ( D ! 1) 2

+ 1 x, −! e . ( D ! 1) / 2 0 1

2

!

1 1 ex 2 ( D –1) 2

1 x2 ! ∋ ∋ ex 2 2!

!

x2 e x 4

which is a constant multiple of x 2 e x. . Ex. 9. Solve (x2D2 – xD + 1)y = (log x sin log x + 1)/x [Madurai Kamraj 2008] Sol. Try yourself as in Ex. 17, page 6.10 Ans. y ! x(c1 + c2 log x) + (1/4x) + {(log x) × (3 log sin x + 4 log cos x)}/25x + (110 cos log x + 20 sin log x)/625x Ex. 10. Solve the differential equation (D2 – 2D + 1)y = x log x, (x > 0) by using the method of variation of parameters. [Delhi 2008] Sol. Re-writing the given equation, y2 – 2y1 + y ! x log x … (1) Comparing (1) with y2 + Py1 + Qy ! R, here R ! x log x

Created with Print2PDF. To remove this line, buy a license at: http://www.software602.com/

Miscellaneous problems based on this part of the book

M.3

2

Consider y2 – 2y1 + y ! 0 or (D – 2D + 1)y ! 0, where D 1 d/dx The auxiliary equation of (2) is D2 – 2D + 1 ! 0 giving x Hence, C.F. of (1) ! (c1 + c2x)e , c1 and c2 being arbitrary constants u ! ex

Let

W!

Now, Then, f (x) ! –

v = x e x. Also, here

and

vR

∃ W dx !∃

ex

u v

R ! x log x

xe x

e x (e x # xe x ) ! xe 2 x u1 v1 e x e x # xe x P.I of (1) ! u f (x) + v g (x), where x

( xe ) ( x log x) e2 x

%

… (2) D ! 1, 1. … (3) … (4)

e2 x 2 0 … (5)



! e ! x ( x 2 log x)dx

dx

&



! ! ( x 2 log x ) ( ! e ! x ) ! (2 x log x # x ) ( ! e ! x ) dx , integrating by parts



!x 2 !x ! e x log x ! e (2 x log x # x )dx

%

&



! e ! x x 2 log x ! (2 x log x # x) ( !e ! x ) ! (2 log x # 2 # 1) ( ! e ! x ) dx



!x 2 !x !x ! e x log x # e (2 x log x # x) ! e (2 log x # 3)dx

%

&



!x 2 !x !x ! e ( x log x # 2 x log x # x ) ! (2 log x # 3)( !e ) ! (2 / x ) ( ! e ) dx



! e ! x ( x2 log x # 2 x log x # x # 2 log x # 3) ! 2 (e! x / x )dx g(x) !



uR dx W



e x ∋ ( x log x) e2 x

dx

∃e

!x

… (6)

( x log x )dx





!x !x ! ( x log x) (!e ) ! (log x # 1) (!e )dx ! ! xe ! x log x # e ! x (log x # 1)dx

! ! xe! x log x # (log x # 1) (!e! x ) !

∃ %(1/ x) ∋ (!e )&dx, integrating by parts !x



! !e ! x ( x log x # log x # 1) # (e ! x / x)dx Using (4), (6) and (7), (5) yields

… (7)

%

&



x !x 2 !x P.I. of (1) ! e e ( x log x # 2 x log x # x # 2 log x # 3) ! 2 ( e / x ) dx

%

&



+ x e x !e ! x ( x log x # log x # 1) # (e ! x / x )dx



2 x !x ! x log x # 2 x log x # x # 2log x # 3 ! 2e (e / x)dx



! x 2 log x ! x log x ! x # xe x (e ! x / x)dx



x !x ! x log x # 2 log x # 3 ! (2 ! x)e ( e / x)dx

Hence the required solution is

y ! C.F. + P.I.,

i.e.,



x x !x y ! (c1 # c2 x)e # x log x # 2 log x # 3 ! (2 ! x)e (e / x)dx

Created with Print2PDF. To remove this line, buy a license at: http://www.software602.com/

M.4

Miscellaneous problems based on this part of the book

Ex. 11. Use the method of variation of parameters to find the general solution of x2y(( – 4xy( + 6y = – x4 sin x. [I.A.S. 2008] 2 2 Sol. Re-writing the given equation, y2 – (4/x) × y1 + (6/x ) × y ! – x sin x. …(1) Comparing (1) with y2 + Py1 + Qy ! R, here R ! – x2 sin x Consider y2 – (4/x) × y1 + (6/x2) × y ! 0 or (x2D2 – 4xD + 6)y ! 0, D 1 d/dx …(2) In order to apply the method of variation of parameters, we shall reduce (2) into linear differential equation with constant coefficients. Let x ! ez, i.e., log x ! z and let D1 1 d/dz …(3) 2 2 Then, xD ! D1, x D ! D1 (D1 – 1) and so (2) reduces to {D1 (D1 – 1) – 4D1 + 6}y ! 0

( D12 ! 5 D1 # 6) y

or

D12 ! 5D1 # 6

whose auxiliary equation is 2z

3z

D1 ! 2, 3.

giving

0

z 2

z 3

2

3 C.F. of (1) ! c1 e + c2 e ! c1 (e ) + c2 (e ) ! c1x + c2x Let u ! x2

and

v = x 3 . Also,

u u1

x2

x3

3x 4 ! 2 x 4

W

Hence,

P.I. of (1) ! u f (x) + v g (x), where

2 x 3x

g (x) !

2

x 3 ∋ (! x 2 sin x)

vR

∃ W dx !∃

! x(! cos x) ! and



… (4) 2 R ! –x sin x

Here

f (x) ! !

3

here

v v1

uR dx W



0,

x4

x4 2 0

…(6)

x4

∃ x sin x dx

dx

∃ %1∋ (! cos x)& dx x 2 ∋ (! x 2 sin x )

…(5)

! x cos x # sin x



! sin x dx

dx

cos x

…(7) …(8)

Using (5), (7) and (8), (6) reduces to P.I. of (1) ! x2 (– x cos x + sin x) + x3 cos x ! x2 sin x Hence the required general solution is y ! C.F. + P.I., i.e., 2 3 2 y ! c1x + c2x + x sin x, c1 and c2 being arbitrary constants. Ex. 12. Solve 2(dx / dt ) # dy / dt ! x ! y 1 and dx / dt # dy / dt # 2 x ! y t . [Delhi Maths (Prog.) 2008] 2(dx / dt ) # dy / dt ! x ! y 1

Sol. Given

dx / dt # dy / dt # 2 x ! y

and Subtracting (2) from (1)

dx / dt ! 3 x 1 ! t ,

( !3) dt which is linear differential equation whose I.F. ! e ∃

xe !3t

∃ (1 ! t )e

!3t

t

... (1) ... (2) ... (3)

e !3t and solution is

dt # c1 (1 ! t ) (!e!3t / 3) ! (!1) ∋ (e !3t / 9) # c1 [Using the chain rule of integration by parts]

or

xe !3t

%(3t ! 2) / 9& e!3t # c1

or

x

c1e3t # (3t ! 2) / 9

... (4)

Created with Print2PDF. To remove this line, buy a license at: http://www.software602.com/

Miscellaneous problems based on this part of the book

3c1e3t # 1/ 3

From (4),

dx / dt

From (2),

dy / dt ! y

t ! (dx / dt ) ! 2 x

%

dy / dt ! y

or

∃ (!5c1e ye!t

or

3t

&

!5c1e3t # t / 3 # 1/ 9

( !1)dt which is linear differentail equation whose I.F. ! e ∃

ye !t

... (5)

t ! (3c1e3t # 1/ 3) ! 2 c1e3t # (3t ! 2) / 9

dy / dt ! y

or

M.5

# t / 3 # 1/ 9)e !t dt # c2

... (6) e !t and solution is

!5c1 ∃ e 2t dt # ∃ (t / 3 # 1/ 9)e !t dt # c2

!(5c1 / 2) ∋ e2t # (t / 3 # 1/ 9) ∋ (!e!t ) ! (1/ 3) ∋ (e!t ) # c2 [Using the chain rule of integration by parts]

ye!t

or

!(5c1 / 2) ∋ e2t ! (t / 3 # 4 / 9) ∋ (e!t ) # c2 y

!(5c1 / 2) ∋ e3t ! t / 3 ! 4 / 9 # c2et

... (7)

The required solution is given by (4) and (7), where c1 and c2 are arbitrary constants. Ex. 13. Solve the following simultaneous differential equations. (a) 2(dx / dt ) # (dy / dt ) ! x ! y 1, (dx / dt ) # (dy / dt ) # 2 x ! y (b) dx / dt # 4 y

sec2 2t , dy / dt

(c) dx / dt # 9(dy / dt ) # 2 x # 31y Ans. (a) x (b) x

x.

t (Delhi B.Sc. II (Prog) 2008) [Delhi B.A. II (Prog) 2009]

et , 3(dx / dt ) # 7(dy / dt ) # x # 24 y 3 [Delhi B.Sc. II (Prog) 2009]

c1e3t # (3t ! 2) / 9; y

c2 et ! (5c1 / 2) ∋ e3t ! (t / 3) ! (4 / 9)

!2c1 sin 2t # 2c2 cos 2t # (1/ 2) ∋ {cos 2t log (sec 2t # tan 2t ) # tan 2t}

y c1 cos 2t # c2 sin 2t # (1/ 4) ∋ {sin 2t log (sec 2t # tan 2t ) ! 1} Ex. 14. If f (D) = xD + 2 and g (D) = D + 5, then find [f (D) g (D)]y, where D ! d/dx. [Pune 2010] Sol. [f (D) g(D)] y ! [(xD + 2) (D + 5)]y ! "xD (D + 5) + 2(D + 5)]y ! (xD2 + 5 xD + 2D + 10)y Ex. 15. (a) Find the particular solution of the equation (D2 – 3D + 2) y = cos (e–x) by using the method of variation of parameters. (b) Solve by variation of parameters (D2 – 3D + 2) y = cos (e–x) [G.N.D.U. Amritsar 2011] Sol. (a). Given y2 – 3y1 + 2y ! cos (e–x) ... (1) Comparing (1) with y2 + Py1 + Qy ! R, we have R ! cos (e–x) ... (2) 2 Auxiliary equation of (2) is D – 3D + 2 ! 0 giving D ! 1,2 Hence, C.F. of (1) ! c1 ex + c2 e2x, c1 and c2 being arbitrary constants ... (3) Let u = ex and v ! e2x. also, here R ! cos (e–x) ... (4) Now, Then,

W

u v u1 v

ex

e2 x

! 2e3 x ! e3 x e3 x 2 0 e x 2e 2 x particular solution (or integral) of (1) ! uf (x) + vg(x),

... (5) ... (6)

Created with Print2PDF. To remove this line, buy a license at: http://www.software602.com/

M.6

Miscellaneous problems based on this part of the book

f ( x)

where

!∃

vR dx W

!∃

e 2 x cos(e ! x ) dx ! ! ∃ e ! x cos(e ! x )dx ! ∃ cos t dt e3 x

sin t sin e! x

[Putting e–x ! t so that e–xdx ! – dt] g ( x)

and

uR ∃ W dx



e x cos(e ! x ) e3 x

dx ! ∃ e !2 x cos(e ! x )dx

! [t sin t ! ∃ sin t dt ] ! ! t sin t ! cos t

!∃ t cos dt , putting e–x ! t

! e! x sin (e ! x ) ! cos (e! x )

Substituting the above values of f(x) and g(x) in (6), we have P.S. ! Particular solution ! ex sin (e–x) + e2x {–e–x sin (e–x) – cos (e–x)] ! –e2x cos (e–x) Part (b). Proceed as in part (a) to get C.F. and P.S. Then, the required solution is y ! C.F. + P.I. ! c1 ex + c2 e2x – e2x cos (e–x) Ex. 16. Given that the complementary solution of the differential equation (x2 – 1)y(( – 2x y( + 2y ! x2 – 1 is yc ! c1 x + c2 (1 + x2), find the particular solution using the method of variation of parameters. [Delhi B.Sc. (Hons) II 2011] 2 Sol. Re-writing the given equation, y2 – {2x/(x – 1)}y1 + {2/(x2 – 1)} y ! 1 ... (1) Comparing (1) with y2 + Py1 + Qy ! R, we have R ! 1 ... (2) Given that C.F. of (11 ! yc ! c1x + c2 (1 + x2), c1 and c2 being arbitrary constants ... (3) 2 Let u = x, v ! 1 + x . Also here R ! 1 ... (4)

W

Now, Then, where

and

u v u1 v1

x 1 # x2 ! 2x2 – (1 + x2) ! x2 – 1 2 0 1 2x

the required particular solution ! yp ! uf(x) + vg(x), f ( x)

!∃

g ( x)

vR dx W uR

∃W

!∃

dx

1 # x2 2

x !1 x

dx !

∃ x 2 ! 1 dx

!



2 ! (1 ! x2 ) 1! x

2

1 ( !2 x ) dx 2 ∃ 1 ! x2

dx

... (5) ... (6)

log e

1# x !x 1! x

1 log e (1 ! x 2 ) 2

Using the above values of f (x) and g (x) in (6), the required particular solution is yp

4 1# x 5 1 x 6log e ! x 7 # (1 # x 2 ) ∋ log e (1 ! x 2 ) ! ! x 2 # x loge 1 # x # 1 (1 # x 2 ) loge (1 ! x 2 ) 1! x 2 8 9 1! x 2

Ex. 17.(a) Given that y = x is a solution of (x2 + 1) (d2y/dx2) – 2x (dy/dx) + 2y = 0. Find a linearly independent solution by reducing the order. Write the general solution. [Delhi B.Sc. (Hons) II 2011] (b) Verify that y1 = x/(x–1)2 is a solution of the differential equation x (x – 1) y(( + 3xy( + y = 0. Find the other linearly independent solution of the equation and hence its general solution. [Mumbai 2010] Sol. (a) Comparing the given equation with p(x) y(( + q(x) y( + r(x) y ! 0, we have p(x) ! x2 + 1 and q(x) ! – 2x. Also here f(x) ! x. Hence the required second linearly independent solution is vf(x), i.e., xv, where v is given by (refer formula (6) off Art. 10.4A of chapter 10). v!



exp [! ∃ %q( x) / p( x)& dx ]

% f ( x)&2

dx

... (1)

Created with Print2PDF. To remove this line, buy a license at: http://www.software602.com/

Miscellaneous problems based on this part of the book

!∃

Here,

q( x ) dx p ( x)

2x

∃ 1 # x2 dx

M.7

! log e ( x 2 # 1) and hence we have

exp : ! ∃ % q ( x ) / p ( x )& dx ; ! exp :< log e ( x 2 # 1) ;= < =

v!

Hence, from (1),

x2 # 1

∃ ( x 2 # 1)2 dx

dx

∃ x2 # 1

!

e log e ( x

2

#1)

x2 #1

tan !1 x

and hence the required second linearly independent solution of the given equation is vf (x), i.e., x tan–1 x. Hence, the required solution is given by y ! c1x + c2x tan–1x, c1 and c2 being arbitrary constants Part (b). Hint. Proceed as in part (a). Here f(x) ! y1 ! x/(x – 1)2. Then, as before, show that –1 v ! x + log x and hence the second linearly solution of the given equation in vf(x), i.e., {x/(x–1)2} × (x–1 + log x). Required general solution is given by y ! (c1x)/(x – 1)2 + c2 × {x/(x – 1)2} × (x–1 + log x), c1 and c2 being arbitrary constants. Ex. 18. Let f(x) and x f(x) be the particular solutions of the differential equation y (( # R( x ) y ( # S ( x) y

0. Then the solution of the differential equation y (( # R( x ) y ( # S ( x) y

(a) y

(! x 2 / 2 # >x # ?) f ( x)

(c) y

(! x 2 # >x # ?) f ( x)

(b) y

( x 2 / 2 # >x # ?) f ( x) ( x 2 # >x # ?) f ( x)

(d) y

f ( x ) is

[GATE 2012]

Sol. Ans. (b). For the problem, 1 4 4 f (( x )type 5 f (( x ) use working rule of Art. 7.4B. Accordingly, 1 given objective ! ∃6 2∃ dx 7dx ! ∃ pdx 2 8 f (x) 9 2 e f (x) e 2log f ( x) [ f ( x)]2 here u xf ( x ), v u f ( ex ) and Re f ( x ) ... (1)

y

Let

Au # Bv

...(2)

be the general solution of y (( # R( x ) y ( # S ( x) y

f ( x)

... (3)

Then A and B are given by [Refer equations (5) and (8) of Art 7.4B] A1u # B1v and

0

A1u1 # B1v1

A1 xf ( x ) # B1 f ( x)

i.e., R

A1

i.e.,

Eliminating B, between (4) and (5) , A1 = 1

or

Since

dA/dx = 1

B

f ( x)

(5)

or

dB/dx = –x

or

...(6) dB = –xdx

( x # >) xf ( x) # (? ! x2 / 2) f ( x)

... (7)

( x2 / 2 # >x # ?) f ( x)

transforms the given

f ( x) y (( ! 4 f (( x) y ( + g(x)y = 0 into the equation of the form v (( # 4h( x)v (b) xf

f ( x) , giving

x # >, > being a constant

giving A

Ex. 19. If a transformation y = uv (a) 1/f

...(4)

!( x2 / 2) # ?, ? being a constant

using (6) and (7) , (2) yields y

2

0

f ( x)

A1 f ( x) # A1 x f (( x) ! A1 x f (( x )

A1 = 1, (5) yields B1 = –x

Integrating,

or A1 x # B1

d d ( xf ( x)) # B ( f ( x)) dx dx

A1{ f ( x ) # xf (( x)} # B1 f (( x)

or

0

(c) 1/2f

(d) f

2

differential equation 0 , then u must be [GATE 2012]

Created with Print2PDF. To remove this line, buy a license at: http://www.software602.com/

M.8

Miscellaneous problems based on this part of the book

Sol. Ans. (d). Re-writing the given equation, we have y (( {4 f (( x) / f ( x) y ( # {g ( x ) / f ( x )} y Comparing (1) with y (( # Py ( # Qy

0

... (1)

0 , here P {4 f (( x ) / f ( x )

Using result of working rule of Art. 10.7, the required value of u is given by

u

1 ! ∃ pdx e 2

1 4 4 f (( x ) 5 ! ∃6 7 dx e 2 8 f ( x) 9

e

2∃

f (( x ) dx f ( x)

e 2log f ( x)

[ f ( x )]2

Created with Print2PDF. To remove this line, buy a license at: http://www.software602.com/

ADVANCED ORDINARY DIFFERENTIAL EQUATIONS AND SPECIAL FUNCTIONS

WHERE IS

WHAT

Chapters

1.

Pages

Picard’s iterative method. Uniqueness and existence theorems

1.3-1.25

2.

Simultaneous equations of the form (dx)/P = (dy)Q = (dz)/R

2.1-2.24

3.

Total (or Pfaffian) differential equations

3.1-3.32

4.

Riccati’s equation

4.1-4.5

5.

Chebyshev polynomials

5.1-5.9

6.

Beta and Gamma functions

7.

Power Series

8.

Integration in series

8.1-8.60

9.

Legendre polynomials

9.1-9.50

6.1-6.22 7.1-7.7

10.

Legendre functions of the second order

10.1-10.12

11.

Bessel functions

11.1-11.45

12.

Hermite polynomials

12.1-12.12

13.

Laguerre polynomials

13.1-13.11

14.

Hypergeometric function

14.1-14.18

15.

Orthogonal sets of functions and Strum-Liouville problem

15.1-15.25

Created with Print2PDF. To remove this line, buy a license at: http://www.software602.com/

1 Picard’s Iterative Method Uniqueness And Existence Theorems 1.1. Introduction. In many practical problems we come across with a differential equation which cannot be solved by one of the standard methods known so far. Various methods have been formulated for getting to any desired degree of accuracy the numerical solution of the above mentioned type of differential equation with numerical coefficients and given conditions. In this chapter we propose to discuss Picard’s iteration method for finding an approximate solution of the initial value problem of the form dy/dx = f(x, y), y(x0) = y0. The condition y(x0) = y0 is called the initial condition. Here y(x0) denotes the value of y at x = x0. Sometimes y(x0) = y0 is also expressed by saying that y = y0 when x = x0. An iteration method is a method which consists of a repeatred application of exactly the same type of steps where in each step we use the result of the previous step (or steps). 1.2A. Picard’s method of successive approximations (or Picard’s iteration method) [Himanchal 2004, Bangalore 2002, 06; Allahabad 2001; Meerut 2000, 10; Ujjan 2003] Consider an initial value problem of the form dy/dx = f(x, y), y(x0) = y0. ...(1) By integrating over the interval (x0, x), (1) gives

z z y

y0

or

dy =

x

x0

f ( x, y) dx

y(x) – y0 =

or

z

y(x) = y0 +

x

f ( x, y) dx .

x0

z

x

x0

f ( x, y) dx ...(2)

Thus, the solving of initial value problem (1) is equivalent to finding a function y(x) which satisfies the equation (2), since by differentiating (2) we get dy/dx = f(x, y) and putting x = x0 in (2) yields y(x0) = y0 + 0 i.e., y(x0) = y0. Conversely, (2) has been obtained from (1) by integration over the interval (x0, x) and employing the initial condition y(x0) = y0. Since the information concerning the expression of y in terms of x is absent, the integral on the R.H.S. of (2) cannot be evaluated. Hence the exact value of y cannot be obtained. Therefore we determine a sequence of approximations to the solution (2) as follows. As a crude approximation, we put y = y0 in the integral on the right of (2) and obtain y1(x) = y0 +

!

x

x0

f ( x, y0 ) dx,

...(3)

where y1(x) is the corresponding value of y(x) and is called first approximation and is better approximation of y(x) at any x. To determine still better approximation we replace y by y1 in the integral on R.H.S. in (2) and obtain the second approximation y2 as y2(x) = y0 +

z

x

x0

f ( x, y1) dx.

...(4)

1.3

Created with Print2PDF. To remove this line, buy a license at: http://www.software602.com/

1.4

Picard’s Iterative Method. Uniqueness and Existence Theorems

Proceeding in this way, the nth approximation yn is given by yn(x) = y0 +

z

x

x0

f (x, yn 1) dx .

...(5)

Thus, we arrive at a sequence of approximate solutions y1(x), y2(x), y3(x),...........yn(x),..... 1.2B. Solved examples based on Art. 1.2A Ex. 1. Apply Picard’s method to solve the following initial value problem upto third approximation : dy/dx = 2y – 2x2 – 3 given that y = 2 when x = 0. [Agra 2005; Gwalior 2003; Delhi Maths (Hons.) 1998, 2005; Meerut 2000, 04, 05, 11] Sol. Given problem is dy/dx = 2y – 2x2 – 3, where y = 2, x = 0. ... (1) We know that the nth approximation yn of the initial value problem dy/dx = f(x, y), where y = y0 when x = x0 ...(2) yn = y0 +

is given by

yn = 2 +

# from (3),

z

x

0

(2yn

x

...(3)

f ( x, yn 1) dx.

x0

f(x, y) = 2y – 2x2 – 3,

Comparing (1) and (2),

z

x0 = 0

y0 = 2.

and

2 x 2 3) dx.

1

...(4) ...(5)

First approximation. Putting n = 1 in (5), we have y1 = 2 +

z

x

0

(2 y0 2 x 2 3) dx = 2 +

y1 = 2 +

or

z

x

0

z

x

(4 2 x 2 3) dx , using (4)

0

LM FG x IJ OP N H 3 KQ 3

(1 2 x 2 ) dx = 2 ∃ x 2

x

%2∃ x 0

2x3 . 3

...(6)

Second approximation. Putting n =2 in (5), we have y2 = 2 +

z

x

0

(2 y1 2 x 2 3) dx = 2 +

y=2+

or

z

x

0

FG1∃ 2x H

2x2

z

LM2FG 2 ∃ x 2x IJ 2 x 3OP dx , using (6) 3 K NH Q 4 x I dx = 2 + x + x – 2 x x . J 3 K 3 3 3

x

2

0

3

3

2

4

...(7)

Third approximation : Putting n = 3 in (5), we have y3 = 2 +

z

x

0

d2 y

2

y=2+

or

i

2 x 2 3 dx = 2 +

z

x

0

LM1 ∃ 2x N

4 x3 3

z

LM2FG 2 ∃ x ∃ x 2x x IJ 2x 3OP dx , using (7) 3 3K NH Q 2 x O dx = 2 + x + x – x 2x . 3 PQ 3 15 x

2

3

4

2

0

4

2

4

5

Ex. 2. Using the Picard’s method of successive approximations, find the third approximation of the solution of the equation : dy/dx = x + y2, where y = 0 when x = 0. [Delhi Maths (Hons) 1996; Meerut 2007; Ravishankar 2001 Indore 2001; Jabalpur 2000, 02, 05; Gwalior 2006; Rohilkhand 2004] Sol. Given problem is dy/dx = x + y2, where y = 0 when x = 0. ...(1) We know that the nth approximation yn of the initial problem dy/dx = f(x, y) where y = y0 when x = x0. ...(2) is given by Comparing (1) and (2),

yn = y0 + f(x, y) = x + y2,

z

x

x0

f ( x, yn 1 ) dx .

x0 = 0

...(3) and

y0 = 0.

...(4)

Created with Print2PDF. To remove this line, buy a license at: http://www.software602.com/

Picard’s Iterative Method. Uniqueness and Existence Theorems

z

yn =

# from (3) ,

x

x ∃ yn2

0

1.5

dx .

1

...(5)

First approximation. Putting n = 1 in (5) and using (4) we have

z z LMN x

z

dx ∃ y i dx %

x

1 2 x . 2 Second approximation. Putting x = 2 in (5) and using (6), we have

y1 =

y2 =

z

x

0

dx ∃ y i dx % 2 1

0

x

0

2 0

0

x dx %

...(6)

OP Q

4 x ∃ x dx = 1 x 2 ∃ 1 x 5 . 4 2 20

...(7)

Third approximation. Putting n = 3 in (5), we get y3 =

z z

x

0

z

dx ∃ y i dx = LMNx ∃ FH 12 x ∃ 201 x IK OPQ dx , using (7) LMx ∃ 1 x ∃ 1 x ∃ 1 x OP dx = 1 x ∃ 1 x ∃ 1 x N 4 400 20 Q 2 20 4400 2 2

x

2

2

5

0

x

1 8 x . 160 Ex. 3. Find the third approximation of the solution of the equation dy/dx = 2 – (y/x) by Picard’s method, where y = 2 when x = 1. [Delhi Maths (Hons.) 1997, 99, 2008, 2009; Gwalior 2004; Meerut 2002, 11; Rohilkhand 2000] Sol. Given problem is dy/dx = 2 – (y/x), where y=2 when x = 1. ...(1) We know that the nth approximation yn of the initial value problem dy/dx = f(x, y), where y = y0 when x = x0. ...(2)

=

0

4

10

7

2

z

yn = y0 +

is given by

x

x0

# from (3),

yn = 2 +

z

x

1

[2

11



f ( x, yn 1 ) dx .

f(x, y) = 2 – (y/x),

Comparing (1) and (2),

5

x0 = 1

...(3) and

y0 = 2.

1] dx .

(1 / x )yn

...(4) ...(5)

First Approximation. Putting n = 1 in (5), we get y1 = 2 +

z

x

1

a f

2 1 / x y0 dx = 2 +

z

x

2

1

b2 / xg

dx , using (4)

x

= 2 + 2 x 2 log x 1 = 2 + 2x – 2 log x – 2 = 2x – 2 log x.

...(6)

Second Approximation. Putting n = 2 in (5), we get y2 = 2 +

z FH IK z z z LMN FH IK OPQ z LMN OP z LNM Q x

1

2

y1 dx = 2 + x

x

1

[2

1 (2 x x

x

x

1 log x . dx = 2 + (log x ) 2 x 1 Third Approximation. Putting n = 3 in (5), we get

= 2 +2

y3 = 2 + =2+

x

1

x

1

x

2 log x )] dx, by (6)

1

% 2 ∃ (log x ) 2 .

FH 1 IK n2 ∃ (log x) sOP dx , by (7) x Q

2

1 y dx = 2 + x 2

2

2 1 (log x )2 dx = 2 + 2 x 2 log x x x

1

2

...(7)

2

LM MN

(log x ) 3 3

OP PQ

x

1

3

= 2 + 2x – 2 log x – (1/3) × (log x) – 2 = 2x – 2 log x – (1/3) × (log x)3.

Created with Print2PDF. To remove this line, buy a license at: http://www.software602.com/

1.6

Picard’s Iterative Method. Uniqueness and Existence Theorems

Ex. 4. (a) Using Picard’s method of successive approximation, find a sequence of two functions which approach solution of the initial value problem dy/dx = ex + y2, y(0) = 1. [Delhi Maths (Hons.) 1994, 2002] Sol. Given problem is dy/dx = ex + y2, where y=1 when x = 0. ...(1) We know that the nth approximation yn of the initial value problem dy/dx = f(x, y), where y = y0 when x = x0 ...(2) yn = y0 +

is given by

z

yn = 1 +

# from (3),

f ( x, yn 1) dx.

x0

f(x, y) = ex + y2,

Comparing (1) and (2),

z

x

...(3)

x0 = 0 x

0

and

y0 = 1. ...(4)

(e x ∃ yn2 1) dx.

...(5)

First Approximation. Putting n = 1 in (5), and using (4), we get y1 = 1 +

z

x

0

( e x ∃ y 02 ) dx = 1 +

z z

x

0

(e x ∃ 1) dx = 1 + e x ∃ x

x

= 1 + ex + x – 1 = ex + x.

0

...(6)

Second Approximation. Putting n = 2 in (5), we have y2 = 1 +

z

z

x

0

(e x ∃ y12 ) dx = 1 +

x

0

e x ∃ (e x ∃ x )2 dx , by (6)

z

L O (e ∃ e ∃ x ∃ 2 x e ) dx = 1 + Me ∃ 1 e ∃ x P ∃ 2 =1+ 2 3 N Q 1 O 1 x 1 1 ∃ 2L x e (1 . e ) dx P = e + e =1+e + e ∃ 2 3 2 MN 2 Q x

x

2x

2

x

x

3 x

2x

0

z

x

0

0

x e x dx

1 3 1 x – + 2 xe x [e x ]0x 3 2 x 2x 3 x x 2x 3 = e + (1/2) × e + x /3 – (1/2) + 2x e – 2 (e – 1) = (1/2) × e + x /3 + 3/2 + (2x – 1) ex. Ex. 4(b). Find three successive approximations of the solution of dy/dx = ex + y2, y(0) = 0. [Delhi Maths (Hons.) 2007] x 2 Sol. Given problem in dy/dx = e + y , y(0) = 0 ... (1) We know that the nth approximation yn of the initial value problem. dy/dx = f(x, y), where y = y0 when x = x0 ... (2) x

2x

3

x x 0

x

x

x

!

yn % y0 ∃

in given by

x

x0

yn %

# (3) reduces to

!

x

0

+

f ( x , yn 1 ) dx

f(x, y) = ex + y2,

Comparing (1) and (2), here

2x

0

... (3)

x0 = 0,

(e x ∃ yn2 1 ) dx

y0 = 0 ... (4) ... (5)

First approximation : Putting n = 1 in (5) and using (4), we get y1 %

!

x

0

(e x ∃ y02 )dx %

!

x

0

e x dx % [e x ]0x % e x 1

... (6)

Second approximation : Putting n = 2 in (5) and using (4) and (6), we get y2 %

!

x

0

(e x ∃ y12 )dx %

x

! {e 0

x

∃ (e x 1)2 }dx %

% [(1/ 2) & e2 x e x ∃ x]0x % (1/ 2) & (e2 x

!

x

0

(e2 x

e x ∃ 1) dx

2e x ∃ 2 x ∃ 1)

... (7)

Created with Print2PDF. To remove this line, buy a license at: http://www.software602.com/

Picard’s Iterative Method. Uniqueness and Existence Theorems

1.7

Third approximation : Putting n = 3 in (5) and using (4) and (7), we get y3 %

!

x

%

1 4

!

x

%

1 4

!

x

0

x

! {e

(e x ∃ y22 )dx %

0

0

0

x

∃ (1/ 4) & (e2 x

2e x ∃ 2 x ∃ 1) 2 }dx

(4e x ∃ e 4 x ∃ 4e 2 x ∃ 4 x 2 ∃ 1 4e3 x ∃ 4 xe 2 x ∃ 2e 2 x 8 xe x ( e4 x

4e x ∃ 4 x ) dx

4e3 x ∃ 2e 2 x ∃ 4 x 2 ∃ 4 x ∃ 1 ∃ 4 xe2 x 8 xe x ) dx x

x

( 1 ∋ 3 x ( x 1 ∋ 2 x ( x 1 ∋ 4 x3 ∋ −1 . −1 .( e , ∃ +e , ∃ ) ∃ 2 x 2 ∃ x ∗ ∃ ) x & / e2 x 0 (1) & / e2 x 0 ∗ + 0 0 3 4 4+ 3 2 14 2 ,0 ,0 + 1 2

x 1 % ∋+ e4 x (, 0 16

x

2 ∋+ ( x ) ( e x ) (1) & (e x ) (, , on integrating by parts the last two terms 0

% (1/16) & (e4 x 1) (1/ 3) & ( e3x 1) ∃ (1/ 4) & (e2 x 1) ∃ (1/12) & (4 x3 6 x2 ∃ 3x) ∃ ( x / 2) & e2 x (1/ 4) & e2 x ∃ 1/ 4 2( xe x e x ∃ 1)

% (1/16) &e4x (1/3) &e3x ∃ (1/ 4) &e2x ∃ (1/12) &(4x3 6x2 ∃ 3x) ∃ (1/ 4) & (2 x 1)e 2 x 2 ( x 1)e x (83 / 48). Ex. 5. Use Picard’s method to obtain a solution of the differential equation: dy/dx = x2 – y, y(0) = 0. Find at least the fourth approximation to each solution. [Meerut 1996] 2 Sol. Given dy/dx = x – y, where y=0 when x = 0. ...(1) We know that the nth approximation yn of the initial value problem dy/dx = f(x, y), when y = y0 when x = x0 ...(2)

z

yn = y0 +

is given by

x

f ( x, yn 1 ) dx .

x0

f(x, y) = x2 – y,

Comparing (1) and (2),

yn =

# from (3),

z

...(3)

x0 = 0 x

0

and

y0 = 0. ...(4)

( x 2 yn 1 ) dx.

...(5)

First approximation. Putting n = 1 in (5) and using (4), we get y1 =

z

x

0

( x 2 y0 ) dx =

z

x

0

x 2 dx %

LM1 x OP N3 Q 3

x 0

1 % x3 . 3

...(6)

Second approximation. Putting n = 2 in (5) and using (6), we get y2 =

z zd x

0

z FH z LMN

( x 2 y1 ) dx =

x

0

x2

IK

LM N

1 3 1 3 1 4 x dx = x x 3 3 12

OP Q

x 0

1 1 4 % x3 x . 3 12

...(7)

Third approximation. Putting n = 3 in (5) and using (7), we get

or

x

i

x

FH 1 x 3

IK OP Q

LM N

1 x 4 dx = 1 x 3 1 x 4 ∃ 1 x 5 3 12 60 12 0 0 y3 = x3/3 – x4/12 + x5/60. Fourth approximation. Putting n = 4 in (5) and using (7), we get y3 =

x 2 y2 dx =

x2

3

OP Q

x 0

...(8)

Created with Print2PDF. To remove this line, buy a license at: http://www.software602.com/

1.8

y4 =

Picard’s Iterative Method. Uniqueness and Existence Theorems

z

x

0

( x 2 y3 ) dx =

z

x

0

LMx FH 1 x N 3 2

1 x 4 ∃ 1 x5 12 60

3

IK OP dx = ∋) 1 x Q +3

x

1 4 1 5 1 6( x ∃ x x 12 60 360 ∗, 0

3

y = x3/3 – x4/12 + x5/60 – x6/360. Ex. 6. (a) Apply Picard’s method to find the solution of the problem dy/dx = y – x, y(0) = 2. Show that the iterative solution approaches the exact solution. [Meerut 1995] Sol. Given dy/dx = y – x, where y=2 when x = 0. ...(1) We know that the nth approximation of the initial value problem dy/dx = f(x, y) where y = y0 when x = x0 ...(2) or

yn = y0 +

is given by

z

x

x0

f ( x, yn 1) dx .

f(x, y) = y – x,

Comparing (1) and (2), # from (3),

...(3)

x0 = 1

yn = 2 +

z

and

x

0

( yn

y0 = 2.

x ) dx .

1

...(4) ...(5)

First approximation. Putting n = 1 in (5) and using (4), we get

z

x

z

x

1 2 x . 2 Second approximation. Putting n = 2 in (5) and using (6) we get

y1 = 2 + (y0 x) dx = 2 + 0

z

x

y2 = 2 + (y1 x ) dx = 2 + 0

z FH x

0

0

(2 x ) dx % 2 ∃ 2 x

IK

1 1 2 x x dx % 2 ∃ 2 x ∃ x 2 2 2

2 ∃ 2x

...(6)

1 3 x 6

... (7)

Third approximation. Putting n = 3 in (5) and using (7), we get y3 % 2 ∃

!

x

0

( y2 x) dx % 2 ∃

!

x

0

[2 ∃ 2 x ∃ x 2 / 2 x 3 / 6 x] dx

x 2 x3 x 4 x x2 x3 x 4 ∃ ∃ ∃ =1+x+1+ 2 6 24 1! 2 ! 3 ! 4 ! To find the exact solution of (1). Rewriting (1), we have

= 2 + 2x +

Its I.F. = e

z

(dy / dx) – y = –x, which is a linear differential equation ( 1) dx

z

%e

x

and hence its solution is

LM N

ye–x = ( x )(e x ) dx ∃ c = – x( e x ) or

ye–x = xe–x + e–x + c Given that y = 2, when x = 0, so (10) gives Hence, from (10), the exact solution is We know that

z

OP Q

...(8)

...(9)

1.( e x ) dx ∃ c , c being an arbitrary constant or

2=1+c

y = x + 1 + c ex. ...(10) or c = 1. y = x + 1 + e x. ...(11)

ex = 1 + x + ( x 2 / 2!) ∃ ( x 3 / 3!) + ........ ad inf.

...(12)

Keeping (12) and (8) in view, we find that the approximate solution tends to y = 1 + x + 1 + x / 1! ∃ x 2 / 2! ∃ x 3 / 3! + .... = 1 + x + ex i.e., which is exact solution of (4). Ex. 6 (b) Apply Picard’s iteration method to the initial value problem dy/dx = y, y(0) = 1 and show that the successive approximations tend to the limit y = ex, the exact solution. [Allahabad 2001, 05; Gwalior 2005; Indore 2002, 03; Kurukshetra 2003; Pune 2002] Sol. Proceeding as in Ext. 6 (a), we have

Created with Print2PDF. To remove this line, buy a license at: http://www.software602.com/

Picard’s Iterative Method. Uniqueness and Existence Theorems

1.9

2 y2 = 1 + x ∃ x , 1! 2 !

y1 = 1 + x,

y3 = 1 +

x x 2 x3 xn ∃ ∃ ∃ ...... ∃ . 1! 2 ! 3! n! we have (1/y)dy = dx so that

x x 2 x3 ∃ ∃ . 1! 2 ! 3!

yn = 1 +

In general, Also, solving dy/dx = y,

...(1) y = c e x.

... (2) Given that y = 1 when x = 0, so (2) given 1 = c and hence the exact solution (2) becomes y = e x. ...(3) From (1), we see that the successive approximations tend to the limit y = ex as n 3 4, which is the exact solution. Ex. 6. (c). Show that successive approximation 5n (x) for the equation y 6( x ) % y ( x) ( 4 7 x 7 4) with initial condition y(0) = 1 are given by 5n(x) = 1 + x + x2/2! + x3/3! + ... + xn/n! [Kolkata 2002, 04, 07]

EXERCISE 1 (A) 1. Apply Picard’s method to the following initial value problems and find the first three successive approximations. (i) dy/dx = 2xy, y(0) = 1. [Allahabad 1994, Delhi Maths (Hons.) 95, 2001, 04, 06] Ans. y1 = 1 + x2, y2 = 1 + x2 + (x4/2 ), y3 = 1 + x2 + (x4/2) + (x6/6) (ii) dy/dx = 3ex + 2y, y(0) = 0. [Himanchal 2002; Meerut 1993, 94, M.K.U. (Tamil Nadu), 2002, 02] Ans. y1 = 3(ex – 1), y2 = 9ex – 6x – 9, y3 = 21 ex – 6x2 – 18x – 21. (iii) dy/dx = x + y, y(0) = 1. [Allahabad 1999; Calicut 2004; Meerut 1993, 95] Ans. y1 = 1 + x + (x2/2), y2 = 1 + x + x2 + (x3/6), y3 = 1 + x + x2 + (x3/3) + (x4/24) (iv) dy/dx = 1 + xy, y(0) = 2. [Agra 2001, 04, 05 Himanchal 2004, 05; Lucknow 2003, Meerut 2001, 06] Ans. y1 = 2 + x + x2,

y2 = 2 + x + x2 + (x3/3) + (x4/4),

y3 = 2 + x + x2 + (x3/3) + (x4/4) + (x5/15) + (x6/24) (v) dy/dx = 2x – y2, where y = 0 at x = 0. Ans. y1 = x2, y2 = x2 – (x5/5),

y3 = x2 – (x5/5) + (x8/120) – (x11/275).

(vi) dy/dx = ex + y2, y(0) = 0. Ans. y1 = ex –1 , y2 = (1/2)e2x – ex + x + (1/2), y3 = (1/16)e4x – (1/3)e3x + (1/2)xe2x + (1/2)e2x – 2xex + 2ex + (1/3)x3 + (1/2)x2 + (1/4)x – (107/48). 2. Solve the differential equation dy/dx = x – y with the condition y = 1 when x = 0 and show that the sequence of approximations given by Picard’s method tend to the exact solution as a limit. [Agra 2001; Meerut 2003] Ans. y1 = 1 – x + y4 = 1 – x +

x2 , 2!

2x 2 2x3 2x4 ∃ 2! 3! 4!

2x2

y 2 = 1 – x + 2!

x3 , 3!

y3 = 1 – x +

2x2 2!

2 x3 x 4 ∃ ; 3! 4!

x5 tending to y = –1 + x + 2e–x, which is exact solution. 5!

3. Use Picard’s method to approximate the solution of the equation dy/dx + 2xy2 = 0 with y = 1 when x = 0 and hence show that y = 1/(1 + x2). [Rohtak 2001, Meerut 1995] Ans. y1 = 1 – x2, y2 = 1 – x2 + x4 – (1/3)x6, y3 = 1 – x2 + x4 – x6 + (2/3)x8 – (1/3)x10 + (1/9)x12 – (1/63)x14.

Created with Print2PDF. To remove this line, buy a license at: http://www.software602.com/

1.10

Picard’s Iterative Method. Uniqueness and Existence Theorems

4. State the conditions under which the initial value problem dy/dx = x2, x0 = 2, y0 = 1 has unique solution by Picard’s method of successive approximation. Obtain the solution of the initial value problem dy/dx = x2 + y2, y(0) = 1, by Picards method as far as the term, involving x4. [Himanchal 2003, 05; Jabalpur 2004, 06; Kurukshetra 2006] Ans. y1(x) = 1 + x + x3/3, y2(x) = 1 + x + x2 + (2x3/3) + (2x4)/12 + (2x5)/15 + x6/63 5. Using Picard’s method find the third approximation of the solution of the initial value problem dy/dx = 1 + y2, y(0) = 0. [Kolkata 2004, 07, Pune 2002] Ans. y1(x) = x, y2(x) = x + x3/3, y3(x) = x + x3/3 + (2x5)/15 + x7/63 6. Find the exact solution of initial value problem dy/dx = x + y, y(0) = 0. Next apply Picard’s iterative method to obtain three successive approximate solutions y1(x), y2(x), y3(x). [Allahabad 2003, 07] 7. Solve the initial value problem dy/dx = y2, y(0) = 1 by method of successive approximation. [Jabalpur 2004; Osmania 2006] 2 3 Ans. y1(x) = 1 + x; y2(x) = 1 + x + x + x /3; y3(x) = 1 + x + x2 + x3 + (2x4)/3 + x5/3 + x6/9 + x7/63 8. (a) Under what condition does the initial value problem of the form y6 = f(x, y), y(x0) = y0 has unique solution ? Give statement only. (b) Consider the initial value problem y6 = x – y2, y(0) = 1/2. (i) Does this have a unique solution ? Justify your answer (ii) Applying Picard method find an approximate solution of the above initial value problem containing at least four no-zero terms (iii) Give your comments regarding the utility of Picard’s method in view of the above initial value problem. [Lucknow 2004, 05] 9. Consider the initial value problem dy/dx = f(x, y), y(a) = b and discuss Picard’s method of successive approximation to solve it. [Bangalore 2002, 06; Himanchal 2004] 10. Let {yn} be a sequence of successive approximation to the solution of dy/dx = f(x, y), y(x0) = y0 such that y10 = y9. Show y10 is exact solution. [Himanchal 2002, 03, 05, 07] 11. Explain initial value problem and equivalent integral equation. [Kanpur 2002, 07; Rohilkhand 2001, Ujjain 2000, 01, 06] 1.3 A. Working rule for Picard’s method of solving simultaneous differntial equations with initial conditions, namely dy/dx = f(x, y, z), dz/dx = g(x, y, z) where y = y0, z = z0 when x = x0. ...(1) The nth approximation (yn, zn) to the initial value problem (1) is given by yn = y0 + and

zn = z0 +

z z

x

f ( x, yn 1, zn 1) dx

...(2)

g( x, yn 1, zn 1) dx .

...(3)

x0

x

x0

1.3B. Solved examples based on Art 1.3A. Ex. 1(a) Find the third approximation of the solution of the equation dy/dx = z, dz/dx = x3(y + z) by Picard’s method where y = 1, z = 1/2 where x = 0. [Meerut 2006,07; Rohilkhand 2002; Gwalior 2003] 3 Sol. Given dy/dx = z, dz/dx = x (y + z), y = 1, z = 1/2 when x = 0. ...(1) We know that the nth approximation (yn, zn) to the initial value problem dy/dx = f(x, y, z), dz/dx = g(x, y, z), when y = y0, z = z0 when x = x0 ...(2) is given by

yn = y0 +

z

x

x0

f ( x, yn 1, zn 1) dx

...(3)

Created with Print2PDF. To remove this line, buy a license at: http://www.software602.com/

Picard’s Iterative Method. Uniqueness and Existence Theorems

zn = z0 +

and

z

x

g( x, yn 1, zn 1) dx .

x0

Comparing (1) and (2), we have f(x, y, z) = z, g(x, y, z) = x3 (y + z), yn = 1 +

# From (3),

1.11

z z

...(4)

y0 = 1, x

0

zn

z0 = 1/2,

x0 = 0. ...(5) ...(6)

1 dx

d

i

...(7)

1 1 dx % 1 ∃ x . 2 2

...(8)

x 1 x 3 yn 1 ∃ zn 1 dx . + 2 0 First approximation. Putting n = 1 in (6) and using (5), we get

zn =

and from (4),

y1 = 1 +

z

x

0

z0 dx % 1 ∃

Next, putting n = 1 in (7) and using (5), we gets z1 = 1 ∃ 2

z

x

0

z

x 3 ( y 0 ∃ z 0 ) dx =

x

0

1 ∃ 2

z

x

0

FH

x3 1∃

IK

1 1 3 dx % ∃ x 4 . 2 2 8

...(9)

Second approximation. Putting n = 2 in (6) and using (9), we get

z z z FH

y2 = 1 +

x

0

z1 dx % 1 ∃

z FH x

0

IK

5

1 3x 1 3 4 ∃ x dx = 1 + x ∃ . 2 8 2 40

...(10)

Next, putting n = 1 in (7) and using (8) and (9), we get x

z2 = 1 ∃

x 3 ( y1 ∃ z1 ) dx = 1 ∃ 2 0

2

= 1∃ 2

x

0

IK

z

x

0

FH

x 3 1∃

IK

1 1 3 x ∃ ∃ x 4 dx 2 2 8

1 3 4 1 5 3 8 3 3 1 4 3 7 x ∃ x ∃ x dx = ∃ x ∃ x ∃ x . 2 2 8 2 8 10 64

...(11)

Third approximation. Putting n = 3 in (6) and (11), we get

z

1 3 1 1 9 −1 3 4 1 5 3 8. ∃ x ∃ x ∃ x 0 dx % 1 ∃ x ∃ x5 ∃ x 6 ∃ x . / 0 0 12 8 10 64 2 2 40 60 192 Next, putting n = 3 in (7) and using (10) and (11), we get x

y3 = 1 + z2 dx = 1 ∃

z

z3 = 1 ∃ 2

= 1∃ 2

z

0

x

x

0

x

!

x 3 ( y 2 ∃ z 2 ) dx = 1 ∃

2

FH 3 x ∃ 1 x 2 2 3

4

z

x

0

LM N

OP Q

5 4 5 8 x 3 1 ∃ x ∃ 3x ∃ 1 ∃ 3x ∃ x ∃ 3x dx 2 40 2 8 10 64

IK

4

5

8

9

12

1 3x x 3x 7x x ∃ 3 x7 ∃ 7 x8 ∃ 3 x11 dx % ∃ ∃ ∃ ∃ ∃ 8 40 64 2 8 10 64 360 256

Ex. 1 (b) Find the third approximation of the solution of the equation d 2 y/dx 2 = x (y + dy/dx), where y = 1 and dy/dx = 1/2 when x = 0 (Agra 2000, 02; Himanchal 2004, 05; Meerut 2000; Rohilkhand 2002) Sol. Given that d2y/dx2 = x3(y + dy/dx), where y = 1 and dy/dx = 1/2 when x = 0. ...(1) Let dy/dx = z so that d2y/dx2 = dz/dx. Then, we have dy/dx = z, dz/dx = x3(y + z), where y = 1 and z = 1/2 when x = 0. ...(2) which, is the same as given in solved Ex. 1 (a) Proceed now as before and obtain the required third approximation: y3 = 1 + x/2 + 3x5/40 + x6/60 + x9/192. Note : For complete solution of Ex. 1(b), you need not find z3. 3

Created with Print2PDF. To remove this line, buy a license at: http://www.software602.com/

1.12

Picard’s Iterative Method. Uniqueness and Existence Theorems

Ex. 2 (a) Find the third approximation of the solution of the equation dy/dx = z, dz/dx = x2z +x y by Picard’s method, y = 5 and z = 1 when x = 0. [Bangalore 2001, Meerut 2006, 07] Sol. Given dy/dx = z, dz/dx = x2z + x4y, y = 5, z=1 when x = 0. ...(1) We know that the nth approximation (yn, zn) to initial value problem dy/dx = f(x, y, z), dz/dx = g(x, y, z) where y = y0, z = z0, when x = x0 ...(2) 4

is given by

yn = y0 +

and

zn = z0 +

z z

x

x0 x

x0

f ( x, yn 1, zn 1) dx

...(3)

g( x, yn 1, zn 1) dx .

...(4)

Comparing (1) and (2), we have f(x, y, z) = z, g(x, y, z) = x2z + x4y,

y0 = 5,

# from (3),

yn = 5 +

z

zn = 1 +

and from (4),

x

0

( x 2 zn

z

z0 = 1, x

0

zn

1∃

x0 = 0.

1 dx

...(5) ...(6)

x 4 yn 1) dx.

...(7)

First approximation. Putting n = 1 in (6) and using (5), we get

z

y1 = 5 +

x

0

Next putting n = 1 in (7) and using (5), we get z1 = 1 +

z z zd z FH x

z

( x 2 z0 ∃ x 4 y0 ) dx = 1 +

0

z

z0 dx % 5 ∃

x

0

x

dx % 5 ∃ x.

...(8)

( x 2 ∃ 5x 4 ) dx % 1 ∃

0

x3 ∃ x 5. 3

...(9)

Second approximation. Putting n = 2 in (6) and using (9), we get y2 = 5 +

x

0

z1 dx = 5 +

z

x

0

FG1 ∃ x ∃ x IJ dx % 5 ∃ x ∃ x ∃ x . 12 6 H 3 K 3

4

5

6

...(10)

Next, putting n = 2 in (7) and using (8) and (9), we get x

z2 = 1 +

2

x z1 ∃ x y1 dx = 1 +

0

0

x

z x

i

4

LMx FG1 ∃ x ∃ x IJ ∃ x N H 3 K 3

2

5

4

OP Q

(5 ∃ x ) dx

IK

4 5 7 1 2 1 x ∃ x dx = 1 + x 3 ∃ x 5 ∃ x 6 ∃ x 8. 3 3 9 8 Third approximation. Putting n = 3 in (6) and using (11), we get

=1+

y3 = 5 +

z

x

0

z2 dx = 5 +

z

x

0

0

x 2 ∃ 5x 4 ∃

FH1 ∃ 1 x 3

3

∃ x5 ∃

...(11)

IK

x4 x6 2 x7 x9 2 6 1 8 x ∃ x dx % 5 ∃ x ∃ ∃ ∃ ∃ 9 8 12 6 63 72

Next, putting n = 3 in (7) and using (10) and (11), we have z3 = 1 +

zd x

0

i

x 2 z2 ∃ x 4 y2 dx = 1 + 3

5

z LMN x

0

FG H

x4 5 ∃ x ∃

IJ K

FG H

x4 x6 x3 2 x 6 x8 ∃ ∃ x2 1∃ ∃ x5 ∃ ∃ 12 6 3 9 8

IJ OP dx KQ

6

= 1 + (1/3)x + x + (2/9)x + (1/8)x8 + (11/224)x9 + (7/264)x11. Ex. 2 (b) Find the third approximation of the solution of the equation d2y/dx2 = x2(dy/dx) + 4 x y where y = 5 and dy/dx = 1 when x = 0. Sol. Let dy/dx = z so that dz/dx = d2y/dx2 = x2z + x4y, where y = 5 and z = 1 when x = 0. ( dy/dx = 1 8 z = 1)

Created with Print2PDF. To remove this line, buy a license at: http://www.software602.com/

Picard’s Iterative Method. Uniqueness and Existence Theorems

1.13

This is exactly the same problem as given in Ex. 2.(a) Proceed as above and obtain the required value of y3. In the solution of Ex. 2 (b), you need not compute z3 because we want third approximation of the solution of the original equation in which y is dependent variable. # The required third approximation = y3(x) = 5 + x + (1/12)x4 + (1/6)x6 + (2/63)x7 + (1/72)x9. Ex. 3. Find the third approximation of the initial value problem 2 d y/dx2 = xy + 1, when (y)0 = 1 and (dy/dx)0 = 0. [Meerut 1994] 2 2 Sol. Let dy/dx = z so that dz/dx = d y/dx = xy + 1. Again, re–writing the initial conditions, y0 = 1, z0 = 0 when x = 0. In order to solve the given problem, we shall solve the following initial value problem involving differential equations : dy/dx = z, dz/dx = 1 + xy, y = 1, z = 0 when x=0 ...(1) We know that the nth approximation (yn, zn) to the initial value problem dy/dx = f(x, y, z), dz/dx = g(x, y, z), when y = y0, z = z0 when x = x0 ...(2) is given by

yn = y0 +

and

zn = z0 +

z z

x

x0

x

x0

Comparing (1) and (2), we have f(x, y, z) = z, g(x, y, z) = 1 + xy,

d gd x, y

yn = 1 +

# From (3),

z

zn =

and from (4),

...(3)

n 1, zn 1

...(4)

y0 = 1,

x

0

z

x

0

i dx i dx .

1

f x, yn 1, zn

zn

z0 = 0

x0 = 0.

and

1 dx

...(5) ...(6)

(1 ∃ xyn 1) dx.

...(7)

First approximation. Putting n = 1 in (6) and using (5), we get y1 = 1 ∃

z

x

0

z0 dx % 1 ∃

Next, putting n = 1 in (7) and using (5), we get

z

z

z

x

0

(0) dx % 1.

...(8)

x2 . 2 0 0 Second approximation. Putting n = 2 in (6) and using (9), we get

z1 =

z

x

(1 ∃ xy0 ) dx %

x

y2 = 1 + z1 dx % 1 ∃ 0

z

x

0

x

(1 ∃ x ) dx % x ∃

FG x ∃ x IJ dx = 1∃ 1 x H 2K 2 2

2



1 3 x . 6

...(9)

...(10)

Next, putting n = 2 in (7) and using (8), we get

zb x

g

zb x

g

1 2 x . 2 Third approximation. Putting n = 3 in (6) and using (11), we get

z2 =

0

1 ∃ xy1 dx = 1 ∃

z

x

y3 = 1 + z2 dx % 1 ∃ 0

0

1 ∃ x dx % x ∃

...(11)

z FH

x∃

1 3 x 6

IK OP dx % x ∃ x ∃ x ∃ x 2 8 30 Q

x

0

IK

1 2 1 1 x dx = 1 ∃ x 2 ∃ x 3 . 2 2 6

Next, putting n = 3 in (7) and using (10), we get z3 =

z

0

z

b1 ∃ xy g dx = LMN1 ∃ x FH1 ∃ 12 x

x

2

x

0

2



2

4

5

Created with Print2PDF. To remove this line, buy a license at: http://www.software602.com/

1.14

Picard’s Iterative Method. Uniqueness and Existence Theorems

EXERCISE 1 (B) 1. Apply Picard’s method upto third approximation to solve the equations: dy/dx = x + z, dz/dx = x – y2, given that y = 2, z = 1 when x = 0 [Bhopal, 2002; Meerut 2000; Agra 2003, 04; Gwalior 2004] [Ans. y 1 = 2 + x + (1/2)x 2 , z 1 = 1 – 4x + (1/2)x 2 ; y 2 = 2 + x –(3/2)x 2 + (1/6)x 3; z 2 = 1 – 4x – (3/2)x 2 – x 3 – (1/4)x 4 – (1/20)x 5 ; y 3 = 2 + x – (3/2)x 2 – (1/2)x 3 – (1/4)x 4 – (1/20)x5 – (1/120)x6 ; z3 = 1 – 4x – (3/2)x2 + (5/3)x3 + (7/12)x4 – (31/60)x5 + (1/12)x6 – (1/252)x8.] 2. Find the third approximation of the solution of the following equations: dy/dx = 2x + z, dz/dx = 3xy + x2z where y = 2 and z = 0 when x = 0. [Meerut 2000, 05; I.A.S. 2000] [Ans. y3 = 2 + x2 + x3 + (3/20)x5 + (1/10)x6, z3 = 3x2 + (3/4)x4 + (6/5)x5 + (3/28)x7 + (3/40)x8.] 1.4. Problems of existence and uniqueness : An Introduction. Consider the initial value problem | dy/dx | + | y | = 0, y(0) = 1. ...(1) If possible, let y 9 0. Then division by | y | and integration leads to an absurd result. Hence y = 0 is the only solution of the differential equation. Clearly this solution does not satisfy the initial condition y(0) = 1. Thus, we see that the initial value problem (1) has no solution at all. Now let us consider the initial value problem dy/dx = x, y (0) = 1. ...(2) Separating the variables, we get dy = x dx. Integrating, y = (x2/2) + c, where c is an arbitrary constant. Using the initial condition y (0) = 1 i.e., x = 0, y = 1, we get c = 1. Hence, the initial value problem (2) has only one solution, namely, y = (x2/2) + 1. Finally consider the following initial value problem dy/dx = (y – 1)/x, y (0) = 1. ...(3) Separating the variables, (dy)/(y – 1) = (dx)/x. Integrating, log (y – 1) = log x + log c or y – 1 = xc. Using the given initial condition, i.e., x = 0, y = 1, we see that c cannot be dertermined. Thus, the given initial value problem (3) has infinite solutions given by y – 1 = xc, where c is an arbitrary constant. From the above examples, we conclude that an initial value problem dy/dx = f(x, y), y (x0) = y0 ...(4) may have none, exactly one, or more than one solution. This leads us to the following two fundamental questions. Problem of existence. Under what conditions does an initial value problem of the form (4) has at least one solution ? Problem of uniqueness. Under what conditions does that problem has a unique solution, that is, only one solution ? Theorems which state such conditions are called existence theorem and uniqueness theorem, respectively. It may be noted that the above three examples are very simple and investigation about their existence and uniqueness is evident by mere inspection (or by actually solving), without using any theorem. However, when the equation cannot be solved by standard methods, existence and uniqueness theorem will play an important role. 1.5. Lipschitz condition. A function f(x, y) is said to satisfy a Lipschitz condition in a region D in xy–plane if there exists a positive constant k such that

Created with Print2PDF. To remove this line, buy a license at: http://www.software602.com/

Picard’s Iterative Method. Uniqueness and Existence Theorems

1.15

| f(x, y2) – f(x, y1) | : K | y2 – y1 | whenever the points (x, y1) and (x, y2) both lie in D. The constant K is called a Lipschitz constant for the function f(x, y). 1.6. Picard’s Theorem. Existence and uniqueness theorem. [Himanchal 2008, 09; Agra 2003, Calicut 2003, I.A.S. 1985 ; Meerut 2001, 02, 07, 11; Gwalior 2004, 05; G.N.D.U. Amritsar 2000; Jiwaji 2002 Ravishankar 2002, Rajasthan 2004, Rohilkhand 2007; Kolkata 2003; Ujjain 2003, 06] Statement. Let f(x, y) be continuous in a domain D of the (x, y) plane and let M be a constant such that | f(x, y) | : M in D. ...(1) Let f(x, y) satisfy in D the Lipschitz condition in y namely | f(x, y1) – f(x, y2) | : K | y1 – y2 |, ...(2) where the constant K is independent of x, y1, y2. Let the rectangle R, defined by | x – x0 | : h, | y – y0 | : k, ...(3) lie in D, where Mh < k. Then, for | x – x0 | : h, the differential equation dy/dx = f(x, y) has a unique solution y = y (x) for which y (x0) = y0. Proof. [Read article 1.2A also for this proof carefully.] We shall prove this theorem by the method of successive approximations. Let x be such that | x – x0 | : h. We now define a sequence of functions y1 (x), y2 (x), ... yn, (x) ..., called the successive approximations (or Picard Iterants) as follows :

z z z z x

; < f ( x, y ) dx < ... .... .... = f ( x, y ) dx < f ( x, y ) dx < >

y1 (x) = y0 + f ( x, y0 ) dx x0

y2 (x) = y0 + ...

...

...

yn – 1 (x) = y0 +

yn (x) = y0 +

x

x0

1

...(4)

x

x0

n 2

x

x0

n 1

We shall divide the proof into five main steps. First Step. We prove that, for x0 – h : x : x0 + h the curve y = yn (x) lies in the rectangle R, that is to say y0 – k < y < y0 + k. Now, or

y1 y0 =

z

z

x

x

f ( x, y0 ) dx : | f ( x, y0 )|.| dx | , by (4)

x0

x0

y1 y0 : M | x – x0 | : Mh < k, using (1), (3) and the given result viz Mh < k.

This proves the desired result for n = 1. Assume that lies in R and so f(x, yn – 1) is defined and continuous and satisfies f ( x , yn 1 ) : M

From (4), we have

yn y0 =

z

x

x0

on

f ( x, yn 1 ) dx :

y = yn – 1 (x)

[x0 – h, x0 + h].

z

x

x0

f ( x, yn 1) . dx : M x x0 : Mh < k,

as before which shows that yn (x) lies in R and hence f(x, yn) is defined and continuous on [x0 – h, x0 + h]. The above arguments show that the desired result holds for all n by induction. Second Step. We prove again by induction, that

yn

yn

1

:

MK n n!

1

n

x x0 .

...(5)

Created with Print2PDF. To remove this line, buy a license at: http://www.software602.com/

1.16

Picard’s Iterative Method. Uniqueness and Existence Theorems

We have already verified (5) for n = 1 in first step where we have shown that | y1 – y0 | : M | x – x0 |. Assume that this inequality (5) holds for n – 1 in place of n, that is, let yn

Then, we have or

yn

1

yn

yn

1

=

yn

yn

1

:

! ? f ( x, y x

z

x0

x

n 1

n 1)

...(6)

.

f ( x, yn

≅ dx

2)

, by (4)

f (x, yn 1) f ( x, yn 2 ) . dx .

x0

f (x, yn 1 ) f ( x, yn 2 ) : K yn

Lipschitz condition (2) gives From (7) and (8), we get

z

n 2 : MK x x0 (n 1)!

2

1

...(7) yn

2

...(8)

n 1 | x x |n 0 , by (6) . dx : K . MK . x0 (n 1)! n Hence by mathematical induction, we conclude that (5) is true for each natural number n. Third Step. We shall now prove that the sequence yn converges uniformly to a limit for

yn

yn

1

:

x

K yn

1

yn

2

x0 – h : x : x0 + h. For the interval under consideration, x x0 : h. Hence from second step, we get

yn

yn

:

1

MK n 1 h n is true for all n. n!

Using this, the infinite series y0 + (y1 – y0) + (y2 – y1) – ... + (yn – yn – 1) + .... ...(9) 1 MKh2 ∃ ... ∃ 1 MK n 1hn ∃ ... M Kh e 1, : y0 + Mh + : y0 + 2! n! K which is known to be convergent for all values of K, h and M. Consequently, the series (9) is surely convergent. Thus, by the Weirstrass M–test, the series (9) converges uniformly on [x0 – h, x0 + h]. Now since the terms of (9) are continuous functions of x, therefore, its sum = Lim yn (x) = y (x), say,,

yn % y0 ∃

as

n3 4

n

Α(y

n

yn 1 )

...(10)

n %1

must be continuous. Fourth Step. We now show that y = y(x) satisfies the differential equation dy/dx = f(x, y). Since yn (x) tends uniformly to y (x) in [x0 – h, x0 + h] and by Lipschitz condition, f ( x, y)

f ( x, yn ) : K y yn ,

it follows that f [x, yn (x)] tends uniformly to f [x, y (x)]. Again from (4) we have yn (x) = y0 +

z

x

x0

f [x, yn

Lim yn (x) = y0 + Lim

or

n3 4

z

x

n 3 4 x0

1

( x )] dx

f [x, yn 1 ( x )] dx , letting n 3 4.

Since the sequence f [x, yn (x)], consisting of continuous functions on the given interval, converge uniformly to f [x, y(x)] on the same interval, the interchanges of limiting operations given below are valid. Thus using (10), we have y(x) = y0 +

z

x

Lim f [x, yn 1 ( x)] dx

x0 n 34

or

y(x) = y0 +

z

x

x0

f [ x, y ( x)] dx ....(11)

Created with Print2PDF. To remove this line, buy a license at: http://www.software602.com/

Picard’s Iterative Method. Uniqueness and Existence Theorems

1.17

The integrand on the right–hand side of (11) being a continuous function of x, we conclude that the integral has the derivative. Thus, the limit function y(x) satisfies the differential equation dy/dx = f(x, y) on [x0 – h, x0 + h] and is such that y(x0) = y0. [In the above four steps we have thus proved the existence of a solution of the given initial value problem. The next step will show that the solution y(x) is unique.] Fifth Step. Uniqueness of the solution : We now prove that the solution y = y(x) just found is the only solution for which y(x0) = y0. Assume if possible y = Y(x), say, is another solution of the given initial value problem. Y ( x) y( x) : B, Let where x0 – h : x : x0 + h. It may be noted here that we can surely take B = 2K. From (11), we get Y ( x)

y( x) =

z

x

x0

l

q f lx, y(x)q

f x, Y ( x)

or

y( x) : K

Y ( x)

l

y( x) :

or Y ( x)

z

x

z

x

x0

Y ( x)

q f lx, y(x)q . dx

f x, Y ( x )

Y ( x ) y( x ) . dx .

x0

...(13)

q f lx, y(x)q : K Ybx)

y( x) , by Lipschitz condition

y( x) : K.B x x0 , using (12)

...(14)

f x, Y ( x )

or

l

...(12)

g

Now substituting (14) for integrand in (13), we get

z z

x

| Y(x) – y(x) | : K2B

x0

x

x 0 dx :

K2B x

2

x0

.

2!

...(15)

Again, substituting (15) for the integrand in (13), we get y( x) :

Y ( x)

K3B 2!

x

x0

x x0

2

dx :

K 3 B x x0

3

3!

Continuing in this way, we shall surely get Y ( x)

y( x) :

aKhf Α n! n

Now the series

B

n%0

K n B x x0 n!

n

:B

aKhf n!

n

,

n

converges, and so

Lim B

n3 4

| x x0 | : h

as

aKhf n!

n

...(16)

=0.

Thus Y ( x) y( x) can be made less than any number however small and consequently we conclude that Y(x) – y(x) = 0 i.e., Y(x) = y(x) This shows that the solution y = y(x) is always unique, and the proof of the theorem is complete. Important note : The above theorem is an existence theorem because it says that the initial value problem does have a solution. It is also a uniqueness theorem, because it says that there is only one solution. Remark 1. If the existence theorem is asked then you need not mention uniqueness in the statement of theorem and finish up the proof just after the fourth step. Again if you are asked to state and prove the uniqueness theorem, then give the complete proof of first four steps and fifth step. Remark 2. If f(x, y) satisfies the condition Βf Βy : K ...(i) for all values of x, y in the given range then for the same constant K the Lipschitz’s condition is also satisfied.

Created with Print2PDF. To remove this line, buy a license at: http://www.software602.com/

1.18

Picard’s Iterative Method. Uniqueness and Existence Theorems

By the mean value theorem of differential calculus, we get f(x, y2) – f(x, y1) = (y2 – y1)

FG Βf IJ H Βy K

,

where y1 < y < y2,

y% y

...(ii)

where (x, y1) and (x, y2) are assumed in the given range. f ( x, y2 ) f ( x, y1 ) : K y2 y1 , ...(iii) 8 which is Lipschitz condition. It follows that the Lipschitz condition (iii) can be replaced by the stronger condition (i). 1.7. An Important Theorem. If S in either a rectangle | x – x0 | : h, | y – y0 | : k, (h, k > 0) or a strip | x – x0 | : h, | y | < 4 (h > 0), and if f(x, y) is a real valued function defined on S such that Now (i) and (ii)

Β f ( x, y) : K, (x, y) Χ S for a positive constant K, then Βy f(x, y) satisfies a Lipschitz condition on S with Lipschitz constant K. [Meerut 1994] Βf Βy exists, is continuous on S, and

z

y2

Β f ( x, y) dy = Βy

Proof. Now,

f ( x, y1 ) f (x, y2 ) =

Thus,

f ( x, y1 ) f (x, y2 ) = K y1 y2

y1

z

y2

y1

Β f ( x, y) dx : K Βy

z

y2

y1

dy .

(x, y1), (x, y2) Χ S,

for

showing that f(x, y) satisfies Lipschitz condition on S with Lipschitz constant K. 1.8. Solved examples based on Articles 1.4 to 1.7 Ex. 1. Show that f(x, y) = xy2 satisfies the Lipschitz condition on the rectangle R: | x | : 1, | y | : 1 but does not satisfy a Lipschitz condition on the strip S | x | : 1, | y | < 4. [Meerut 2005, 07, Kanpur 2002, Bilaspur 2004] Sol. Method I. We have

f ( x, y2 )

f ( x, y1) = xy22 xy12 , as

y2 ∃ y1

f (x, y2 ) f (x, y1) = x

or

y2 y1 .

Hence in the rectangle | x | : 1, | y | : 1, (1) 8 f ( x, y2 ) showing that Lipschitz condition is satisfied. Next,

f ( x, y2 ) f (x, 0) = x y2 3 4 y2 0

when

f(x, y) = xy2 ...(1)

f ( x, y1) : (1) × (2) × y2 y1 ,

| y2 | 3 4

if

x 9 0,

showing that the Lipschitz condition is not satisfied on the strip x : 1, y < 4. | Βf / Βy | % 2 | xy |% 2 | x | | y |

Method II. We have

... (1)

# In the rectangle | x | : 1, | y | : 1, | Βf / Βy | : 2, for eah ( x, y ) Χ R,

showing that f(x, y) satisfies Lipschitz condition in R, with Lipschitz constant 2. On the other hand, on the strip S :| x | : 1, | y | 7 4, (1) shows that | Βf / Βy | is unbounded on the strip S as | y | 3 4. Hence Lipschitz condition is not satisfied on the strip S. Ex. 2. If S is defined by the rectangle | x | : a, | y | : b, show that the f(x, y) = x2 + y2, satisfies the Lipschitz condition. Find the Lipschitz constant. [Meerut 2000, 05, 07; Himanchal 2003; Rohilkhand 2007] Sol. Let (x, y1) and (x, y2) be two arbitrary points in the rectangle S. Then, we have f (x, y2 ) f (x, y1) =

dx

2

∃ y22

i dx

2

i

∃ y12 , as f(x, y) = x2 + y2

Created with Print2PDF. To remove this line, buy a license at: http://www.software602.com/

Picard’s Iterative Method. Uniqueness and Existence Theorems

1.19

= y22 y12 % y2 ∃ y1 y2 y1 . Thus,

f (x, y2 ) f (x, y1) : 2b y2 y1 , since

| y | < b in S.

showing that the Lipschitz condition is satisfied. Here the Lipschitz constant K = 2b. Ex. 3. Prove that the continuity of f(x, y) is not enough to guarantee the uniqueness of the solution of the initial value problem : dy/dx = f(x, y) = | y | , y(0) = 0. [Himanchal 2003; Lucknow 2006; Meerut 1998] OR Show that the solution of the initial value problem dy/dx = f(x, y), y(x0) = y0 may not be unique although f(x, y) is continuous. dy/dx = | y | ,

Sol. Consider the initial value problem

y(0) = 0. ...(1)

Clearly f(x, y) = | y | is continuous for all y and (1) has the following two solutions : y∆0

Ε< x 2 4, when x Φ 0 y= Γ 2 0, such that

f ( x, y1 ) f ( x, y2 ) : K y1 y2 for all (x, y1), (x, y2) in D. The constant K is known as Lipschitz constant. As a consequence of the definition, a function f(x, y) satisfies Lispschitz condition if and only if there exists a constant K > 0 such that f ( x, y1 ) y1

f ( x, y2 ) y2

: K , y1 9 y2 , whenever (x, y1), (x, y2) belongs to D.

Created with Print2PDF. To remove this line, buy a license at: http://www.software602.com/

Picard’s Iterative Method. Uniqueness and Existence Theorems

1.21

We now state a general criterion which would ensure the Lipschitz condition : Let f(x, y) be a continuous function defined over a rectangle D % ?( x, y ); x x0 : a, y y0 : b ≅. Here a, b are some positive real numbers. Let Βf / Βy be defined and continuous on D and

Βf / Βy

: K , for each ( x, y ) Χ D

for some K > 0. Then f satisfies a Lipschitz conditon on D with Lipschitz constant K. (ii) Let f(x, y) = y1/2 be defined on the rectangle D % ?( x, y ) :| x | : 2, | y | : 2≅ , Let y1 > 0. f ( x, y1 )

Then,

f ( x, 0)

y1 0

%

y11/ 2 1 % 1/ 2 , which is unbounded as y1 3 0. y1 y1

Hence f(x, y) does not satisfy the Lipschitz condition in D. (iii) Let f ( x, y ) % | y | be defined on the square D % ?( x, y) : | x | : 1, | y | : 1≅ . Recall that the partial derivatives of f(x, y) w.r.t. ‘y’ at the point ( x6, y 6) is defined as − Βf . f ( x6, y 6 ∃ k ) % lim / 0 k 3 0 Β y k 1 2( x6, y6)

f ( x 6, y 6)

− Βf . f ( x, 0 ∃ k ) % lim / 0 k 1 Βy 2( x, 0) k 30

# Using the above definition, we get

f ( x, 0)

|k| , k 30 k

% lim

which does not exist. Thus Βf / Βy fails to exist at (x, 0). Again,

f ( x, y1 ) | y1

f ( x, y2 ) y2 |

%

| y1 | | y2 | y1

y2

: 1,

| y1

as

y2 | Φ | y1 | | y2 |

showing that f(x, y) satisfies Lipschitz condition in x on D with Lipschitz constant K = 1. Ex.9. If S is defined by the rectangle | x | : a, | y | : b, show that the function f(x, y) = x sin y + y cos x, satisfy the Lipschitz condition. Find the Lipschitz constant. [Agra 2006; Jabalpur 2005; Kanpur 2002; Meerut 2003] Sol. From the given function,

Βf / Βy % x cos y ∃ cos x

... (1)

Since x, cos y and cos x are continuous functions, Βf / Βy is also continuous on S. Also, we have

| Βf / Βy | % | x cos y ∃ cos x | : | x cos y | : | cos x |

or | Βf / Βy | % | x | | cos y | ∃ | cos x | : | x | ∃1 : a ∃ 1 for each ( x, y ) Χ S , [ | x | < a] showing that f(x, y) satisfies Lipschitz condition and Lipschitz constant is a + 1. Ex. 10. Show that the function f(x, y) = y2/3 does not satisfy the Lipschitz condition on the rectangle R :| x | : 1, | y | : 1. Sol. We have Βf / Βy % 2 / 3 y1/ 3 , which is unbounded in every neighbourhood of the origin. Hence f(x, y) does not satisfy the Lipschitz condition f (0, y )

f (0, 0)

%

y2/3 0

%

1

, y 0 y | y |1/3 which is unbounded in every neighbourhood of the origin and so f(x, y) does not satisfy the Lipschitz condition.

Second method. We have

Created with Print2PDF. To remove this line, buy a license at: http://www.software602.com/

1.22

Picard’s Iterative Method. Uniqueness and Existence Theorems

Ex. 11. Give an example to prove that we cannot drop the Lipschitz condition in the statement of Picard’s theorem. Sol. Consider the problem dy/dx = 3 y2/3, y(0) = 0 ... (1) and let R be the rectangle | x | : 1, | y : 1| . Clearly f(x, y) = 3 y2/3, which is continuous on R. We easily verify that y1(x) = x3 and y2(x) = 0 are two distinct solutions of (1) valid for all x. Thus (1) has solution that is not unique. The reason for this non uniqueness lies in the fact that f (x, y) does not satisfy a Lipschitz condition on the rectangle R, since f (0, y )

f (0, 0)

y 0

%

3y2 / 3 3 % y | y |1/ 3

is unbounded in every neighbourhood of the origin. Ex. 12. Show that the f(x, y) = 4x2 + y2 on R : | x | : 1, | y | : 1 satisfies Lipschitz condition.

Βf / Βy % | 2 y | : 2,

Sol. Since

( x, y ) Χ R.

for each

Hence f(x, y) satisfies the Lipschitz condition with Lipschitz constant 2. Ex. 13. Consider f(x, y) = x 3| y |. Prove that f satisfies a Lipschitz condition on R :| x | : 2, | y | : 2 even though Βf / Βy does not exist at (x, 0) if x 9 0 . Sol. By definition of partial derivative, we have − Βf . f ( x, k ) f ( x, 0) x3 | k | % lim % lim , / 0 k 30 k k 1 Βy 2( x ,0) k 30 which does not exist. However, we have f ( x, y2 ) % x 3 | y1 |

f ( x , y1 )

| y1 | | y2 |

Also, we have

8

(1) and (2)

# f ( x, y1 )

x 3 | y2 | % | x3 | & | y1 | | y2 |

f ( x, y1 )

f ( x, y2 ) : 8 y1

f ( x, y2 )

:

y1 3

... (1)

y2

: | x | y1

... (2) y2

y2 , which is true for all (x, y1), (x, y2) in R.

... (3)

(3) shows that f(x, y) satisfies Lipschitz condition with Lipschitz constant 8. Ex. 14. For the initial value problem dy/dx = y2 + cos2x, y(0) = 0, determine the interval of existence of its solution given that R is the rectangle containing origin,

R : ?( x, y) :

0 : x : a,

| y |: b,

a Ι 1/ 2,

b Ι 0≅

2 2 2 2 2 Sol. Let f(x, y) = y2 + cos2x Also, f ( x, y ) % y ∃ cos x : | y | ∃ | cos x | : b ∃ 1

b2 + 1 = M

Let

so that

| f ( x, y) | : M

Again, since Βf / Βy % | 2 y |% 2b % K (say), we see that f(x, y) satisfies Lipschitz condition We find that y(x) exists for Now,

b 1∃ b

2

%

0 : x : h % min(a, b / M ) % min(a, b /(1 ∃ b2 ))

1 1 % 1/ b ∃ b 1/ b b

Κ

Λ

2

, ∃2

8

The maximum value of

... (1) b 1 ∃ b2

is

1 2

Hence (1) 8 h = 1/2 and so y(x) exists on the interval 0 : x : 1/ 2.

Created with Print2PDF. To remove this line, buy a license at: http://www.software602.com/

Picard’s Iterative Method. Uniqueness and Existence Theorems

1.23 2

Ex. 15. Consider the initial value problem dy/dx = y , y(0) = 2. Let R be the rectangle R :{( x, y ) :| x | : a, y 2 : b, a Ι 0, b Ι 0}. Find the largest interval of existence of its solution. By explicitly solving the given initial value problem, show that there exists an interval of existence which is larger than that obtained by the application of Picards’ method. Sol. Given dy/dx = y2, y(0) = 2 ... (1) To find the interval of existence by application of Picard’s theorem: Let f(x, y) = y2. Then, in R, we have | f ( x, y) | % | y 2 |: (b ∃ 2)2 % M , say and hence the interval of existence of solution is given by

| x | : h,

where

Κ

Λ

h % min a, b /(b ∃ 2)2 % 1/ 8.

The interval of existence is (1/ 8) : x : (1/ 8). 8 To find the interval of existence by solving (1). We have dy/dx = y2 so that y–2dy = dx. Integrating, –y–1 = x + c, c being an arbitrary constant. ... (2) 1 1 2 , Putting x = 0 and y = 2, (2) gives –2–1 = c. Then (2) gives %x or y ( x) % 1 2x y 2 showing that y(x) exists on 4 7 x 7 1/ 2. This interval of existence is much larger than that obtained by the application of Picard’s method. Ex. 16. The Picard’s theorem assumes Lipschitz condition. Can we drop this condition. If answer is no, give an example to illustrate this point. Sol. In the following two examples, we shall show that if Lipschitz condition is not satisfied then we do not arrive at unique solution as stated in Picard’s theorem. Consider dy/dx = 4y3/4, y(0) = 0, ... (1) xΦ0 f ( x, y ) f ( x, 0) 4 y 3 / 4 4 % % 1/ 4 , y 9 0 y 0 y y

Let f(x, y) = 4y3/4. Then, we get

which is unfounded for x Φ 0, since it can be made as large as possible by choosing y close to zero. Thus f(x, y) fails to satisfy Lipschitz condition. Using Picard’s method of successive approximation (refer Art. 1.2), we easily see that yn(x) = 0 for n = 0, 1, 2, 3..... Hence y( x) % lim yn ( x) % 0 on [0, 4 [. We also not that y(x) = x4 is n 34

also solution of (1). Hence (1) does not possess unique solution.

EXERCISE 1(C) Ex. 1 (a) Show that the function f given by f(x, y) = y1/2 does not satisfy a Lipschitz condition on R : | x | : 1, 0 : y : 1 . Show that f satisfies a Lipschitz condition on any rectangle R of the form R : | x |: a, b : y : c, (a, b, c Ι 0) (b) Show that a function f given by f(x, y) = x2 | y | satisfies a Lipschitz condition on R : | x | : 1, | y | : 1 (c) Show that f(x, y) = xy2 (i) satisfies a Lipschitz condition on any rectangle a : x : b, c : y : d . (ii) does not satisfy a Lipschitz condition on any strip a : x : b, 4 7 y : 4 (d) Show that f(x, y) = xy (i) satisfies Lipschitz condition on any rectangle a : x : b, c : y : d (ii) satisfies a Lilpschitz condition on any strip a : x : b, Lipschitz condition on the entire plane.

4 7 y 7 4 (iii) does not satisfy a

Created with Print2PDF. To remove this line, buy a license at: http://www.software602.com/

1.24

Picard’s Iterative Method. Uniqueness and Existence Theorems

(e) For what points (x0, y0) does Picard’s theorem imply that the initial value problem dy/dx = y | y |, y(x0) = y0 has a unique solution on some interval | x x0 | : h ?

[Ans. All point (x0, y0)]

(f) Consider the initial value problem : dy / dx % 2 y / x, if x > 0 and dy/dx = 0, if x = 0; y(0) = 0. Show that 2y/x does not satisfy Lipschitz condition in any closed rectangle containing (0, 0) and the given initial value problem has solution which is not unique. Sol. Left as an exercise for the reader. Ex. 2. By computing appropriate Lipschitz constants, show that the following functions satisfy Lipschitz conditions on the domain D of xy–plane indicated. (i) f(x, y) = 4x2 + y2, 2

on

2

2

(ii) f(x, y) = x cos y + y sin x, (iii) f ( x, y ) % x3 e

xy

2

,

on

D : | x | : 1,

| y |:1

D : | x | : 1,

on

| y|74

D : 0 : x : a, | y | 7 4, (a Ι 0)

2

(iv) f(x, y) = a(x) y + b(x) y + c(x),

on

D : | x | : 1,

| y | : 2,

where a(x), b(x) and c(x) are continuous functions on | x | : 1. (v) f(x, y) = a(x) y + b(x), on D : | x | : 1, | y | 7 4, where a(x) and b(x) are continuous on | x | < 1 Ans. (i) K = 2 (ii) K = 3 (iii) K = max {2a3, 2a4} (iv) K = 4 Ma + Mb; where M a % max | a( x) |, M b % max | b( x) | (v) K % max | a ( x ) | | x| : 1

| x | :1

| x| : 1

3. Let (x0, y0) be an interior point of a closed rectangle R, a : x : b, c : y : d in which f(x, y) is continuous. Let f(x, y) satisfy the Lipschitz condition. | f(x, y1) – f(x, y2)| : K (y1 – y2), for all possible (x, y1) and (x, y2) in R and some fixed constant K. Prove that dy/dx = f(x, y), y(x0) = y0 has a unique solution. 4. Write a note on Picard’s existence theorem regarding the existence and uniquencess of the solution of the equation dy/dx = f(x, y), where f satisfies Lipschitz condition. 5. (a) Show that (i) | y6 | + | y | = 0, y(0) = 1 has no solution. (ii) y6 = x, y(0) = 1 has one solution (iii) y6= (y – 1)/x, y(0) = 1 has an infinity of solutions. Explain what you mean by existence and uniqueness of a differential equation. (b) State carefully any existence and uniqueness theorem for differential equation you know and apply it to the above three examples. 6. Show that the conditions for the exitence and uniqueness of a solution of the following initial value problem are not satisfied by the function f(x, y) = (y – 1)/x in any rectangle R of xy– plane with (0, 1) as its centre : y 6 = (y – 1)/x, y(0) = 1 ; but a solution does exist of above problem. Give reason for your answer. Draw some possible solution curves. 7. Apply Picard’s iteration process to get the solution of y 6 = (y – 1)/x, y(1) = 1. Give your arguments, why Picard’s iteration process is applicable to the above initial value problem. 8. If f(x, y) = y2/3, show that Lipschitz condition in not satisfied in any region containing the origin and that the solution of the differential equation dy/dx = f(x, y) satisfying the initial condition y = 0 when x = 0 is not unique. [Meerut 1999] 9. By giving suitable example prove that a function does not satisfy the Lipschitz condition on the prescribed domain. [Madurai 2001, 05] 10. Examine existence and uniqueness of the solution of the initial value problem dy/dx = y2, y(1) = –1. [Meerut 2011] 11. Show that the Picard’s theorem ensures a unique solution in the interval x : 1/2 for the initial value problem dy/dx = x + y2, y(0) = 0.

Created with Print2PDF. To remove this line, buy a license at: http://www.software602.com/

Picard’s Iterative Method. Uniqueness and Existence Theorems

1.25

12. State Picard’s theorem on existence of solutions of differential equations. Show that f(x, y) = y 1/2 does not satisfy Lipschtiz’s condition on x : 1, 0 : y : 1 while f(x, y) = x2 cos2 y + y sin2 x satisfies Lipschitz’s condition on x : 1, | y | < 4. Find the Lipschitz’ss constant. [I.A.S. 2000] 13. Examine the exitence and uniqueness of solution of the initial value problem dy/dx = y1/3, y(0) = 0 [Himanchal 2003; Kanpur 2000; Kolkata 2000, Osmania 2005, 07; G.N.D.U. Amritsar 2003, 05; Meerut 2000] 14. Show that for the problem dy/dx = y, y(0) = 1, the constant h in Picard’s theorem must be smaller than unity. [Rohilkhand 2007; Meerut 1995] 15. Discuss the existence and uniqueness of a solution of the initial value problem dy/dx = y4/3, y(x0) = y0. [Bhopal 2010; Rajasthan 2010; Himachal 2010] 16. Examine existence and uniqueness of the solution of the initial value problem dy/dx = y2, y(1) = –1. [ Gwalior 2000, 01, 03; Ujjain 2000, 02, 04, Rajasthan 2000; Agra 1997; Kanpur 2000; Meerut 1994, 96] 17. State and prove the uniqueness theorem for the initial value problem dy/dx = f(x, y), y(x0) = y0. [Meerut 1998] 18. Examine whether the following differential equation possesses unique solution. Justify your answer.

dy Ε y (1 2 x), x Ι 0 subject to the condition: y = 1 at x = 1 %Γ dx Η y (2 x 1), x 7 0

[Jabalpur 2003]

19. Verify that the initial value problem xy 6 y % 0, y(0) = 0 has two solutions y1(x) = 0 and y2(x) = x. Does it contradict the Picard’s theorem. [Himanchal 2002, 03, 05] 20. Let f ( x, y) % (cos y ) /(1 x 2 ), (| x |7 1). Show that f satisfied a Lipschitz condition on every strip Sa : | x | : a, , where 0 < a < 1.

[Himanchal 2009]

21. Define Lipschitz condition (with respect to y) of the function f ( x, y ) and investigate its geometrical significance. Show that the function f ( x, y ) % x 2e x ∃ y satisfies Lipschitz condition in the rectangle defined by | x | : a, | y | : b .

[Himanchal 2009]

22. State and prove Picard’s existence theorem for the solution of differential equation dy/dx = f (x, y), y(x0) = y0. Show that dy/dx = x2 + y2, y (0) = 0 has a unique solution. [Himanchal 2009]

Created with Print2PDF. To remove this line, buy a license at: http://www.software602.com/

Picard’s Iterative Method. Uniqueness and Existence Theorems

1.29

Created with Print2PDF. To remove this line, buy a license at: http://www.software602.com/

2 Simultaneous Equations of the Form (dx)/P = (dy)/Q = (dz)/R 2.1. Introduction. In this chapter we shall study simultaneous equations of the first order and of the first degree in the derivatives. Equations containing only three variables will be studied. It will be noted that the method of solution presented here can be applied to equations involving any number of variables. The general type of a set of simultaneous equations of the first order having three variables is P1dx + Q1dy + R1dz = 0 and P2dx + Q2dy + R2dz = 0, ...(1) where the coefficients are functions of x, y, z. Solving these equations simultaneously, we have dx Q1 R2 ! Q2 R1

dy R1 P2 ! R2 P1

dz , P1Q2 ! P2 Q1

which is of the form (dx)/P = (dy)/Q = (dz)/R, ...(2) where P, Q, and R are functions of x, y, z. Thus we note that the simultaneous equations (1) can always be put in the form (2). 2.2. The nature of solution of (dx)/P = (dy)/Q = (dz)/R. The given equations are said to be completely solved when we get a solution of the form u1(x, y, z) = c1 and u2 (x, y, z) = c2, where u1 and u2 are two independent integrals (solutions) of the given equations. u1 and u2 are said to be independent integrals if u1/u2 is not merely a constant. For example, u1 = x2 + y2 + z2 and u2 = x + y + z are independent integrals whereas u1 = 2x + 2y + 2z and u2 = 2(x + y + z) are not independent. 2.3. Geometrical Interpretation of (dx)/P = (dy)/Q = (dz)/R. From three dimensional coordinate geometry, it is known that the direction cosines of the tangent to a curve are proportional to dx: dy: dz. The given differential equations, therefore, express the fact that the direction cosines of the tangent to the curve at that point are proportional to P : Q : R. Suppose that the solution of the given equations is given by u1(x, y, z) = c1 and u2(x, y, z) = c2. Then we observe that the solution represents the curves of intersection of the surfaces u1(x, y, z) = c1 and u2(x, y, z) = c2. Since c1 and c2 can take any values in infinite number of ways, we get a doubly infinite number of such curves. 2.4. Rule I for solving (dx)/P = (dy)/Q = (dz)/R. ...(1) By equating two of the three fractions of (1), we may be able to get an equation in only two variables. Sometimes such an equation is obtained after cancellation of some factor from the chosen two fractions of (1). On integrating the differential equation in only two variables by well known methods, we shall obtain one of the relations in the general solution of (1). This method may be repeated to give another relation with help of two other fractions of (1). 2.5. Solved examples based on Art. 2.4. dy dz Ex. 1. Solve (a) xdx . [Nagpur 1996; Bangalore 2005] 2 2 y z xz y 2.1

Created with Print2PDF. To remove this line, buy a license at: http://www.software602.com/

2.2

Simultaneous Equations of the Form cx/P = dy/Q = dz/R

(b)

xdx 2

y z

dy xz

dz

[Poona 2006; Vikram 1996]

y2 z

Sol. (a) Taking the first two fractions, we get x2dx = y2dy 2 2 or 3x dx – 3y dy = 0 and so x3 – y3 = c1. ...(1) Next, taking the first and the third fractions, we get xdx = zdz or 2xdx – 2zdz = 0 and so x2 – z2 = c2. ...(2) 3 3 2 2 Since x – y and x – z are independent, the required general solution is given by the relations (1) and (2), c1 and c2 being arbitrary constants. (b) Proceed as in part (a). Ans. x3 – y3 = c1, x2 – 2z = c2 Ex. 2. Solve dx yz

dy zx

dz . xy

[Agra 1996; Delhi Maths (G) 1998]

Sol. Taking the first two fractions, we have xdx = ydy or 2xdx – 2ydy = 0 so that Again, taking the first and the third fractions, we have xdx = zdz or 2xdx – 2zdz = 0 so that The required general solution is given by the relations (1) and (2). Ex. 3. (dx)/x = (dy)/0 = (dz)/(–x) Sol. From the second fraction, we have dy = 0 so that Taking the first and the third fractions, xdx + zdz = 0 or 2 2 Integrating, x + z = c2, c2 being an arbitrary to constant. The required solution is given by the relations (1) and (2).

dy x2

Ex. 4. Solve dx2 y

dz . x y z

x2 – y2 = c1. ...(1) x2 – z2 = c2. ...(2)

y = c1. ...(1) 2xdx + 2zdz = 0. ...(2)

[Delhi Maths (G) 2000]

2 2 2

Sol. Taking the first two fractions, 3x2dx – 3y2dy = 0. Integrating, x3 – y3 = c1, being an arbitrary constant Taking the first and third fractions, 3x2dx – 3z–2dz = 0. 3 –1 Integrating, x + 3z = c2, c2 being an arbitrary constant The required general solution is given by the relations (1) and (2).

dy dx dz . [Delhi Maths (Hons.) 2 ! xy xz x # 2y Sol. Taking the last two fractions, (1/y)dy + (1/z)dz = 0. Integrating, log y + log z = log c1 or yz = c1. Again, taking the first two fractions, we have 2 2 2 dx 2 dx = x # 2 y x 2 x # x 2 !4 y or or x dx = ! ! 2y dy ! xy dy y dy y 2 Putting x = v so that 2x(dx/dy) = dv/dx, (2) reduces to (dv/dx) + (2/y)v = –4y, which is linear differential equation Ex. 5. Solve

2

Integrating factor of (3) = e vy2 =

z

%{(!4 y) ∃ y } ∃ dy # c 2

( 2 / y ) dy

2

...(1) ...(2) 1994] ...(1) ... (2) ...(3)

= e2 log y = y2 and so its solution is or

x2y2 + y4 = c2.

...(4)

The required general solution is given by the relations (1) and (4).

Created with Print2PDF. To remove this line, buy a license at: http://www.software602.com/

Simultaneous Equations of the Form cx/P = dy/Q = dz/R

2.3

EXERCISE 2 (A) Solve the following simultaneous differential equations : 1. dx = dy = dz Ans. x – y = c1, x – z = c2 2. (dx)/a = (dy)/a = dz Ans. x – y = c1, y – az = c2 3. (dx)/x = (dy)/y = (dz)/z Ans. x/y = c1, x/z = c2 4. (dx)/tan x = (dy)/tan y = (dz)/tan z Ans. (sin x)/(sin y) = c1, (sin x)/(sin z) = c2 5. dx = dy = (dz)/sin x Ans. x – y = c1, z + cos x = c2 2.6. Rule II for solving (dx)/P = (dy)/Q = (dz)/R. ...(1) Suppose only one relation u1(x, y, z) = c1 can be found by using rule I of Art 2.4. Then, sometimes we try to use this relation in expressing one variable in terms of the others. This may help us to obtain an equation in two variables. The solution of this equation will give us second relation for the general solution of (1). Note that the second relation will involve the arbitrary constant c1. To find the final form the second relation, the arbitrary constant c1 must be removed with help of the first relation u1(x, y, z) = c1. 2.7. Solved examples based on Art. 2.6 Ex. 1. Solve

dy ! yz( z 2 # xy)

dx xz( z # xy) 2

dz . x4

[Delhi Maths (H) 2001]

Sol. Cancelling z(z2 + xy), the first two fractions give (1/x)dx = – (1/y)dy or Integrating, log x + log y = log c1 or Using (1), the first and third fractions give x4dx = xz(z2 + c1)dz or 1 4

Integrating,

x4 !

d

1 4

i

z 4 # 12 c1z 2 =

1 c 4 2

(1/x)dx + (1/y)dy = 0, xy = c1. ...(1) x3dx – (z3 + c1z)dz = 0. x4 – z4 – 2c1z2 = c2.

or

Using (1) to remove c1, we get x4 + z4 – 2xyz2 = c2. The complete solution is given by the relations (1) and (2). Ex. 2. Solve dx xy

dy 2 y

dz . 2 zxy ! 2 x

[Rohilkhand 1993]

Sol. Taking the first two fractions, (1/x)dx – (1/y)dy = 0 From (1), x = c1y. So the second and third fractions give dy dz = 2 y c1zy 2 ! 2c12 y 2

...(2)

or

so that

x/y = c1. c1dy =

dz . z ! 2c12

Integrating, c1y – log (z – 2c21) = c2, c1 and c2 being arbitrary constants. Using (1) to remove c1, (2) gives x – log (z – 2x2/y2) = c2. The complete solution is given by the relations (1) and (3). Ex. 3. Solve

dx 1

dy 2

dz . 5 z # tan( y ! 2 x)

...(1)

...(2) ...(3)

(Delhi Maths (H) 2005; Mumbai 2007)

Sol. Taking the first two fractions, dy – 2dx = 0. Integrating, y – 2x = c1, c1 being an arbitrary constant. ...(1) Using (1), the first and the third fractions give dx = dz/(5z + tan c1) so that x – (1/5) × log (5z + tan c1) = c2/5.

Created with Print2PDF. To remove this line, buy a license at: http://www.software602.com/

2.4

Simultaneous Equations of the Form cx/P = dy/Q = dz/R

Using (1) to remove c1, this gives on simplification 5x – log [5z + tan (y – 2x)] = c2, c2 being an arbitrary constant. The complete solution is given by the relations (1) and (2). dy x

Ex. 4. Solve dx y

dz . 2 2 xyz ( x ! y ) 2

Sol. Taking the first two fractions, 2xdx – 2ydy = 0. Integrating, x2 – y2 = c1, c1 being an arbitrary constant Using relation (1), the first and third fractions give

dz dx = y xyz 2 c1

or

...(2)

...(1)

2c1xdx – 2z–2dz = 0.

or

Integrating, c1x2 + 2z–1 = c2 or (x2 – y2)x2 + 2z–1 = c2, by (1) ...(2) The required general solution is given by the relations (1) and (2). Ex. 5. Solve (dx)/(xz) = (dy)/(yz) = (dz)/(xy). Sol. Taking the first two fractions, (1/x)dx – (1/y)dy = 0. Integrating, log x – log y = log c1 or x/y = c1. ...(1) From the second and third fractions, xdy = zdz. 2c1ydy = 2zdz [ from (1), x = c1y] 2 2 Integrating, c1y – z = c2 or (x/y) × y2 – z2 = c2, using (1). 2 Thus, xy – z = c2, c2 being an arbitrary constant ...(2) The required general solution is given by the relations (1) and (2). Ex. 6. Solve

dx 2 ! xy

dy 3 y

dz . axz

Sol. Taking the first two fractions, Integrating, log x + log y = log c1 From (1), x = c1/y. Hence the last two fractions give dz dy = 3 az ∃ (c1 / y ) y

(1/x)dx + (1/y)dy = 0. or xy = c1.

...(1)

dz – ac y–4dy = 0. 1 z

or

Integrating, log z – (ac1) × [(y–3/(–3)] = c2, c2 being an arbitrary constant. Using (1), we get log z + (axy)/3y3 = c2 or log z + (ax/3y2) = c2. ...(2) The required general solution is given by the relations (1) and (2).

EXERCISE 2 (B) Solve the following simultaneous differential equations : 1. Solve

dx 1

dt !2

2. Solve

dx z

dy !z

dz 2

3 x sin( y # 2 x ) dz 2

z # ( x # y )2

(Gulbarga 2005) Ans. y + 2x = c1, x3 sin (y + 2x) – z = c2

.

dx dy dz 1 !1 z /( x # y ) 4. (dx)/(zx) = (dy)/(–zy) = (dz)/(z + xy)

3. Solve

Ans. x + y = c1 and z2 + (x + y)2 = c1e2x Ans. x + y = c1 and x – (x + y) log z = c2 [Delhi Maths (G) 2006] Ans. xy = c1, log x + xy log (z + xy) – z = c2

Created with Print2PDF. To remove this line, buy a license at: http://www.software602.com/

Simultaneous Equations of the Form cx/P = dy/Q = dz/R

2.5

2.8. Rule III for solving (dx)/P = (dy)/Q = (dz)/R ...(1) Let P1, Q1, R1 be functions of x, y, z. Then, by a well–unknown principle of algebra, each fraction in (1) will be equal to (P1dx + Q1dy + R1dz) / (P1P + Q1Q + R1R). ...(2) If P1P + Q1Q + R1R = 0 in (2), then we know that numerator of (2) is also zero. This gives P1dx + Q1dy + R1dz = 0 which can be integrated to give u1(x, y, z) = c1. This method may be repeated to get another integral u2(x, y, z) = c2. P1, Q1, R1 are called multipliers. As a special case, these can be constants also. Sometimes only one integral is possible with help of multipliers. In such cases second integral should be obtained by using Rule I of Art. 2.4 or Rule II of Art. 2.6 as the case may be. 2.9. Solved examples based on Art. 2.8 Ex. 1. Solve the simultaneous equations

bdy adx = (b ! c)yz (c ! a)zx

cdz . (a ! b) xy

[Kolkata 2001; Kumaoun 2002; Guwahati 2001, 02; Nagpur 2003, 04; Delhi Maths 1995, 1996; Bangalore 2005; Lucknow 2006] Sol. Choosing x, y, z as multipliers, each fraction of given equations axdx # bydy # czdz xyz[( b ! c ) # ( c ! a ) # ( a ! b)]

=

axdx # bydy # czdz . 0

& ax dx + by dy + cz dz = 0 or 2ax dx + 2by dy + 2cz dz = 0. Integrating, ax2 + by2 + cz2 = c1, c1 being an arbitrary contant. Again choosing ax, by, cz as multipliers, each fraction of the given equations 2

=

2

2

a xdx # b ydy # c zdz xyz[a(b ! c ) # b(c ! a) # c(a ! b)]

2

2

2

a xdx # b ydy # c zdz . 0

& a2x dx + b2y dy + c2z dz = 0 or 2a2x dx + 2b2y dy + 2c2z dz = 0. 2 2 2 2 2 2 Integrating, a x + b y + c z = c2, c2 being an arbitrary constant ...(2) The complete solution is given by relations (1) and (2). Ex. 2. Solve :

dx z( x # y )

dy z ( x ! y)

dz . 2 x #y 2

[Delhi Maths (H) 2009]

[Agra 2005; Gauhati 1996, Meerut 2006, 11; Kanpur 2002; Rajasthan 2003] Sol. Choosing, x, –y, –z as multipliers, each fraction of the given equations =

xdx ! ydy ! zdz 2 2 xz( x # y ) ! yz( x ! y) ! z( x # y )

xdx ! ydy ! zdz . 0

& x dx – y dy – z dz = 0 or 2x dx – 2y dy – 2z dz = 0. Integrating, x2 – y2 – z2 = c1, c1 being an arbitrary constants Now choosing, y, x, –z as multipliers, each fraction of the given equations =

ydx # xdy ! zdz 2 2 yz( x # y) # xz( x ! y) ! z( x # y )

ydx # xdy ! zdz . 0

& 2y dx + 2x dy – 2z dz = 0 or 2d(xy) – d(z2) = 0. 2 Integrating, 2xy – z = c2, c2 being an arbitrary constant The complete solution is given by the relations (1) and (2). Ex. 3. Solve

dx mz ! ny

dy nx ! lz

dz . ly ! mx

...(2)

[Bangalore 1997, Delhi Maths (H) 2001;

Lucknow 2002; Karnataka 2004; Mysore 2005; Rajasthan 2006]

Created with Print2PDF. To remove this line, buy a license at: http://www.software602.com/

2.6

Simultaneous Equations of the Form cx/P = dy/Q = dz/R

Sol. Choosing l, m, n as multipliers, each fraction of given equations ldx # mdy # ndz l( mz ! ny ) # m( nx ! lz ) # n(ly ! mx )

=

ldx # my # ndz . 0

Thus, l dx + m dy + n dz = 0 so that lx + my + nz = c1. ...(1) Similarly, choosing x, y, z as multipliers, each fraction of the given equations xdx # ydy # zdz x ( mz ! ny ) # y( nx ! ly) # z(ly ! mx )

=

xdx # ydy ! zdz . 0

& x dx + y dy + z dz = 0 or 2x dx + 2y dy + 2z dz = 0. 2 2 2 Integrating, x + y + z = c2, c2 being an arbitrary constant. ...(2) The complete solution consists of (1) and (2). Ex.4. Solve

dx x( y 2 ! z 2 )

dy y( z 2 ! x 2 )

dz . [Bangalore 1993,Delhi Maths (Hons.)2006] z( x 2 ! y 2 )

Sol. Choosing x, y, z as multipliers, each fraction of the given equations =

xdx # ydy # zdz 2 2 2 2 2 2 x ( y ! z ) # y (z ! x ) # z ( x ! y ) 2

2

xdx # ydy # zdz . 0

2

& x dx + y dy + z dz = 0 or 2x dx + 2y dy + 2z dz = 0. Integrating, x2 + y2 + z2 = c1, c1 being an arbitrary constant. ...(1) Again choosing 1/x, 1/y, 1/z as multipliers, each fraction of the given equations =

dx / x # dy / y # dz / z 2 2 2 2 2 2 ( y ! z ) # (z ! x ) # ( x ! y )

dx / x # dy / y # dz / z . 0

& or

dx/x + dy/y + dz/z = 0. so that log x + log y + log z = log c2 log xyz = log c2 or xyz = c2, c2 being an arbitrary constant ...(2) The complete solution is given by the relations (1) and (2). Ex. 5. Solve (dx)/y = (dy)/(–x) = (dz)/(bx – ay). Sol. Taking a, b, 1 as multipliers, each fraction of the given equations. (a dx # b dy # dz ) / 0 ax # by # z

Integrating,

c1 , c1 being an arbitrary constant.

From first two fractions, x dx + ydy = 0 The required solution is given by (1) and (2). Ex. 6. Solve

or

a dx # b dy # dz 0 .

so that

dy y#z

xdx 2 z ! 2 yz ! y 2

dz . y!z

so that

... (1) 2

2

x + y = c2 ... (2)

[Bangalore 2007, Rohilkhand 1997]

Sol. Choose 1, y, z as multipliers, each fraction of the given equations = (xdx + ydy + zdz)/0 so that 2x dx + 2y dy + 2z dz = 0. Integrating, x2 + y2 + z2 = c1, c1 being an arbitrary constant. ...(1) From the last two fractions, (y – z)dy = (y + z)dz 2(ydz + zdy) – 2ydy + 2zdz = 0 or 2d(yz) – d(y2) + d(z2) = 0. 2 2 Integrating, 2yz – y + z = c2, c2 being an arbitrary constant. ...(2) The required complete solution is given by (1) and (2). Ex. 7. Solve

dx 4 y x ! 2x 3

dy 3 2y ! x y 4

dz . 3 9 z( x ! y ) 3

Created with Print2PDF. To remove this line, buy a license at: http://www.software602.com/

Simultaneous Equations of the Form cx/P = dy/Q = dz/R

2.7

Sol. Choosing 1/x, 1/y, 1/3z as multipliers, each fraction of the given equations = or

dx / x # dy / y # dz / 3z . 0

& (1/x)dx + (1/y)dy + (1/3z) dz = 0 so that log x + log y + (1/3) × log z = log c1 xyz1/3 = c1, c1 being an arbitrary constant. ...(1) Now the first two fractions give (2y4 – x3y)dx = (y3x – 2x4)dy. Dividing by x3y3,

or

dx / x # dy / y # dz / 3z ( y 3 ! 2 x 3 ) # (2 y 3 ! x 3 ) # 3( x 3 ! y3 )

F 2 y ! 1 I dx = F 1 GH x y JK GH x 3

I JK

! 2 x3 dy or y

2

2

FG 1 dy ! 2 y dxIJ # FG 1 dx ! 2 x dyIJ = 0. Hx K Hy x y K 2

3

2

3

d(y/x2) + d(x/y2) = 0. so that y/x2 + x/y2 = c2. ...(2) The required solution is given by the relations (1) and (2). Ex. 8. Solve (dx)/y2 = (dy)/x2 = (dz)/x2y2z2. [Mysore 2004] Sol. First two fractions give 3x2dx – 3y2dy = 0 so that x3 – y3 = c1. ...(1) Choosing x2, y2, –2/z2 as multipliers, each fraction of the given equations = 2

x2 y 2 # x2 y 2 ! 2 x2 y 2

x 2 dx # y 2dy ! (2 / z 2 )dz . 0

& x dx + y dy – (2/z )dz = 0 or 3x2dx + 3y2dy – (6/z2)dz = 0. 3 3 Integrating, x + y + 6/z = c2, c2 being an arbitrary constant. ...(2) (1) and (2) together give the complete solution. Ex. 9.

2

x 2 dx # y 2 dy ! (2 / z 2 ) dz

dx 2 x( y # z)

2

dy 2 ! y( x # z )

dz 2 . z( x ! y )

[Meerut 2007]

2

Sol. Choosing 1/x, 1/y, 1/z as multipliers, each fraction of the given equations =

(1/ x ) dx # (1/ y )dy # (1/ z )dz 2

2

2

y # z ! (x # z) # x ! y

2

(1/ x )dx # (1/ y ) dy # (1/ z )dz . 0

& (1/x)dx + (1/y)dy + (1/z)dz = 0 so that xyz = c1. Next choosing x, y, –1 as multipliers, each fraction of the given equations

...(1)

xdx # ydy ! dz xdx # ydy ! dz 2 2 2 2 0 x ( y # z ) ! y ( x # z ) ! z( x ! y ) 2 2 & 2xdx + 2ydy – 2dz = 0 so that x + y – 2z = c2. ...(2) (1) and (2) together give the complete solution.

=

2

2

dy dx dz . [Delhi Maths (G) 1994] z!y x!z y!x (b) Show that u = x + y + z, v = x2 + y2 + z2 are integrals of the linear system dx/dt = y – z, dy/dt = z – x, dz/dt x – y. [Amravati 2003] Sol. (a) Choosing 1, 1, 1 as multipliers, each fraction of the given equations

Ex. 10. (a) Solve

= & dx + dy + dz = 0 so that Again, choosing x, y, z as multipliers, each fraction = & xdx + ydy + zdz = 0

dx # dy # dz dx # dy # dz . z!y#x!z#y!x 0 x + y + z = c1. ...(1)

xdx # ydy # zdz xdx # ydy # zdz . x ( z ! y) # y( x ! z ) # z ( y ! x ) 0 or 2xdx + 2ydy + 2zdz = 0.

Created with Print2PDF. To remove this line, buy a license at: http://www.software602.com/

2.8

Simultaneous Equations of the Form cx/P = dy/Q = dz/R 2

2

Integrating, x + y + z2 = c2, c2 being arbitrary constant. ...(2) The required solution is given by the relations (1) and (2). (b) Given system yields (dx)/(y – z) = (dy)/(z – x) = (dz)/(x – y). Now proceed as in part to show that x + y + z = c1 and x2 + y2 + z2 = c2. Hence u = x + y + z and v = x2 + y2 + z2, as required. yzdx zxdy xydz . [Delhi Maths (G) 1996] y!z z!x x!y Sol. Choosing 1, 1, 1 as multipliers, each fraction of the given equations

Ex. 11. Solve

=

yzdx # zxdy # xydz y!z#z!x#x!y

d ( xyz ) . 0

& d(xyz) = 0 so that xyz = c1. Again choosing 1/yz, 1/zx, 1/xy as multipliers, each fraction of the given equations =

...(1)

dx # dy # dz dx # dy # dz = ( y ! z ) / yz # ( z ! x ) / zx # ( x ! y) / xy 1/ z ! 1/ y # 1/ x ! 1/ z # 1/ y ! 1/ x

dx # dy # dz 0 & dx + dy + dz = 0 so that x + y + z = c2. ...(2) The required general solution is given by the relations (1) and (2).

Ex. 12. Solve

dx 2 x( y ! z ) 2

dy 2 ! y( z # x ) 2

dz 2 . z( x # y )

[Delhi Maths (H) 1993]

2

Sol. Choosing x, y, z as multipliers, each fraction of the given equations =

xdx # ydy # zdz x2 ( y 2 ! z2 ) ! y2 (z2 # x 2 ) # z2 ( x2 # y 2 )

xdx # ydy # zdz 0

& xdx + ydy + zdz = 0 so that x2 + y2 + z2 = c1. Again, choosing 1/x, –1/y, –1/z as multipliners, each fraction =

or

(1 / x )dx ! (1 / y)dy ! (1 / z )dz 2 2 2 2 2 2 ( y ! z ) # (z # x ) ! ( x # y )

...(1)

(1 / x )dx ! (1 / y )dy ! (1 / z )dz . 0

& (1/x)dx – (1/y)dy – (1/z)dz = 0 so that log x – log y – log z = log c2 log (x/yz) = log c2 or x/yz = c2. ...(2) The required general solution is given by the relations (1) and (2). Ex. 13. Solve

dx y ! zx

dy x # yz

dz . x 2 # y2

[Nagpur 1996, Delhi Maths (H) 1998]

Sol. Choosing x, –y, z as multipliers, each fraction of the given equations =

xdx ! ydy # zdz 2 2 x ( y ! zx ) ! y( x # yz ) # z( x # y )

xdx ! ydy # zdz 0

& xdx – ydy + zdz = 0 so that 2xdx – 2ydy + 2zdz = 0. Integrating it, x2 – y2 + z2 = c1, c1 being an arbitrary constants. ...(1) Again, choosing y, x, –1 as multipliers, each fraction of the given equation =

y dx # x dy ! dz 2

2

y ( y ! zx) # x( x # yz ) ! ( x # y )

y dx # x dy ! dz . 0

& y dx + x dy – dz = 0 or d(xy) – dz = 0. Integrating it, xy – z = c2, c2 being an arbitrary constant. The required general solution is given by the relations (1) and (2).

...(2)

Created with Print2PDF. To remove this line, buy a license at: http://www.software602.com/

Simultaneous Equations of the Form cx/P = dy/Q = dz/R

Ex. 14. Solve

dx

dy

2

2

x

y

2.9

dz . nxy

[Delhi Maths (G) 1995] x–2dx = y–2dy.

Sol. Taking the first two fractions,

1 1!1 = ! 1 ! c1 or = c1 x x y y y – x = c1xy or x – y + c1xy = 0. Choosing 1/x, –1/y, c1/n as multipliers, each fraction of the given equations !

Integrating or

=

(1 / x)dx ! (1 / y)dy # (c1 / n)dz x ! y # c1xy

& Integrating, or

...(1)

(1 / x)dx ! (1 / y)dy # (c1 / n)dz , using (1) 0

(1/x)dx – (1/y)dy + (c1/n)dz = 0. log x – log y + (c1/n)z = (c1/n)c2, c2 being arbitrary constant

c1 c y y nxy y z = 1 c2 # log or z = c2 # n log c2 # log , using (1) n c1 x y!x x n x The required general solution is given by the relations (1) and (2).

Ex. 15. Solve

dy ! xy

dx 2 y #z 2

dz . ! xz

[Delhi Maths. (H) 1995]

Sol. Taking the last two fractions, Integrating, log y – log z = log c,

or

Choosing x, y, z or multipliers, each given fraction =

(1/y)dy – (1/z)dz = 0. y/z = c1.

xdx # ydy # zdz x( y 2 # z 2 ) ! xy 2 ! xz 2

xdx # ydy # zdy . 0

x2 + y2 + z2 = c2.

& xdx + ydy + zdz = 0 so that The required general solution is given by the relations (1) and (2). Ex. 16. Solve

dx 4 4 x (2 y ! z )

dy 4 4 y( z ! 2 x )

...(1)

dz 4 4 . z( x ! y )

Sol. Choosing 1/x, 1/y, 2/z as multipliers, each fraction =

or

(1 / x )dx # (1 / y )dy # (2 / z )dz 4 4 4 4 4 (2 y ! z ) # ( z ! 2 x ) # 2( x ! y ) 4

(1 / x )dx # (1 / y)dy # 2(1 / z )dz 0

& (1/x)dx + (1/y)dy + 2(1/z)dz = 0 so that log x + log y + 2 log z = log c2 log x + log y + log z2 = log c2 or xyz2 = c2. ...(1) Again, choosing x3, y3, z3 as multipliers, each fraction =

x 3 dx # y 3 dy # z 3 dz x 4 (2 y 4 ! z 4 ) # y 4 ( z 4 ! 2 x 4 ) # z 4 ( x 4 ! y 4 )

x 3 dx # y 3 dy # z 3 dz 0

& x3dx + y3dy + z3dz = 0 so that x4 + y4 + z4 = c2. ...(2) The required general solution is given by the relations (1) and (2). Important Note: Sometimes multipliers are chosen by using a trial method. The whole procedure is explained in the next solved Ex. 17. Ex. 17. Solve

dx 4 y ! 3z

dy 4 x ! 2z

dz . 2 y ! 3x

Sol. Choosing l, m, n as multipliers, each fraction of the given equations ldx # mdy # ndz = l ( 4 y ! 3 z ) # m( 4 x ! 2 z ) # n ( 2 y ! 3 x )

...(1)

Created with Print2PDF. To remove this line, buy a license at: http://www.software602.com/

2.10

Simultaneous Equations of the Form cx/P = dy/Q = dz/R

Now, choose l, m, n such that l(4y – 3z) + m(4x – 2z) + n(2y – 3x) = 0 or (4m – 3n)x + (4l + 2n)y + (–3l – 2m)z = 0, which is satisfied if 4m – 3n = 0, 4l + 2n = 0, –3l – 2m = 0 or l : m : n = 2 : –3 : –4. & from (1), each given ratio = (2dx – 3dy – 4dz)/0. & 2dx – 3dy – 4dz = 0 so that 2x – 3y – 4z = c1. Again, choose l, m, n such that l(4y – 3z) + m(4x – 2z) + n(2y – 3x) = 0 or 4(ly + mx) + 3(–lz – nx) + 2(ny – mz) = 0, which is satisfied if ly + mx = 0, –lz – nx = 0, ny – mz = 0 or l : m : n = x : –y : –z. & from (1), each given ratio = (xdx – ydy – zdz)/0. & xdx – ydy – zdz = 0 so that x2 – y2 – z2 = c2. ...(3) The required general solution is given by the relations (1) and (2). Ex. 18. Solve

or

dx 2 z ! 2 yz ! y 2

dy y#z

dz . y!z

Sol. From the last two fractions, (y – z)dy = (y + z)dy ydy – zdz – (zdy + ydz) = 0 or 2ydy – 2zdz – 2d(yz) = 0. Integrating, y2 – z2 – 2yz = c1, c1 being an arbitrary constant ...(1) Taking, 1, y, z as multipliers, each fraction dx # ydy # zdz dx # ydy # zdz = 2 . 2 0 z ! 2 yz ! y # y( y # z ) # z( y ! z ) & dx + ydy + zdz = 0 or 2dx + 2ydy + 2zdz = 0. Integrating, 2x + y2 + z2 = c2, c2 being an arbitrary constant ...(2) The required general solution is given by the relations (1) and (2). Ex. 19. Solve

dx y ! xz

dy yz # x

dz 2

x # y2

.

Sol. Choosing y, x, –1 and x, –y, z as multipliers by turn each given fraction, =

ydx # xdy ! dz xdx ! ydy # zdz 0 0 xdx – ydy + zdz = 0. x2 – y2 + z2 = c2. ... (3)

& ydx + xdy – dz = 0 and Integrating, xy – z = c1 and The required general solution is given by (3). dy dz Ex. 20. Solve 2 dx . 2 2 x ( y ! z) y ( z ! x ) z ( x ! y) Sol. Choosing 1/x, 1/y, 1/z and 1/x2, 1/y2, 1/z2 as multipliers by turn, each fraction

(1 / x )dx # (1 / y )dy # (1 / z ) dz (1 / x 2 ) dx # (1 / y 2 )dy # (1 / z 2 ) dz 0 0 & (1/x)dx + (1/y)dy + (1/z)dz = 0 and x–2dx + y–2dy + z–2dz = 0. Integrating, log x + log y + log z = log c1 and –x–1 – y–1 – z–1 = –c2 –1 –1 –1 xyz = c1 and x + y + z = c2, which give the desired solution. dy dy dx Ex. 21. . x # 2 z 4 zx ! y 2 x 2 # y Sol. Choosing y, x, –2z as multipliers, each fraction ydx # xdy ! 2 zdz d ( xy ) ! 2 zdz = 2 0 y( x # 2 z) # x ( 4 zx ! y ) ! 2 z(2 x # y )

=

or

Created with Print2PDF. To remove this line, buy a license at: http://www.software602.com/

Simultaneous Equations of the Form cx/P = dy/Q = dz/R

2.11 2

& d(xy) – 2zdz = 0 so that Choosing 2x, –1, –1 as multipliers, each fraction =

xy – z = c1.

2 xdx ! dy ! dz 2 2 x ( x # 2 z ) ! (4 zx ! y) ! (2 x # y )

...(1)

2 xdx ! dy ! dz . 0

& 2xdx – dy – dz = 0 so that x2 – y – z = c2. The required general solution is given by relations (1) and (2).

...(2)

dy dx dz . x ! y x # y 2 xz Sol. Choosing 1, 1, –1/z as multipliers, each given fraction

Ex. 22. Solve

=

dx # dy ! (1/ z )dz ( x ! y ) # ( x # y ) ! (1/ z ) ∃ (2 xz )

& dx + dy – (1/z)dz = 0

x # y 1 # ( y / x) dy = . x ! y 1 ! (y / x) dx so that dy/dx = v + x(dv/dx).

Let y/x = v From (3), Using (3) and (4), (2) reduces to v#x

1# v dv = 1! v dx

x

d v = dx x 1# v

2 tan–1 v – log (1 + v2) = 2 log x – log c2

log {x2(1 + v2)/c2} = 2 tan–1 v

or

!1

y = xv.

or

( y / x)

dy 2 2 ! y(3 x # y )

!1

v

, as v = y/x by (3)

(x2 + y2) e !2 tan ( y / x) = c2, c2 being an arbitrary constant The required general solution is given by the relations (1) and (5). dx 2 2 x ( x # 3y )

1 # v2 1! v

x2(1 + v2) = c2 e2 tan

!1

Ex. 23. Solve

...(3) ...(4)

∋ 2 2v ( 2dx ! dv )) 2 2 ∗ ∗ x 1# v , +1# v 2 2 log x – log (1 + v ) – log c2 = 2 tan–

or

x2{1 + (y2/x2)} = c2 e 2 tan

or

...(2)

1# v 1 + v ! v(1 ! v) dv !v= = 1! v 1! v dx

or

2

Integrating, 1 v

or

or

1! v

or

x + y – log z = c1. ...(1)

so that

Taking the first two fractions

dx # dy ! (1/ z )dz . 0

...(5)

dz . 2 2 2 z( y ! x )

Sol. Choosing 1/x, 1/y, –1/z as multipliers, each fraction of the given equations. =

(1 / x)dx # (1 / y)dy ! (1 / z)dz 0

Integrating,

log x + log y – log z = log c1

Taking the first two fractions, Putting v#x

y/x = v

or

y = xv

dx # dy ! dz = 0. x y z (xy)/z = c1. ...(1)

so that so that

dy y (3 x 2 # y 2 ) = ! 2 dx x( x # 3 y 2 )

so that

!

y 3 # ( y / x)2 ∃ x 1 # 3( y / x ) 2

dy/dx = v + x (dv/dx),

2

3# v dv = !v dx 1 # 3v2

or

x

we get − 3 # v2 . dv = !v / # 10 2 dx 11 # 3v 2

Created with Print2PDF. To remove this line, buy a license at: http://www.software602.com/

2.12

Simultaneous Equations of the Form cx/P = dy/Q = dz/R

or or

or or

x

4(1 # v 2 )v dv = ! dx 1 # 3v2

or

4

dx 1 # 3v2 # dv x v(1 # v 2 )

0

dx ∋ 1 2v ( # )) # ∗ d v = 0, on resolving into partial fractions x + v 1 # v 2 ∗, Integrating, 4 log x + log v + log (1 + v2) = log c23 or x4v(1 + v2) = c23 4 2 2 2 2 x (y/x)[1 + (y/x) )] = c23 or xy(x + y ) = c23 or c1z(x + y2) = c23, using (1) 2 2 z(x + y ) = c2, where c2 = c23/c1, c2 being an arbitrary constant ...(2) The required general solution is given by the relations (1) and (2). 4

Ex. 24. Solve

dx y#z

dy !( x # z )

dz . x!z

[Mysore 2004]

dx # dy # dz dx # dy # dz y # z ! (x # z) # x ! y 0 so that dx + dy + dz = 0 and so x + y + z = C1 ... (1) From (1), y + z = C1 – x and x + z = C1 – y Hence the first two fractions of the given problem may be re-written as

Sol. Choosing 1, 1, 1 as multipliers each fraction

dx C1 ! x

or

dy !(C1 ! y )

dx dy # C1 ! x C1 ! y

or

0

Integrating, – log (C1 – x) – log (C1 – y) = – log C2, C2 being an arbitrary constant (C1 – x) (C1 – y) = C2 or (y + z) (x + z) = C2 ... (2) The required solution is given by the relations (1) and (2).

EXERCISE 2 (C) Solve the following simultaneous differential equations : 1.

dx

x

2

4y

3

!z

3

dy

5

y

2

4z

3

!x

3

5

dz

z

2

4x

Hint. Do like Ex. 4. of Art. 2.9, 2.

dx x( y ! z )

dy y( z ! x)

dz . z( x ! y)

ldx mn( y ! z )

mdy nl ( z ! x)

! y3

5

[Delhi Maths (G) 2005] Ans. x2 + y2 + z2 = c1, 1/x + 1/y + 1/z = c2 [Pune 2010; Nagpur 1996; Bangalore 1993] Ans. x + y + z = c1 and xyz = c2

Sol. Try yourself 3.

3

ndz . lm( x ! y )

[Delhi Maths (G) 2001]

Sol. Try yourself as in Ex. 1 of Art. 2.9 Ans. l2x + m2y + n2z = c1, l2x2 + m2y2 + n2z2 = c2 4. (dx)/y = (dy)/(–x) = (dz)/(2x – 3y) [Osmania 2003] Ans. x2 + y2 = C1, 3x + 2y + z = C2 5. (dx)/(zx) = (dy)/(–zy) = (dz)/(y2 – x2) Ans. xy = C1, x2 + y2 + z2 = C2 2.10. Rule IV for solving (dx)/P = (dy)/Q = (dz)/R. ...(1) Let P1, Q1, R1 be functions of x, y, z. Then, by a well known principle of algebra, each fraction in (1) will be equal to (P1dx + Q1dy + R1dz)/(P1P + Q1Q + R1R). ...(2)

Created with Print2PDF. To remove this line, buy a license at: http://www.software602.com/

Simultaneous Equations of the Form cx/P = dy/Q = dz/R

2.13

Suppose the numerator of (2) is exact differential of the denominator of (2). Then (2) can be combined with a suitable fraction in (1) to give an integral. However, in some problems, another set of multipliers P2, Q2 and R2 are so chosen that the fraction (P2dx + Q2dy + R2dz)/(P2P + Q2Q + R2R) ...(3) is such that its numerator is exact differential of denominator. Fractions (2) and (3) are then combined to give an integral. This method may also be repeated in some problems to get another integral. Sometimes only one integral is possible with help of multipliers. In such cases second integral should be obtained by using rule I of Art. 2.4 or rule II of Art. 2.6 or rule III of Art 2.8 as the case may be. 2.11. Solved examples based on Art. 2.10 Ex. 1. Solve

dy dx = 2 y ( x ! y) ! x ( x ! y)

dz

2

2

z( x # y 2 )

dy ! x ( x ! y)

dx y ( x ! y)

Sol. Given

2

dx . 2 2 z( x # y )

2

... (1)

Taking the first two fractions in (1), we get x2dx = –y2dy or 3x2dx + 3y2dy = 0. 3 3 Integrating, x + y = c1, c1 being an arbitrary constant Choosing 1, –1, 0 as multipliers, each fraction of (1) =

dx ! dy 2 2 y ( x ! y) # x ( x ! y)

...(2)

dx ! dy . ...(3) 2 2 ( x ! y )( x # y )

Combining the third fraction in (1) with fraction (3), we get dx ! dy dz = 2 2 z( x 2 # y 2 ) ( x ! y )( x # y )

Integrating

log (x – y) – log z = log c2

The required solution is given by (2) and (4). dy dz . Ex. 2. Solve 2 dx2 2 2 xy 2 xz x !y !z dx 2 2 2 y #z !x

or

dz z

or

Sol. Given

or

dz . !2 xz dx dy 2 2 2 2 xy x !y !z

Delhi Maths (Prog) 2009] [Delhi Maths (G) 2000; Nagpur 1995] dz 2 xz

Taking the last two fractions of (1), Integrating, log y – log z = log c1 Choosing x, y, z as multipliers, each fraction in (1)

... (1) (1/y)dy – (1/z)dz = 0 or y/z = c1.

xdx # ydy # zdz 2 2 2 2 x ! xy ! xz # 2 xy # 2 xz Combining the third fraction in (1) with fraction (3), we get xdx # ydy #zdz dz 2 2 2 = 2 xz x( x # y # z )

(x – y)/z = c2. ...(4)

[Guwahati 2007; Delhi Maths (H) 2000,

dy !2 xy

=

dx ! dy x!y

3

or

Integrating, log (x2 + y2 + z2) – log z = log c2 The required solution is given by (2) and (4).

...(2)

xdx # ydy # zdz 2 2 2 . ...(3) x( x # y # z )

2( xdx # ydy # zdz ) dz = . 2 2 2 z x #y #z

or

(x2 + y2 + z2)/z = c2. ...(4)

Created with Print2PDF. To remove this line, buy a license at: http://www.software602.com/

2.14

Simultaneous Equations of the Form cx/P = dy/Q = dz/R

dx x ! yz

Ex. 3. Solve

2

dy y ! zx 2

dz [Bangalore 2005, Delhi Maths (G) 1998, 2008] z ! xy 2

dx

Sol. Given

2

x ! yz

dy

dz

2

... (1)

2

y ! zx

z ! xy

Choosing 1, –1, 0 and 0, 1, –1 as multipliers by turn, each fraction of (1) = &

dx ! dy dy ! dz = ( x ! y)( x # y # z ) ( y ! z )( y # z # x )

dx ! dy 2 2 x ! y # z( x ! y )

dx ! dy dy ! dz = . y!z x!y (x – y)/(y – z) = c1. ...(2)

or

Integrating, log (x – y) – log (y – z) = log c1 so that Choosing x, y, z as multipliers, each fraction in (1) =

dy ! dz 2 2 y ! z # x ( y ! z)

xdx # ydy # zdz xdx # ydy # zdz = 3 3 3 2 2 2 x # y # z ! 3xyz ( x # y # z )( x # y # z ! xy ! yz ! zx )

...(3)

Again, choosing 1, 1, 1 as multipliers, each fraction in (1) =

or

2

From (3) and (4), we get (xdx + ydy + zdz)/(x + y + z) = dx + dy + dz 2(x + y + z)(dx + dy + dz) – 2(xdx + ydy + zdz) = 0

( x # y # z )2 ! ( x 2 # y 2 # z 2 ) = 2c2

Integrating,

( x2 # y 2 # z 2 # 2 xy # 2 yz # 2 zx) ! ( x 2 # y 2 # z 2 ) = 2c2

or or

dx # dy # dz ...(4) 2 x # y # z ! xy ! yz ! zx 2

xy + yz + zx = c2, c2 being an arbitrary constant. The required solution is given by (2) and (5). Ex. 4. Solve dx y#z

dy z#x

dz . x#y

...(5)

[Delhi Maths 1999; 2002]

dy dx dz y#z z#x x#y Choosing, 1, –1, 0 and 0, 1, –1 as multipliers each fraction of (1)

Sol. Given

=

dy ! dz dx ! dy = . ( y # z) ! ( z # x ) ( z # x ) ! ( x # y)

dy ! dz dx ! dy = or !( y ! z ) ! ( x ! y) Integrating, log (x – y) – log (y – z) = log c1

So

so that

Choosing 1, 1, 1 as multiplers, each given fraction of (1) =

... (1)

...(2)

dx ! dy dy ! dz = . x!y y!z (x – y)/(y – z) = c1. ...(3) dx # dy # dz . 2( x # y # z)

...(4)

Combining the first fraction in (2) which fraction (4), we have dx ! dy dx ! dy dx # dy # dz dx # dy # dz # = or = 0. ! ( x ! y) 2( x # y # z) x!y 2( x # y # z ) 1/2 Integrating, log (x – y) + (1/2) × log (x + y + z) = log c2 or (x – y)(x + y + z) = c2. ...(5) The required solution is given by (3) and (5).

Created with Print2PDF. To remove this line, buy a license at: http://www.software602.com/

Simultaneous Equations of the Form cx/P = dy/Q = dz/R

Ex. 5. Solve dx 1# y

dy 1# x

dz . z

2.15

[Delhi Maths (Hons) 2007, 08; Meerut 2005]

dy dx dz 1# y 1# x z Taking the first two fractions in (1), we have 2(1 + x)dx – 2(1 + y)dy = 0 so that (1 + x)2 – (1 + y)2 = c1. Taking 1, 1, 0 as multipliers, each fraction in (1) = (dx + dy)/(2 + x + y). Combining the last fraction in (1) with (3), we get

Sol. Given

dx # dy = dz z 2#x#y

or

dx x

dy y

dz 2

z ! a ( x # y 2 # z 2 )1/ 2 dx x

Sol. Given

dy y

...(4)

. dz

... (1)

2

z ! a ( x # y 2 # z 2 )1/ 2

Taking the first two fractions in (1), Integrating, log x – log y = log c1 Choosing x, y, z as multipliers, each fraction in (1) =

...(2) ...(3)

log (2 + x + y) – log z = log c2

so that

(2 + x + y)/z = c2, c2 being an arbitrary constant. The required solution is given by (2) and (4). Ex. 6. Solve

...(1)

(1/x)dx – (1/y)dy = 0 or x/y = c1.

xdx # ydy # zdz

tdt

x 2 # y 2 # z 2 ! az ( x 2 # y 2 # z 2 )1/ 2

t 2 ! azt

dt . t ! az

...(2)

...(3)

[Put x2 + y2 + z2 = t2 so that xdx + ydy + zdz = tdt] dx x

Putting x2 + y2 + z2 = t2 in (1), we get Then,

(3) and (4)

dz = dt z !at t ! az

6

dy y

dz z ! at

... (4)

dx . x

...(5)

dz # dt . ( z # t )(1 ! a )

...(6)

Choosing 1, 1, 0 as multipliers, each fraction in (5) =

dz # dt z # t ! a (t # z )

Combining the last fraction in (5) with (6), we get dz # dt dx = ( z # t )(1 ! a) x

(1 ! a )

dx dz # dt ! = 0. x z#t

(1 – a) log x – log (z + t) = log c2

Integrating, or

or

a !1

x = c2 z#t

or

xa ! 1 z # ( x 2 # y 2 # z 2 )1/ 2

The complete solution is given by (2) and (7). Ex. 7. Solve (dx)/cos (x + y) = (dy)/sin (x + y) = (dz)/z. Sol. Given (dx)/cos (x + y) = (dy)/sin (x + y) = (dz)/z.

= c2. ...(7)

... (1)

Created with Print2PDF. To remove this line, buy a license at: http://www.software602.com/

2.16

Simultaneous Equations of the Form cx/P = dy/Q = dz/R

From (1), using 1, 1, 0 and 1, –1, 0 as multipliers by turn, we have dx # dy dx ! dy dz = . z cos ( x # y ) # sin ( x # y) cos ( x # y ) ! sin ( x # y) Putting x + y = t so that dx + dy = dt, the first two fractions give dz dt = z cos t # sin t

or

LM N

2

cos t # sin t

FH

IK

2

1 sin t 2

or

z

FH

IK

cot 1 x # y # 7 = c1, as 2 4 Now from the last two fractions in (2), we get z

2

cos ( x # y) ! sin ( x # y ) ( dx # dy) = dx – dy cos ( x # y) # sin ( x # y )

FH

IK

dz = cosec t # 7 dt 4 z

FG 1 cos t # H 2

Integrating, 2 log z = log tan 1 t # 7 # log c1 2 4 or

...(2)

IJ K

2

FH 7 IK OP 4 Q tan 1 FH t # 7 IK

2 sin t #

= c1

2

t = x + y.

4

...(3)

cos t ! sin t dt = dx – dy. cos t # sin t

or

log (cos t + sin t) – log c2 = x – y or (cos t + sin t)/c2 = ex – y. y–x [cos (x + y) + sin (x + y)]e = c2, as t = x + y. ...(4) The complete solution is given by (3) and (4). dy dx dz Ex. 8. Solve 2 . [Garhwal 2010] 2 2 2 x ! y ! yz x ! y ! zx z( x ! y ) Integrating,

or

Sol. Choosing 1, –1, 0 and x, – y, 0 as multipliers by turn, each fraction of the given equations. xdx ! ydy 2 ( x ! y )( x ! y ) In view of the last fraction of the given equations and the above fractions, we have

=

dz z( x ! y)

...(2)

dx ! dy z( x ! y)

dx ! dy z ( x ! y)

2

xdx ! ydy

... (1)

2

(x ! y2 ) (x ! y)

dz = dx – dy. so that z – x + y = c1.

From the first two fractions of (1), Integrating, z = x – y + c1

2( xdx ! ydy ) 8dz = . 2 2 z x !y Integrating, 2 log z = log (x2 – y2) – log c2. or (x2 – y2)/z2 = c2. ...(3) The required general solution is given by the relations (2) and (3). Ex. 9. Solve (dx)/xz = (dy)/yz = (dz)/xy. Sol. Given (dx)/xy = (dy)/yz = (dz)/xy ... (1) Taking the first two fraction, (1/x)dx – (1/y)dy = 0. Integrating, log x – log y = log c1 or x/y = c1. ...(2) Choosing 1/x, 1/y, 0 as multipliers, each fraction of (1)

Now taking the first and the last fraction in (1),

=

(1/ x )dx # (1/ y )dy (1/ x ) ∃ xz # (1/ y ) ∃ yz

ydx # xdy 2 xyz

...(3)

Combining the last fraction of (1) with fraction (3), we have

Created with Print2PDF. To remove this line, buy a license at: http://www.software602.com/

Simultaneous Equations of the Form cx/P = dy/Q = dz/R

ydx # xdy dz = 2 xyz xy

or

2.17

or

ydx + xdy = 2zdz xy – z2 = c2. ...(4)

d(xy) = 2zdz so that The required general solution is given by the relations (2) and (4). Ex. 10. Solve

dx 2 2 x # y # yz

Sol. Given

dy 2 2 x # y ! xz

dz . z( x # y )

dx 2 x # y # yz

dy 2 x # y ! xz

2

[Kanpur 2009] dz z( x # y )

2

... (1)

Choosing 1, –1, 0 as multipliers, each fraction of (1) =

dx ! dy 2 2 2 2 ( x # y # yz ) ! ( x # y ! xz)

dx ! dy . z( x # y)

...(2)

Choosing x, y, 0 as multipliers, each fraction of (1) =

xdx # ydy 2 2 2 2 x ( x # y # yz) # y( x # y ! xz ) dx ! dy dz = z( x # y ) z( x # y )

From (1), (2) and (3),

xdx # ydy ...(3) 2 2 ( x # y)( x # y )

xdx # ydy 2 2 ( x # y )( x # y )

...(4)

Taking the first two fractions of (4), dz – dx + dy = 0. Integrating, z – x + y = c1, c1 being an arbitrary constant

d( x 2 # y2 ) ! 2 dz = 0. 2 2 z x #y

Taking the first and third fractions of (4), log (x2 + y2) – 2 log z = log c2

Integrating,

or

The required general solution is given by relations (1) and (2) Ex. 11. Solve

dx 2 x # 3 xy 3

dy 2 y # 3x y 3

dx 2 x # 3 xy

Sol. Given

3

dz . 2 2 z( x # y ) dy 2 y # 3x y 3

Choosing 1, –1, 0 as multipliers, each fraction of (1) =

or

(x2 + y2)/z2 = c2. ...(6)

2

Choosing 1, 1, 0 as multipliers, each fraction of (1)

or

...(5)

=

dz 2 2 z( x # y )

... (1)

2

dx # dy 2 2 3 x # 3xy # 3x y # y 3

d ( x # y) . ... (2) 3 ( x # y)

dx ! dy

d ( x ! y)

x 3 # 3 xy 2 ! y 3 ! 3 x 2 y

( x ! y) 3

.

...(3)

From (2) and (3), (x + y)–3d(x + y) = (x – y)–3d(x – y). –3 –3 u du – v dv = 0, [putting u=x+y and v = x – y] –2 –2 –2 –2 Integrating, u /(–2) – v /(–2) = c1/2 or v – u = c1 (x – y)–2 – (x + y)–2 = c1, as u = x + y, v = x – y. ...(4) Choosing 1/x, 1/y, 0 as multipliers, each fraction of (1) =

(1 / x )dx # (1 / y )dy 3 2 3 2 (1 / x )( x # 3 xy ) # (1 / y )( y # 3 x y )

(1 / x )dx # (1 / y )dy . ...(5) 2 2 4( x # y )

Combining the last fraction of (1) with fraction (5), we have

Created with Print2PDF. To remove this line, buy a license at: http://www.software602.com/

2.18

Simultaneous Equations of the Form cx/P = dy/Q = dz/R

(1 / x )dx # (1 / y )dy dz = 2 2 2 2 z( x # y ) 4( x # y )

dx # dy ! 2 dz = 0. x y z

or

2

Integrating, log x + log y – 2 log z = log c2 or (xy)/z2 = c2. The required general solution is given by the relations (4) and (6). Ex. 12. Solve dx = dy = (dz)/(x + y + z). dx dy dz 1 1 x#y#z Taking the first two fractions, dx – dy = 0 so that Choosing 1, 1, 1 as multipliers, each fraction of (1)

Sol. Given

=

dx # dy # dz 1 # 1 # ( x # y # z)

...(6)

... (1) x – y = c1. ...(2)

d (2 # x # y # z ) 2#x#y#z

...(3)

Combining the first fraction of (1) with fraction (3), we get d (2 # x # y # z ) = dx 2#x#y#z

or

log (2 + x + y + z) – log c2 = x.

so that

[(2 + x + y + z)/c2] = ex or The required general solution is given by (2) and (4). Ex. 13. Solve

dy

dx y # yz # z 2 2

...(4)

dz . [Delhi Maths (H) 2004; Meerut 1996] x 2 # xy # y 2

2

z # zx # x 2

dy 2 z # zx # x

dx 2 y # yz # z

Sol. Given

e–x(2 + x + y + z) = c2.

2

dz 2 2 x # xy # y

2

... (1)

Choosing 1, –1, 0 as multipliers, each ratio of (1) =

dx ! dy 2

2

dx ! dy

2

2

2

( y # yz # z ) ! ( z # zx # x )

2

( y ! x ) # z( y ! x)

=!

dy ! dx · ( y ! x )( y # x # z)

...(2)

Again, choosing 0, 1, –1 as multipliers, each ratio of (1) =

dy ! dz 2 2 2 ( z # zx # x ) ! ( x # xy # y )

dy ! dz dz ! dy =! · 2 ( z ! y )( z # y # x ) ( z ! y ) # x ( z ! y)

2

2

...(3)

d ( y ! x ) d ( z ! y) – 0. y!x z!y Integrating, log (y – x) – log (z – y) = log c1 or (y – x)/(z – y) = c1. ...(4) Choosing x, y, z as multipliers, each ratio of (1)

From (2) and (3),

=

xdx # ydy # zdz 2

2

2

2

2

2

x ( y # yz # z ) # y ( z # zx # x ) # z( x # xy # y )

=

xdx # ydy # zdz ( x # y # z )( xy # yz # zx )

...(5)

Again, choosing y + z, z + x, x + y as multipliers, each ratio of (1) =

( y # z )dx # ( z # x )dy # ( x # y )dz 2 2 2 2 2 ( y # z )( y # yz # z ) # ( z # x )(z # zx #x ) # ( x # y )( x # xy # y )

=

( ydx # xdy ) # ( ydz # zdy) # ( zdx # xdz ) 2 2 3 2 2 3 2 2 2( x # xy # xz # y # yx # yz # z # zx # zy )

=

d ( xy ) # d ( yz ) # d ( zx ) 2 2 2 2( x # y # z)( x # y # z )

2

3

d ( xy # yz # zx ) . 2 2 2 2( x # y # z)( x # y # z )

...(6)

Created with Print2PDF. To remove this line, buy a license at: http://www.software602.com/

Simultaneous Equations of the Form cx/P = dy/Q = dz/R

2.19

From fractions (5) and (6), we have

or

xdx # ydy # zdz d ( xy # yz # zx ) = 2 2 2 . ( x # y # z )( xy # yz #zx ) 2( x # y # z )( x # y # z ) (xy + yz + zx) d(xy + yz + zx) – (x2 + y2 + z2) (2xdx + 2ydy + 2zdz) = 0. Integrating, (xy + yz + zx)2 – (x2 + y2 + z2)2 = c2. The required general solution is given by the relations (4) and (7).

Ex. 14. Solve

dx x ( x # y)

dy ! y( x # y)

...(7)

dz . !( x ! y )(2 x # 2 y # z )

dy dx dz ... (1) x ( x # y) ! y( x # y) !( x ! y )(2 x # 2 y # z ) Taking the first two fractions in (1), we get (1/x)dx + (1/y)dy = 0. Integrating, log x + log y = log c1 or xy = c1. ...(2) Choosing, 1, 1, 0 as multipliers, each fraction of (1)

Sol. Given

dx # dy x ( x # y) ! y( x # y) Again, choosing 1, 1, 1 as multipliers, each fraction of (1)

dx # dy . ( x # y )( x ! y)

=

=

dx # dy # dz dx # dy # dz = x ( x # y) ! y( x # y) ! ( x ! y)( 2 x # 2 y # z ) ( x ! y )( x # y) ! ( x ! y)( 2 x # 2 y # z )

=

dx # dy # dz ( x ! y){x # y ! (2 x # 2 y # z)}

From fractions (3) and (4), or

...(3)

!

dx # dy # dz . ...(4) (x ! y)( x # y # z)

dx # dy # dz dx # dy =! ( x # y )( x ! y ) ( x ! y)( x # y # z )

dx # dy dx # dy # dz # = 0. x#y x#y#z Integrating, log (x + y) + log (x + y + z) = log c2, so that (x + y)(x + y + z) = c2. ...(5) The required general solution is given by the relations (2) and (5). Ex. 15. Solve (dx)/x2 = (dy)/y2 = (dz)/z(x + y)

Sol. Given

dy 2 y

dx 2 x

From the first two fractions in (1), Integrating, –x–1 + y–1 = c1

dz z( x # y )

... (1)

x–2dx – y–2dy = 0. (1/y) – (1/x) = c1.

or

Choosing 1, –1, 0 as multipliers, each fraction of (1)

=

dx ! dy 2 2 x !y

...(2)

dx ! dy . ( x ! y)( x # y )

...(3) Taking the last fraction of (1) and fraction (3), we have dx ! dy d ( x ! y ) dz dz = or ! = 0. z( x # y ) ( x ! y )( x # y ) x!y z Integrating, log (x – y) – log z = log c2 or (x – y)/z = c2. ...(4) The required general solution is given by the relations (2) and (4). Ex. 16. Solve (dx)/(x2 + y2) = (dy)/(2xy) = (dz)/z(x + y).

Sol. Given

dx x # y2 2

dy 2 xy

dz ( x # y) z

... (1)

Created with Print2PDF. To remove this line, buy a license at: http://www.software602.com/

2.20

Simultaneous Equations of the Form cx/P = dy/Q = dz/R

Choosing 1, 1, 0 as multipliers, each fraction of (1)

=

dx # dy 2 x # y # 2 xy 2

dx # dy . ...(2) 2 ( x # y)

dx ! dy dx ! dy . ...(3) 2 2 x # y ! 2 xy ( x ! y ) From fractions (2) and (3), (x + y)–2(dx + dy) = (x – y)–2(dx – dy). –1 –1 Integrating, –(x + y) = –(x – y) + c1 or (x – y)–1 – (x + y)–1 = c1....(4) From the last fraction of (1) and fraction (2), we have

Choosing 1, –1, 0 as multipliers, each fraction of (1) =

dx # dy dz 2 = ( x # y )z ( x # y)

2

d ( x # y ) dz ! = 0. x#y z

or

Integrating, log (x + y) – log z = log c2 or The required general solution is given by relations (4) and (5). Ex. 17. Solve (dx)/y = (dy)/x = (dz)/z.

(x + y)/z = c2.

( dx ) / y ( dy ) / x ( dx) / z

Sol. Given

... (1)

From the first two fractions, 2xdx – 2ydy = 0 so that x2 – y2 = c1. Choosing 1, 1, 0 as multipliers, each fraction of (1) = (dx + dy)/(y + x). Combining this fraction which the last fraction (1), we get dx # dy x#y

or

dz z

zdy ! ydz z. z ! y( ! y )

zdy ! ydz z2 # y2

...(2)

log (x + y) – log z = log c2.

so that

log [(x + y)/z] = log c2 or (x + y)/z = c2. The required general solution is given by the relations (2) and (3). Ex. 18. Solve (dx)/x = (dy)/z = (dz)/(–y). Sol. Given (dx)/x = (dy)/z = (dz)(–y) From the last two fractions, 2ydy + 2zdz = 0. so that y2 + z2 = c1. Choosing, 0, z, –y as multipliers each fraction of (1) =

...(5)

(1 / z )dy ! ( y / z 2 )dz d( y / z) = 2 1 # ( y / z) 1 # ( y / z )2

...(3)

... (1) ...(2)

dt , where t = y ...(3) z 1 # t2

Combining the above fraction with the first fraction of (1), we get dt dx = x 1 # t2

or

log x – log c2 = tan–1 t = tan–1 (y/z).

so that

!1

x = c2 e tan ( y / z ) The required general solution is given by the relations (2) and (4). Ex. 19. Solve (dx)/(x2 + a2) = (dy)/(xy – az) = (dz)/(xz + ay). dy dx dz Sol. Given x 2 # a 2 xy ! az xz # ay Taking 0, z, –y as multipliers, each fraction of (1) zdy ! ydz zdy ! ydz = . z( xy ! az ) ! y( xz # ay) ! a( y 2 # z 2 ) Taking 0, y, z as multipliers, each fraction of (1) log (x/c2) = tan–1 (y/z)

or

=

ydy # zdz y( xy ! az) # z( xz # ay )

ydy # zdz . 2 2 x( y # z )

...(4)

... (1)

...(2)

...(3)

Created with Print2PDF. To remove this line, buy a license at: http://www.software602.com/

Simultaneous Equations of the Form cx/P = dy/Q = dz/R

2.21

Taking the first fraction of (1) and fraction (3), we have ydy # zdz dx 2 2 = 2 2 x #a x( y # z )

2 ydy # 2 zdz 2 xdx ! = 0. 2 2 2 2 x #a y #z

or

log (x2 + a2) – log (y2 + z2) = log c2. (x2 + a2)/(y2 + z2) = c2, c2 being an arbitrary constant Taking the first fraction of (1) and fraction (2), we have Integrating,

or

zdy ! ydz dx = 2 2 2 x #a ! a( y # z )

or

zdy ! ydz adx =0 2 # x #a z2 # y2

2 adx # (1 / z )dy ! ( y / z )dz = 0 x2 # a2 1 # ( y / z )2

or

d(y / z) adx =0 # 2 2 x #a 1 # ( y / z)

2

or or

...(4)

2

2

y adx dt = 0, where t= . # z x 2 # a2 1 # t 2 –1 –1 –1 Integrating, a tan (x/a) + tan t = c2 or a tan (x/a) + tan–1 (y/z) = c2. The required general solution is given by the relations (4) and (5).

Ex. 20. Solve

dx y( x # y ) # az

dy x ( x # y) ! az dx y( x # y ) # az

Sol. Given

...(5)

dz . z ( x # y) [Delhi Maths (Hons) 2005, Nagpur 2005, 10] dy x ( x # y) ! az

dz z ( x # y)

... (1)

Choosing 1, 1, 0 as multipliers, each fraction of (1) =

dx # dy y( x # y) # az # x ( x # y ) ! az

dx # dy . 2 ( x # y)

...(2)

From the last fraction of (1) and the fraction (2), we have dx # dy dz = 2 z ( x # y) ( x # y)

d ( x # y ) dz = 0. ! x#y z

or

Integrating, log (x + y) – log z = log c1 Choosing x, –y, 0 as multipliers, each fraction of (1) =

(x + y)/z = c1. ...(3)

or

xdx ! ydy x[ y( x # y ) # az] ! y[ x ( x # y) ! az ]

xdx ! ydy . ...(4) az( x # y )

From the first fraction of (1) and fraction (4), we have xdx ! ydy dz = az( x # y) z( x # y )

2xdx – 2ydy – 2adz = 0.

or

Integrating, x2 – y2 – 2az = c2, c2 being an arbitrary constant The required general solution is given by relations (3) and (5). Ex. 21. Solve Sol. Given

dx x # y ! xy

dy 2

2

x y!x!y

dz 2

z( y ! x 2 )

...(5)

[Delhi Maths (H) 1997, 2002, 07]

dx

dy

dz

x # y ! xy 2

x2 y ! x ! y

z( y 2 ! x 2 )

... (1)

Created with Print2PDF. To remove this line, buy a license at: http://www.software602.com/

2.22

Simultaneous Equations of the Form cx/P = dy/Q = dz/R

Each fraction of (1)

y dx # xdy 2

ydx # xdy 2

2

( y ! x 2 ) (1 ! xy )

y ( x # y ! xy ) # x ( x y ! x ! y )

Combining the above fraction with last fraction of (1), we get

y dx # xdy ( y ! x 2 )(1 ! xy) 2

Integrating,

dz z ( y ! x2 )

! y dx ! xdy dz # 0. 1! xy z

or

2

log (1 ! xy ) # log z log c1

z (1 ! xy ) c1

or

... (2)

Again, each fraction of (1) = [ xdx # ydy # (1/ z ) dz ]/ 0 Hence or or

2 x dx # 2 y dy # 2(1/ z ) dz 0

so that

x2 # y 2 # 2log z c23

x 2 # y 2 # 2 log{c1 / (1 ! xy )} c23 , using (2) x2 # y 2 ! 2log (1 ! xy) c2 ,

where

c2 c23 ! 2log c1

... (3)

The required general solution is given by (2) and (3)

EXERCISE 2 (D) Solve the following simultaneous differential equations : 1. (dx)/x = (dy)/z = (dz)/y Ans. y2 – z2 = C1, (y + z)/x = C2 2. (dx)/cos (x + y) = (dy)/sin (x + y) = (dz)/(z + 1/z) Ans. e y ! x 9cos( x # y ) # sin( x # y ): C1 , ( z 2 # 1)1/ 2 tan 937 / 8 ! ( x # y ) / 2: C2 3. (dx)/x = (dy)/(–y) = (dz)/(y2 – x2) (Bangalore 2005) Ans. xy = C1, x2 + y2 – 2z = C2 4. (dx)/y2 = (dy)/x2 = (dz)/z2(x2 – y2) (Bangalore 2005) Ans. x3 – y3 = c1, x + y + (1/z) = C2 5. Solve (dx)/(xz – y) = (dy)/(yz – x) = (dz)/(1– z2) (Pune 2010) Ans. (x – y) (1 – z) = c1,(x + y) (1 + z) = c2 2.12. Orthogonal trajectories of a system of curves on a surface Let the given surface be f(x, y, z) = 0 ... (1) and let the given system of surface be

;( x , y , z )

c, c being a parameter..

... (2)

Then, the given system of curves lying on the surface (1) are the curves of intersection of (1) and (2). Clearly the direction ratios dx, dy, dz of the tangent at any point (x, y, z) on the given curve lying on the surfaces (1) and (2) are given by ( 0, B(m, n) = B(m + 1, n) + B(m, n + 1). Proof. Using results (i) and (ii) of deduction IV, we have [Agra 2006, 07] m n m+n B (m + 1, n ) + B (m, n + 1) # B (m, n) + B ( m, n) # B ( m , n ) # B ( m, n ) m+n m+n m+n

Proof. (i) B ( x + 1, y ) #

Created with Print2PDF. To remove this line, buy a license at: http://www.software602.com/

6.14

Beta and Gamma Functions

Deduction VI. To show that &(m) &( n) B(m, n) # , m ∃ 0, n ∃ 0 0 2& ( m + n ) 2 [Delhi Maths (H) 2008; Purvanchal 2005; Agra 2008] p + 1 4 5 4 q +15 &6 7&6 7 3/ 2 8 2 9 8 2 9 , p ∃ !1, q ∃ !1 p q sin : cos : d : # (ii) [Agra 1999] 0 4 p+q+25 2& 6 7 2 8 9

(i)

3/ 2

sin 2 m !1 : cos 2 n !1 : d : #

3/ 2

(iii)

0

sin p : d : #

3/ 2 0

cos p : d : #

# 3/2

(iv)

(v)

0

3/ 2 0

sin p !1 : cos q !1 : d : #

sin p !1 : d : #

3/ 2 0

1 0 3 0 5 ...( p ! 1) 3 , if p is even +ve integer 2 0 4 0 6 ... p 2 2 0 4 0 6 ...( p ! 1) , is p is odd +ve integer 1 0 3 0 5 ... p

&( p / 2)&(q / 2) 4 p+q5 2& 6 7 8 2 9

cos p !1 : d : #

&( p / 2)&(1/ 2) 3 &( p / 2) 415 # , as & 6 7 # 3 4 p +1 5 2 4 p +15 829 2& 6 &6 7 7 8 2 9 8 2 9

B (m, n) #

Proof. (i) By definition of Beta function,

1 0

x m!1 (1 ! x )n !1 dx

Let x # sin 2 : so that dx # 2sin : cos : d :. Then, we have 3/2

B ( m, n ) #

,

0

3/ 2 0

sin 2 m ! 2 (1 ! sin 2 :)n !1 (2 sin : cos :d :)

sin 2 m !1 : cos 2 n !1 : d : #

&(m)&(n) , 2& ( m + n )

3/ 2

or

0

sin 2 m !1 : cos 2 n !1 : d : #

B ( m, n ) #

as

B (m , n ) . 2

& ( m )& ( n ) ... (1) &(m + n )

Part (ii). Let p = 2m – 1 and q = 2n – 1, so that m = (p + 1)/2 and n = (q + 1)/2.

Then (1) becomes

3/2 0

4 p +1 5 4 q +1 5 &6 7&6 7 2 9 8 2 9 sin p : cos q : d : # 8 4 p+q+25 2& 6 7 2 8 9

... (2)

4 p +15 4 1 5 &6 7&6 7 2 9 829 sin p : d : # 8 4 p+25 2& 6 7 8 2 9

... (3)

Part (iii) Replacing q by 0 in (2),

Next, putting p = 0 and q = p in (2),

3/ 2 0

3/ 2 0

4 p +1 5 4 1 5 &6 7&6 7 2 9 829 8 cos : d : # 4 p+25 2& 6 7 8 2 9 p

... (4)

Created with Print2PDF. To remove this line, buy a license at: http://www.software602.com/

Beta and Gamma Functions

6.15

Let p be even, say p = 2r. Then R.H.S. of (3) or (4) 15 415 4 1 54 35 3 1 415 415 4 2r + 1 5 4 1 5 4 &6 7 & 6 7 & 6 r + 7 & 6 7 6 r ! 7 6 r ! 7 ... 0 0 & 6 7 & 6 7 2 9 829 29 829 8 2 98 29 2 2 8 29 829 8 8 # # # 2&(r + 1) 4 2r + 2 5 2 . r (r ! 1) (2r ! 3) ... 3 . 2 .1 2& 6 7 8 2 9

#

1. 3 . 5 ... (2r ! 3) (2r ! 1) 3 (2r ! 1) (2r ! 3)... 3 .1 3 415 # , as & 6 7 # 3 2 . 4 . 6 ...(2r ! 2) (2r ) 2 (2r ) (2r ! 2) (2r ! 4)...6 . 4 . 2 2 829

1 . 3 . 5 ... ( p ! 3) ( p ! 1) 3 , as p = 2r 2 . 4 . 6 ...( p ! 2) p 2 Next, let p = 2r + 1 i.e., odd +ve integer. Then R.H.S. of (3) and (4) #

#

... (5)

&( r + 1)&(1/ 2) r ( r ! 1)...3 . 2 .1 3 2 . 4 . 6 ... (2r ! 2) (2r ) 2 . 4 . 6 ...( p ! 1) # # # 1 . 3 . 5 .... (2r ! 1) (2r + 1) 1. 3 . 5 ... p , ... (6) 35 1 54 15 3 1 4 4 2& 6 r + 7 2. 6 r + 76 r ! 7 ... 0 3 29 2 98 29 2 2 8 8

since 2r + 1 = p. Thus from (3), (4), (5) and (6) the required results follow. Part (iv) Let 2m = p and 2n = q so that m = p/2 and n = q/2. Then (1) becomes 3/ 2 0

& ( p / 2) & (q / 2) 4 p+q5 2& 6 7 8 2 9

sin p !1 : cosq !1 : d : #

sin p !1 : d : #

&( p / 2)&(1/ 2) 4 p +15 2& 6 7 8 2 9

cos p !1 : d : #

&(1/ 2)& ( p / 2) 41+ p 5 2& 6 7 8 2 9

3/2

Part (v) Replacing q by 1 in (7),

0

3/ 2

Next, replacing p by 1 and q by p, in (7),

0

... (7)

... (8)

... (9)

From (8) and (9), the required results follow. 6.12. Solved Examples Ex. 1. Evaluate the following Integrals : (i) (iii)

1 0

a 0

x 4 (1 ! x )2 dx

(ii)

y 4 (a 2 ! y 2 ) dy

(iv) 1

Sol. We know that Part. (i).

1 0

0

x 4 (1 ! x)2 dx #

Part. (ii) Let I # I#

2 0 1 0

1 0

2

x 2 dx

0

(2 ! x )

2 0

x (8 ! x 3 )1/ 3 dx.

x m !1 (1 ! x )n !1 dx # B (m, n) #

x 5!1 (1 ! x )3!1 dx #

,

[Agra 2000, 03]

&( m) &( n ) & ( m + n)

... (1)

& (5) & (3) 4!2! 4! ∆ 2 1 # # # & (5 + 3) 7! 7 ∆ 5 ∆ 4! ∆ 6 105

x 2 (2 ! x)!1/ 2 dx. Let x = 2t, so that dx = 2dt, Then

(2t )2 (2 ! 2t )!1/ 2 (2dt ) # 4 2

1 0

t 2 (1 ! t )!1/ 2 dt # 4 2

1 0

t 3!1 (1 ! t )1/ 2 !1 dt

Created with Print2PDF. To remove this line, buy a license at: http://www.software602.com/

6.16

Beta and Gamma Functions

&(3)&(1/ 2) 2 ! &(1/ 2) 64 2 #4 2 # 5 3 1 &(3 + 1/ 2) 15 0 0 &(1/ 2) 2 2 2

#4 2

I#

1 0

a

Let I #

Part (iii)

y 4 (a 2 ! y 2 ) dy. Let y2 = a2t, so that dy #

0

(a 2t )2 (a 2 ! a 2t )

(a dt ) 2 t

#

a6 2

1 0

t 3/ 2 (1 ! t )1/ 2 dt #

a6 2

1 0

a 2 dt a dt # . Then 2y 2 t

t (5 / 2)!1 (1 ! t )(3/ 2) !1 dt

3 1 1 0 30 3 a 6 &(5 / 2)&(3/ 2) a6 2 2 3a6 2 # ∆ # ∆ # 2 &(5 / 2 + 3 / 2) 2 3! 32

Part (iv)

, I#

1 0

2

Let I #

x (8 ! x3 )1/3 dx. Put x3 = 8t or x = 2t1/3 so that dx = (2/3) × t–2/3 dt

0

(2t1/3 )(8 ! 8t )1/3 (2 / 3)t !2 / 3 dt #

8 3

1 0

t !1/ 3 (1 ! t )1/ 3 dt #

8 3

1 0

t (2 / 3) !1 (1 ! t )( 4 / 3) !1 dt

8 &(2 / 3)& (4 / 3) 8 &(1 ! 1/ 3)&(1 + 1/ 3) 8 4 1 5 1 4 1 5 # ∆ # ∆ # & 61 ! 7 & 6 7 , as 3 &(2 / 3 + 4 / 3) 3 &(2) 3 8 39 3 839

&(n + 1) # n &(n)

8 3 163 3 # ∆ # , as &(1 ! n)&(n) # 9 sin(3 / 3) 2 3 sin n3

(b)

1

dx

0

(1 ! x 4 )1/ 2

I#

0

(1 ! x )

1

dx

0

(1 ! x n )1/ 2 1

1 0

n

#

&(1/ n) 3 0 . &(1/ 2 + 1/ n) n

[Meerut 2004, Purvanchal 2006]

3 &(1/ 4) 0 4 &(1/ 2 + 1/ 4)

#

Sol. (a) Let I #

dx

1

Ex. 2. Show that (a)

1/ 2

(1 ! t )

[Meerut 2007]

. Putting xn = so that x = t1/n and dx = (1/n)t(1/n)–1 dt, we get

1 1 0 t (1/ n ) !1dt # n n

1 0

t (1/ n ) !1 (1 ! t ) (1/ 2) !1 dt #

1 41 15 B6 , 7 n 8n 29

1 &(1/ n)&(1/ 2) 3 & (1/ n) # . n &(1/ n + 1/ 2) n &(1/ n + 1/ 2) (b) Taking n = 4 in part (a), we get the required result. #

Ex. 3. Show that Sol. Let I #

I#

0

n

n 1/ n

(a ! a t )

(a n ! x n )1/ n

( a ! x n )1/ n

1

1

dx

0

dx

a 0

a

n

#

3 . n sin(3 / n)

[Meerut 1998]

. Putting xn = an t so that x = at1/n and dx = a(1/n) t(1/n)–1 dt, gives

a (1/ n ) !1 1 t dt # n n

1 0

t (1/ n ) !1 (1 ! t ) !1/ n dt #

1 n

1 0

t (1/ n ) !1 (1 ! t ) (1!1/ n ) !1 dt

Created with Print2PDF. To remove this line, buy a license at: http://www.software602.com/

Beta and Gamma Functions

6.17

415 4 15 415 4 15 & 6 7 & 61 ! 7 & 6 7 & 61 ! 7 n n 1 3 8 9 8 9 # 8n9 8 n9 # 0 . 15 n 41 n sin(3 / n) & 6 +1 ! 7 n9 8n

1 41 15 1 # B6 , 1! 7 , # 0 n 8n n9 n

&( p) &(1 ! p) # 3 / sin p3 ]

[ %

Ex. 4. Show that %

Sol. Let I # I#

% t1/16 0

0

x dx 1+ x

x dx

0 (1/ 6)t !5 / 6 1 # 1+ t 6

#

#

1 41 25 3 B6 , 7 # 6 83 39 3 3

[Delhi Maths (H) 2007, 08]

Putting x6 = t so that x = t1/6 and dx = (1/6)t–5/6 dt, we get

1 + x6

0

6

% t !2 / 3 dt

#

1+ t

0

1 6

t (1/3) !1

% 0

(1 + t )1/ 3+ 2 /3

dt

1 41 25 B6 , 7 6 83 39

x m !1dx

% 0

(1 + x )m + n

# B ( m, n)

1 &(1/ 3)&(2 / 3) 1 4 1 5 4 1 5 # ∆ # & 6 7 & 61 ! 7 6 &(1/ 3 + 2 / 3) 6 8 3 9 8 3 9

1 3 3 3 # ∆ # 6 sin (3 / 3) 9

& ( n) & (1 ! n) #

3 sin n3

# (3 3) / 9. Ex. 5. Evaluate (i) %

(iii)

x8 (1 ! x 6 )

0

(1 + x) 24

Sol. Part (i)

% 0

x 4 (1 + x 5 )

%

(1 + x)15

0

%

dx

(ii)

x 4 (1 + x 5 ) 15

(1 + x)

%

dx #

0

#2

Part (iii)

0

x m !1 ! x n !1 (1 + x )m + n

x 4 dx 15

(1 + x )

+

%

x9 15

(1 + x )

0

dx # %

x5 !1dx

% 0

(1 + x)

x m !1

%

dx #

0

x8 (1 ! x 6 )dx (1 + x)

24

#

x m !1dx

0

% 0

0

5 +10

+

% 0

x10!1dx (1 + x)10 +5

dx # B (m, n)

&(5) & (10) 2 ∆ 4 ! ∆ 9 ! 1 # # . & (5 + 10) 14 ! 5005

(1 + x)m + n = B (m, n) – B (n, m) = 0. %

dx

[Garhwal 2000]

(1 + x )m + n = 2B (5, 10), as B (5, 10) = B (10, 5)

%

(1 + x )m +1

0

dx.

= B (5, 10) + B (10, 5), as

Part (ii)

x m !1 ! x n !1

x8 dx (1 + x)

24

= B (9, 15) – B (15, 9) = 0

!

!

% 0

x n !1dx (1 + x )m + n

B(m, n) = B(n, m)]

[ % 0

x14 dx (1 + x )

24

#

% 0

x9 !1dx (1 + x )

9 +15

[

!

% 0

x15 !1 (1 + x )15 +9

B(9, 15) = B (15, 9)]

Created with Print2PDF. To remove this line, buy a license at: http://www.software602.com/

6.18

Beta and Gamma Functions

t

t

1!Η

( z ! x)

(x ! t)



#

3 , 0 Γ Η Γ1 sin 3 Η

dx

z

I#

Sol. Let

dx

z

Ex. 6. Show by means of Beta function, that

1!Η

( z ! x)

... (1)

( x ! t )!Η

Putting x – t = (z – t)y so that x = t + (z – t) y and dx = (z – t)dy, (1) becomes I#

0 [z

1

# #

( z ! t )dy

1

0

1!Η

! t ! ( z ! t ) y]

(1 ! y )Η!1 y Η dy #



[( z ! t ) y ] 1 0

3/ 2 0

tan n x dx # 3/ 2

Sol. Part (a)

0

3/ 2

0

1!Η

(z ! t)

(1 ! y )1!Η ( z ! t ) !Η y !Η

&( p)&(1 ! p) #

as

tan : d : #

0

( z ! t )dy

1

y (1!Η ) !1 (1 ! y )Η!1 dy # B (1 ! Η, Η), by definition of Beta function.

&(1 ! Η) & (Η) 3 # , &(1 ! Η + Η ) sin 3 Η

Ex. 7. Prove that (a)

(b)

#

1 415 435 3 2 &6 7&6 7 # [Kumaun 2000 Meerut 2004] 2 8 49 849 2

3 n3 sec , ! 1 Γ n Γ 1. 2 2 tan : d : #

0

1/ 2

sin : 5 6 7 cos :9 8

3/24

d: #

3/ 2 0

4 1 + 1/ 2 5 4 1 ! 1/ 2 5 &6 7&6 7 2 9 8 2 9 8 # 4 1/ 2 ! 1/ 2 + 2 5 2& 6 7 2 8 9

#

Part (b)

3/2 0

sin1/ 2 : cos!1/ 2 : d :

Refer deduction VI (ii) of Art. 6.11.

&(3 / 4)&(1/ 4) 1 4 3 5 4 1 5 1 4 1 5 4 1 5 1 3 3 2 # & 6 7 & 6 7 # & 6 7 & 61 ! 7 # # 2&(1) 2 8 4 9 8 4 9 2 8 4 9 8 4 9 2 sin(3 / 4) 2

tan n x dx #

3/ 2 0

sin n x cos ! n x dx

4 1+ n 5 41! n 5 &6 7&6 7 2 9 8 2 9 8 # 4n!n+25 2& 6 7 2 8 9

#

3 sin 3p

1 41+ n 5 4 1+ n 5 1 &6 7 & 61 ! 7 # 2 8 2 9 8 2 9 2

Refer deduction VI (ii) of Art. 6.11 Here (1 + n)/2 > 0 and (1 – n)/2 > 0 2 n > –1 and n < 1 2 –1 < n < 1.

3 3 3 # # 1 + n 3 n 3 4 5 4 5 2 cos n3 sin 6 7 3 2sin 6 + 7 2 8 2 9 82 2 9

# (3 / 2) ∆ sec(n3 / 2), where –1 < n < 1.

[

&( p) &(1 ! p) # 3 / sin p 3 ]

Created with Print2PDF. To remove this line, buy a license at: http://www.software602.com/

Beta and Gamma Functions

6.19

Ex. 8. If p > 0, q > 0, m + 1 > 0, n + 1 > 0, prove

q

q

Putting x = p t so that x = pt I#

p

I#

Sol. Let

1 0

1/q

0

p 0

x m ( p q ! xq )n dx #

p nq + m +1 4 m +15 B 6 n + 1, 7 q q 9 8

x m ( p q ! x q ) n dx

and dx = (p/q) t

( pt t / q )m ( p q ! p q t )n ( p / q)t (1/ q )!1dt #

... (1)

(1/q) – 1

dt, (1) reduces to

p m 0 p nq 0 p q

1 0

t ( m / q) + (1/ q) !1 (1 ! t )(n +1)!1 dt

5 p nq + m +1 4 p nq + m +1 4 m + 1 m +1 5 B6 , n + 17 # B 6 n + 1, 7 q q q 9 8 q 9 8

#

Ex. 9. Compute I #

% 0

%

4

x 2 e ! x dx 0

0

4

e ! x dx.

Sol. Putting x4 = t so that x = t1/4 and dx = (1/4) t –

I#

% 0

41 5 (t1/ 4 )2 e !t 6 t !3 / 4 7 dt 0 84 9 %

%

#

1 16

#

1 3 3 2 0 # . 16 sin(3 / 4) 16

0

e !t t (3/4) !1 0

Ex. 10. Show that I #

0

% 0

e !t

1 !3/ 4 1 t dt # 4 16

e !t t (1/ 4) !1dt #

3/2

3/4

dt, we get % 0

e !t t !1/ 4 dt 0

% 0

e !t e !3/ 4 dt

1 435 415 1 415 4 15 & 6 7 & 6 7 # & 6 7 & 61 ! 7 16 8 4 9 8 4 9 16 8 4 9 8 4 9 &(n)&(1 ! n) #

3/ 2

d:

[Delhi B.Sc. (Prog.) 2009] # 3. sin : [Garhwal 2002, Meerut 1998, Delhi Maths (H) 2005, 08] 4 p +15 &6 7 3 3/ 2 2 9 8 p Sol. We know that ... (1) sin :d : # 0 4 p+25 2& 6 7 8 2 9 4 1/ 2 + 1 5 4 !1/ 2 + 1 5 &6 7 3 &6 7 3 3/2 3/ 2 2 2 8 9 8 9 1/ 2 !1/ 2 0 , using (1) I# sin : d : 0 sin : d: # 0 0 4 1/ 2 + 2 5 4 !1/ 2 + 2 5 2& 6 2 & 7 6 7 2 8 2 9 8 9

#

0

sin : d :.

3 sin n3

0

&(3 / 4) 3 &(1/ 4) 3 3&(1/ 4) 3&(1/ 4) 0 # # # 3. 2&(5 / 4) 2&(3/ 4) 4&(1 + 1/ 4) 4 ∆ (1/ 4) ∆ &(1/ 4)

EXERCISE 6 B 1. Prove that (a)

(c)

1 0

1 0

(1 ! x n )1/ n dx #

dx 3 1/ 3

(1 ! x )

#

{& (1/ n)}2 2& (2 / n)

23 3 3

(b)

(d)

1

dx

0

(1 ! x n )1/ n

1 0

#

3 sin(3 / n )

x m!1 (1 ! x k )n dx #

1 n !& ( m / k ) a &( n + 1 + m / k )

Created with Print2PDF. To remove this line, buy a license at: http://www.software602.com/

6.20

Beta and Gamma Functions 1

(e)

(g)

0

x m!1 (1 ! x 2 )n !1 dx #

1

x n !1dx

0

2

#

(1 ! x )

1 41 5 B 6 m, n 7 [Garhwal 2003] (f) 2 82 9

3 &{(n ! 1) / 2} 2&(n / 2)

0

dx

1

(h)

4

0

dx

1

(1 ! x )

(1 + x 4 ) #

1 6 23

#

&(1/ 4)&(1/ 2) 4 2 & (3/ 4)

[&(1/ 4)]2

[Delhi Math (H) 2003, Meerut 2007] 2. Prove that (a)

5 6 ! 17 08 x 9 x n dx

1

3. Show that

1/ 4

14 1

2

(1 ! x )

0

#

3 45 35 dx # B 6 , 7 # 84 49 2 2

(b)

% 0

1. 3 . 5 ... (n ! 1) 3 ∆ 2 . 4 . 6 ... n 2

41 # B6 , t (1 + t ) 82 dt

15 7 # 3. 29

2 . 4 . 6 ...(n ! 1) 1. 3 . 5 ... n

or

according as n is even or odd positive integer. 4. Show that if p and q are positive, then B ( p, q ) # 2

3/2 0

cos 2 p !1 : sin 2 q !1 : d : #

& ( p )& ( q ) and deduce that &( p + q )

3 0

2

e ! x dx #

3 . 2

5. Prove that (i) B(l, m) B(l + m, n) = B(m, n) B(m + n, l) = B(n, l) B(n + l, m). (ii) B(l, m) B(l + m, n) B(l + m + n, p) #

&(l )&( m)&(n)&( p) & (l + m + n + p )

(iii) l B(l, m + l) = mB (l + l, m). 6. Prove that 7. Prove

1 !1

a !a

[Agra 2005]

(a + x) m!1 (a ! x )n !1 dx # (2a )m + n !1 B (m, n)

(1 + x) p !1 (1 ! x )q !1 dx # 2 p + q !1 B( p, q )

8. Using the integral

% 0

x n !1 3 dx # , 0 Γ n Γ 1, prove that &(n) &(1 ! n) # 3 / sin n3, 1+ x sin n3

0 < n < 1 Hence obtain value of &(1/ 2). 9. Prove that

1 !1

( x + 1)a !1 (1 ! x)b !1 ( x + 2)

a+ b

dx #

2a +b !1 B(a, b), a ∃ 0, b ∃ 0 3a

10. Show that the perimeter of a loop of the curve r n # a n cos n: can be expressed as

(a / n) ∆ 2(1/ n)!1{&(1/ 2n) / &(1/ n)} 11. Show that the area enclosed by the curve x4 + y4 = 1 is [ &(1/ 4)]2 / 2 3. 12. With the help of double integral, prove that

% 0

2

e ! x dx # 3 / 2.

[Delhi Maths (H) 2007]

Created with Print2PDF. To remove this line, buy a license at: http://www.software602.com/

Beta and Gamma Functions

6.21

15 3 4 6.13. Legendre-Duplication Formula. &(n) & 6 n + 7 # 2 n !1 &(2 n), n ∃ 0. 29 2 8 [Delhi Maths (H) 2005, 07, Delhi Phy (H) 2002 Agra 2000, 01, 02, 03, 06, 08; Meerut 1999]

B ( m, n ) #

Proof. We know that

& ( m )& ( n ) , where m > 0, n > 0. & ( m + n)

... (1)

B(n, n) # [& (n)]2 / &(2n)

Putting m = n in (1), we get

B ( n, n) #

By the definition of the Beta function,

1 0

... (2)

x n !1 (1 ! x )n !1 dx.

... (3)

Putting x # sin 2 : so that dx # 2 sin : cos : d :, (1) gives 3/ 2

B ( n, n) #

#2

0

3 / 2 4 sin 2: 52 n !1

6 8

0

7 9

2

3/ 2

(sin 2 :) n !1 (1 ! sin 2 :)n !1 0 2sin : cos : d : # 2

d: #

1 2

2n! 2

3/ 2 0

sin 2n!1 2: d : #

3

1 2

2n! 2

0

0

(sin : cos :)2 n !1 d :

sin 2 n !1 Ι

dΙ 1 # 2 n !1 2 2

3 0

sin 2 n !1 Ι d Ι

[On putting 2: # Ι and d : # (d Ι) / 2 ] #

#

1

2

∆2 2 n !1

3/ 2 0

3/ 2

1 22 n !2

0

sin 2 n !1 Ι d Ι,

2a

as

sin 2 n !1 Ι (cos Ι)0 d Ι #

1 22n! 2

0

f ( x )dx # 2

a 0

f ( x) dx

when f(2a – x) = f(x)

4 2n ! 1 + 1 5 4 0 + 1 5 &6 70&6 7 2 8 9 8 2 9 4 2n ! 1 + 0 + 2 5 2& 6 7 2 8 9

&(n) 3 415 , as & 6 7 # 3 & ( n + 1/ 2) 2 829 Equating two values of B (n, n) given by (2) and (3), we obtain B(n, n) #

,

Α &(n)Β2 &(2n)

#

1 2

2 n !1

1

... (3)

2 n !1

& ( n) 3 &(n + 1/ 2)

Deduction 1. To show that B (n, n) #

&(n)&(n + 1/ 2) #

or 3 &( n ) 2 n !1

2 &(n + 1/ 2) Proof. From (3), we get the required result.

3 2

2 n !1

&(2n).

... (4)

,n∃0

4 p +1 5 4 p + 2 5 Deduction II For all positive real values of p, 2 p & 6 7&6 7 # 3 & ( p + 1). 8 2 9 8 2 9 Proof. Putting 2n – 1 = p so that n # ( p + 1) / 2 in (4), we get 3 4 p +1 5 4 p +1 1 5 4 p +1 5 4 p + 2 5 &6 + 7 # p & ( p + 1) or 2p &6 7&6 7&6 7 # 3 &( p + 1). 2 2 2 8 9 8 9 2 8 2 9 8 2 9 Deduction III. When n is positive integer, to show that [Delhi Maths (H) 2003] 1 5 (2n)! 4 & 6 n + 7 # 2n 3. 2 9 2 n! 8

Created with Print2PDF. To remove this line, buy a license at: http://www.software602.com/

6.22

Beta and Gamma Functions

&(2n) (2n ! 1)! (2n) (2n ! 1) ! (2n) ! # # # &(n ) (n ! 1) 2 0 n (n ! 1) ! 2 0 n ! Now, from the duplication formula (4) and the above result, we have

Proof. Let n be positive integer, then

15 3 &(2n) 3 (2n) ! (2n ) ! 4 # 2n 3. & 6 n + 7 # 2n !1 0 # 2n !1 0 29 2 &( n ) 2 20n! 2 n! 8 6.14. SOLVED EXAMPLES. Ex. 1. Express &(1/ 6) in terms of &(1/ 3). 15 3 4 &( n )& 6 n + 7 # 2 n !1 &(2 n). 29 2 8

Sol. From the duplication formula, Putting n = 1/6 in (1), we get 3 & (1/ 3) 415 425 &6 7&6 7 # 2 !2 / 3 869 839 Now, we know that Putting n = 1/3 in (3) we get 3 23 415 425 &6 7& 6 7 # # 8 3 9 8 3 9 sin(3 / 3) 3

... (1)

3 & (1/ 3) 415 & 6 7 # !2 / 3 869 2 & (2 / 3)

or

... (2)

&(n)&(1 ! n) # 3 / sin n3.

... (3)

23 425 &6 7 # 3 & (1/ 3) 839 Substituting the value of &(2 / 3) given by (4) in (2), we get

or

... (4)

2

3 & (1/ 3) 3 & (1/ 3) 415 &6 7 # 0 23 2 !2 / 3 869

3 ∋ 4 1 5( 415 & 6 7 # 1/ 3 Ε& 6 7 Φ . 3 ) 8 3 9∗ 869 2

or

3 &(n / 2) 41! n 5 , 0 Γ n Γ 1. Ex. 2. Prove that &(n)& 6 7# 8 2 9 21! n cos(n3 / 2) Sol. We know that &( m) &(1 ! m) # 3 / sin m3, 0 Γ m Γ 1

and

... (1)

15 3 &(2 m) 4 & ( m )& 6 m + 7 # , m ∃ 0, 29 2 2 m!1 8 Putting m = (n + 1)/2 in (1), we get 3 3 3 4 n + 1 5 41! n 5 &6 # # 7&6 7# 8 2 9 8 2 9 sin{(n + 1)3 / 2} sin(3 / 2 + n3 / 2) cos(n3 / 2) 4 n 5 4 n +1 5 &6 7&6 7# 829 8 2 9

Putting m = n/2 in (2), we get Dividing the corresponding sides of (3) and (4), we get &[(1 ! n)/2] 3 2n !1 # ∆ &(n / 2) cos(n3 / 2) 3 & ( n)

... (2)

... (3) 3 &(n) 2 n !1

... (4)

3 &(n / 2) 41! n 5 & ( n )& 6 7 ! 1! n 8 2 9 2 cos(n3 / 2)

or

Ex. 3. Prove that B(m, m) B(m + 1/ 2, m + 1/ 2) # (3m !1 ) / 24m !1 Ex. 4. By evaluating I #

3/2 0

sin 2 p x dx and J #

duplication formula for gamma function.

3/2 0

sin 2 p x dx derive the Legendre’ss

[Kanpur 2006]

Created with Print2PDF. To remove this line, buy a license at: http://www.software602.com/

7 Power Series 7.1. INTRODUCTION In this chapter we propose to study the theory of power series which is very useful tool in the study of analyis. We shall present a summary of the pertinent results of infinite series specially power series. These will be used in solving second order differential equations in the next chapter 8, namely, ‘Integration in series’. 7.2. *SUMMARY OF USEFUL RESULTS In what follows, we shall deal with infinit series and hence we shall write simply ! un to denote ! un etc. n #1

List A : Results related to convergence of infinite series of positive terms n

A-1. Let ! un be an infinite series and let Sn # ! ui . Then ! un is said to be convergent n #1

i #1

n #1

or divergent according as the sequence < Sn > is convergent or divergent. A-2. Geometric series. The positive term infinite geometric series 1 + r + r2 + .. + rn + ..., (r ∃ 0) is convergent if and only if r < 1. A-3. Harmonic series. The positive term series !1/ n % is converted iff % & 1. A-4. Comparisian test. If !un and !vn are two positive term series such that lim (un / vn ) # l ( 0, then the two series !un and !vn have identical behaviours in relation to

n∋

convergence. A-5. D’Alembert’s ratio test. Let !un be a positive term series such that lim (un)1 / un ) # l. n∋

Then the series is (i) convergent if l < 1 (ii) divergent if l > 1. (iii) No firm decision if l = 1. Also, if l # , then the series is divergent. A-6 Cauchy’s nth root test. Let !un be a positive term series and let lim (un )1/ n # l. Then n∋

the series is (i) convergent if l < 1 (ii) divergent if l > 1 (iii) No firm decision if l = 1. A-7. Raabe’s test. Let !un be a positive term series and let lim n{(un / un )1 ) ∗ 1} # l. Then n∋

the series is (i) convergent if l > 1 (ii) divergent if l < 1 (iii) No firm decision if l = 1. A-8. Logarithmic test. Let !un be a positive term series and let lim n log{un / un )1 ) # l. n∋

Then the series is (i) convergent if l > 1 (ii) divergent if l < 1 (iii) No firm decision if l = 1. *For all results of this article, refer chapters 6, 7 and 15 of Real Analysis by Shanti Narayan and M.D. Raisinghania, published by S.Chand & Co. New Delhi.

7.1

Created with Print2PDF. To remove this line, buy a license at: http://www.software602.com/

7.2

Power Series

List B : Results related to convergence of infinite series with positive and negative terms B-1. Absolutely convergent series. A series !un is said to be absolutely convergent if the positive term series ! | un | formed by the moduli of the terms of the series is convergent. B-2. Alternating series. A series whose terms are alternatively positive and negative is referred to an alternating series. B-3. Leibnitz’s test. Let < un > be a sequence such that for each natural number n (i) un ∃ 0 (ii) un )1 + un (iii) lim un # 0. Then the alternating series !(∗1) n∗1 un is convergent. n∋

B-4. Every absolutely convergent series is convergent. The converse need not be true. List C. Results related uniform convergence of infinite series of functions. C-1. A series ! f n ( x) will converge uniformly in [a, b], if there exists a convergent series !M n of numbers such that , x − [a, b], | f n ( x) | + M n C-2. The sum of a uniformly convergent series of continuous functions is continuous. C-3. If ! f n ( x) be a uniformly convergent series of intergrable functions in [a, b], then the

.

series is term by integrable, that is,

b

! f n ( x) dx # !

a n #1

.

b

n #1 a

f n ( x ) dx

C-4. If ! f n ( x) be a uniformly convergent series of differentiable functions, then the series is d d ! f n ( x) # ! f n ( x) n #1 dx dx n #1

term by term differentiable, i.e., 7.3. POWER SERIES ! an x n

A series of the form

... (1)

n# 0

is known as real infinite power series where a0, a1, ...., an, .... are real coefficients free from x, and n x is the real variable. More generally ! an ( x ∗ x0 ) is taken to represent a general power series. n# 0

Since with a shift of origin to x0 i.e., with change of variable x – x0 to x this precisely reduces to the form (1), hence without any loss of generality our studies shall be confined to the form (1). n For the sake of brevity we shall write !an xn instead of ! an x n #0

7.4. SOME IMPORTANT FACTS ABOUT THE POWER SERIES !an x n . (i) For all values of the coefficients, every power series converges for x = 0. Hence if a power series converges for no value other than x = 0, we say that the given power series is nowhere convergent. For example, the power series ! nn xn is nowhere convergent. (ii) If a given series converges for all values of x, we say that the given power series is everywhere convergent. For example, the power series ! ( x n / n !) is everywhere convergent. (iii) If the given power series converges for some value of x and diverges for other values of x, then the set of all values of x for which it is convergent is known as its region of convergence. 7.5. RADIUS OF CONVERGENCE AND INTERVAL OF CONVERGENCE If a given power series does not converge everywhere or nowhere, then a definite positive number R exists such that the given power series converges (indeed absolutely) for every | x | < R and diverges for every | x | > R. Such a number R is known as the radius of convergence and the interval ] – R, R [, the interval of convergence, of the given power series.

Created with Print2PDF. To remove this line, buy a license at: http://www.software602.com/

Power Series

7.3

7.6. FORMULAS FOR DETERMINING THE RADIUS OF CONVERGENCE Theorem I. If the power series !an xn is such that an ( 0 for all n and lim | an )1 / an | # 1/ R, n∋

n

then ! an x is convergent (indeed absolutely) for | x | < R and divergent for | x | > R.. Proof. Let

un = anxn lim

n∋

[Delhi Maths (H) 2006] un + 1 = an + 1 xn + 1. Then, we have

so that

un )1 a x a |x| # lim n )1 # | x | lim n )1 # n∋ n∋ un an an R

... (1)

/ By D’ Alembert’s ratio test, ! an x n converges absolutely if | x | / R 0 1, i.e., | x | < R. Also,

! an xn diverges if | x | > R. Theorem II. If the power series ! an xn is such that an ( 0 for all n and lim | an |1/ n # 1/ R, n∋

n

then !an x is convergent (indeed absolutely) for | x | < R and divergent for | x | > R. Proof. According to Cauchy’s second theorem on limits, if < | an | > is a sequence of positive lim | un |1/ n # lim

constants, then

n∋

n∋

un )1 , un

... (1)

provided the limit on the right side of (1) exists, whether finite or infinite. Also given that lim | un |1/ n # 1/ R

... (2)

lim | un )1 / un | # 1/ R

... (3)

n∋

(1) and (2) 1

/

n∋

Using (3), the result of the theorem follows from theorem I. In view of the above discussion, the radius of convergence R of the power series ! an xn can be determined as follows : R # 1 2 lim n∋

R#

an )1 an

R # 1 2 lim | an |1/ n

or if

lim

n∋

... (*)

n∋

an )1 #0 an

or

lim | an |1/ n # 0

n∋

an )1 or lim | an |1/ n # # n ∋ an Note 1. Every power series converges absolutely within its interval of convergence. Note 2. Observe that formula (*) is derived with the supposition of existence of the finite

R = 0,

if

lim

n∋

limit lim |an / an + 1| , that is, with the supposition that the power series !an xn contains all powers of x. Indeed for the power series !{(2 x)2n )1 /(2n ) 1)}, the coefficients of even powers of x are equal to zero, a2m = 0 and hence lim (a2n)1 / a2n ) # n∋

and lim (a2n) 2 / a2n)1 ) # 0. This shows that n∋

we cannot apply the formula (*) to the given power series. However, a direct application of D’ Alembert’s ratio test leads to the desired result:

Created with Print2PDF. To remove this line, buy a license at: http://www.software602.com/

7.4

Power Series

un = (2x)

Here, let

lim

/

n∋

2n + 1

/(2n + 1)

un + 1 = (2x)

so that

2n + 3

/(2n + 3)

u n )1 (2 x) 2 n ) 3 2 n ) 1 2 ) 1/ n # lim 3 # 4 | x |2 lim # 4 | x |2 2 n )1 n ∋ n ∋ un 2n ) 3 2 ) 3/ n (2 x)

Therefore, by D’ Alembert’s ratio test, the given power series converges absolutely if

4 | x |2 0 1

| x |2 0 1/ 4

or

or

| x | < 1/2.

Note. If the given power series is given in general form !an ( x ∗ x0 )n , then formula (*) is used to find the radius of convergence R. In such a case, we say that the given power series converges if | x – x0 | < R and diverges if | x – x0 | > R. The interval of convergence is given by ]x0 ∗ R, x0 ) R[ . 7.7. SOLVED EXAMPLES BASED ON ART. 7.6. Ex. 1. Find the radius of convergence of the following series x 14 3 2 14 34 5 3 (i) 2 ) 2 4 5 x ) 2 4 5 4 8 x ) ....

[Delhi Maths (H) 2006]

a 4 b a(a ) 1) b (b ) 1) (ii) 1 ) 1 4 c ) 1 4 2 c (c ) 1) ) ...

[Delhi Maths (H) 2003]

Sol. (i) Let the given series be denoted by ! an x n . n #1

Then, here

an #

1 4 3 4 5 ... (2n ∗ 1) 2 4 5 4 8...(3n ∗ 1)

/ Radius of convergence # lim n∋

an )1 #

and

1 4 3 4 5 ... (2n ∗ 1) (2n ) 1) 2 4 5 4 8 ...(3n ∗ 1) (3n ) 2)

an 3n ) 2 3)2/n 3 # lim # lim # n∋ n ∋ 2 ) 1/ n an )1 2n ) 1 2

(ii) Omitting the first term, let the given series be denoted by !an xn . Then, here we have an #

and

a (a ) 1)...(a ) n ∗ 1) b (b ) 1) ... (b ) n ∗ 1) 1 4 2 ... n c (c ) 1) ... (c ) n ∗ 1) an )1 #

a (a ) 1)...(a ) n ∗ 1) (a ) n) b (b ) 1) ... (b ) n ∗ 1) (b ) n) 1 4 2 ... n (n ) 1) c (c ) 1) ... (c ) n ∗ 1) (c ) n)

Radius of convergence # lim n∋

an ( n ) 1) (c ) n) (1 ) 1/ n) (1 ) c / n) # lim # lim #1 n∋ n∋ an )1 ( a ) n ) (b ) n ) (1 ) a / n) (1 ) b / n)

Ex. 2. Find the radius of convergence the exact interval of convergence of the power series (n ) 1) ! xn . (n ) 2) (n ) 3) Sol. Let the given series be denoted by

!an xn

an = (n + 1) / {(n + 2) (n + 3)}

and

/

R = radius of convergence # lim n∋

or

!un . Then, we have an + 1 = (n + 2) / {(n + 3) (n + 4)}

an (n ) 1) (n ) 4) # lim #1 n∋ an )1 (n ) 2)2

Created with Print2PDF. To remove this line, buy a license at: http://www.software602.com/

Power Series

7.5

Hence the given series converges for | x | < and diverges for | x | > 1. We now investigate the nature of the given power series when | x | = 1, i.e., when x = 1 and x = –1. un #

For x = 1,

(n ) 1) 1 (1 ) 1/ n) # 3 (n ) 2) (n ) 3) n (1 ) 2 / n) (1 ) 3 / n)

Let the companion series !vn be such that

... (1)

vn # 1/ n.

lim (un / vn ) # 1, which is finite and non zero.

Then,

n∋

Again, !vn # !(1/ n) is a divergent series. So by comparison test !un diverges for x = 1. Next, for x = –1, the given series is an alternating series for which un + 1 < un for each natural number n and lim un # 0, by (1). Hence, by Leibnitz’s test the given series converges for x = –1. n∋

Hence the exact interval of convergence is [–1, 1[. Ex. 3. Determine the interval of convergence of the power series !{(1/ n) 3 (∗1) n)1 ( x ∗ 1) n }. Sol. Let the given series be denoted by !an ( x ∗ x0 )n . Then, we have

an # (∗1)n )1 / n /

an )1 # (∗1)n) 2 /(n ) 1).

and

an n )1 # lim ∗ #1 n ∋ an )1 n

R = radius of convergence # lim n∋

Since the given power series is about the point x = x0 = 1, the interval of convergence is x0 ∗ R 0 x 0 x0 ) R,

i.e.,

–1 + 1 < x < 1 + 1,

i.e.,

0 < x < 2.

For x = 2, the given series reduces to the alternating series !(∗1)n ∗1 / n ( # !(∗1)n∗1 un , say) for which un + 1 < un for each natural number n and lim un # lim (1/ n) # 0. Hence by Leibnitz’ss n∋

n∋

test the given series is convergent when x = 2. Next, for x = 0, clearly the given series diverges. Hence the exact interval of convergence is ]0, 2].

EXERCISE 7 (A) Determine the radius of convergence and the exact interval of convergence of each of the following power series. 1. (i) ! 2. (i) ! (iv) !

nx n (n ) 1) 2

(2n)! x 2 n (n !)2

(n !)2 x 2 n (2n)!

n 3. (i) ( x ∗ 1) 2n

(ii) ! (ii) !

2n x n n!

(iii) !

(∗1)n x 2 n

(v) !(∗1)n

x 2n )1 (2n ) 1)

n

n

(iv) !

n3

(iii) !(∗1)n

(n !)2 2 2 n

(ii) ! (∗1) ( x ∗ 1) 2n (3n ∗ 1)

xn

xn nn

x2n)1 (2n )1)!

(iii) ! n !( x ) 2) nn

n

Created with Print2PDF. To remove this line, buy a license at: http://www.software602.com/

7.6

Power Series

4. If the power series !an xn has radius of convergence R, then prove that, for any positive integer m, !an xmn has radius of convergence R1/m. 1. (i) R = 1, [–1, 1[; 2. (i) R = 1/4; (v) R = 1, [–1, 1] 3. (i) R = 2, ] –1, 3[,

ANSWERS (ii) R # , R; (iii) R = 1, [–1, 1]

(iv) R #

(ii) R #

(iv) R = 4, ] – 4, 4 [

, R;

(iii) R = 1, [–1, 1]

(ii) R = 2, ] –1, 3]

, R

(iii) R = e, ] –2 – e, – 2

+ e[ 7.8. Some theorems about power series !an x n Theorem I. If a power series !an xn converges for x = x0, then (i) it is absolutely convergent in the interval | x | 0 | x0 | (ii) it is uniformly convergent in the interval | x | + | x1 |, where | x1 | 0 | x0 | . [Delhi Maths (H) 2004] Solution (i)

!an x0n is convergent 1 lim an x0n # 0 n∋

1 there exists a positive integer m such that

| an x0n

∗0| 01 , n ∃ m Now,

so that

| an | 0 1/ | x0 |n , , n ∃ m ... (1)

| an x n | # | an | | x |n 0 | x |n / | x0 |n , , n ∃ m, by (1)

Then, ... (2) | an x n | 0 (| x | / | x0 |)n , , n ∃ m The series on the R.H.S. of (2) converges for | x | < | x0 | (being a geometric series with common ratio < 1). Hence, by the comparison test !an xn is convergent for | x | 0 | x0 | . Therefore,

!an xn is absolutely convergent for | x | 0 | x0 | (ii) Let M n # | x1 |n / | x0 |n . Then !M n converges since | x1 | 0 | x0 | (being a geometric series with common ratio < 1). Now,

| an xn | # | an | | x |n 0 | x |n / | x0 |n , , n ∃ m using (1) 0 | x1 |n / | x0 |n , , n ∃ m since | x | + | x1 |

Thus,

| an x n | 0 M n , , n − N , where !M n converges.

Hence, from the Weierstrass M-test, it follows that !an xn is uniformly convergent in the interval | x | + | x1 | where | x1 | 0 | x0 | . Theorem II. If a power series !an xn converges for | x | 0 R and if a function f(x) is defined as

f ( x) # !an x n , | x | 0 R, then f ( x) # !an xn converges uniformly on [∗ R ) 5, R ∗ 5] for everyy 5 & 0. Proof. Let 5 & 0 be any given number. Then, we have | x |+ R∗5 ... (1) 1 | an xn | + | an | ( R ∗ 5)n Since every power series converges absolutely within its interval of convergence, it follows that !an ( R ∗ 5)n converges absolutely. Hence by Weierstrass’s M-test it follows that the series

!an xn converges uniformly on [∗ R ) 5, R ∗ 5].

Created with Print2PDF. To remove this line, buy a license at: http://www.software602.com/

Power Series

7.7

Theorem III. The series obtained by integrating and differentiating power series term by term has the same radius of convergence as the original series. Proof. Let R be the radius of convergence of the given power series

! an x n .

n# 0

... (1)

an x n )1 ... (2) n# 0 n ) 1 Let R6 be the radius of convergence of (2). Then, we have 1 (n ) 1)1/ n R # lim 6 R # lim and ... (3) n ∋ | a |1/ n n∋ | an |1/ n n log(1 ) n) 1/(n ) 1) Let l # lim ( n ) 1)1/ n so that log l # lim # lim , by L’ Hopital’s rule n∋ n∋ n∋ n 1 log l = 0 l = e0 = 1 1 lim (n ) 1)1/ n # 1 ... (4) 1 1 !

On integrating (1) term by term, we get

(3) 1

Using (4),

n∋

R 6 # R.

! nan x n ∗1

Next, differentiating (1) term by term, we get

n #1

Let R 66 be the radius of convergence of (5). Then 1 1 1 R 66 # lim 1/ n # lim 1/ n 3 lim 1/ n n∋ n n ∋ n ∋ | an | n | an |1/ n Let

1

m # lim n1/ n

log m = 0

n∋

so that

1

... (5)

... (6)

log n (1/ n) # lim , by L’ Hopital’s rule n∋ n 1 m = e0 = 1 ... (7) lim n1/ n # 1 1 log m # lim n∋

n∋

R 66 # R.

From (3), (6) and (7), we have

Exercise. Show that both the power series ! an x n and corresponding series of derivatives n #0

! n an x n ∗1 have the same radius of convergence.

n #1

[Delhi Maths (H) 2001]

[Hint. Refer first part of the above theorem III.] Theorem IV. Let the given power series !an xn converges for | x | 0 R and let f ( x) # !an x n . Then (i) f(x) is continuous in ] – R, R [. [Delhi Maths (H) 1995] (ii) !an xn can be integrated term by term in ] – R, R [. (iii) !an xn can be differentiated term by term in ] – R, R [. [Note : If f ( x) # !an xn , then f(x) is known as the sum function of the series.] Proof (i) Since each term of the series !an xn is continuous on ] – R, R [ and !an xn is uniformly convergent on [∗ R ) 5, R ∗ 5], hence the sum function f(x) of !an xn is also continuous. (ii) Since each term of !an xn is continuous on [ – R, R [ and !an xn is uniformly covergent in [ ∗ R ) 5, R ∗ 5], hence !an xn is term by term integrable. (iii) Since each term of !an xn in continuous, possess continuous derivatives in ] ∗ R, R [ and !an xn is uniformly convergent in [∗ R ) 5, R ∗ 5], hence !an x n is term by term differentiable. Exercise. Let the power series !an xn converge for | x | < R, and f ( x) # !an x n , | x | 0 R, prove that ! an xn converge uniformly on [∗ R ) 5, R ∗ 5], no matter which 5 & 0 is chosen, and that the function f is continuous and differentiable on [–R, R]. [Delhi Maths (H) 2007]

Created with Print2PDF. To remove this line, buy a license at: http://www.software602.com/

8 Integration In Series 8.1. INTRODUCTION In may happen that a given linear differential equation comes under none of the standard classes which are all of some particular form, and thus we may fail to express its solution in terms of elementary functions, namely, polynomials, rational functions, exponentials, trigonometric fucntions, hyperbolic functions, logarithms etc. In such situations we have to find a convergent series arranged according to powers of the independent variable, which will approximately express the value of the dependent variable. The solution in the form of an infinite series is called ‘integration in series’. In this chapter we propose to discuss some methods of getting solution in the form of infinite series for second order linear equation. 8.2. Some basic definitions. (Jabalpur 2004) Power series. An infinite series of the form #

∃ Cn ( x ! x0 )n = C0 + C1(x – x0) + C2(x – x0)2 + ...

...(1)

n 0

is called a power series in (x – x0). In particular, a power series in x is an infinite series #

2 ∃ Cn x n = C0 + C1x + C2x + ...

...(2)

n 0

For example, the exponential function ex has the power series ex =

#

n

∃ xn! n 0

2

3

1 % x % x % x % ... 2 ! 3!

The power series (1) converges (absolutely) for | x | < R, where R = lim

n&#

Cn , provided the limit exists. Cn % 1

...(3)

R is said to be the radius of convergence of power series (1). The interval (–R, R) is said to be the interval of convergence. Since R = # for the power series (2), hence the interval of convergence of the power series (2) is (–#, #) i.e. the real line. In what follows we shall use the following results : (i) A power series represents a continuous function within its interval of convergence. (ii) A power series can be differentiated termwise within its interval of convergence. For more results about power series, refer chapter 7. Analytic function. A function f(x) defined on an interval containing the point x = x0 is called #



analytic at x0 if its Taylor series.

n 0

f ( n ) ( x0 ) ( x ! x0 ) n!

...(4)

exists and converges to f(x) for all x in the interval of convergence of (4). 8.1

Created with Print2PDF. To remove this line, buy a license at: http://www.software602.com/

8.2

Integration In Series x

Hence, we find that all polynomial functions, e , sin x, cos x, sinh x and cosh are analytic everywhere. A rational function is analytic except at those values of x at which its denominator is zero. For example, the rational function defined by x/(x2 – 3x + 2) is analytic everywhere except at x = 1 and x = 2. 8.3. Ordinary and singular points. [Nagpur 1996, Ravishankar 1993] Definitions. A point x = x0 is called an ordinary point of the equation y∋∋ + P(x)y∋ + Q(x)y = 0 ...(1) if both the functions P(x) and Q(x) are analytic at x = x0. If the point x = x0 is not an ordinary point of the differential equation (1), then it is called a singular point of the differential equation of (1). There are two types of singular points : (i) regular singular points and (ii) irregular singular points. A singular point x = x0 of the differential equation (1) is called a regular singular point of the differential equation (1) if both (x – x0) P(x) and (x – x0)2Q(x) are analytic at x = x0. A singular point, which is not regular is called an irregular singular point. 8.4. Solved examples based on Art. 8.3. Ex. 1. Determine whether x = 0 is an ordinary point or a regular singular point of the differential equation 2x2(d2y/dx2) + 7x(x + 1) (dy/dx) – 3y = 0. [Delhi Maths (Hons) 1993, 2000] Sol. Dividing by 2x2, the given equation becomes

d 2 y 7( x % 1) dy 3 % ! 2 y 0. 2 2 x dx 2 x dx

...(1)

Comparing (1) with standard equation y( + P(x) y∋ + Q(x) y = 0, we have P(x) = [7(x + 1)]/2x and Q(x) = –3/(2x2). ...(2) Since both P(x) and Q(x) are undefined at x = 0, so both P(x) and Q(x) are not analytic at x = 0. Thus x = 0 is not an ordinary point and so x = 0 is a singular point. Also, (x – 0)P(x) = 7(x + 1)/2 and (x – 0)2 Q(x) = –3/2, 2 showing that both (x – 0)P(x) and (x – 0) Q(x) are analytic at x = 0. Therefore x = 0 is a regular singular point. Ex. 2. Show that x = 0 is an ordinary point of (x2 – 1)y( + xy∋ – y = 0, but x = 1 is a regular singular point. [Ranchi 2010] Sol. Dividing by (x2 – 1), the given equation becomes 2

d y dy x 1 % ! y = 0. ...(1) 2 ( x ! 1 )( x % 1 ) dx ( x ! 1 )( x % 1) dx Comparing (1) with with standard equation y( + P(x)y∋ + Q(x)y = 0, we have P(x) = x/[(x – 1)(x + 1)] and Q = –1/{(x – 1) (x + 1)}. Since both P(x) and Q(x) are analytic at x = 0, so x = 0 is an ordinary point of the given equation (1). Since both P(x) and Q(x) are undefined at x = 1, so they are not analytic at x = 0. Thus x = 1 is not an ordinary point and so x = 1 is a singular point. Also (x – 1)P(x) = x/(x + 1) and (x – 1)2Q(x) = –(x – 1)/(x + 1), showing that both (x – 1)P(x) and (x – 1)2Q(x) are analytic at x = 1. Therefore x = 1 is a regular singular point. Ex. 3. Show that x = 0 and x = –1 are singular points of x2(x + 1)2y( + (x2 – 1)y∋ + 2y = 0, where the first is irregular and the other is regular. Sol. Dividing by x2(x + 1)2, the given equation becomes

Created with Print2PDF. To remove this line, buy a license at: http://www.software602.com/

Integration In Series

8.3

d2y x ! 1 dy % 2 2 2 y = 0. 2 % 2 dx dx x ( x % 1) x ( x % 1)

...(1)

Comparing (1) with standard equation y( + P(x)y∋ + Qy = 0, we get P(x) = (x – 1)/[x2(x + 1)] and Q(x) = 2/[x2(x + 1)2]. Since both P(x) and Q(x) are undefined at x = 0 and x = –1, so they are not analytic at x = 0 and x = –1. Hence x = 0 and x = –1 are both singular points. Also (x – 0)P(x) = (x – 1)/[x(x + 1)] and (x – 0)2Q(x) = 2/(x + 1)2, showing that P(x) is not analytic at x = 0 and so x = 0 is an irregular singular point. Again, (x + 1)P(x) = (x – 1)/x2 and (x + 1)2Q(x) = 2/x2, showing that both (x + 1)P(x) and (x + 1)2Q(x) are analytic at x = – 1 and hence x = –1 is a regular singular point. x

Ex. 4. Discuss the singularities of the equation x2 y ∋∋ % x y ∋ % ( x 2 ! n2 ) y 0 at x = 0 and (Delhi Physics (Hons.) 2000, 02; Bhopal 2010) #. Sol. Discussion about singularity at x = 0. Re-writing the given equation

y∋∋ % (1/ x) y∋ % {( x 2 ! n 2 ) / x2 } y y ∋∋ % P ( x) y ∋ % Q( x ) y

Comparing (1) with

Q ( x)

0,

... (1)

0

P ( x ) 1/ x

and

( x 2 ! n 2 ) / x2 .

Here (x – 0) P(x) = 1 and (x – 0)2 Q(x) = x2 – n2, showing that both (x – 0) P(x) and (x – 0)2 Q(x) are analytic at x = 0. Therefore, x = 0 is a regular singular point. Discussion about singularity at x #. Let x = 1/t or t = 1/x. Then, dt/dx = –1/x2 ... (2)

y∋

Now,

y ∋∋

and

d2y dx

2

y ∋∋

or

dy dx

dy dt dt dx

d ) dy ∗ + , dx − dx .

dy ) 1 ∗ +! , dt − x 2 .

d ) dy ∗ dt + , dt − dx . dx

!t 2

dy , by (2) dt

d ) 2 dy ∗ ) 1 + !t ,+! dt − dt . − x 2

) 2 d2y dy ∗ ! 2t ,, / (!t 2 ) ++ !t 2 dx . dt −

t4

d2y dt 2

% 2t 3

... (3) ∗ , , by (2) and (3) .

dy dt

... (4)

Using (2), (3) and (4), the given equation reduces to 1 ) 4 d2y dy ∗ 1 ) dy ∗ ) 1 ∗ t % 2 t 3 , % + !t 2 , % + 2 ! n 2 , y 2 + 2 + , dt . t − dt . − t t − dt .

0

or

t2

d2y dt 2

%t

(d 2 y / dt 2 ) % (1/ t ) / (dy / dt ) % {(1 ! n2t 2 ) / t 4 } 0

or

Comparing (5) with (d 2 y / dt 2 ) % P(t ) (dy / dt ) % Q (t ) y P(t) = 1/t

and

Q (t )

dy 1 ! n2 t 2 % y dt t2

0

... (5)

0, here

(1 ! x 2t 2 ) / t 4 . Then, we have

(t – 0) P(t) = 1 and (t – 0)2 Q(t) = (1 – n2 t2)/t2. 2 Since (t – 0) Q(t) is not analytic at t = 0, so t = 0 is irregular singular point of (5). In view of (2), x # is an irregular singular point of the given equation.

Created with Print2PDF. To remove this line, buy a license at: http://www.software602.com/

8.4

Integration In Series

EXERCISE 8 (A) 1. Show that x = 0 is an ordinary point of y( – xy∋ + 2y = 0. 2. Determine whether x = 0 is an ordinary point or regular singular point for the differential equation 2x2y( – xy∋ + (x – 5)y = 0. Ans. x = 0 is regular singular point 3. Show that x = 0 is an ordinary point of (x2 + 1)y( + xy∋ – xy = 0. 4. Show that x = 0 is a regular singular point of x2y∋∋ + xy∋ + (x2 – 1/4)y = 0. 5. Show that x = 0 is a regular singular point and x = 1 is an irregular singular point of x(x – 1)3y( + 2(x – 1)3y∋ + 3y = 0. 6. Verify that origin is regular singular point of the equation 2x2y( + xy∋ – (x + 1)y = 0. 7. Determine the nature of the point x = 0 for the equations (i) xy( + y sin x = 0 [Nagpur 1996] (ii) x3y( + y sin x = 0 [Nagpur 2005] Ans. (i) Regular singular point (ii) Irregular singular point 8. Determine the singular points and their nature for the following differential equations : (i) 3xy( + 2x(x – 1)y∋ + 5y = 0 (ii) y( + (1 – x)y∋ + (1 – x)2 y = 0 [Utkal 2003] Ans. (i) x = 0 is regular singular point (ii) There is no singular point. 8.5. Power series solution in power of (x – x0) or the power series solution near the ordinary point x = x0 or power series solution about the ordinary point x = x0. Let the given equation be y( + P(x)y∋ + Q(x)y = 0. ...(1) If x = x0 is an ordinary point of (1), then (1) has two non–trivial linearly independent power series solutions of the form #

n

∃ Cn ( x ! x0 )

n

...(2)

0

and these power series converge in some interval of convergence | x – x0 | < R, (where R is the radius of convergence of (2)) about x0. In order to get the coefficients Cn’s is (2), we take y=

#

n ∃ Cn ( x ! x0 ) .

n

...(3)

0

Differentiating twice in succession, (3) gives y∋ =

#

∃nC n

1

n

( x ! x0 ) n ! 1

and

y( =

#

∃ n ( n ! 1) C

n

n

( x ! x0 ) n ! 2

. ...(4)

2

Putting the above values of y, y∋ and y( in (1), we get an equation of form A0 + A1(x – x0) + A2(x – x0)2 + .... + An(x – x0)n + .... = 0, ...(5) where the coefficients A0, A1, A2 .... etc. are now some functions of the coefficients C0, C1, C2, .... etc. Since (5) is an identity, all the coefficients A0, A1, A2, .... of (5) must be zero, i.e., A0 = 0, A1 = 0, A2 = 0, ...., An = 0. ...(6) Solving equation (6), we obtain the coefficients of (3) in terms of C0 and C1. Substituting these coefficients in (3), we obtain the required series solution of (1) in powers of (x – x0). 8.6. Solved examples based on Art. 8.5. Ex. 1. Find the power series solution of the equation (x2 + 1)y( + xy∋ – xy = 0 in powers of x (i.e. about x = 0). [Delhi B.Sc. (Hons.) 1993, 2000, 06, 07] 2 Sol. Given that (x + 1)y( + xy∋ – xy = 0. ...(1) Dividing by (x2 + 1), (1) can be written in standard form as

d2 y dy % 2x ! 2 x y = 0. 2 dx dx x %1 x %1 Comparing (2) with y( + P(x)y∋ + Q(x)y = 0, here we have P(x) = x/(x2 + 1) and

...(2) Q(x) = –x/(x2 + 1),

Created with Print2PDF. To remove this line, buy a license at: http://www.software602.com/

Integration In Series

8.5

showing that P(x) and Q(x) are analytic at x = 0. So x = 0 is an ordinary point. Therefore, to solve y = C0 + C1x + C2x2 + C3x3 + .... =

(1), we take power series

n

Differentiating (3) twice in succession w.r.t. ‘x’, we get #

n!1 y∋ = ∃ nCn x n 1

#



or

n

n 1

n ( n ! 1) C n x

n

%

2

#



n

n ( n ! 1) C n x

n ! 2

!

2

#

#

n ∃ Cn x = 0

n

#

∃ nC n

x

n

0

n

!

1

#

∃C

n

#

n

xn % 1

=0

0

#

#

n 1

n 1

n n n n ∃ n(n ! 1)Cn x % ∃ (n % 2 )(n % 1)Cn % 2 x % ∃ nCn x ! ∃ Cn ! 1 x = 0

or

n

or

2

...(3)

n 1

( x 2 % 1) ∃ n(n ! 1)Cn x n ! 2 % x ∃ nCn x n ! 1 ! x n

xn

#

Substituting the above values of y, y∋ and y( in (1), we get

#

n

0

n!2 y( = ∃ n(n ! 1) Cn x ....(4)

and #

#

∃C

2

n

2 C2 % (6C3 % C1 ! C0 ) x %

0

#

n ∃ [n(n ! 1)Cn % (n % 2 )(n % 1)Cn % 2 % nCn ! Cn ! 1 ]x = 0.

...(5)

n 2

Since (5) is an identity, equating the constant term and the coefficients of various powers of x to zero, we get 2C2 = 0 so that C2 = 0 ...(6) 6C3 + C1 – C0 = 0 so that C3 = (C0 – C1)/6 ...(7) n(n – 1)Cn + (n + 2)(n + 1)Cn + 2 + nCn – Cn – 1 = 0 for all n 0 2 2

Cn + 2 =

or

Cn ! 1 ! n Cn (n % 1)(n % 2)

,

n 0 2.

for all

The above relation (8) in known as recurrence relation. Putting n = 2 is (8), C4 = (C1 – 4C2)/12 = C1/12, Putting n = 3 in (8),

or

C5 = !

9C3 20

!

as

...(8) C2 = 0. ...(9)

9 ) C0 ! C1 ∗ 3 + , ! (C0 ! C1 ) . 20 − 6 . 40

...(10)

Putting the above values of C2, C3, C4, C5, .... etc. in (3), we have y = C0 + C1x + C2x2 + C3x3 + C4x4 + C5x5 + .... ad. inf. y = C0 + C1x + (1/6) × (C0 – C1)x3 + (1/12) × C1x4 – (3/40) × (C0 – C1)x5 + ....

FH

IK

FH

IK

y = C0 1 % 1 x 3 ! 3 x 5 % .... % C1 x ! 1 x 3 % 1 x 4 % 3 x 5 ! .... , 6 40 6 12 40 which is the required solution near x = 0, where C0 and C1 are arbitrary constants. Ex. 2(a). Find the solution in series of (d2y/dx2) + x(dy/dx) + x2y = 0 about x = 0. [Delhi Maths (Hons.) 2005, 08; Ranchi 2010] 2 (b) Solve y( – xy∋ + x y = 0 in powers of x [Guwahati 2007] Sol. (a) Given that y( + xy∋ + x2y = 0. ...(1) Comparing (1) with y( + P(x)y∋ + Q(x)y = 0, here P(x) = x and Q(x) = x2. Since P(x) and Q (x) are both analytic at x = 0, it follows that x = 0 is an ordinary point. To solve (1), we take or

y = C0 % C1 x % C2 x 2 % C2 x 3 % .... Differentiating (2) twice in succession w.r.t. ‘x’, #

y∋ = ∃ Cn nx n ! 1 n 1

and

#

n ∃ Cn x .

n

...(2)

0

#

y( = ∃ Cn n(n ! 1) x n ! 2 . n

...(3)

2

Created with Print2PDF. To remove this line, buy a license at: http://www.software602.com/

8.6

Integration In Series #

Putting the above values of y, y∋ and y( is (1), ∃Cnn(n ! 1)x

n!2

n 2

#

∃ Cn n(n ! 1) x

or

n!2

n 2

#

#

n 1

n 0

#

n

#

n

n 1

∃ [(n % 1)(n % 2)C

n

#

n

n 0

=0

=0

n 2

#

n

2

% x ∃Cn x

% ∃ Cn nx n % ∃ Cn x n % 2 = 0

n 0

2C2 % (6C3 % C1 ) x %

n !1

n 1

∃ Cn % 2 (n % 2)(n % 1)x % ∃ Cn nx % ∃ Cn ! 2 x

or or

#

#

% x ∃Cnnx

% nCn % Cn ! 2 ]x n = 0.

n%2

...(4)

2

Since (4) is an identity, equating the constant term and the coefficients of various powers of x to zero, we get 2C2 = 0 so that C2 = 0 ...(5) 6C3 + C1 = 0 so that C3 = – (C1/6) ...(6) (n + 1)(n + 2)Cn + 2 + nCn + Cn – 2 = 0, for all n02 Cn + 2 = !

or

C4 = !

Putting n = 2 in (7), Putting n = 3 in (7),

nCn % Cn ! 2

C5 = !

3C3 % C1 20

!

2C2 % C0 12

...(7)

!

C0 , by (5). 12

...(8)

!3 / (C1 / 6) % C1 20

C6 = !

Putting n = 4 in (7),

n 0 2.

for all

(n % 1)(n % 2)

4C4 % C2 30

!

!

C1 , by (6) 40

! (C0 / 3) 30

C0 , by (8) 90

and so as. Putting these values in (1), we get y = C0 + C1x – (1/6) × C1x3 – (1/12) × C0x4 – (1/40) × C1x5 + (1/90) × C0x6 + ....

FH

IK

FH

IK

1 4 1 6 1 3 1 5 x % x ! .... % C1 x ! x ! x ! .... , y = C0 1 ! 12 90 6 40 which is the required general solution about x = 0, where C0 and C1 are arbitrary constants. (b) Ans. y = C0 (1 –x4/12 – x6/90 + ...) + C1 (x + x3/16 – x5/40 + ....) Ex. 3. Find the general power series solution near x = 0 of the Legendre’s equation (1 – x2)(d2y/dx2) – 2x(dy/dx) + p(p + 1)y = 0, where p is an arbitrary constant. [Delhi Maths (Hons.) 2006] Sol. Given (1 – x2)y( – 2xy∋ + p(p + 1)y = 0. ...(1) 2 2 or y( – [(2x)/(1 – x )]y∋ + [p(p + 1)/(1 – x )]y = 0. ...(2) Comparing (2) with y( + P(x)y∋ + Q(x)y = 0, we have P(x) = –(2x)/(1 – x2) and Q(x) = p(p + 1)/(1 – x2), showing that both P(x) and Q(x) are analytic at x = 0 and hence x = 0 is an ordinary point of (1).

or

To solve (1), let

y = C0 % C1 x % C2 x 2 % C3 x 3 % ....

#

n ∃ Cn x .

n

...(3)

0

Differentiating (3) twice in succession w.r.t. ‘x’, we get #

y∋ = ∃Cnnx n !1

and

n 1

#

y( = ∃ Cn n(n ! 1)x n 2

n!2

. ...(4)

Putting the above values of y, y∋ and y( in (1), we get

Created with Print2PDF. To remove this line, buy a license at: http://www.software602.com/

Integration In Series

8.7 #

#

#

(1 ! x 2 ) ∃ Cn n(n ! 1)x n ! 2 ! 2 x ∃ Cn nx n ! 1 % p( p % 1) ∃ Cn x n = 0 n 2

#

∃ Cn n(n ! 1) x

or

n

n 1

n!2

2

!

#

n 0

#

#

n n n ∃ Cn n( x ! 1) x ! 2 ∃ Cn nx % p( p % 1) ∃ Cn x = 0

n

#

2

n 1 #

#

n

0

#

n n n ∃ Cn % 2 (n % 2)(n % 1)x ! ∃ Cn n(n ! 1)x ! ∃ 2Cn nx % ∃ p( p % 1)Cn x n = 0

or

n

0

n 2

or [2C2 + p(p + 1)C0] + [6C3 – 2C1 + p(p + 1)C1]x +

n 1

n 0

#

∃[(n % 1)(n % 2)Cn % 2 ! n(n ! 1) Cn ! 2nCn ! p( p % 1)Cn ]x

n

= 0

n 2

#

or [2C2 + p(p + 1)C0] + [6C3 + (p2 + p – 2)C1]x + ∃[(n % 1)(n % 2)Cn % 2 ! {n(n ! 1) % 2n ! p( p % 1)}Cn ]xn = 0 n 2 #

or [2C 2 + p(p + 1)C 0] + [6C3 + (p – 1)(p + 2)C1]x % ∃ [(n % 1)( n % 2)Cn % 2 % {( p 2 ! n 2 ) % ( p ! n)}Cn ] x n n

0

2

#

n

or [2C2+ p(p + 1)C0] + [6C3 + (p – 1)(p + 2)C1]x + ∃ [(n%1)(n %2)Cn%2 %(p! n)(p % n%1)Cn]x = 0. ...(5) n 2

Since (5) is an identity, we equate the coefficients of various powers of x to zero and obtain C2 = ! p( p % 1) C0 2! ( p ! 1)( p % 2) 6C3 + (p – 1)(p + 2)C1 = 0 so that C3 = ! C1 3! (n + 1)(n + 2)Cn + 2 + (p – n)(p + n + 1)Cn = 0 ( p ! n)( p % n % 1) or Cn + 2 = ! Cn , for n 0 2. (n % 1)(n % 2) Putting n = 2, 3, .... in (8) and using (6) and (7), we have 2C2 + p(p + 1)C0 = 0

so that

C4 = !

( p ! 2)( p % 3) C2 413

p ( p ! 2)( p % 1)( p % 3) C0 4!

C5 = !

( p ! 3)( p % 4) C3 514

( p ! 1)( p ! 3)( p % 2)( p % 4) C1 5!

...(6) ...(7) ...(8)

and so on. Putting the above values of C2, C3, C4, C5, .... in (3), we get y = C0 % C1 x ! or

LM N

y = C0 1 !

p ( p ! 2)( p % 1)( p % 3) p( p % 1) ( p ! 1)( p % 2) C0 x 4 % .... C0 x 2 ! C1x 3 % 2! 3! 4!

p( p % 1) 2 p( p ! 2)( p % 1)( p % 3) 4 x % x ! .... ad. inf 2! 4!

LM N

OP Q

OP Q

( p ! 1)( p % 2) 3 ( p ! 1)( p ! 3)( p % 2)( p % 4) 5 x % x ! .... ad. inf , 3! 5! which is the required general solution, C0 and C1 being arbitrary constants. Ex. 4. Solve y( – xy∋ – py = 0, where p is any constant. Sol. Given y( – xy∋ – py = 0. ...(1) Comparing (1) with y( + P(x)y∋ + Q(x)y = 0, here P(x) = –x and Q(x) = –p. Since P(x) and Q(x) are both analytic at x = 0, so x = 0 is an ordinary point. To solve (1), we take

+ C1 x !

#

n

y = C0 + C1x + C2x2 + C2x3 + C4x4 + .... = ∃ Cn x . n 0

...(2)

Created with Print2PDF. To remove this line, buy a license at: http://www.software602.com/

8.8

Integration In Series

Differentiating (2) twice in succession w.r.t. ‘x’, we get #

y∋ = ∃ Cn nx n ! 1

#

y( = ∃ Cn n(n ! 1)x n ! 2 .

and

n 1

...(3)

n 2

Putting the above values of y, y∋ and y( in (1), we have #

∃ C n(n ! 1) x n

n!2

!x

n 2

#

∃ C nx n

n 1

!p

#

∃C x n

n

#

∃ n(n ! 1)Cn x

= 0 or

n!2

n 2

n 0

#

#

n

n 1

#

n

#

n

#

n

! ∃ nCn x ! ∃ pCn x = 0 n 0

n

∃ (n % 2)(n % 1)Cn % 2 x ! ∃ nCn x ! ∃ pCn x = 0

or or

n !1

n 0

n 1

(2C2 – pC0) + (6C3 – C1 – pC1)x +

n 0

#

∃ [(n % 2)(n % 1)Cn % 2 ! nCn ! pCn ] = 0

n 2

#

or

(2C2 – pC0) + {6C3 – (p + 1)C1}x +

∃ [(n % 2)(n % 1)Cn % 2 ! ( p % n)Cn ] = 0. ...(4)

n 2

Since (4) is an identity, we equate the coefficients of various powers of x to zero and obtain 2C2 – pC0 = 0 so that C2 = (p/2)C0 ...(5) 6C3 – (p + 1)C1 = 0 so that C3 = [(p + 1)/6]C1 ...(6) (n + 2)(n + 1)Cn + 2 – (p + n)Cn = 0 so that Cn + 2 =

p%n C , for all n 0 2 ( n % 1)(n % 2) n

...(7) Putting n = 2, 3, 4, .... in (7) and using (5) and (6), we get p ( p % 2) p%3 C0 , C5 = C3 4! 415 and so on. Putting these values in (3), we obtain

C4 =

p%2 C2 31 4

p%2 p / C0 31 4 2

p % 3 p %1 / C1 4 15 2 13

( p % 1)( p % 3) C1 5!

p p %1 p( p % 2) ( p % 1)( p % 3) 5 C1 x % .. C0 x 2 % C1 x 3 % C0 x 4 % 5 ! 2 6 4! p 2 p( p % 2) 4 p % 1 3 ( p % 1)( p % 3) 5 or y = C0 1 % x % x % ... + C1 x % x % x % ... 2! 4! 3! 5! which is the required solution, C0 and C1 being arbitrary constants. Ex. 5(a). Find the general solution of y( + (x – 3)y∋ + y = 0 near x = 2. (b) Obtain power series solution of y( + (x – 1)y∋ + y = 0 in powers of (x – 2). Sol. Given y( + (x – 3)y∋ + y = 0. ...(1) Comparing (1) with y( + P(x)y∋ + Q(x)y = 0, here P(x) = x – 3 and Q(x) = 1. Since both P(x) and Q(x) are analytic at x = 2, so x = 2 is an ordinary point of (1). To find solution near x = 2, we shall find series solution in powers of (x – 2). We assume that y = C0 % C1x %

LM N

OP Q

LM N

OP Q

y = C0 % C1( x ! 2) % C2 ( x ! 2)2 % C3( x ! 2)3 % ....

#

n ∃ Cn (x ! 2) .

...(2)

n 0

Differentiating (2) twice in succession w.r.t. ‘x’, we get #

y∋ = ∃ nCn ( x ! 2) n 1

n !1

and

#

y( = ∃ n(n ! 1)Cn ( x ! 2) n

2

n!2

. ...(3)

Putting the above values of y, y∋ and y( in (1), we get

Created with Print2PDF. To remove this line, buy a license at: http://www.software602.com/

Integration In Series #



n

8.9

n(n ! 1)Cn ( x ! 2) n ! 2 % ( x ! 3)

2

#

n!2

n 2 #

∃ n(n ! 1)Cn ( x ! 2)

or

n!2

n 2 #

or



n

or

∃ nC ( x ! 2) n

n !1

#

+ ∃ Cn ( x ! 2)n = 0 n 0

n 1

∃ n(n ! 1)Cn ( x ! 2)

or

#

#

% [( x ! 2) ! 1] ∃ nCn ( x ! 2)

n !1

#

n

+ ∃ Cn ( x ! 2) = 0 n 0

n 1

#

#

#

n 1

n 1

n 0

% ∃ nCn ( x ! 2)n ! ∃ nCn ( x ! 2)n ! 1 % ∃ Cn ( x ! 2)n = 0

(n % 2)(n % 1)Cn % 2 ( x ! 2) n %

0

#

∃ nC ( x ! 2) n

n

n 1

#

#

n

n

! ∃ (n % 1)Cn % 1 ( x ! 2 ) % ∃ Cn ( x ! 2) = 0 n 0

n

#

0

n

(2C2 ! C1 % C0 ) % ∃ [(n % 2)(n % 1)Cn % 2 % nCn !( n % 1)Cn % 1 % Cn ]( x ! 2) = 0.

...(4)

n 2

which is an identity. Equating to zero the coefficients of various powers of (x – 2), we get 2C2 – C1 + C0 = 0 so that C2 = (C1 – C0)/2. ...(5) (n + 2)(n + 1)Cn + 2 + (n + 1)Cn – (n + 1)Cn + 1 = 0 for all n 0 1 or Cn + 2 = (Cn + 1 – Cn)/(n + 2), for all n 0 1. ...(6) Putting n = 1, 2, 3, .... in (6) and using (5) etc., we get C3 = C4 =

C3 ! C2 4

C2 ! C1 3

LM N

1 C1 ! C0 ! C1 3 2

LM N

1 C0 % C1 C1 ! C0 ! ! 4 6 2

OP Q

OP Q

!

C0 % C1 6

...(7)

1 1 C ! C. 12 0 6 1

...(8)

and so on. Putting these values in (2), the required solution near x = 2 is

FG C ! C IJ(x ! 2) ! FG C H 2 K H

IJ K

FH

IK

% C1 ( x ! 2)3 % 1 C0 ! 1 C1 ( x ! 2) 4 % .... 6 12 6 or y = C0[1 – (1/2) × (x – 2)2 – (1/6) × (x – 2)3 – (1/12) × (x – 2)4 + .... ad. inf.] + C1[(x – 2) + (1/2) × (x – 2)2 – (1/6) × (x – 2)3 – (1/6) × (x – 2)4 + .... ad. inf.] (b) Do as in part (a) yourself. Ex. 6. Find the power series solution in powers of (x – 1) of the initial value problem xy( + y∋ + 2y = 0, y(1) = 1, y∋(1) = 2. [Purvanchal 2007; CDLU 2004] Sol. Given equation is y( + (1/x)y∋ + (2/x)y = 0. ...(1) Comparing (1) with y( + P(x)y∋ + Q(y) = 0, here P(x) = 1/x and Q(x) = 2/x, which are analytic at x = 1. Hence x = 1 is an ordinary point of (1). To find solution near x = 1, we shall find series solution in powers of (x – 1).

y = C0 % C1(x ! 2) %

1

0

2

0

#

y = C0 + C1(x – 1) + C2(x – 1)2 + C3(x – 1)3 + .... = ∃ C n ( x ! 1) n .

Let

n

...(2)

0

Differentiating (2) twice in succession w.r.t. ‘x’, we get #

y∋ = ∃ nCn ( x ! 1)n ! 1 n 1

and

#

y( = ∃ n(n ! 1)Cn ( x ! 1)n ! 2 . ...(3) n 2

Putting these values of y, y∋ and y( in given equation, we get

Created with Print2PDF. To remove this line, buy a license at: http://www.software602.com/

8.10

Integration In Series #

x ∃ n(n ! 1)Cn ( x ! 1) n

n!2

2

#

#

% ∃ nCn ( x ! 1) n 1

[( x ! 1) % 1] ∃ n(n ! 1)Cn ( x ! 1)

or

n!2

n 2

or

#

or

#

#

n

2

n

=0

0

n !1

%2

n 1

#

n

#

% 2 ∃ Cn ( x ! 1)

% ∃ nCn ( x ! 1)

n !1 % ∃ n(n ! 1)Cn ( x ! 1)n ! 2 % ∃ n(n ! 1)Cn ( x ! 1)

n 2

n !1

#

∃ C ( x ! 1) n

n

=0

n 0 #



nCn ( x ! 1)n ! 1 % 2

n 1

#

∃ C ( x ! 1) n

n

=0

n

= 0,

n 0

#

n n ∃ (n % 1)nCn % 1( x ! 1) % ∃ (n % 2)(n % 1)Cn % 2 ( x ! 1)

n 1

n

0

%

#



(n % 1)Cn % 1 ( x ! 1)n % 2

n 0

#

∃ C ( x ! 1) n

n 0

which is an identity. Equating to zero the coefficients of various powers of (x – 1), we get 2C2 + C1 + 2C0 = 0 so that C2 = –(C1 + 2C0)/2. ...(4) and (n + 1)nCn + 1 + (n + 2)(n + 1)Cn + 2 + (n + 1) Cn + 1+ 2Cn = 0, for all n 0 1 or (n + 1)(n + 2)Cn + 2 + (n + 1)2Cn + 1 + 2Cn = 0, for all n 0 1 or

(n % 1)2 Cn % 1 % 2Cn , for all n 0 1. ...(5) (n % 1)(n % 2) Given that y = 1 and y∋ = 2 when x = 1. Hence putting x = 1 in (2) and (3), we have C0 = 1 and C1 = 2. ...(6) Using (6), (4) gives C2 = –(2 + 2)/2 = –2. ...(7) Putting n = 1, 2, 3, ... (5) and using (6) and (7) etc., we get Cn + 2 = !

C3 = !

22 C2 % 2C1 2 13

!

4 / (!2) % (2 / 2) 213

32 C3 % 2C2 2 , C4 = ! 31 4 3

!

9 / (2 / 3) % 2 / (!2) 31 4

!

1 6

42 C4 % 2C3 16 / (!1/ 6) % 2 / (2 / 3) 1 . ! 415 415 15 and so on. Putting these values in (2), we have y = 1 + 2(x – 1) – 2(x – 1)2 + (2/3) × (x – 1)3 – (1/6) × (x – 1)4 + (1/15) × (x – 1)5 + .... Note. Ex. 5 and 6 can also be solved by shifting the origin. The following example is given to provide an alternative method of solving 5 and 6. Ex. 7. Find the power series solution of the initial value problem (x2 – 1)y( + 3xy∋ + xy = 0, y(2) = 4, y∋(2) = 6. Sol. Given equation is (x2 – 1)y( + 3xy∋ + xy = 0. ...(1) 2 Dividing by (x – 1), (1) gives y( + [(3x)/(x2 – 1)]y∋ + [x/(x2 – 1)]y = 0. ...(2) 2 2 Comparing (2) with y( + P(x)y∋ + Q(x)y = 0, here P(x) = (3x)/(x – 1) and Q(x) = x/(x – 1). Since P(x) and Q(x) are both analytic at x = 2, so x = 2 is an ordinary point of (1). Since the initial values of (1) are prescribed at x = 2 and x = 2 is an ordinary point, hence we shall find the required solution near x = 2, i.e., in powers of (x – 2).

C5 = !

Let

#

n

y = C0 + C1(x – 2) + C2(x – 2)2 + .... = ∃ Cn ( x ! 2) . n

0

...(3)

Created with Print2PDF. To remove this line, buy a license at: http://www.software602.com/

Integration In Series

8.11

We now shift the origin to x = 2 by writing

t=x–2

x=t+2

so that

F I H K

F I H K

y = C0 % C1t % C2 t 2 % ....

∃ Cn t .

...(4)

2

2 d y d y d dy d dy = ...(5) 2 . 2 dx dx dt dt dt dx Using (4) and (5), (1) reduces to [(t + 2)2 – 1](d2y/dt2) + 3(t + 2)(dy/dt) + (t + 2)y = 0 (t2 + 4t + 3)(d2y/dt2) + (3t + 6)(dy/dt) + (t + 2)y = 0. ...(6)

dy dy dt = dx dt dx

Then or

dy dt

and

Also, (3) reduces to

#

n

n

...(7)

0

Differentiating (7) twice in succession w.r.t. ‘t’, we get #

dy/dt = ∃ nCn t

n !1

#

d2y/dt2 = ∃ n(n ! 1)Cn t

and

n 1

n

n!2

. ...(8)

2

Using (7) and (8), (6) reduces to #

#

#

n

(t 2 % 4 t % 3) ∃ n(n ! 1)Cn t n ! 2 % (3t % 6 ) ∃ nCn t n ! 1 % (t % 2) ∃ Cnt = 0 n 0 n

2

n 1

#

#

#

#

#

#

#

n 2

n 2

n 2

n 1

n 1

n 0

n 0

n n !1 n %1 n or ∃n(n ! 1)Cntn % ∃4n(n ! 1)Cnt n !1 % ∃3n(n ! 1)Cntn ! 2 + ∃ 3nCn t % ∃ 6nCnt % ∃ Cnt % ∃ 2Cn t = 0

#

n

#

#

n

∃ n(n ! 1)Cn t % ∃ 4(n % 1)nCn % 1t % ∃ 3(n % 2)(n % 1)Cn % 2 t

or

n 2

n 1

n

#

#

n

n

0

n

#

#

n

% ∃ 3nCn t % ∃ 6(n % 1)Cn % 1t % ∃ Cn ! 1t % ∃ 2Cn t n 1

or

n

0

n 1

n

0

n

=0

(6C2 + 6C1 + 2C0) + (8C2 + 18C3 + 3C1 + 12C2 + C0 + 2C1)t #

+ ∃ [n(n ! 1)Cn % 4n(n % 1)Cn % 1 % 3(n % 1)(n % 2)Cn % 2 % 3nCn + 6(n + 1)Cn + 1 + Cn – 1 + 2Cn]tn = 0 n 2

or

2(3C2 + 3C1 + C0) + (18C3 + 20C2 + 5C1 + C0)t #

+ ∃ [3( n % 1)( n % 2 )Cn % 2 % 2(2n % 3)(n % 1)Cn % 1 + (n2 + 2n + 2)Cn + Cn – 1]tn = 0 n

...(9)

2

From (3), y∋ = C1 + 2C2(x – 2) + 3C3(x – 2)2 + .... ...(10) Putting x = 2 in (3) and (10) and using the given initial conditions, namely, y = 4 and y∋ = 6 when x = 2, we get C0 = 4 and C1 = 6. Hence (9) reduces to #

2(3C2 + 22) + (18C3 + 20C2 + 34)t + ∃ {3(n % 1)(n % 2)Cn % 2 + 2(2n + 3)(n + 1)Cn + 1 + (n2 + 2n + 2)Cn + Cn – 1}tn = 0, n 2

which is an identity in t. Equating to zero the coefficients of various powers of t, we have 2(3C2 + 22) = 0 so that C2 = –(22/3). ...(11) 18C3 + 20C2 + 34 = 0 or 18C3 – 20 × (22/3) + 34 = 0 or C3 = 169/27 ...(12) and 3(n + 1)(n + 2)Cn + 2 + 2(2n + 3)(n + 1)Cn + 1 + (n2 + 2n + 2)Cn + Cn – 1 = 0, for all n 0 2. ...(13) Putting n = 2 in (13), we get 36C4 + 42C3 + 10C2 + C1 = 0 or 36C4 + 42 × (169/27) + 18 × (–22/3) + 6 = 0 2 C4 = 344/81 Putting the above values in (3), the required solution is y = 4 + 6(x – 2) – (22/3) × (x – 2)2 + (169/27) × (x – 2)3 + (344/81) × (x – 2)4 + .... Ex. 8. Solve y( – 2x2y∋ + 4xy = x2 + 2x + 4 in powers of x. Sol. Given equation is y( – 2x2y∋ + 4xy = x2 + 2x + 4. ...(1)

Created with Print2PDF. To remove this line, buy a license at: http://www.software602.com/

8.12

Integration In Series

Clearly x = 0 is an ordinary point of (1). To solve (1) it, let #

n

y = C0 + C1x + C2x2 + C3x3 + .... = ∃ Cn x . n

...(2)

0

Differentiating (2) twice in succession w.r.t. ‘x’, we have #

y∋ = ∃ nCn x n ! 1

#

y( = ∃ n(n ! 1)Cn x

and

n

n 1

n!2

2

.

...(3)

Substituting these values of y, y∋ and y( in (1), we have #

∃ n ( n ! 1) C

n

or

xn! 2 ! 2x2

2

#

∃ n(n ! 1)Cn x

or

n

#

∃ nC n

n!2

n 2

n

1

#

n %1

! ∃ 2nCn x n 1

#

xn !1 % 4x

#

∃C

n

#

% ∃ 4Cn x n

#

n

= x2 + 2x + 4

xn

0

n%1

0

– x2 – 2x – 4 = 0

#

n n n ∃ (n % 2 )(n % 1)Cn % 2 x ! ∃ 2(n ! 1)Cn ! 1 x % ∃ 4 Cn ! 1 x – x2 – 2x – 4 = 0

n

0

n

2

n 1

#

n

or (2C2 – 4) + (6C3 + 4C0 – 2)x + (12C4 + 2C1 – 1)x2 + ∃ [(n % 2)(n % 1)Cn %2 ! 2(n !1)Cn !1 % 4Cn!1}x = 0. ...(4) n 3

Equating to zero the coefficients of various powers of x in (4), we get 2C2 – 4 = 0 so that C2 = 2, ...(5) 6C3 + 4C0 – 2 = 0 so that C3 = (1/3) – (2C0/3) ...(6) 12C4 + 2C1 – 1 = 0 so that C4 = (1/12) – (C1/6) ...(7) and (n + 2)(n + 1)Cn + 2 – 2(n – 1)(Cn – 1 + 4Cn – 1 = 0, for all n 0 3. ...(8) Putting n = 3, 4, 5, ... in (8) and using (5), (6), (7), etc, we get 20C5 – 4C2 + 4C2 = 0 so that C5 = 0, ...(9) 1 2 30C6 = 2C3 so that C6 = 1 1 ! 2 C0 ! C 15 3 3 45 45 0 2 1 1 1 1 ! C ! C 42C7 = 4C4 so that C7 = 21 12 6 1 125 63 1 and so on. Putting these values in (2), the required solution is

FH FH

y = C0 + C1x + 2x2 %

FH

IK IK

FG 1 ! 2C IJ x % FG 1 ! C IJ x + FG 1 ! 2C IJ x % FG 1 ! C IJ x H 3 3 K H 12 6 K H 45 45 K H 126 63 K 3

0

IK

FH

1

0

4

IK

6

1

7

% ....

or y = C0 1 ! 2 x 3 ! 2 x 6 .... % C1 x ! 1 x 4 ! 1 x 7 .... + 2 x 2 % 1 x 3 % 1 x 4 % 1 x 6 % 1 x 7 % .... 3

45

6

63

3

12

45

126

Ex. 9. (i) Explain the method of integrating in series for solving a first order differential equation. #

(ii) Find a power series solution of the form 3 an x n for the differential equation y ∋ n 0

2 xy.

[Nagpur 2005] 2 ∋ (iii) Solve y x ! 4 x % y % 1 satisfying y = 3 when x = 2. Sol. (i) The Picard’s theorem of Art. 1.6 (refer chapter 1) for a differential equation of the form dy/dx = f(x, y) ... (1) gives a sufficient condition for a solution. In the proof using power series, y is found in the form of a Taylor series y = a0 + a1 (x – x0) + a2(x – x0)2 + ... + an (x – x0)n + ... ... (2) where for convenience y0 has been replaced by a0. This series has the following properties:

Created with Print2PDF. To remove this line, buy a license at: http://www.software602.com/

Integration In Series

8.13

(a) It satisfies the differential equation (1) (b) It has the value y0 when x = x0. (c) It is convergent for all values of x sufficiently near x = x0. To find the solution of (1) satisfying the condition y = y0 when x = 0, we assume the solution y

to be of the form

a0 % a1 x % a2 x 2 % ... % an x n % ...

#

3 an x n

... (3)

n 0

in which a0 = y0 and the remaining a’s are constant to be determined. Substitute the assumed series (3) in (1) and proceed to find a1, a2, a3, ... an ... as usual. Remark. If we are to find the solution of (1) satisfying y = y0 when x = x0, we modify the above procedure as follows Make the substitution x – x0 = v, that is, x

v % x0 , dy / dx

dy / d v

dy / d v resulting in

F ( y , v)

... (4)

Use the above procedure to obtain the solution of (4) satisfying y = y0 when v = 0. Finally make the substitution v = x – x0 in the solution. y ∋ 2 xy

(ii) Given

... (1)

Let (1) possess series solution

y

From (2),

y∋

Inserting (2) and (3) into (1), we have

a1 % 2a2 x % 3a3 x2 % 4a4 x3 % ... % nan xn !1 % ...

#

3 an x n

... (2)

n 0 #

3 nan x n !1

... (3)

n 0

4

2 x a0 % a1 x % a2 x 2 % ... % an ! 2 x n !2 % ...

5

Collecting like powers of x yields. a1 % (2a2 ! 2a0 )x % (3a3 ! 2a1)x2 % (4a4 ! 2a2 ) x3 % (5a5 ! 3a3 )x4 % (6a6 ! 5a4 ) x5 % ... % (nan ! 2an ! 2 ) x n !1 % ... 0

In order that this series vanish for all values of x in some region surrounding x = 0, it is necessary and sufficient that coefficients of each power of x vanish. Thus, we obtain a1 = 0, 2a2 – 2a0 = 0, 3a3 – 2a1 = 0, 4a4 – 2a2 = 0, 5a5 – 2a3 = 0, 6a6 – 2a4 = 0 2 a1 = 0, a2 = a0, a3 = (2/3)a1 = 0, a4 = (1/2)a2 = (1/2)a0 , a6 = (1/3)a4 = (1/3) × (a0/2) = a0/3! and so on In general, a2n + 1 = 0 and a2n = a0/n!, for all n = 1, 2, 3 Substituting these values into (2), we obtain the required power series solution. y = a0 + a2x2 + a4x4 + a6x6 + ... + a2nx2x + ... or y = a0 + a0x2 + (a0/2!) x4 + (a0/3!)x6 + ... + (a0/n!)x2n + ... or y = a0 (1 + x2/1! + x4/2! + x6/3! + x2n/n! + ...), a0 being an arbitrary constant (iii) Given dy/dx = x2 – 4x + y + 1 ... (1) where y=3 when x=2 ... (2) Let x = v + 2. Then (1) and (2) reduce to dy/dv = v2 + y – 3 ... (3) and y=3 when v=0 ... (4) We now proceed with (3) and (4) as in part (ii). We assume the series solution

y From (5),

3 % a1v % a2v 2 % a3v3 % a4 v4 % ... % an vn % ...

dy / d v

a1 % 2a2 v % 3a3v 2 % 4a4 v3 % ... % nan v n!1 % ...

... (5) ... (6)

Inserting (5) and (6) into (3), we have, as before

a1 % (2a2 ! a1 )v % (3a3 ! a2 ! 1)v2 % (4a4 ! a3 )v3 % ... % (nan ! an!1 )v n!1 % ... 0

Created with Print2PDF. To remove this line, buy a license at: http://www.software602.com/

8.14

Integration In Series

Equating each of the coefficients to zero, we obtain a1 = 0, 2a2 – a1 = 0 so that a2 = 0; 3a3 – a2 – 1 = 0 so that a3 = 1/3; 4a4 – a3 = 0 so that a4 = 1/12, and so on. Again, nan – an–1 = 0 an = (1/n)an–1 for all n 0 1 2 ... (7) Using the recursion formula (7), we have an

1 an !1 n

1 an ! 2 n(n ! 1)

111

1 a3 n(n ! 1) (n ! 2) 111 4

2 , n!

for all n 0 2

Substituting the above values of the coefficients into (5), we have

y

3 % v3 / 3 % v4 /12 % ... % (2 / n !)v n % ...

... (8)

Replacing v by x – 2, (8) gives the required solution y = 3 + (2/3!) × (x – 2)3 + (2/4!) × (x – 2)4 + ...+ (2/n!) × (x – 2)n + ... Ex. 10. Solve by power series method : y ∋ ! y y∋ ! y

Sol. Given

0.

[Sagar 2004]

0

... (1)

Assume that a solution of (1) is given by power series y

C0 % C1 x % C2 x 2 % ... % Cn x n % ...

y∋

Differentiating (2) w.r.t. ‘x’,

#

3 Cn x n

... (2)

n 0

#

3 n Cn x n !1

n 1

Substituting the above values of y and y∋ in (1), we get

#

#

n 1

n 0

3 n Cn x n !1 ! 3 Cn x n

0

(C1 + 2C2x + 3C3 x2 + ....) – (C0 + C1x + C2x2 + ...) = 0 (C1 – C0) + (2C2 – C1)x + (3C3 – C2) x2 + ... = 0 ... (3) Since (3) is an identity, we must have C1 – C0 = 0, 2C2 – C1 = 0, 3C3 – C2 = 0, .... ... (4) Solving (4), C1 = C0, C2 = C1/2 = C0/2, C3 = C2/3 = C0/3!, ... Substituting these values in (2), we obtain y = C0 (1 + x + x2/2! + x3/3! + ...) or y = C0ex, which is the required solution, C0 being an arbitrary constant. or or

EXERCISE 8 (B) Find the series solution of the following equations : 1. (1 – x2)y( + 2xy∋ – y = 0 about x = 0. 2

[Purvanchal 2007; Meerut 2000] 4

Ans. y C0 (1 % x / 2 ! x / 24 % 111) % C1 ( x ! x3 / 6 ! x5 /120 % 111) 2. (2 + x )y( + xy∋ – (1 + x)y = 0 near x = 0. (Delhi Maths (H) 2002) 2

Ans. y C0 (1 % x2 / 4 % x3 /12 ! 3x 4 / 96 % 111) % C1 ( x % x 4 / 24 % 111) 3. (1 + x2)y( + xy∋ – y = 0 near x = 0. [Delhi Maths (Hons) 1999] 2

4. (x – 1)y( + xy∋ – y = 0 near x = 0.

Ans. y C0 (1 % x2 /2 ! x 4 /8 % x6 /15 % 111) % C1 x. Ans. y = C0{1 + (x2/2) + (x4/4) + ....} + C1x

5. (x2 – 1)y( + 4xy∋ + 2y = 0 near x = 0. Ans. y = C0(1 + x2 + x4 + ....) + C1(x + x3 + x5 + ...) 6. (1 – x2)y( + 2xy∋ – y = 0 about x = 0. Ans. y = C0(1 + x2/2! – x4/4! + ...) + C1 (1 – x3/3! – x5/5! +...)

Created with Print2PDF. To remove this line, buy a license at: http://www.software602.com/

Integration In Series

8.15

7. y( – xy∋ + 2y = 0 near x = 1. Ans. y = C0[1 – (x – 1)2 – (1/3) × (x – 1)3 – ....] + C1[(x – 1) + (1/2) × (x – 1)2 + ....] 2 8. (x – 1)y( + 3xy∋ + xy = 0, y(0) = 2, y∋(0) = 3. Ans. y = 2 + 3x + (11/6) × x3 + (1/4) × x4 – .... 9. (i) (1 – x2)y( + 2y = 0 near x = 0. Ans. y = C0(1 – x2) + C1(x – x3/3 – x5/5 – x7/35 – .... (ii) (1 – x2)y( + 2y = 0, y(0) = 4, y∋(0) = 5. Ans. y = 4 + 5x – 4x2 – (5/3) × x3 – (1/3) × x5 + .... 10. (x2 + 2x)y( + (x + 1)y∋ – y = 0 near x = –1. Ans. y = C0[1 – (1/2) × (x + 1)2 – (1/8) × (x + 1)4 – (1/16) × (x + 1)6 + ....] + C1(x + 1) 11. y( – xy∋ = e–x, y(0) = 2, y∋(0) = –3. [Hint : Use the expansion e–x = 1 – (x/1!) + (x2/2!) – (x3/3!) + .... Ans. y = 2 – 3x + (1/2) × x2 – (2/3) × x3 + (1/8) × x4 – .... 2 Ex. 12(a). Solve y( + x y = 2 + x + x2 about x = 0. Sol. Proceed as in Ex. 8. of Art. 8.6. Its general solution is given by

FG H

IJ K

FG H

IJ K

3 4 6 7 4 8 5 9 2 x x x x % ! ! ! .... y = C0 1 ! x % x ! .... % C1 x ! x % x ! .... + x % 6 12 30 252 12 672 20 1440

Ex. 12(b). Apply power method to solve y( – y = x. Sol. Proceed as in Ex. 8. of Art 8.6. Its general solution is given by

FG H

IJ K

FG H

IJ FG K H

IJ K

2 4 3 5 3 5 y = C0 1 % x % x % .... % C1 x % x % x % .... % x % x % .... 2! 4! 3! 5! 3! 5!

13. y ∋∋ ! 4 y 0 near x = 0 [Agra 2007] 8.7. Series solution about regular singular point x = 0. Frobenius Method. If x = 0 is regular point, we shall use the Frobenius method for finding series solution about x = 0. However if we wish to find the series solution about regular singular point x = a, then we first shift the origin to point x = a and later on proceed as before. If x = 0 is an irregular point of the given equation, then discussion of solution of the equation is beyond the scope of this book. We now discuss Frobenius method. Consider the differential equation of the form dy d2y , ...(1) y2 dx dx 2 and the functions F(x) and G(x) are analytic at x = 0. Then the following method for solving (1) is called Frobenius method. We assume a trial solution

y2 %

F( x ) G( x ) y % 2 y = 0, x 1 x

where

#

r

y = x ∃ cm x m

0

m

r

y1

2

x (c0 % c1 x % c2 x % ...), where c0 6 0.

...(2)

Differentiating (2) term by term, we have y1 = rc0xr – 1 + (r + 1)c1xr + .... = xr – 1[rc0 + (r + 1)c1x + ....] y2 = r(r – 1)c0xr – 2 + r(r + 1)c1xr – 1 + ... = xr – 2[r(r – 1)c0 + (r + 1)rc1x + ...] Since F(x) and G(x) are analytic at x = 0, we can write F(x) = a0 + a1x + a2x2 + .... and G(x) = b0 + b1x + b2x2 + .... Putting the values of y, y1, y2, F(x) and G(x) in (1) and then multiplying both sides by x2, gives xr[r(r – 1)c0 + ....] + (a0 + a1x + ....)xr(rc0 + ....) + (b0 + b1x + ....)xr (c0 + c1x + c2x2 + ....) = 0. ...(3) Since (3) is an identity, we can equate to zero the coefficients of various powers of x. This will give us a system of equations involving the unknown coefficients cm. The smallest power is xr, and the corresponding equation is [r(r – 1) + a0r + b0]c0 = 0. Since by assumption c0 6 0, we obtain r2 + (a0 – 1)r + b0 = 0. ...(4) This important quadratic equation is known as the indicial equation of (1). We shall see that

Created with Print2PDF. To remove this line, buy a license at: http://www.software602.com/

8.16

Integration In Series

this method will give rise to a fundamental system of solutions ; one of these solutions will always be of the form (2), but for the form of the other solutions there will be three different possibilities corresponding to the following cases : Case I. The roots of the indicial equation are distinct and do not differ an integer. Let r1 and r2 be the roots of (4). We substitute r = r1 in the above mentioned system of r

u(x) = x 1 (c0 % c1 x % .... ) .

equations and obtain a solution

r

Proceeding similarly by using r = r2, we obtain a second solution v(x) = x 2 (c0∋ % c1∋ x % ....) where c∋0, c1∋, .... are the new values of the coefficients corresponding to r = r2. Since r1 – r2 is not an integer, u/v is never constant. Thus u and v are two independent solutions of (1). Hence the general solution will be y = au + bv, where a and b are arbitrary constants. Case II. The indicial equation has equal roots. The indicial equation (4) has double root r if (a0 – 1)2 – 4b0 = 0 giving r = (1 – a0)/2. ...(5) We first obtain the coefficients c1, c2.... successively from the system of equations connecting them. Thus we obtain a first solution u(x) = xr(c0 + c1x + ....), ...(6) wherein we write r = (1 – a0)/2 afterwards. To determine another solution we use the method of variation of parameters, that is, we replace the constant c in the solution cu by a function w(x) to be determined such that v(x) =u(x) w(x) ...(7) is solution of (1). Differentiating (7), we get v1 = u1w + uw1 and v2 = u2w + 2u1w1 + uw2. ...(8) 2 Since v(x) is a solution of (1) by assumption, x v2 + F(x) x v1 + G(x) v = 0. ...(9) Putting the values of v, v1 and v2 from (7) and (8) in (9), we get x2(u2w + 2u1w1 + uw2) + xF(x)[u1w + uw1] + G(x)uw = 0 or [x2u2 + F(x)xu1 + G(x)u]w + x2(2u1w1 + uw2) + xF(x)uw1 = 0 Since u(x) is a solution of (1), we have x2u2 + F(x)xu1 + G(x)u = 0. 2 7 The above equation reduces to x (2u1w1 + uw2) + xF(x)uw1 = 0.

FG H

Dividing by x2u and putting the value of F(x) gives, w2 % 2

IJ K

u1 a0 % % .... w1 = 0. ... (10) u x

In what follows we write dots to represent terms which are constant or involve positive powers of x. Now from (6), we obtain u1 x = u

r !1

[ rc0 % ( r % 1)c1 x % ....] r

x [ c0 % c1 x % ....] 2 r % a0 7 (10) becomes w2 % % .... w1 = 0. x But from (5), 2r + a0 = 1. Hence the above equation reduces to 1 w2 % % .... w1 = 0 or x

FH

FG H

IJ K

r % .... x

...(11)

IK

w2 = ! 1 % .... w1 x 1 (...) Integrating, log w1 = –log x + .... or w1 = e x Expanding the exponential function in powers of x and integrating once more, we obtain the expression of w always in the following form w = log x + k1x + k2x2 + .... Putting this value of w in (7), we obtain the desired another independent solution v(x). Then the general solution is y = au + bv, a and b being arbitrary constants.

Created with Print2PDF. To remove this line, buy a license at: http://www.software602.com/

Integration In Series

8.17

Case III. The roots of the indicial equation differ by an integer. Let the roots r1 and r2 of the indicial equation differ by an integer, say r1 = r and r2 = r – p, where p is a positive integer, then as before one solution will be given by r

2 u(x) = x 1 (c0 % c1 x % c2 x % ....)

corresponding to the root r1. However, while, dealing with the root r2 it may not be possible to determine another independent solution v(x) as in case I. In such cases we determine v(x) by using method outlined in case II. Thus as before we first obtain (11). From (5), r1 + r2 = –(a0 – 1), using theory of equations Since r1 = r, r2 = r – p, this gives 2r + a0 = p + 1. Thus (11) becomes

F H

I K

w2 p%1 = ! % .... . w1 x

Integrating, log w1 = –(p + 1) log x + .... or Expanding the exponential function and simplifying, we get w1 =

1

x

p %1

%

k1 x

p

% .... %

kp x

w1 = x–(p + 1)e(...)

% k p % 1 % k p % 2 x % ....

w = ! 1 p ! .... % k p log x % k p % 1 x % .... px Putting this value of w in (7), we obtain another solution v(x) and as usual the general solution y = au + bv, where a and b are arbitrary constants. 8.8. Working rule for solution by Frobenius method Consider linear differential equation of order two f(x)y( + g(x)y∋ + r(x)y = 0. ...(1) Step 1. Suppose that a trial solution of (1) be of the form Integrating again,

y = xk(c0 + c1x + c2x2 + .... + cmxm + .... )

i.e.

#

y = ∃ cm x m % k ,

Thus we take

m

where

0

#

k m y = x ∃ cm x . m

0

c0 6 0.

...(2) ...(3)

Step 2. Differentiate (3) and obtain #

y∋ = ∃ cm (m % k ) x m 0

m % k !1

and

#

y( = ∃ cm (m % k )(m % k ! 1)x m % k ! 2 . m

...(4)

0

Using (3) and (4), (1) reduces to an identity. Step 3. Equating to zero the coefficient of the smallest power of x in the identity obtained in step 2 above, we obtain a quadratic equation is k. The quadratic equation so obtained is called the indicial equation. Step 4. Solve the indicial equation. The following cases arise : (i) The roots of indicial equation unequal and not differing by an integer. (ii) The roots of indicial equation unequal, differing by an integer and making a coefficient of y indeterminate. (iii) The roots of indicial equation unequal, differing by an integer and making a coefficient of y infinite. (iv) The roots of indicial equation equal. Step 5. We equate to zero the coefficient of general power (e.g. xk + m, xk + m – 1 etc. whichever may be the lowest) in the identity obtained in step 2. The equation so obtained will be called the recurrence relation, because it connects together the coefficients cm, cm – 2 or cm, cm – 1 etc.

Created with Print2PDF. To remove this line, buy a license at: http://www.software602.com/

8.18

Integration In Series

Step 6. If the recurrence relation connects cm and cm – 2, then we, in general, determine c1 by equating to zero the coefficient of the next higher power (than already used for getting the indicial equation). On the other hand, if the recurrence relation connects cm and cm – 1, this step may be omitted. Step 7. After getting various coefficinets with help of steps 5 and 6 above, solution is obtained by substituting these in (2) or (3) above. Other necessary working will be shown in details and necessary modifications in method will be discussed as we proceed with different four cases outlined in step 4. The readers are advised to study carefully the first example of each case. In each problem the two series of solution should be linearly independent. 8.9. Examples of Type–1 on Frobenius method. Roots of indicial equation unequal and not differing by an integer : In this connection the following rule should be noted carefully. Rule. Let k1 and k2 be the roots of the indicial equation. It k1 and k2 do not differ by an integer. Then, in general, two independent solutions u and v are obtained by putting k = k1 and k2 in the series for y. Then the general solution is y = au + bv, where a and b are arbitrary constants. Ex. 1. Solve in series : 9x(1 – x)y( – 12y∋ + 4y = 0. [Delhi Maths (H) 2007, 08; Meerut 1997] Sol. Given equation is 9x(1 – x)y( – 12y∋ + 4y = 0. ...(1) Dividing by 9x(1 – x), (1) can be put in standard form 2

d y dy 4 4 ! % y = 0. 2 3 x (1 ! x ) dx 9 x (1 ! x ) dx Comparing it with y( + P(x)y∋ + Q(x)y = 0, we have P(x) = –4/[3x(1 – x)] and Q(x) = 4/[9x(1 – x)]. Since P(x) and Q(x) are not both analytic at x = 0, so x = 0 is not ordinary point of (1). Again xP(x) = –4/[3(1 – x)] and x2Q(x) = 4x/(1 – x), showing that both P(x) and Q(x) are analytic at x = 0. So x = 0 is a regular singular point of (1). To find solution of (1), we take #

y = ∃ cm x m

#

y∋ = ∃ cm (k % m) x

7

k % m!1

k%m

0

...(2)

#

y( = ∃ cm (k % m)(k % m ! 1)x k % m ! 2 .

and

m 0

, where c0 6 0.

m

...(3)

0

Substituting the series (2) and (3) in (1), we have #

#

#

9 x (1 ! x ) ∃ c m (k % m)(k % m ! 1) x k % m ! 2 ! 12 ∃ c m (k % m) x k % m ! 1 + 4 ∃ c m x k % m = 0 m

0

m

#

#

m 0

m 0

0

m

#

k % m !1

2 k % m! 2 ! 12 ∃c (k % m)x m or 9x ∃cm(k % m)(k % m !1)xk % m!2 ! 9x ∃cm(k %m)(k % m!1)x m 0

#

or

∃cm{9(k % m)(k % m ! 1) !12(k % m)}x

k % m !1

m 0

0

#

% 4 ∃cmxk % m = 0 m 0

#

k%m + ∃cm{4 ! 9(k % m)(k % m ! 1)}x = 0. m 0

But 9(k + m)(k + m – 1) – 12(k + m) = 3(k + m)(3k + 3m – 7) and 4 – 9(k + m)(k + m – 1) = 4 – 9(k + m)2 + 9(k + m) = –[9(k + m)2 – 9(k + m) – 4] = –[9(k + m)2 – 12(k + m) + 3(k + m) – 4] = –[3(k + m){3(k + m) – 4} + 3(k + m) – 4] = –{3(k + m) – 4}{3(k + m) + 1} Thus, 4 – 9(k + m) (k + m – 1) = – (3k + 3m – 4)(3k + 3m + 1). Using (5) and (6), (4) can be re–written as #

#

3 ∃ c m (k % m)(3k % 3m ! 7) x k % m ! 1 – ∃ c m (3k % m ! 4)(3k % 3m % 1) x m

0

m

k%m

= 0,

...(4) ...(5)

...(6)

...(7)

0

Created with Print2PDF. To remove this line, buy a license at: http://www.software602.com/

Integration In Series

8.19 k–1

which is an identity in x. Equating to zero the coefficient of the smallest power of x, namely x , (7) gives the indicial equation 3c0k(3k – 7) = 0 or k(3k – 7) = 0 [ c0 6 0] Thus k=0 and 7/3, ...(8) which are unequal and not differing by an integer. To obtain the recurrence relation, we equate to zero the coefficient of xk + m – 1 and obtain 3cm(k + m)(3k + 3m – 7) – cm – 1[3k + 3(m – 1) – 4] × [3k + 3(m – 1) + 1] = 0 3k % 3m ! 2 c . 3( k % m ) m ! 1

cm =

or Taking m = 1 in (9) gives

c1 =

Next, taking m = 2 in (9) gives

c2 =

...(9)

c 0 3k % 1 . / 3 k %1

...(10)

c 3k % 4 (3k % 1)(3k % 4 ) c1 = 02 / , by (10) 3( k % 2 ) ( k % 1)( k % 2 ) 3

...(11) and so on. Putting these values in (2) i.e.,

y = xk(c0 + c1x + c2x2 + ....), gives

LM N

OP Q

3k % 1 (3k % 1)(3k % 4 ) 2 y = c0 x k 1 % 1 x % 12 x % .... 3 k %1 3 (k % 1)(k % 2 )

...(12)

11 4 2 ) 1 ∗ y = a +1 % x % x % .... , 3 31 6 − .

Putting k = 0 and replacing c0 by a in (12),

au, say

7 8 8 111 2 ∗ and replacing C0 by b in (12), y = bx7/3 )+1 % x % x % .... , bv , say.. 2 10 113 − 10 . The required solution is given by y = au + bv, i.e.

Next, putting k =

1 11 4 2 8 8 1 11 2 ) ∗ ) ∗ x % .... , % bx 7 / 3 +1 % x % x % .... , . y = a +1 % x % 3 31 6 10 113 − . − 10 .

Ex. 2. Solve the Bessel equation x2y( + xy∋ + (x2 – n2)y = 0 in series, taking 2n as non– integral. [Delhi 1997; G.N.D.U. Amritsar 2010; Ranchi 2010] Sol. Given x2y( + xy∋ + (x2 – n2)y = 0. ...(1) 2 2 2 2 Dividing by x , y( + (1/x)y∋ + {(x – n )/x }y = 0. Comparing it with y( + P(x)y∋ + Q(x)y = 0, here P(x) = 1/x and Q(x) = (x2 – n2)/x2 so that xP(x) = 1 and x2Q(x) = x2 – n2. Thus both P(x) and Q(x) are analytic at x = 0 and so x = 0 is a regular singular point of (1). Let the series solution of (1) be of the form #

y = ∃ Cm x m

0

#

k % m!1 y∋ = ∃ Cm (k % m) x

7

m

k%m

...(2)

#

y( = ∃ Cm (k % m)(k % m ! 1) x k % m ! 2 .

and

0

, where C0 6 0.

m

...(3)

0

Substitution for y, y∋, y( in (1), we have x

2

#

∃ Cm (k % m)(k % m ! 1) x

k %m!2

m 0

or

#

∃ Cm (k % m)(k % m ! 1) x

m

0

k%m

#

% x ∃ Cm ( k % m ) x

k % m !1

m 0

#

% ∃ Cm ( k % m ) x m

0

k%m

#

2 2 k%m + ( x ! n ) ∃ Cm x =0 m 0

#

#

+ ∃ Cm x k % m % 2 ! n 2 ∃ Cm x k % m = 0 m

0

m 0

Created with Print2PDF. To remove this line, buy a license at: http://www.software602.com/

8.20

Integration In Series #

2

∃ Cm {( k % m)(k % m ! 1) % ( k % m ) ! n }x

or

m

0

#

2

2

∃ Cm {( k % m ) ! n }x

or

m

0

#

k%m

m

0

#

% ∃ Cm x

∃ Cm (k % m % n)(k % m ! n) x

or

k%m

k %m%2

#

% ∃ Cm x m

k% m%2

=0

0

=0

m 0

k%m

#

% ∃ Cm x

k %m%2

= 0,

...(4)

m 0

which is an identity. Equating to zero the coefficient of the smallest power of x, namely xk, (4) gives the indicial equation C0(k + n)(k– n) = 0 or (k + n)(k + n) = 0 [ C0 6 0] so that k=n and –n. ...(5) Since 2n is non–integral (given), the roots given by (5) are unequal and not differing by an integer. To obtain the recurrence relation, we equate to zero the coefficient of xk + m and obtain Cm(k + m + n)(k + m – n) + Cm – 2 = 0

1 ...(6) C ( k % m % n)(k % m ! n) m ! 2 [Since (6) gives relationship between Cm and Cm – 2, we proceed to find C1 as explained in step 6 in Art. 8.8]. Equating to zero the coefficient of xk + 1 in (4) gives C1(k + 1 + n)(k + 1 – n) = 0 giving C1 = 0 for both k = n and k = –n. Then from (6) and C1 = 0, we have C1 = C3 = C5 = .... = 0. ...(7) 1 C . Further, taking n = 2 in (6) gives C2 = ! ...(8) (k % 2 % n)(k % 2 ! n) 0 Next, taking n = 4 in (6) and using (8) gives 1 1 C C4 = ! ...(9) C = (k % 4 % n)( k % 4 ! n) 2 ( k % 2 % n)(k % 2 ! n)(k % 4 % n)( k % 4 ! n) 0 and so on. Putting these values in (2), i.e., y = xk(C0 + C1x + C2x2 + C3x3 + C4x4 + ....), gives Cm = –

giving

LM N

k y = c0 x 1 !

OP PQ

x4 x2 ! .... . + ( k % 2 % n )( k % 2 ! n )( k % 4 % n )( k % 4 ! n ) (k % 2 % n)(k % 2 ! n)

...(10)

Putting k = n and replacing C0 by a in (10), gives 8 9 x2 x4 y = ax n :1 ! % ! ....; < 4(n % 1) 4 1 8(n % 1)(n % 2) = Next, putting k = –n and replacing C0 by b in (10) gives

au , say..

2 8 9 x4 y = bx ! n :1 ! x % ! ....; bv , say.. < 4(1 ! n) 4 1 8(1 ! n )(2 ! n ) = Required general series solution is given by y = au + bv, where a and b are arbitrary constants. Ex. 3. Solve the following differential equations in series : (a) 2x2y( – xy∋ + (1 – x2)y = 0 [Garhwal 2010] 2 2 2 (b) 2x y( – xy∋ + (1 – x )y = x . [Meerut 1996, 98] Sol. (a) Given 2x2y( – xy∋ + (1 – x2)y = 0 ...(1) Dividing by x2, (1) takes standard form y( – (1/2x)y∋ + {(1 – x2)/2x2}y = 0. Comparing it with y( + P(x)y∋ + Q(x)y = 0, here P(x) = –(1/2x) and Q(x) = {(1 – x2)/2x2} so that xP(x) = –(1/2) and x2Q(x) = (1 – x2)/2. Thus both P(x) and Q(x) are analytic at x = 0 and so x = 0 is a

Created with Print2PDF. To remove this line, buy a license at: http://www.software602.com/

Integration In Series

8.21

regular singular point. Let the series solution of (1) be of the form #

y = ∃ Cm x m

#

k % m!1 y∋ = ∃ Cm (k % m) x and

7

m

k%m

0

0

, where C0 6 0.

...(2)

#

y( = ∃ Cm ( k % m )( k % m ! 1) x k % m ! 2 . m

...(3)

0

Substitution for y, y∋, y( in (1), we have #

#

#

2 x 2 ∃ Cm (k % m )( k % m ! 1) x k % m ! 2 ! x ∃ Cm (k % m ) x k % m ! 1 + (1 ! x ) ∃ Cm x m

or

0

m

#

2

m 0

0

#

#

k%m

=0

#

2 ∃ Cm (k % m)(k % m ! 1) x k % m ! ∃ Cm ( k % m) x k % m + ∃ Cm x k % m ! ∃ Cm x k % m % 2 = 0 m

0

m

0

m

#

∃ Cm {2(k % m)(k % m ! 1) ! (k % m) % 1}x

or

m

k%m

0

#

0

m

#

! ∃ Cm x m

0

k%m%2

=0

0

#

2 k%m ! ∃ Cm x k % m % 2 = 0 ∃ Cm {2(k % m) ! 3( k % m) % 1}x

or

m

0

m 0

#

∃ Cm (2k % 2 m ! 1)( k % m ! 1) x

or

m

k%m

0

#

! ∃ Cm x m

k %m%2

0

= 0,

...(4)

[ 2(k + m)2 – 3(k + m) + 1 = 2(k + m)2 – 2(k + m) – (k + m) + 1= 2(k + m)[(k + m) – 1] – [(k + m) – 1] = (k + m – 1)[(2(k + m) – 1] = (k + m – 1)[(2k + 2m – 1)] (4) is an identity. Equating to zero the coefficient of the smallest power of x namely xk, (4) gives the indicial equation C0(2k – 1)(k – 1) = 0 or (2k – 1)(k – 1) = 0 [ C0 6 0] so that k = 1, and k = 1/2, ...(5) which are unequal and not differing by an integer. To obtain the recurrence relation, we equate to zero the coefficient of xk + m and obtain Cm (2k + 2m – 1)(k + m – 1) – Cm – 2 = 0

1 C . ...(6) (2 k % 2 m ! 1)(k % m ! 1) m ! 2 To obtain C1, we now equate to zero the coefficient of xk + 1 and get C1(2k + 1)k = 0 so that C1 = 0 for both roots k = 1, and k = 1/2 of the indicial equation. Then from (6) and C1 = 0, we have C1 = C3 = C5 = .... = 0. ...(7) 1 C . Further, taking n = 2 in (6) gives C2 = ...(8) (2 k % 3)(k % 1) 0 Next, taking n = 4 in (6) and using (8) gives 1 1 C4 = C C (2 k % 5)(k % 3) 2 (k % 1)(k % 3)(2 k % 3)(2k % 5) 0 and so on. Putting these values in (2), i.e., y = xk(C0 + C1x + C1x2 + C3x3 + ....), gives giving

Cm =

LM N

x = C0 x k 1 %

OP Q

x2 x4 % % .... (k % 1)(2k % 3) (k % 1)(k % 3)(2 k % 3)(2 k % 5)

Putting k = 1 and replacing C0 by a in (10) gives 8 9 x2 x4 y = ax :1 % % % ....; < 2 15 2 14 15 19 = Next, putting k = 1/2 and replacing C0 by b in (10) gives

...(10)

au , say

Created with Print2PDF. To remove this line, buy a license at: http://www.software602.com/

8.22

Integration In Series

8 9 x2 x4 y = bx1/ 2 :1 % % % ....; < 2 13 2 131 41 7 =

bv , say

The required series solution is given by y = au + bv where a and b are arbitrary constants. Part (b) Given 2x2y( – xy∋ + (1 – x2)y = x2. ...(11) Since R.H.S. is involved in (11), the general solution of (11) is made up of complementary function (C.F.) and particular integral (P.I.) as usual. To find C.F. of (11), we solve 2x2y( – xy∋ + (1 – x2)y = 0, which is the same as equation (1) of part (a). So as above, C.F. is au + bv. Next assume that the particular solution of (11) is of the form [similar to (2)]. y= x

k

#

m

∃ Am x ,

A0 6 0,

where

m 0

...(12)

Find y∋ and y( and then substitute values of y, y∋ and y( in (11). Since L.H.S. of (11) and (1) is the same, proceeding as in part (a) we shall get [compare with (4) of part (a)] #

∃ Am (2 k % 2m ! 1)(k % m ! 1) x

m

k%m

0

#

! ∃ Am x

k%m%2

m 0

= x2,

...(13)

which is an identity. Hence the leading term (the term containing the smallest power of x) of L.H.S. of (13) must be x2 and the coefficients of each of the remaining terms of L.H.S. of (13) must vanish. These conditions are satisfied by taking k = 2, A0(k – 1)(2k – 1) = 1 and A1 = 0 ...(14) Am =

and

Am ! 2 (2k % 2 m ! 1)(k % m ! 1)

.

...(15)

Note that (15) is similar to the recurrence relation (6) of part (a). Since k = 2, (14) gives A0 = 1/3. Since A1 = 0, so (15) gives A1 = A3 = A5 = .... = 0. ...(16) Putting m = 2 and k = 2 in (15), we obtain

A2 =

A0 7 13

1 1 , as A0 = . ...(17) 3 11 7 1 3 1 3 11

Putting m = 4 and k = 2 in (15) and using (17), we obtain A4 =

A2 11 1 5

1 , 11 1 5 1 7 1 3 1 3 11

...(18)

Putting these values in (12), i.e., y = xk(A0 + A1x + A2x2 + A3x3 + A4x4 + ....), we obtain P.I. =

x2 x4 x6 % % % .... 3 11 7 1 3 1 3 11 11 1 5 1 7 1 3 1 3 1 1

f ( x) , say..

Hence the required solution is given by y = au + bv + f(x), a and b being arbitrary constants. Ex. 4. Solve xy( + (x + n)y∋ + (n + 1)y = 0, where n is not an integer. [Meerut 1993] Sol. Given xy( + (x + n)y∋ + (n + 1)y = 0. ...(1) As usual verify that x = 0 is a regular singular point of (1). To solve (1), take #

y = xk(C0 + C1x + C2x2 + ....) = ∃ Cm x k % m , where m

0

#

7

y∋ =

∃ (k % m)Cm xk % m ! 1

m 0

and

C0 6 0.

...(2)

#

y( = ∃ (k % m)(k % m ! 1)Cm x k % m ! 2 . ...(3) m

0

Created with Print2PDF. To remove this line, buy a license at: http://www.software602.com/

Integration In Series

8.23

Putting the above values of y, y∋ and y( in (1), we have #

x ∃ ( k % m)(k % m ! 1)Cm x m

or

k%m!2

0

#

k % m!1

∃(k % m)(k % m !1)Cmx

m 0

#



or

m

#

% ( x % n ) ∃ ( k % m )Cm x

#

k % m !1

% ( n % 1)

m 0

#

k %m

% ∃(k % m)Cmx m 0

#

+ ∃ n(k % m)Cm x

0

m

k % m !1

m 0

( k % m )( k % m ! 1 % n )C m x k % m ! 1 %

m

xk

% m

=0

0

#

% ∃ (n % 1)Cm x k % m = 0 m 0

#

∃ ( k % m % n % 1)C

m

∃C

mx

k%m

= 0.

...(4)

0

Equating to zero the coefficient of the smallest power of x, namely xk – 1, the above identity (4) gives the indicial equation C0k(k – 1 + n) = 0 so that k=0 and 1 – n, as C0 6 0. Given that n is not an integer. So the roots 0 and 1 – n of the indicial are unequal and do not differ by an integer. Next, we equate to zero the coefficient of xk + m – 1 in the above identity (4) and obtain the recurrence relation (k + m)(k + m + n – 1)Cm + (k + m + n)Cm – 1 = 0 Cm = !

so that

k%m%n c . ( k % m )( k % m % n ! 1) m ! 1

Putting m = 1, 2, 3, .... in (5), we have C2 = !

k %n%2 C1 (k % 2)(k % n % 1)

C3 = !

k %n%3 C2 (k % 3)(k % n % 2)

!

k%n%2 C , ( k % 1)( k % 2 )( k % n ) 0

...(7)

k %n%3 C (k % 1)(k % 2)(k % 3)(k % n) 0

...(8)

and so on. Putting these values of C1, C2, C3, .... in (2), we have

LM N

y = C0 x k 1 !

...(6)

(k % n % 3) (k % n % 2) / C0 , using (7) (k % 3)(k % n % 2) (k % 1)(k % 2)(k % n)

C3 = !

or

k % n %1 C , (k % 1)(k % n) 0

k %n%2 k % n %1 / C0 , by (6) (k % 2)(k % n % 1) (k % 1)(k % n)

C2 =

or

C1 = !

...(5)

OP Q

k%n%3 k % n %1 k%n%2 x2 ! x 3 % .... ...(9) x% (k % 1)(k % 2)(k % 3)(k % n) (k % 1)( k % n) (k % 1)( k % 2)(k % n)

Putting k = 0 and replacing C0 by a in (9), we get

LM N

y = a 1!

OP Q

n %1 n % 2 x 2 n % 3 x3 x% ! % .... n n 2! n 3!

au , say

Putting k = 1 – n and replacing C0 by b in (9), we get

LM N

y = bx1 ! n 1 !

9 4 2 2 x% 3 x3 ! ....; x ! 2!n (2 ! n)(3 ! n) (2 ! n)(3 ! n )(4 ! n) =

bv , say

The required solution is y = au + bv, where a and b are arbitrary constants.

Created with Print2PDF. To remove this line, buy a license at: http://www.software602.com/

8.24

Integration In Series 2

Ex. 5. Verify that the origin is a regular singular point of 2x y( + xy∋ – (x + 1)y = 0 and find two independent Frobenius series solutions of it. [Lucknow 1995] 2 Sol. Given 2x y( + xy∋ – (x + 1)y = 0. ...(1) Dividing by 2x2, (1) takes the standard form y( + (1/2x)y∋ – {(x + 1)/2x2}y = 0. Comparing it with y( + P(x)y∋ + Q(x)y = 0, here P(x) = 1/2x and Q(x) = –(x + 1)/2x2 so that xP(x) = 1/2 and Q(x) = –(x + 1)/2. Since xP(x) and x2Q(x) are both analytic, so x = 0 is a regular singular point of x. The solve (1), we take #

y = xk(C0 + C1x + C2x2 + ....) = ∃ Cm x m

#

y∋ = ∃ ( k % m)Cm x k % m ! 1

7

m

k%m

0

, where C0 6 0.

...(2)

#

y( = ∃ ( k % m )( k % m ! 1)Cm x k % m ! 2 . ...(3)

and

0

m

0

Putting the above values of y, y∋ and y( in (1), we get 2x

2

#

∃ (k % m)(k % m ! 1)Cm x

m

or or

0

#

∃ 2(k % m)(k % m ! 1)Cm x

m

0

or

k%m

#

% x ∃ ( k % m )Cm x m

%

#

∃ ( k % m)Cm x

m

k%m

2

m

0

k%m

k%m

#

! ∃ Cm x m

#

0

#

0

m 0

#

! ∃ Cm x m

k % m %1

0

k % m %1

m

=0

=0

#

0

=0

#

k%m ! ∃ Cm x k % m % 1 = 0. ∃ {2(k % m) % 1}(k % m ! 1)Cm x

m

k%m

! ∃ Cm x k % m % 1 ! ∃ Cm x k % m = 0

0

∃ {2(k % m) ! (k % m) ! 1}Cm x

#

! ( x % 1) ∃ Cm x m 0

m

0

#

m

k % m !1

0

∃ {2(k % m)(k % m ! 1) % (k % m) ! 1}Cm x #

or

k%m!2

0

...(4)

Equating to zero the coefficient of the smallest power of x, namely, xk, the above identity (4) in x gives the indicial equation, namely, C0(2k + 1)(k – 1) = 0 so that k=1 and –1/2, as C0 6 0. Here the difference of these roots = 1 – (– 1/2) = 3/2 6 not an integer. Next, we equate to zero the coefficient of xk + m in (4) and obtain the recurrence relation {2(k + m) + 1}(k + m – 1)Cm – Cm – 1 = 0 1 so that Cm = C . ...(5) (2k % 2m % 1)(k % m ! 1) m ! 1 Putting m = 1, 2, 3, .... in (5), we have C1 = {1/k (2k + 3)}C0, ...(6) 1 1 C2 = C C , by (6). ...(7) (2 k % 5)(k % 1) 1 (2k % 3)(2k % 5)k ( k % 1) 0 and so on. Putting these values in (2), we get 8 9 x x2 % % ....; . y = C0 x k :1 % < (2k % 3)k (2k % 3)(2k % 5)k (k % 1) =

...(8)

Putting k = 1 and replacing C0 by a in (8), we get y = ax[1 + x/5 + x2/70 + ....] = au, say Next, putting x = –1/2 and replacing C0 by b in (8), we get y = bx–1/2[1 – x – (x2/2) + ....] = bv, say. The required solution is y = au + bv, a and b being arbitrary constants.

Created with Print2PDF. To remove this line, buy a license at: http://www.software602.com/

Integration In Series

8.25 3

Ex. 6. Show that x = 0 is a regular singular point of (2x + x )y( – y∋ – 6xy = 0 and find its solution about x = 0. [Delhi Maths (H) 1995, 96; Meerut 1996] 3 Sol. Given (2x + x )y( – y∋ – 6xy = 0. ...(1) Dividing (2x + x3), (1) can be put in standard form y( – [1/(2x + x3)}y∋ – {6/(2 + x2)}y = 0. Comparing it with y( + P(x)y∋ + Q(x)y = 0, here P(x) = –1/(2x + x3) and Q(x) = –6/(2 + x2) so that xP(x) = –1/(2 + x2) and x2Q(x) = –(6x2)/(2 + x2). Since P(x) and Q(x) are both analytic at x = 0, so x = 0 is a regular singular point of (1). Let series solution of (1) be #

k%m

y = xk(C0 + C1x + C2x2 + ....) = ∃ Cm x m

#

7

y∋ =

∃ (k % m)C

mx

k % m !1

0

C0 6 0.

, where

...(2)

#

y( = ∃ ( k % m )( k % m ! 1)Cm x k % m ! 2 .

and

m

m 0

0

...(3) Putting the above values of y, y∋ and y( in (1), we get #

#

#

(2 x % x 3 ) ∃ (k % m)(k % m ! 1)Cm x k % m ! 2 ! ∃ (k % m)Cm x k % m ! 1 ! 6 x ∃ Cm x k % m = 0 m

or or

m

0

m

#

#

#

m 0

m 0

m 0

k % m !1 % ∃(k % m)(k % m ! 1)Cmxk % m %1 ! ∃(k % m)Cmx ∃2(k % m)(k % m ! 1)Cmx

#

∃ {2(k % m)(k % m ! 1) ! (k % m)}Cm x

m

k % m !1

k % m !1

0

#

! ∃6Cmxk % m %1 = 0 m 0

#

% ∃ {(k % m)( k % m ! 1) ! 6}Cm x k % m % 1 = 0 m

0

0

#

#

2 k % m !1 % ∃ {( k % m ) ! (k % m) ! 6}Cm x ∃ {2(k % m) ! 3(k % m)}Cm x

or

m

or

0

2

k % m %1

=0

m 0

0

#

#

k % m!1 k % m %1 % ∃ (k % m ! 3)(k % m % 2)Cm x = 0. ...(4) ∃ ( k % m )(2 k % 2 m ! 3)Cm x

m

0

m

0

Equating to zero the coefficient of the smallest power of x, namely xk – 1, the above identity (4) gives the indicial equation, namely C0k(2k – 3) = 0 so that k=0 and 3/2, as C0 6 0. Here the difference of these roots = (3/2) – 0 = 3/2 6 not an integer. Here the difference of the powers of x in (4) = (k + m + 1) – (k + m – 1) = 2. Hence we equate to zero the coefficient of xk in the identity (4) and obtain C1(k + 1)(2k – 1) = 0 so that C1 = 0 for both k = 0 and k = 3/2. Next, equating to zero the coefficient of xk + m – 1 in (4), we get (k + m)(2k + 2m – 3)Cm + (k + m – 5)(k + m)Cm – 2 = 0 Cm = !

or

k%m!5 C . 2k % 2m ! 3 m ! 2

Putting m = 3, 5, 7, .... in (5) and noting that C1 = 0, we get C1 = C3 = C5 = C7 = .... = 0. Next, putting m = 2, 4, 6, .... in (5), we have C2 = !

k!3 C , 2k % 1 0

C4 = !

k !1 C 2k % 5 2

...(5) ...(6) (k ! 1)(k ! 3) C , .... ...(7) (2k % 1)(2k % 5) 0

Putting these values in (2), we have

Created with Print2PDF. To remove this line, buy a license at: http://www.software602.com/

8.26

Integration In Series

8 9 k !3 2 ( k ! 1)(k ! 3) 4 y = C0 x k :1 ! ...(8) x % x ! ....; . (2k % 1)(2 k % 5) < 2k % 1 = Putting k = 0 and replacing C0 by a in (8), y = a[1 + 3x2 + (3/5) × x4 – ....] = au, say 3/2 Putting k = 3/2 and replacing C0 by b in (8), y = x [1 + (3/8) × x2 – (3/128) × x4 + ....] = bv, say. Hence the required solution is y = au + bv, a, b being arbitrary constants. Ex. 7. Solve the hypergeometric equation x(1 – x)y( + {> – (? + ≅ + 1)x}y∋ – ?≅y = 0 near x = 0, if > is not an integer. [Meerut 1999, Kanpur 2006] Sol. Given x(1 – x)y( + {> – (? + ≅ + 1)x}y∋ – ?≅y = 0. ...(1) d2y

Dividing by x(1 – x), (1) gives

dx

%

2

> ! (? % ≅ % 1) x dy ?≅ ! y = 0. x(1 ! x ) dx x (1 ! x )

Comparing it with y( + P(x)y∋ + Q(x)y = 0, we have P(x) =

> ! ( ? % ≅ % 1) x x (1 ! x )

Q(x) = !

and

?≅ . x (1 ! x )

Since xP(x) and x2Q(x) are both analytic at x = 0, so x = 0 is a regular singular point of (1). #

y = ∃ cm x

Let the series solution of (1) be #

y∋ = ∃ cm (k % m) x

7

m

k % m!1

,

where

c0 6 0

... (2)

#

k % m!2 y( = ∃ c m (k % m)(k % m ! 1) x ...(3)

and

m 0

k%m

0

m

0

Putting the above values of y, y∋, y( in (1) gives #

2

( x ! x ) ∃ cm (k % m)(k % m ! 1)x m

k %m!2

0

#

k % m !1

m 0

#

∃ cm (k % m)(k % m ! 1 % >5 x

m

or

#

m 0

or ∃cm{(k % m)(k % m ! 1) % >(k % m)}x or

#

% [> ! (? % ≅ % 1)x ] ∃ cm (k % m)x k % m ! 1 ! ?≅ ∃ cm x k % m = 0

0

m

0

#

! ∃cm{(k % m)(k % m ! 1) % (? % ≅ % 1)(k % m) % ?≅}xk % m = 0 m 0

k % m !1

#

2

! ∃ cm{(k % m) % (? % ≅)(k % m) % ?≅}x m

#

k%m

=0

0

#

k % m !1 ! ∃ c m (k % m % ? )( k % m % ≅) x k % m = 0. ∃ cm (k % m)(k % m ! 1 % >5x

m 0

m

...(4)

0

which is an identity. Equating to zero the coefficient of the smallest power of x, namely xk – 1, (4) gives the indicial equation c0k(k – 1 + >) = 0 or k(k – 1 + >) = 0 [ c0 6 0] so the roots of the indicial equation are k = 0 1 – >, which are unequal and not differing by an integer because by assumption > is not an integer. To obtain the recurrence relation, we equate to zero the coefficient of xk + m – 1. Then we have cm(k + m)(k + m – 1 + >) – cm – 1(k + m – 1 + ?)(k + m – 1 + ≅) = 0 or

cm =

( k % m ! 1 % ? )(k % m ! 1 % ≅) cm ! 1 . (k % m)(k % m ! 1 % > )

...(5)

Case 1. When k = 0. Putting m = 1, 2, 3, .... succesively in (5), we get c1 =

?≅ c0 , 11 >

c2 =

(? % 1)(≅ % 1) c1 2 1 (> % 1)

?(? % 1)≅(≅ % 1) c0 , 11 2> ( > % 1)

Created with Print2PDF. To remove this line, buy a license at: http://www.software602.com/

Integration In Series

c3 =

8.27

(? % 2)(≅ % 2) c2 3( > % 2)

?(? % 1)(? % 2)≅(≅ % 1)(≅ % 2) c0 and so on. 1 1 2 1 3 1 >( > % 1)( > % 2)

Putting these values and k = 0 and replacing c0 by a in (2) gives 8 9 ?≅ ?( ? % 1)≅(≅ % 1) 2 ? (? % 1)(? % 2)≅(≅ % 1)(≅ % 2) 3 x% x % x % ....; y = a :1 % ...(6) 1 1 > 1 1 2 > ( > % 1) 1 1 2 1 3 1 > ( > % 1)( > % 2) < = If we take a = 1 in (6), the series on the right hand side of (6) is called hypergeometric series and is represented by 2F1(?, ≅ ; > ; x). Thus we see that 2F1(?, ≅ ; > ; x) is a solution of (1). Case 2. When k = 1 – >. Then (5) reduces to

cm =

(1 ! > % m ! 1 % ? )(1 ! > % m ! 1 % ≅) cm ! 1 (1 ! > % m)(1 ! > % m ! 1 % > )

cm =

or

(? ∋ % m ! 1)(≅ ∋ % m ! 1) cm ! 1 . m(> ∋ % m ! 1)

where ?∋ = 1 – > + ?, ≅∋ = 1 – > + ≅ and Replacing m by 1, 2, 3, .... successively in (7) gives as before c1 =

?∋≅∋ c0 , 1 1 >∋

c2 =

4? ∋ % Α54≅∋ % Α5 c1 2 1 ( >∋ % Α5

>∋ = 2 – >.

...(7) ...(8)

?∋4?∋ % Α5≅∋4≅∋ % Α5 c0 etc. 11 2 1 > ∋( > ∋ % Α5

Hence putting k = 1 – >, using the above values of c1, c2, .... in (2) and replacing c0 by b gives 8 ? ∋ 1≅∋ 9 ? ∋4?∋ % Α5≅∋4≅∋ % Α5 2 y = bx1 ! > :1 % x% x % ....; ...(9) 11 > ∋ 1 1 2 1 >∋(1 % > ∋5 < = If we take b = 1 in (9), the series on the R.H.S. of (9) would x> – 12F1(?∋, ≅∋; >∋ ; x) i.e. >–1 x 2F1(1 – > + ?, 1 – > + ≅ ; 2 – > ; x) which is another independent solution of (1). Hence the general series solution of (1) is y = a 2F1(?, ≅ ; > ; x) + b x> – 12F1(1 – > + ?, 1 – > + ≅ : 2 – > : x),

where a and b are arbitrary constants.

2 F1 ( ?,

≅; >; x ) is called hypergeometric function.

Ex. 8. Find the series solution of 4 xy ∋∋ % 2 y ∋ % y 0 . [Bilaspur 2004, Purvanchal 2005, Ravishankar 1998, 2004, Vikram 2004] Sol. Given

4 xy ∋∋ % 2 y ∋ % y

Re-writing (1),

y ∋∋ % (1/ 2 x) y ∋ % (1/ 4 x ) y

Comparing (2) with y ∋∋ % P ( x) y ∋ % Q( x ) y

0

...(1) 0

... (2)

0, we have

and Q ( x) 1/ 4 x ... (3) P 1/ 2 x Since P(x) and Q(x) are not with analytic at x = 0, so x = 0 is not ordinary point of (1). Also, xP(x) = 1/2 and x2Q(x) = x/4, showing that both P(x) and Q(x) are analytic at x = 0. Hence x = 0 is a regular singular point of (1). We, therefore, use Frobenium method to solve (1). Let a solution of (1) be of the form y 7

y∋

#

#

3 Cm x k % m ,

m 0

3 Cm (k % m) x k % m !1

m 0

and

C0 6 0

where y ∋∋

... (4)

#

3 Cm (k % m) (k % m ! 1) x k % m ! 2 ... (5)

m 0

Created with Print2PDF. To remove this line, buy a license at: http://www.software602.com/

8.28

Integration In Series

Substituting the series (4) and (5) in (1), we have #

#

#

m 0

m 0

m 0

4 x 3 Cm (k % m) ( k % m ! 1) x k % m ! 2 % 2 3 Cm (k % m) x k % m !1 % 3 Cm x k % m #

#

m 0

m 0

3 Cm Β4(k % m) ( k % m ! 1) % 2(k % m )Χ x k % m !1 % 3 Cm x k % m

or

#

#

m 0

m 0

3 Cm Β2(k % m) (2k % 2m ! 1)Χ x k % m !1 % 3 Cm x k % m

or

0

0 0,

... (6)

which is an identity in x. Equating to zero the coefficients of the smallest power of x, namely, x k !1 , (6) gives the indicial equation

2C0k(2k – 1) = 0

k = 0, 1/2

giving

[

C0 6 0 ]

Here roots of indicial equation are unequal and do not differ by integer. To obtain the recurrence relation, we equate to zero the coefficient of xk + m – 1 and obtain 2Cm(k + m) (2k + 2m – 1) + Cm–1 = 0 or

Cm

!

Cm !1 , m 1, 2, 3,... 2(k % m) (2k % 2m ! 1)

... (7)

C0 2(k % 1) (2k % 1)

... (8)

C1

Taking m = 1 in (7),

!

Next, Taking m = 2 in (7), we have C2

!

C1 2(k % 2) (2k % 3)

C0 , by (8) 4(k % 1) (k % 2) (2k % 1)(2k % 3)

Taking m = 3, in (7) and using (9), C3

!

... (9)

C0 8(k % 1) (k % 2) (k % 3) (2k % 1) (2k % 3) (2k % 5)

Substituting the above values of C1, C2, C3, ... in (4), we get y

∆Ε x x2 C0 x m Φ1 ! % ΕΓ 2(k % 1) (2k % 1) 4(k % 1) (k % 2) (2k % 1) (2k % 3) !

ΗΕ x3 % ....Ι 8(k % 1) (k % 2) (k % 3) (2k % 1) (2k % 3) (2k % 5) Εϑ

... (10)

Taking k = 0 and replacing C0 by a in (10), we have y

) ∗ x x2 x3 a +1 ! % ! % ... , + 11 2 1 1 2 1 3 1 4 11 2 1 3 1 4 1 5 1 6 , − .

) ( x )2 ( x )4 ( x )6 ∗ a +1 ! % ! % ... , + , 2! 4! 6! − .

a cos x1/ 2

Taking k = 1/2 and replacing C0 by b in (10), we have y

) ∗ ) ∗ x x2 x3 ( x )3 ( x )5 ( x )7 bx1/ 2 +1 ! % ! % ... , b + x ! % ! % ... , b sin x1/ 2 + 2 1 3 2 1 3 1 4 1 5 2 1 31 4 1 5 1 6 1 7 , + , 3! 5! 7! − . − .

Hence the required solution is y

a cos x1/ 2 % b sin x1/ 2 , a and b being arbitrary constants.

Created with Print2PDF. To remove this line, buy a license at: http://www.software602.com/

Integration In Series

8.29 2

2

Ex. 9. Using Frobenius method solve the differential equation d y/dx + (1/4x) × (dy/dx) + (1/8x2)y = 0 [Delhi Maths (Hons.) 1993 , 2006]

8x 2 y∋∋ % 2 xy∋ % y

Sol. Re-writing the given differential equation,

7 y∋

#

3 Cm (k % m) x k % m !1

... (2)

m 0

y ∋∋

and

m 0

#

3 Cm x k % m , C0 6 0

y

Let a series solution of (1) be of the form

... (1)

0

#

3 Cm (k % m) (k % m ! 1) x k % m ! 2

m 0

Substituting the above values of y, y ∋ and y ∋∋ in (1), we get #

#

#

m 0

m 0

m 0

8 x 2 3 Cm (k % m) ( k % m ! 1) x k % m ! 2 % 2 x 3 Cm (k % m) x k % m!1 % 3 Cm x k % m

or or

#

3 C m Β8( k % m ) ( k % m ! 1) % 2( k % m ) % 1Χ x k % m

m 0

#

Β

Χ

0 or 3 Cm 8( k % m) 2 ! 6 ( k % m) % 1 xk % m m 0

#

3 Cm Β4 (k % m) ! 1ΧΒ2(k % m) ! 1Χ x k % m

m 0

0,

0 0

... (3)

which is an identity in x. Equating to zero the coefficient of the smallest power in x, namely, xk, (6) gives the indicial equation C0(4k – 1) (2k –1) = 0

giving

k = 1/2, 1/4,

as

C0 6 0

which are unequal and do not differ by an integer. To obtain the recurrence relation, we equate to zero the coefficient of xk+m in (3) and obtain Cm {4(k + m) – 1} {2(k + m) – 1} = 0, for all m 0 1 ... (4) Relation (4) is satisfied by both values of k = 1/2 and k = 1/4 by choosing Cm = 0 for Hence for k = 1/2 and k = 1/4, (2) reduces to y = xk (C0 + C1x + C2x2 + ...) or y = C0xk Putting k = 1/2 and replacing C0 by a in (5), we have y = ax1/2 Next, putting k = 1/4 and replacing C0 by b in (5), we have y = bx1/4 1/2 1/4 The required solution is y = ax + bx , where a and b are arbitrary constants.

m 0 1.

... (5) ... (6) ... (7)

EXERCISE 8 (C) Find the series solution following equations near x = 0.

a{1 ! 3 x % 3 x2 /(11 3) ! ....} % bx1/ 2 (1 ! x) 2. 2xy( + (x + 1)y∋ + 3y = 0. [Delhi Maths (H) 2005] Ans. y = a (1 – 3x + 2x2 – 2x3/3...) + bx1/2 (1 – 7x/6 + 21x2/40 – 11x3 /80 ....) 3. 2x2y( – xy∋ + (x2 + 1)y = 0. 4. 3xy( + 2y∋ + x2y = 0. 8.10. Examples of Type 2 on Frobenius method. Roots of indicial equation unequal, differing by an Integer and making a coefficient of y indeterminate : In this connection the following rule should be noted. Rule : If the indicial equation has two roots k1 and k2 (say k1 < k2) and if the one of the coefficients of y becomes indeterminate when k = k2, the complete solution is given by putting k = k2 in y, which then contains two arbitrary constants. The result of putting k = k1 in y merely gives a numerial multiple of one of the series contained in the first solution. Hence we reject the solution obtained by putting k = k1. 1. 2x(1 – x)y( + (1 – x)y∋ + 3y = 0.

Ans. y

Created with Print2PDF. To remove this line, buy a license at: http://www.software602.com/

8.30

Integration In Series 2

2

Ex. 1. Find solution near x = 0 of x y( + (x + x )y∋ + (x – 9)y = 0. Sol. Given x2y( + (x + x2)y∋ + (x – 9)y = 0. ...(1) 2 Dividing by x , (1) can be put in standard form as y( + {(1 + x)/x}y∋ + {(x – 9)/x2}y = 0. Comparing it with y( + P(x)y∋ + Q(x)y = 0, here P(x) = (1 + x)/x and Q(x) = (x – 9)/x2 so that xP(x) = 1 + x and x2Q(x) = x – 9. Since both xP(x) and x2Q(x) are analytic at x = 0, hence x = 0 is a regular singular point of (1). Let the series solution of (1) be #

y = xk(C0 + C1x + C2x2 + .... ad. inf.) = ∃ Cm x m

#

7 y∋ = 3 (k % m)Cm x k % m ! 1

0

, where C0 6 0.

...(2)

#

y( = ∃ (k % m)(k % m ! 1)Cm x k % m ! 2 .

and

m 0

k%m

m

...(3)

0

Putting the above values of y, y∋ and y( in (1), we have x

2

#

∃ (k % m)(k % m ! 1)Cm x

k%m!2

2

#

% ( x % x ) ∃ ( k % m )Cm x

m 0

k % m !1

m 0

#

% ( x ! 9 ) ∃ Cm x m

k%m

=0

0

#

#

#

#

#

m 0

m 0

m 0

m 0

m 0

or ∃(k % m)(k % m ! 1)Cm xk % m % ∃(k % m)Cm xk % m % ∃(k % m)Cmxk % m %1 % ∃Cmxk % m %1 ! 9 ∃Cmxk % m = 0 #

∃ {(k % m)(k % m ! 1) % (k % m) ! 9}Cm x

or

m

k%m

0

#

2

2

∃ {(k % m ) ! 3 }Cm x

or

m

k%m

0

#

% ∃ {( k % m) % 1}Cm x k % m % 1 = 0 m 0

#

% ∃ (k % m % 1)Cm x

k % m%1

=0

m 0

#

#

k%m % ∃ (k % m % 1)Cm x k % m % 1 = 0. ∃ (k % m % 3)(k % m ! 3)Cm x

or

m

0

m

...(4)

0

Equating to zero the coefficient of the smallest power of x, namely xk, the above identity (4) gives the indicial equation (k + 3)(k – 3)C0 = 0 so that k = 3, –3 as C0 6 0. k+m Next, equating to zero the coefficient of x in (4), we get (k + m + 3)(k + m – 3)Cm + (k + m)Cm – 1 = 0 Cm = !

or

Putting m = 1, 2, 3, .... in (5), we get C2 = !

(k % 2) C ( k % 5)( k ! 1) 1

(k % m) C . ( k % m % 3)( k % m ! 3) m ! 1

C1 = !

...(5)

k %1 C , ( k % 4 )( k ! 2 ) 0

( k % 1)( k % 2 ) C , ( k ! 1)( k ! 2 )( k % 4 )( k % 5) 0

( k % 3) ( k % 1)( k % 2 )( k % 3) C ! C ( k % 6 )k 2 k ( k ! 1)( k ! 2 )( k % 4 )( k % 5)( k % 6 ) 0 and so on. Putting these values in (2), we have

C3 = !

LM N

y = C0xk 1 !

OP Q

(k % 1) (k % 1)(k % 2) (k % 1)(k % 2)(k % 3) 2 3 x% x ! x % .... ad. inf. (k ! 2)(k % 4) (k ! 1)(k ! 2)(k % 4)(k % 5) k(k ! 1)(k ! 2)(k % 4)(k % 5)(k % 6)

Putting k = 3 and replacing C0 by a in (6), we have

4 415 4 1516 8 9 y = ax3 :1 ! x% x2 % x 3 ! ....; 2 11 1 7 1 8 3 1 2 1 11 7 1 8 1 9 < 217 = Putting k = –3 and replacing C0 by b in (6), we have

...(6)

au , say

Created with Print2PDF. To remove this line, buy a license at: http://www.software602.com/

Integration In Series

8.31 –3

2

y = bx {1 – (2/5)x + (1/20)x } = bv, say. The required solution is y = au + bv, where a and b are arbitrary constants. Important note. When x = 0 is an ordinary point of y( + P(x)y∋ + Q(x)y = 0, we have already explained the method of solution in Art. 8.5. We have solved many problems based on Art. 8.5. All those problems can also be solved by Frobenius method as given in Art. 8.10 and explained in above solved Ex. 1. We now solve the same problems by method of solved Ex. 1 as follows. Ex. 2. Solve in series (1 – x2)y( – xy∋ + 4y = 0. Sol. Given (1 – x2)y( – xy∋ + 4y = 0. ...(1) Clearly x = 0 is an ordinary point of (1). Let the series solution of (1) be of the form #

y = ∃ Cm x k % m , m

where

C0 6 0.

and

y( =

0

#

k % m!1 y∋ = ∃ Cm (k % m) x

7

m

0

...(2)

#

∃C

m

m

( k % m )( k % m ! 1 ) x k

...(3)

Putting the above values of y, y∋, y( in (1) gives m

or

#

#

(1 ! x 2 ) ∃ Cm (k % m)(k % m ! 1) x k % m ! 2 ! x ∃ Cm ( k % m) x

or #

∃ Cm (k % m)(k % m ! 1) x

m 0

or

k %m!2

k % m !1

0

#

#

#

% 4 ∃ Cm x m 0

k%m

! ∃ Cm (k % m)(k % m ! 1)x k % m ! ∃Cm(k % m)x m 0

m 0

k%m

=0

#

% 4 ∃Cmxk % m = 0 m 0

#

#

k %m!2 ! ∃ Cm [( k % m)(k % m ! 1) % (k % m) ! 4]x k % m = 0 ∃ Cm (k % m)(k % m ! 1) x

m

m

0

#

∃ Cm (k % m)(k % m ! 1) x

or

m

or

m

0

% m ! 2

0

#

0

k %m!2

0

∃ Cm (k % m)(k % m ! 1) x

m

0

k %m!2

#

2

! ∃ Cm [(k % m) ! 4]x m

k%m

=0

0

#

! ∃ Cm (k % m % 2 )(k % m ! 2 ) x k % m = 0, m

...(4)

0

which is an identity. Equating to zero the coefficient of the smallest power of x, namely xk – 2, (4) gives the indicial equation C0k(k – 1) = 0 or k(k – 1) = 0 [ C0 6 0] giving k = 1 and k = 0. These are unequal and differ by an integer. To get the recurrence relation, we equate to zero the coefficient of xk + m – 2. Thus, Cm(k + m)(k + m – 1) – Cm – 2(k + m)(k + m – 4) = 0 giving

Cm =

k%m!4 C . k % m !1 m!2

...(5)

Next, we equate to zero the coefficient of xk – 1 and get C1(k + 1)k = 0. ...(6) If we take k = 0, (6) shows that C1 is indeterminate.With k = 0 and using (5), we can express C2, C4, C6 .... in terms of C0 and C3, C5, C7 .... in terms of C1 if we assume C1 to be finite. Thus, Cm = {(m – 4)/(m – 1)} Cm – 2 so that C2 = –2C0, C4 = 0 and hence C6 = C8 = .... = 0 and C3 = (–C1)/(2) = –C1/2, C5 = C3/(4) = –C1/8, C7 = (3C5)/(6) = –C1/16, and so on. Putting k = 0 and these values in (2), i.e. y = xk(C0 + C1x + C2x2 + C3x3 + C4x4 + C5x5 + .... ) i.e. y = x0(C0 + C2x2 + C4x4 + ....) + x0(C1x + C3x3 + C5x5 + ....), we get y = C0(1 – 2x2) + C1(x – x3/2 – x5/8 + x7/16 – ....) ...(7) which is the required series solution, C0 and C1 being two arbitrary constants.

Created with Print2PDF. To remove this line, buy a license at: http://www.software602.com/

8.32

Integration In Series

Remarks. The reader can easily verify that the root k = 1 of the indicial equation gives another solution as usual (Refer examples of Type 1). But this solution, will be a constant multiple of one of the series occurring in (7). So we reject it. Ex. 3. Solve in series the Legendre’s equation (1 – x2)y( – 2xy∋ + n(n + 1)y = 0 [Delhi Maths (Hons.) 1996, Nagpur 1996, Utkal 2003] Sol. Given (1 – x2)y( – 2xy∋ + n(n + 1)y = 0 ...(1) Here x = 0 is an ordinary point of (1). Let the series solution of (1) be of the form #

y = ∃ Cm x m

#

7 y∋ = ∃ Cm (k % m) x m

k%m

0

k % m!1

, where C0 6 0. #

y( = ∃ Cm (k % m)(k % m ! 1) x k % m ! 2 .

and

0

...(2)

m

0

...(3) Putting the above values of y, y∋, y( into (1) gives #

#

k %m!2 ! x 2 ∃ Cm (k % m)(k % m ! 1) x k % m ! 2 ∃ Cm (k % m)(k % m ! 1) x

m

0

m

0

#

! 2x



Cm ( k % m) x k % m ! 1 % n(n % 1)

m 0

∃C

mx

k %m!2 ! ∃ {(k % m)(k % m ! 1) % 2( k % m ) ! n(n % 1)}x ∃ Cm (k % m)(k % m ! 1) x

or

m

m

0

k %m

m 0

#

#

#

k%m

0

=0 =0

#

#

k %m!2 ! ∃ Cm{(k % m) 2 % (k % m) ! n 2 ! n)}x k % m = 0 ∃ Cm (k % m)(k % m ! 1) x

or

m

m

0

#

∃ Cm (k % m)(k % m ! 1) x

or

m

k %m!2

0

#

∃ Cm (k % m)(k % m ! 1) x

or

m

0

0

#

! ∃ Cm{(k % m % n)( k % m ! n) % (k % m ! n )}x k % m = 0 m

k %m!2

0

#

! ∃ Cm (k % m ! n)( k % m % n % 1) x k % m = 0, ...(4) m

0

which is an identity. Equating to zero the coefficient of the smallest power of x, namely xk – 2, (4) gives the identical equation c0k(k – 1) = 0 or k(k – 1) = 0, [ c0 6 0] giving k = 1 and k = 0. These are unequal and differ by an integer. To get the recurrence relation, we equate to zero the coefficient of xk + m – 2. Thus Cm(k + m)(k + m – 1) – Cm – 2(k + m – 2 – n)(k + m – 2 + n + 1) = 0 ( k % m ! 2 ! n)(k % m ! 1 % n) Cm ! 2 . (k % m)(k % m ! 1) Next, equating to zero the coefficient of xk – 1 gives C1(k + 1)k = 0. If we take k = 0, (6) shows that C1 is indeterminate. For k = 0, (5) gives

so that

Cm =

Cm =

( m ! 2 ! n)( m ! 1 % n) Cm ! 2 . m( m ! 1)

...(5) ...(6)

...(7)

We now express C2, C4, C6, .... in terms of C0 and C3, C5, C7, .... in terms of C1 by assuming that C1 is finite. Putting m = 2 in (7) gives

Created with Print2PDF. To remove this line, buy a license at: http://www.software602.com/

Integration In Series

8.33

C2 =

( !2)( n % 1) C0 2 11

!

n( n % 1) C0 . 2!

...(8)

Putting m = 4 in (7) and using (8) gives C4 =

(2 ! n )(3 % n) C2 31 2

( n ! 2) n( n % 1)( n % 3) C0 . 4!

...(9)

Next, putting m = 3 in (7) gives C3 =

(1 ! n)(2 % n) C1 31 2

!

(n ! 1)(n % 2) C1 . 3!

...(10)

Again, putting m = 5 in (7) and using (10) gives C5 =

(3 ! n)(4 % n ) C3 514

(n ! 3)(n ! 1)(n % 2)(n % 4) C1 5!

and so on. Now (2) can be re–written as y = xk(C0 + C1x + C2x2 + C3x3 + C4x4 + ....) 0 or y = x (C0 + C2x2 + C4x4 + ....) + x0(C1x + C3x3 + C5x5 + ....) [ k = 0] Using the values of C2, C3, C4, C5, .... as given by (8), (9), (10), (11) etc. in the above equation gives

LM N

OP Q

n ( n % 1) 2 (n ! 2) n ( n % 1)( n % 3) 4 (n ! 1)(n % 2) 3 (n ! 3)(n ! 1)(n % 2)(n % 4) 5 9 x % x ! ....; % C1 x ! x % x % .... 3! 5! 2! 4! =

y = C0 8:1 !
k2) differing by an integer, and if some of the coefficients of y become infinite when k = k2, we modify the form of y by replacing c0 by do (k – k2). We then obtain two independent solutions by putting k = k2 in the modified form of y and Κy/Κk. The result of putting k = k1 in y gives a numerical multiple of that obtained by putting k = k2 and hence we reject the solution obtained by putting k = k1 in y. Ex. 1. Solve in series x(1 – x)y( – 3xy∋ – y = 0 near x = 0. [Delhi Maths (H) 2009] Sol. Given x(1 – x)y( – 3xy∋ – y = 0. ...(1) Ans. y

2

d y 3 dy 1 ! ! y = 0. 2 1 ! x dx x ( 1 ! x) dx Comparing it with y( + P(x)y∋ + Q(x)y = 0, hence P(x) = –3/(1 – x) and Q(x) = –1/{x(1 – x)} so that xP(x) = –3x/(1 – x) and x2Q(x) = –x/(1 – x). Since xP(x) and x2Q(x) are both analytic at x = 0, so x = 0 is a regular singular point of (1). Let the series solution of (1) be

Dividing by x(1 – x), (1) yields

#

y = ∃ cm x k % m , where c0 6 0. m

#

7 y∋ = ∃ cm (k % m)x k % m ! 1

...(2)

0

#

y( = ∃ cm ( k % m )(k % m ! 1)x k % m ! 2 . ...(3)

and

m 0

m

0

Putting the above values of y, y∋, y( into (1) gives #

#

#

( x ! x 2 ) ∃ cm ( k % m )(k % m ! 1)x k % m ! 2 ! 3x ∃ cm (k % m)x k % m ! 1 ! ∃ cm x k % m = 0 m 0

or

m

#

∃ cm (k % m)(k % m ! 1)x

m 0

or or

k % m !1

0

m 0

#

#

#

m 0

m 0

m 0

! ∃ cm (k % m)(k % m ! 1)xk % m ! 3 ∃cm (k % m)xk % m ! ∃cmxk % m = 0

#

#

m 0

m 0

k % m !1 k%m ! ∃ Cm {(k % m)(k % m ! 1) % 3(k % m) % 1}x =0 ∃ cm (k % m)(k % m ! 1)x

#

∃ cm (k % m)(k % m ! 1)x

m 0

k % m !1

#

! ∃ cm {(k % m)2 % 2(k % m) % 1} x k % m = 0 m 0

Created with Print2PDF. To remove this line, buy a license at: http://www.software602.com/

8.36

Integration In Series #

∃ cm ( k % m)( k % m ! 1)x

or

m

k % m !1

0

#

2

! ∃ cm (k % m % 1) x

k%m

= 0.

...(4)

m 0

which is an identity. Equating to zero the coefficient of the smallest power of x, namely xk – 1, (4) gives the indicial equation c0k(k – 1) = 0 or k(k – 1) = 0. [ c0 6 0] which gives k = 0 and k = 1. These are unequal and differ by an integer. Next, to find the recurrence relation we equate to zero the coefficient of xk + m – 1 and obtain cm(k + m)(k + m – 1) – cm – 1(k + m)2 = 0

cm =

or

k%m c . k % m ! 1 m !1

...(5)

Putting m = 1 in (5) gives

c1 = {(k + 1)/k}C0.

...(6)

Putting m = 2 in (5) and using (6) gives

c2 =

k%2 c k %1 1

...(7)

k%2 c0 . k

k%3 k%3 c c . k%2 2 k 0 Putting these values in (2), i.e., y = xk(c0 + c1x + c2x2 + ....), gives

Putting m = 3 in (5) and using (7) gives

LM N

k y = c0 x 1 %

c3 =

...(8)

OP Q

k %1 k%2 2 k %3 3 x% x % x % .... . k k k

...(9)

If we put k = 0 in (9), we find that due to presence of the factor k in their denominators, the coefficients becomes infinite. To remove this difficulty, we write c0 = k do in (9). Then (9) becomes y = do xk[k + (k + 1)x + (k + 2)x2 + (k + 3)k3 + ....] ...(10) Putting k = 0 and replacing do by a in (10) gives y = a(x + 2x2 + 3x3 + ....) = au, say ...(11) To obtain a second solution, if we put k = 1 in (9) we obtain y = c0(x + 2x + 3x2 + ....) ...(12) which is not distinct (i.e. not linearly independent because ratio of the two series in (11) and (12) is a constant) from (11). Hence (12) will not serve the purpose of a second solution. In such a case the second independent solution is given by (Κy/Κk)k = 0. Differentiating (10) partially w.r.t. ‘k’ Κy/Κk = do xk log x [k + (k + 1)x + (k + 2)x2 + ....] + do xk [1 + x + x2 + ....]. ...(13) Putting k = 0 and replacing do by b (13), gives (Κy/Κk)k = 0 = b log x (x + 2x2 + 3x3 + ....) + b(1 + x + x2 + ....) or (Κy/Κk)k = 0 = b[u log x + (1 + x + x2 + ....)] = bv, by (11) ...(14) The required solution is y = au + bv, where a and b are arbitrary constants. Ex. 2. Solve in series the Bessel’s equation of order 2, near x = 0, x2y( + xy∋ + (x2 – 4)y = 0. [Delhi Maths (Hons.) 1995, Meerut 1995] Sol. Given x2y( + xy∋ + (x2 – 4)y = 0. ...(1) 2 2 2 Dividing by x , y( + (1/x)y∋ + [(x – 4)/x ]y = 0. Comparing it with y( + P(x)y∋ + Q(x)y = 0, here P(x) = 1/x and Q(x) = (x2 – 4)/x2 so that xP(x) = 1 and x2Q(x) = x2 – 4. Since xP(x) and x2Q(x) are both analytic at x = 0, hence x = 0 is a regular singular point of (1). Let the series solution of (1) be #

y = ∃ cm x m

7

#

y∋ = ∃ cm (k % m)x k % m ! 1 m 0

0

and

k%m

, where c0 6 0. #

y( = ∃ cm ( k % m)(k % m ! 1)x m

0

...(2) k%m!2

...(3)

Created with Print2PDF. To remove this line, buy a license at: http://www.software602.com/

Integration In Series

8.37

Putting the above values of y, y∋, y( into (1) gives #

#

#

#

2 k%m ! 4 ∃ cm x k % m = 0 x 2 ∃ cm (k % m)(k % m ! 1)x k % m ! 2 % x ∃ cm ( k % m)x k % m ! 1 % x ∃ cm x m 0

m

m

0

0

m

#

#

#

#

m 0

m 0

m 0

m 0

0

k %m k % m%2 2 k%m % ∃ cm x k % m % 2 = 0 or ∃cm{(k % m)(k % m!1) % (k % m) ! 4}x % ∃cmx = 0 or ∃ cm {(k % m) ! 4}x

#

∃ cm (k % m % 2)(k % m ! 2)x

or

m

0

k%m

#

% ∃ cm x

k %m %2

= 0,

...(4)

m 0

which is an identity. Equating to zero the coefficient of the smallest power of x, namely xk, gives the indicial equation c0(k + 2)(k – 2) = 0 or (k + 2)(k – 2) = 0 [ c0 6 0] This gives k = 2 and k = –2. These are unequal and differ by an integer. For the recurrence relation, we equate to zero the coefficient of xk + m and get

1 c . ...(5) (k % m % 2)( k % m ! 2) m ! 2 To determine c1, we equate to zero the coefficient of xk + 1 and get c1(k + 3)(k – 1) = 0 giving c1 = 0 for both the roots k = 2 and k = –2 of the indicial equation. Now using c1 = 0 and (5), we get c1 = c3 = c5 = c7 = .... = 0. ...(6) Next, putting m = 2, 4, 6, .... in (5) and simplifying, we have c2 = – c0/k(k + 4), c4 = –c2/(k + 2)(k + 6) = c0/k(k + 2)(k + 4)(k + 6), c6 = – c4/(k + 4)(k + 8) = – c0/k(k + 2)(k + 4)2(k + 6)(k + 8) and so on. Putting these values in (2), i.e., y = xk(c0 + c1x + c2x2 + c3x3 + c4x4 + c5x5 + c6x6 + c7x7 + ....), we get cm(k + m + 2)(k + m – 2) + cm – 2 = 0

RS T

y = c0 x k 1 !

cm = !

so

UV W

6 x x2 x4 ! % .... % k (k % 4) k (k % 2)(k % 4)(k % 6) k (k % 2 )(k % 4) 2 (k % 6)( k % 8)

...(7)

∆ x2 x4 x6 ΕΗ 2 Ε % ! % ....Ι c0 Λ , say ...(8) Putting k = 2 in (7) yields y = c0 x Φ1 ! 2 ΕΓ 2 1 6 2 1 4 1 8 1 6 2 1 4 1 6 1 8 110 Εϑ

Next, if we put k = –2 in (7), the coefficients of x4, x6, ... become infinite. To get rid of this difficulty, we put c0 = do(k + 2) in (7) and obtain modified solution as

RS T

k y = do x (k % 2 ) !

UV W

2

4 6 (k % 2) x x x ! ! % .... 2 k ( k % 4) k ( k % 4 )( k % 6) k ( k % 4) ( k % 6)(k % 8)

...(9)

Putting k = –2 and replacing do by a in (9) gives ∆

Ε y = ax!2 Φ0 ! 0.x2 % ΓΕ

Now,

ΗΕ ΗΕ !ax2 Ε∆ x2 x4 x4 x6 ! % .... 1 ! % ! ....Ι au , say ...(10) or y = Φ Ι (!2)(2)(4) (!2)(2)2 (4)(6) 16 2 1 6 2 1 4 1 8 1 6 ϑΕ ΓΕ ϑΕ

(8) and (10)

2

u= !

ΗΕ x 2 Ε∆ x2 x4 % ! ....Ι Φ1 ! 16 ΓΕ 2 1 6 2 1 4 1 8 1 6 ϑΕ

!

Λ , 16

showing that w and u are dependent solutions. Hence we must find one more independent solution in order to obtain the required general solution. To get another independent solution, substituting (9) into the L.H.S. of (1) and simplifying, we find x2y( + xy∋ + (x2 – 4)y = d0(k – 2)(k + 2)2xk. ... (11)

Created with Print2PDF. To remove this line, buy a license at: http://www.software602.com/

8.38

Integration In Series

Differentiating both sides of (11) partially w.r.t. k, we get

LM N

OP Q

2

Κ Κ x 2 d y % x dy % ( x 2 ! 4 ) y = d 0 [(k % 2)2 1 x k (k ! 2)] 2 Κk dx dx Κk

8 d2 9 Κy d or : x 2 2 % x % ( x 2 ! 4) ; dx < dx = Κk

2d 0 (k % 2) 1 x k (k ! 2) % d0 (k % 2)2 [ x k log x 1 (k ! 2) % x k 11] ...(12)

The presence of the factor (k + 2) in each term on R.H.S. of (12) shows that a second solution is (Κy/Κk)k = –2, where y is given by (9). Differentiating (9) partially w.r.t. ‘k’ we get

RS T

UV W

2

4 6 (k % 2 ) x Κy k x x % ! % .... = d0 x log x (k % 2) ! 2 k (k % 4 ) k (k % 4)(k % 6) k ( k % 4) ( k % 6)(k % 8) Κk

RS T

% d0 x k 1 !

2

FG H

IJ K

(k % 2 ) x ) 1 1 1 ∗ 1 !1! 1 x4 % ! ! ! + k (k % 4 ) k % 2 k k % 4 k(k % 4)(k % 6) − k k % 4 k % 6 ,.

!

x6 2

k (k % 4) (k % 6)(k % 8)

FG H

/ !

IJ K

UV W

1 2 1 1 ! ! ! % .... ... (13) k k%4 k%6 k%8

LMTo find d RS k % 2 UV, we proceed as follows : Take z = k % 2 so that log z = logL k % 4 O MN k (k % 4) PQ dk T k (k % 4) W k (k % 4) N or

log z = log (k + 4) – log k – log (k + 4). Differentiating it w.r.t. ‘k’, we get 1 dz = 1 !1! 1 z dk k%4 k k%4

2

1 1 ∗ k%2 ) 1 ! ! + , etc. k ( k % 4) − k % 4 k k % 4 .

d ∆ k%2 Η Φ Ι dk Γ k ( k % 4) ϑ

Other terms can be similarly differentiated easily] Putting k = –2 and replacing do by b in (13) gives

FG Κy IJ H Κk K

k

!2

∆ x2 x4 x6 x4 ) 1 ∗ Ε∆ ΕΗ !2 2 !2 Ε bx log x 0 ! 0. x % ! % .... % bx 1 % % Φ Ι Φ + , = 2 (!2)(2)(4) (!2)(2) 2 (4)(6) 22 1 4 − 4 . ΕΓ Εϑ ΕΓ 2 %

ΗΕ 8 ∆ x2 1 1∗ )1 x4 ΕΗ9 !2 Ε + ! 1 % ! , % ....Ι = b :u log x % x Φ1 % 2 % 2 2 ! ....Ι; bv , by (10) 4 6. 2 1416 − 2 2 14 Εϑ Εϑ;= ΓΕ 2

1

∃1

z

1

∃1

2n n 1 2 = 2n ) 1 2(n ∃ 1) ) 1 (2n ) 1)(2n ∃ 1)

or

2n . 4n2 ∃ 1

R| S| T

0, if l ∋ m 2 (1 ∃ x ) P'l . Pm' dx = 2l(l ) 1) , if l % m 2l ) 1

(1 ∃ x 2 ) P'l P'm dx %

2l (l ) 1) Θlm , 2l ) 1

where

0, if l ∋ m Θlm = 1, if l % m

{

[Indore 2004; Purvanchal 2004; Ravishakar 2005]

Created with Print2PDF. To remove this line, buy a license at: http://www.software602.com/

9.22

z

Legendre Polynomials

Sol. Case I. Let l ∋ m. Then integrating by parts, we have

1

1

2

∃1

[(1 ∃ x ) P'l ] Pm' dx = (1∃ x2) P'l . Pm –

>

∃1

1

∃1

[(1 ∃ x 2 ) Pl ∃ 2 x Pl ] Pm dx = –

z

1

But

∃1

Using (2), (1) reduces to

z z

1

1

∃1

2 [(1 ∃ x ) P" l ∃ 2x P' l ] Pm dx . ...(1)

(1 – x2)y! – 2xy + l(l + 1)y = 0, hence

Since Pl satisfies Legendre’s equation

(1 ∃ x 2 ) Pl ∃ 2 xPl ) l (l ) 1) Pl % 0

z

(1 ∃ x2 ) Pl ∃ 2 xPl % ∃l (l ) 1) Pl .

or

...(2)

Pl Pm dx = 0, if l ∋ m

...(3)

z

1

2

(1 ∃ x ) P'l Pm' dx = l(l + 1) Pl Pm dx = 0, using (3).

∃1

...(4)

∃1

Case II. Let l = m. Then the required result takes the form 1

∃1

2

2

(1 ∃ x ) ( P'l ) dx %

2l(l ) 1) . 2l ) 1

[Agra 2010]

...(5)

We have, by using integration by parts,

z

1

∃1

2

z

1

2

(1 ∃ x ) (P'l ) dx = 2 = (1 ∃ x ) P'l Pl

= l(l + 1) .

2

[(1 ∃ x ) P'l ] . P'l dx

∃1

1 ∃1



z

1

∃1

2

(1 ∃ x )P" l ∃ 2 xP' l Pl dx = 0 + l(l + 1)

2 = 2l(l ) 1) . 2l ) 1 2l ) 1

z

1

Combining (4) and (5) and using symbol Θlm, we get

z

1

∃1

∃1

2l(l ) 1) Θ . 2l ) 1 lm

2n (n ) 1) [Agra 2009; Nagpur 2010] (2n ) 1) (2n ) 3) (2n + 1) (x2 – 1) = n(n + 1) (Pn + 1 – Pn – 1). ...(1)

n (n ) 1) (P ∃ Pn ∃ 1) . ...(2) 2n ) 1 n ) 1 Multiplying both sides of (2) by Pn + 1 and then integrating w.r.t. x from –1 to 1, we have

(x2 – 1)P n =

From (1),

z

(1 ∃ x 2 ) Pl Pm dx =

( x 2 ∃ 1) Pn ) 1 Pn dx =

Sol. Refer Art. 9.10 to show that

1

(Pl )2 dx , using (2)

∃1

(2n + 1) (x2 – 1)P n = n(n + 1) (Pn + 1 – Pn – 1)

Ex. 8. Prove that and hence prove that

∃1

z

1

(x2 ∃ 1) Pn ) 1 P'n dx = n(n)1)

2n)1

z

1

∃1

Pn)1 (Pn)1 ∃ Pn∃1) dx =

z

1

But

∃1

Using (4), (3) reduces to

z

1

∃1

2

z

1

∃1

z

LM N

1

∃1

Pn2)1 dx ∃

z

1

∃1

OP Q

Pn ) 1 Pn ∃1 dx

0, if m ∋ n Pm Pn dx % n / (2n ) 1), if m % n

( x ∃ 1) Pn ) 1 P'n dx =

Ex. 9. (a). Show that

n (n ) 1) 2n ) 1

{

LM N

...(3) ...(4)

OP Q

2n (n ) 1) n (n ) 1) 2 ∃0 = (2n ) 1) (2n ) 3) 2n ) 1 2(n ) 1) ) 1

x 2 Pn ) 1( x ) Pn ) 1( x ) dx =

2n (n ) 1) (2n ∃ 1) (2n ) 1) (2n ) 3) [Kanpur 2007, 08]

Created with Print2PDF. To remove this line, buy a license at: http://www.software602.com/

Legendre Polynomials

9.23

(b) Deduce the value of

z

1

0

x2 Pn )1(x) Pn )1(x) dx. i.e., prove that

z

1

0

2

x Pn)1(x) Pn)1(x) dx =

n (n ) 1) . (4n2 ∃ 1) (2n ) 3)

Sol. (a) From recurrence relation I, (2n – 1)xPn – 1 = nPn + (n – 1)Pn – 2. ...(1) Replacing n by n + 2 in (1), (2n + 3)xPn + 1 = (n + 2)Pn + 2 + (n + 1)Pn. ...(2) Multiplying the corresponding sides of (1) and (2), we get (2n – 1) (2n + 3)x2 Pn – 1Pn + 1 = n(n + 1)Pn2 + n(n + 2)Pn + 2 Pn + (n – 1) (n + 2)Pn – 2 Pn + 2 + (n – 1) (n + 1)Pn – 2 Pn. ...(3)

z

1

Also,

0, if m ∋ n Pm Pn dx = 2 / (2n ) 1), if m % n

{

∃1

...(4)

Integrating (3) w.r.t. ‘x’ from –1 to 1 and using (4), we get (2n – 1) (2n + 3)

z

1

or

∃1

z

1

∃1

x 2 Pn ∃ 1( x) Pn ) 1dx = n(n ) 1) .

2

x Pn ∃ 1( x ) Pn ) 1dx %

2 )0)0 2n ) 1

2n(n ) 1). (2n ) 3) (2n ∃ 1) (2n ) 1)

...(5)

Part (b). Deduction. Since Pn(x) is a polynomial of degree n, so x2Pn – 1(x) Pn + 1(x) is a polynomial of degree 2 + (n – 1) + (n + 1) i.e. 2(n + 1). Since 2(n + 1) is even, we see that x2Pn – 1 Pn + 1 is an even function of x and hence

z

1

z

1

or

0

∃1

x 2 Pn ∃ 1( x) Pn ) 1dx = 2

x 2 Pn ∃ 1(x ) Pn ) 1dx =

Ex. 10. Prove that

z

1

∃1

x 2 Pn2 dx =

1 2

z

1

∃1

1

x 2 Pn ∃ 1 Pn ) 1 dx

0

x 2 Pn ∃ 1 Pn ) 1 dx =

n(n ) 1) , by (5). (2n ) 3) ( 4n 2 ∃ 1)

1 3 1 ) ) . 8(2n ∃ 1) 4(2n ) 1) 8(2n ) 3) (2n + 1)xPn = (n + 1)Pn

Sol. From recurence relation I, Squaring both sides, we have (2n + 1)2 x2Pn2 = (n + 1)2P2n

z

1

Also,

z

∃1

+1

+ n2P2n

–1

+1

+ nPn

– 1.

+ 2n(n + 1)Pn + 1Pn – 1. ...(1)

Pm Pn dx = 0, if m ∋ n 2 / (2n ) 1), if m % n

{

...(2)

Integrating both sides of (1) w.r.t. ‘x’ between the limits –1 to 1 and using (2), we have (2n ) 1)2

;

z

1

x 2 Pn2 dx =

∃1

z

1

x 2 Pn2 dx = (n ) 1)2

∃1

2 2 ) n2 )0 2(n ) 1) ) 1 2(n ∃ 1) ) 1

LM N

OP Q

(n )1): 1 3 1 2 n2 ) ) 2 2n ) 3 ) 2n ∃ 1 = (2n ) 1) 8(2n ∃ 1) 4(2n ) 1) 8(2n ) 3)

(on resolving into partial fractions) Ex. 11. Prove that xPn = nPn + (2n – 3)Pn – 2 + (2n – 7)Pn – 4 + ... and hence or otherwise show that (a) (b)

z

1

∃1

z

1

∃1

x Pn Pn' dx = (2n)/(2n + 1).

x Pn Pm' dx = either 0 or 2 or (2n)/(2n + 1).

Sol. From recurrence relation II, we have xP n = nPn + P n – 1 or

xP n – P n – 1 = nPn. ...(1)

Created with Print2PDF. To remove this line, buy a license at: http://www.software602.com/

9.24

Legendre Polynomials

Again from recurrence relation III, we have P n + 1 = (2n + 1)Pn + P n – 1 or P n + 1 – P n – 1 = (2n + 1)Pn Replacing n by n – 2, n – 4, n – 6, ... successively in (2), we get P n – 1 – P n – 3 = (2n – 3)Pn – 2 P n – 3 – P n – 5 = (2n – 7)Pn – 4 .... .... .... ... ... .... .... ... Adding (1) and (3) and simplifying, we get xP n = nPn + (2n – 3)Pn – 2 + (2n – 7)Pn – 4 + .... Part (a). Multiplying both sides of (4) by Pn, we get xPnP n = nPn2 + (2n – 3)Pn – 2Pn + (2n – 5)Pn – 4Pn + ...

z

1

0, if m ∋ n x Pm Pn dx = 2 / (2n ) 1), if m % n . ∃1

Also

{

...(2)

...(3)

...(4) ...(5) ...(6)

Integrating both sides of (5) w.r.t. x from –1 to 1 and using (6), we have

z

1

∃1

x Pn Pn' dx = n .

2 ) 0 ) 0 ) ... % 2n . 2n ) 1 2n ) 1

...(7)

Part (b). Replaing n by m in (4), we get xP m = mPm + (2m – 3)Pm – 2 + (2m – 7)Pm – 4 + ... Multiplying both sides of (8) by Pn, we get xPnP m = mPmPn + (2m – 3)Pm – 2Pn + (2m – 7)Pm – 4Pn + ... Integrating both sides of (9) w.r.t. ‘x’ from –1 to 1 and using (6), three cases arise: Case I. When n is different from m, m – 2, m – 4, ... and so on. Then

z

1

∃1

...(8) ...(9)

x Pn Pm' dx = 0 + 0 + 0 + ... = 0.

z

1

Case II. When n = m. Then,

∃1

x Pn P' n dx = n ∗

2 2n ) 0 ) 0 ) ... % . 2n ) 1 2n ) 1

Case III. When n = m – 2. Then n ∋ m, n ∋ (m – 4), n ∋ (m – 6), ... and so on. So we obtain

>

1

∃1

x Pn Pm' dx % 0 ) (2 m ∃ 3) 1

= (2n + 1) × Similarly we can prove that Thus

z

1

∃1

x Pn Pm' dx = 0

Ex. 12. Prove that

z

1

∃1

+ n % m ∃ 2, −6 m % n ) 2 . / 0

2 2 % [2( n ) 2) ∃ 3] 1 2n ) 1 2n ) 1

2 = 2. 2n ) 1

z

1

∃1

x Pn Pm' dx = 2, when n = m – 4 or n = m – 6 etc.

or

2

or

2n . 2n ) 1

2 (Pn' ) dx = n(n + 1).

Sol. From Christoffel’s expansion, we get P n = (2n – 1)Pn – 1 + (2n – 5)Pn – 3 + (2n – 9)Pn – 5 + ... The last term on R.H.S. of (1) is 3P1 or P0 according as n is even or odd.

...(1)

Created with Print2PDF. To remove this line, buy a license at: http://www.software602.com/

Legendre Polynomials

9.25 2

2

2

2

But (a + b + c + d + ....) = (a + b + c + ...) + 2&ab. Squaring both sides of (1) and using (2), we have P n2 = (2n – 1)2P2n – 1 + (2n – 5)2 P2n – 3 + (2n – 9)2P2n – 5 + ... + 32P12 + 2&(2n – 1) (2n – 5)Pn – 1Pn – 3, if n is even P n2= (2n – 1)2P2n – 1 + (2n – 5)2P2n – 3 + ... + P02 + 2&(2n – 1) (2n – 5)Pn – 1Pn – 3, if n is odd

or

z

1

Also,

z

1

∃1

∃1

...(2)

...(3A) ...(3B)

0, if m ∋ n Pm Pn dx = 2 / (2n ) 1), if m % n .

{

...(4)

We consider two cases : Case I. When n is even. Integrating both sides of (3A) w.r.t. ‘x’ from –1 to 1 and using (4), 2 2 (Pn' ) dx = (2n ∃ 1) ∗

2 2 2 ) (2n ∃ 5)2 ∗ ) ... ) 32 ) 0 ) 0 ) ... 2(n ∃ 1) ) 1 2(n ∃ 3) ) 1 2 ∗1 ) 1

= 2[(2n – 1) + (2n – 5) + ... + 3]. Let m be the number of terms in A.P. on R.H.S. of (5). Then, we have 3 = (2n – 1) + (m – 1) × (–4) so that

m = n/2.

number of terms (first term + last term) 2

But

sum of A.P. =

;

(2n – 1) + (2n – 5) + ... + 3 =

z

1

Hence (5) reduces to

...(5)

∃1

(n / 2) (2n – 1 + 3) = 2

2 =2× P' n dx

1 2

n(n + 1).

n(n + 1) = n(n + 1).

1 2

Case II. When n is odd. Integrating both sides of (3B) w.r.t. ‘x’ from –1 to 1 ans using (4),

z

1

∃1

2 (2n ∃ 1) 2 ∗ P' n dx =

2 2 2 ) (2n ∃ 5)2 ∗ ) ... ) 2(n ∃ 1) ) 1 2(n ∃ 3) ) 1 2 ∗0 )1

= 2[(2n – 1) + (2n – 5) + .... + 1]. Let p be the number of terms in A.P. on R.H.S. of (6). Then, we have 1 = (2n – 1) + (p – 1) × (–4) so that As before,

(2n – 1) + (2n – 5) + .... + 1 =

Hence (6) reduces to

z

{(n ) 1) / 2} (2n – 1 + 1) = 2

1

Pn 2 dx = 2 ×

∃1

1

Thus for all values of n, we have Ex. 13. Show that, when | z | < 1 and

z

1

z

1 2

∃1

| x | 3 1,

p = (n + 1)/2. 1 2

n(n + 1).

n (n + 1) = n(n + 1). 2

P'n dx = n(n + 1).

n Pn ( x) (1 ∃ 2zx ) z 2 )∃1/ 2 dx = (2z )/(2n + 1).

[Meerut 2005]

∃1

#

Sol. We know that

...(6)

(1 – 2zx + z2)–1/2 =

(z n%0

n

Pn ( x ) .

Multiplying both sides by Pn(x), we have

Created with Print2PDF. To remove this line, buy a license at: http://www.software602.com/

9.26

Legendre Polynomials 2 –1/2

n

Pn(x). (1 – 2zx + z ) = Pn(x) [P0(x) + zP1(x) + ... + z Pn(x) + ...]. Integrating both sides w.r.t. ‘x’ between –1 to 1, we get

>

1

∃1

>

Pn ( x) (1 ∃ 2 zx ) z 2 )∃1/ 2 dx =

1

∃1

+z But

>

1

∃1

z

Ex. 14. Evaluate

>

1

>

1

∃1

∃1

(i)

2. Prove that

z

1

∃1

4. Prove that

1

∃1

z

1

>

1

P1 ( x) Pn ( x )dx ) ...

∃1

[ Pn ( x)]2 dx ) z n ) 1

>

and

>

1

1

∃1

Pn ) 1 ( x) Pn ( x ) dx ) ... 2 . 2n ) 1

[ Pn ( x)]2 dx %

∃1

z

...(1) ...(2)

1

n Pn ( x) (1 ∃ 2zx ) z 2 )∃1/ 2 dx = 2z . ∃1 2n ) 1

P32 ( x ) dx .

[Nagpur 1995, 96]

Pn2 ( x ) dx = 2/(2n + 1),

1. Evaluate

3. If un =

n

Pm ( x) Pn ( x ) dx % 0, if m ∋ n

Using (2), (1) reduces to

Sol. Since

P0 ( x ) Pn ( x ) dx ) z

z

1

so

P32 ( x ) dx = 2/(6 + 1) = 2/7.

∃1

EXERCISE 9 (B) 2

(ii)

x Pn ( x ) dx

∃1

z

1

∃1

z

(iii)

x Pn ( x) Pn ) 1( x) dx

1

∃1

x 3 P4 ( x ) dx .

Ans. (i) 0 (ii) 2(n + 1)/[(2n + 1) (2n + 3)] (iii) 0.

z

1

∃1

2

(1 ∃ x ) Pm' Pn' dx = 0, if m ∋ n.

x ∃1 Pn ( x) Pn ∃ 1( x ) dx , show that nun + (n – 1)un – 1 = 2 and hence evaluate un.

z

1

∃1

Ans. un = 2/n, if n is even; un = 0 if n is odd x Pn ( x) Pn' ( x ) dx =

2n . 2n ) 1

5. Obtain the relation : xPn ( x) % Pn)1 ( x) ∃ ( n ) 1) Pn ( x) 9.14. Rodrigue’s formula. To show that

Pn(x) =

[Guwahati 2007] n

1 d ( x 2 ∃ 1)n 2 n n ! dx n

[Punjab 2005; Purvanchal 2006; Gulbarga 2005; Ultak 2003; Nagpur 1996; Meerut 2007, 11; Garhwal 2004; Bilaspur 1998; Bhopal 2004, 10; KU Kurukshetra 2005; Agra 2010; MDU Rohtak 2005; Kanpur 2008, 11; Ranchi 2010; Lucknow 2010] Proof. By the definition of Legendre polynomial, we get [n / 2 ]

Pn(x) = where

(2 n ∃ 2 r ) ! x n ∃ 2 r . 2 r ! (n ∃ r ) ! (n ∃ 2 r ) ! 1 n = n / 2, if n is even 2 (n ∃ 1) / 2, if n is odd

( (∃1) n%0

r

{

n

Now, by binomial theorem,

...(1)

n

(x2 – 1)n =

( r %0

n

2 n∃r

Cr ( x )

...(2) r

(∃1) %

n

(

n

r 2n ∃ 2r

Cr (∃1) x

.

r %0

Created with Print2PDF. To remove this line, buy a license at: http://www.software602.com/

Legendre Polynomials

9.27

dn

1

;

n

2 n ! dx

n

2n

dn m x = 0 if m < n dx n

But

n

1

( x 2 ∃ 1)n =

( 1n!

n

Cr (∃1) r

r %0

dn dx n

x 2n ∃ 2r .

m! dn m x = x m ∃ n , if m 2 n ( m ∃ n) ! dx n

and

n d n 2n ∃ 2 r = 0, if 2n – 2r < n, i.e., r > . n x 2 dx

;

...(3) ...(4) ...(5)

n

n/2

(n ∃ 1)/ 2

r %0

r%0

r %0

Making use of (5) in (3), we see that we must replace ( by ( if n is even and by ( if n

[n / 2 ]

r %0

r %0

n is odd. i.e. we must replace ( by ( . Hence (3) reduces to 1 1 dn 2 (x ∃ 1) = n n 2 n ! dx n 2 n! [ n / 2]

=

( r %0

[n / 2]

(

n

Cr (∃1)r

r %0

dn dx

n

x2n ∃ 2r =

1 2

n

[n / 2]

( n!

n

Cr (∃1)r

r %0

(2n ∃ 2r ) ! 2n ∃ 2r ∃ n x , by (4) (2n ∃ 2r ∃ n) !

n !(∃1)r (2n ∃ 2r ) ! n ∃ 2 r . x = Pn(x), using (1). 2n n ! r ! (n ∃ r ) ! (n ∃ 2r ) ! 1

9.15. Solved examples based on Art. 9.14. Ex. 1. Using Rodrigue’s furmula, find values of P0(x), P1(x), P2(x) and P3(x). [MDU Rohtak 2004; Bangalore 1995] Pn(x) =

Sol. Rodrigue’s formula is given by

1 n

dn

2 n ! dx

n

( x 2 ∃ 1) n .

Putting n = 0 in (1),

P0(x) =

1 (x 2 ∃ 1)0 = 1. 2 0!

Putting n = 1 in (1),

P1(x) =

1 1 d 2 ( x ∃ 1) = (2x) = x. 2 21 . 1 ! dx

... (1)

0

Putting n = 2 in (1), we have P2(x) =

1

LM N

d2

OP Q

1 d + 2( x 2 ∃ 1) 1 2 x , = 1 d ( x 3 ∃ x ) % 1 (3x 2 ∃ 1). (x2 ∃ 1)2 = 1 d d (x2 ∃ 1)2 = 0 2 dx 8 dx / 8 dx dx 2 2 1 2! dx 2

2

Putting n = 3 in (1), we have P3(x) =

LM N

OP Q

1 d2 + 2 1 d2 d 2 2 3 3 3( x ∃ 1) 2 1 2 x ,0 ( x ∃ 1) = ( x ∃ 1 ) = 48 dx 2 dx 48 dx 2 / 23 1 3 ! dx3 1

d3

1 d +d , 1 d 1 d +( x 2 ∃ 1)2 ) x 1 2( x 2 ∃ 1) 1 2 x , = 1 (5 x 4 ∃ 6 x 2 ) 1) x ( x 2 ∃ 1) 2 . = − / 0 8 dx / dx 8 dx 8 dx 0 3 3 =(1/8)× (20x – 12x) = (1/2) × (5x – 3x). Ex. 2. If x > 1, show that Pn(x) < Pn + 1(x). [Garhwal 2005; Ravishankar 2000] Sol. We are to prove Pn(x) < Pn + 1(x). ...(1) Since x > 1, from Rodrigues’s formula, we find that Pn(x) > 0 for each value of n. We now use the mathematical induction to prove (1). =

Created with Print2PDF. To remove this line, buy a license at: http://www.software602.com/

9.28

Legendre Polynomials

Now x > 1 6 1 < x 6 P0(x) < P1(x) [ P0(x) = 1 and P1(x) = x] This implies that (1) is true for n = 0. Let (1) be true for n – 1. Then, we have Pn – 1 < Pn so that Pn – 1/Pn < 1. ...(2) From recurrence relation I, (2n + 1)xPn = (n + 1)Pn + 1 + nPn – 1 Pn ) 1 Pn ) 1 n Pn ∃ 1 (2n ) 1)x (2n ) 1)x n Pn ∃ 1 = ) or = ∃ Pn n ) 1 Pn Pn n )1 n )1 n ) 1 Pn

or

Pn ) 1 2n ) 1 ∃ n , using (2) and noting that x > 1 > Pn n )1 n )1 ; Pn + 1/Pn > 1, so that Pn < Pn + 1 [ Pn > 0 for each n.] This shows that (1) is true for n whenever (1) is true for n – 1. Hence (1) is true for each n by induction.

or

z z

1

Ex. 3. Prove that (i) Pn ( x ) dx = 2, if n = 0

[Guwahati 2007]

∃1

(ii)

1

Pn ( x ) dx = 0, if n 2 1.

∃1

Sol. (i) When n = 0,

Pn(x) = P0(x) = 1.

Hence

Part (ii) Using Rodrigues’ formula, we have

>

1

∃1

1 n 2 .n!

Pn ( x) dx =

∃1

1

Pn ( x ) dx =

∃1

z

1

dx = 2.

∃1

D n ( x2 ∃ 1) n dx , where Dn Ω dn/dxn

1 1 1 1 + D n ∃ 1 ( x 2 ∃ 1) n , + D n ∃ 1{( x ∃ 1) n ( x ) 1) n }, = n / 0 / 0 ∃ 1 ∃1 2 n! 2 n!

=

=

>

1

z

n

1

+ D n ∃ 1 ( x ∃ 1) n ∗ ( x ) 1) n ) 2 n! /

[ =

1

n n ∃1 n C1 D n ∃ 2 ( x ∃ 1) n D( x ) 1)n ) ... ) ( x ∃ 1) ∗ D ( x ) 1) ,0

n ∃1

n

n

n

n

n –1

By Leibnitz Theorem, D (uv) = D u . v + C1D

1 n ! ( x ∃ 1) ( x ) 1)n ) ... ) n ! ( x ) 1) ( x ∃ 1)n 2 .n! n

LM N

=0

u. Dv + ... + u . Dnv]

1 ∃1

n

m

D (ax ) b) % a

n

m! m∃n (ax ) b) (m ∃ n) !

Ex. 4. If m > n – 1 and n is a positive integer, prove that

z z

1

∃1

OP Q

m(m ∃ 1) (m ∃ 2) ... (m ∃ n ) 2) (m ) n ) 1) (m ) n ∃ 1) ... (m ∃ n ) 3) Sol. Using Rodrigues’ formula, we have 1 1 1 x m Pn ( x ) dx = n x m D n ( x 2 ∃ 1) n dx , where Dn Ω d n/dxn 0 2 n! 0 0

= =

Ξ

x m Pn ( x ) dx =

>

+ m n ∃1 2 n − x D ( x ∃ 1) n 2 n! / 1

(∃1)1 m n

2 n!

>

1

0

[

Χ

1 0

∃m

>

1

0

, x m ∃ 1 D n ∃ 1 ( x 2 ∃ 1)n dx . 0

...(1)

x m ∃ 1 D n ∃ 1 ( x 2 ∃ 1)n dx

the first term in (1) vanishes on using Leibnitz theorem as shown in Ex. 3.]

Created with Print2PDF. To remove this line, buy a license at: http://www.software602.com/

Legendre Polynomials

=

=

9.29

(∃1)1 m + m ∃ 1 n ∃ 2 2 x D ( x ∃ 1)n 2 n n ! −/

Ξ

(∃1)2 m(m ∃ 1) n

2 n!

>

1

0

Χ

1 0

– (m – 1)

= =

=

(∃1)n m(m ∃ 1) ... (m ∃ n ) 1)

1

>

n

2 n!

0

n

2 n!

n

2 n!

2

n!

>

1

0

>

1

0

>

1

0

m( m ∃ 1) ... ( m ∃ n ) 1)

n)1

OP Q

the first term is again zero as shown in Ex. 3.]

x m ∃ n (∃1) n (1 ∃ x 2 )n dx

(t1/ 2 )m ∃ n (1 ∃ t ) n

1 ( m ∃ n ∃ 1)

t2

...(2)

xm ∃ n (x2 ∃ 1)n dx , on continuing the similar steps n – 2 times more

(∃1)n m(m ∃ 1) ... (m ∃ n ) 1)

m ( m ∃ 1) ... ( m ∃ n ) 1)

x m ∃ 2 Dn ∃ 2 ( x 2 ∃ 1)n dx

0

x m ∃ 2 D n ∃ 2 ( x 2 ∃ 1)n dx

[ =

z

1

(1 ∃ t ) n dt

dt 2t

1/ 2

, taking x 2 % t so that dx %

m(m ∃ 1) ... (m ∃ n ) 1)

=

2

Ε m ∃ n ) 1Φ ≅Γ Η ≅(n ) 1) m(m ∃ 1) ... (m ∃ n ) 1) Ι 2 ϑ = n )1 m ∃ n ) 1 Ε Φ 2 n! ≅Γ ) n ) 1Η 2 Ι ϑ

FG H

n )1

z

1

0

n!

>

1

0

1 ( m ∃ n ) 1) ∃ 1

t2

dt dt % 1/ 2 2 x 2t

(1 ∃ t )(n ) 1) ∃ 1 dt

t p ∃ 1(1 ∃ t )q ∃ 1 dt % B( p, q) %

≅ ( p ) ≅( q ) ≅( p ) q )

IJ K

Ε m ∃ n ) 1Φ ≅ Η. n! m(m ∃ 1) ... (m ∃ n ) 1) ΓΙ 2 ϑ = Ε m ) n ) 3Φ 2n ) 1 n ! ≅Γ Η 2 Ι ϑ Ε m ∃ n ) 1Φ m(m ∃ 1) ... (m ∃ n ) 1) ≅ Γ Η 2 Ι ϑ = m ) n ) 1 m ) n ∃ 1 m ∃ n ) 1 m ∃ n ) 1Φ Ε 2n ) 1 1 . ... ≅Γ Η 2 2 2 2 Ι ϑ

=

[

≅(p + 1) = p≅(p)]

m(m ∃ 1) ... (m ∃ n ) 1) (m ) n ) 1) (m ) n ∃ 1) ... (m ∃ n ) 1)

Ex. 5. (i) If m < n, show that

>

Deduce that

1

∃1

z

1

∃1

x m Pn ( x ) dx = 0.

x 4 P6 ( x )dx % 0

[Ranchi 2010, Ravishankar 2010, Kanpur 2011] [Meerut 2006]

≅(1 / 2) ≅(n ) 1) 2 n ) 1(n !)2 % . ∃1 (2n ) 1) ! 2 n ≅(n ) 3 / 2) Sol. (i) Using Rodrigues’ formula, we have

(ii) Prove that

>

1

x n Pn ( x ) dx =

[Kanpur 2006, 10]

Created with Print2PDF. To remove this line, buy a license at: http://www.software602.com/

9.30

Legendre Polynomials

z

1

m

∃1

x Pn ( x ) dx

1

=

2

(∃1)2 m ( m ∃ 1)

%

n

2 ∗ n!

>

∗n!>

n

1 ∃1

x m D n ( x 2 ∃ 1) n dx , where Dn Ω dn/dxn

∃1

x m ∃ 2 D n ∃ 2 ( x 2 ∃ 1)n dx, doing upto equation (2) as in Ex. 4

(∃1)m m (m ∃ 1)....3 ∗ 2 ∗1

=

1

n

2 ∗ n!

1

>

1

x m ∃ m D n ∃ m ( x 2 ∃ 1)n dx

∃1

...(A)

(On continuing the similar steps m – 2 times more and noting that m < n) (∃1)m m !

=

2n ∗ n !

>

1

∃1

d ΨΝ d n ∃ m ∃ 1 2 ΨΡ ( x ∃ 1)n Σ dx Ο dx ΠΨ dx n ∃ m ∃ 1 ΤΨ 1

1 (∃1)m m ! n ∃ m ∃ 1 (∃1)m m ! + d n ∃ m ∃ 1 2 n, +D {( x ∃ 1)n ∗ ( x ) 1) n },0 = 0 = n − n ∃ m ∃ 1 ( x ∃ 1) . = n / ∃1 2 ∗ n ! / dx 2 ∗ n! 0 ∃1

[By using Leibnitiz theorem and simplifying as before] Deduction : Taking m = 4 and n = 6 in the above result, we get the required result because 4 < 6 satisfies condition m < n. Part (ii). Here m = n. So proceeding as above upto (A), we obtain

z

1

∃1

=

=

=

x n Pn ( x) dx =

1 2

n

12

1 2n ∃ 1

1 2

n

1

>

1

1

0

(∃1)n n ! n

2 n!

1

>

∃1

(1 ∃ x 2 ) n dx =

≅(n )1) ≅ Ζ 12 9 Ε 2n ) 3 Φ 2≅ Γ Η Ι 2 ϑ

,

xn ∃ nDn ∃ n (x2 ∃1)n dx = 1

2

n ∃1

z

=/2

0

z

as

n! = 2n ) 1 2n ∃ 1 3 1 ∗ ...... . ∗ 2 2 2 2

z

1

∃1

=/ 2

=

2

x % sin 8 ≅

cos p 8 sin q 8 d8 %

n! n

1

(∃1)n 2n

( x 2 ∃ 1)n dx =

cos2n ) 1 8 d8 , putting

0

=

(∃1)n 2n

= 2(n !) 1

(2 ∗ n) ∗ [2 ∗ (n ∃ 1)] ... (2 ∗ 2) ∗ (2 ∗ 1) 2n n ! 2n )1 (n !)2 = 2(n !) 1 . % (2n ) 1) ! (2n ) 1) ! (2n ) 1) !

∃1

dx % cos 8 d 8

(

(2n) (2n ∃ 2) ... 4 ∗ 2 (2n ) 1) (2n) (2n ∃ 1) (2n ∃ 2) ... 4 ∗ 3 ∗ 2 ∗ 1

1

(∃1)n (1 ∃ x 2 )n dx

FG p ) 1IJ ≅FG q ) 1IJ H 2 K H 2K F p ) q ) 2 IJ 2≅ G H 2 K

= 2(n !) 1

z

∃1

and

2n ) 1 (2n ) 1) (2n ∃ 1) ... 3 ∗ 1

Ex. 6. Deduce from Rodrigue’s formula

z

1

n f ( x) Pn ( x) dx = (∃n1)

2 n!

z

1

∃1

≅(n + 1) = n !)

(x 2 ∃ 1)n f (n) ( x) dx .

Sol. Using Rodrigue’s formula, we have

Created with Print2PDF. To remove this line, buy a license at: http://www.software602.com/

Legendre Polynomials

z

1

∃1

=

=

9.31

f ( x) Pn ( x ) dx =

1 2 n! n

LMn f (x) D N

(∃1)1 2n n !

1 2 n! n

n ∃1

z

1

∃1

n

2

n

n

n

f ( x) D (x ∃ 1) , where D Ω d / dx

( x 2 ∃ 1)n

s

1 ∃1



z

1

∃1

n

f ( x ) Dn ∃ 1( x 2 ∃ 1)n dx

OP Q

...(1) (On integration by parts)

z

1

f ( x ) Dn ∃ 1( x 2 ∃ 1)n dx

∃1

(the first term in (1) vanishes on using Leibnitz theorem as explained in Ex. 3)

Ln f (x) D n ! MN

1 = (∃n 1)

2

=

n∃2

2

(∃ 1) n 2 n!

=

(∃1)n 2n n !

=

(∃1)n 2n n !

z

1

∃1

z z

1

∃1 1

∃1

( x 2 ∃ 1)n

s

1 ∃1



z

1

∃1

OP Q

f ( x) Dn ∃ 2 ( x 2 ∃ 1)n dx , on integration by parts again

f ( x ) D n ∃ 2 ( x 2 ∃ 1)n dx

f (n)( x ) Dn ∃ n ( x 2 ∃ 1)n dx ,

(

the first term vanishes as shown in Ex. 3)

on continuing the similar steps n – 2 times more

( x 2 ∃ 1)n f (n)( x ) dx .

Ex.7. Using Rodrigue’s formula, show that Pn(x) satisfies

d Ν Ρ 2 d Ο(1 ∃ x ) Pn ( x) Σ ) n(n ) 1) Pn ( x ) % 0 dx Π dx Τ Sol. Rodrigue’s formula is

or or

Pn ( x ) %

1

[CDLU 2004]

dn

( x 2 ∃ 1)n

... (1) 2 n ! dx Let y = (x2 – 1)n ... (2) 2 n–1 2 2 Differentiating (2) w.r.t. ‘x’, y1 = 2nx(x – 1) so that (x – 1)y1 = 2nx (x – 1)n 2 (x – 1) y1 = 2nxy, using (2) ... (3) 2 Differentiating (3) w.r.t. ‘x’, (x – 1)y2 + 2xy1 = 2n (xy1 + y) (x2 – 1) y2 + 2(1 – n)xy1 – 2ny = 0 ... (4) Differentiating both sides of (4) w.r.t. ‘x’ n times, we have n

n

Dn{(x2 – 1)y2} + 2(1 – n) Dn(xy1) – 2nDn (y) = 0, where D n Ω d n / dx n ... (5) Using Leibnitz’ theorem, (5) yields yn + 2 (x2 – 1) + nC1 yn + 1(2x) + nC2 yn . 2 + 2(1 – n) (yn + 1 x + nC1 yn ∗1 ) – 2nyn = 0 (x2 – 1) yn + 2 + 2x yn + 1 + {n(n – 1) + 2n(1 – n) – 2n} yn = 0 (1 – x2) yn + 2 – 2x yn + 1 + n(n + 1) yn = 0

or or or or

d Ν Ε dyn Φ Ρ d 2 or (1 ∃ x 2 ) yn )1 ) n(n ) 1) yn % 0 Ο(1 ∃ x ) 1 Γ Η Σ ) n(n ) 1) yn % 0 dx Π dx Ι dx ϑ Τ n Ρ d ΝΨ dn 2 2 d Ε d 2 n ΦΨ n Ο(1 ∃ x ) ΓΓ n ( x ∃ 1) ΗΗΣ ) n(n ) 1) n ( x ∃ 1) % 0, using (2) dx ΠΨ dx Ι dx dx ϑΤΨ

Ξ

Χ

Dividing by 2nn! and using (1), we get

d d {(1 ∃ x 2 ) Pn ( x )} ) n( n ) 1) Pn ( x ) % 0 dx dx

Created with Print2PDF. To remove this line, buy a license at: http://www.software602.com/

9.32

Legendre Polynomials

Ex. 8. Using Rodrigue’s formula, prove that

Ε1) t Φ # (i) (1 ∃ t )n Pn Γ Η% & Ι 1 ∃ t ϑ k %0

Ζc9 n

2

k

tk

(Ravishankar 1993)

(ii) Pn (cosh u) 2 1

(ii) | Pn ( x ) | 2 1 , if | x | 2 1. Pn ( x ) %

Sol. (i) Rodrigues formula is given by

dn

1 n

2 n ! dx

n

(Bilaspur 1994)

( x 2 ∃ 1)n

... (1)

Let D Ω d/dx. Then (1) can be re-written as Pn ( x ) % Pn ( x ) %

or

2 n!

D n ( x 2 ∃ 1)n %

2 n!

Ξ

D n ( x ) 1)n ( x – 1)n

Χ

2 n!

& n Ck D n ∃ k ( x ) 1) n D k ( x ∃ 1) n , by Leibnitz’s rule

... (2)

k %0

D n (ax ) b) m %

From Differential calcules, we know that D n ∃k ( x ) 1)n %

Using (3),

1 n

n

1 n

1 n

... (3)

n! n! ( x ) 1)n ∃( n ∃ k ) % ( x ) 1)k [n ∃ (n ∃ k )]! k!

D k ( x ∃ 1)n %

and

m! (ax ) b)m ∃1 a n (m ∃ n)!

... (4)

n! ( x ∃ 1) n ∃ k (n ∃ k )!

... (5)

Using (4) and (5), (3) reduces to Pn ( x ) %

1

n

&

2n n ! k %0

Put

Ck

n! n! ( x ) 1)k ( x ∃ 1)n ∃k or k! (n ∃ k )!

x%

1) t 1∃ t

n

x ∃1 %

so that

# Ε x ∃1 Φ Pn ( x) % & (n Ck )2 Γ Η k %0 Ι 2 ϑ

2t 1∃ t

x )1 %

and

n∃k

2 1∃ t

Ε x )1 Φ Γ Η Ι 2 ϑ

k

... (6) ... (7)

Using (7), (6) reduces to Ε1) t Φ # n 2Ε t Φ Pn Γ Η % k&% 0( Ck ) Γ Η Ι 1∃ t ϑ Ι1∃ t ϑ

(ii) Let

n ∃k

u t % tanh 2 , 2

1 (1 ∃ t )

k

or

Ε 1) t Φ # n 2 k (1 ∃ t )n Pn Γ Η % k&%0( Ck ) t ... (8) 1 ∃ t Ι ϑ

then t 2 0

and n

Pn (cosh u ) % (1 ) t )∃ n & (n Ck )2 t k

Hence (8) reduces to

k %0

1) t % cosh u 2 1. 1∃ t

... (9)

Since the first term on R.H.S. of (9) is 1 and all the subsequent terms are positive, hence (9) yields

Pn (cosh u ) 2 1,

as required

(iii) Left as an exercise. Ex. 9. Show that all the roots of Pn(x) = 0 are real and lie between –1 and 1. (Bilaspur 1998) Sol. Let f(x) = (x2 – 1)n = (x – 1)n (x + 1)n. ...(1)

Created with Print2PDF. To remove this line, buy a license at: http://www.software602.com/

Legendre Polynomials

9.33

From (1), we notice that f(x) vanishes for x =1 and x = –1, hence by *Rolle’s theorem, f (x) must vanish at least once for some value Υ of x lying between –1 and 1. Now, from (1), we have f (x) = n(x – 1)n – 1(x + 1)n + n(x – 1)n (x + 1)n – 1. ...(2) (2) shows that f (x) vanishes at x = 1 and x = –1. But we have just proved that f (x) vanishes at x = Υ, where –1 < Υ < 1. Hence applying Rolle’s theorem to function f two times, we conclude that f !(x) must vanish at x = [ such that –1 < [ < Υ and also at x = ∴ such that Υ < ∴ < 1. Proceeding likewise we conclude that f (n)(x) = 0 must have n real roots lying between –1 and 1. Using (1), Rodrigue’s formula gives Pn(x) =

dn

1 n

2 ∗ n ! dx

n

( x 2 ∃ 1)n %

1 n

2 ∗ n!

∗ f ( n ) ( n) .

...(3)

; f (n)(x) = 0 6 Pn(x) = 0, by (3). ...(4) n Since f (x) = 0 has n real roots lying between –1 and 1, so (4) shows that Pn(x) = 0 has n real roots between –1 and 1. Remarks. The roots of Pn(x) = 0 are also known as zeros of Pn(x). Ex. 10. Prove that all the roots of Pn(x) are distinct. Sol. If possible, let the roots of Pn(x) = 0 be not all different. Then at least two roots must be equal. Let Υ be the repeated root, then from the theory of equations, we have Pn(Υ) = 0 and P n(Υ) = 0. ...(1) Since Pn(x) satisfies Legendre’s equation, (1 – x2)P!n – 2xP n + n(n + 1)Pn = 0. ...(2) Differentiating r times and using Leibnitz theorem, (2) gives dr)2 dr )1 dr (1 ∃ x 2 ) r ) 2 Pn ( x) ) r C1 1 (∃2 x ) 1 r ) 1 Pn ( x) ) r C2 1 (∃2) 1 r Pn ( x ) dx dx dx + d r )1 , dr dr ∃2 − x r ) 1 Pn ( x ) ) r C1 1 1 1 r Pn ( x ) . ) n(n ) 1) r Pn ( x) % 0 dx dx / dx 0

or (1 ∃ x 2 )

dr )2 dx

r)2

Pn ( x ) ∃ 2 x ( r C1 ) 1)

dr )1 dx

r )1

Pn ( x ) –{2 × rC2 + 2 × rC1 – n(n + 1)}

dr dx r

Pn(x) = 0

....(3) Putting r = 0 and x = Υ in (3) and using (1), we get (1 – Υ2)P!n(Υ) – 0 – 0 = 0 or P!n(Υ) = 0. ...(4) Next, putting r = 1 and x = Υ in (3) and using (1) and (4), we get (1 – Υ2)Pr! (Υ) – 0 – 0 = 0 or P! n(Υ) = 0. ...(5) Putting r = 2, 3, ...., n – 3, n – 2 in (3) and doing as above stepwise, we finally arrive at Pn(n)(Υ) = 0 But ;

LM d N dx

n

i.e. Pn(x) =

n

OP Q

Pn ( x )

= 0.

...(6)

∋ 0.

...(7)

x %Υ

, 1 ∗ 3 ∗ 5 ...(2 n ∃ 1) + n n ( n ∃ 1) n ∃ 2 x ) .... −x ∃ n! 2(2n ∃ 1) / 0

1 ∗ 3 ∗ 5 ...(2n ∃ 1) dn 1 n! Pn ( x ) = dx n n!

6

LM d MN dx

n n

Pn ( x )

OP PQ

x %Υ

*Rolle’s theorem: If f(x) vanishes for x = a and x = b, then f (x) vanishes at least once for some value of x between a and b.

Created with Print2PDF. To remove this line, buy a license at: http://www.software602.com/

9.34

Legendre Polynomials

Since (6) and (7) are contradictory results, it follows that our assumption about not distinct roots of Pn(x) is absurd. Hence all the roots of Pn(x) = 0 must be distinct.

EXERCISE 9 (C) 1. Define Legendre’s differential equation and show that y =

dn 2 (x – 1)n satisfies it. dx n

2. Using Rodrigue’s formula, prove that P n + 1 – P n – 1 = (2n + 1)Pn. 3. Show that Pn(x) =

1 dn 2 (x – 1)n is a solution of the Legendre’s equation (1 – x2)y2 2 n ! dx n n

– 2xy1 + n(n + 1)y = 0, where n is +ve integer. Hence or otherwise show that (i) xP n(x) – P n – 1(x) = nPn(x). (ii) (2n + 1)xPn(x) = (n + 1)Pn + 1(x) + nPn – 1 (x). 4. Use Rodrigue’s formula to derive the orthogonal property for Pn(x) and show that

z

1

2 Pn2 ( x ) dx = 2n ) 1 . ∃1

5. Prove that the function y =

dn 2 (x – 1)n satisfies the Legendre’s differential equation dx n

(1 – x2)y! – 2xy + n(n + 1)y = 0. Hence obtain Rodrigue’s formula for Legendre–polynomial Pn(x).

z

1

Using this formula prove that

x m Pn ( x) dx = 0 for m < n.

∃1

9.16. Legendre series for f(x) when f(x) is a polynomial. n

Theorem (a). If f(x) is a polynomial of degree n, then f(x) =

(C r %0

cr = Ζ r ) 1/ 2 9

where

>

1

∃1

r

...(i)

Pr ( x ),

f ( x) Pr ( x) dx.

...(ii)

(b) If f(x) is even (or odd), only those Cr with even (or odd) suffixes are non–zero. Proof (a). Since f(x) is a polynomial of degree n, we write f(x) = anxn + an – 1xn – 1 + .... + a1x + a0. ...(i) Again, we know that Pn(x) is a polynomial of degree n of the form Pn(x) = knxn + kn – 1xn – 1 + ... + k1x + k0. ...(1) Consider f(x) – (an/kn) Pn(x). Two cases may arise : Case (i). f(x) – (an/kn)Pn(x) = 0 so that f(x) = (an/kn) Pn(x), which proves the required result (i). Case (ii). f(x) – (an/kn) P(x) = gn – 1(x), gn – 1(x) being a polynomial of degree n – 1. Taking cn = an/kn, we may write f(x) = cnPn(x) + gn – 1(x). ...(3) Taking gn – 1(x) in place of f(x) and proceeding as above, we have gn – 1(x) = cn – 1Pn – 1(x) + gn – 2(x). ...(4) Making use of (4), (3) may be re–written as f(x) = cnP(x) + cn – 1Pn – 1(x) + gn – 2(x). ...(5) Making use of similar method for gn – 2(x) etc., we finally obtain (noting that P0(x) = 1). f(x) = cnPn(x) + cn – 1Pn – 1(x) + ... + c1P1(x) + c0P0(x) #

f(x) =

or #

Since

(c r %0

r

r %0

#

Pr ( x) =

(c s%0

(c

s

Ps ( x ) , (6) gives

r

Pr ( x) .

...(6) #

f(x) =

(c s%0

s

Ps ( x ) .

...(7)

Created with Print2PDF. To remove this line, buy a license at: http://www.software602.com/

Legendre Polynomials

9.35

Multiplying both sides of (7) by Pr(x) and then integrating w.r.t. ‘x’ from –1 to 1, we have

z

1

∃1

>

But

1

∃1

( RSTc n

f (x) Pr (x) dx =

s

s% 0

z

1

UV W

Ps ( x ) Pr ( x ) dx .

∃1

...(8)

Ν0, if r ∋ s Ps ( x ) Pr ( x ) dx % Ο Π2 /(2r ) 1), if r % s

z

1

Using (9), (8) reduces to

∃1

c

cr = r )

so that

1 2

f ( x ) Pr ( x) dx = cr 1

z

h

...(9)

1

∃1

2 2r ) 1

f ( x) Pr ( x ) dx .

...(10)

Part (b). We now prove that if f(x) is even, only those cr with even suffixes are non–zero. We know that Pr(x) is even when r is even, and odd when r is odd. Thus f(x) Pr(x) is even when r is even and odd when r is odd. But, it is known that

z

1

F(x ) dx = 0, if F(x) is odd. Hence (10) show

∃1

that cr = 0 if r is odd. Thus if r is even, only those cr with even suffixes are non–zero. Similarly, we can prove that if f(x) is odd, only those cr with odd suffixes are non–zero. 9.17. Solved examples based on Art 9.16

>

Ex. 1. If f(x) is a polynomial of degree less than l, prove that

1

∃1

f ( x) Pl ( x ) dx = 0.

Sol. Let f(x) be a polynomial of degree n such that n < l. Then we have (Do upto equation (6) n

(c

f(x) =

as explained in Art 9.16)

Pr ( x)

r

r %0

Multiplying both sides by Pl(x) and then integrating from –1 to 1, we have

>

1

∃1

( RSTc n

f ( x) Pl ( x ) dx =

r %0

r

z

1

UV W

Pr ( x ) Pl ( x ) dx .

∃1

...(A)

Since n < l, for each r = 0, 1, 2, ..., n we find that r ∋ l. But we know that

z

1

∃1

Pr (x) Pl (x)dx = 0 if

z

Hence (A) reduces to

1

f ( x ) Pl ( x ) dx = 0.

∃1

2

r ∋ l.

Ex. 2. Expand f(x) = x in a series of the form &cr Pr(x) Sol. Since x2 is a polynomial of degree two, from Legendre series, we have 2

x2 =

(c

Pr ( x) = c0P0(x) + c1P1(x) + c2P2(x),

r

r %0

c

cr = r )

where ...(3) c0 =

P0(x) = 1,

But

1 2

z

h

P1(x) = x

1

∃1

...(1)

x 2 Pr ( x ) dx .

...(2) P2(x) =

and

1 2

(3x2 – 1).

Putting r = 0, 1, 2 successively in (2) and using (3), we have 1 2

z

1

∃1

x 2 dx %

1 2

LM x OP MN 3 PQ 3

1

% ∃1

1 , c1 = 3 2 3

z

1

∃1

x 3 dx = 0, c2 =

With the above values of c0, c1 and c2, (1) gives

1 5 1 2 2

>

1

1

∃1

x 2 (3 x 2 ∃ 1) dx %

5 4

+ x5 x3 , 2 ∃ −3 1 . % . 5 3 / 0 ∃1 3

x3 = (1/3) × P0(x) + (2/3)× P2(x).

Created with Print2PDF. To remove this line, buy a license at: http://www.software602.com/

9.36

Legendre Polynomials 4

2

Ex. 3. Expand x – 3x + x in a series of form &cr Pr(x). Ans. x4 – 3x2 + x = –(4/5) × P0(x) + P1(x) – (10/7) × P2(x) + (8/35) × P4(x). 9.18. Expansion of function f(x) in a series of Legendre Polynomials. Supposing the expansion of f(x) in a series of Legendre polynomials to be possible, we write #

f(x) =

(c

Pr ( x) .

r

r %0

...(1)

#

f(x) =

But (1) may also be expressed as

(c s%0

s

...(2)

Ps ( x )

where c0, c1, c2, ..... cr, ... are constants. Multiplying both sides of (2) by Pr(x) and then integrating it w.r.t. ‘x’ from –1 to 1, we have

z

#

1

f ( x) Pr ( x ) dx =

∃1

R

( STc

s

s%0

>

But

1

∃1

Using (4), (3) reduces to

z

1

∃1

z

UV W

1

Pr ( x ) Ps ( x ) dx .

∃1

...(3)

Ν0, if r ∋ s Pr ( x) Ps ( x ) dx % Ο Π2 /(2 r ) 1), if r % s

...(4)

FH

2 6 cr = r ) 1 2r ) 1 2

f ( x) Pr ( x ) dx = cr 1

IK

z

1

f ( x ) Pr ( x ) dx .

∃1

Remark 1. Fourier–Legendre expansion of f(x). If f(x) be defined from x = –1 to x = 1, then #

f(x) =

(c r %0

r

Pr ( x) ,

c

cr = r )

where

1 2

h

z

1

∃1

f ( x ) Pr ( x ) dx .

Remark 2. Suppose that function f is continuous and has continuous derivatives in [–1, 1], then we prove that series (1) converges uniformly in [–1, 1] and the series expansion (1) is unique. #

Ex. 1. Expand f(x) in the form

(c r %0

r

Pr ( x) ,

f(x) = 0, where ∃ 1 7 x 7 0 1, where 0 7 x 7 1.

{

wheree

Sol. Given that

Ν0, if ∃ 1 7 x 7 0 f ( x) % Ο Π1, if 0 7 x 7 1

We know that

f(x) =

... (1)

#

c

cr = r )

1 2

h

(c

z

1

∃1

f ( x ) Pr ( x ) dx =

2r ) 1 2

c0 =

cr =

1 2

z

1

0

P0 ( x ) dx %

1 2

z

1

0

(1) dx %

1 , 2

z z

LM N

2r ) 1 2 Putting r = 0, 1, 2, ... successively in (3), we get

;

Pr ( x) , where

r

r %0

0 ∃1

1

0

...(2)

f ( x) Pr ( x) dx )

z

1

0

f ( x) Pr ( x) dx

Pr ( x ) dx , by (1)

c1 =

3 2

z

1

0

P1( x ) dx %

OP Q ...(3)

3 2

z

1

0

x dx %

3 , 4

Created with Print2PDF. To remove this line, buy a license at: http://www.software602.com/

Legendre Polynomials

9.37

z

z

z

z

2 1 1 3x 2 ∃ 1 7 1 7 1 5x ∃ 2 x 7 P3 ( x) dx % dx % ∃ c2 = 5 c3 = P2 ( x) dx % dx % 0 , 2 0 2 0 2 16 2 0 2 0 and so on. Using these values in (2), we get f(x) = (1/2) × P0(x) + (3/4) × P1(x) – (7/16) × P3(x) + ..... + crPr(x) + ...., where cr is given by (3)

EXERCISE 9 (D) 1. Expand f(x) in a series of Legendre polynomials, if (i) f(x) = x, 0 < x < 1; f(x) = 0, –1 < x < 0. (ii) f(x) = 0, –1 3 x < Υ; f(x) = 1, Υ < x 3 1. (iii) f(x) = 12 , 0 < x < 1; f(x) = – 12 , –1 < x < 0. (iv) f(x) = 2x + 1, 0 < x, 3 1; f(x) = 0, –1 3x < 0. (v) Given f(x) = | x | for –1 3 x 3 1.

z

#

2. If f(x) =

( r %0

1

Cr Pr ( x) , obtain Parseval’s identity 4

∃1

2

#

2

[ f (x )] dx =

Cr2 2r ) 1 r %0

(

and illustrate it by making use of f(x) = x – 3x + x. 3. Obtain the first three terms in the expansion of the following function f in terms of ∃17 x 7 0 f(x) = 0x,, if if 0 7 x 7 1.

{

Legendre’s polynomial: 4

3

2

4 (a). Express P(x) = x + 2x + 5x – x – 2 in terms of Legendre’s polynomials. (b). Prove that x4 + 3x3 – x2 + 5x – 2 = – (31/15) × P0(x) + (34/5) × P1(x) – (2/21) × P2(x) – (6/5) × P3(x) + (8/35) × P4(x). 5. Prove that, for all x, (a) x2 = (1/3) × P0(x) + (2/3) × P2(x). (b) x3 = (3/5) × P1(x) + (2/5) × P3(x). (c) x4 = (1/5) × P0(x) + (4/7) × P2(x) + (8/35) × P4(x). (d) x5 = (3/7) × P1(x) + (4/9) × P3(x) + (8/63) × P5(x). 6. Express f(x) = 4x3 + 6x2 + 7x + 2 in terms of Legendre’s polynomials. [Kanpur 2005] 4 3 2 7. Express f(x) = x + 2x – 2x – x – 3 in terms of Legendre’s polynomials.[Kanpur 2011]

ANSWERS

FH

1 1. (i) f(x) = (1/4) + (1/2) × P1(x) + (5/16) × P2(x) + ... + CrPr(x) + ... where Cr = r ) 2

(ii) f(x) =

1 2

(1 – Υ) –

#

([ P

r ) 1(Υ) ∃ Pr ∃ 1(Υ)] Pr ( x )

r %0

(iii) Cr = 0 if r is even, and Cr = (∃1)r ∃ 1/ 2

IK

z

1

0

x Pr ( x) dx .

.

cr ) h (r ∃ 1) ! 1 `2

r

2 {(r ) 1) / 2}!{(r ∃ 1) / 2}!

, if r is odd

(iv) f(x) = P0(x) + (7/4) × P1(x) + (5/8) × P2(x) – (7/16)P2(x) + ... #

(v) f(x) =

n )1

( (∃12) n%0

2n

(4n ) 1) (2n ∃ 2) ! P2n ( x ) . (n ) 1) ! (n ∃ 1) !

3. f(x) = – (1/4) × P0(x) + (1/6) × P1(x) + (1/16) × P2(x). 4 (a). P(x) = (8/35) × P4(x) + (4/5) × P3 (x) + (82/21) × P2(x) + (1/5) × P1(x) – (2/15) × P0(x). 6. f(x) = (8/5) × P3(x) + (32/5) × P2(x) + 7 P1(x) + 4P0(x). 9.19. Even and odd fuctions. An important observation. (i) Even function. f is called an even function of x if f(–x) = f(x). Suppose an even function f can be sum of a series of functions f1, f2, ..., fn, .... such that f(x) = f1(x) + f2(x) + ... + fn(x) + ..... ....(1)

Created with Print2PDF. To remove this line, buy a license at: http://www.software602.com/

9.38

Legendre Polynomials

Then all functions f1, f2, ..., fn must be even functions of x because if any of the above function is not even, then f(–x) ∋ f(x) and so f ceases to be an even function. (ii) Odd function. f is called an odd functions of x if f(–x) = –f(x). Suppose an odd function f can be sum of a series as (1). Then all the functions f1, f2, ..., fn must be odd functions of x because if any of the above function is not odd, then f(–x) ∋ –f(x) and so f ceases to be an odd function. 9.20. Expansion of xn in Legendre’s polynomials. (Bilaspur 1994, 96) Let xn = Cn Pn(x) + Cn – 2 Pn – 2 (x) + Cn – 4 Pn – 4(x)+ ... + Cr Pr(x) + ..., ...(1) where Pn(x), Pn – 2(x), ... are all even or odd functions of x according as xn is an even or odd function (Refer Art 9.19 for more details). Again Pn – 1(x), Pn – 3(x), ... cannot occur in the proposed series (1) because they are odd or even functions of x according as xn is even or odd function x respectively. Since Pn(x) contains terms of degree n and lower, hence the expansion (1) cannot contain any P with suffix higher than n. Multiplying both sides of (1) by Pm(x) and integrating between the limits –1 and 1, we have

z

1

x n Pm ( x ) dx % Cm

∃1

z

1

or

∃1

2

(2m ) 1) 2m ) 1 m !

z

1

∃1

1

∃1

[Pm ( x )]2 dx,

x n Pm ( x ) dx = Cm ∗

(2) 6 Cm = (2 m ) 1)

=

z

z

1

∃1

xn

LM N

2 2m ) 1

Cm =

or

LM 1 d MN 2 m ! dx

m

m

m

OP Q

m (2m ) 1) xn d m (x2 ∃ 1)m dx = m ) 1 dx 2 m!

z

z

Ν0, if, m ∋ n Pn ( x ) Pm ( x ) dx = Ο ∃1 Π 2 /(2m ) 1), if m % n 1

as

RS T

UV W

(2m ) 1) n(n ∃ 1) ... (n ∃ m ) 1) 2m ) 1 m !

z

(2m ) 1) 2

z

1

∃1

x n Pm ( x) dx

...(2)

OP PQ

( x 2 ∃ 1) m dx , by Rodrigue’s formula

LMRx MNST

n

d m ∃ 1 ( x2 ∃ 1)m dx m ∃ 1

UV W

1



∃1

z z 1

∃1

m ∃1 n xn ∃ 1 d m ∃ 1 (x2 ∃ 1)m dx dx

OP PQ

(Integrating by parts taking xn as first function)

RS T

m ∃1

UV W

m ∃1 2 (2m ) 1) n(n ∃ 1) x n ∃ 1 d m ∃ 1 ( x 2 ∃ 1)m dx xn ∃ 1 d m ∃ 1 (x2 ∃ 1)m dx = (∃1) m ) 1 ∃1 2 m! dx ∃1 dx n–1 [Again Integrating by parts taking x as first function and simplifying as before]

1)n = (∃1) (2mm) ) 2 1 m! = (∃1)m

1

1

1

x n ∃ m ( x 2 ∃ 1)m dx

∃1

[Repeating the above process of integration by parts m – 2 times more] = (∃1)m ;

(2m ) 1) n(n ∃ 1) ... (n ∃ m ) 1) (∃1)m 2m ) 1 m !

Cm =

z

1

x n ∃ m (1 ∃ x 2 )m dx

∃1

(2m ) 1) n(n ∃ 1) (n ∃ 2) ... (n ∃ m ) 1) 2m ) 1 m !

z

1

x n ∃ m (1 ∃ x 2 )m dx .

∃1

...(3)

In our discussion m can be one of integers n, n – 2, n – 4, ... only and hence (n – m) can be one of the integers 0, 2, 4, 6, ... etc only. Hence xn – m (1 – x2)m is an even function of x. ;

z

1

z

1

x n ∃ m (1 ∃ x 2 )m dx = 2 x n ∃ m (1 ∃ x 2 )m dx .

∃1

Using (4), (3) reduces to

0

...(4)

Created with Print2PDF. To remove this line, buy a license at: http://www.software602.com/

Legendre Polynomials

Cm =

(2m ) 1) 2

=

=

=

=

=

m )1

n! .2 ( n ∃m) ! m! ∗

(2m ) 1) 2

9.39

m )1

n! ∗ m ! ( n ∃m) ! ∗

>

0

2

m )1

x n ∃ m (1 ∃ x 2 ) m dx =

1

>t

( n ∃ m ∃ 1) / 2

0

(2m ) 1) n! ∗ ∗ m )1 2 m ! ( n ∃m) ! (2 m ) 1)

1

1

>t

2

m )1

n! ( n ∃m) ! m! ∗

(1 ∃ t ) m dt

( n ∃ m ) 1) / 2 ∃ 1

0

(2m ) 1)

>

1

0

x n ∃ m ∃ 1 (1 ∃ x 2 ) m ∗ 2 xdx

[Putting x2 = t and 2xdx = dt]

(1 ∃ t ) ( m ) 1) ∃ 1 ∗ dt

n! ≅[( n ∃ m ) 1) / 2] ≅( m) ∗ m ! (n ∃m) ! ≅[(n ∃ m ) 1) / 2 ) m ] ∗

LM N

B( p, q ) %

z

1

0

t

p∃1

(1 ∃ t )

q ∃1

dt %

≅( p) ≅( q) ≅( p ) q)

≅{(n ∃ m ) 1) / 2} (2m ) 1) n ! 1 n ∃ m ) 1 Ε n ∃ m ) 1Φ 2 m ) 1 (n ∃ m) ! n ) m ) 1 n ) m ∃ 1 n ) m ∃ 3 ∗ ∗ .... ≅Γ Η 2 2 2 2 2 Ι ϑ

(2m )1) n(n ∃ 1) (n ∃ 2) ... (n ∃ m ) 2) (n ∃ m ) 1) (n ∃ m) ! ∗ . (n ∃ m) ! (n ) m ) 1) (n ) m ∃ 1) .... (n ∃ m ) 3) (n ∃ m ) 1)

n(n ∃ 1) (n ∃ 2) ... (n ∃ m ) 2) (2m ) 1). (n ) m ) 1) (n ) m ∃ 1) .... (n ∃ m ) 3) Putting m = n, (n – 2), (n – 4), ... in (5), we obtain

Cm =

Thus,

Cn =

=

OP Q

...(5)

n(n ∃ 1) (n ∃ 2) ... 3 ∗ 2 n! (2n ) 1) % (2n ) 1) , (2n ) 1) (2n ∃ 1) . . 5 ∗ 3 3 ∗ 5 ∗ (2n ∃ 1) (2n ) 1)

Cn – 2 =

n(n ∃ 1) ... 5 ∗ 4 n! (2n ) 1) (2n ∃ 3) % (2n ∃ 3) , (2n ∃ 1) (2n ∃ 3) . . 7 ∗ 5 3 ∗ 5 ∗ 7 ∗ (2n ∃ 1) (2n ) 1) 2

Cn – 4 =

n(n ∃ 1) ... 7 ∗ 6 6 ∗ 7 . .. (n ∃ 1)n (2n ∃ 7) % (2n ∃ 7) (2n ∃ 3) (2n ∃ 5) . . 9 ∗ 7 7 ∗ 9 ... (2n ∃ 5) (2n ∃ 3)

n! (2n ) 1) (2 n ∃ 1) 2 ∗ 3 ∗ 4 ∗ 5 ∗ 6 ∗ 7 ... ( n ) 1)n (2 n ) 1) (2 n ∃ 1) ∗ (2 n ∃ 7) = . (2n ∃ 7) 3 ∗ 5 ∗ 7 ... (2 n ∃ 1) (2 n ) 1) 2.4 3 ∗ 5 ∗ 7 ... (2n ) 1) 2.4

and so on. Putting these values in (1), we have xn =

n! (2n ) 1) (2n ) 1)(2n ∃ 1) + (2n ) 1) Pn ( x ) ) (2n ∃ 3) ∗ Pn ∃ 2 ( x) ) (2n ∃ 7) Pn ∃ 4 ( x) − 3 ∗ 5 ... (2n ) 1) / 2 2∗4 ) ... )

and

xn %

, 1 P0 ( x ) . , if n is even ( n ) 1) 0

... 6(A)

+ Ζ 2n ) 19 P ( x) n! − (2n ) 1) Pn ( x) ) (2n ∃ 3) ∗ n∃ 2 3 ∗ 5...(2n ) 1) / 2 ) (2 n ∃ 7)

(2 n ) 1) (2n ∃ 1) 3 P ( x) , Pn ∃ 4 ( x) ) ... ) 1 . , if n is odd 2∗4 ( n ) 2) 0

... 6 (B)

Created with Print2PDF. To remove this line, buy a license at: http://www.software602.com/

9.40

Legendre Polynomials

To compute C0 and C1 it is convenient to use (2) as shown below. From (2), we have 1 C0 = 2

z

1

∃1

x n P0 ( x) dx %

1 2

z

1

∃1

x n dx =

1 ∗2 2

>

1

x

0

n

L n OP dx = M N n ) 1Q

n )1 1 0

%

1 . n )1

[using the facts that P0 (x) = 1 and n is even for C0] C1 =

and

=

3 2

>

1

x n P1 ( x) dx %

∃1

3 ∗2 2

>

1

2 3

>

1

∃1

x n ∗ x dx , as P1(x) = x

x n ) 1 dx , as n is odd so (n + 1) is even for C1

0

L x OP = 3M Nn ) 2Q

n)2 1 0

%

3 . n)2

Remark. The result of this article can also put in compact form xn =

n ! [n / 2] (2n ∃ 4k ) 1) Pn ∃ 2k ( x ) , ( k ! (3 / 2)n ∃ k 2n k %0

where

{

if n is even [n/2] = (nn∃/ 12), / 2, if n is odd.

and the symbol (Υ)n is defined as below (Υ)0 = 1, (Υ)n = Υ(Υ + 1) (Υ + 2) ... (Υ + n – 1). Corollary. Let f(x) = a0 + a1x + a2x2 + .... + anxn + ....., where a0, a1, ..., an, ... are constants. Then f(x) can be expanded in Legendre’s polynomials in the form #

f(x) = ( Cn Pn ( x), where n% 0

Cn =

, (n ) 1) (n ) 2) (n ) 3) (n ) 4) + n! ( n ) 1) ( n ) 2) an ) 4 ) .... an ) 2 ) − an ) 2 ∗ 4 ∗ (2n ) 3) (2n ) 5) 3 ∗ 5 ...(2 n ) 1) / 2(2n ) 3) 0

f(x) = a0 + a1x + a2x2 + ... + anxn + an + 1xn + 1 + ...

Proof. Given

#

f(x) = ( Cn Pn ( x).

Assume that

n% 0

...(7) ...(8)

With help of formula (4), we replace every power of x in (7) by its expansion in terms of Legendre’s polynomials and then we collect the terms involving Pn(x). Clearly when xn, xn + 2, xn + 4, ... are expanded in terms of Legendre’s polynomials, each one of them involves a term containing Pn(x). Again Pn(x) will not be involved in any expansion containing power of x less than n. Thus, we see that only anxn, an + 2xn + 2, an + 4xn + 4, ... etc in (7) will contain Pn(x). Now, using (6), we have anxn = an an

+ 2x

n+2

n! 3 ∗ 5 ...(2n ) 1)

= an ) 2

an + 4xn + 4 = an ) 4

4(2n ) 1) Pn ( x) ) ....... 5

(n ) 2) ! + (2n ) 5) , (2n ) 5) Pn ) 2 ( x ) ) (2n ) 1) Pn ( x) ) .... 3 ∗ 5 ...(2n ) 5) −/ 2 0 (n ) 4) !

+(2n ) 9) Pn ) 4 ( x) 3 ∗ 5 ...(2n ) 9) / ) (2n ) 5)

, (2n ) 9) (2n ) 7) (2n ) 9) Pn ( x) ) ..... Pn ) 2 ( x ) ) (2 n ) 1) . 2 2∗4 0

Created with Print2PDF. To remove this line, buy a license at: http://www.software602.com/

Legendre Polynomials

9.41

Putting the above values in (7) and equating the coefficients of Pn(x) from (7) and (8), Cn =

n! (n ) 2) ! + (2n ) 1) (2n ) 5) , (2n ) 1)an ) . an ) 2 3 ∗ 5 ...(2n ) 1) 3 ∗ 5 .. (2n ) 5) −/ 2 0 )

( n ) 4) ! + (2n ) 9) (2n ) 7) , −(2n ) 1) ) . an ) 4 ) ... 3 ∗ 5 ...(2n ) 9) / 2.4 0

+ , n! ( n ) 1) ( n ) 2) ( n ) 1) ( n ) 2) ( n ) 3) ( n ) 4) an ) 2 ) an ) 4 ) .... − an ) 3 ∗ 5 ...(2 n ∃ 1) / 2(2n ) 3) 2 ∗ 4 ∗ (2 n ) 3) (2n ) 5) 0 9.21. Solved examples based on Art 9.20 Ex. 1. Prove that for all x (a) x4 = (8/35) × P4(x) + (4/7) × P2(x) + (1/5) × P0(x). (b) x5 = (8/63) × P5(x) + (4/9) × P3(x) + (3/7) × P1(x).

or

Cn =

Sol. We have,

xn =

n! (2n ) 1) + (2n ) 1) Pn ( x ) ) (2n ∃ 3) Pn ∃ 2 ( x) − 3 ∗ 5 ... (2n ) 1) / 2 ) (2n ∃ 7)

, (2n ) 1) (2n ∃ 1) Pn ∃ 4 ( x ) ) .... ...(1) 2∗4 0

(a) Putting n = 4 in (1), we have x4 =

4! 9 9∗7 4 1 + , 8 9 P4 ( x) ) 5 1 P2 ( x) ) 1 1 P0 ( x) . % P4 ( x ) ) P2 ( x ) ) P0 ( x) − 3∗ 5∗7 ∗9 / 2 2∗4 7 5 0 35

(b) Putting n = 5 in (1), we have + , 8 5! 11 11 ∗ 9 4 3 P1 ( x) . % P5 ( x ) ) P3 ( x ) ) P1 ( x) −11 P5 ) 7 1 P3 ( x ) ) 3 1 3 ∗ 5 ∗ 7 ∗ 9 ∗ 11 / 2 2∗4 9 7 0 63 Ex. 2. Prove (a) x2 = (1/3) × P0(x) + (2/3) × P1(x). (b) x3 = (3/5) × P1(x) + (2/5) ×

x5 = P3(x).

Ex. 3. Express f(x) = x4 + 3x2 – x2 + 5x – 2 in terms of Legendre’s polynomials. Sol. As in Ex. 1 and 2, prove yourself that x4 = (8/35)P4(x) + (4/7)P2(x) + (1/5)P0(x), x2 = (1/3)P0(x) + (2/3)P1(x),

x = P1(x)

x3 = (3/5)P1(x) + (2/5)P3(x), and

1 = P0(x).

; f(x) = (8/35)P4(x) + (4/7)P2(x) + (1/5)P0(x) + 3[(3/5)P1(x) + (2/5)P3(x)] – [(1/3)P0(x) + (2/3)P1(x)] + 5P1(x) – 2P0(x), on putting values of x4, x3, x2 , x and 1 = (8/35)P4(x) + (6/5)P3(x) + [(4/7) – (2/3)]P2(x) + (9/5 + 5)P1(x) + [(1/5) – (1/3) – 2]P0(x) = (8/35)P4(x) + (6/5)P3(x) – (2/21)P2(x) + (34/5)P1(x) – (32/15)P0(x).

EXERCISE Express the following polynomials in terms of Legendre polynomials : 1. x3 + 2x2 – x – 3 (KU Kurukshetra 2004) 2. 4x3 – 2x2 – 3x + 8 (KU Kurukshtra 2005) 4 3 2 3. x + 3x – x + 5x – 2 4. x4 + 2x3 – 5x2 – x – 2 5. 1 + x – x2 (Gulbarga 2005, Purvanchal 2004) 3 6. 5x + x [Agra 2007]

Created with Print2PDF. To remove this line, buy a license at: http://www.software602.com/

9.42

Legendre Polynomials

ANSWERS 1. 2. 3. 4. 5.

(2/5) × P3(x) + (4/3) × P2(x) – (2/5) × P1 (x) – (7/3) × P0(x) (8/5) × P3(x) – (4/3) × P2(x) – (3/5) × P1(x) + (22/3) × P0(x) (8/35) × P4(x) + (6/5) × P3(x) – (2/21) × P2(x) + (34/5) × P1(x) – (32/25) × P0(x) (8/35) × P4(x) + (4/5) × P3(x) + (82/21) × P2(x) + (1/5) × P1(x) – (2/15) × P0(x) (2/3) × P0(x) + (1/3) × P0(x) 6. 2P3(x) + 4 P1 (x)

MISCELLANEOUS EXAMPLES ON CHAPTER 9 1. Verify that the Legendre polynomise P4(x) = (35x3 – 30x2 + 3)/8 satisfies the Legendre equation when the parameter n is equal to 4. [Sol. The Legendre equation (1 ∃ x2 ) y ∃ 2 xy ) n(n ) 1) y % 0 for n = 4 becomes

(1 ∃ x 2 ) y ∃ 2 xy ) 20 y % 0 4

... (1)

2

y = P4(x) = (35x – 30x + 3)/8

Let

y % (35 x3 ∃ 15 x) / 2

From (2),

... (2)

y % (105 x2 ∃ 15) / 2

and

Substituting the above value of y , y and y in (1), we get 2

(1 – x ) × (1/2) × (105x2 – 15) – (2x) × (1/2) × (35x3 – 15x) + 20 × (1/8) × (35x4 – 15x2 + 3) = 0 or (–105/2 – 35 + 175/2)x4 + (105/2 + 15/2 + 15 – 75) x2 + (–15/2 + 15/2) = 0 or 0 = 0, which is true. Hence P4(x) is a solution of (1).] p

2. Prove that Pn (cos 8) % &

r %0

n

Cr cos(n ∃ 2 r )8, where p = n/2 or (n – 1)/2 according as n is

even or odd. Deduc that | Pn (cos 8) | 3 1 Ν0, if m 7 n Ψ m ! ≅(m / 2 ∃ n / 2 ) 1/ 2) Ψ m x Pn ( x)dx % Ο n , if m ∃ n (2 0) is even 3. Prove that ∃1 Ψ 2 (m ∃ n)!≅(m / 2 ) n / 2 ) 3 / 2) Ψ0, if m ∃ n (] 0) is odd Π

>

1

dx , show that (n + 1) Un + 1 + n Un = 2. Hence evaluate Un. x 5. If R denotes the operator

4. If U n %

>

1

∃1

Pn ( x) Pn ∃1 ( x)

d Ν 2 d Ρ Ο(1 ∃ x ) Σ , then show that dx Π dx Τ

>

1

∃1

Pn ( x ) R Ξ f ( x )Χ dx % ∃n(n ) 1)

provided that f(x) and f ( x) are finite at x % Α 1. Deduce that

6. If n is a positive integer, show that

>

1

∃1

>

1

∃1

>

1

∃1

Pn ( x ) f ( x) dx

log(1 ∃ x) Pn ( x) dx % ∃

Pn ( x )(1 ∃ 2 xz ) z 2 )∃1/ 2 dx %

2 . n(n ) 1)

2zn 2n ) 1

and hence making use of Rodrigue’s formula, deduce that

>

1

∃1

(1 ∃ x 2 )n (1 ∃ 2 xz ) z 2 )∃ n∃1/ 2 dx %

22n)1 (n!)2 (2n ) 1)!

Created with Print2PDF. To remove this line, buy a license at: http://www.software602.com/

Legendre Polynomials

9.43

>

From this result, deduce

1

Pn ( x)dx

∃1 (1 ∃

1/ 2

x)

%

2 2 2n ) 1

7. Using Rodrigue’s formula show that (i) Pn(–x) = (–1)n Pn(x) (ii) Pn(1) = 1

(Bangalore 2005)

OBJECTIVE PROBLEMS IN CHAPTER 9 Ex 1. The value of (1/2nn!) × {d n(x2 – 1)n/dxn} is (a) 0 (b) 1 (c) Pn(x) (d) None of these. Sol. Ans. (c) Refer Art. 9.14. [Agra 2005, 06] 10

Ex. 2. Let Pn(x) be the Legendre polynomial of degree n 2 0. If 1 ) x10 % & Cn Pn ( x ), then n %0

C5 equals : (a) 0 (b) 2/11 (c) 1 (d) 11/2 [GATE 2004] Sol. Ans. (a) Refer theorem 9.16. Here f(x) = 1 + x10 is a polynomial of degree 10, which is even. Hence, only those Cn with even suffuses are non-zero. Ex. 3. Let y % ⊥( x) be a bounded solution for the equation (1 ∃ x2 ) y ∃ 2 xy ) 30 y % 0. Then

(a)

>

1

(c)

>

1

∃1

∃1

x3 ⊥( x )dx ∋ 0

(b)

>

1

x3 ⊥ ( x) dx % 0

(d)

>

1

∃1

∃1

(1 ) x3 ) x 4 ) ⊥ ( x ) ∋ 0 x 2 n ⊥ ( x )dx % 0 for all n _ N

[GATE 2003]

(1 ∃ x 2 ) y ∃ 2 xy ) 5(5 ) 1) y % 0

Sol. Ans. (d) Re-writing given equation,

... (1)

Comparing (1) with (1 ∃ x2 ) y ∃ 2 xy ) n(n ) 1) y % 0, we find that (1) is Legendre equation with n = 5. Since ⊥( x) is a bounded solution of (1), we have ⊥( x) % P5 ( x) % (63x 2 ∃ 70 x3 ) 15x) / 8, by Art. 9.2. Since x2n ⊥( x) is an odd function, conclusion (d) is true. Ex. 4. Let Pn(x) denote the Legendre polynmial of degree n. If Ν x, ∃ 1 3 x 3 0 f ( x) % Ο , Π0, 0 3 x 3 1

f(x) = a0P0(x) + a1P1(x) + a2P2(x) + ..., then

and

(a) a0 = –1/4, a1 = –1/2 (c) a0 = 1/2, a1 = –1/4

(b) a0 = –1/4, a1 = 1/2 (d) a0 = –1/2, a1 = –1/4

(GATE 2005)

#

Sol. Ans. (b). Here f ( x ) % & ar Pr . Proceed exactly as in Ex. 1 of Art. 9.18 to get a0, a1. r %0

PART II : ASSOCIATED LEGENDRE FUNCTIONS. 9.22. Associated Legendre Functions Theorem. If z is a solution of Legendre equation then to show that (1 – x2)m/2

(1 – x2)y! – 2xy + n(n + 1)y = 0

...(1)

m

d z is a solution of the equation dx m

RS T

UV W

2 (1 – x2)y! – 2xy + n(n ) 1) ∃ m 2 y % 0 . 1∃ x

...(2)

Created with Print2PDF. To remove this line, buy a license at: http://www.software602.com/

9.44

Legendre Polynomials

d 2z dz ∃ 2x ) n(n ) 1)z = 0. ...(3) dx dx 2 Differentiation (3) m times with help of Leibnitz’ theorem, we have (1 ∃ x 2 )

Proof. Since z is a solution of (1), we get

LM N

OP Q

LM N

OP Q

d m d 2 z . (1 ∃ x 2 ) ) d m dz (∃2 x ) ) n(n ) 1) d m z = 0 dx m dx 2 dx m dx dx m or or

m )1 m m d m ) 2z (1 ∃ x 2 ) ) mC d m ) 1z (∃2 x) ) mC d m z (∃2) ) d z (∃2x) ) mC d z (∃2) ) n(n ) 1) d z = 0 1 1 2 m )1 m m)2 m )1 m dx dx dxm dx dx dx

(1 ∃ x 2 )

d m ) 2z d m ) 1z d mz =0 m ) 2 ∃ 2 x(m ) 1) m ) 1 + {n(n + 1) – m(m – 1) – 2m} dx dx dx m

d m ) 2z d m ) 1z dmz =0 ...(4) m ) 2 ∃ 2 x(m ) 1) m ) 1 ) {n(n ) 1) ∃ m(m ) 1)} dx dx dx m Let u = d mz/dxm. ...(5) Then (4) reduces to (1 – x2)u! – 2x(m + 1)u! + [n(n + 1) – m(m + 1)}u = 0. ...(6) Let v = (1 – x2)m/2u ...(7) 2 –m/2 so that u = (1 – x ) v. ...(8) From (8), u = (1 – x2)–m/2 v + (–m/2) × (1 – x2)–(m/2) – 1(–2x) v. or u = (1– x2)–m/2 v + mvx(1 – x2)–(m/2) – 1. From (9), u! = (1 – x2)–m/2 v! + (–m/2) × (1 – x2)(m/2) – 1 × (–2x) × v + mvx{–(m/2) – 1} (1 – x2) –(m/2) – 1(–2x) + m(1 – x2)–(m/2) – 1(xv + v). ...(10) Substituting the values of u, u and u! given by (8), (9) and (10) into (6), we have (1 – x2)–(m/2) + 1v! + mx(1 – x2)–m/2 v + mx2v(m + 2)(1 – x2)–(m/2) – 1 + mx(1 – x2)–m/2 v + mv(1 – x2)–m/2 – 2(m + 1)x (1 – x2)–m/2v –2m(m + 1) x2v(1 – x2)–m/2 – 1 + {n(n + 1) – m2 + m} (1 – x2)–m/2v = 0. Dividing throughout by (1 – x2)–(m/2), the above equation becomes

or

(1 ∃ x 2 )

2m(m ) 1) 2 m(m ) 2) 2 , + (1 ∃ x 2 )v ∃ 2 xv − n(n ) 1) ∃ m2 ∃ m ∃ x )m) x . v % 0. 1 ∃ x2 1 ∃ x2 / 0

(1 ∃ x2 )v ∃ 2 xv ) {n(n ) 1) ∃ m2 /(1 ∃ x2 )} v % 0

or

Thus, using (11), (7) and (5), we get

... (11)

v = (1 – x2)m/2 u = (1 – x2)m/2

d mz dx m

is a solution of (11) and hence it is solution of (2). Remark 1. Equation (2) is called the associated Legendre equation. Since Pn(x) and Qn(x) are solutions of (3), we conclude that dm dm and Qnm(x) = (1 – x2)m/2 m Qn(x) m Pn(x) dx dx are solutions of (2). Pnm(x) and Qnm(x) are called the associated Legendre’s functions of degree n and order m of the first and second kind respectively. Since these are independent solutions of (2), the general solutions of (2) is y = APnm(x) + BQnm(x), where A and B are arbitrary constants. Note that if m > n, Pnm(x) = 0. The functions Qnm(x) are unbounded for x = ±1.

Pnm(x) = (1 – x2)m/2

Remark 2. For m 2 0, we define

Pnm ( x ) % (1 ∃ x 2 )m / 2

dm dx m

Pn ( x ).

Created with Print2PDF. To remove this line, buy a license at: http://www.software602.com/

Legendre Polynomials

9.45

Pnm ( x ) % (1 ∃ x 2 )m / 2

Using Rodeigue’s formula this gives

1

d m)n

2n n! dx m ) n

( x 2 ∃ 1)n ,.

where R.H.S.is well defined for negative values of m such that m ) n 2 0 i.e., m 2 ∃n.

Thus, we define

Ν 2 m/ 2 Ψ(1 ∃ x ) Ψ Pnm ( x ) % Ο Ψ 2 m/ 2 ΨΠ(1 ∃ x )

dm

Pn ( x ), for m 2 0 dx m 1 d m)n 2 ( x ∃ 1) n , for m 2 n. 2 n n ! dx m ) n

Corollary. To find the general solutions of the following equations:

) {n(n ) 1) ∃ m2 / sin 2 8} .

...(12)

d d sin 8 ) n(n ) 1) sin 8 = 0 d8 d8 ! + cot 8. + n(n + 1) = 0. dα Proof. (a) Put cos 8 = α. Then, = d %d % ∃ sin 8 . d d8 dα d8 dα

...(13A)

) cot 8 .

(a)

FH

(b) or

IK

d Ω ∃ sin 8 . d d8 dα

; Now using (14),

FG H

FH IK

% ∃ d sin 8 d != d d d8 d8 d8 dα

FG IJ H K

...(13B) ...(14) ...(15)

IJ = ∃ cos 8 d ∃ sin 8 d FG d IJ K dα d8 H dα K

= ∃ cos 8 d ∃ sin 8 1 Ζ∃sin 8) d d , by (15) dα dα dα 2 2 = ∃ cos 8 d ) sin 2 8 d 2 = ∃ α d ) (1 ∃ α 2 ) d 2 , as sin28 = 1 – cos28 = 1 – α2 dα d8 dα dα Using these values of and ! in (12), we have cos 8 d d2 d m2 ∃α ) (1 ∃ α 2 ) 2 ) ∃ sin 8 ) n(n ) 1) ∃ =0 dα sin 8 dα dα 1 ∃ cos2 8

FG H

or

(1 ∃ α 2 )

RS T

IJ RS K T

d2 d m2 2 ∃ 2α dα ) n(n ) 1) ∃ dα 1 ∃ α2

UV W

UV W

= 0, as α % cos 8

...(16) which is same as (2). Hence the general solution of (16) is = APnm(α) + BQnm(α) or = APnm(cos 8) + BQnm(cos 8), as µ = cos 8. ...(17) Part (a). Putting cos 8 = α and doing as before, (13A) or (13B) gives 2

(1 ∃ α 2 ) d 2 ∃ 2α d ) n(n ) 1) dα dα

= 0,

...(18)

which is Legendre equation and so solution of (13A) or (13B) is = APn(α) + BQn(α) or = APn(cos 8) + BQn(cos 8). 9.23. Properties of the Associated Legendre Functions. (i) Pn0 ( x) % Pn ( x) Proof. We have

(ii) Pnm ( x) % 0 if m > n. Pnm ( x ) % (1 ∃ x 2 )m / 2

dm dx m

Pn ( x ).

... (1)

Created with Print2PDF. To remove this line, buy a license at: http://www.software602.com/

9.46

Legendre Polynomials

Pn0 ( x) % Pn ( x).

(i) Putting m = 0 in (1),

(ii) Since Pn(x) a polynomial of degree n, so when m > n, (1) gives Pnm ( x) % (1 ∃ x 2 ) m / 2 1 0 % 0. 9.24. Orthogonality relation for Pnm (x). To show that

>

1

∃1

2(n ) m)! Θ nl , (2n ) 1) (n ∃ m)!

Pnm ( x ) Pl m ( x) dx %

Ν0, if n ∋ l Θ nl % Ο Π1, if n % l

where

Proof. Case I. Let l ∋ n. Since Pnm ( x) satisfy the corresponding associated equation d2y

(1 ∃ x 2 )

dx 2

∃ 2x

Ν d Ν m2 ΨΡ dy ΝΨ m2 ΡΨ 2 dy Ρ Ψ (1 ∃ x ) ) n ( n ) 1) ∃ i.e., ) Ο n(n ) 1) ∃ y % 0. Ο Σ Ο Σ y % 0. Σ dx Π dx Τ ΨΠ dx ΠΨ 1 ∃ x 2 ΨΤ 1 ∃ x 2 ΤΨ

;

d Ν m2 ΡΨ m Ρ ΝΨ 2 d m Ο(1 ∃ x ) Pn ( x) Σ ) Ο n(n ) 1) ∃ Σ Pn ( x ) % 0. dx Π dx 1 ∃ x 2 ΤΨ Τ ΠΨ

... (1)

d Ν m 2 ΡΨ m Ρ ΝΨ 2 d m Ο(1 ∃ x ) Pl ( x) Σ ) Οl (l ) 1) ∃ Σ Pl ( x ) % 0. dx Π dx 1 ∃ x 2 ΤΨ Τ ΠΨ

... (2)

Similarly, for Pl m ( x),

Multiplying (1) by Pl m ( x) and (2) by Pnm ( x) and subtracting the resulting equations gives

Pl m

d Ν d Ν Ρ Ρ 2 d m m 2 d m m m Ο(1 ∃ x ) Pn ( x) Σ ∃ Pn ( x) Ο(1 ∃ x ) Pl ( x ) Σ )[n(n ) 1) ∃ l (l ) 1)]Pn ( x) Pl ( x) % 0. dx Π dx dx dx Τ Π Τ Integrating between limits –1 to 1, we get

1

>

∃1

Plm (x)

dΝ Ρ 2 d m Ο(1∃ x ) Pn (x)Σdx ∃ dx Π dx Τ

1

>

∃1

Pnm(x)

dΝ Ρ 2 d m Ο(1∃ x ) Pl (x)Σdx )[ n ( n ) 1) ∃ l (l ) 1)] dx Π dx Τ

>

1

Pnm ( x ) Pl m ( x ) dx % 0.

∃1

Integrating by parts, we now obtain 1

+ m , 2 d m − Pl ( x)(1 ∃ x ) dx Pn ( x) . ∃ / 0 ∃1 )

(n ∃ 1) (n ) l ) 1)

or

1

d m d d + , Pl (1 ∃ x 2 ) Pnm ( x )dx ∃ − Pnm ( x )(1 ∃ x 2 ) Plm ( x) . ∃1 dx dx dx / 0 ∃1

>

1

d m d 2 2 Pn (1 ∃ x 2 ) Plm ( x ) dx )[(n ∃ l ) ) (n ∃ l )] ∃1 dx dx

>

>

1

1

∃1

Pnm ( x ) Pl m ( x )dx % 0

>

1

∃1

Pnm ( x ) Pl m ( x )dx %

>

1

1

∃1

{Pnm ( x)}2 dx %

∃1

1

1

∃1

Pnm ( x ) Pl m ( x )dx % 0

Pnm (n) Plm ( x )dx % 0, if n ∋ 1 ... (3)

Pnm ( x ) % (1 ∃ x 2 )m / 2

Case II. Let l = n, If m > 0, we have ;

>

or

>

dm dx m

Pn ( x ).

... (4)

m + , dm 2 m d (1 ∃ x ) P ( x ) − . m Pn ( x )dx, using (4) n ∃1 dx m / 0 dx

>

1

+ ΝΨ ΡΨ ΝΨ d m∃1 ΡΨ , dm % − Ο(1 ∃ x 2 )m m Pn ( x )Σ Ο m ∃1 Pl ( x) Σ . ∃ dx ΤΨ ΠΨ dx ΤΨ 0. ∃1 /− ΠΨ

m ΝΨ d m ∃1 ΡΨ d ΝΨ ΡΨ 2 m d Pn ( x) Σ dx Ο m ∃1 Pn ( x )Σ Ο(1 ∃ x ) m ∃1 Ψ dx dx ΨΤ dx ΨΠ ΨΤ Π

>

1

[On integrating by parts]

Created with Print2PDF. To remove this line, buy a license at: http://www.software602.com/

Legendre Polynomials

>

;

1

9.47

{Pnm ( x )}2 dx % ∃

∃1

m ΝΨ d m ∃1 ΡΨ d ΝΨ ΡΨ 2 m d Pn ( x ) Σ dx Ο m ∃1 Pn ( x ) Σ Ο(1 ∃ x ) m ∃1 Ψ dx dx ΨΤ dx ΨΠ ΨΤ Π

>

1

(1 ∃ x 2 ) y ∃ 2 xy ) n(n ) 1) y % 0.

Since Pn(x) satisfies the Legendre equations

;

d 2 Pn ( x)

(1 ∃ x 2 )

dx 2

∃ 2x

... (5)

dPn ( x ) ) n(n ) 1) Pn ( x ) % 0 dx

... (6)

Differentiating both sides of (6) w.r.t. ‘x’ (m – 1) times with help of Leibnitz’s theorems gives (1 ∃ x 2 )

d m)1

dm

dx

dx m

P ( x ) ∃ 2mx m )1 n

Pn ( x ) ) [n(n ) 1) ∃ m(m ∃ 1)]

d m ∃1 dx m ∃1

Pn ( x ) % 0

Multiplying by (1 – x2)m – 1, we get

(1 ∃ x 2 )m

d m )1 dx m)1

Pn ( x ) ∃2 xm(1 ∃ x 2 )m ∃1

dn dx m

Pn ( x) )[n(n ) 1) ∃ m(m ∃ 1)](1 ∃ x 2 ) m∃1

d m ∃1 dx m ∃1

Pn ( x) % 0

m ΡΨ d ΝΨ d m ∃1 2 m d Pn ( x) Σ % ∃(n ) m) (n ∃ m ) 1) (1 ∃ x 2 ) m ∃1 Pn ( x ) Ο(1 ∃ x ) m dx ΠΨ dx dx m ∃1 ΤΨ

or

... (7)

Using (7), (5) reduces to

>

1

{Pnm ( x)}2 dx %

∃1

m ∃1 ΝΨ d m ∃1 ΡΨ 2 m ∃1 d Pn ( x)dx Ο m ∃1 Pn ( x) Σ(n ) m) (n ∃ m ) 1) (1 ∃ x ) ∃1 Ψ dx dx m ∃1 Π ΤΨ

>

1

2

d m∃1 ΨΝ ΨΡ % (n ) m) (n ∃ m ) 1) Ο(1 ∃ x 2 )( m ∃1) / 2 m ∃1 Pn ( x) Σ dx ∃1 Ψ dx Π ΤΨ

>

% (n ) m) (n ∃ m ) 1)

1

> ΞP 1

∃1

Χ

2 m ∃1 ( x) n

dx, using definition (4) for Pnm ( x)

>

= {(n + m) (n – m + 1)} {(n + m – 1) (n – m + 2)} ... ...{(n ) 1)n}

1

{Pn0 ( x)}2 dx

∃1

[on repeating the similar method (m – 1) times more] = (n + m) (n + m – 1) .... (n + 1) n (n – 1) .... ...(n ∃ m ) 2) (n ∃ m ) 1) %

(n ) m)! 2 1 , using Art. 9.8. (n ∃ m)! 2n ) 1

[

>

1

{Pn ( x )}2 dx,

∃1

From Art 9.23 (i), Pn0 ( x) % Pn ( x) ]

Finally, let m < 0. Then if k > 0, we write m = –k. Then, we know that Pn∃ k ( x ) % (∃1)k

(n ∃ k )! k Pn ( x) (n ) k )!

... (9)

Created with Print2PDF. To remove this line, buy a license at: http://www.software602.com/

9.48

Legendre Polynomials

;

>

1

{Pnm ( x)}2 dx %

∃1

+ (n ∃ k )!, {Pn∃ k ( x)}2 dx % (∃1)2 k − . ∃1 / (n ) k )!0

>

1

2

>

1

{Pnk ( x )}2 dx, using (9)

∃1

2

(n ∃ k )! 2 (n ) m)! 2 + (n ∃ k )!, (n ) k )! 2 , using (8) and k = –m ∗ % 1 %− 1 , % . 1 (n ) k )! 2n ) 1 (n ∃ m)! 2n ) 1 / (n ) k )!0 (n ∃ k )! 2 n ) 1

>

From (8) and (10), we find

;

1

{Pnm ( x )}2 dx %

∃1

>

6

(3) and (11)

1

∃1

(n ) m)! 2 1 (n ∃ m)! 2n ) 1

Pnm ( x) P1m ( x)dx %

... (11)

2(n ) m)! Θnl , (2n ) 1)!(n ∃ m)!

9.24. Recurrence relations (Formulae) for Pnm (x). Prove that (i) Pnm)1 ( x) ∃

2mx

Pnm ( x ) ) {n( n ) 1) ∃ m( m ∃ 1)}Pnm∃1 ( x ) % 0.

(1 ∃ x 2 )1/ 2

(ii) (2n ) 1) xPnm ( x) % (n ) m) Pnm∃1 ( x) ) (n ∃ m ∃ 1) Pnm)1 ( x). 2 1/ 2 m (iii) (1 ∃ x ) Pn ( x ) %

1 {Pnm))11 ( x) ∃ Pnm∃1)1 ( x )}. 2n ) 1

(iv) (1 ∃ x 2 )1/ 2 Pnm ( x ) %

1 {( n ) m) (n ) m ∃ 1) Pnm∃∃11 ( x ) ∃(n ∃ m ) 1) (n ∃ m ) 2) Pnm)∃11 ( x)}. 2n ) 1

Proof. (i) Since Pn(x) is a solution of Legendre’s equation, (1 ∃ x2 ) y ∃ 2 xy ) n(n ) 1) y % 0, (1 ∃ x 2 )

hence

d2

Pn ( x ) ∃ 2 x

d Pn ( x) ) n(n ) 1) Pn ( x) % 0. dx

... (1) dx Differentiating both sides of (1) w.r.t. ‘x’ (m – 1) times with help of Leibnitz’s theorem, gives 2

m )1 ΝΨ ΡΨ dm (m ∃ 1) (m ∃ 2) d m∃1 2 d (1 ∃ x ) P ( x ) ∃ 2 x ( m ∃ 1) P ( x ) ∃ 2 ∗ m ∃1 Pn ( x) Σ Ο m )1 n m n 2! dx dx dx ΠΨ ΤΨ

ΝΨ d m ΡΨ d m ∃1 d m ∃1 ∃2 Ο x m Pn ( x ) ) (m ∃ 1) ∗ 1∗ m ∃1 Pn ( x ) Σ ) n(n ) 1) m ∃1 Pn ( x ) % 0. dx dx ΠΨ dx ΤΨ

or

(1 ∃ x 2 )

d m)1

P ( x) ∃ 2 xm m )1 n

dm

Pn ( x) )[n(n ) 1) ∃ m(m ∃ 1)]

dx dx m 2 (m – 1)/2 Multiplying both sides by (1 – x ) , we get (1 ∃ x 2 )( m )1) / 2

d m )1

dm

dx

dx m

P ( x ) ∃ 2 xm(1 ∃ x 2 )∃1/ 2 (1 ∃ x 2 ) m / 2 m )1 n

d m ∃1 dx m∃1

Pn ( x ) % 0.

Pn ( x )

)[n (n ) 1) ∃ m(m ∃ 1)](1 ∃ x 2 )( m ∃1) / 2

d m ∃1 dx m ∃1

Pn ( x) % 0

Created with Print2PDF. To remove this line, buy a license at: http://www.software602.com/

Legendre Polynomials

9.49

Using the definition Pnm ( x ) % (1 ∃ x 2 )m / 2

dm dx m

Pn ( x ), the above result becomes

2mx

Pnm)1 ( x) ∃

Pnm ( x ) ) [ n ( n ) 1) ∃ m( m ∃ 1)]Pnm∃1 ( x ) % 0. (1 ∃ x 2 )1/ 2 (ii) From recurrence relations for Lengendre polynomial (Refer recurrence relations I and II of Art. 9.8), we have (2n + 1) x Pn(x) = (n + 1) Pn + 1(x) + n Pn – 1(x) ... (1)

and

d d Pn )1 ( x) ∃ Pn ∃1 ( x) ... (2) dx dx Differentiating both sides of (1) w.r.t. ‘x’ m times with help of Leibnitz theorem, we get (2n ) 1) Pn ( x) %

m m ΝΨ d m ΡΨ d m ∃1 (2n ) 1) Ο x m Pn ( x ) ) m ∗1 ∗ m ∃1 Pn ( x ) Σ % (n ) 1) d Pn )1 ( x ) ) n d Pn ∃1 ( x) dx dx m dx m ΠΨ dx ΤΨ Now, differentiating (2) w.r.t ‘x’ (m – 1) times, we get

(2n ) 1)

Putting the value of (2n ) 1) (2n ) 1) x

or

d m ∃1

dm

dx

dx m

P ( x) % m ∃1 n

d m ∃1 dx m ∃1

dm dx m

Pn ∃1 ( x ).

... (4)

Pn ( x) given by (4) in (3), we get

m m ΝΨ d m ΡΨ dm P ( x ) ) m P ( x ) ∃ P ( x) Σ % (n ) 1) d Pn ( x) ) n d Pn ∃1 ( x) Ο m n )1 m n m n ∃1 dx dx dx m dx m ΠΨ dx ΤΨ

dm

(2n ) 1) x

dm

Pm ( x ) % (n ) m)

dm

Pn ∃1 ( x ) ) (n ∃ m ) 1)

dx m dx m 2 m/2 Multiplying both sides by (1 – x ) , we get

(2n ) 1) x (1 ∃ x 2 ) m / 2

dm dx

m

Pn ( x ) % (n ) m )(1 ∃ x 2 ) m / 2

Using the definition Pnm ( x ) % (1 ∃ x 2 )m / 2

have

Pn )1 ( x) ∃

... (3)

dm dx

dm dx m

m

dm dx m

Pn )1 ( x)

Pn ∃1 ( x ) ) ( n ∃ m ) 1) (1 ∃ x 2 ) m / 2

dm dx m

Pn )1 ( x).

Pn ( x ), the above result becomes

(2n ) 1) x Pnm ( x) % (n ) m) Pnm∃1 ( x) ) (n ∃ m ) 1) Pnm)1 ( x). (iii). From recurrence relation for Legendre polynomial (Refer relation III of Art. 9.9), we d d Pn )1 ( x) ∃ Pn ∃1 ( x ). dx dx Differentiating (1) m times w.r.t. ‘x’, we get (2 n ) 1) Pn ( x ) %

(2n ) 1)

dm

Pn ( x ) %

d m)1

Pn )1 ( x ) ∃

dx m x m)1 Multiplying both sides by (1 – x2)(m + 1)/2, we get (2n ) 1)(1 ∃ x 2 )m / 2 (1 ∃ x 2 )1/ 2

dm dx m

Pn ( x)% (1 ∃ x 2 )( m )1) / 2

m 2 m/2 Using the definition Pn ( x ) % (1 ∃ x )

dm dx m

d m )1 dx m)1

... (1)

Pn ∃1 ( x).

d m )1

d m )1

dx

dx m )1

P ( x ) ∃ (1 ∃ x 2 )( m )1) / 2 m )1 n )1

Pn ∃1 ( x)

Pn ( x ), the above result becomes

2 1/ 2 m (2n ) 1)(1 ∃ x 2 )1/ 2 Pnm ( x) % Pnm))11 ( x) ∃ Pnm∃1)1 ( x) or (1 ∃ x ) Pn ( x ) %

Ξ

Χ

1 Pnm))11 ( x) ∃ Pnm∃)11 ( x) . 2n ) 1

Created with Print2PDF. To remove this line, buy a license at: http://www.software602.com/

9.50

Legendre Polynomials

1 {(n ) m) Pnm∃1 ( x ) ) (n ∃ m ) 1) Pnm)1 ( x )} 2n ) 1

xPnm ( x ) %

(iv). From recurrence relation (ii),

Putting this values of xPnm ( x) in recurrence relation (i), we get

Pnm)1(x) ∃

2m 2 1/ 2

(1∃ x )



1 ( ∃1)}Pnm∃1(x) % 0 ... (1) {(n ) m)Pnm∃1(x) ) (n ∃ m )1)Pnm)1(x)} ){n (n)1) ∃mm 2n )1

Replacing m by (m – 1) in recurrence relation (iii), we get (1 ∃ x 2 )1/ 2 Pnm∃1 ( x ) %

Ζ

9

1 1 Pnm)1 ( x) ∃ Pnm∃1 ( x ) or Pnm∃1 ( x ) % {Pnm)1 ( x ) ∃ Pnm∃1 ( x)}. 2n ) 1 (2 n ) 1)(1 ∃ x 2 )1/ 2

Putting this value of Pnm∃1 ( x) in (1), we get Pnm)1 ( x) ∃

2m (2 n ) 1)(1 ∃ x 2 )1/ 2

{(n ) m) Pnm∃1 ( x ) ) (n ∃ m ) 1) Pnm)1 ( x)} )

1 (2n ) 1)(1 ∃ x 2 )1/ 2

{n ( n ) 1)

∃m (m ∃ 1)}Pnm)1 ( x) ∃ Pnm∃1 ( x)} % 0 or

(1 ∃ x 2 )1/ 2 Pnm)1 ( x) %

1 [{2 m(n ) m) ) n (n ∃ 1) ∃ m(m ∃ 1)]Pnm∃1 ( x) 2n ) 1

){2m (n ∃ m ) 1) ∃ n(n ) 1) ) m(m ∃ 1) Pnm)1 ( x)] or

1 m [(n ) m) (n ) m ) 1) Pnm∃1 ( x ) ∃(n ∃ m) (n ∃ m ) 1) Pn)1 ( x)] ... (2) 2n ) 1 Replacing m by (m – 1) in (2), we get (1 ∃ x 2 )1/ 2 Pnm )1 %

(1 ∃ x 2 )1/ 2 Pnm (n) %

1 m ∃1 [(n ) m ∃ 1) (n ) m) Pnm∃1∃1 (n) ∃(n ∃ m ) 1) (n ∃ m ) 2) Pn)1 ( x)]. 2n ) 1

EXERCISE #

(2m)!(1 ∃ x 2 )m )1/ 2

n %0

2 m !(1 ∃ 2tx ) t 2 )m )1/ 2

1. Show that & Pnm) m ( x )t n %

2. Prove that (a) Pnm ( x ) % 3. Prove that Pnm ( x ) %

m

1 n

2 ∗ n!

(1 ∃ x 2 )m / 2

(n ) m)!(1 ∃ x 2 )m / 2 m

(n ∃ m)!2 m !

d n) m dx

n)m

Ε Ι

.

( x 2 ∃ 1)n (b) Pn∃ m ( x ) % (∃1)m

2 F1 Γ m ∃ n,

m ) n ) 1; m ) 1;

(n ∃ m)! m Pn ( x). (n ) m)!

1∃ x Φ Η. 2 ϑ

4. Define associated Legendre’s polynomials and prove their orthoganality condition. 5. Prove that 6. Prove that

Pmm ( x) %

(n ) m)! (1 ∃ x 2 )m / 2 1∃ x Φ Ε 1 . 2 F1 Γ m ∃ n, m ) n ) 1; m ) 1; m (n ∃ m)! 2 Ηϑ 2 m! Ι

Pnm (∃ x) % (∃1) m) n Pnm ( x)

(Utkal 2003)

Created with Print2PDF. To remove this line, buy a license at: http://www.software602.com/

10 Legendre Functions of the Second Kind—Qn(x) 10.1. Some useful results: From Art. 9.2 of chapter 9, we have

# ! (n n! x 1 ∋ 3 ∋ 5 ∋ ... ∋ (2n 1) %(

Qn(x) =

(n 1) (n 2) !( n x 2(2n 3)

1)

∃ ...& )

3)

Multiplying by 2 ∋ 4 ∋ 6 ∋ .... ∋ 2n in the numerator and denominator, we get

or

Qn(x) =

n! (2 ∋ 4 ∋ 6...2n) 1 ∋ 2 ∋ 3 ∋ 4 ∋ 5 ∋ ... ∋ 2n ∋ (2n 1)

Qn(x) =

n ! ∋ 2n n ! # !( n %x (2n 1)! ( n

# !( n %x (

1)

(n 1)(n 2) !(n x 2(2n 3)

(n 1)(n 2) ! (n x 2(2n 3)

1)

∃ ...& )

3)

LM N

2

∃ ...& )

3)

OP Q

2 (n !) (n 1) (n 2 ) !( n 3) x !( n 1) x ... , ...(1) (2 n 1)! 2(2 n 3) which is a solution of Legendre’s equation in descending powers of x, all the powers of x being negative. Re–writing (1), we have

Qn(x) =

or

Qn(x) =

2n ( n!) # (n!) x ! ( n % (2n 1)! (

(n 2)! ! ( n x 2(2n 3)

1)

=

2n ( n!) # (n !) x ! ( n % (2n 1)! (

=

2n (n !) (n 2r )! x ! (n 2 r 1) (2n 1)! r +0 2 ∋ 4 ∋ ... ∋ 2r (2n 3) (2n 5)...(2n

( n 2)! !( n x 2(2n 3)

1)

2 1)

( n 4)! 2 ∋ 4 (2n 3) (2 n

3)

( n 4)! 2 ∋ 4 (2n 3) (2 n

5)

5)

x !(n

x !( n

4 1)

5)

∃ ...& ) ∃ ...& )





,

n

Qn(x) =

2 (n !) (2n 1)!



2r 1) ! (n

,

2r 1)

(n 2r)! x r 2 (r !)(2n 3) (2n 5)...(2n r+0

2r

...(2)

1)

Differentiating (2) w.r.t. ‘x’ we get ∗

Qn.(x) = !

2 n. n ! (n 2r 1)! x !( n 2 r (2n 1)! r +0 2r r !(2n 3) (2n 5)...(2n

,

2)

2r

1)

.

...(3)

Putting n – 1 for n in (3), we get 10.1

Created with Print2PDF. To remove this line, buy a license at: http://www.software602.com/

10.2

Legendre Functions of the Second kind—Qn(x)

Q .n – 1(x) = !

2n ! 1 (n ! 1)! ∗ (n 2r )! x !( n 2 r 1) (2n ! 1)! r + 0 2r r !(2n 1) (2n 3)...(2n 2r ! 1)

,



= !

2n ∋ 2n ! 1 (n ! 1)! (n 2r )! x !( n 2 r 1) (2n ! 1)!∋ 2n r +0 2r ∋ r !(2n 1) (2n 3)...(2n 2r ! 1)

= !

2n ∋ n ! (n 2r )! x ! (n 2 r 1) . (2n)! r +0 2r ∋ r !(2n 1) (2n 3)...(2n 2r ! 1)

,



,

...(4)

Again, putting n + 1 for n in (3), we get Q .n + 1(x) = !

= !



2n 1 ∋ (n 1)! (n 2r 2)! x !( n 2 r 3) (2n 3)! r +0 2r ∋ r !(2n 5)...(2n 2r 1) (2n 2r

,



(2n

2n ∋ n! ∋ (2n 2) (n 2r 2)! x !( n 2 r 3) 3) (2 n 2) (2n 1) (2n)! r +0 2r ∋ r !(2n 5)...(2n 2r 3)

,



= !

2n ∋ n ! (n 2r 2)! x !(n 2 r r (2n)! r +0 2 ∋ r !(2n 1) (2n 3)...(2n

,

3)

2r

3)

.

...(5)

10.2. Recurrence Relations (formulae) for Qn(x) I. Q.n + 1 – Q.n – 1 = (2n + 1)Q n. Proof. We have, Q.n – 1 + (2n + 1)Qn

2n (n!) =! (2n)!

3)



[Bilaspur 1997, 98]

∗ (n 2r)! x!(n 2r 1) 2n (n!) (n 2r)! x !(n 2r 1) ( 2 n 1 ) (2n 1)! 2r (r!)(2n 3) (2n 5)...(2n 2r 1) 2r (r!)(2n 1)(2n 3)...(2n 2r !1) r+0 r+0

,

,

[using results (4) and (2) Art. of 10.1.] n

=

=

2 ( n !) ( 2 n)!

LM (n 2 r )! x , MN 2 (r !)(2n 3) (2n ∗

!(n

r

r+0

5)...(2 n

2r

1)

!

, r +0

! ( n 2 r 1)

( n 2r ) x r 2 ( r !)(2 n 1) ( 2n 3)...( 2n

2r

OP 1) PQ

∗ ∃ 2n (n !) # (n 2r )! x !(n 2 r 1) / {(2n 1) ! (2n 2r 1}& % r (2n)! (% r +0 2 (r !) (2n 1) (2n 3)...(2n 2r 1) &)

,

LM !2r (n 2r)! x , MN 2 (r!)(2n 1) (2n 3)...(2n 2 (n!) L (n 2r)! x =! M0 (2n)! M ,2 (r ! 1)! (2n 1) (2n N

=



2 r 1)

2n (n!) (2n)!



r +0



n

r+0

=!

n

2 (n !) (2n)!

OP = ! 2 (n!) (n 2r)! x , (2 n )! 2 ( r ! 1)! (2n 1)(2n 2r 1) PQ OP, as 1 + 0 (!1) ! 3)...(2n 2r 1) PQ

!( n 2r 1)

r

r+0

!(n 2r 1)

r !1

3)...(2n 2r 1)

!(n 2r 1)

r !1



(n ! 2s

, 2 (s)! (2n s+ 0



n

s

2)! x

1) (2n

!( n 2 s

3)...(2n

3)

2s

3)

, putting r = s + 1 so that s = r – 1

= Q.n + 1 by result (5) of Art. 10.1.

Created with Print2PDF. To remove this line, buy a license at: http://www.software602.com/

Legendre Functions of the Second kind—Qn(x)

10.3

II. nQ .n + 1 + (n + 1)Q .n – 1 = (2n + 1)xQ .n. Proof. We have, (2n + 1)xQ.n – (n + 1)Q.n – 1

(Bilaspur 1998)



= !(2n 1) x ∋

2n (n !) (n 2r 1)! x !(n 2 r 2) (2n 1)! r +0 2r ∋ r ! (2n 3) (2n 5)...(2n 2r 1)

,



2n (n!) (n 2r )! x !( n 2 r 1) (2n)! r +0 2r (r !) (2n 1) (2n 3)...(2n 2r ! 1) ,putting the values of Q.n and Q.n–1

,

!(n 1) ∋ (!1)

=!

2 n (n !) (2n)!



(n 2r 1)! x !(n 2 r 1) (2n 1) 2n (n!) (n 2r)! x !( n 2r 1) (n 1) (2n 2r 1) r 2 (r !) (2n 1) (2n 3)...(2n 2r 1) (2n)! r+ 0 2r (r !) (2n 1) (2n 3)...(2n 2r 1) r +0 ∗

,

n

,



!(n 2r 1)

=!

2 (n!) (2n)!

(n 2r)! x × [(n + 2r + 1) (2n + 1) – (n + 1) (2n + 2r + 1)] r r+0 2 (r!) (2n 1) (2n 3)...(2n 2r 1)

= !

n ∋ 2n (n !) (n 2r )! x !( n 2 r 1) (2n)! r + 0 2r ! 1 (r ! 1)! (2n 1) (2n 3)...(2n

= !

n ∋ 2n (n !) (2n)!

# ∗ 20 (n 2r )! x !( n 2 r 1) % 3 r !1 %( r +1 25 2 (r ! 1)! (2n 1) (2n 3)...(2n

= !

n ∋ 2n (n !) (2n)!

, 2 (s !) (2n

,



,

21 4 2r 1) 26

,



(n

2s

s

s +0

2)! x ! (n

1) (2n

= nQ .n + 1, by result (5) of Art. 10.1. III. (2n + 1) xQ n = (n + 1) Q n Or

2r 1)

xQn =

2 s 3)

3)...(2n

+1

∃ 0& &)

2s

3)

, putting r = s + 1

+ nQ n – 1.

n 1 Q + n Q 2 n 1 n + 1 2n 1 n – 1

(n + 1) Qn + 1 – (2n + 1) xQn + nQn – 1 = 0.

Or

Proof. We have, nQn – 1 – (2n + 1) xQn ∗

2n ! 1 (n ! 1)! (n 2r ! 1)! x !( n 2 r ) = n / (2n ! 1)! r r +0 2 ( r !) (2n 3) (2n 5)...(2 n 2 r ! 1)

,



!(2n 1) /

=

2n (n !) (n 2r )! x ! (n 2 r 1) (2n 1)! r +0 2r (r !) (2n 3) (2n 5)...(2n

,

2n ! 1 (n!) ∋ 2n (2n 1)!



(n

, 2 (r !) (2n r +0

r

2r ! 1)! x !(n 3) (2n !

=

2 n (n !) (2n 1)!

LM (n MN, 2 (r !) (2n ∗

r

r +0

2r )

∋ (2n

2r 1)

2r 1)

2r ! 1) (2n

5)...(2n n

(2n 1) 2 (n !) (2n 1)!

2r ! 1)! x !(n 2r ) 3) (2n 5)...(2n 2r

1)

, putting values of Qn – 1 and Qn

2r 1)



, 2 (r !) (2n / {n(2n

1) ! (2n 1) (n

r +0

r

2r

2r )! x

! (n

(n

3) (2n

2r )

5)...(2n

2r

1)

2r )}

Created with Print2PDF. To remove this line, buy a license at: http://www.software602.com/

10.4

Legendre Functions of the Second kind—Qn(x) n

=

2 (n !) (2n 1)!

= !



, 2 (r !) (2n r +0

n

2 (n 1)! (2n 1)!

= !( n 1)

2r ! 1)! x

(n

r

r +0

n

2 (n !) (2 n 1)!

5)...(2n

(r ! 1)! (2n

LM |R MN, S|T 2 ∗

(!2r ) (n 1)

2r ! 1)! x

(n r !1

2 r)

3) (2n



,2

!( n

2r 1)

!( n

3) (2n

2 r)

5)...(2n !( n

r !1

r +1

2r

= !(n 1) /

∗ 2n (n !) # (n 2 s 1)! x !(n 2 s 2) % s (2n 1)! (% s +0 2 ( s !) (2n 3) (2n 5)...(2n 2s

= !(n 1) /

2n (n !) (2n 2) (2n 2)!

,

(n

, 2 (s !) (2n s+0

s

2s 1)! x ! (n 3) (2n



= !(n 1) /

2n (n 1)! (n 2s 1)! x !( n (2n 3)! s +0 2s (s !) (2n 5)...(2n

2s

,

|UV 1) |W

2r )

( n 2 r ! 1)! x (r ! 1)! (2 n 3) (2 n 5)...( 2 n



1)

2s

2r

0

OP PQ

∃ & , taking r = s + 1 3) )& 2)

5)...(2n

2s

3)

2)

2s

3)

= –(n + 1)Qn + 1, be definition (2). IV. (2n + 1) (1 – x2)Q n. = n(n + 1) (Q n – 1 – Q n + 1). Proof. Since Qn is a solution of Legendre’s equation, namely,

LM N

OP Q

d 2 dy (1 ! x ) + n(n + 1) y = 0 dx dx

d [(1 – x2)Q. ] = –n(n + 1)Q . n n dx Integrating both sides of (1) between the limits ∗ to x, we get (1 ! x 2 )Qn.

or

(1 – x2) Q.n(x) = !n (n 1)

z

x ∗

so

x ∗

= !n (n 1)

7

x



Qn dx

Qn dx , as {Qn.)x = ∗ = 0

{x2 Qn.}x = ∗ = 0.

and

But by recurrence relation I, we get Q.n + 1 – Q.n – 1 = (2n + 1)Qn. Integrating both sides of (3) between the limits ∗ to x, we get Qn

1

! Qn ! 1

x ∗

=

z

x ∗

(2n 1) Qn dx

or

Qn + 1(x) – Qn – 1(x) = [

Now, from (4) and (2), or

(1 – x2) Qn.(x) = – n(n + 1)

...(1)

z

x ∗

(2 n

...(3)

1) Qn dx. ...(4)

{Qn + 1}x = ∗ = 0 = {Qn – 1)x = ∗] Qn

1 ( x)

! Qn ! 1 ( x )

2n 1

(2n + 1) (1 – x2) Qn.(x) = n(n + 1) [Qn – 1(x) – Qn + 1 (x)]. V. xQ n. – Q n. – 1 = nQ n. Proof. From recurrence relation III, (n + 1) Qn + 1 – (2n + 1)x Qn + n Qn – 1 = 0. Differentiating (1) w.r.t. ‘x’, we have (n + 1)Q.n + 1 – (2n + 1) {x Qn. + 1.Qn} + nQ.n – 1 = 0. Now, by recurrence relation I, Q.n + 1 = Q.n – 1 + (2n + 1)Qn. Putting the value of Q.n + 1 given by (3) in (2), we get

...(1) ...(2) ...(3)

Created with Print2PDF. To remove this line, buy a license at: http://www.software602.com/

Legendre Functions of the Second kind—Qn(x)

10.5

(n + 1) {Q.n – 1 + (2n + 1)Qn} – (2n + 1) (x Qn. + Qn) + nQ.n – 1 = 0 or (2n + 1) Q.n – 1 – (2n + 1) xQn. + (2n + 1) Qn (n + 1 – 1) = 0 or Q.n – 1 – xQn. + nQn = 0 i.e. xQn. – Q.n – 1 = nQn. VI. Q .n – xQ .n – 1 = nQ n – 1. Proof. From recurrence relation I, Q.n + 1 – Q.n – 1 = (2n + 1) Qn. ...(1) Multiplying both sides of (1) by x, x Q.n + 1 – x Q.n – 1 = (2n + 1)x Qn. ...(2) Now, from recurrence relation III, (n + 1) Qn + 1 – (2n + 1) x Qn + n Qn – 1 = 0. ...(3) Putting the value of (2n + 1) xQn from (2) in (3), (n + 1) Qn + 1 – xQ.n + 1 + xQ.n – 1 + nQn – 1 = 0. ...(4) From recurrence relation V, nQn = xQn. – Q.n – 1. ...(5) Replacing n by n + 1 in (5), we get (n + 1) Qn + 1 = xQ .n + 1 – Qn.. ...(6) Putting the value of (n + 1)Qn + 1 from (6) in (4), we get xQ.n + 1 – Q.n – xQ.n + 1 + xQ.n – 1 + nQn – 1 = 0 or Qn. – xQ.n – 1 = nQn – 1. VII. (x2 – 1)Q .n = nxQ n – nQ n – 1. Proof. Recurrence relation V, we have xQn. – Q.n – 1 = nQn. ...(1) 2 Multiplying both sides of (1) by x, we get x Qn. – xQ.n – 1 = nxQn. ...(2) Again, from recurrence relation VI, we get Qn. – xQ.n – 1 = nQn – 1. ...(3) Subtracting (3) from (2), we have x2Q.n – Q.n = nxQn – nQn – 1 or (x2 – 1)Q.n = nxQn – nQn – 1. 2 VIII. (x – 1)Q n. = (n + 1) Q n + 1 – (n + 1)xQ n. Proof. Recurrence relations V and VI are xQ.n – Q.n – 1 = nQn ...(1) and Q.n – xQ.n – 1 = nQn – 1. ...(2) Replacing n by n + 1 in (1) and (2), xQ.n + 1 – Q.n = (n + 1)Qn + 1 ...(3) and Qn. + 1 – xQn. = (n + 1)Qn. ...(4) Multiplying both sides of (4) by x, xQ.n + 1 – x2Q.n = (n + 1)x Qn. ...(5) Subtracting (5) from (3), we have – Q.n + x2Q.n = (n + 1)Qn + 1 – (n + 1)xQn or (x2 – 1)Qn. = (n + 1)Qn + 1 – (n + 1)xQn. 10.3. Theorem. The associated Legendre function of the second kind defined by

dm Q (x) dx m n 2 2 d y dy satisfy Legendre’s associated equation (1 ! x 2 ) 2 ! 2 x n( n 1) ! m 2 y = 0. dx dx 1! x Proof. Proceed as in chapter 9. 10.4. Theorem. Assuming Pn as a solution of Legendre’s equation, show that the complete dx solution of this equation is given by aPn + bQn, where Qn = cPn , c being a constant. 2 2 (1 ! x ) Pn Qnm(x) = (1 – x2)m/2

RS T

UV W

z

2

d y dy ! 2x n(n 2 dx dx Let y = uPn be the complete solution of (1), where u is a function of x.

Proof. The Legendre’s equation is given by

Now,

dP dy =u n dx dx

Pn du dx

2

(1 ! x )

1) y = 0.

2

and

2 d Pn d y 2 = u dx dx 2

2

dPn du dx dx

...(1)

Pn

2

d u. dx 2

Putting these values of (1), we have

Created with Print2PDF. To remove this line, buy a license at: http://www.software602.com/

10.6

Legendre Functions of the Second kind—Qn(x)

8 d2P (1 – x2) : u 2n : dx
0

0

(ii) 6(n) = 2

(iii) 6(1) = 1 (v) 6(n + 1) = n 6(n), n > 0 (vii) B(m, n) = B(n, m)

z

#

2

e) x x 2n ) 1 dx

0

(iv) 6(1/2) = ≅ (vi) 6(n + 1) = n !, if n is +ve integer (viii) 6(n) 6(1 – n) = ≅/sin n≅.

(ix) 6(m) = #, so that 1/ 6(m) % 0 if m = 0 or –ve integer (x) B(m, n) = (xi) B(m, n) =

z

i

0

x

m )1

(1 ) x)

n )1

dx %

6(m) 6(n) 6 (m ∃ n )

z

1

0

x

n )1

(1 ) x )

m )1

dx , where m > 0, n > 0

2n ) 1 (xii) 6(2n) = 2 6(n) 6(n ∃ 12 ) ≅

11.4. Relation between Jn(x) and J–n(x), n being an integer Theorem. I Show that when n is (i) positive integer, J–n(x) = (–1)n Jn(x). [Agra 2006; Kanpur 2006, 07, 08 MDU Rohtak 2004; Purvanchal 2006; Meerut 2006; Nagpur 1995;] (ii) any integer, J–n(x) = (–1)n Jn(x) [Kanpur 2004, 08; Ranchi 2010; Meerut 1993] Proof. Part (i). Let n be a +ve integer. We know that #

J–n(x) =

∋ ()1)

r

r %0

FH IK

1 x r ! 6()n ∃ r ∃ 1) 2

2r ) n

.

...(1)

Since n >0, so 6(–n + r + 1) is infinite (and so 1/6(–n + r + 1) is zero) for r = 0, 1, 2, ....., (n – 1). Keeping this in mind we see that the sum over r in (1) must be taken from n to infinity. Thus, #

J–n(x) =



()1)

r %n

#

From (2), J ) n ( x ) =

∋ ()1)

m%0

m∃n

r

FH IK

1 x r ! 6()n ∃ r ∃ 1) 2

FH IK

1 x (m ∃ n) ! 6 (m ∃ 1) 2

2( m ∃ n ) ) n

2r ) n

...(2)

, (on changing the variable

of summation to m = r – n so that r = m + n and so m = 0 when r = n and m = # when r = #) #

(

J ) n ( x) =

∋ m%0

m

()1) ()1)

n

FH IK

1 x 6(m ∃ n ∃1) m ! 2

2m ∃ n

% ()1)

n

#

∋ r %0

()1)

r

FH IK

1 x 6(r ∃ n ∃1) r ! 2

2m ∃ n

,

(on changing the variable of summation from m to r while keeping the limits of summation unchanged.) Thus, for n > 0

J ) n ( x ) = (–1)n Jn(x), by the definition of Jn(x).

... (3)

Part (ii). Let n < 0. Let p be a positive integer such that n = –p. Since p > 0, from part (i) above, we have J– p(x) = (–1)pJp(x) so that Jp(x) = (–1)– p J– p(x). n But p = – n hence the above result becomes J– n(x) = (–1) Jn(x), ...(4) which is of the same form as (3). Hence the required result holds for any integer.

Created with Print2PDF. To remove this line, buy a license at: http://www.software602.com/

11.4

Bessel Functions

Remark. When n is an integer J– n(x) is not independent of Jn(x), because J–n(x) is a constant multiple of Jn(x) as shown above. Hence y = AJn(x) + BJ–n(x) is not the general solution of Bessel equation when n is an integer. Of course, when n is not an integer, the most general solution of Bessel equation is given by y = AJn(x) + BJ– n(x). When n is an integer, the nature of general solution is indicated by the following theorem. Theorem II. The two independent solutions of Bessel’s equation may be taken to be Jn(x) and cos n≅ Jn ( x) ) J)n ( x) Yn(x) = , for all values of n. ...(5) sin n≅ Proof. Case I. Let n be not an integer. Since n is not an integer, sin n≅ & 0. Hence (5) shows that Yn(x) is a linear combination of Jn(x) and J–n(x). But we know that Jn(x) and J–n(x) are independent solutions if n is not an integer. Hence Jn(x) and a linear combination of Jn(x) and J– n(x) will also be independent solutions. Thus we find that Jn(x) and Yn(x) are two independent solutions of Bessel’s equation. Case II. Let n be an integer. Then we have cos n≅ = (–1)n, sin n≅ = 0 and J–n(x) = (–1)n Jn(x). Using these values in (5), we find that Yn(x) has the form 0/0 and so Yn(x) is undefined. To make Yn(x) meaningful, we define it as lim Yn(x) = vlim Α n Yv(x) = v Α n

=

cos v≅ Jv ( x ) ) J)v ( x) sin v≅

[(Β / Βv) {(cos v≅ Jv ( x ) ) J) v ( x )}]v % n , by L’ Hospital’s rule [(Β / Βv) cos v≅)]v % n

LM ) ≅ sin v≅ J ( x) ∃ cos v≅ Β J ( x) ) Β J Βv Βv N = v

v

[≅ cos v≅ ]v % n

=

LM N

OP Q

()1)n Β Jv ( x) Βv

LM N

OP Q

) ()1)2n Β J)v (x ) Βv

)v ( x )

OP Q

v%n

LM N

OP Q

LM N

OP Q

cos n≅ Β Jv ( x ) ) Β J) v ( x ) Βv Βv v%n = ≅ cos n≅

LM N

v%n

OP Q

v%n = 1 Β Jv ( x) ) ()1)n Β J)v ( x ) ...(7) ≅ Βv Βv ≅ Χ)∆Εn v%n We now establish the following two results about Yn(x) as given by (6). (i) Yn(x) is a solution of Bessel’s equation. (ii) Yn(x) is a solution independent of Jn(x). Proof of (i). Since Jv(x) and J–v(x) are solutions of Bessel’s equation of order v, we must have v%n

x2 and

...(6)

d 2 Jv dJv ∃ ( x 2 ) v2 ) Jv % 0 2 ∃ x dx dx

d 2 J) v dJ)v ∃ ( x 2 ) v 2 ) J )v % 0 . 2 ∃ x dx dx Differentiating (8) and (9) w.r.t. ‘v’, we obtain

x2

FG IJ FG IJ H K H K FG ΒJ IJ ∃ x d FG ΒJ IJ ∃ (x H Βv K dx H Βv K

2 ΒJv ΒJv ΒJ x2 d 2 ∃x d ∃ ( x 2 ) v2 ) v ) 2vJv % 0 Β v dx Β v Βv dx

2 ΒJ 2 )v )v x2 d 2 ) v 2 ) )v ) 2vJ)v % 0 . Βv dx Multiplying (11) by (–1)v and subtracting from (10) gives

RS T

UV W

RS T

...(8) ...(9)

...(10) ...(11)

UV W

2 ΒJv Β .Β / Β d Β Β x2 d 2 ) ()1)v J ∃x J ) ()1)v J ∃ ( x 2 ) v 2 ) 1 J v ) ( )1)v J )v 2 Βv )v dx Βv v Βv ) v dx Βv Β v Β v 4 5

) 2v{J v ) ( )1)v J ) v } % 0

...(12)

Created with Print2PDF. To remove this line, buy a license at: http://www.software602.com/

Bessel Functions

11.5

Taking v = n in (12) and using (7), we have 2 x2 d 2 {≅ Yn ( x )} ∃ x d {≅ Yn ( x )} ∃ (x 2 ) n2 ) ≅Yn ( x ) –2n{Jn(x) – (–1)n J–n(x)} = 0 dx dx Since n is an integer, J– n(x) = (–1)nJn(x) by theorem I and hence the last term in the above equation vanishes. So the above equation reduces to x2Yn! + xYn + (x2 – n2)Yn = 0, ...(13) showing that Yn(x) is also a solution of Bessel’s equation of order n. Proof of (ii). We know that an explicit expression Yn(x) for n integral is given by

Yn(x) =

2. x 1 1log ∃ Φ ) ≅4 2 2

n ∃ 2m m 1/ 1 # 1 1 / : x; .1 m J ( x ) ) ( ) 1) < = 2 n 1 ∃ 2 ∋ ∋ ≅ m !( n ∃ m )! > 2 ? r r ∃ n5 m%0 r %1 4 r %1 r 5

n



n )1

)

1 ( n ) m ) 1) ! : x ; < = ≅ m %0 m! >2?



)n ∃ 2m

, ...(14)

where Φ is Euler’s constant. From (14) we find that Yn(x) is infinite when x = 0, whereas Jn(x) is infinite when x = 0. So Yn(x) as given by (6) and Jn(x) are two independent solutions of Bessel’s equation of order n. Remark 1. General solution of Bessel’s equation when n is an integer is y = AJn(x) + BYn(x), A and B being arbitrary constants. ...(15) where Yn(x) is given by (6). Yn(x) is known as Bessel’s function of order n of the second kind. Yn(x) is also called the Neumann function of order n and is denoted by Nn(x). Remark 2. Equations reducible to Bessel’s equation Consider x2y! + xy + (Γ2x2 – n2)y = 0 ...(16) Let z = Γx

so that

becomes

dy dy %Γ dx dz

and

d2y dx

2

% ΓΗ

d2y dz 2

.

Then (16)

...(17) z 2 (d 2 y / dz 2 ) ∃ z (dy / dz ) ∃ ( z 2 ) n 2 ) y % 0 , which is Bessel’s equation of order n. As explained in remark 1 above, the general solution of (17) is y = AJn(z) + BYn(z) or y = AJn(Γx) + BYn(Γx). ...(18) Thus Jn(Γx) and Yn(Γx) are solutions of (16), which is called the modified Bessel’s equation. 11.5. Bessel’s function of the second kind of order n. Definition This is denoted by Yn(x) and is defined by J ( x) cos n≅ ) J)n ( x) Yn(x) = n , n & integer sin n≅ Jv ( x ) cos v≅ ) J)v ( x ) and Yn(x) = vlim , n is an integer.. Αn sin v≅ 11.6. Integration of Bessel’s equation xy! + y + xy = 0 in series for n = 0. Bessel’s function of zeroth order, i.e. J0(x). [Kakitiya 1997] Bessel’s equation for n = 0 is given by xy! + y + xy = 0 ...(1) #

y = ∋ cm x k ∃ m , c0 & 0 .

Let its series solution be #

( y =



m%0

(k ∃ m) cm x k ∃ m ) 1

...(2)

m%0 #

and

y! =

∋ (k ∃ m) (k ∃ m ) 1) c

m%0

mx

k ∃m)2

.

...(3)

Substituting for y, y , y! in (1), we obtain

Created with Print2PDF. To remove this line, buy a license at: http://www.software602.com/

11.6

Bessel Functions #

#

#

x ∋ (k ∃ m) (k ∃ m ) 1) cm x k ∃ m ) 2 ∃ ∋ (k ∃ m) cm x k ∃ m ) 1 ∃ x ∋ cm x k ∃ m = 0 m%0

m %0

#

∋ (k ∃ m)( k ∃ m ) 1) cm x

or

k ∃ m )1

m %0 #

m%0

#

∃ ∋ (k ∃ m) cm m% 0

∋ {(k ∃ m) (k ∃ m ) 1) ∃ (k ∃ m)} cm x

or

k ∃ m )1

m%0

#

#

m%0

m%0

k ∃ m )1 ∃ x

#

#



cm x

k ∃ m ∃ 1

m%0

∃ ∋ cm x

k ∃ m ∃1

=0

%0

m %0

∋ (k ∃ m)2 cm x k ∃ m ) 1 ∃ ∋ cm x k ∃ m ∃ 1 % 0 ,

or

...(4) k–1

which is an identity. Equating to zero the coefficient of the lowest power of x, namely x , we have k2c0 = 0 so that k = 0, 0 (as c0 & 0) Now equating to zero the coefficient of next higher power of x, namely xk, in (4), we have c1(k + 1)2 = 0 so that c1 = 0 as (k + 1)2 & 0 for k = 0. k+m–1 Finally, equating to zero the coefficient of x in (4), we get c (k + m)2cm + cm – 2 = 0 or cm = ) m ) 2 2 , for all m Ι 2. (k ∃ m) When k = 0, we have cm = – (1/m2)cm – 2 for all m Ι 2 ...(5) Putting m = 3, 5, 7, ... in (5) and noting that c1 = 0, we obtain c1 = c3 = c5 = c7 = ... = 0. ...(6) Next, putting = 2, 4, 6, ... in (5), we have c2 = )

c0 , 22

c4 = )

c2 c % 20 2, 2 4 2 34

Putting the above values in (2) for k = 0,

c6 = )

c4 c % ) 2 02 2 , ... and so on 2 6 2 3 4 36

#

y = ∋ cm x m = c0 + c1x + c2x2 + c3x3 + .... m% 0

: ; x2 x4 x6 y = c0 < 1 ) 2 ∃ 2 2 ) 2 2 2 ∃ ......... ad. inf = 2 2 34 2 34 36 > ?

or

If c0 = 1, the above solution is denoted by J0(x) so that J0(x) = 1 )

x2 x4 x6 ∃ ) ∃ ..... .... ad. inf , 22 22 3 42 22 3 42 3 62

where J0(x) is known as Bessel’s function of zeroeth order. Note 1. Replacing n by 0 in Art. 11.2, we can deduce (7). Note 2. From (7), we have J0(0) = 1. 11.6.A. Solved examples based on Art. 11.1 to 11.6 Ex. 1. Prove that (i) J–1/2(x) =

(2 / ≅x ) cos x.

...(7)

[Nagpur 1995]

[Garhwal 2005; Nagpur 2005; Kanpur 2009, 10; Agara 2010; Bhopal 2010; Ranchi 2010]

(ii) J1/2(x) = (2 / ≅x ) sin x. [Nagpur 2003, 05; Garhwal 2004; Kanpur 2004; 07] 2 2 (iii) [J1/2(x)] + [J–1/2(x)] = 2/(≅x). [Lucknow 2010; Meerut 1992, 93] Sol. By the definition of Jn(x), we have Jn(x) =

+ 7 xn x2 x4 1 ) ∃ ) ...8 , n 2 2 6( n ∃ 1) − 2 3 2( n ∃ 1) 2 3 4 3 2 ( n ∃ 1) ( n ∃ 2) 9

...(1)

Created with Print2PDF. To remove this line, buy a license at: http://www.software602.com/

Bessel Functions

11.7

Part (i). Replacing n by – (1/2) in (1) and simplifying, we get J–1/2(x) =

7 x )1/ 2 + x2 x4 1) ∃ ) ...8 = 2 cos x, as 6 1 , ≅x 2 6 Χ 2 Ε − 1 3 2 13 2 3 3 3 4 9 )1/ 2

ch= 1 2



Part (ii). Replacing n by 1/2 in (1) and simplifying, we get J1/2(x) =

+ 7 x2 x4 . ,1 ) ∃ ) ...8 = 6 (3 / 2) − 1 3 2 3 3 1 3 2 3 3 3 4 3 5 9

x1 / 2 1/ 2

2

7 1+ x3 x5 : x; 1 ∃ ) ...8 < = ∆ 1 . ,x ) > 2 ? Η 6 Χ 2 Ε x − 13 2 3 3 13 2 3 3 3 4 3 5 9

6(p + 1) = p6(p)]

[ 7 2 + x 3 x5 2 x ) ∃ ) ...8 % sin x , = ≅x 3! 5! ≅x − 9 Part (iii). Squaring and adding the results of (i) and (ii), we get [J1/2(x)]2 + J–1/2(x)]2 = (2/≅x) (sin2x + cos2x) = 2/≅x.

Ex. 2. Prove that zlim Α0 Sol. By defination,

Jn ( z ) % n 1 , where n > –1. zn 2 6(n ∃ 1)n Jn(z) =

(Kanpur 2005, 07)

LM N

OP Q

zn z2 z4 1 ) ∃ ) ... 4(n ∃ 1) 4 . 8 . (n ∃ 1) . (n ∃ 2) 2n 6(n ∃ 1)

+ 7 Jn (z) 1 z2 z4 1 1 ) ∃ ) ...8 % n = , n z Α0 z n 2 6(n ∃ 1) − 4(n ∃ 1) 4 . 8 . (n ∃ 1) . (n ∃ 2) 9 2 6(n ∃ 1) Ex. 3. Write the general solution of the following equations: (i) x2(d2y/dx2) + x(dy/dx) + (x2 – 25)y = 0 (MDU Rohtak 2005) 2 2 2 2 (ii) x (d y/dx ) + x(dy/dx) + (x – 9/16)y = 0 (iii) d2y/dx2 + (1/x) × (dy/dx) + (1 – 1/6.25x2) y = 0 (iv) x2 (d2z/dx2) + x(dz/dx) + (x2 – 64) z = 0 (v) z(d2y/dz2) + (dy/dz) + zy = 0 Sol. In what follows, we shall use the following solutions of Bessel’s equation

(

lim

x2 y ∃ xy ∃ ( x 2 ) n2 ) y % 0 Result I: y = A Jn (x)+ B J–n(x), where n is not an integer. A, B being arbitrary constants. Result II: y = A Jn(x) + B Yn(x), where n is an integer, A, B being arbitrary constants. (i) Given x2 y ∃ xy ∃ ( x2 ) 52 ) y % 0, which is Bessel’s equation of order 5, which is an integer. Its general solution is y = A J5(x) + B Y5(x), where A and B are arbitrary constants. (ii) Given x2 y ∃ xy ∃ {x 2 ) (3/ 4)2 } % 0, which is Bessel’s equation of order 3/4, which is not an integer. Its solution is y = A J3/4(x) + B J–3/4 (x), where A and B are arbitrary constants (iii) Re-writing, given equating becomes

x2 y ∃ xy ∃ {x 2 ) (2 / 5)2 } y % 0

As in part (ii), solution is y = A J2/5(x) + B J–2/5(x), A, B being arbitrary constants (iv) Given x2(d2z/dx2) + x(dz/dx) + (x2 – 82) z = 0 As in part (i), solution is z = A J8 (x) + B Y8(x), A, B being arbitrary constants (v) Re-writing the given equation, z2(d2y/dz2) + z(dy/dz) + z2y = 0 which is a Bessel equation of order 0, which is an integer. Its solution is y = A J0(z) + BY0(z).

Created with Print2PDF. To remove this line, buy a license at: http://www.software602.com/

11.8

Bessel Functions

Ex. 4. Solve the following differential equation: (i) x2(d2y/dx2) + x(dy/dx) + (4x4 – 1/4)y = 0 (ii) x(d2y/dx2) + dy/dx + (y/4) = 0 by using the subtitution z % x Sol. (i) Suppose that z = x2 dy dy dz dy % % 2 x , by (1) dx dz dx dz

( d2y

and

dx

... (1)

2

%

... (2)

2 d : dy ; d : dy ; dy d : dy ; dy d : dy ; dz dy 2d y , by (1) < = % < 2 x = % 2 ∃ 2 x < = % 2 ∃ 2x < = % 2 ∃ (2x) dx > dx ? dx > dz ? dz dx > dz ? dz dz > dz ? dx dz dz 2

Substituting the above values in the given equation, we get : dy d2y ; dy : 4 1 ; x2 < 2 ∃ 4 x2 2 = ∃ 2 x2 ∃ < 4x ) = y % 0 < dz = dz > 4? dz ? >

or

4z2

d2y dz

2

∃ 4z

dy : 2 1 ; ∃ < 4z ) = y % 0 dz > 4?

... (3) z 2 (d 2 y / dz 2 ) ∃ z (dy / dz ) ∃ {z 2 ) (1/ 4)2 } y % 0 (3) is a Bessel’s equation of order 1/4. Since 1/4 is a positive non-integral real number, hence solution of (3) is y = A J1/4(z) + B J–1/4(z) or y = A J1/4(x2) + B J–1/4 (x2). or

(ii) Given

... (1)

z% x dy dy dz 1 dy using (1) % % , dx dz dx 2 x dz

(

d2y

and

dx 2 d2y

%

... (2)

d : dy ; d : 1 dy ; x )3/ 2 dy 1 d : dy ; % % ) ∃ < = < = dx > dx ? dx > 2 x dz ? 4 dz 2 x dx dz =?

dy 1 d : dy ; dz 1 dy 1 d 2 y ∃ % ) ∃ < = dx 2 4 x 3/ 2 dz 2 x dz > dz ? dx 4 x 3 / 2 dx 4 x dz 2 Using (1), (2) and (3), the given equation, reduces to

or

%)

1

: 1 dy 1 d 2 y ; 1 dy 1 x < ) 3/2 ∃ ∃ y%0 == ∃ 2 < 4x dz 4 x dz ? 2 x dz 4 >

or

)

... (3)

1 dy 1 d 2 y 1 dy 1 ∃ ∃ ∃ y%0 4 z dz 4 dz 2 2 z dz 4

z 2 (d 2 y / dz 2 ) ∃ z (dy / dz ) ∃ ( z 2 ) 02 ) y % 0, which is a Bessel’s equation of order 0 and so its solution is or

y = A J0(z) + B Y0(z)

y % A J 0 ( x ) ∃ B Y0 ( x ),

or

which is the general solution of the given equation, A, B being arbitrary constants. Ex. 5 (a) Solve x(d2y/dx2) + 2(dy/dx) + (xy)/2 = 0 in terms of Bessel’s functions. (KU Kurukshetra 2005) (b) Solve x(d2z/dx2) – 2(dz/dx) + xz = 0 by using the substitution y = z/x3/2. Sol. (a) Given x(d2y/dx2) + 2(dy/dx) + (xy)/2 = 0 ... (1) Assume that ( d2 y dx

2

%

z%y x

so that

y % z/ x

... (2)

dy d )1/ 2 dz x )3 / 2 % (x z ) % x )1/ 2 ) z dx dx dx 2 d : dy ; d : )1/ 2 dz ; 1 d )3 / 2 d 2 z x )3/ 2 dz 1 : )3/ 2 dz 3x )5/ 2 (x z ) = x )1/ 2 ) ) dx ? 2 dx 2 dx 2 dx 2 dx 2

; z= = ?

Created with Print2PDF. To remove this line, buy a license at: http://www.software602.com/

Bessel Functions

11.9

d2y

1 d 2z

1 dz 3 ∃ 5/ 2 z dx x dx x dx 4 x Substituting the above values of y, dy/dx and d2y/dx2 in (1), we get : 1 d2z 1 dz 3z ; z ; x z : 1 dz x < 1/ 2 2 ) 3 / 2 ∃ 5 / 2 = ∃ 2 < 1/ 2 ) 3/ 2 = ∃ ϑ 1/ 2 % 0 < x dx = dx dx 2 x 4x ? 2x ? x >x >

or

2

x1/ 2

or

%

1/ 2

d 2z

dx Multiplying both sides of (3) by x3/2,

2



2

)

3/ 2

1 dz z 1 ) 3 / 2 ∃ x1/ 2 z % 0 dx 2 x 4x 2 2 2 x (d z/dx ) + x(dz/dx) + (x2/2 – 1/4) z = 0 1/ 2

u % x/ 2 dz dz du dz 1 % % ϑ , by (5) dx du dx du 2 d 1 d ∗ dx 2 du

Let

( From (6), we have

d2y

2u 2 ϑ

2

... (4) ... (5) ... (6) ... (7)

d : dy ; 1 d : 1 dz ; % < = , by (6) and (7) dx dx =? dx 2 du > 2 du ? Thus, d2y/dx2 = (1/2) × (d2z/du2) Substituting the above values in (4), we get (

... (3)

%

1 d2z 1 dz : 2 1 ; ∃u 2ϑ ∃

or

u2

d2z du 2

∃u

... (8)

2 dz .0 2 : 1 ; /0 ∃ 1u ) < = 2 z % 0 du 40 > 2 ? 50

... (9)

which is a Bessel equation of order 1/2. Since 1/2 a positive non-negative integer, hence the required solution is given by z = A J1/2(u) + B J1/2(u)

or

y x % A J1/ 2 ( x / 2) ∃ BJ )1/ 2 ( x / 2), by (2) and (5)

(b) Ans. z % c1 x3/ 2 J3/ 2 ( x) ∃ c2 x3/ 2 J )3/ 2 ( x), c1, c2 being arbitrary constants Ex. 6. Verify that the Bessel function J1/ 2 ( x) % (sin x) ϑ (2 / ≅x)1/ 2 satisfies the Bessel equation of order 1/2. (MDU Rohtak 2006) Sol. Bessel equation of order 1/2 is given by x2(d2y/dx2) x(dy/dx) + (x2 – 1/4)y = 0 ... (1)

y % J1/ 2 ( x) % (2 / ≅)1/ 2 ϑ ( x)1/ 2 sin x)

Let

Κ

... (2)

Λ

dy / dx % (2 / ≅)1/ 2 ϑ x )1/ 2 cos x ∃ ()1/ 2) ϑ x )3/ 2 sin x

(

Κ

Λ

d 2 y / dx 2 % (2 / ≅)1/ 2 ϑ )(1/ 2) ϑ x )3/ 2 cos x ) x )1/ 2 sin x ∃ (3 / 4) ϑ x )5 / 2 sin x ∃ ()1/ 2) ϑ x )3 / 2 cos x 2

2

Substituting the above values of y, dy/dx and d y/dx in (1), we get

Κ

Λ

Κ

x 2 ϑ (2 / ≅)1/ 2 ) x )3 / 2 cos x ) x )1/ 2 sin x ∃ (3 / 4) ϑ x )5 / 2 sin x ∃ x ϑ (2 / ≅)1/ 2 x )1/ 2 cos x

Λ

)(1/ 2) ϑ x )3/ 2 sin x ∃ ( x 2 ) 1/ 4) ϑ (2 / ≅)1/ 2 ϑ ( x )1/ 2 sin x ) % 0

or or

Κ

(2 / ≅)1/ 2 ) x1/ 2 cos x ) x3/ 2 sin x ∃ (3 / 4) ϑ x )1/ 2 sin x ∃ x1/ 2 cos x ) (1/ 2) ϑ x )1/ 2 sin x

0 = 0, which is true. Hence y = J1/2

Λ

∃ x 3/ 2 sin x ) (1/ 4) ϑ x )1/ 2 sin x % 0 (x) satisfies the Bessel equation (1) of order 1/2.

Created with Print2PDF. To remove this line, buy a license at: http://www.software602.com/

11.10

Bessel Functions n

n

Ex. 7. Show that d (x Jn(ax)/dx = a x Jn–1(ax) and hence deduce that d(x J1(x))/dx = x J0(x). ()1)r :x; < = r % 0 r !6( n ∃ r ∃ 1) > 2 ? #

()1) r : ax ; < = r % 0 r !6( n ∃ r ∃ 1) > 2 ? #

x n J n (ax) % x n Μ

(

2r ∃ n

J n ( x) % Μ

Sol. We know that

2r ∃ n

... (1) ()1)r a 2 r ∃ n x 2 r ∃ 2 n

#

x n J n (ax ) % Μ

or

r %0 22 r ∃ n r !6(n ∃ r

# ( )1) r a 2 r ∃ n ϑ 2( r ∃ n ) x 2 r ∃ 2 n )1 d n x J n (ax ) % Μ r %0 dx 22 r ∃ n r ! 6 (n ∃ r ∃ 1)

Χ

(

Ε

()1)r : ax ; < = r %0 r ! 6 (n ∃ r ) > 2 ? #

∃ 1)

... (2)

2 r ∃ n )1

J n )1 (ax ) % Μ

From (1),

#

()1) r a 2 r ∃ n x 2 r ∃ 2 n )1 ϑ 2(r ∃ n)

r %0

22 r ∃ n )1 r ! 6 (n ∃ r ) ϑ 2(r ∃ n)

#

()1) r a 2 r ∃ n ϑ 2(r ∃ n) x 2 r ∃ 2 n )1

r %0

22 r ∃ n r ! 6 (r ∃ n ∃ 1)

... (3)

Ε

... (4)

ax n J n )1 (ax) % Μ

(

ax n J n )1 (ax) % Μ

or

Χ

d n x J n ( ax) % ax n J n )1 ( x ) dx d ( xJ1 ( x )) % x J 0 ( x) Putting n = 1 and a = 1 in (4), we have dx

From (2) and (3),

Ex. 8. Show that

1

u J 0 ( xu )

0

2 1/ 2

Ν

(1 ) u )

du %

sin x x

J 0 ( x) % 1 )

Sol. We have (see Art. 11.2),

x2 2

2



x4 2

2 34

2

)

x6

∃ .....

2

2 3 4 2 3 62

... (1)

Ν

( %

Ν

0

≅/ 2

0

1

uJ 0 ( xu) 2 1/ 2

(1 ) u )

du %

u

1

Ν (1 ) u )

2 1/ 2

0

; sin Ο : x2 x4 2 4 2 ?

[Bilaspur 1997] 2r ∃ n

... (1)

Created with Print2PDF. To remove this line, buy a license at: http://www.software602.com/

Bessel Functions

11.11

()1)r : z cos Ο ; < = r % 0 r ! 6 (1 ∃ r ∃ 1) > 2 ? #

(1) Ρ ≅/ 2

Ν

(

()1)r z 2 r ∃1

#

% Μ

r ! (r ∃ 1)!2

r %0

2 r ∃1

3

()1)r z 2 r ∃1

#

% Μ

r ! (r ∃ 1)!2

r %0

3 2 r ∃1

()1)r z 2 r ∃1

#

% Μ

r ! (r ∃ 1)!2

r %0

()1)r z 2 r ∃1

#

J1 Χ z cos ΟΕ d Ο % Μ

r %0

0

2 r ∃1

J1 ( z cos Ο) % Μ

3 2 r ∃1

r ! (r ∃ 1) ! 2

2 r ∃1

Ν

≅/2

0

2r (2r ) 2)...4 3 2 (2r ∃ 1)(2r ) 1)...5 3 3

cos 2 r ∃1 Ο d Ο

[using a standard result of Integral calculus]

[2r (2r ) 2)...4 3 2]2 (2r ∃ 1) 2r (2r ) 1) (2r ) 2)...5 3 4 3 3 3 2 31 # 22 r (r1)2 ()1)r z 2 r ∃1 1 1 z z3 z5 1 ) cos z % Μ % ) ∃ ) ∃ ) ... % (2r ∃ 1)! r %0 2(r ∃ 1)(2r ∃ 1)! z z 2! 4! 6! z

Ex. 10. Prove that J n ( x ) %

xn 2

n )1

6(n) Ν

≅/ 2

0

sin Ο cos 2 n )1 Ο J 0 ( x sin Ο)d Ο, where n > – 1/2.

[Ravishankar 2002] ()1)r :x; < = r % 0 r ! 6 ( n ∃ r ∃ 1) > 2 ? #

#

J 0 ( x sin Ο) % Μ

(1) Ρ

r %0

I%

Ν

≅/2

0

#

()1) x 2 r

r %0

(r !)2 22 r

% Μ

Ν

I%

Let

(

r

≅/ 2

0

()1) r : x sin Ο ; < = r ! 6(r ∃ 1) > 2 ?

2r

... (2)

sin Ο cos 2 n )1 Ο J 0 ( x sin Ο)d Ο

Ν

≅/ 2

#

cos 2 n )1 Ο sin 2 r ∃1 Ο d Ο % Μ

... (3)

r %0

0

()1)r x 2r 6(n) 6(r ∃ 1) 6(n) # ( )1)r :x; 3 % Μ < = 2 2r 6( n ∃ r ∃ 1) 2 r %0 r !6(n ∃ r ∃ 1) > 2 ? (r !) 2

n

()1)r :x; # :x; I % Μ < = < = n )1 r % 0 r ! 6 (n ∃ r ∃ 1) > 2 ? 2 6(n) >2? xn

or

... (1)

: # ; ()1)r x 2 r sin Ο cos 2 n )1 Ο r %0 r ! 6(r ∃ 1) 2 ?

xn

or

2r ∃ n

J n ( x) % Μ

Sol. We have

2

n )1

6(n) Ν

≅/2

0

()1) r :x; < = r % 0 r !(n ∃ r ∃ 1)! > 2 ? #

2r ∃ n

% Μ

sin Ο cos 2 n )1 Ο J 0 ( x sin Ο) d Ο % J n ( x), by (1) and (3)

Ex. 11. Show that y % A J n ( x )

x

dx

0

xJ n2 ( x)

Ν

equation. Sol. The Bessel’s equation is

∃ B J n ( x) is the complete solution of Bessel’ss

[Bilaspur 1994] 2

2

y ∃ (1/ x) ϑ y ∃ (1 ) n / x ) y % 0 u = Jn(x) y=uv

We know that a solution (1) is Let the complete solution of (1) be Comparing (1) with

2r

2r

y ∃ Py ∃ Qy % R,

P = 1/x

and

... (1) ... (2) ... (3)

Q = 1 – n2/x2,

R=0

Created with Print2PDF. To remove this line, buy a license at: http://www.software602.com/

11.12

Bessel Functions

*Then, we know that v is given by d 2v : 2 du ; d v R ∃

dv/dx = q

Let

d 2 v : 1 2 J n ( x) ; d v ∃< ∃ %0 = J n ( x) ? dx dx 2 > x

or

d2v/dx2 = dq/dx

so that

Then (4) yields

dq : 1 2 J n ∃< ∃ dx < x J n >

Integrating,

log q + log x + 2 log Jn = log A

; =q % 0 = ?

dq : 1 2 J n ∃< ∃ q < x Jn >

or

q % d v / dx % A /( x J n2 )

or

Ν

... (5)

; = dx % 0. = ?

q x J n2 % A

or

Κ

Λ

d v % A /( x J n2 ) dx

or v% A

... (4)

dx

x

... (6) ∃ B, x J n2 where A and B are arbitrary constants. From (2), (3) and (6) the required complete integral is Integrating,

: y % J n ( x) < A < >

Ν

x

0

; ∃ B= = ?

dx J n2

x

0

y % A J n ( x)

or

2( x / 2)n ) m 6(n ) m)

Ex. 12. Prove that if n > m – 1, then Jn(x) = I=

Sol. Let

I=

z

1

0

∋ r %0

(1 ) t 2 )n ) m ) 1 t m ∃ 1 ()1)r ( x / 2)2r ∃ m r ! 6(m ∃ r ∃ 1)

#

=

0

1

0

dx

0

x J n2 ( x)

∃ BJ n ( x)

(1 ) t 2 )n ) m ) 1 t m ∃ 1 Jm ( xt) dt .

(1 ) t 2 )n ) m ) 1 t m ∃ 1 Jm ( xt ) dt .

Then using the definition of Jm(xt), we have

#

=

z

1

z

x

Ν

z

1

0

#

∋ r %0

2 n ) m ) 1 2m ∃ 2r ∃ 1

(1 ) t )

∋ (r)!16) ((mx /∃2r) ∃ 1) . 12 r %0

r

FH IK

1 xt r ! 6(m ∃ r ∃ 1) 2

2r ∃ m

#

2r ∃ m

r

()1)r

z

1

0

t

dt =

dt

()1)r (x / 2)2r ∃ m

1

∋ r ! 6(m ∃ r ∃ 1) Ν (1 ) t )

2 n ) m )1

0

r %0

(1 ) z)n ) m ) 1 z( m ∃ r ∃ 1) ) 1 dz , on putting t2 = z and t dt =

(t 2 )m ∃ r t dt dz 2

2r ∃ m

# = ∋ ()1) (x / 2)

6(n ) m) 6(m ∃ r ∃ 1) , provided n – m > 0 and m + r + 1 > 0 i.e. n > m > –1 .1. r ! 6 ( m ∃ r ∃ 1 ) 2 6(n ∃ r ∃ 1) r%0 + , −

=

6( n ) m) : x ; < 2= 2 > ?

( Jn(x) = 2

m)n #

∋ r %0

( x / 2) m ) n I 6(n ) m)

1

Ν (1 ) z) 0

()1)r : x; r ! 6(n ∃ r ∃ 1) 2 =? or

Σ )1 Τ )1

z

n ∃ 2r

Jn(x) = 2

=

dz % B (Σ, Τ) %

FH IK

6(n ) m) x 2 2 m)n

( x / 2) 6 (n ) m )

z (1 ) t ) 1 0

7 6(Σ ) 6(Τ) , if Σ Υ 0, Τ Υ 0 8 6 ( Σ ∃ Τ) 9

m)n

Jn(x), by def. of Jn(x)

2 n ) m )1 m ∃1

t

J m ( xt ) dt, using (1)

*Refer Chapter 10 in part I of ‘‘Ordinary and Partial Differential Equations’’ by Dr. M.D. Raisinghania published by S.Chand & Co., New Delhi

Created with Print2PDF. To remove this line, buy a license at: http://www.software602.com/

Bessel Functions

11.13

z

n

(ii) Jn(x) =

2

n )1

1

x 1 6( 2 ) 6(n ) 1)

0

1

z

1

(1 ) t 2 )n ) 1/ 2 eixt dt .

)1

A2 Ar ∃ .... ∃ ∃ .... % 2! r!

eA = 1 ∃ A ∃

But

z

1

( I=

)1

(1 ) t 2 )n ) 1/ 2

R| (ixt) U| dt S|∋ r ! V| T W #

(1 ) t 2 )n ) 1/ 2 eixt dt , if n Ι ) 12

)1

(1 ) t 2 )n ) 1/ 2 cos xt dt .

I =

Sol. Part (i). Let

z

( x / 2 )n ≅ 6(n ∃ 1)

Ex. 13. Prove that (i) Jn(x) =

#

r

I=

or

r %0

∋ (ixr)!

r

r %0

z

1

#

r

∋ Ar! r%0

(1 ) t 2 )n ) 1/ 2 t r dt . ...(2)

)1

Now, if r is odd (i.e. r = 2m + 1), the integrand in the above integral is an odd function of t and hence it vanishes whereas if r is even (i.e. r = 2m), the integral is even function of t and so by a

z

1

property of definite integral #

I= #

=

)1

0

1 (ix )m . 2 (1 ) t 2 ) n ) 1/ 2 t 2 m dt = 0 (2m) ! m %0

∋ m

Ν

2m

∋ ()(12)m)x!

m%0 #

=

z

1

(1 ) t 2 )n ) 1/ 2 t 2 m dt = 2 (1 ) t 2 )n ) 1/ 2 t 2 m dt . So (2) gives

()1)m x 2m (2m) ! m%0



z z

1

0 1

0

#

(i 2 ) m x 2 m (2m) ! m %0



(1 ) z)n ) 1/ 2 (z1/ 2 )2 m ) 1. dz ,

on putting

1

Ν (1 ) t

2 n ) 1 2m ) 1

0

)

t2 = z

t

. 2t dt , by (1)

so that

2t dt = dz

#

()1)m x 2 m 6(n ∃ 12 ) 6(m ∃ 12 ) ϑ , (2m) ! 6 (n ∃ m ∃ 1) m %0



(1 ) z)(n ∃ 1/ 2 ) ) 1 z( m ∃ 1/ 2) ) 1 dz =

provided n + 12 > 0, m + 12 > 0 i.e. n > – 12 , m > – 12 + , −

1

Ν (1 ) x)

Σ )1

0

xΤ ) 1 dx % B (Σ, Τ) %

7 6( a ) 6(Τ) , if Σ Υ 0, Τ Υ 0 8 6 (Σ ∃ Τ) 9

#

= 6(n ∃ 12 )

()1)m x 2 m (2m) ! ≅ ϑ 2m ϑ ( 2 m ) ! 6 ( n ∃ m ∃ 1 ) m! 2 m%0



+ , −

By duplication formula,when m is a+ve integer,6(m + 12 ) %

FH IK ∋ ()1) FH x IK m ! 6(n ∃ m ∃ 1) 2

= 6(n ∃ 12 ) ≅ x 2

)n #

m

m%0

FH IK

x = 6(n ∃ 12 ) ≅ 2

)n

Jn(x), by definition of Jn(x)

( x / 2 )n Jn(x) = 6(n ∃ 1 ) ≅ I 2

( or

2m ∃ n

z z

(2m)! ≅ 7 8 22m m ! 9

...(3)

1 ( x / 2 )n (1 ) t Η )n ) 1/ 2 eixt dt, by (1). ≅ 6(n ∃ 12 ) )1 Part (ii). Since eixt = cos xt + i sin xt, (1) gives

Jn(x) =

I=

z

1

(1 ) t Η )n ) 1/ 2 (cos xt ∃ i sin xt ) dt =

)1

1

z

1

(1 ) t Η )n ) 1/ 2 cos xt ∃ i (1 ) t Η )n ) 1/ 2 sin xt dt

)1

)1

Created with Print2PDF. To remove this line, buy a license at: http://www.software602.com/

11.14

Bessel Functions

z

1

I = 2 (1 ) t Η )n ) 1/ 2 cos xt dt ∃ 0

or

...(4)

0

[ the integrand in the first integral is an even function of t, while the integrand in the second integral is an odd function of t and it is known that

z

. a 02 f (t ) dt , if f (t ) is even function f (t ) dt = 1 0 )a 040, if f (t ) is odd function

Ν

a

Using (4) and noting that ≅ = 6(1/2), (3) gives Jn(x) =

z

n 1 ( x / 2 )n . 2 (1 ) t 2 )n ) 1/ 2 cos xt dt = n ) 1 1x 1 6(n ∃ 2 ) 0 Η 6( 2 ) 6 (n ∃ 12 )

6( 12 )

Ex. 14. (i) Prove that J n J ) n ) J n J ) n = )

FG IJ H K

z

1

0

(1 ) t 2 ) n ) 1/ 2 cos xt dt .

2 sin n≅ . x≅

[Vikram 2004]

J) n 2 sin n≅ (ii) Prove that d . %) dx Jn ≅x Jn2

[Bilaspur 1998]

(iii) Prove that J n(x) J–n(x) – Jn(x) J ) n (x) = c/x where c is a constant. By considering the behaviour for the large values of x, show that c = (2 sin n≅)/≅. (iv) Show that the Wronskian W(Jn, J–n) of Jn and J–n is given by W(Jn, J–n) % )(2 / ≅x) ϑ sin n≅.

[Ravishankar 1998, 2000]

Sol. (i) We know that Jn and J–n are solutions of Bessel’s equation y! + (1/x)y + (1 – n2/x2)y = 0.

J n ∃ (1/ x) J n ∃ (1 ) n2 / x2 ) J n % 0

...(1)

J– n ∃ (1/ x) J ) n ∃ (1 ) n2 / x2 ) J ) n % 0

...(2)

( and

Multiplying (1) by J–n and (2) by Jn and then subtracting, we have J n J ) n ) J ) n J n ∃ (1/ x) ϑ ( J n J ) n ) J ) n J n ) = 0

...(3)

J n J )n ) J ) n J n % v .

...(4)

Let Differentating w.r.t. ‘x’ (4) gives

J n J ) n ∃ J n J– n ) ( J) n J n ∃ J– n J n ) % v Using (4) and (5), (3) becomes v + (1/x)v = 0 or (dv/dx) + (1/x)v = 0 Integrating, log v + log x = log c

or or

(1/v)dv + (1/x)dx = 0. v = c/x

J nJ–n ) J ) n J n % c / x, by (4)

or Now,

and

J n J ) n ) J )n J n % v . ...(5)

or

Jn(x) = J–n(x) =

...(6)

+ n 7 xn ∃ 2 xn ∃ 4 x ) ∃ ) ...8 , n 4(n ∃ 1) 4 3 8 3 (n ∃ 1)(n ∃ 2) 2 6(n ∃ 1) − 9 1

+ )n 7 x2 ) n x4 ) n ∃ ) ...8 ,x ) 4(1 ) n) 4 3 8 3 (1 ) n)(2 ) n) 6()n ∃ 1) − 9 1

2) n

( Using the above values of Jn and J–n, (6) gives

Created with Print2PDF. To remove this line, buy a license at: http://www.software602.com/

Bessel Functions

11.15

+ n ) 1 (n ∃ 2) x n ∃ 1 7 1 (n ∃ 4) x n ∃ 3 ) ∃ ) ...8 , nx 26(n ∃ 1) − 4(n ∃ 1) 4 3 8 3 (n ∃ 1)(n ∃ 2) 9 ϑ )

+ )n 7 x2 ) n x4 ) n ∃ ) ...8 ,x ) 4(1 ) n) 4 3 8 3 (1 ) n)(2 ) n) 6()n ∃ 1) − 9 1

2) n

+ 7 (2 ) n) x1 ) n (4 ) n) x 3 ) n )n ) 1 ) ∃ ) ...8 ,)n x 4(1 ) n) 4 3 8 3 (1 ) n)(2 ) n) 6()n ∃ 1) − 9 1

2) n

ϑ

+ n 7 c xn ∃ 2 xn ∃ 4 ∃ ) ...8 % ,x ) 4(n ∃ 1) 4 3 8 3 (n ∃ 1)(n ∃ 2) 2 6(n ∃ 1) − 9 x 1

n

Now comparing the coefficients of 1/x from both sides, we get n )n

n

2 6(n ∃ 1) 3 2 6()n ∃ 1)

or



n )n

2 6()n ∃ 1) 3 2n 6(n ∃ 1)

=c

c=

or

2n n6(n) 6(1 ) n)

LM N

2 sin n≅ 2 6(n) 6(1 ) n) % ≅ = sin n≅ (≅ / sin n≅) ≅ Putting this value of c in (6) and multiplying both sides by (–1), we get

c=

2 sin n≅ . ≅x Part (ii). Dividing both sides of (7) by Jn2, we get

Jn J)n ) J n J)n = –

Jn J) n ) Jn J) n 2 sin n≅ =– 2 Jn ≅x Jn2

OP Q

...(7)

FG IJ = ) 2 sin n≅ . H K ≅x J

d J) n dx Jn

or

2 n

Part (iii) Refer part (i). (iv) By definition, W ( J n , J )n ) %

Jn Jn

J )n % J n J )n ) J n J ) n % )(2 / ≅x) ϑ sin n≅, by (7) J )n

Ex. 15(a). Prove that Jn(x) = (–2)n xn

d n J 0 ( x) d ( x 2 )n

[Ranchi 2010]

.

n

: d ; (b) Show that if J n ( x) % kx n < 2 = J 0 ( x), then k = –(2)n. [Ravishankar 1998] > dx ? Sol. (a) We know that J0(x) is a solution of Bessel’s equation of order zero (i.e. n = 0) namely, x2 = X

Let ( From (3) (

...(1) d 2 y / dx2 ∃ (1/ x) ϑ (dy / dx) ∃ y % 0 . so that x= X and dX/dx = 2x = 2 X . ...(2) dy dy dX dy = %2 X , by (2) ...(3) dx dX dx dX d d ∗ 2 X . ...(3) dx dX

FH IK FH IK L1 dy I d F X X % 4 X M ( X) dX H dX K N2

d2 y dy dy d = d %2 X 2 X , by (3) and (3) dx dx dX dx dx 2 = 4

)1/ 2

dy d2y ∃ X dX dX 2

OP Q

= 4X

d2y dX 2

∃2

dy ... (4) dX

Created with Print2PDF. To remove this line, buy a license at: http://www.software602.com/

11.16

Bessel Functions 2

2

Putting the values of x, dy/dx and d y/dx from (2), (3) and (4) in (1), we get 4X

d 2y dX

∃2

2

dy 1 dy ∃ .2 X ∃ y%0 dX dX X

4 X

or

d2y dy ∃4 ∃ y = 0. ...(5) dX dX 2

Differentiation of (5) n times w.r.t. ‘X’ by Leibnitz theorem gives

LM N

4 X

Y=

OP Q

n∃2

n ∃1

n

d n ∃ 2y d n ∃ 1y d n ∃ 1y d n y d y d y d y =0 ∃ n . 1 ∃ 4 n∃2 n ∃1 n ∃1 ∃ n = 0 or 4 X n ∃ 2 ∃ 4(n ∃ 1) n ∃1 ∃ n dX dX dX dX dX dX dX Since y = J0(x) is a solution of (1), we now take n d n y d J0 ( x ) n % dX d ( x 2 )n

( (6) gives

dY d n ∃ 1 y % dX dX n ∃ 1

so that 4X

d2y dy ∃ X = 0. 2 ∃ 4(n ∃ 1) dX dX

y = xn z

...(7) ...(8)

FG H

IJ K

d 2 y 1 dy n2 y ∃ ∃ 1 ) = 0. dx 2 x dx x2

But Jn(x) is a solution of Bessel equation Let

d2y dn∃2y % dX 2 dX n ∃ 2

and

Jn(x) = xnz.

so that

....(6)

...(9) ...(10)

2

2 d y dy dz dz n d z ∃ 2nx n ) 1 ∃ n(n ) 1)z n ) 2 z . = xn + nxn – 1z and 2 = x dx dx dx dx dx 2 Using the above values of y, dy/dx and d2y/dx2, (9) gives

Differentiating (10) ,

xn

or

IK FG H

FH

d 2z x n dz d 2z 1 dz . ∃ xnz = 0 ∃ (2n ∃ 1) ∃ z = 0. or 2 ∃ (2n ∃ 1) x dx x dx dx dx 2 Using (2) and also (3) and (4) [after replacing y by z here], (11) gives xn

F 4 X d z ∃ 2 dz I ∃ (2n ∃ 1) . 2 GH dX dX JK x 2

2

or

IJ K

d 2z 1 n dz n2 n ) 1 dz ∃ n(n ) 1) x n ) 2 z ∃ x ∃ nx n ) 1z + 1 ) 2 xnz = 0 2 ∃ 2nx dx x dx dx x

...(11)

dz ∃z=0 dX

X

...(12) 4 X (d 2 z / dX 2 ) ∃ 4(n ∃ 1)(dz / dX ) + z = 0. Comparing (8) and (12), we have z = kY, where k is a constant to be determined. Using (7)

and (10), we have

xnz = kxnY

Jn(x) = kxn

or #

But

J0(x) =

()1)r x 2r r 2 % r % 0 (2 r !)



d nJ 0 ( x) d ( x 2 )n

.

...(13)

#

()1)r X r r 2 r % 0 (2 r !)



(

x2 = X)

Differentiating both sides w.r.t. x2 (i.e. X) n times, we have d n J0 ( x ) = d ( x 2 )n

#

r

∋ (2()1r)!) r %0

r

2

dn r X dX n

...(14)

()1)n 2 n 2 n ! + terms involving X (i.e. x ) (2 n !) (Note that on differentiation all those terms in R.H.S. of (14) for which r < n will vanish). =

Created with Print2PDF. To remove this line, buy a license at: http://www.software602.com/

Bessel Functions

11.17

Using value of Jn(x) and the above expression, (13) gives n + 7 7 x2 n + ( )1) n ! 1 ) ∃ ... % kx ∃ terms contaning x 2 8 , 8 , n n 2 2 6(n ∃ 1) − 2 3 2(n ∃ 1) 9 − (2 n !) 9 n Equaing the coefficients of x from both sides, we get

xn

()1)n n ! 1 k = (2 n n !)2 2 n 6(n ∃ 1)

k = (–1)n 2n = (–2)n

or [

6(n + 1) = n !, n being +ve integer]

Jn(x) = (–2)nxn

With this value of k, (13) gives

... (15)

d n J0 ( x ) . d ( x 2 )n

(b) Proceed as in part (a). From (15), we have k = (–2)n

EXERCISE 11 (A) 1. Show that y = x 2. Show that

z

≅/2

z

#

0

(b) Prove that

z

#

0

5. Prove that

Jn (bx )x ne )ax dx =

z

#

0

#

0

2 n 6(n ∃ 12 ) ≅

Jn (bx) x n ∃ 1 e )ax dx =

Jn(bx) x

z

Jn(2 x ) satisfies the equation xy! + (n + 1)y + y = 0.

(≅x ) J1/ 2 (2 x ) dx = 1.

0

3. (a) Prove :

4. Prove :

–n/2

2

n ∃ 1 )ax

e

dx = 2

Jn (bx) x n ∃ 1e )ax dx

=

.

bn (a ∃ b 2 )n ∃ 1/ 2 2

n 2 n ∃ 1 6(n ∃ 12 ) . 2 ab n ∃ 3 / 2 , a > 0. ≅ (a ∃ b)

bn exp()b2 / 4a) , where exp (p) = ep (2a)n ∃ 1 n

2

b a

FG n ∃1 ) b IJ expFG ) b IJ , a Υ 0 . H 4a K H 4a K 2

n ∃1 n ∃ 2

2

6. For what value of n the general solution of Bessel’s differential equation will be of the form y = AJn(x) + BJn(x). 7. Write the differential equation satisfied by Bessel’s function of order n. Express the following Bessel’s functions in terms of trigonometric functions : (i) J1/2(x), (ii) J–1/2 (x), (iii) J3/2(x), (iv)J–3/2(x). 11.7. Recurrence Relations (Formulae) for Jn(x). Prove that I. II. III.

d n {x Jn(x)} = xnJn – 1(x). [Agra 2005; Guwahati 2007; Kanpur 2004, 09; Nagpur 1996] dx d –n {x Jn(x)} = –x–nJn + 1(x). dx

J n(x) = Jn – 1(x) – (n/x) Jn

[Agra 2008; Gulbarga 2005; Kanpur 2009] or

x J n = –nJn + xJn – 1. [Agra 2010; Bilaspur 2004;

KU Kurukshetra 2005; Agra 1997; Kanpur 2005, 09, 11; Meerut 2010; Nagpur 2010] IV. J n(x) = (n/x) Jn(x) – Jn + 1(x) or xJ n = nJn – xJn + 1. [Nagpur 2005; Bangalore 93, 94; Meerut 2005, 07, 11; Kanpur 1999; Bilaspur 1998] V. J n(x) = 12 {Jn – 1(x) – Jn + 1(x)} or Jn – 1 – Jn + 1 = 2 J n . Agra 2009; Jiwaji 2004, 07; Ravishankar 2004; Nagpur 1995; Kanpur 2007] VI. Jn – 1(x) + Jn + 1(x) = (2n/x)Jn(x) or xJn + 1(x) + xJn – 1(x) = 2nJn(x) or 2Jn = x(Jn – 1 + Jn + 1). [Agra 2008; Bangalore 2005; Meerut 2006; Kanpur 2006, 11; Nagpur 2010]

Created with Print2PDF. To remove this line, buy a license at: http://www.software602.com/

11.18

Bessel Functions

Proof 1. Using the definition of Jn(x), we have

R| ()1) S| ∋ T

d d n xn {x Jn(x)} = dx dx #

=

#

FG IJ H K

1 x r ς 6 (n ∃ r ∃ 1) 2

r

r %0

( )1) r 1 d . 2r ∃ n . ( x 2r ∃ 2n ) = r ς 6 (n ∃ r ∃ 1) 2 dx

∋ r %0

#



2r ∃ n

U| V| W

( )1) r (2r ∃ 2n) x 2 r ∃ 2 n ) 1 r ! 6 (n ∃ r ∃ 1) 2 2 r ∃ n

r %0

#

=

()1)r 3 2 3 (r ∃ n) x n 3 x 2 r ∃ n ) 1 . r ς Χ n ∃ r Ε 6( n ∃ r ) 22 r ∃ n r %0



#



= xn

r %0

FH IK

()1)r x r ς 6(n ) 1 ∃ r ∃ 1) 2

2r ∃ n ) 1

= xnJn – 1(x). by the definition of Jn – 1(x).

II. Using the definition of Jn(x), we have

R| S| ∋ ()1) T

d –n {x Jn(x)} = d x ) n dx dx

#

r%0

r

FH IK

1 x r ς6 (n ∃ r ∃ 1) 2

#

=

()1)r 1 d 3 2r ∃ n 3 ( x2r ) = r ς6(n ∃ r ∃ 1) 2 dx r %0



()1)r x 2 r ) 1 1 3 2r ∃ n ) 1 = Χ r ) 1Ε ς6(n ∃ r ∃ 1) 2 r %0

2r ∃ n

U| V| W

#

()1)r 2rx 2 r ) 1 2r ∃ n r % 0 rΧr ) 1Ε ς6(n ∃ r ∃ 1) 2



#

#

=

6(n + 1) = n6(n)]

[

()1)r x 2 r ) 1

∋ Χr ) 1Ε ς6(n ∃ r ∃ 1) 3 2



r %1

1 2r ∃ n ) 1

(since (r – 1) ! = # when r = 0 so the term corresponding to r = 0 vanishes) #

=

()1) m ∃ 1 x 2 m ∃ 2 ) 1 xn 3 x) n 3 2 m ∃ 2 ∃ n ) 1 , (on changing the variable of summation to m = r –1 m ς6(n ∃ m ∃ 2) 2 m %0



so that r = m + 1. Then m = 0 when r = 1 and m = # when r = #) = – x )n

#



m%0

m

FH IK

()1) x m ς6(n ∃ m ∃ 2) 2

n ∃ 1 ∃ 2m

= ) x )n

#

∋ r %0

r

FH IK

()1) x r ς6(n ∃ 1 ∃ r ∃ 1) 2

n ∃ 1 ∃ 2r

(on changing the variable of summation from m to r) = –x–nJn + 1(x), by the definition of Jn + 1(x). III. Recurrence relation I is

or

or

d n {x Jn(x)} = xnJn – 1(x) dx Dividing both sides by xn – 1, (n/x) Jn(x) + Jn (x) = Jn – 1(x) IV. Recurrence relation II is d {x–n J (x)} = –x–n J n n + 1(x) dx Dividing both sides by x–n, (–n/x) Jn + J n = Jn + 1

or

n xn – 1Jn(x) + xnJ n(x) = xnJn – 1(x).

or

n Jn(x) + x J n(x) = x Jn – 1(x) J n(x) = Jn – 1(x) – (n/x) Jn(x)

or or

–n x–n – 1Jn(x) + x–n J n(x) = –x–nJn + 1(x) n x–1 Jn(x) + J n(x) = –Jn + 1(x) J n = (n/x) Jn – Jn + 1.

Created with Print2PDF. To remove this line, buy a license at: http://www.software602.com/

Bessel Functions

11.19

V. From recurrence relations III and IV, we have J n(x) =Jn – 1(x) – (n/x)Jn(x) and J n(x) = (n/x) Jn(x) – Jn + 1(x).

...(1) ...(2)

Adding (1) and (2), 2J n(x) = Jn – 1(x) – Jn + 1(x)

J n(x) =

or

1 2

{Jn – 1(x) – Jn + 1(x)}.

VI. From recurrence relations III and IV, we have J n(x) = Jn – 1(x) – (n/x)Jn(x) ...(1) J n(x) = (n/x) Jn(x) – Jn + 1(x). ...(2) Subtracting (2) from (1), we get 0 = Jn – 1 + Jn + 1 – 2(n/x) Jn or Jn – 1 + Jn + 1 = (2n/x) Jn. 11.7. A. Solved examples based on recurrence relations Ex. 1(a). Show that xnJn(x) is a solution of x(d2y/dx2) + (1 – 2n) × (dy/dx) + xy = 0. [CDLU 2004] (b) Show that x–nJn(x) is a solution of x(d2y/dx2) + (1 + 2n) × (dy/dx) + xy = 0. [MDU Rohtak 2005] Sol. (a) Given x(d2y/dx2) + (1 – 2n) × (dy/dx) + xy = 0 ... (1) Let y = xn Jn(x) ... (2) and

d n [ x J n ( x)] % x n J n )1 ( x) dx dy/dx = xn Jn–1, using (2) From (3), d2y/dx2 = xn J n–1 + n xn–1 Jn–1 Substituting the above values of y, dy/dx and d2y/dx2 in (1), we get

From recurrence relation I, or

x( x n J n)1 ∃ nxn )1 J n)1 ) ∃ (1 ) 2n) x n J n)1 ∃ xn ∃1 J n % 0

xn ∃1 J n )1 ) (n ) 1) xn J n )1 ∃ x n∃1 J n % 0

or

x n ∃1 +− J n )1 ) Κ( n ) 1) / xΛ J n )1 79 ∃ x n ∃1 J n % 0

or

... (3)

... (4)

From recurrence relation VI, we have J n ( x) %

n J n ) J n ∃1 ( x ) x

J n )1 )

so that

Using (5), (4) reduces to –xn + 1 Jn + xn + 1 Jn = 0, Hence xn Jn is a solution of (1).

i.e.,

n )1 J n )1 % ) J n ( x ) x

0 = 0, which is true.

x(d 2 y / dx 2 ) ∃ (1 ∃ 2n) ϑ (dy / dx) ∃ xy % 0

(b) Given

–n

y = x Jn(x)

Let From recurrence relation II,

Κ

... (5)

... (1) ... (2)

Λ

d )n x J n ( x ) % ) x ) n J n ∃1 ( x) dx

dy/dx = –x–nJn + 1, using (2)

or

d 2 y / dx2 % ) x )n J n ∃1 ∃ n x )n )1 J n∃1

From (3),

2

... (3)

2

Substituting the above values of y, dy/dx and d y/dx in (1), we get

x() x )n J n∃1 ∃ nx) n )1 J n ∃1 ) ∃ (1 ∃ 2n) ϑ () x) n J n ∃1 ) ∃ x) n ∃1 J n % 0 or ) x) n∃1 Jn ∃1 ) (n ∃ 1) x) n J n∃1 ∃ x)n∃1 J n % 0 or ) x ) n ∃1 +− J n ∃1 ∃ Κ(n ∃ 1) / xΛ J n ∃1 79 ∃ x ) n ∃1 J n % 0 ... (4) From recurrence relation III, we have J n ( x ) % J n )1 ( x) )

n Jn x

so that

J n ∃1 ∃

n ∃1 J n ∃1 % J n ( x ) x

... (5)

Created with Print2PDF. To remove this line, buy a license at: http://www.software602.com/

11.20

Bessel Functions –n + 1

Using (5), (4) reduce to –x Jn + x –n Hence x Jn is a solution of (1). Ex. 2. (Lommel theorems) Prove that

:1 d ; (i) < = > x dx ?

m

Χx J Ε % x n

n

n) m

–n + 1

Jn = 0,

0 = 0,

i.e.,

which is true.

J n )m , where m is positive integer and m < n.

m

:1 d ; )n m )n ) m Jn∃m (ii) < = ( x J n ) % ()1) x > x dx ? n

:1 d ; (iii) J n ( x ) % ()1)n x n < = J 0 ( x), n being positive integer.. > x dx ? Sol. (i) From recurrence relation I, we have d n ( x J n ) % x n J n )1 dx

:1 d ; n n )1 < = ( x J n ) % x J n )1 > x dx ?

so that m )1

m

... (1)

m)1

:1 d ; :1 d ; :1 d ; n :1 d ; n n)1 Now, < = ( x Jn ) % < = < = ( x J n ) % < x dx = ( x J n)1 ), using (1) x dx x dx x dx > ? > ? > ? > ? ....................................................................... = xn) m J n )m , on preceeding as before m times more (ii) From recurrence relation II, we have d )n ( x J n ) % ) x ) n J n ∃1 dx

so that

m

m)1

: 1 d ; )n 1 ) n )1 J n ∃1 < = ( x J n ∃1 ) % ()1) x > x dx ?

... (2)

m )1 :1 d ; : 1 d ; : 1 d ; )n )n :1 d ; Now, < ()1)1 x ) n )1 J n ∃1 , using (2) = ( x Jn ) % < = < = (x Jn ) % < = x dx x dx x dx > ? > ? > ? > x dx ? ....................................................................... = (–1)m x–n–m Jn+m, on proceeding as before m times more ... (3) (iii) Replacing n by 0 and m by n in part (ii), we get

n

n

:1 d ; n )n < = J 0 % ()1) x J n > x dx ?

Ex. 3. Prove that

z

1

0

or

t{Jn (t )}2 dt %

1 2

:1 d ; J n ( x) % ()1) n x n < = J 0 ( x) > x dx ?

x 2{Jn2 ( x) ) Jn ) 1( x ) Jn ∃ 1( x)}.

LM N

OP Q

2 Sol. We have, d t {Jn2 (t ) ) Jn ) 1(t ) Jn ∃ 1(t)} dt 2

= t{Jn2(t) – Jn – 1(t) Jn + 1(t)} +

1 t2 2

= t{Jn2(t) – Jn – 1(t) Jn + 1(t)} +

1 t2{2J (t) n 2

{2Jn(t) J n(t)–J ×

1 2

n – 1(t)Jn + 1(t)

– Jn – 1(t) J

n + 1(t)}

.n )1 / J n ) 1 (t ) ) J n (t ) 2 4 t 5

{Jn – 1(t) – Jn +1(t)} ) J n ∃ 1 1

n ∃1 . / ) J n ) 1 (t ) 1 J n (t ) ) J n ∃ 1 (t ) 2 , using recurrence relations III, IV and V t 4 5

= t Jn2(t), on simplification.

Created with Print2PDF. To remove this line, buy a license at: http://www.software602.com/

Bessel Functions

11.21

LM N

OP Q

2 tJn2(t) = d t {Jn2 (t ) ) Jn ) 1(t ) Jn ∃ 1(t)} . dt 2

(

...(1)

Integrating both sides of (1) w.r.t. ‘x’ from 0 to x, we get

z

x

0

t Jn2 (t ) dt =

LM t {J (t) ) J N2 2

2 n

OP Q

x

x2 {Jn2(x) – Jn – 1(x) Jn + 1(x)} n ) 1 (t ) J n ∃ 1(t )} = 2 0

Jn(0) = Jn + 1(0) = Jn – 1(0) = 0]

[

FH 2 IK FH ) cos x ) sin xIK ≅x x FH ≅x IK J FH 2 IK FH sin x ) cos xIK i.e. 2 ≅x x

Ex. 4. Prove that (i) J–3/2(x) =

[Kanput 2005, 10]

sin x ) cos x . x [Purvanchal 2005, Agra 2005, Kakitiya 1997; Kanpur 2008, 11; Bangalore 1997; Meerut 2007; KU Kurukshetra 2004] Sol. Proceed as in Ex. 1 of Art. 11.6A and prove that

(ii) J3/2(x) =

J–1/2(x) = and

or

=

(2 / ≅x ) cos x

...(1)

J1/2(x) = (2 / ≅x ) sin x. ...(2) Recurrence relation (VI) is Jn – 1(x) + Jn + 1(x) = (2n/x) × Jn(x). ...(3) Part (i). Replacing n by ) 12 in (3), we have J–3/2(x) + J1/2(x) = –(2/2x) × J–1/2(x)

FH 2 IK sin x – 1 FH 2 IK x ≅x ≅x FH 2 IK FH ) cos x ) sin xIK .

J–3/2(x) = –J1/2(x) – (1/x)× J–1/2 (x) = – =

≅x Part (ii). Replacing n by 12 in (3), we have

or

3/2(x)

cos x, by (1) and (2)

x

J–1/2(x) + J3/2(x) = (2/2x) × J1/2(x)

FH 2 IK cos x + 1 FH 2 IK sin x, by (1) and (2) x ≅x ≅x FG 2 IJ FG sin x ) cos xIJ . H ≅x K H x K

J3/2(x) = –J–1/2(x) + (1/x) × J1/2(x) = – =

Ex. 5. Prove that (i) J 5 / 2 ( x ) % (2 / ≅x )1/ 2

ΚΚ(3 ) x ) / x Λ sin x ) (3 / x) ϑ cos xΛ 2

2

[Kanpur 2006, 07] (ii) J )5/ 2 ( x) % (2 / ≅x )1/ 2

ΚΚ(3 ) x ) / x Λ cos x ∃ (3/ x) ϑ sin xΛ 2

2

[KU Kurukshetra 2006]

Sol. (i) From recurrence relation VI, we have Jn(x) = (x/2n) {Jn–1 (x) + Jn–1(x)}

or

J n ∃1 % (2n / x) J n ( x) ) J n )1 ( x )

... (1)

Putting n = 3/2 in (1), we have, J 5 / 2 ( x) %

3 3 2 : sin x 2 ; J 3/ 2 ( x) ) J1/ 2 ( x ) % ϑ ) cos x = ) sin x < x x ≅x > x ≅x ?

[using values of J3/2 (x) and J1/2(x) as obtained in Ex. 4 (ii), Art. 11.7A and Ex. 1 (ii), Art 11.6A] %

2 : 3sin x 3cos x 2 : 3 ) x2 3cos x ; ; ) sin x = % x x ≅x > x x =? ?

Created with Print2PDF. To remove this line, buy a license at: http://www.software602.com/

11.22

Bessel Functions

(ii) Re-writing (1), Jn–1(x) = (2n/x) Jn(x) – Jn + 1(x) Putting n = – 3/2 in (1), we have

... (2)

3 3 2 : cos x 2 ; J )5/ 2 ( x) % ) J )3/ 2 ( x) ) J )1/ 2 ( x) % ) ϑ ) sin x = ) cos x

x ≅x ?

[using values of J–3/2 (x) and J–1/2 (x) as obtained in Ex. 4 (i), Art. 11.7A and Ex. 1 (i), Art. 11.6A] 1/ 2

: 2 ; %< = > ≅x ?

1/ 2

: 3cos x 3sin x ; : 2 ; ∃ ) cos x = % < = < 2 x > x ? > ≅x ?

Ex. 6. Express J4(x) in terms of J0 and J1. Sol. Recurrence relation VI is Replacing n by 3 in (1), we get Now replacing n by 2 in (1), we get Using (3), (2) becomes

LM N

OP Q

6 4 J ( x ) ) J1( x ) ) J2 ( x ) x x 2 Next, replacing n by 1 in (1) gives Using (5), (4) becomes

J4(x) =

F H

1

0

Ex. 7(a). Prove that (i)

1 2

Jn + 1(x) = (2n/x)Jn(x) – Jn – 1(x). J4(x) = (6/x)J3(x) – J2(x). J3(x) = (4/x)J2(x) – J1(x).

F H

I K

J4(x) = 242 ) 1 J2 ( x ) ) 6 J1(x ) . x x J2(x) = (2/x)J1(x) – J0(x).

or

I L 2 J (x) ) J (x)O ) 6 J (x) PQ x K MN x

J4(x) = 242 ) 1 x

: 3 ) x2 3sin x ; x

1

F H

I K

F H

...(1) ...(2) ...(3)

...(4) ...(5)

I K

J4(x) = 483 ) 8 J1( x) ) 242 ) 1 J0 ( x ) . x x x

or

xJn = (n + 1)Jn + 1 – (n + 3)Jn + 3 + (n + 5)Jn + 5 ... .

(ii) Jn – 1 = (2/x) [n Jn – (n + 2)Jn + 2 + (n + 4)Jn + 4 – ...]. Proof (i). Recurrence relation VI is 2nJn = x(Jn – 1 + Jn + 1). Replacing n by n + 1 in the above relation, we get 2(n + 1)Jn + 1 = x(Jn + Jn + 2)

1 2

or 1 2

Replacing n by n + 2 in (1), we get Putting the value of 12 xJn + 2 from (2) in (1), Repalcing n by n + 4 in (1) gives Putting the value of 1 2

1 2

1 2

xJn = (n + 1) Jn

+1



1 2

xJn + 2 = (n + 3)Jn + 3 –

1 2

xJn + 4.

xJn

+ 2.

xJn = (n + 1)Jn + 1 – (n + 3)Jn + 3 + 12 xJn + 4. 1 2

xJn + 4 = (n + 5)Jn + 5 –

1 2

xJn + 6.

...(1) ...(2) ...(3)

...(4)

xJn + 4 from (4) in (3) gives

xJn = (n + 1)Jn + 1 – (n + 3)Jn + 3 + (n + 5)Jn + 5 –

1 2

xJn

+6

Proceeding likewise and noting that JnΑ 0 as n Α #, we get 1 2

xJn = (n + 1)Jn + 1 – (n + 3)Jn + 3 + (n + 5)Jn + 5 – ...

...(5)

(ii) Replacing n by n – 1 in (5) and then multiplying both sides by (2/x), we get Jn – 1 = (2/x) [nJn – (n + 2)Jn + 2 + (n + 4)Jn + 4 – ...] Ex. 7(b). Prove that Jn – 1 = (2/x)[nJn – (n + 2)Jn + 2 + (n + 4)Jn + 4 – ...] and hence deduce that (x/2)Jn = (n + 1)Jn + 1 – (n + 3)Jn + 3 – (n + 5)Jn + 5 + ... [Meerut 1993] Hint : Proceed as in Ex. 7(a). Ex. 8. Prove that J n = (2 / x ) (n / 2) Jn ) (n ∃ 2) Jn ∃ 2 ∃ (n ∃ 4) Jn ∃ 4 ) ... Sol. Recurrence relation III is xJ n = –nJn + xJn – 1 or J n = –(n/x)Jn + Jn – 1. From Ex. 7(a) part (ii), Jn – 1 = (2/x)[nJn – (n + 2)Jn + 2 + (n + 4)Jn + 4 – ...]

...(1) ... (2)

Created with Print2PDF. To remove this line, buy a license at: http://www.software602.com/

Bessel Functions

11.23

Putting the value of Jn – 1 from (2) in (1), we get Jn = –(n/x)Jn + (2/x) [nJn – (n + 2)Jn + 2 + (n + 4) Jn + 4 – ...] J n = (2 / x ) (n / 2) Jn ) (n ∃ 2) Jn ∃ 2 ∃ (n ∃ 4) Jn ∃ 4 ) ... .

or

F H

I K

d n ∃1 2 J (J 2 + J2n + 1) = 2 n Jn2 ) dx n x x n ∃1 Sol. From recurrence relation III and IV, we have J n = –(n/x)Jn + Jn – 1 and J n = (n/x)Jn – Jn + 1.

Ex. 9. Prove :

Replacing n by n + 1 in (1), we get d (J2n + J2n + 1) = 2Jn J n + 2Jn + 1J dx

FH

n +1=

J 2 Jn

FH n J x

n

n +1

= )

[Agra 2005; Meerut 1998] ...(1) ...(2)

n ∃1 J ∃ Jn . x n ∃1

F H

IK

) Jn ∃ 1 ∃ 2 Jn ∃ 1 )

...(3)

I K

n ∃1 J ∃ Jn , by (2) and (3) x n ∃1

IK

n∃1 2 J = 2 n Jn2 ) , on simplification. x x n ∃1 Ex. 10. Prove that (i) J02 + 2(J12 + J22 + J32 + ...) = 1. [Agra 2009, 10; Meerut 2008; Kanpur 2011] (ii) | J0(x) | Ω 1 [Agra 2009; Meerut 1996, 97, 98; Kanpur 2011] (iii) | Jn(x) | Ω 2–1/2, when n Ι 1. [Meerut 1996, 97, 98; Kanpur 2011]

FH

IK

d (J2 + J2 ) = 2 n J 2 ) n ∃ 1 J 2 . n n+1 x n x n ∃1 dx Replacing n by 0, 1, 2, 3 ... successively in (1), we get d (J 2 + J 2) = 2 0 ) 1 J 2 1 x 1 dx 0 d (J 2 + J 2) = 2 1 J 2 ) 2 J 2 2 x 1 x 2 dx 1 d (J 2 + J32) = 2 2 J22 ) 3 J32 dx 2 x x ... ... ... ... ... ... ... ... ... ... ... ... ... Adding these columnwise and noting that Jn Α 0 as n Α #, we get

Sol. (i) From Ex. 9 above,

FH FH FH

IK

...(1)

IK IK

d [J 2 + 2(J12 + J22 + ...)] = 0 dx 0 Integrating, J02(x) + 2[J12(x) + J22(x) + ...] = C. ...(2) Replacing x by 0 in (2) and noting that J0(0) = 1 and Jn(0) = 0 for n Ι 1, we get 1 + 2(0 + 0 + ...) = C or C = 1. Hence (2) becomes J02 + 2(J12 + J22 + ...) = 1 ...(3) Part (ii). From (3), J02 = 1 – 2(J12 + J22 + ... + J2n – 1 + J2n + J2n + 1 + ....) ...(4) 2 2 2 Since J1 , J2 , J3 ... are all positive or zero, (4) gives J02 Ω 1 so that | J0(x) | Ω 1. Part (iii). Solving (4) for Jn2, we have Jn2 = (1/2) × (1 – J02) – (J12 + J22 + ... + J2n – 1 + J2n + 1 + ...). ...(5) 2 Since J0 , J12, J22 ... are all positive or zero, (5) gives J2n Ω 1/2 or | Jn(x) | Ω 2–1/2, where n Ι 1. d {xJn(x) Jn + 1(x)} = x{Jn2(x) – J2n + 1(x)} dx [Agra 2009; Sager 2004; Meerut 2005; Kanpur 2007] (ii) x = 2J0J1 + 6J1J2 + ... + 2(2n + 1)Jn Jn + 1 + .... [Bilaspur 1998]

Ex. 11. Prove that (i)

Created with Print2PDF. To remove this line, buy a license at: http://www.software602.com/

11.24

Bessel Functions

d (xJn Jn + 1) = JnJn + 1 + x(J n Jn + 1 + JnJ n + 1) = JnJn + 1 + Jn + 1(xJ n) + Jn(xJ n + 1). ...(1) dx From recurrence relation III and IV, we have xJ n = nJn – xJn + 1 ...(2) and xJ n = –nJn + xJn – 1 ...(3) Replacing n by n + 1 in (3), xJ n + 1 = –(n + 1)Jn + 1 + xJn. ...(4) Putting the values of xJ n and xJ n + 1 from (2) and (4) in (1), we get d (xJn Jn + 1) = JnJn + 1 + Jn + 1(nJn – xJn + 1) + Jn[–(n + 1)Jn + 1 + xJn]= x(Jn2 – J2n + 1), other terms cancel. dx d Part (ii). From part (i) above we have (xJnJn + 1) = x(Jn2 – J2n – 1). dx Replacing n by 0, 1, 2, ... successively, we get

Sol. (i)

d (xJ0J1) = x(J02 – J12) dx d (xJ1 J2) = x(J12 – J22) dx d (xJ2 J3) = x(J22 – J23) dx ................ .. ................. Multiplying (1), (2), (3) ... by 1, 3, 5, ... respectively and adding, we have

...(1) ...(2) ...(3)

d [x(J0 J1 + 3J1J2 + 5J2J3 + ...)] = x[(J02 – J12) + 3(J12 – J22) + 5(J22 – J23) + ...] dx = x[J02 + 2(J12 + J2 + ...)] = x × 1 = x , by Ex. 10. (i)

Integrating,

2008]

z

x(J0 J1 + 3J1 J2 + 5J2J3 + ...) = xdx ∃ C %

1 2

x 2 ∃ C.

...(4)

Putting x = 0 in (4) gives C = 0. Putting C = 0 in (4) and simplifying, we get 2J0J1 + 6J1J2 + 10J2J3 + ... = x. Ex. 12. Prove that (i) J0 = –J1. [Agra 2010; Kanpur 2008] (ii) J2 – J0 = 2J!0. [Agra

(iii) J2 = J0! – (1/x)J0 . [Bilaspur 1997] (iv) J2 + 3J 0 + 4J0! = 0. Sol. (i) Recurrence relation IV is xJ n = nJn – xJn + 1. Replacing n by 0 in (1), we have xJ 0 = –xJ1 or J0 = –J1. Part (ii). Recurrence relation V is 2J n = Jn – 1 – Jn + 1. Differentiating (2), w.r.t. ‘x’, we get 2Jn! = J n – 1 – J n + 1 Replacing n by n – 1 and n + 1 successively in (2), we have 2J n – 1 = Jn – 2 – Jn and 2J n + 1 = Jn – Jn + 2. Putting the values of J n – 1 and J n + 1 from (4) and (5) in (3), we have 2Jn! =

1 2

(Jn – 2 – Jn) –

1 2

(Jn – Jn + 2)

4Jn! = Jn – 2 – 2Jn + Jn + 2.

or

...(1) ...(2) ...(3) ...(4) ...(5) ...(6)

Replacing n by 0 in (6), we have 4J 0 (

= J–2 – 2J0 + J2 = (–1)2J2 – 2J0 + J2

4J 0 = 2(J2 – J0)

or

[

J–n = (–1)n Jn]

2J0! = J2 – J0.

Part (iii). Replacing n by 1 in (1), we get xJ1 = J1 – xJ2 or J2 = x–1J1 – J1 . From part (i), J1 = –J 0 so that J 1 = –J0!. ( (7) gives J2 = x–1(–J 0) + J !0 = J0! – x–1 J0 .

...(7)

Created with Print2PDF. To remove this line, buy a license at: http://www.software602.com/

Bessel Functions

11.25

Part (iv). Differentating (6), we have 4Jn! = J n – 2 – 2J n + J n + 2. Replacing n by n – 2 and n + 2 successively in (2), we get 2J n – 2 = Jn – 3 – Jn – 1 and 2J n + 2 = Jn + 1 – Jn + 3. Putting the values of J n – 2, J n + 2 and J n from (9), (10) and (2) in (8), we get Jn = or

1 2

(Jn – 3 – Jn – 1) – (Jn – 1 – Jn + 1) +

1 2

8J0! + 2J3 + 6 J 0 = 0

or

...(9) ...(10)

(Jn + 1 – Jn + 3)

8Jn! = Jn – 3 – 3Jn – 1 + 3Jn + 1 – Jn + 3. Replacing n by 0 in (11), we get 8J0! = J–3 – 3J–1 + 3J1 – J3 = –2J3 + 6J1 = –2J3 – 6J0 (

...(8)

...(11) [

J–n = (–1)n Jn] [ J1 = – J 0]

J3 + 3 J 0 + 4J0! = 0.

Ex. 13. Show that Jn(x) = 0 has no repeated roots except at x = 0. Sol. If possible suppose Jn(x) = 0 has repeated roots; then at least two roots must be equal (say Σ), that is, Σ is a double root of Jn(x) = 0. Then from the theory of equations, we have Jn(Σ) = 0 and J n(Σ) = 0. ...(1) Recurrence relations III and IV, Jn – 1(x) = (n/x)Jn(x) + J n(x). ...(2) and Jn + 1(x) = (n/x) Jn(x) – J n(x). ...(3) Replacing x by Σ in (2) and (3) and using (1), we get Jn + 1(Σ) = 0 and Jn – 1(Σ) = 0 except when x = 0. Since two different power series have distinct sum functions, so Jn + 1(Σ) = 0 = Jn – 1(Σ) must be absurd. Hence Jn(x) = 0 has no repeated roots except at x = 0. Ex. 14. From the recurrence formula 2Jn = Jn – 1 – Jn + 1, deduce the result 2rJnr(x) = Jn – r – r Jn – r + 2 +

r(r ) 1) Jn ) r ∃ 4 ∃ ... ∃ ()1)r Jn ∃ r . 2!

...(1)

Sol. Given that 2J n = Jn – 1 – Jn + 1. ...(2) Clearly (2) shows that (1) is true for r = 1. We assume that (1) is true for some particular value of r, say r = p. Then we have 2 p J pn(x) = Jn – p – p Jn – p + 2 + or

p ( p ) 1) J n ) p ∃ 4 ∃ ... ∃ ()1) p J n ∃ p 2!

2pJnp(x) = Jn – p – pC1 Jn – p + 2 + pC2 Jn – p + 4 + ... + (–1)p Jn + p. ...(3) Differentiating (3) w.r.t. x and then multiplying by 2, we get 2p + 1Jnp + 1(x) = 2J n – p – 2 × pC1Jn – p + 2 + 2 × pC2 J n – p + 4 + ... + 2(–1)p J n + p. ...(4) Replacing n by n – p, n – p + 2, n – p + 4 ..., n + p successively in (2), 2J n – p = Jn – p – 1 – Jn – p + 1 2J n – p + 2 = Jn – p + 1 – Jn – p + 2 2J n – p + 4 = Jn – p + 3 – Jn – p + 5 ......... .... .... .... 2J n + p = Jn – p – 1 – Jn + p + 1 Substituting these values in (4), we have 2p + 1Jnp + 1(x) = Jn – p – 1 – Jn – p + 1 – pC1(Jn – p + 1 – Jn – p – 3) + pC2(Jn – p + 4 – Jn – p + 5) + ... + (–1)p (Jn – p – 1 – Jn + p + 1)

Created with Print2PDF. To remove this line, buy a license at: http://www.software602.com/

11.26

Bessel Functions p

p

p

p +1

= Jn – p – 1 – (1 + C1)Jn – p + 1 + ( C1 + C2)Jn – p + 2 + ... + (–1) Jn + p + 1. ...(5) p p p+1 p p p p+1 p p p+1 Since Cr + Cr – 1 = Cr, we have 1 + C1 = C0 + C1 = C1, C1 + C2 = C2 and so on. Then (5) becomes 2p + 1J n p + 1(x) = J n – p – 1 – p + 1C1 Jn – p + 1 + p + 1C2 J n – p + 2 + ... + (–1)p + 1 J n + p + 1, showing that (1) is true for r = p + 1 if it were true for r = p. So (1) is true for all natural numbers by mathematical induction. Ex. 15. Prove that x2J!n(x) = (n2 – n – x2)Jn(x) + xJn + 1(x), where n = 0, 1, 2, .... Sol. Recurrence relation IV is xJn (x) = nJn(x) – xJn + 1(x). ...(1) Differentiating both sides of (1) w.r.t. ‘x’, we have xJ !n(x) + J n(x) = nJ n(x) – [xJ n + 1(x) + Jn + 1(x)] or x2J !n(x) = (n – 1)xJ n(x) – x[xJ n + 1(x)] – xJn + 1(x). ...(2) Recurrence relation III is xJ n(x) = –nJn(x) + xJn + 1(x) Replacing n by (n + 1) in this relation, we obtain xJ n + 1(x) = – (n + 1) Jn + 1(x) + x Jn(x). ...(3) Substituting for xJ n from (1) and for xJ n + 1(x) from (3) in (2), we get x2J !n(x) = (n – 1) [n Jn(x) – xJn + 1(x) – x[– (n + 1)Jn + 1(x) + xJn(x)] – x Jn + 1(x) = [(n – 1)n – x2]Jn(x) + [– (n – 1) + (n + 1) – 1]xJn + 1(x) ( x2J !n(x) = (n2 – n – x2) Jn(x) + x Jn + 1(x). 2 3 Jn ∃ 1 ( x / 2) ( x / 2) ( x / 2) % ... Jn (n ∃ 1) ) (n ∃ 2) ) (n ∃ 3) )

Ex. 16. Show that*

Jn – 1 + Jn + 1 = (2n/x) × Jn

Sol. Recurrence relation VI is Jn – 1 = 2n Jn – Jn + 1 x

or

Replacing n by (n + 1) in (1), we get (

Jn ∃ 1 = Jn

1 Jn Jn∃ 1

=

Jn ) 1 2n Jn ∃ 1 . % ) Jn x Jn

or

%

Jn Jn ∃ 1

=

...(1)

2(n ∃ 1) Jn ∃ 2 ) n Jn ∃ 1

...(2)

1 , using (2) 2(n ∃ 1) J n ∃ 2 ) x Jn ∃ 1

1 1 % 2(n ∃ 1) 2(n ∃ 1) 1 1 ) ) x ( Jn ∃ 1 / Jn ∃ 2 ) x 2(n ∃ 2) Jn ∃ 3 ) x Jn ∃ 2 [With help of (2) by replacing n by n + 1]

*Student should read a chapter on continued fractions in some book on Algebra to understand the

a1

notations, namely,

a2 ∃

a3 a4 ∃

a5 a6 ∃ ...

%

a1 a3 a2 ∃ a4 ∃

a5 a6 ∃ ...

Created with Print2PDF. To remove this line, buy a license at: http://www.software602.com/

Bessel Functions

11.27

=

1 2(n ∃ 1) 1 ) 2(n ∃ 2) 1 x ) x ( Jn ∃ 2 / Jn ∃ 3 )

=

1 2(n ∃ 1) 1 ) 2(n ∃ 2) 1 x ) x 2(n ∃ 3) Jn ∃ 4 ) x Jn ∃ 3

=

=

x/2 x/2 ( n ∃ 1) ) 2(n ∃ 2) 1 ) 2 ( n ∃ 3) x ∃ ... x

+ Multiply numerator and 7 , denominator by x/2 8 − 9

x/2

L OP ( x / 2) (n ∃ 1) ) M ( x / 2) MM (n ∃ 2) ) 2(n ∃ 3) ∃ ... PP x N Q 2

2

=

+ With help of (2) by 7 ,replacing n by n + 2 8 − 9

( x / 2) ( x / 2) (n ∃ 1) ) (n ∃ 2) )

Repeating the LMsimilar O N operationsQP

3

( x / 2) . (n ∃ 3) ) 333

EXERCISE 11 (B) 1. Show that all roots of Jn(x) are real. 2. Prove that between any two zeros of Jn(x) lie one and only one zero of Jn + 1(x) as well as Jn – 1(x). 3.(a) Prove that between any two consecutive positive roots of the equation Jn(x) = 0, there is one and only one root of the equation Jn + 1(x) = 0. (b) Show that between two consecutive positive zeros of Jn (x) there is precisely one zero of Jn–1 (x). 4. Prove (i) Jn + 3 + Jn + 5 = (2/x) (n + 4)Jn + 4. [Kanpur 2009] (ii) 4J!n = Jn – 2 – 2Jn + Jn + 2. 5. For Bessel’s functions Jn(x), find out a and b, where d{Jn (x)}/dx = aJn – 1(x) + bJn + 1(x). Ans. a = 1/2; b = –(1/2) 6. Show that J 2 ) J 0 % 2 J 0

[Kanpur 2006] 2

2

7. Evaluate J3(x) in terms of J0(x) and J1(x). Ans. J3(x) = {(8 – x )/x }J1(x) – (4/x)J0(x) 8. Prove that (a) x2J !n(x) + J n(x) = (n2/x)Jn(x) – xJn(x). [Meerut 1998]

z

z

(b) Jn ∃ 1( x) dx = Jn ) 1( x ) dx ) 2 Jn( x ).

[Kanpur 1998]

11.7.B. Solved Example involving integration and recurrence relations Ex. 1. If n > –1, show that

z

x

0

x n ∃ 1 Jn ( x ) dx % x n ∃ 1 Jn ∃ 1( x ).

Sol. From recurrence relations I,

d n {x Jn(x)} = xnJn – 1(x). dx

d n+1 {x Jn + 1(x)} = xn + 1Jn(x). dx Integrating (2) w.r.t. ‘x’ between the limits 0 and x, we get

Replacing n by n + 1 in (1),

[Bilaspur 1998] ...(1) ...(2)

Created with Print2PDF. To remove this line, buy a license at: http://www.software602.com/

11.28

Bessel Functions

Ν

[ x n ∃1 J n ∃1 ( x)]0x =

x

0

x n ∃ 1 J n ( x ) dx

Ex. 2. Prove that

Jn + 1(x) = x

Sol. Let

xy = t

( R.H.S. of (1) = x

z

x J (t ) (t 0 n

= x–n – 1

z

x

0

z

1

0

/ x)

z

or

x

0

x n ∃ 1 Jn ( x ) dx = xn + 1Jn + 1(x).

Jn ( xy) y n ∃ 1dy .

x dy = dt

so that

n )1

(dt / x) %

z

)n ) 1 x n ∃ 1 x t Jn (t ) 0

dt

x n ∃ 1 Jn ( x) dx % x ) n )1 x n ∃ 1 Jn ∃ 1( x ), by Ex. 1

= Jn + 1(x) = L.H.S. of (1). Ex. 3. Show that (a) (b)

z

#

0

z

x

0

x )n Jn ∃ 1( x) dx %

1 ) x )n Jn , n Υ 1. n 2 6(n ∃ 1)

x )n Jn ∃ 1( x) dx %

[Bilaspur 1997]

1 ,nΥ ) 1. 2 2n 6(n ∃ 1)

d [x–nJ (x)] = –x–nJ (x). n n+1 dx Integrating (1) w.r.t. ‘x’ between the limits 0 and x, we get

Sol. (a) From recurrence relation II,

x ) n Jn ( x )

x 0

But

z

x

)n = ) x Jn ∃ 1(x) dx 0

or x )n Jn ( x ) ) xlim Α0

Jn ( x ) = ) xn

x Α0

z

x

0

x )n Jn ∃ 1(x ) dx =

Part (b). Integrating (1) w.r.t. ‘x’ from 0 to #, we get # 0

x

0

x )n Jn ∃ 1( x ) dx.

z

= )

#

0

)n

x Jn ∃ 1(x) dx

lim

or

x Α#

1 ) x ) n Jn ( x ) . 2n 6(n ∃ 1)

Jn ( x ) Jn ( x ) =) n ) xlim Α 0 x xn

z

#

0

x )n Jn ∃ 1( x ) dx .

J n ( x)

= n 1 . 2 6(n ∃ 1) xn We know that for large values of x the approximate value of Jn(x) is As in part (a),

lim

x Α0

Jn(x) ~

FH 2 IK ≅x

1/ 2

RS FH T

cos x ) n ∃ lim

Using (5),

x Α#

Using (4) and (6), (3) reduces to Ex. 4. Prove (i)

...(2)

+ 7 1 xn x2 Jn ( x) ∃ ...8 = n 1 = lim n n ,1 ) n x Α0 x 2 6( n ∃ 1) 2 3 2( n ∃ 1) x 2 6(n ∃ 1) − 9

lim

Hence (2) may be written as

x )n J n ( x )

z

...(1)

d {xJ1(x)} = xJ0(x). dx

Sol. (i) Recurrence relation I is Replacing n by 1 in (1), we have

z

#

0

IK UV W

1 ≅ 1 ,nΥ 2 2 2

...(6)

x Jn ∃ 1(x ) dx =

(ii)

z

b

0

...(4)

...(5)

Jn ( x ) = 0. xn )n

...(3)

1 . 2n 6(n ∃ 1)

xJ0 (ax ) dx %

b J (ab). a 1

d n {x Jn(x)} = xnJn – 1(x). dx d {xJ (x)} = xJ (x). 1 0 dx

...(1)

Created with Print2PDF. To remove this line, buy a license at: http://www.software602.com/

Bessel Functions

11.29

Part (ii). Put ax = t, so that adx = dt. Then, we get

z

z

b

z

ab ab d x J0 (ax) dx = 12 t J0 (t ) dt = 12 {t J1(t )} dt, by part (i) 0 a 0 a 0 dt ab 1 = 2 tJ1(t ) 0 = 12 [abJ1(ab) – 0], as J1(0) = 0. a a

z

(

0

z

b

a

x J0 (ax ) dx = b J1(ab). a

(i) d J0(x) = –J1(x). dx

Ex. 5. Prove that (ii)

b

J0 ( x) J1(x ) dx =

1 [ J 2 (a) 2 0

) J02 (b)] .

[Nagpur 2005] d –n {x Jn(x)} = –xnJn + 1(x). dx d J (x) = –J1(x). dx 0

Sol. (i) From recurrence relation II, Put n = 0 in (1). Then Part (ii). Using (2), we have

z

z

...(1) ...(2)

b

b + [ J ( x )]2 7 2 2 1 J0 ( x) J1(x ) dx = ) J0 ( x ) J0 ( x) dx = ) , 0 8 % 2 [ J 0 (a) ) J 0 (b)] a a 2 −, 98 a b

z

Ex. 6. Evaluate J 3 ( x ) dx and express the result in terms of J0 and J1. x–nJn + 1 =

Sol. From recurrence II, we have

zx

Integrating it,

)n

Jn ∃ 1 dx = –x–nJn.

...(1)

z

z

z

d –n {x Jn(x)}. dx

Now, J 3 ( x ) dx = x 2 ( x )2 J3 ) dx = x2(–x–2J2) – 2 x () x )2 J2 ) dx. [Integrating by parts and using (1) for n = 2]

z

= –J2 + 2 x )1 J2 dx = –J2 + 2(–x–1J1) + c

z

[using (1) for n = 1]

–1

J 3 ( x ) dx = –J2 – 2x J1 + c. ( From recurrence relation VI, (2n/x)Jn = Jn – 1 + Jn + 1 Put n = 1 in (3). Then J2 = 2J1/x – J0

z J ( x ) dx = –(2J /x – J ) – 2x J + c z J ( x ) dx = J – 4J /x + c, being an arbitrary constant.

Using (4), (2) gives

3

(

3

z

1

0

0

–1

2

1

3

Ex. 7. Evaluate x J3( x ) dx.

Now,

Ν x J ( x) dx = Ν x 3

z

[Gulbarga 2005]

d {x–nJ } = –xnJ n n+1 dx

Sol. Since 3

5

...(2) ...(3) ...(4)

so

zx

)n

Jn ∃ 1 dx = –x–nJn.

...(1)

z

( x )2 J 3 ) dx = x5(–x–2J2) – 5x 4 () x )2 J2 ) dx

z

(On integration by parts and using (1) for n = 2)

Ν

3 )1 2 )1 = –x3J2 + 5 x 2 J 2 dx % ) x 3 J 2 ∃ 5 x 3 ( x )1 J 2 ) dx = –x3J2 + 5 +,− x () x J1 ) ) 3x () x J1 ) dx 789

(on integration by parts and using (1) for n = 1)

Created with Print2PDF. To remove this line, buy a license at: http://www.software602.com/

11.30

Bessel Functions

z – 15 z x J

z

= –x3J2 – 5x2J1 + 15 x J1 dx = –x3J2 – 5x2J1 + 15 x () J0 ) dx 3

2

3

3

= –x J2 – 5x J1

0

3

[

z

2

J1 = –J0 ]

dx = –x J2 – 5x J1 – 15 x J0 ) 1 . J0 dx ,integrating by part,

z

= –x J2 – 5x J1 – 15xJ0 + 15x J0 dx .

z

Remark. From Ex. 6. and 7 note that, in general, an integral of the form x m Jn ( x ) dx, m ∃ n Ι 0, can be completely integrated if m + n is an odd integer, while if m + n is even, then the integral can be put in terms of J0 ( x ) dx . Note that J0 ( x ) dx cannot be expressed in closed form and so it must be left as such in final answer.

z

z

z

Ex. 8. Evaluate x 4 J1( x ) dx. d {xnJ } = xnJ n n–1 dx

Sol. Since

z x J dx = z x 4

Now

1

2

zx J n

so

dx = xnJn.

...(1)

z

( x 2 J1) dx = x2(x2J2) – 2 x( x 2 J2 ) dx [on integrating by parts and using (1) for n = 2]

z

3 = x4J2 – 2 x J2 dx = x4J2 – 2x3J3 + c

z

n)1

[using (1) for n = 3]

Ex. 9. Express x )3 J4 ( x) dx in terms of J0 and J1. d –3 d {x–nJ } = –x–nJ –3 n n + 1 gives dx {x J3} = –x J4. dx

Sol. Putting n = 3 in recurrence relation II

z

x )3 J4 ( x) dx = –x–3J3 + c, c being an arbitrary constant Integrating, Recurrence relation VI is Jn + 1 = (2n/x)Jn – Jn – 1. Replacing n by 2 and 1 successively in (2) gives J3 = (6/x)J2 – J1 J2 = (4/x)J1 – J0. J2 = (6/x)[(4/x)J1 – J0] – J1 = (24x–2 – 1)J1 – (6/x)J0

and Using (4), (3) becomes Using (5), (1) becomes

z

zx

z

)3

...(1) ...(2) ...(3) ...(4) ...(5)

J4 ( x) dx = –x–3[(24x–2 – 1)J1 – 6x–1J0] + c.

Ex. 10. Prove Jn ∃ 1 dx = Jn ) 1( x ) dx ) 2 Jn ( x ). Sol. From recurrence relations, we have 2J n(x) = Jn – 1(x) – Jn + 1(x) Integrating

z

zJ

n ∃ 1( x )

z

Jn + 1(x) = Jn – 1(x) – 2J n(x)

or

dx = Jn ) 1( x ) dx ) 2 Jn ( x ).

Ex. 11(a). Prove that x )1 J 4 ( x) dx = –x–1J3(x) – 2x–2J2(x) + c. d –n {x Jn(x) = –x–nJn + 1(x). dx

Sol. Recurrence relation II is

zx

Integrating it, we get

)n

Jn ∃ 1( x ) dx % ) x )n Jn ( x ).

z

z

...(1)

Ν

2 )3 )1 ( x J4 ( x ) dx = x [x J4 ( x)] dx = x 2 + ) x)3 J 3 ( x) 7 ) 2 x ϑ [) x)3 J 3 ( x)] dx − 9 (Integrating by parts taking x2 as first function and using result (1) for n = 3)

z

= –x–1J3(x) + 2 x )2 J3 ( x ) dx = –x–1J3(x) + 2[–x–2J2(x)] + c, using result (1) for n = 2 = –x–1J3(x) – 2x–2J2(x) + c. Ex. 11(b). Prove that

Ν J ( x)dx % ) J ( x) ) (2 / x) ϑ J ( x) ∃ C 3

2

0

[KU Kurukshetra 2005]

Created with Print2PDF. To remove this line, buy a license at: http://www.software602.com/

Bessel Functions

11.31

Κ

Λ

d )n x J n ( x ) % ) x ) n J n ∃1 ( x) dx

Sol. From recurrence relation II,

Ν x J ( x)dx % ) x J ( x) Now, Ν J ( x)dx % Ν x Κ x J ( x)Λ dx % x Ν x J ( x)dx ) Ν 2 x Χ Ν x J ( x)dx Ε dx % x Χ ) x J ( x ) Ε ) 2 x () x J ( x))dx, using (1) for n = 2 Ν % ) J ( x) ∃ 2Ν x J ( x)dx % ) J ( x ) ) 2 x J ( x) ∃ C , using (1) again for n = 1 )n

Integrating,

)2

2

)2

2

)1

2

zx

1 3 0

x 3 0

)2

3

... (1)

3

1

J0 ( x) dx = x3J1(x) – 2x2J2(x).

[GATE 2003]

J0 ( x ) dx = 2J0(1) – 3J1(1). d {xnJ (x)] = xnJ (x) n n–1 dx

Sol. (i) Since (

)1

2

zx

)2

2

3

n

2

2

Ex. 12. Show that (i) (ii)

)2

2

3

)n

n ∃1

zx J n

so

n )1( x )

dx = xnJn(x) ...(1)

z x J (x) dx = z x [x J (x)] dx % [x {xJ (x)}] ) z 2x {x J (x)} dx x 3 0 0

x 2 0

3

= x J1(x) – 2

2

0

x 0

1

x 0

1

(Integrating by parts and using (1) for n = 1)

z

x 2 x J1( x ) 0

3

dx = x J1(x) – 2[x2J2(x)]x0, using result (1) for n = 2

= x 3 J1 ( x) ) 2 x 2 J 2 ( x), as J2 (0) % 0

z x J (x) dx = x J (x) – 2x J (x). x 3 0 0

(ii) Proceed as in part (i) and prove that

3

2

1

2

z x J (x) dx = J (1) – 2J (1). 1 3 0 0

Putting x = 1 in (2),

1

...(2) ...(3)

2

Recurrence relation VI is Jn – 1(x) + Jn + 1(x) = (2n/x)Jn(x). Putting n = 1 and re–writing it, we have J2(1) = 2J1(1) – J0(1) Substituting the above value of J2(1) in (3), we have

...(4)

z x J (x) dx = J (1) – 2{2J (1) – J (1)] = 2J (1) – 3J (1). 1 3 0 0

1

1

0

0

1

EXERCISE 11(C)

z 2. Prove that z

2

z

2

1. Prove that x J0 ( x) dx = x J1(x) + xJ0(x) – J0 ( x) dx . x 2 x J0 ( x) J1( x ) dx 0

=

1 2

x2J12(x).

z

3. Prove that x )2 J1( x) dx = ) 13 x )1 J2 ( x) ) 13 J1( x) ∃

z

1 2

z

J0 ( x) dx .

4. Prove that J0 ( x) sin x dx = xJ0(x) sin x – xJ1(x) cos x + c.

Ν

5. Show that J 3 dx % )3J 0 ∃ 4 J1

[Bilaspur 1997]

11.8. Generating function for the Bessel’s function Jn(x)

RS xFH z ) 1IK UV % ∋ z J (x) z W T #

Prove : exp Or

1 2

n % )#

n

n

[Kanpur 2007; Meerut 1996, 97]

Show that when n is a positive integer, Jn(x) is the coefficient of zn in the expansion of

Created with Print2PDF. To remove this line, buy a license at: http://www.software602.com/

11.32

Bessel Functions

exp {( x / 2) ϑ ( z ) 1/ z )} i.e., e(x/2) (z – 1/z) in ascending and descending power of z. Also show that Jn is coefficient of z–n multiplied by (–1)n in the expansion of the above expression. [Kanpur 2007; Kakitiya 1997; Kanpur 2005, 06] Note. Exp{(x/2) × (z – 1/z)} is called the generating function for Jn(x). Here exp A = eA. xz

exp {(x/2) × (z – 1/z)} = e 2

Proof. We have

)

x 2z

xz

% e 2 .e

)

x 2z

2 2 n n n ∃1 + :x; 7 + zn ∃1 :x; z :x; z :x; x 1 ∃ z ∃ ∃ ... ∃ ∃ ∃ ...8 ϑ ,1 ) :< ;= z )1 < = < = < = = , ? > ? > ? ,− > ? 89 − > 2 ? 2

n

)2 n )n :x; z : x ; ()1) z :x; ∃< = ∃ ... ∃ < = ∃< = 2 2! 2 n ! >2? > ? > ?

n ∃1

7 ()1) n ∃ 1 z ) ( n ∃ 1) ∃ ...8 ...(1) (n ∃ 1) ! 89

The coeff. of zn in the product (1) is obtained by multiplying the coefficients of zn, zn + 1, , ... in the first bracket with the coeff. of z0, z–1, z–2, ... in the second bracket respectively

n+2

z

FH IK

1 1 FH IK ∃ F xI ) ... (n ∃ 1) ! H 2 K (n ∃ 2) ! 2 ! ()1) F x I % ∋ ()1) FH x IK = J (x) =∋ r ! (n ∃ r ) ! H 2 K r ! 6(n ∃ r ∃ 1) ! 2

x n ( coefficient of z in product (1) = 2 #

n

n∃2

1 ) x n! 2

n ∃ 2r

r

r %0

n∃4

#

r

n ∃ 2r

n

r %0

[ (n + r) ! = 6(n + r + 1), n + r being positive integer]. The coeff. of z in the product (1) is obtained by multiplying the coefficients of z–n, z–n – 1, z– n – 2... of the second bracket with the coefficients of z0, z1, z2, ... in the first bracket respectively –n

n

n : x ; ()1) :x; ∃< = ( coeff. of z in product (1) = < = > 2? n! >2? –n

n ∃1

()1)n ∃ 1 x : x ; ∃< = (n ∃ 1) ! 2 > 2 ?

n∃2

2

()1)n ∃ 2 : x ; < = ∃ ... (n ∃ 2) ! 2 ! > 2 ?

n∃ 4 +: x ;n 1 : x ;n ∃ 2 1 7 1 :x; % ()1)n ,< = )< = ∃< = ...8 = (–1)nJ (x), as before n (n ∃ 1)! > 2 ? (n ∃ 2) 2! 89 ,−> 2 ? n ! > 2 ?

Thus the coefficient of z–n = (–1)nJn(x)

Jn(x) = (–1)n × the coefficient of z–n. Ρ Finally, in the product (1) the coefficient of z0 is obtained by multiplying the coefficient of z0, 1 2 z , z , ... in the first bracket with the coefficients of z0, z–1, z–2, ... in the second bracket and is thus 2

4

2

6

2

x2 x4 : x; : x; : 1 ; : x; : 1 ; = 1 – < = ∃ < = < = ) < = < = ∃ ... % 1 ) 2 ∃ 2 2 ) ... = J0(x). 2 2 34 > 2? > 2 ? > 2 ! ? > 2 ? > 3 !? We observe that the coefficients of z0, (z – z–1), (z2 + z–2) ..., [zn + (–1)nz–n] ... are J0(x), J1(x), J2(x) ..., Jn(x) ... respectively. Thus, (1) gives exp{(x/2) × (z – 1/z)} = J0(x) + (z – z–1)J1(x) + (z2 + z–2)J2(x) + ...+ [z2 + (–1)nz–n]Jn(x) + ... #

n

n

= ∋ z Jn ( x), as J)n ( x) % ()1) Jn ( x ) n %) #

Created with Print2PDF. To remove this line, buy a license at: http://www.software602.com/

Bessel Functions

11.33

11.9. Trigonometric expansions involving Bessel’s functions. Show that (i) cos (x sin Ξ) = J0 + 2 cos 2Ξ 3 J 2 + 2 cos 4Ξ 3 J 4 + ... [Meerut 1995, KU Kurukshetra 2005] (ii) sin (x sin Ξ) = 2 sin Ξ 3 J1 + 2 sin 3Ξ3 J 2 + ...

[KU Kurukshreta 2006]

(iii) cos (x cos Ξ) = J0 – 2 cos 2Ξ 3 J 2 + 2 cos 4Ξ 3 J 4 – ... (iv) sin (x cos Ξ) = 2 cos Ξ3 J1 – 2 cos 3Ξ3 J 3 + 2 cos 5Ξ3 J 5 – ... #

(v) cos x = J0 – 2J4 + 2J4 .... = J0 ( x ) ∃ 2 #

(vi) sin x = 2J1 – 2J3 + 2J5 ... = 2

∋ ()1) n% 0

∋ ()1)

n

[Kanpur 2011]

J2n ( x)

n %1

n

J2n ∃ 1( x )

[Kanpur 2011]

Proof. We know that e(x/2)(z – 1/z) = J0 + (z – z–1) J1 + (z2 + z–2) J2 + (z3 – z–3) J3 + ...

...(1)

Let z = eiΞ so that zn = einΞ and z–n = e–inΞ. Then (1) gives iΞ ) iΞ iΞ –iΞ 2iΞ –2iΞ 3iΞ –3iΞ e( x / 2) ( e )e ) = J0 + (e – e ) J1 + (e + e ) J2 + (e – e ) J3 + ... Since cos nΞ = (eniΞ + e–niΞ)/2 and sin nΞ = (eniΞ – e–niΞ)/2i, (2) gives

...(2)

exi sin Ξ = J0 + 2i sin Ξ 3 J1 + 2 cos 2Ξ 3 J 2 + 2i sin 3Ξ3 J 3 + ... or

cos (x sin Ξ) + i sin(x sin Ξ) = (J0 + 2 cos 2Ξ 3 J 2 + ...)+ 2i( sin Ξ 3 J1 + sin 3Ξ3 J 3 + ...)

...(3)

Part (i). Equating real parts in (3), we get cos (x sin Ξ) = J0 + 2 cos 2Ξ 3 J 2 + 2 cos 4Ξ 3 J 4 + ... Part (ii). Equating imaginary parts in (3), we get

...(4)

sin (x sin Ξ) = 2 sin Ξ 3 J1 + 2 sin 3Ξ3 J 3 + 2 sin 5Ξ3 J 5 + ... Part (iii). Replacing Ξ by ≅/2 – Ξ in (4) and simplifying, we have

...(5)

cos (x cos Ξ) = J0 – 2 cos 2Ξ 3 J 2 + 2 cos 4Ξ 3 J 4 – ... Part (iv). Replacing Ξ by ≅/2 – Ξ in (5) and simplifying, we get

...(6)

sin (x cos Ξ) = 2 cos Ξ3 J1 – 2 cos 3Ξ3 J 3 + 2 cos 5Ξ 3 J 5 – ... Part (v) & (vi). Replacing Ξ by 0 in (6) and (7), we get

...(7)

#

∋ ()1)

n

cos x = J0 – 2J1 + 2J4 – ... = J0 ( x ) ∃ 2 #

and

sin x = 2J1 – 2J3 + 2J5 – ... = 2

n %1

∋ ()1) n% 0

J2n ( x)

n

J2n ∃ 1( x ) .

11.9A. Solved examples based on Art 11.8 and 11.9. Ex. 1. Show that (i) x sin x = 2(22J2 – 42J4 + 62J6 – ...). [Agra 2010] 2 (ii) x cos x = 2(1 J1 – 32J3 + 52J5 – ...). Sol. We know that cos (x sin Ξ) = J0 + 2J2 cos 2Ξ + 2J4 cos 4Ξ + ... ...(1) Differentiating (1) w.r.t. ‘Ξ’, – sin (x sin Ξ) 3 x cos Ξ = 0 – 2 3 2J 2 sin 2Ξ – 2 3 4J 4 sin 4Ξ + ... ...(2) Differentiating (2) w.r.t. ‘Ξ’, ) cos( x sin Ξ) 3 ( x cos Ξ)2 ∃ sin( x sin Ξ) 3 ( x sin Ξ) = )2 3 22 J 2 cos 2Ξ – 2 3 42 J 4 cos 4Ξ – 2 3 62 J 6 cos 6Ξ + ... ...(3) Replacing Ξ by ≅/2 in (3), we get x sin x = 2(22J2 – 42J4 + 62J6 – ...)

Created with Print2PDF. To remove this line, buy a license at: http://www.software602.com/

11.34

Bessel Functions

sin (x sin Ξ) = 2J1 sin Ξ + 2J3 sin 3Ξ + 2 J 5 sin Ξ ∃ ...

Part (ii). Start with

Differentiate this twice w.r.t. ‘Ξ’ as in part (i) and then replace Ξ by ≅/2. This will lead to the desired answer. Complete the solution yourself. Ex. 2. Bessel’s Integrals. Show that

z z z

1 ≅ cos (nΞ ) x sin Ξ) dΞ , where n is a positive integer ≅ 0 [Purvanchal 2004, 07; Punjab 2005] ≅ 1 cos (nΞ ) x sin Ξ) dΞ , where n is any integer (ii) Jn(x) = [KU Kurukshetra 2004] ≅ 0 1 ≅ 1 ≅ cos ( x sin Ξ) dΞ % cos ( x cos Ξ) dΞ . (iii) J0(x) = [Agra 2006] ≅ 0 ≅ 0

(i) Jn(x) =

z

(iv) Deduce that J0(x) = 1 )

x

2



#

x4

) ... %

22 22 3 42 Sol. (i). We shall use the following results :

z



cos mΞ cos nΞ dΞ =

0

z

()1)r x 2 r

∋ (2 r %0

r

3 r !)2



.

U| V 0 when m & n |W

sin mΞ sin nΞ dΞ % ≅ / 2 when m % n

0

%

cos (x sin Ξ) = J0 + 2J2 cos 2Ξ + 2J4 cos 4Ξ + ... sin (x sin Ξ) = 2J1 sin Ξ + 2J3 sin 3Ξ + 2J5 sin 5Ξ + ...

and

...(1) ...(2)

...(3) Multiplying both sides of (2) by cos nΞ and then integrating w.r.t. ‘Ξ’ between limits 0 to ≅ and using (1), we have

z



cos ( x sin Ξ) cos nΞ dΞ = 0, if n is odd

0

...(4)

= ≅ Jn, if n is even. ...(5) Again, multiplying both sides of (3) by sin nΞ and then integrating w.r.t. ‘Ξ’ between limits 0 to ≅ and using (1), we get

z



sin ( x sin Ξ) sin nΞ dΞ

0

= ≅Jn, if n is odd = 0, if n is even.

...(6) ...(7)

Let n be odd. Adding (4) and (6), we get

z

z



[cos ( x sin Ξ) cos nΞ + sin ( x sin Ξ) sin nΞ] dΞ = ≅Jn.

0



z

≅ Jn(x) = 1 cos (nΞ ) x sin Ξ) dΞ . ...(8) 0 ≅ 0 Next, let n be even. Then adding (5) and (7) as before, we again get (8). Thus (8) holds for each positive integer (even as well as odd). Part (ii). Let n be any integer. Then as in part (i), if n is positive integer, we have

or

cos (nΞ ) x sin Ξ) dΞ = ≅Jn

or

z

≅ Jn(x) = 1 cos (nΞ ) x sin Ξ) dΞ ...(9) ≅ 0 Next, let n be a negative integer so that n = –m, where m is a positive interger. To prove the required result for a negative integer, we prove that

z

≅ J–m(x) = 1 cos ()mΞ ) x sin Ξ) dΞ . ≅ Ψ Let Ξ = ≅ – Ο so that d Ξ = –d Ο. Then, we have

...(10)

Created with Print2PDF. To remove this line, buy a license at: http://www.software602.com/

Bessel Functions

11.35

R.H.S. of (10) 1 Ψ 1 ≅ cos {) mΧ≅ ) Ο) ) x sin Χ≅ ) Ο)} () d ΟΕ = cos [( mΟ ) x sin Ο) ) m≅Θ d Ο ≅ 0 ≅ ≅ 1 ≅ [cos (mΟ ) x sin Ο) cos m≅ ∃ sin(mΟ ) x sin Ο) sin m≅Θ d Ο = ≅ 0 1 ≅ ( )1) m cos( mΟ ) x sin Ο) d Ο = [ sin m≅ = 0 and cos m≅ = (–1)m] ≅ 0 ≅ = 1 ()1)m cos (mΞ ) x sin Ξ) dΞ = (–1)m Jm(x) [Using (9) as m is + ve integer] ≅ Ψ = J–m(x) = L.H.S. of (10) [ J–m(x) = (–1)m Jm(x)] Thus, (10) is true. (9) and (10), show that the required result holds for each integer. Part (iii). Integrating (2) w.r.t. ‘Ξ’ between the limits 0 to ≅ and using the result

=

Ν

Ν

Ν

Ν

z

z



Ψ

cos pΞ dΞ % 0 , if p is an even interger, we have

z



Ψ

cos ( x sin ΞΕ dΞ % J0 ( x)

z



Ψ

...(11)

dΞ ∃ 0 ∃ 0 ∃ ... = J0(x). ≅.

z

1 ≅ cos ( x sin ΞΕ dΞ . ≅ Ψ Replacing Ξ by ≅/2 – Ξ in (2) and simplifying, we get cos (x cos Ξ) = J0 – 2J2 cos 2Ξ + 2J4 cos 4Ξ – ... Integrating (13) w.r.t. ‘Ξ’ and using (11), we get

(

z

J0(x) =



cos ( x cos ΞΕ dΞ = J0(x). ≅ – 0 – 0... or

Ψ

z

(iv) Deduction. From (14), J0(x) = 1 ≅ But

z

Using (16), (14) becomes

J0(x) =



Ψ

J0(x) = 1 )

or

x2 22



J0(x) =



Ψ

FG1 ) x H

cos2n Ξ dΞ =

2

z

...(12) ...(13)

1 ≅ cos ( x cos ΞΕ dΞΖ ≅ Ψ

...(14)

IJ K

cos2 Ξ x 4 cos 4 Ξ ∃ ) ... dΞ 2! 4!

...(15)

1 3 3 3 5 ...(2n ) 1) ≅. 2 3 4 3 6 ...(2n)

...(16)

7 1+ x2 1 x4 1 3 3 x6 1 3 3 3 5 3 ≅∃ 3 ≅) 3 ≅ ∃ ...8 ,x ) ≅− 2! 2 4! 2 3 4 6! 23 4 3 6 9

x4 22 3 42

)

#

x6 22 3 42 3 62

∃ ... %

()1)r x 2 r

∋ (2 r %0

r

3 r !)2

.

Ex. 3. Use the generating function to show that Jn(–x) = (–1)n Jn(x). [Agra 2008; Meerut 2006, 11; Kanpur 2008] #

∋ J ( x) z

Sol. We have

n

n% ) #

Replacing x by –x in (1), we get #



n% ) #

RS FH T

n Jn () x ) z % exp ) x z ) 1 2 z

n

% exp

RS x FH z ) 1IK UV . T2 z W

...(1)

IK UV % expRS x FH )z ) 1 IK UV = ∋ J (x) . ()z) W T 2 )z W #

n %)#

n

n

, by (1)

Created with Print2PDF. To remove this line, buy a license at: http://www.software602.com/

11.36

Bessel Functions #



(

#

n

J n () x ) z %

n% ) # n

∋ J (x) ()1)

n

n

...(2)

z .

n

n% ) #

Jn(–x) = (–1)nJn(x).

Equating the coefficients of z from both sides of (2), we have #

Jn(x + y) =

Ex. 4. Use the generating function to prove that

∋ J ( x) J r

n ) r ( y) .

n% ) #

expRS T

Sol. By the generating function,

( x ∃ y) F z ) 1I UV = ∋ J ( x ∃ y) z . H zKW #

1 2

n

...(1)

n

n% ) #

So we see that Jn(x + y) is the coefficient of zn on R.H.S. of (1). We next obtain the coefficient of z on L.H.S. of (1). Now, we have n

L.H.S. of (1) = exp #

=



r

Jr ( x ) z .

r %) #

#

=

RS x FH z ) 1IK UV . expRS y FH z ) 1IK UV , as exp(A + B) = e z W z W T T 1 2

1 2

#

∋ J ( y) z

s

A+B

= exp A. exp B.

[by definition of generating function]

s

s %) #

#

∋ ∋ J (x) J (y) z r

r∃s

s

.

...(2)

r %)# s % ) #

For a fixed value of r, we get zn by taking r + s = n i.e. s = n – r. Thus keeping r fixed, the coefficient of zn in (1) is Jr(x) Jn – r(y). So the total coefficient of zn will be given by summing all #

∋J

such terms from r = – # to r = # and is given by

r %)#

Hence equating the coefficient of zn from both sides of (1),

z

Ex. 5. If a > 0, prove that

r

( x ) Jn ) r (y). #

Jn(x) =

∋J

r

( x) Jn ) r ( y) .

r %) #

#

0

1

e )ax J0 (bx) dx %

2

(a ∃ b 2 )

.

[Ravishanker 1999; Purvanchal 2007; Lucknow 2010]

z

≅ J0(x) = 1 cos ( x sin Ξ) dΞ . ≅ 0

Sol. We know that (Refer Ex. 2. Part (iii)) (

z

( = 1 ≅

=

1 ≅

zz z RSTz #

0

RS T

#

0



0



#

0

0

e ) ax J0 (bx ) dx =

z

#

0

UV W

RS T

z z RSTz

UV W

...(1)

# e ) ax 1 cos (bx sin Ξ) dΞ dx , using (1) ≅ 0

e ) ax cos (bx sin Ξ) dΞ dx =

e )ax

z

≅ J0(bx) = 1 cos (bx sin Ξ) dΞ . ≅ 0

UV W

1 ≅



#

0

0

ei bx sin Ξ ∃ e )i bx sin Ξ 1 dx dΞ = 2 Η≅

UV W

e )ax cos (bx sin Ξ) dx dΞ

zz ≅

0

RS T

(On interchanging the order of integration) #

0

UV W

[e )(a ) ib sin Ξ)x ∃ e )(a ∃ ib sin Ξ) x ] dx dΞ

Created with Print2PDF. To remove this line, buy a license at: http://www.software602.com/

Bessel Functions

1 Η≅

=

=a



z

z

0



0

= 2a ≅



11.37

LM e OP ∃ e N )(a ) ib sin Ξ) )(a ∃ ib sin Ξ) Q )( a ) ib sin Ξ ) x

dΞ % a a ∃ b 2 sin 2 Ξ ≅ 2

z

0

#

= ) 2a2 ≅a

)( a ∃ ib sin Ξ ) x

z

# 0

cosec2Ξ dΞ = a ≅ b ∃ a 2 cosec2 Ξ



2

0

z LMN ≅

dΞ = 1 Η≅

z



0

cosec 2 Ξ dΞ 2a % ≅ b ∃ a 2 (1 ∃ cot 2 Ξ) 2

0

OP Q

1 1 ∃ dΞ a ) ib sin Ξ a ∃ ib sin Ξ

z

≅[Η

cosec2 dΞ (a ∃ b 2 ) ∃ a 2 cot 2 Ξ 2

0

( ) dt ) , putting cos Ξ = t so that – cosec2 Ξ dΞ = dt (a 2 ∃ b 2 ) ∃ a 2 t 2

z

0

dt 2 2 2 % a≅ # t ∃ (a ∃ b ) / a 2

2

z

#

0

dt 2

2

t ∃ [ (a ∃ b2 ) / a]2

LM N

1 t tan )1 = 2 . 2 2 2 2 a≅ (a ∃ b ) a (a ∃ b ) a

OP Q

#

2

=

≅ (a

0

2

FH ≅ ) 0IK = ∃b ) 2 2

1 2

a ∃ b2

.

Ex. 6. Using the generating function, prove that (ii) 2n J n ( x) % x Κ J n ∃1 ( x) ∃ J n)1 ( x)Λ

(i) 2 J n ( x ) % J n )1 ( x ) ) J n ∃1 ( x)

#

Μ J n ( x ) z n % e( x / 2) ϑ ( z )1/ z )

Sol. Generating function is given by

n %)#

... (1)

#

Μ J n ( x ) z n % e( x / 2) ϑ ( z )1/ z ) ϑ (1/ 2) ϑ ( z ) 1/ z )

Diff. both sides of (1) w.r.t. ‘x’,

n %)#

#

#

#

n %)#

n %)#

n %)#

2 Μ J n ( x ) z n % ( z ) z )1 ) Μ J n ( x ) z n % Μ J n ( x) ( z n ∃1 ) z n )1 ) , using (1)

or

#

#

#

n %)#

n %)#

n %)#

2 Μ J n ( x ) z n % Μ J n ( x ) z n ∃1 ) Μ J n ( x ) z n )1

or

Equating the coefficients of zn on both sides of (2) yields

... (2)

2 J n ( x ) % J n )1 ( x ) ) J n ∃1 ( x)

(ii) Differentiating both sides of (1) w.r.t. ‘z’, we get #

Μ n J n ( x ) z n )1 % e( x / 2) ϑ ( z )1/ z ) ϑ (1/ 2) ϑ (1 ∃ z )2 )

n %)# #

#

2 Μ n J n )1 ( x) z n )1 % x (1 ∃ z )2 ) Μ J n ( x ) z n , using (1)

or

n %)#

n %)#

#

#

#

n %)#

n %)#

n %)#

2 Μ n J n ( x ) z n )1 % x Μ J n ( x) z n ∃ x Μ J n ( x ) z n ) 2

or

Equating the coefficients of zn–1 on both sides of (3), Ex. 7. Show that

Ν

x sin ax

y

0

2

2 1/ 2

(y ) x )

dx %

... (3)

2n Jn(x) = x{Jn – 1(x) + Jn + 1 (x)}

≅y J1 ( ay ) 2

sin ( x sin Ξ) % 2sin Ξ 3 J1 ∃ 2sin 3Ξ 3 J 2 ∃ ...

Sol. From result (ii) of Art. 11.9,

... (1)

Multiplying both sides of (1) by sin Ξ and then integrating between the limits 0 and ≅, we have

Ν



0

sin( x sin Ξ) sin Ξ d Ξ % J1

Ν



0

(2sin 2 Ξ)d Ξ ∃ J 2

Ν



0

(2sin Ξ sin 3Ξ) d Ξ ∃ ...

Created with Print2PDF. To remove this line, buy a license at: http://www.software602.com/

11.38

Bessel Functions ≅



+ sin 2Ξ 7 + sin 2Ξ sin 4Ξ 7 ∃ J2 , ) ∃ .... % J1 (1 ) cos 2Ξ) d Ξ ∃ J 2 (cos 2Ξ ) cos 4Ξ)d Ξ ∃ ...% J1 ,Ξ ) 8 2 2 89 0 0 0 − 90 − 2 % J1 3 ≅, since the remaining terms vanish

Ν



Ν



≅ J1 ( x) %

Thus,

Ν



0

sin ( x sin Ξ) sin Ξ d Ξ

... (2)

Let F (Ξ) % sin( x sin Ξ)sin Ξ. Then, clearly F (≅ ) Ξ) ) F (Ξ). Hence, using a property of definite integrals, (2) yields ≅ J1 ( x ) % 2 ≅ J1 (ay ) % 2

Ρ

Ν

≅/2

0

Ν

≅/2

0

sin ( x sin Ξ) sin Ξ d Ξ

sin (ay sin Ξ) sin Ξ d Ξ

... (3)

Put y sin Ξ % x so that y cos Ξ d Ξ % dx and hence dΞ %

dx dx dx dx % % % 2 2 1/ 2 2 2 1/ 2 y cos Ξ y (1 ) sin Ξ) y (1 ) x / y ) ( y ) x 2 )1/ 2 y:

Ν

Hence (3) yields ≅ J1 (ay) % 2

; x 1

0

FH IK

1 x ≅ 6(n ∃ 12 ) 2

Ex. 8. Prove that (i) Jn(x) =

(ii) Jn(x) =

FH IK

1 x ≅ 6(n ∃ 12 ) 2

Ν

≅[Η

0

0

0

x sin ax dx 2

2 1/ 2

(y ) x )

%

≅y J1 (ay ) 2

cos ( x sin Ξ) cos2n Ξ dΞ.

[Garhwal 2005, Ravishanker 2004; Ranchi 2010]

z

n ≅ 0

cos ( x cos Ξ) sin 2n Ξ dΞ.

I=

Sol. (i). Let (I= 2

z

n ≅

Ν

y

z



0

[Bilaspur 1998]

cos ( x sin Ξ) cos2 n Ξ dΞ.

...(1)

cos ( x sin Ξ) cos 2 n Ξ d Ξ, since cos( x sin Ξ) cos2n Ξ is an even function.

I=2

or

But

2

z

z

≅ [Η

0

≅[Η

0

LM N

cos2n Ξ 1 )

p

OP Q

x 2 sin2 Ξ x 4 sin 4 Ξ ∃ ) ... dΞ . 2! 4!

q

cos Ξ cos Ξ dΞ =

6

FH p ∃ 1IK 6FH q ∃ 1IK 2 2 . p ∃ q ∃ 2 I 26 F H 2 K

...(2)

...(3)

Using (3), (2) may be written as I=2

LM 6(n ∃ ) 6( ) ) x 6(n ∃ ) 6(3 / 2) ∃ x 6(n ∃ ) 6(5 / 2) ) ...OP N 26(n ∃ 1) 2 ! 26(n ∃ 2) 4 ! 26(n ∃ 3) Q 1 2

1 2

2

1 2

4

1 2

1 ≅ + 7 Χ∴[ΗΕ ϑ 12 ≅ ≅ x2 x4 2 1 ) ϑ ∃ ϑ ) ..8 = 6(n ∃ 2 ) , 89 −, 6(n ∃ 1) 2 ! (n ∃ 1) 6 (n ∃ 1) 4 ! (n ∃ ΗΕ Χ n ∃ ∆Ε6(n ∃ 1)

=

6( n ∃ 12 ) ≅ + 7 x2 x4 ∃ ) ...8 ,1 ) 6( n ∃ 1) − 4( n ∃ 1) 4 3 8( n ∃ 1) ( n ∃ 2) 9

...(4)

Created with Print2PDF. To remove this line, buy a license at: http://www.software602.com/

Bessel Functions

11.39

Jn(x) =

Also,

+ 7 x2 x4 1 ) ∃ ) ...8 . , n 2 6(n ∃ 1) − 4(n ∃ 1) 4 3 8(n ∃ 1) (n ∃ 2) 9 xn

Multiplying both sides of (4) by

...(5)

xn , we get 2 6(n ∃ 12 ) ≅ n

+ 7 x x x xn I 1) ∃ ) ...8 = n , 1 4(n ∃ 1) 4 3 8 3 (n ∃ 1) (n ∃ 2) 2 6(n ∃ 12 ) ≅ 2 6(n ∃ 2 ) − 9 2

n

4

n

FH IK

1 x ≅ 6(n ∃ 1 / 2) 2

Using (1) and (5), (6) becomes Part (ii). Proceed as in part (i), Ex. 9. Prove that Jn(x) =

2

n )1

I=

z



0

z

1

0

FH IK

cos ( x sin Ξ) cos2n Ξ dΞ = 2

Let sin Ξ = t so that cos Ξ dΞ = dt. Then (2) gives

Ν



2n I = 2 cos xt cos Ξ 0

Ν

dt %2 cos Ξ



Ν cos xt cos

2n ) 1

0



Ν

0

cos ( x sin Ξ) cos2n Ξ dΞ % Jn ( x).

(1 ) t 2 )n ) 1/ 2 cos xt dt.

1 x ≅ 6(n ∃ 12 ) 2

Jn(x) =

Sol. From part (i) of Ex. 8, Let

xn 6 (n ∃ 12 ) ≅

z

n ≅

z

z

n ≅ 0

≅ [Η

0

...(6)

[Bilaspur 1994, 97]

cos ( x sin Ξ) cos2n Ξ dΞ.

cos ( x sin Ξ) cos2n Ξ dΞ

...(1) ...(2)

Ξdt



= 2 cos xt (cos 2 Ξ)(2 n ) 1) / 2 dt % 2 cos xt (1 ) sin 2 ΞΕn ) 1[ 2 d t . 0

z



I = 2 cos xt. ( I ) t 2 )n ) 1/ 2 dt.

( Using (2), (3) gives Using (4), (1)

0

z

0



0

z



cos ( x sin Ξ) cos2 n dΞ = 2 (1 ) t 2 )n ) 1/ 2 cos xt dt . 0

1

Ρ Jn(x) =

(

Jn(x) =

≅ 6( n ∃ 12 )2

Η

n )1

n

ϑ

xn 6(n ∃ 12 ) ≅

xn 2

z

n



0

ϑ2



Ν (1 ) t 0

2 n ) 1/ 2

)

cos xt dt

z

z

1 ≅ cos ( x sin Ο) dΟ satisfies Bessel’ss ≅ 0 [Indore 2004]

≅ J0(x) = 1 cos ( x sin Ο) dΟ . ≅ 0 Bessel’s equation of order n is x2y! + xy + (x2 – n2)y = 0. For n = 0, (2) reduces to x2y! + xy + x2y = 0. In order to show that J0(x) satisfies (3), we must prove that

Sol. Given

...(4)

(1 ) t 2 )n ) 1/ 2 cos xt dt .

Ex. 10. Verify directly that representation J0(x) = equation in which n = 0.

...(3)

x 2 J 0 ( x ) ∃ x J 0 ( x ) ∃ x 2 J 0 ( x) % 0.

...(1) ...(2) ...(3) ...(4)

Created with Print2PDF. To remove this line, buy a license at: http://www.software602.com/

11.40

Bessel Functions

z z



J 0 ( x) = ) 1 sin ( x sin Ο) sin Ο dΟ . ...(5) ≅ 0

Differentiating both sides of (1) w.r.t. ‘x’,



J 0 ( x) = ) 1 cos (x sin Ο) sin 2 Ο dΟ . ...(6) ≅ 0 Integrating R.H.S. of (5) by parts taking sin Ο as second function, we get Differentiating both sides of (5) w.r.t. ‘x’,

l

q ∃Ν

J 0 ( x) = ) 1 ) sin ( x sin Ο) (cos ΟΕ ≅ or

z

LM N

≅ 0



0

OP Q

7 cos ( x sin Ο) 3 x cos Ο cos Ο d Ο 8 9

≅ J 0 ( x) = ) 1 0 ∃ x cos ( x sin Ο) cos2 Ο dΟ = – x ≅ ≅ 0 Using (1), (6) and (7), L.H.S. of (4)

= )

x2 ≅

z z



0

cos ( x sin Ο) sin 2 Ο dΟ )

x2 ≅

z



0

z



0

cos (x sin Ο) cos2 Ο dΟ .

cos ( x sin Ο) cos2 Ο dΟ ∃

z

x2 ≅

z

...(7)



cos ( x sin Ο) dΟ

0

2 ≅ x2 ≅ = )x cos ( x sin Ο) (sin 2 Ο ∃ cos2 Ο) dΟ ∃ cos ( x sin Ο) dΟ = 0 = R.H.S. of (4) ≅ 0 ≅ 0 Thus (4) is true. Hence we get the required result.

2 x

1. Prove that (i) J0(x) =

z

EXERCISE 11 (D)

1 cos

0

xt dt

(1 ) t )

2. If n is non–negative, prove that 3. Prove that (i)

(ii) 4. Show that (i)

z z

(ii)

2

z

#

0

Jn (bx ) dx %

0

Jn ( x) 1 dx % . x n

1 . b

.00, b Υ a sin ax J0 (bx) dx = 1 2 2 1/ 2 0 041/(a ∃ b ) , b ] a

.01/(b2 ) a 2 )1/ 2 , b Υ a cos ax J0 (bx) dx = 1 0 040, b ] a #

Ν

≅/ 2

J 0 ( x cos Ο) cos Ο d Ο %

sin x x

5. For Bessel function Jn(x) prove that Jn(x) =

z

#

#

0

J0(x) = 2 ≅

z



cos xt dt

0

(1 ) t 2 )

1 ≅



z



Ψ

z



0

z

≅/2

0

J1( x cos Ο) dΟ %

1 ) cos x . x

cos ΧnΟ ) x sin Ο) dΟ and hence show that

.

6. Show that the recurrence relation J n ( x ) = differentiation of Jn(x) = 1

(ii)

[Purvanchal 2007] 1 2

[Jn – 1(x) – Jn + 1(x)], follows directly from

cos (n Ο ) x sin Ο) d Ο .

z

≅ 7. Show that coefficient of tn in the expansion of e ) ( x / 2) (t ) 1/ t) equals 1 cos (nΞ ) x sin Ξ) dΞ. ≅ 0 11.10. Orthogonality of Bessel functions If Γi and Γj are roots of the equation Jn(Γa) = 0, then

Created with Print2PDF. To remove this line, buy a license at: http://www.software602.com/

Bessel Functions

11.41

Ν

a

0

R| S| T

0, if i & j (different roots) x J n (Γi x ) J n (Γ j x) dx = a 2 2 J (Γ a), if i % j (equal roots) 2 n∃1 i

RS T

2 0 if i & j x J n (Γi x ) J n (Γ j x) dx = a Jn2 ∃ 1(Γi a) ⊥ ij . where ⊥ij = Kronecker delta = 1 if i = j 0 2 [Meerut 2010; Nagpur 2005; Purvanchal 2005, 06; Delhi Physics (H) 2002] Proof. Case I. Let i & j, i.e., let Γi and Γj be unequal roots of Jn(Γa) = 0. ( Jn(Γia) = 0 and Jn(Γja) = 0 ...(1) Let u(x) = Jn(Γix) and v(x) = Jn(Γj x). ...(2) Then u and v are Bessel functions satisfying the modified Bessel’s equation [Refer Art. 11.4] x2y! + xy + (Γ2x2 – n2)y = 0. ...(3) ( x2u! + xu + (Γi2x2 – n2)u = 0. ...(4) and x2v! + xv + (Γj2x2 – n2)v = 0. ...(5) Multiplying (4) by v and (5) by u and then subtracting, we get x2(vu! – uv!) + x(vu – uv ) + x2(Γi2 – Γj2)uv = 0 or x(vu! – uv!) + (vu – uv ) = x(Γj2 – Γi2)uv d or x (vu – uv ) + (vu – uv ) = x(Γj2 – Γi2)uv dx d or {x(vu – uv )} = x(Γj2 – Γi2)uv.. ...(6) dx

Ν

i.e.

a

Integrating (6) w.r.t. x from 0 to a,

Ν

(Γj2 – Γi2)

a

0

xuv dx = x(vu ) uv ) a . 0

...(7)

Using (2), (7) reduces to

z

a

a

0

0

(Γj2 – Γi2) x Jn (Γi x ) Jn (Γ j x) dx = x{Jn (Γ j x )Jn' (Γ i x) ) Jn (Γ i x) Jn' (Γ j x )}

= a{Jn(Γja)J n(Γia) – Jn(Γia)J n(Γja)} = 0, using (1)

z

Since Γi & Γj, the above equation gives

a

0

x Jn (Γ i x) Jn (Γ j x ) dx = 0, when i & j. ...(8)

Case II. Let i = j (equal roots). Multiplying (4) by 2u , we have 2x2u!u + 2xu 2 + 2(Γi2x2 – n2)uu = 0 2Γi2xu2 =

(

Integrating (9) w.r.t. ‘x’ from 0 to a,

d {x2u 2 – n2u2 + Γ2 x2u2} – 2 Γ2 xu2 = 0 i i dx

or

d 2 2 (x u – n2u2 + Γi2x2u2). dx

2Γi2

z

a

0

xu2 dx = x 2 u 2 ) n 2u 2 ∃ Γ 2i x 2 u 2

...(9) a 0

...(10)

Using (1) and (2) and noting that Jn(0) = 0, we have 2Γi2

z

a

0

2 2 x Jn2 (Γi x ) dx = a {Jn' (Γ i x )}

d J (x) = n J (x) – J (x). n+1 x n dx n

From recurrence relation IV, we have Replacing x by Γix in (12), we have d J (Γ x) = n Jn(Γix) – Jn + 1(Γix) d (Γ i x ) n i Γi x

x%a

or

...(11) ...(12)

1 d n J (Γ x ) = J (Γ x) – Jn+ 1 (Γix) Γ i dx n i Γi x n i

Created with Print2PDF. To remove this line, buy a license at: http://www.software602.com/

11.42

Bessel Functions

J n(Γix) =

or 2

(

{Jn' (Γ i x)}

x%a

LM{ n J (Γ x ) ) Γ Nx

=

n

i

n Jn(Γix) – Γi Jn + 1(Γix). x i

} OPQ 2

J n ∃ 1( Γ i x )

x%a

= {0 – Γi Jn + 1(Γia)}2, by (1)

= Γi2 J2n + 1(Γia). Using this value in (11) and dividing both sides of the resulting equation by 2Γi2, we get

z

a

0

a2 2 J (Γ a ) . 2 n ∃1 i

x Jn2 (Γi x ) dx =

z

...(13)

2 x Jn (Γi x ) Jn (Γ j x) dx = a Jn2 ∃ 1(Γ i a) ⊥ ij . . ...(14) 2 0 11.11. Bessel–series or Fourier–Bessel expansion for f(x). If f(x) is defined in the region 0 Ω x Ω a and has an expansion of the form

Combining (8) and (13), we have

a

#

f(x) =

∋c

i

J n (Γi x ) ,

...(1)

i %1

where the Γi are the roots of the equation 2

ci =

then

Jn(Γa) = 0,

Ν

a

0

x f ( x ) J n (Γ i x) dx

...(2)

.

a 2 J n2 ∃ 1 (Γ i a )

...(3) #

Proof. Multiplying both sides of (1) by xJn(Γjx), xf(x) Jn(Γjx) = Integrating both sides of (4) w.r.t. ‘x’ from 0 to a, we get

Ν

a

0

#

x f ( x ) J n (Γ j x ) dx =

∋c

i

i %1

z

∋ c x J (Γ x ) J (Γ x ) i

i %1

n

i

n

j

a

0

x Jn (Γ i x ) Jn (Γ j x ) dx.

...(5)

From the orthogonality property of Bessel functions, we have 0, if i & j a x Jn (Γi x ) Jn (Γ j x ) dx = a 2 2 0 J (Γ a), if i % j 2 n∃1 j

z

R| S| T

Ν

Using (6), (5) reduces to

0

Replacing j by i in (7), we have ci

2

a 2 J (Γ a) = 2 n+1 i

z

a

0

...(6)

2

x f ( x ) J n (Γ j x ) dx = cj a J2n 2 2

a

x f ( x ) Jn (Γ i x) dx

or

ci =

...(8) 11.11A. Solved Exaples based on Art. 11.11

Ν

a

0

...(4)

+ 1(Γja).

x f ( x ) J n (Γ i x) dx a 2 J n2 ∃ 1 (Γ i a )

...(7)

.

#

Ex. 1. Expand the function f(x) = 1, 0 Ω x Ω a in a series of the form ∋ ci J0 (Γi x ) , where Γi i %1

are the roots of the equation J0(Γa) = 0. #

Sol. Given

f(x) = 1 =

∋c i %1

i

J0 (Γ i x) ,

...(1)

Created with Print2PDF. To remove this line, buy a license at: http://www.software602.com/

Bessel Functions

11.43

where J0(Γia) = 0. Then from Art. 11. 11 (with n = 0), we know that ci =

z

a

2 0 x f ( x ) J0 (Γ i x ) dx a 2 J12 (Γ i a)

=

z

...(2)

a

2 0 x J0 (Γi x ) dx a 2 J12 (Γ i a)

, as f(x) = 1

...(3)

Let Γix = t so that dx = dt/Γi. Then, we have

z

a

x J0 (Γi x ) dx = 12 0 Γi

z

z

a Γi

t J0 (t ) dt

0

aΓ i d {t J (t )} dt , as d [ x n J ( x)] % x n J ( x ) Ρ d [ x J ( x )] % x J ( x) = 12 n n )1 1 0 1 dx dx Γ i 0 dt aΓ = 12 t J1(t ) 0 i % 12 [aΓ i J1(aΓ i ) ) 0], as J1(0) % 0 . Γi Γi

z

(

a

x J 0 (Γ i x ) dx = (a/Γi) × J1(aΓi)

...(4)

0

ci =

Using (4), (3) becomes

2 ϑ (a / Γ i ) ϑ J1 ( a Γ i ) a

2

J12

( aΓ i )

%

2 aΓ i J1 ( aΓ i )

...(5) #

∋Γ

1= 2 a

Using (5), (1) becomes

i %1

J 0 (Γ i x ) . i J 0 (Γ i a )

#

Ex. 2. Expand x in a series of the form

∋C

r

r %1

J1 (Γ r x ) valid for the region 0 Ω x Ω 1, wheree

Γr are the roots of the equation J1(Γ) = 0.

#

f(x) = x = ∋ Cr J1(Γ r x ) ,

Sol. Given

...(1)

r %1

J1(Γr) = 0

where

Then from Art. 11.11 (with n = 1, i = r and a = 1),

Cr =

2

z

1

x 2 J1(Γ r x ) dx = 13 0 Γr

= 13 Γr

z

Γr

0

z

Γr

0

d {t 2 J (t )} dt , 2 dt

J1(Γ r x ) dx

J22 (Γ r )

...(3)

Let Γrx = t, so that dx = dt/Γr. Then we have

z

...(2)

1 2 x 0

t 2 J1(t ) dt

as

d n d 2 [ x J n ( x)] % x n J n ) 1 ( x ) Ρ [ x J 2 ( x)] % x 2 J1 ( x ) dx dx

Γr = 13 t 2 J 2 (t ) % 13 [Γ 2r J 2 (Γ r ) ) 0], as J 2 (0) % 0

Γr

0

Γr

(

z

1

1 J2(Γr). x 2 J1(Γ r x ) dx = Γ 0 r

Using (4), (3) becomes

2 Cr = Γ J (Γ ) r 2 r

Using (5), (1) becomes

x= 2

#

...(4) ...(5)

J1(Γ r x ) , 0 Ω x Ω 1. r J2 (Γ r )

∋Γ r %1

Created with Print2PDF. To remove this line, buy a license at: http://www.software602.com/

11.44

Bessel Functions

EXERCISE 11 (E) #

1. Expand x2 in a series of the form ∋ Cr J0 (Γ r x ) valid for the region 0 Ω x Ω a, where Γr are r %1

the roots of the equation J0(Γa) = 0. #

2. Prove that 1 =

∋ Σ2 n %1

n

Ans. x 2 %

2 a

#



{(Γ r a )2 ) 4} J 0 (Γr x ) Γ3r J 0 (Γ r a)

r %1

J0 (Σ n x) . J1(Σ n ) #

3. If Γi are the solutions of J0(Γ) = 0, show that ∋

J0 (Γi x)

2 i %1ΚΓi J1(Γ n )}

#

% ) 1 log x , where 0 < x < 1. 2

4. If f(x) = ∋ Ci J0 (Γ i x) where J0(Γi) = 0, i = 1, 2, 3, ..., show that i %1

5. If Γi are the positive roots of J0(Γ) = 0, show that 1

z

1

#

x[ f ( x )]2 dx =

0

∋Γ i %1

2 i

J12 (Γ i ) .

# J (Γ x) 1 ) x2 = ∋ 30 i , where –1 < x < 8 i %1 Γ i J1(Γ i )

# J (Γ x) 6. If Γi are the positive roots of J1(Γ) = 0, show that x3 = 2∋ 1 i , where –1 < x < 1 Γ J (Γ ) i %1 i 2 i #



7. If Γi are the positive roots of J1(Γ) = 0, show that x3 = 2

i %1

(8 ) Γi2 ) J1 (Γi x) Γ3i J1 (Γi )

, where –1 < x < 1

OBJECTIVE PROBLEMS ON CHAPTER 11 1. Write (a), (b), (c) or (d) whichever is correct (a) x{J n )1 ( x ) ∃ J n ∃1 ( x )} is equal to (a) 2Jn(x)

(b) 2 J n ( x )

(c) 2n Jn (x)

(d) None of these [Agra 2005, 06]

Sol. Ans. (c). Refer recurrence relation VI of Art. 11.7. 2. The Bessel’s equation is (a) z2(d2w/dz2) – z (dw/dz) + (z2 – n2) w = 0 (b) z2(d2w/dz2) + z (dw/dz) + (z2 + n2) w = 0 (c) z2(d2w/dz2) + z(dw/dz) + (z2 – n2)w = 0 (d) None of these. [Agra 2005, 07] Sol. Ans. (c). Refer equation (1) of Art. 11.1 3. d {xn Jn(x)}/dx is equal to (a) xn Jn – 1 (x) (b) xn – 1 Jn (x) (c) xn Jn + 1 (x) (d) xn + 1 Jn(x) [Bhopal 2010] Sol. Ans. (a). Refer recurrence relation I of Art. 11.7. 4. The Bessel’s functions

Κ J 0 (Σk x)Λ#k %1

with Σ k denoting the kth zero of J0(x) form an

orthogonal system on [0, 1] with respect to weight function (a) 1 (b) x2 (c) x (d) x . Sol. Ans. (c) Refer Art. 11.10. [GATE 2002] 5. If Jn(x) and Yn(x) denote Bessel functions of order n of the first and second kind, then the general solution of the differential equation x(d2y/dx 2) + (dy/dx) + xy = 0 is given by (a) y ( x) % Σx J1 ( x) ∃ ΤxY1 ( x )

(b) y ( x) % Σ J1 ( x ) ∃ ΤY1 ( x )

(c) y ( x) % ΣJ 0 ( x) ∃ ΤY0 ( x ) (d) y ( x) % ΣxJ 0 ( x ) ∃ ΤxY0 ( x). Sol. Ans. (c) Refer remark of theorem I, Art. 11.4.

[GATE 2005]

Created with Print2PDF. To remove this line, buy a license at: http://www.software602.com/

Bessel Functions

11.45

MISCELLANEOUS PROBLEMS ON CHAPTER 11 1. If x > a, show that 2. Prove that J 0 ( x ) %

(ii)

(iii)

Ν

#

0

Ν

Ν

0

1 ≅

Ν

e a cos Ο cos( x sin Ο) d Ο % J 0 {( x 2 ) a 2 )1/ 2 }



0

cos( x cos Ξ) d Ξ and deduce that (i)

cos(ax) J 0 (bx )dx % 0 or

#

0



sin(ax ) J 0 (bx ) dx % )

Ν

3. Show that (i)

≅/ 2

0

1 2

(b ) a 2 )1/ 2 1 2

2 1/ 2

(a ) b )

J 2 n (2 x cos Ο) d Ο %

Ν

#

0

e ) ax J 0 (bx )dx %

1 2

(a ∃ b 2 )1/ 2

, according a2 > or < b2

or 0, according as a2 > or < b2.

2 ≅ Π J n ( x) Θ2 (ii) J 0 ( x) % ≅ 2

Ν

( x ∃ y) J 0 ( y ) dy x∃ y

# sin

0

(n ∃ 2r ) (n ∃ r ) 1) J n∃ 2r ( x) r %0 r! #

(iii) x n % 2n Μ

4. Show that Jn (x) is even or odd function of x according as n is even or odd, respectively. 5. Show that

Ν J Χ{x (t ) x)} Ε % 2sin (t / 2). 1

1/ 2

0

0

[Kanpur 2008]

Sol. Setting n = 0 and x = {x (t – x)}1/2 in result (1) of Art. 11.2, we have #

+ {x (t ) x}1/2 7 1 , 8 j0 ({x (t – x)} ) % ∋ ()1) r ! 6 (r ∃ 1) − 2 r %0 9

(

Ν0 J 0 Χ{x(t ) x)} 1

1/2

Ε dx

! #

!

1 #

Ν0 r∋%0

( )1) r xr (t ) x)r 2r

2 r !6( r ∃ 1)

()1) r t 2 r ∃1

#

dx % ∋

r %0 2

#

()1)r x r (t ) x)r 2r r % 0 2 r ! 6 ( r ∃ 1)

%∋

( )1)r

2r

1 r

Νx r !6( r ∃ 1) 0

(t ) x) r dx

1

∋ 22r r !6(r ∃ 1) Ν0 y r (1 ) y)r dy, putting x = ty and dx = tdy

r %0

#

!

2r

r

1/2

( )1) r t 2r ∃1

∋ 22r r !6 (r ∃ 1) B(r ∃ 1, r ∃ 1), r %0

by definition 6.2, page 6.1

#

()1)r t 2r ∃1 r ! , since 6 (r + 1) = r!, r being a positive integer.. ! ∋ 2r r %0 2 r !6 (2 r ∃ 1) 2 r ∃1 # .0 t (t / 2)3 (t / 5)5 /0 r (t / 2) ! 2∋ ()1) (2r ∃ 1)! % 2 1 2 ) 3! ∃ 5! ) ......2 r %0 40 50

! 2 sin (t/2), as sin x = x – x3/3! + x3/5! – x7/7! +...

Created with Print2PDF. To remove this line, buy a license at: http://www.software602.com/

11.46

Bessel Functions 2

2

6. Show that every non-trivial solution of Bessel’s equation x y + xy + (x – n2) y = 0 has infinitely many zeros. [Mumbai 2010] 7. Find the normal form of Bessel’s equation x2y + xy + (x2– p2) y = 0 and use it to show that every non-trival solution has infinitely many positive zeros. [Himachal 2010] 8. Show that if x is real, between two consecutive zeros of x–n Jn (x), there lies one and only one zero of x–n Jn+1 (x).

Created with Print2PDF. To remove this line, buy a license at: http://www.software602.com/

12 Hermite Polynomials 12.1. Hermite’s equation and its solution. [Meerut 1993, 95, 96] Hermite’s equation is (d2y/dx2) – 2x(dy/dx) + 2ny = 0, ...(1) where n is a constant. We now solve (1) in series by using the method of Frobenius. y = # Cm xk + m,

Let the series solution of (1) be

m! 0

C0 ∃ 0

...(2)

Differentiating (2) and then putting the value of y, dy/dx and d2y/dx2 in (1), we get

# Cm (k + m) (k + m – 1) xk + m – 2 –2xCm(k + m) xk + m – 1 + 2n # Cm xk + m = 0

m! 0

or or

m! 0

%

# Cm (k + m) (k + m – 1) xk + m – 2–2 & # Cm (k + m) xk + m – # Cm nx ∋ m! 0

m! 0

k (m )

m !0

∗ =0 +

# Cm (k + m) (k + m – 1) xk + m – 2 – 2 # Cm (k + m – n) xk + m = 0.

m! 0

m! 0

... (3)

(3) is an identity. To get the indicial equation, we equate to zero the coefficient of the smallest power of x, namely xk – 2, in (3) and obtain C0 k (k – 1) = 0 or k (k – 1) = 0, as C0 ∃ 0. ...(4) So the roots of indicial equation (4) are k = 0, 1. They are distinct and differ by an integer. The next smallest power of x is k – 1. So equating to zero the coefficient of xk – 1 in (3), we get C1 (k + 1) k = 0. ... (5) When k = 0 (one of the roots of the indicial equation), (5) shows that C1 it indeterminate. Hence C0 and C1 may be taken as arbitrary constants. Equating to zero the coefficient of xk + m – 2, (3) gives Cm (k + m) (k + m – 1) – 2 Cm – 2 (k + m – 2 – n) = 0 or

2 (k ( m , 2 , n) C . ( k ( m ) ( k ( m , 1) m–2 2 ( m , 2 , n) Putting k = 0 in (6) gives Cm = Cm–2 . m ( m , 1) Putting m = 2, 4 , 6, ..., 2m, in (7), we have

Cm =

C2 = , C4 =

and

...(6) ...(7)

(,1)1 − 21 − n 2n C0 = , 2n C0 = , C0, 2! 2 −1 2!

2 2 2 (2 , n) (,1)2 − 2 (2 , n) 2n (,1) 2 n (n , 2) . C2 = C0 = C0, 4! 4−3 4 −3 2! ... ... ... ... ... ...

C2m =

(,1)m 2m − n(n , 2)...(n , 2m ( 2) C0 (2m) ! 12.1

Created with Print2PDF. To remove this line, buy a license at: http://www.software602.com/

12.2

Hermite Polynomials

Next, putting m = 3, 5, 7, ..., 2m + 1, in (7), we get C3 =

2 (1 , n) 2 (3 , n) 2 (n , 1) (,1)2 22 (n , 1) (n , 3) ( ,1)1 2 1 ( n , 1) C1 = , C1 = C1, C5 = C3 = C1, 3! 3! 5! 3− 2 5−4

...

...

... m

C2m + 1 =

and i.e.

...

...

...

m

( ,1) 2 (n , 1) (n , 3)...(n , 2 m ( 1) C1. (2 m ( 1) !

Putting the above values in (2) with k = 0, we get y = C0 + C1 x + C2 x2 + C3 x3 + .... 2 4 y = (C0 + C2 x + C4 x + ....) + (C1 x + C3 x3 + C5 x5 + ....),

LM N

m

OP Q

2 ( ,2) n(n , 2)...(n , 2 m ( 2) 2 m 2 n(n , 2) 4 2 x ( ... y = C0 1 , 2n x ( x ... ( 2! 4! ( 2 m) !

i.e.

LM N

( C1 x ,

OP Q

m

2

2 ( n , 1) 3 2 ( n , 1) ( n , 3) 5 (,2 ) (n , 1) (n , 3)...(n , 2m ( 1) 2 m ( 1 x ( x ( ... ( x ( ... 3! 5! (2 m ( 1) !

...(8)

y = C0 u + C1 v, say, ...(9) Since u/v is not merely a constant, u and v form a fundamental set (i.e. linearly independent) of solutions of (1). Hence (8) or (9) is the most general solution of (1) with C0 and C1 as two arbitrary constants. Remarks. In practice we require a solution of (1) such that (i) it is finite for all finite values of x and (ii) as x / , exp. (1/2x2) y(x) / 0. or

The solution (8) in ascending powers of x does not satisfy the condition exp. (1/2x2) y(x) /0 as x / . However, this requirement is easily seen to be satisfied provided the series terminate. Replacing m by m + 2 in (7), we have Cm+ 2 =

2( m , n ) C . ( m ( 1) ( m ( 2 ) m

...(10)

Let n be a non–negative integer. Then (10) shows that Cm + 2 and all subsequent coefficients in (2) will vanish and so the corresponding series terminate. We shall now obtain the series solution of (1) in descending powers of x by assuming n to be a non–negative integer. Re–writing (2) for k = 0, we have as explained above y = Cn xn + Cn – 2 xn – 2 + Cn – 4 xn – 4 + ... ...(11) Cm = ,

From (10),

Putting m = n – 2, n – 4, ...., in (12), Cn – 4 = ,

or

Cn – 2 = ,

...(12)

(n , 1) n n(n , 1) C = , Cn 2( n , n ( 2) n 2−2

(n , 3) (n , 2) n ( n , 1) (n , 2) ( n , 3) Cn – 2 = , Cn 2 ( n , n ( 4) 22 − 2 − 4

and so on. Putting these is (11), we have ,

( m ( 1) ( m ( 2 ) Cm + 2. 2( n , m )

y = an{xn

n ( n , 1) n , 2 n ( n , 1) ( n , 2) ( n , 3) n , 4 n (n , 1) (n , 2r ( 1) n , 2 r 10 x ( x ( .... ((,1)r x ( ...2 2 r 2− 2 2 −2−4 2 − 2 − 4 ... 2r 31 [ n / 2]

n (n , 1) (n , 2r ( 1)

r !0

2 − 2 − 4...2r

y = an 4 (,1)r

r

[n / 2 ]

x n , 2 r = an # (,1)r r!0

n! n , 2r , x 2 r ! (n , 2 r ) ! 2r

Created with Print2PDF. To remove this line, buy a license at: http://www.software602.com/

Hermite Polynomials

12.3

if n is even 5 n / 2, [ n / 2] ! 6 ( n , 1) / 2, if n is odd 7 n Taking an = 2 and denoting the solution by Hn (x), we obtain the standard solution of (1), known as Hermite polynomial of order n. 12.2. Hermite polynomial of order n. [Meerut 1994, 97, 98] Hermite polynomial of order n is denoted and defined by

where

[n / 2]

Hn (x) =

# (,1) r !0

r

n! n , 2r (2 x ) , r ! (n , 2 r ) !

{

n / 2, if n is even n / 2 ! (n , 1) / 2, if n is odd

where,

12.3. Generating function for Hermite polynomials. 2

2 t x ,t ! Theorem. Prove that e

# n! 0

t n H ( x) n! n

[Meerut 1992, 94]

Proof. Using the well known expansion for exponential function, we get e

2t x , t 2

2

= e2t x . e , t =

# s!0

( 2t x ) s!

s

# r!0

2 r

( ,t ) = r!

# # (,1)

r

s!0 r ! 0

s

(2 x ) s ( 2r . ...(1) t r!s!

Let s + 2r = n so that s = n – 2r. So for a fixed value of r, the coefficient of tn is given by n , 2r

(2 x ) . ...(2) r ! (n , 2r ) ! Now, s80 9 n – 2r 8 0 9 n 8 2r 9 r : n/2, which gives all values of r for which (2) is the coefficient of tn. If n is even, r : n/2 shows that r varies from 0 to n/2. Again, if n is odd, r : n/2 shows that r varies from 0 to (n – 1)/2. Here note that r is an integer. Combining these results we see that r varies from 0 to [n/2], where, ( ,1)

r

{

n / 2, if n is even n / 2 ! (n , 1) / 2, if n is odd 2

Hence the total coefficient of tn in the expansion of e 2 tx , t is given by [n / 2]

# r !0

1 n!

1 (,1) (2 x )n , 2 r i.e. r ! (n , 2 r ) ! r

#

Again the coefficient of tn in

n!0

[ n / 2]

# (,1)

r

r !0

.

n! 1 (2 x)n , 2 r , i.e. H n ( x). r ! (n , 2r ) ! n!

tn H n ( x) n!

1 H (x). n n!

is also

This proves the required result. 12.4. Alternative expressions for the Hermite polynomials.

d n e , x2 . dx n [Kanpur 2005, 07, 09; Garhwal 1996; Meerut 1997]

Theorem 1. Prove that Hn (x) = (–1)n e x

2

n

# nt ! H

Proof. Using the generating function, we have

n

( x) = e

2tx , t 2

. ...(1)

n!0

Expanding the function on the R.H.S. by Taylor’s theorem, (1) gives

# n!0

n

t H ( x) = n! n

L # MN ;;t

n

n!0

n

e

2tx , t

2

OP Q

n

t!0

t . n!

...(2)

Created with Print2PDF. To remove this line, buy a license at: http://www.software602.com/

12.4

Hermite Polynomials n

Equating coefficient of t in (2) and cancelling n ! from both sides, we get

LM N

n 2 Hn (x) = ; n e 2tx , t ;t

OP Q

t!0

LM N

n 2 2 = ; n e x , (x , t ) ;t

=e

x

2

LM(,1) N

n

FG H

2

IJ K

=

n n

e, (x , t)

RS T

2

OP Q

t!0

LM N

n 2 2 = e x (,1)n ; n e , ( x , t )

;x

1 1 d2 , n! 4 dx 2

2

t !0

UV W

n

[Meerut 1995, 96, 97]

2

IJ K

n

e

2 tx

=

# n!0

n

( ,1) n!

# n! 0

n

FH 1 d IK 2 dx

2n

e

2 tx

=

# n!0

n

(,1) 2 n e 2tx t = n!

# n!0

(2t x)n ! n!

# n! 0

n

( x) t . n!

...(3)

#

# n!0

n

Hn ( x) t . n!

On equating the coefficients of tn on both sides of (4), we have < 1 d 2 = 2n x n H ( x) exp > , = n > 4 dx 2 ?? n ! n! ≅ Α

n

RS FG T H

2

2 exp , 1 d n! 4 dx 2

or

RS FG T H

...(2)

2n xn n t . n!

< 1 d 2 = 2n x n n exp >> , t = 2 ? ? ≅ 4 dx Α n ! n !0

Using (2) and (3), (1) becomes

2n

2 tx 1 d e 2n 2n 2 dx

n

#H n! 0

e2tx =

n

2 (,t 2 ) n ! e2tx − e,t . ...(1) n! n! 0 2

2

( ,1) n!

#

2tx , t ! e 2tx − e ,t = e

Using the generating function, we have

Also,

OP Q

; n f ( x , t ) ! (,1) n ; n f ( x , t ) ;t n ;x n

OP Q RSexp FG , 1 d IJ UV x . T H 4 dx K W

( ,1) 2 n 2 tx . 12 n (2t ) e = e 2 tx n! 2 n!0

#

LM ; N ;t

x , we have n!

F # GH n!0

t !0

2

n

#

n!0

exp , 1 d 2 e 2tx = 4 dx

= ex

n 2 d n , x2 n x2 d ,x ! (,1) e . n e n e dx dx

Theorem II. Prove that Hn (x) = 2n Proof. Since exp x = ex =

OP Q

IJ UV KW

IJ UV x KW

n

=

...(4)

Hn ( x ) n!

2 n 1 d xn . Hn(x) = 2 exp , 4 dx 2 12.5. Hermite Polynomials for some special values of n.

or

[n / 2]

Hn (x) =

By definition,

# (,1) r !0

r

n! n , 2r (2 x ) r ! (n , 2r ) !

{

n / 2, if n is even n / 2 ! (n , 1) / 2, if n is odd Putting n = 0, 1, 2, 3, ... in (1), we have

where,

0

H0 (x) =

# (,1) r !0

r

...(1) ...(2)

0! , 2r 0 1 (2 x)0 ! 1, (2 x ) ! (,1) r ! ( , 2r ) ! 0! 0!

Created with Print2PDF. To remove this line, buy a license at: http://www.software602.com/

Hermite Polynomials

12.5 0

H1 (x) =

# (,1)

r

r !0

1

# (,1)

H2 (x) =

r

r !0

1

# (,1)

H3 (x) =

r

r !0 2

H4 (x) =

# (,1) r !0

r

1! (2 x )1 , 2r ! (,1)0 1 (2 x )1 ! 2 x, r ! (1 , 2r) ! 0 ! 1!

2! 2! 2! 2 , 2r (2 x) 2 ( (,1)1 (2 x)0 = 4x2 – 2, = (,1)0 (2 x ) r ! (2 , 2r ) ! 0!2! 1!0 ! 3! 3! 0 3! (2 x )3 ( (,1)1 (2 x )1 = 8x3 – 12x, (2 x )3 , 2r = (,1) r ! (3 , 2r ) ! 0 !3 ! 1!1! 4! 4! 4! 0 4! (2 x )4 ( (,1)1 (2 x)2 ( (,1)2 (2 x )0 ( 2 x ) 4 , 2 r = (,1) r ! ( 4 , 2r ) ! 0!4! 1!2 ! 2!0!

or H4 (x) = 16 x4 – 48 x2 + 12 12.6. Evaluation values of H2n (0) and H2n + 1 (0) = 0.

and so on.

( 2n) ! ; H2n + 1 (0) = 0. n! [Meerut 1997, Kanpur 2006, 08, 09, 10]

Theorem. Prove that H2n (0) = (–1)n

#

Proof. Using the generating function, we have

n!0

2 Hn ( x ) n t = e2tx , t . n!

...(1)

Replacing x by 0 in (1), we have

# n!0

2 Hn (0) n t = e ,t = n!

2 n

# (,nt !) n!0 2n

Equating coefficients of t

(–1) H2n (0) = (2n) ! n!

#

or

n!0

Hn (0) n t = n!

n

# (,n1!)

t2n.

...(2)

n!0

on both sides of (2), we have

n

H 2n (0) ! (–1)n

or

(2n)! . n!

Since the R.H.S. of (2) does not contain odd powers of t. equating coefficients of t2n + 1 on H 2n ( 1 (0) both sides of (2) gives =0 so that H2n + 1 (0) = 0. (2 n ( 1) ! 12.7. Orthogonality properties of the Hermite Polynomials. Theorem. Prove that :

or

z

,

e

,x

2

z

2

e , x Hn ( x) Hm ( x ) dx ! 2 n n ! Β Χ nm

,

Hn ( x ) Hm (x ) dx =

RS 0, T Β 2 n !, n

if m ∃ n if m ! n 2

or Show that Hermite polynomials are orthogonal over (– , ) with respect to the weight function e , x . Proof. Using the generating functions, we have

# n!0

n 2tx , t 2 Hn ( x ) t = e n!

#

and

m !0

Multiplying their corresponding sides, gives

##

n!0 m!0

Multiplying both sides by e

,x

2

H m ( x)

2 sm ! e 2 sx , s . m!

2 2 Hn ( x ) Hm ( x) n m t s = e2tx , t ( 2sx , s n!m!

and then integerating both sides w.r.t. ‘x’ from – to , we have

Created with Print2PDF. To remove this line, buy a license at: http://www.software602.com/

12.6

Hermite Polynomials

=

z

# # LMN n !0 m!0

e

,

=e

2ts

,

2

2

2

, x ( 2 x (t ( s ) , (t ( s )

z

,

2ts Β, =e

! Β

z

2

e , x Hn ( x ) H m ( x ) dx

e

2

, [ x , (t ( s)]

as



,

(2ts )n n! = n !0

#

dx =

dx = e

2ts

z z

,

,

e e

OP t s Q n!m! n

m

2

2

, x ( 2 x (t ( s ) , (t ( s )

,y

2

.e

2

2

2

(t ( s ) , (t ( s )

dx

dy , putting x – (t + s) = y so that dx = dy

2

e , y dy ! Β

# n !0

2n Β n n t s . nΕ

...(1)

We note that powers of t and s are always equal in each term on R.H.S. of (1). Hence when m ∃ n, equating coefficients of tnsm on both sides of (1), we have

z

z

2 2 1 ,x e, x Hn ( x ) Hm (x ) dx = 0 9 e Hn (x ) Hm ( x) dx = 0, when n ∃ m. ...(2) n!m! , , Again equating coefficients of tnsn on both sides (1), we have

1 n!n!



,

2 e , x [ H n ( x )]2 dx = 2

n

Β n!

z

or

e

,

,x

2

2 n [ Hn ( x )] dx = 2 n ! Β . ...(3)

[Kanpur 2007; Utkal 2003] 0, if n ∃ m Χnm = 1, if n ! m.

{

Let

...(4)

Combining results (2) and (3) with help of (4), we get

z

,

e

,x

2

n

Hn (x ) Hm ( x) dx = 2 n ! Β Χ nm.

12.8. Recurrence Relations (or formulae) Theorem. (i) HΦn (x) = 2n Hn–1(x) (n 8 1) ; (ii) Hn+1(x) = 2x Hn(x) – 2n Hn–1(x) (n 8 1) ; (iii) HnΦ(x) = 2x Hn(x) – Hn+1(x). (iv) HnΦΦ(x) – 2x HnΦ(x) + 2n Hn(x) = 0.

H Φ0(x) = 0. H1(x) = 2x H0(x).

tn

# H ( x) n ! ! e

Proof. (i) We know that

n

[Kanpur 2007] [Kanpur 2007, 11]

2tx ,t 2

...(1)

n !0

Differentiating both sides of (1) w.r.t. ‘x’ we have

# n!0

Thus,

n

2 2tx , t = 2t Hn Φ (x ) t = 2t e n!

# n!0

n

Hn Φ (x ) t = 2 n!

# n!0

# n!0

n

H n ( x ) t , by (1). n!

Hn ( x ) t

n (1

n!

.

Equating coefficients of tn from both sides for n = 0, (2) gives Again equating coefficient of tn from both sides for n 8 1, (2) gives

...(2) H0Φ(x) = 0.

Created with Print2PDF. To remove this line, buy a license at: http://www.software602.com/

Hermite Polynomials

12.7

H ( x) Hn Φ ( x ) = n ,1 n! (n , 1) !

HnΦ(x) = 2n Hn–1(x)

so that

(ii) We know that

e

2tx , t

2

=

[

n ! = n(n – 1) !]

n

# H (x) nt ! .

...(3)

n

n!0

2

(2x – 2t) e2tx , t =

Differentiating both sides of (3) w.r.t. ‘t’ gives or

0,1

n

# H (x) nt ! = 0. t0!

(2x – 2t)

n

H0 ( x ) (

n!0

or

2x

n (1

n

# H (x) nt ! , 2 # H (x) t n ! n

n

n!0

n!0

=

n ,1

# ntn!

n ,1

# ntn !

Hn ( x)

n!0

Hn ( x )

n !1

n ,1

# (nt , 1) ! H (x) n

...(4)

n !1

[ 0 ! = 1, Hn(x) = 1 and n ! = n(n – 1) !] n Equating coefficients of t from both sides for n = 0, (4) gives 2x H0(x) = H1(x). Again equating coefficient of tn from both sides for n 8 1, (2) gives 2x .

H n , 1 ( x) H n ( x) Hn ( 1(x ) ,2 = . n! ( n , 1) ! n!

...(5)

On multiplying both sides of (5) by n! and noting that n! = n(n – 1)!, (5) gives 2x Hn(x) –2n Hn – 1(x) = Hn + 1(x). (iii) From recurrence relations (1) and (2), we get HnΦ(x) = 2n Hn – 1(x) ...(1) and Hn + 1(x) = 2x Hn(x) –2n Hn – 1(x). ...(2) Adding (1) and (2), HnΦ(x) + Hn + 1(x) = 2x Hn(x) or HnΦ(x) = 2x Hn(x) –Hn + 1(x). (iv) Since Hn(x) is a solution of Hermite’s differential equation yΦΦ – 2xyΦ + 2ny = 0. Γ HnΦΦ(x) –2x HnΦ(x) + 2n Hn(x) = 0. 12.9. SOLVED EXAMPLES Ex. 1. Express H(x) = x4 + 2x3 + 2x2 – x – 3 in terms of Hermite’s polynomials. [Kanpur 2008, 10] 2 Sol. We know that H0(x) = 1, H1(x) = 2x, H2(x) = 4x – 2, H3(x) = 8x3 – 12x and H4(x) = 16x4 – 48x2 + 12. From these, we have x4 = (1/16) × H4(x) + 3x2 – (3/4), ...(1) 3 x = (1/8) × H3(x) + (3x/2) ...(2) x2 = (1/4) × H2(x) + 1/2 ...(3) and x = (1/2) × H1(x), 1 = H0(x). ...(4) Γ H(x) = x4 + 2x3 + 2x2 – x – 3 = (1/16) × H4(x) + 3x2 – (3/4) + 2x3 + 2x2 – x – 3, by (1) = (1/16) × H4(x)+ 2x3 + 5x2 – x – 15/4 = (1/16) × H4(x)+ 2 [(1/8) × H3(x) + 3x/2] + 5x2 – x – 15/4, by (2) = (1/16) × H4(x) + (1/4) × H3(x) + 5x2 + 2x – (15/4) = (1/16) × H4(x) + (1/4) × H3(x) + 5 [(1/4 ) × H2(x) + (1/2)] + 2x – (15/4), by (3) = (1/16) × H4(x) + (1/4) × H3(x) + (5/4) × H2(x) + 2x – (5/4), = (1/16) × H4(x) + (1/4) × H3(x) + (5/4) × H2(x) + H1(x) – (5/4) × H0(x), by (4) Ex. 2. Prove that, if m < n,

dm dx

m

{Hn ( x )} =

2m. n ! H (x). [Garhwal 2004, 05; Meerut 98] (n , m)! n , m

Created with Print2PDF. To remove this line, buy a license at: http://www.software602.com/

12.8

Hermite Polynomials

#

Sol. We know that

n!0

n 2 Hn ( x ) t = e2tx , t . n!

...(1)

Differentiating both sides of (1) w.r.t. ‘x’ m times, we have n

m

# nt ! dxd n! 0

m

m

2 2 m 2tx , t d 2tx , t = (2t) e = 2m t m {Hn ( x )} = m e dx

n

m

# nt ! dxd

Γ

{Hn ( x)} = 2 m

m

n! 0

n

# H ( x) nt ! , by (1) n

n! 0

n(m

# H ( x) t n ! n

.

...(2)

n!0

Equating coefficients of tn from both sides for m < n, (2) gives m 1 d m Hn , m ( x ) m {Hn ( x )} = 2 n ! dx (n , m) !

z

Ex. 3. Prove that

2

x e

,

m d m {H ( x)} = 2 n ! H (x). n m (n , m) ! n – m dx

or

,x

2

2

[Hn ( x)] dx = ( Β ) 2n n ! (n ( 1/ 2) .

Sol. From the recurrence relations, we know that Hn+ 1(x) = 2x Hn(x) –2n Hn – 1(x) or x Hn(x) = n Hn – 1(x) + (1/2) × Hn + 1(x) ...(1) 2 or x Hn(x) = nx Hn – 1(x) + (x/2) × Hn + 1(x) ...(2) Replacing n by n – 1 and n + 1 successively in (1), we have x Hn–1(x) = (n – 1) Hn – 2(x) + (1/2) × Hn(x) ...(3) and x Hn+1(x) = (n +1) Hn(x) + (1/2) × Hn + 2(x). ...(4) Using (3) and (4), (2) becomes x2 Hn(x) = n [(n – 1) Hn – 2(x) + (1/2) × Hn(x)] + (1/2) × [(n + 1) Hn(x) + (1/2) × Hn + 2(x)] or x2 Hn(x) = n (n – 1) Hn – 2(x) + (1/4) × Hn+2(x) + (n + 1/2) Hn(x) ...(5) 2

z

,

Multiplying both sides of (5) by e , x Hn ( x ) and then integrating w.r.t. ‘x’ from – to , gives 2

x e

,x

2

2

{Hn ( x )} dx = n (n , 1)

z

(

b g d Βi 2 = bn ( 1 / 2g d Β i 2 n !. = 0 + 0 + n ( 1/ 2

n

,

1 4

e

,x

z

e

,

n !,

2

Hn ( x ) Hn , 2 ( x ) dx

,x

2

FH

Hn ( x ) Hn ( 2 ( x ) ( n (

as



,

1 2

IK

2

z

,

e

e , x H n ( x ) H m ( x ) dx !

,x

2

2

{Hn ( x )} dx

Η ΒΙ 2

n

n ! Χ nm

n

Ex. 4. Evaluate or

z

,

xe

,x

2

H n ( x ) H m ( x ) dx .

[Meerut 1997]

Sol. From the recurrence relations, we have Hn+1(x) = 2x Hn(x) – 2n Hn – 1(x) xHn(x) = n Hn – 1(x) + (1/2) × Hn + 1(x). ...(1) 2

Multiplying both sides of (1) by e , x Hm(x) and then integrating w.r.t. ‘x’ from – to , we have

z

,

xe

,x

2

Hm ( x ) Hn ( x) dx = n

z

,

e

,x

2

Hm ( x ) Hn , 1( x ) dx ( 1 2

z

,

e

,x

2

H m ( x ) H n ( 1 ( x ) dx

Created with Print2PDF. To remove this line, buy a license at: http://www.software602.com/

Hermite Polynomials

12.9 n ,1 ( n , 1) ! Χ n , 1, m ( (1/ 2) . Β 2 n ( 1 ( n ( 1) ! Χ n ( 1, m = n Β2

% & ∋

,

2 ) e , x H n ( x) H m ( x ) dx ! Β 2n n ! Χnm ∗ +

Β 2 n , 1 n ! Χn , 1, m ( Β 2 n ( n ( 1) ! Χn ( 1, m .

= 2

Β n!∆

Ex. 5. Show that Pn ( x ) =



2

t n e ,t H n ( xt ) dt .

0

Sol. By the definition of Hermite polynomial (on replacing x by xt here), we have [n / 2]

Hn(xt) =

# (,1)

n! n , 2r (2 xt ) . r ! (n , 2r) !

r

r !0

...(1)

Making use of (1), we have 2

Β n!∆

0

=

2

Β n!∆

=

#

% & ∋



r!0

0

ϑ n / 2Κ

=

t e

#

n/2

ϑn / 2Κ n , 2r ( 1 51[n /2] n ! 2n , 2r xn , 2r n , 2r 01 2 (,1)r xn , 2r (,1)r t 6 2 dt = r !(n , 2r)! Β r ! (n , 2r )! 17 r !0 13 r !0

#

#



2

0

e,t t 2n , 2r dt

2 n , 2 r ( 1 (,1)r x n , 2 r 1 Λ (n , r ( 1/ 2) Β r ! (n , 2r )! 2 2

e,t t 2 n , 2 r dt !



Β r ! (n , 2r )!

# (,1)

r

r !0

Η

2

e ,t t 2(n , r ( 1/ 2) , 1 dt ! 12 Λ n , r (

0

2 n , 2 r (,1)r x n , 2 r

r !0

=

n ,t 2

0

ϑ n / 2Κ

2

t n e,t H n ( xt ) dt

.

(2 n , 2r ) !

2 ) e,t e 2 x , 1 dt ∗ +

(2n , 2r) ! n , 2r x = Pn(x), by the definition of Legendre polynomial. 2 n r ! (n , 2r) ! (n , r) !

Ex. 6. If Μn (x) = e , x

z z

(a)

,

(b)

,

2

/2

Hn(x), where Hn(x) in a Hermite’s polynomial of degree n, then n

Μ m ( x) Μ n ( x ) dx ! 2 ! Β Χ m, n

z

,

R| 0, S| 2 n ! Β , T,2 (n ( 1) ! Β ,

Μ m ( x ) Μ n ( x ) dx = =

if m ∃ n Ν 1 if m ! n , 1 if m ! n ( 1

n ,1

Μ m ( x ) Μ n' ( x ) dx !

Sol. Part (a) Given Γ

Ι , as Λ(x) ! 2 ∆0

< by using duplication formula, if n is = > ? > positive integer then Λ < n + 1 = ! (2 n )! Β ? > ? > ? 2 Α 22 n n ! ≅ ≅ Α

Β

22 n , 2 r ( n , r ) !

1 2

n

z z

,

,

Μ n (x) ! e e e

2

, x /2

,x

2

, x2 / 2

Hm ( x ). e

2

Hn ( x ) .

,x /2

...(1)

Hn ( x ) dx , using (1)

n H m ( x ) H n ( x ) dx = 2 n ! Β Χ nm,

[using orthogonal properties of the Hermite polynomials]

Created with Print2PDF. To remove this line, buy a license at: http://www.software602.com/

12.10

Hermite Polynomials

Μn(x) = e

Part (b) Here

, x2 / 2

,x Μm(x) = e

and

ΜnΦ(x) = , xe , x

From (2),

From recurrence relations, we have and Using (5) and (6), (4) reduces to ΜnΦ(x) = ,e , x

z

,

=n

Μ m ( x ) Μ Φn ( x ) dx =

z

e

,

,x

2

z

,

/2

/2

...(2)

Hm ( x) .

Hn ( x) ( e , x

...(3) 2

/2

HnΦ ( x )

e

Hn , 1 Hm ( x) dx ,

2

/2

2

,x /2

1 2

...(5)

HnΦ(x) = 2n Hn–1(x).

...(6) 2 /2

. 2 n H n , 1 ( x)

[ n H n , 1 ( x ) , (1/ 2) . H n ( 1 ( x)]

Hm ( x). e

z

...(4)

xHn(x) = n H n , 1 ( x) ( (1/ 2) . H n ( 1 ( x)

[n H n , 1 ( x ) ( (1/ 2) . H n ( 1 ( x)] ( e , x

ΜnΦ(x) = e , x

or Γ

2 /2

2

2

Hn ( x)

e

,

,x

2

2

,x /2

[n Hn , 1( x ) ,

...(7)

1 H ( x)] dx , by (3) and (7) 2 n (1

Hn ( 1 ( x ) Hm ( x) dx

Β Χ n , 1. m , (1/ 2) . 2 n ( 1 ( n ( 1) ! Β Χn ( 1 , m , using orthogonal

= n 2n – 1(n – 1)! properties =2

n ,1

n

n ! Β Χ n , 1, m , 2 (n ( 1) ! Β Χ n ( 1 , m

R| 0, = S 2 n ! Β, |T,2 (n ( 1) ! Β ,

if m ∃ n Ν 1 if m ! n , 1 if m ! n ( 1

n ,1

n

Ex. 7. Using the Rodrigue’s formula for Hn(x) and integrating by parts iteratively, show that Μ=

z

2

exp (,x ) Hn (x) Hm(x) dx ! 0, if m ∃ n

,

n = 2 n ! Β , if m = n

n

2 2 x d ,x . Hn(x) = (–1)n e n e dx

Sol. Rodrigue’s formula for Hn(x) is given by

n

Γ

z

ΓΜ=

,

LMRF d MNSTGH dx FG d H dx

n ,1

= ( ,1) n

= 0,

2 exp (–x2) Hn(x) = (–1)n d n e , x . dx n 2 n 2 (,1) n exp d n e , x H m ( x ) dx (,1) exp (, x ) Hn ( x) Hm ( x ) dx = , dx

n ,1

z

,

= ( ,1) n (,1) n

n ,1 n ,1

z

,

e

, x2

e,x

IJ H ( x )UV K W

,

m

2

IJ d H K dx

,

m (x)

z

,

z

FG d H dx

n ,1 n ,1

RS T

e,x

2

IJ d H K dx

m (x)

OP Q

...(1)

UV W

dx , integrating by parts

dx , as first term is zero due to presence of e , x

2

d n H ( x ) . e , x 2 dx , integrating by parts iteratively m dx n

Created with Print2PDF. To remove this line, buy a license at: http://www.software602.com/

Hermite Polynomials

12.11

Γ

z

Μ=

n

2 d ,x dx . n Hm ( x ) . e dx

,

...(2)

d n H (x ) m dx n Μ = 0. Μ = 0. m∃n

We know that Hm(x) is a polynomial of degree m. Hence if n > m, Γ If n > m, then from (2), Since Μ is symmetrical in m and n, it follows that if m > n, From (3) and (4), we see that Μ = 0, When, m = n, (2) gives Μ =

z

,

d n H ( x) . e, x2 dx = n dx n

= 2n n ! Β ,

as

z

2 n!e



e , x dx ! Β

,

,

n

,x

2

n

#

Hk ( x ) Hk ( y) k

2 k!

k !0

or

=

as

dn dx n

...(3) ...(4) ...(5)

H n ( x) ! 2n n !

2

Γ Μ = 2n n ! Β From (5) and (6), we get the required results. Ex. 8. Show that

dx ,

then for

= 0.

when

m=n

Hn ( 1 (y) Hn (x ) , Hn ( 1 ( x) Hn ( y) 2

n (1

n ! ( y , x)

...(6)

.

Sol. From the recurrence relations, we have Hn+1 (x) = 2x Hn(x) –2n Hn–1(x) x Hn(x) = n Hn–1 (x) + (1/2) × Hn+1(x). ...(1) Replacing x by y in (1), y Hn (y) = n Hn–1 (y) + (1/2) × Hn+1 (y). ...(2) Multiplying (2) by Hn(x) and (1) by Hn(y) and then subtracting, we have (y – x) Hn (x) Hn(y) = (1/2) × [Hn+1 (y) Hn (x) – Hn+1 (x) Hn (y)] – 2 [Hn–1 (x) Hn (y) – Hn–1 (y) Hn (x)]. ...(3) Putting n = 0, 1, 2, 3, ...., (n – 1), n successively in (1), we have (y – x) H0 (x) H0 (y) = (1/2) × [H1 (y) H0 (x) – H1 (x) H0 (y)] – 0 ...(E0) (y – x) H1 (x) H1 (y) = (1/2) × [H2 (y) H1 (x) – H2 (x) H1 (y)] – [H0 (x) H1(y) – H0 (y) H1(x)]. ...(E1) (y – x) H2 (x) H2 (y) = (1/2) × [H3 (y) H2 (x) – H3 (x) H2 (y)] –2 [H1 (x) H2 (y) – H1 (y) H2 (x)]. ...(E2) ... ... ... ... ... ... ... ... ... (y – x) Hn – 1 (x) Hn – 1 (y) = (1/2) × [Hn (y) Hn – 1(x) – Hn (x) Hn – 1(y)] –2 [Hn – 2(x) Hn – 1(y) – Hn – 2 (y) Hn – 1(x)]. ...(En – 1) (y – x) Hn (x) Hn (y) = (1/2) × [Hn + 1 (y) Hn (x) – Hn + 1 (x) Hn (y)] –2 [Hn – 1(x) Hn (y) – Hn – 1 (y) Hn (x)]. ...(En) 1 1 1 1 1 , Multiplying (E0), (E1), (E2), ..., (En – 1), (En) by 1, 2 .1! , 2 , 3 , ...., n , 1 2 . 2! 2 . 3! 2 (n , 1) ! 2 n. n !

respectively and adding (note that all terms, except the first term on R.H.S. of (En), cancel in pairs), we have Hn ( 1 (y) Hn (x ) , Hn ( 1 ( x) Hn ( y) Hk (x ) Hk ( y) ( y , x) = k 2 k! 2n ( 1 n ! k !0

#

or

# k !0

Hk ( x ) Hk ( y) k

2 k!

=

Hn ( 1 (y) Hn (x ) , Hn ( 1 ( x) Hn ( y) 2n ( 1 n ! (y , x )

.

Created with Print2PDF. To remove this line, buy a license at: http://www.software602.com/

12.12

Hermite Polynomials

x 1. Show that Hn (x) = 2n + 1 e

2

EXERCISE

z

e

x

,t

2

t

n (1

Pn

FH x IK dt . t

2. If f (x) is a polynomial of degree m, show that f (x) may be expressed in the form m

f (x) =

#C

r

Hr ( x ),

Cr =

where,

r !0

z

Deduce that

e

,

,x

2

1 2 r! Β r

z

,

e

,x

2

f ( x) Hr ( x) dx .

f (x) Hn (x) = 0, if f (x) is a polynomial of degree less than n.

3. Using the generating function for Hermite polynomials, evaluate the values of (i) H0 (x) (ii) H1 (x) (iii) H2 (x) (iv) H3 (x). 2

4. Show that Hn (x) defined by e 2tx , t !

# n!0

Hn ( x) n t satisfies the differential equation n!

HnΦΦ (x) –2x HnΦ (x) + 2x Hn(x) = 0. 5. The Hermite polynomial is defined for integral values of x by the identity 2

e 2tx , t !

# n!0

Hn ( x) n t . Show that Hn (x) satisfies the differential equation HnΦΦ (x) –2x HnΦ(x) n!

n 2 2 + 2x Hn(x) = 0 and Hn (x) is given by Hn(x) = (,1)n e x d n e , x .

Further show that 6. Show that

z

e

,

,x

2

z z

,

2

n

{Hn ( x )} dx = 2 n !

,

dx

2

{Hn ( x )} e

e

,x

2

,x

2

dx =

Β 2n n ! .

n

dx ! 2 n ! Β .

7. Show that (a) H5 (x) = 32 x5 – 160 x3 + 120 x. (b) H6 (x) = 64 x6 – 480 x4 + 720 x2 – 120. 8. Show that for n = 0, 1, 2 .... 2 n ( 1 . (,1)n e x (i) H2n(x) = (Β)

2

(ii) H2n(x) =

2n ( 2

n

z

2

. (,1) e (Β)

3

e ,t t2n cos 2xt dt.

Ο

x

2

z

e

,t 2

t2n + 1 sin 2xt dt.

9. Prove that Hn (–x) = (–1)n Hn(x). [ n / 2]

n ! H n, 2k ( x)

k !0

2n k ! (n , 2k )!

10. Prove that x n ! 4

[Kanpur-2004, 09] and express x6 in terms of Hermite polynomials. [Kanpur 2004, 09]

11. Show that d n H o ( x) / dxn ! 2n n ! 12. Show that



x

0

H n ( y )dy !

[Kanpur-2004]

Ο

1 ϑ H n(1 ( x) , H n,1 (o)Κ 2(n ( 1)

(Kanpur 2008, 09) (Kanpur 2010)

Created with Print2PDF. To remove this line, buy a license at: http://www.software602.com/

13 Laguerre Polynomials 13.1. Laguerre’s equation and its solution. Laguerre’s equation of order n is 2

[Meerut 1995, 96]

2

x(d y / dx ) ! (1 x )(dy / dx ) ! ny = 0, ...(1) where n is a positive integer. We obtain a solution of (1) which is finite for all values of x and which tends to infinity no faster than ex/2 as x # ∃. For this we use the well known method of Frobenius. Let the series solution of (1) be ∃

y = & Cm x k ! m ,

C0 ∋ 0.

m%0

...(2)

Differentiating (2) and then putting the values of y, dy/dx and d2y/dx2 in (1), we have

x or



k!m & Cm (k ! m)(k ! m 1)x

m%0 ∃

k!m & Cm (k ! m)(k ! m 1)x

2

! (1

x)



k!m & Cm (k ! m)x

1

!

m%0



& Cm (k ! m)x



k!m 1

m%0 ∃

or

2 k!m & Cm (k ! m) x

=0

m%0



& Cm (k ! m)x

k!m

k!m 1

& Cm (k ! m)x

or



& Cm x

m%0

m%0 1

!n

{(k ! m 1) ! 1}

m%0 ∃

& Cm x

k!m

k!m

(k ! m

!n



& Cm x

m%0

k!m 1

=0

n) = 0

m%0 1

m%0



k!m = 0. & Cm (k ! m n)x

m%0

...(3)

(3) is an identity. To get the indicial equation, we equate to zero the coefficient of the smallest power of x, namely xk – 1 in (3) and obtain C0k2 = 0, so that k2 = 0 ( C0 ∋ 0) ...(4) From (4) we see that the roots of indicial equation are equal. Next equating to zero the coefficient of xk + m – 1, we have k!m 1 n Cm 1 . ...(5) 2 (k ! m) The two independent solutions in the present case are (y)k = 0 and ((y/(k)k = 0. But ((y/(k)k = 0 involves a term of the form log x, and so is infinite when x = 0. Since we wish to obtain a solution finite for all finite values of x, we consider only the former solution, i.e., (y)k = 0, as follows. m 1 n Cm 1 With k = 0, (5) and (2) reduce to Cm = ...(6) 2 m

Cm(k + m)2 – Cm – 1(k + m – 1 – n) = 0

and

y=



m

or

& Cm x = C0 + C1x + C2x2 + ....

m%0

Cm =

...(7)

13.1

Created with Print2PDF. To remove this line, buy a license at: http://www.software602.com/

13.2

Laguerre Polynomials

Putting m = 1, 2, 3, ... in (6), we have 1 n (n 1) ( 1) n 2 n(n 1) nC0 , C1 = 2 C0 % C2 = 2 C1 % ) ( 1)nC0 % ( 1) 2 2 C0 , 1 (1!)2 2 2 (2 !) 2 n (n 2) n(n 1) 3 n(n 1)(n 2) ) ( 1)2 C0, C3 = 2 C2 % 2 2 C0 % ( 1) 3 3 (2 !) (3!)2 .... .... .... .... .... r ! 1) C0 , for r ∗ n. ( r !) 2 = Cn + 3 = ... = 0.

n ( n 1)...(n

Cr = ( 1)r

Thus,

Also, Cn + 1 = Cn + 2 With these values, (7) reduces to

LM N

y = C0 1 ∃

= C0

&

n x ! n(n 1) x 2 ! ... ! ( 1)r n(n 2 2 (1!) (2 !)

n(n 1)....(n ( 1) (r !)2 r%0 r

r ! 1)

n

r

x = C0

&(

1)r

(n

n

y = C0

&(

1)

r )(n

r )( n

r 1)...3 + 2 +1

r 1)...3 + 2 +1.( r !) 2

xr

n! r 2 x . r )!(r !)

r

(n

r%0

OP Q

x r ! ....

r ! 1)( n

n( n 1)....( n

r %0

Thus,

r ! 1)

1)...(n 2 (r !)

Taking C0 = 1, we define the corresponding solution as the Laguerre polynomial of order n, n

Ln(x) =

and denote it by Ln(x). Thus, we have

&(

1)

r

r%0

(n

n! r 2 x . r)! (r !)

13.2A. Laguerre polynomial of order (or degree) n. Definition. Laguerre polynomial of order n is denoted and defined by n

Ln(x) = & ( 1)r r%0

n! (n

r )! (r !)2

[Meerut 1997]

xr .

13.2B. Alternative definition of Laguerre polynomial of order (or degree) n. In Art. 13.1, we took C0 = 1 to define Ln(x) and the same is given in Art. 13.2A. However, some authors take C0 = n! to define Ln(x). Thus, another definition of Laguerre polynomial is n

Ln(x) = &

( 1) r (n !)2 x r

r % 0 (n

r )! (r !)2

.

13.3. Generating function for Laguerre polynomials. Theorem. Prove that

∃ exp { xt / (1 t )} n = & Ln ( x) t . 1 t n%0

Proof. In order to prove the required result we must show that the coefficient of tn in the expansion of L.H.S. (in ascending powers of t) is Ln(x). Now, r



, xt − 1 1 exp { xt / (1 t )} = . / . , 1 t 1 t r % 001 t 1 r!

&



=

&

r%0

r

( 1) r r x t (1 t ) r!

(r ! 1)



=

& ( r1!)

r%0

r

r r

xt

as ∃

&

s%0

exp x % e x %



xr r! r%0

&

(r ! s)! , by the binomial theorem r ! s!

Created with Print2PDF. To remove this line, buy a license at: http://www.software602.com/

Laguerre Polynomials ∃



& &(

=

13.3

1)r

r%0 s%0

(r ! s )!

xr t r ! s . (r !) s ! 2

Let r be fixed. Then the coefficient of tn can be obtained by setting r + s = n i.e. s = n – r. r n! r Hence, for the chosen fixed value of r, the coefficient of tn is ( 1) x . 2 (r !) (n r)! Now, s 2 0 3 n – r 2 0 3 r ∗ n, which gives all allowed values of r for finding coefficient of tn. Thus, the total coefficient of tn is given by n

&(

1)

r%0

r

n! r x 2 (r !) (n r )!

i.e.

Ln(x), by definition 13.2A

This proves the desired result. exp { xt / (1 t )} = 1 t

Remark. If we use definition 13.2B, then



Ln ( x) t n . n! n%0

&

13.4. Alternative expression for the Laguerre polynomials Ln(x) =

Prove that

ex d n n x ( x e ) . [Kanpur 1992; Meerut 1992, 93; Garhwal 2005] n ! dx n

Proof. By the Leibnitz’s theorem, we have Dn(uv) = d n (uv) / dxn = Dnu.v + nC1Dn – 1u.Dv + ... + nCrDn – ru.Drv + ... + uDnv n

Dn(uv) =

i.e.,

&

n

Cr D n

r

u Drv .

r%0 n

x n ex n 4 e d n ( x ne x ) = Cr D n n! dx n! r % 0

&

x n = e & n Cr n! r % 0 {n

n! xn (n r )}!

(n r)

x D r e x , . by (1)

. ( 1)r e x ,

ex n! n! r ) x ) ( 1) r e n ! r !( n r )! r ! r%0

&

x

=

Dn xm %

as

n

n

=

r n

m! xm (m n)!

n

and D n e ax % a n e ax

r

&

( 1) n ! r x = Ln(x), by definition. 2 ( r !) ( n r )! r%0 n

x d n x Ln(x) = e n (x e ) . dx

Remark. If the use definition 13.2B, we get 13.5. First few Laguerre polynomials.

e x d n ( x ne x ) . n! dx n Putting n = 0, 1, 2, 3, 4, ... in succession in (1), we obtain Ln(x) =

We know that (Refer Art. 13.4)

x L0(x) = e ( x 0 e x ) % 1, 0!

L2(x) =

x

2

x

LM N

x L1(x) = e d ( xe x ) % e x (e 1! dx

OP Q

x

e d 2 x e d d 2 x e d (x e ) % (x e ) % (2 xe 2 ! dx 2 2 ! dx dx 2 ! dx

x

x

2

...(1)

x

xe ) % 1

x,

x

x e )

Created with Print2PDF. To remove this line, buy a license at: http://www.software602.com/

13.4

Laguerre Polynomials

= L3 ( x ) %

ex [2e 2!

x

! 2 x ( e x ) {2 xe

ex d 3 3 x ex d 2 ( x e ) % 3! dx3 3! dx 2

%

ex d [(6 x 3! dx

3 x 2 )e

4

x

(3x 2

x

! x 2 ( e x )}] %

x 2 5d 3 x 6 e d 2 7 dx ( x e ) 8 % 3! dx 2 (3x e 9 :

= (e x / 3!) ) [(6 12 x ! 3x 2 )e x

x

%

4 ! dx

ex d 2 [(12 x2 4! dx 2

4 x 3 )e

x

x

(6 x

OP Q

(4 x3

x3e x ) %

ex d 5 d {(3x 2 3! dx 79 dx

6 x 3 )e x }8 :

ex d {(6 x 6 x 2 ! x 3 ) e x } 3! dx 6 x2 ! x3 )e x ] = (6 – 18x + 9x2 – x3)/3!,

x 3 L4(x) = e d 4 ( x 4 e x ) % e d 3 d ( x 4e x ) % e d 3 [4 x 3e

4 ! dx

4 x ! x2 ) ,

x 3 )e x ] %

LM N dx

3

3

1 (2 2!

x

4

4 ! dx

x x 4 )e x ] = e d

x

x e ]=

LM N

d 2 (12 x 4 ! dx dx

ex d 2 5 d {(4x3 4! dx2 79 dx 3

4

8 x ! x )e

x

6 x 4 )e x }8 :

OP Q

x x = e d [(24 x 24 x 2 ! 4 x 3 )e x (12 x 2 8x 3 ! x 4 )e x ]= e d [(24x 36 x 2 ! 12 x 3 x 4 )e x ] 4 ! dx 4 ! dx x 2 3 x 2 3 4 x (24x 36x ! 12x x )e ] = (24 – 96x + 72x2 – 16x3 + x4)/4! = (e / 4!)[(24 72x ! 36x 4x )e

n Ln(x) = e x d n ( x n e x ) dx 2 and proceed as before, then we have L0(x) = 1, L1(x) = 1 – x, L2(x) = 2 – 4x + x , L3(x) = 6 – 18x + 9x2 – x3, L4(x) = 24 – 96x + 72x2 – 16x3 + x4 and so on. 13.6. Orthogonality properties of Laguerre’s polynomials.

Note : If we use the result given remark of Art. 13.4, namely,

Theorem. Prove that

0

ze ∃ 0

prove that

–x

0



and or



; e ; e x

–x

Ln ( x) Lm ( x ) dx % 0, if m ∋ n

{Ln ( x)}2 % 1

(Kanpur 2004)

0, m ∋ n Ln (x )Lm ( x ) dx = Β). 1 t 1 t 1 t /1 0 ? Χ



r & L ( x) t , using (1) and writing r in place of n 1 t r%0 r

= t(1 t)

1







r%0

s%0

r%0

r s r & Lr ( x) t = t & t & Lr ( x) t , by the binomial theorem

4







& Ln≅ (x) t = – & & Lr ( x) t r ! s ! 1 .

n%0

n

...(2)

r%0 s%0

Clearly the coefficient of tn on L.H.S. of (2) is L≅n(x). We now obtain the coefficient of tn on R.H.S. of (2). Let r + s + 1 = n so that s = n – r – 1. Hence, for a fixed value of r, the coefficient of tn on R.H.S. of (2) is –Lr(x). But s 2 0 3 n – r – 1 2 0 3 r ∗ n – 1, which gives all the values of r for which –Lr(x) is the n 1

coefficient of tn. Hence the total coefficient of tn on R.H.S. of (2) is given by

& Lr ( x)

r%0

Thus, equating the coefficients of tn from both sides of (2), we get 13.9. SOLVED EXAMPLES Ex. 1. Prove that (i) Ln(0) = 1.

[Meerut 1993]

(ii) Ln(0) = n!.

L≅n(x) =

n 1

& Lr ( x) .

r%0

[Meerut 1996]

Created with Print2PDF. To remove this line, buy a license at: http://www.software602.com/

13.8

Laguerre Polynomials ∃ n & t Ln ( x) = 1 e

Sol. (i) We know that (Refer Art. 13.3.) Putting x = 0 in (1),



n & t Ln (0) =

n%0

1 % (1 t ) 1 t

1





n%0

n%0

tx /(1 t )

...(1)

1 t

n%0

% 1 ! t ! t 2 ! ., by the binomial theorem

n & t n Ln (0) = & t ,

or

...(2)

which is an identity. Equating the coefficient of tn on both sides of (2), we get t Ln ( x ) = 1 e n ! 1 t n%0

&

(ii) We know that (Refer remark of Art. 13.3) t n Ln (0) = (1 t ) n! n%0 ∃

&

Putting x = 0 in (3),

n



1

%

Ln(0) = 1. tx /(1 t )

.

...(3)



n & t , by binomial theorem

n%0

Equating the coefficient of tn on both sides, we get (1/n!)Ln(0) = 1 or Ln(0) = n!. Ex. 2. Prove that (i) L≅n(0) = –n. (ii) L∆n(0) = {n(n – 1)}/2 Part (i). Since Ln(x) is a solution of the Laguerre’s equation xy∆ + (1 – x)y≅ + ny = 0, ...(1) we get xLn∆(x) + (1 – x)Ln≅(x) + nLn(x) = 0. ...(2) Putting x = 0 and using Ln(0) = 1, (3) gives 0 + (1 – 0)Ln≅(0) + n × 1 = 0 or Ln≅(0) = –n.

1

Part (ii). We know that

1 t

Differentiating twice w.r.t. ‘x’, (3) gives



n & t Ln( x) .

exp{–xt/(1 – t)} =

exp{ xt / (1 t )} 1 t

FG H

... (3)

n%0

t 1

IJ tK

2



= & Ln ≅≅ ( x) t n . ...(4) n%0



& Ln ≅≅ (0) t n = t2(1 – t)–3. n%0

Putting x = 0 in (4), we have

...(5)

Equating the coefficients of tn on both sides of (5), we get Ln∆(0) = coefficients of tn in the expansion of t2(1 – t)–3 = coeff. of tn – 2 in the explansion of (1 – t)–3 =

( 3)( 3 1)...{ 3 (n (n 2)!

2) ! 1}

( 1)n

2

=

( 3)( 4)...( n ) ( 1) n ( n 2)!

2

%

3 + 4 + 5 ... n ( 1) n ( n 2)!

2

( 1) n

2

1 + 2 + 3 + 4 ++ + n n(n 1)(n 2)! n(n 1) n! = % % . [ (–1)2n – 2 = 1] 1+ 2 + (n 2)! 2(n 2)! 2(n 2)! 2 Ex. 3. Prove that xLn∆(x) + (1 – x)Ln≅(x) + Ln(x) = 0 and hence deduce that Ln≅(0) = –n. Sol. Since Ln(x) satifies Laguerre’s equation x(x2y/dx2) + (1 – x)(dy/dx) + ny = 0, therefore xLn∆(x) + (1 – x)Ln≅(x) + nLn(x) = 0. ...(1) Replacing x by 0 in (1), we have Ln≅(0) + nLn(0) = 0 or Ln≅(0) = –n, as Ln(0) = 1. Ex. 4. Expand x3 + x2 – 3x + 2 in a series of Laguerre polynomials. [Meerut 1994] Sol. We know that L0(x) = 1, L1(x) = 1 – x, L2(x) = (2 – 4x + x2)/2 and L3(x) = (6 – 18x + 9x2 – x3)/6. 4 x3 = 6 – 18x + 9x2 – 6L3(x), ...(1)

=

Created with Print2PDF. To remove this line, buy a license at: http://www.software602.com/

Laguerre Polynomials

13.9 2

x = 4x – 2 + 2L2(x), ...(2) x = 1 – L1(x) and 1 = L0(x). ...(3) Now, x3 + x2 – 3x + 2 = 6 – 18x + 9x2 – 6L3(x) + x2 – 3x + 2, by (1) = 8 – 21x + 10x2 – 6L3(x) = 8 – 21x + 10[4x – 2 + 2L2(x)] – 6L3(x), by (2) = –12 + 19x + 20L2(x) – 6L3(x)= –12 + 19[1 – L1(x)] + 20L2(x) – 6L3(x), by (3) = 7 – 19L1(x) + 20L2(x) – 6L3(x)= 7L0(x) – 19L1(x) + 20L2(x) – 6L3(x), by (3). Ex. 5. Find the values of (i)

ze ∃ 0

x

Sol. By results of Art 13.6, we have

z



x

(i) 0 e L3 ( x)L5 ( x ) dx = 0 Ex. 6. Taking Ln(x) to be the coefficient of tn in the expansion of

z

FG IJ H K

1 exp xt , prove that 1 t 1 t

Sol.

z

Γ Η2

Φ

Γ Η2

Φ

∃ 0

x

∃ 0

x

2

{L4 ( x )} dx . 2

{L4 ( x )} dx = 1.

tan Ε

e Ln (tan ΕΙ Lm (tan ΕΙ dΕ = 0.

Proof. By definition, we have F(!, ∋; ∗; x) =

(

)

n&0

=

(!)n (∋)n x n = . (∗ )n n!

(

)

n&0

(!)n

%(∋ ∃ n) %(∗ ) x n . . , by Art. 14.1 %(∋) %(∗ ∃ n) n !

( %( ∗ ) %(∋ ∃ n)%(∗ # ∋) x n (!)n . , multiplying and dividing by %(∗ – ∋) ) %(∗ # ∋)%(∋) n & 1 %(∋ ∃ n ∃ ∗ # ∋) n !

% (∗ )

(

+

1 ∋ ∃ n #1

= %(∋) %(∗ # ∋) ) (!)n 30 t n&0

(1 # t )∗ # ∋ # 1 dt

, xn! , where ∗ – ∋ > 0, ∋ + n > 0 so that ∗ > ∋ > 0 n

4 6 8

5 %( p)% (q) 1 & B( p, q) & 30 t p # 1 (1 # t ) q # 1 dt 7 %( p ∃ q ) 9

Created with Print2PDF. To remove this line, buy a license at: http://www.software602.com/

Hypergeometric Function

=

%( ∗ ) %(∋)%(∗ # ∋)

14.5

z

1

0

t

∋ #1

(1 # t)

F GG ) (!) H (

n

n&0

F (!, ∋, ∗ , x) =

Thus,

∗ #∋ #1

%(∗ ) %(∋)%(∗ # ∋)

z

1

0

I JJ K

( xt )n dt n!

t ∋ # 1(1 # t)∗ # ∋ # 1(1 # xt)#! dt

...(1)

(#!)(#! # 1)...( #! # n ∃ 1) (# xt )n n! !(! ∃ 1)...(! ∃ n # 1) x nt n & ( #1) n : (#1) n x n t n & (!) n , by Art.14.1] n! n!

[ the general term in the expansion of (1 # xt ) #! &

Also, B(∋, ∗ – ∋) =

% (∋) % (∗ # ∋) % (∋) % (∗ # ∋) % (∗ ) 1 & = . ...(2) ; B(∋, ∗ # ∋) % (∋ ∃ ∗ # ∋) % (∗ ) %(∋)%( ∗ # ∋)

Using (2), (1) may be re–written as F(!, ∋; ∗; x) =

1 B(∋, ∗ # ∋)

z

1

0

t ∋ # 1(1 # t)∗ # ∋ # 1(1 # xt)#! dt.

...(3)

Thus (1) and (3) are the required results. F(!, ∋; ∗; 1) =

14.10. Gauss Theorem.

%(∗ ) %(∗ # ∋ # !) %(∗ # !) %(∗ # ∋)

Proof. From Art. 14.9, for x = 1 we have

z z

F(!, ∋; ∗; 1) =

%(∗ ) %(∋)%(∗ # ∋)

=

%(∗ ) %(∋)%(∗ # ∋)

1

0

1

0

t∋ # 1(1 # t)∗ # ∋ # 1(1 # t)#! dt

t

∋ #1

(1 # t )

∗ #∋ # ! #1

dt =

< >≅ =

[Kanpur 2005, 06, 07 Ranchi 2010]

%( ∗ ) % (∋) % ( ∗ # ∋ # !) % (∋)% (∗ # ∋) % (∋ ∃ ∗ # ∋ # !) 1 p #1

30 t

(1 # t )q # 1 dt & B( p, q) &

% ( p) % (q) = %( p ∃ q ) ?Α

%(∗ ) %(∗ # ∋ # !) . %(∗ # !) %(∗ # ∋)

14.11. Vandermonde’s theorem.

F(–n, ∋; ∗; 1) =

( ∗ # ∋)n . ( ∗ )n

Proof. From Art. 14.10, with ! = –n, we get F(–n, ∋; ∗; 1) =

=

% ( ∗ ) %( ∗ # ∋ ∃ n # 1) (∗ # ∋ ∃ n # 2)...( ∗ # ∋) % (∗ # ∋) % ( ∗ ) % ( ∗ # ∋ ∃ n) = (∗ ∃ n # 1)( ∗ ∃ n # 2)...∗% ( ∗ )% ( ∗ # ∋) %( ∗ ∃ n) %( ∗ # ∋) ( ∗ # ∋)n 1∗ # ∋ ∃ n # 1) ( ∗ # ∋ ∃ n # 2)...( ∗ # ∋) = , by Art. 14.1 ( ∗ )n (∗ ∃ n # 1) ( ∗ ∃ n # 2)...∗

Created with Print2PDF. To remove this line, buy a license at: http://www.software602.com/

14.6

Hypergeometric Function

14.12. Kummer’s theorem. F(!, ∋; ∋ – ! + 1; –1) =

%1∋ # ! ∃ 1) %(∋ΒΧ ∃ 1) . % (∋ ∃ 1) % (∋ / 2 # ! ∃ 1)

[Purvanchal 2005]

Proof. From Art. 14.9, with x = –1 and ∗ = ∋ – ! + 1, we get F(!, ∋; ∋ – ! + 1; –1) =

%1∋ # ! ∃ 1) 1 ∋ # 1 %1∋ # ! ∃ 1) 1 ∋ #1 2 #! t (1 # t )∋ # ! ∃ 1 # ∋ # 1 (1 # t )#! dt = 3 t (1 # t ) dt 3 0 % (∋) % (1 # !) 0 % (!) % (∋ # ! ∃ 1 # ∋)

=

%1∋ # ! ∃ 1) 1 1/ 2 ∋ # 1 #! 3 (u ) (1 # u ) ( du / 2 u ) % (∋) % (1 # !) 0

=

%(∋ # ! ∃ 1) % (∋ΒΧ2 %(1 # !) % (∋ # ! ∃ 1) 1 (∋ / 2) # 1 . (1 # u )1 # ! # 1 du = 3u 2%(∋) %(1 # !) % (∋ / Χ ∃ 1 # !) 2% (∋) % (1 # !) 0

(putting t2 = u so that dt = du / 2 u )

FG H =

z

1

0

u p # 1(1 # u)q # 1 & B( p, q) &

%1 p)%(q) %( p ∃ q )

IJ K

% (∋ # ! ∃ 1) (∋ / 2) % (∋ / 2) %(∋ # ! ∃ 1) %(∋ / 2 ∃ 1) = %(∋ Β Χ ∃ / # !2 %(∋ ∃ /) %(∋ΒΧ ∃ / # !2∋ %(∋2

14.13. More about the confluent hypergeometric function and solution of confluent hypergeometric equation The hypergeometric differential equation is (x2 – x)y∆ + [(1 + ! + ∋)x – ∗]y− + !∋y = 0. ...(1)

FG H

x 1#

Replacing x by x/∋ in (1), we get

IJ K

RS FG T H

IJ UV KW

! ∃1 x y −− ∃ ∗ # 1 ∃ x y− # !y = 0 ∋ ∋

...(2)

Its solution is represented by the function F(!, ∋; ∗ ; x/∋) When ∋ Ε (, the equation (2) reduces to xy∆ + (∗ – x)y− – !y = 0

...(3)

Lim F 1 !, ∋ ; ∗ ; x / ∋ 2 .

whose solution is given by

...(4)

∋Ε(

The equation (3) is known as the confluent hypergeometric differential equation or Kummer’s equation.

FG H

IJ FG KH

IJ FG K H

IJ K

∋(∋ ∃ 1)(∋ ∃ 2)...(∋ ∃ r # 1) 1 1 ∃ 2 ... 1 ∃ r # 1 & 1 = Lim = Lim 1 ∃ r ∋Ε 0 ∋ Ε( ∋ ∋ ∋ ∋ ∋ ∋...r times ∋ Hence solution (4) may be written as

Now, Lim

(∋)r

∋ Ε(

FG H

Lim F !, ∋; ∗ ; x ∋ ∋ Ε(

IJ = Lim ) (!) (∋) FG x IJ K r ! (∗ ) H ∋ K (

r

∋ Ε(

r&0

r

r

r

(

= Lim ) (!)r (∋)rr x r = ∋ Ε(

r ! (∗ )r ∋ r&0

(

(! )r

) r ! (∗ )

r&0

r x & F (!; ∗ ; x ) . ...(5) r

The function F(a; ∗; x) is called the confluent hypergeometric function. Solution of differential equation (3) may also be obtained directly by the series integration method. Considering the equation (3), we find that x = 0 is a removable (non–essential) singularity and so the series representing the solution can be developed about the point x = 0. Solution of the confluent hypergeometric differential equation when x = 0 and ∗ is not an integer. [Kanpur 2010] Let us consider the solution of the hypergeometric differential equation (3) in the ascending powers of x as

Created with Print2PDF. To remove this line, buy a license at: http://www.software602.com/

Hypergeometric Function

14.7 (

y = xk(a0 + a1x + a2x2 + ...) = ) a0 x k ∃ r , a0 . 0.

...(6)

r&0

(

y− = ) ar (k ∃ r )x k ∃ r # 1

0

(

y∆ = ) ar (k ∃ r )(k ∃ r # 1)x k ∃ r # 2 .

and

r&0

r&0

Now putting the values of y, y− and y∆ in (3) we get (

(

(

r&0

r&0

r&0

x ) ar ( k ∃ r )( k ∃ r # 1) x k ∃ r # 2 ∃ ( ∗ # x) ) ar ( k ∃ r ) x k ∃ r # 1 # ! ) ar x k ∃ r = 0 (

(

) ar [(k ∃ r )(k ∃ r # 1) ∃ ∗ (k ∃ r )] x k ∃ r # 1 # ) [ar (k ∃ r ) ∃ !] x k ∃ r = 0. ...(7) r&0 r &0

or

which is an identity and so coefficients of various powers of x must be zero. Equating the coefficients of xk – 1 (lowest powers of x) to zero, we get a0[k(k – 1) + ∗k] = 0 or k(k – 1) + ∗k = 0 Hence k = 0 and k = 1 – ∗ are the roots of the indical equation. Now equating to zero the coefficients of xk + i, in (7) we get [(k + i + 1)(k + i) + ∗(k + i + 1)]ai + 1 – [(k + i + !)]ai = 0. 0

ai + 1 =

(k ∃ i ) ∃ ! ai . (k ∃ i ∃ 1) (k ∃ i ∃ ∗ )

Case I. When k = 0. Then (8) gives Putting i = 0, 1, 2, 3, ... in (9), we get

as

ai + 1 =

...(8)

(i ∃ !) a. (i ∃ 1)(i ∃ ∗ ) i

a1 = ! a0 ,

a2 =



a0 . 0.

...(9)

/∃ ! !1! ∃ /) a & a Χ1/ ∃ ∗2 1 /ΦΧΦ∗1∗ ∃ /2 0

............................................................................................................ am =

(! ) m !1! ∃ /)(a ∃ 2)...(a ∃ m # 1) a0 & a0 . / Χ ΓΦΦΦm ∗1∗ ∃ /2ΦΦΦ1∗ ∃ m # 1) m ! ( ∗ )m

Substituting the values of a1.a2...am in the series (6), we get ( 4 ! 5 (!)m m !(! ∃ 1) x 2 ∃ ...7 & a0 ) x . y = a0 61 ∃ x ∃ ∗ 1 2 ∗ ( ∗ ∃ 1) m & 0 m ! (∗ )m 8 9

Taking a0 = 1, we have y = F(!; ∗; x) which is called the confluent hypergeometric function of the first kind. Case II. When k = 1 – ∗. Then (8) gives ai + 1 = 0

ai + 1 =

!− ∃ 1 a (∗ − ∃ i)(i ∃ 1) i

where

1# ∗ ∃ i ∃! (! # ∗ ∃ 1) ∃ i a & a (i ∃ 2 # ∗ )(i ∃ 1) i (2 # ∗ ∃ i)(i ∃ 1) i

! – ∗ + 1 = !−

2 – ∗ = ∗−.

and

Putting i = 0, 1, 2, 3, ... in the above relation, we have a1 =

!− !− ∃ / ! −1! − ∃ /2 a0 , a2 & a1 & a0 ∗− 1∗ − ∃ /2 Χ / Χ ∗ −1∗ − ∃ /2

Similarly,

am =

1! −2 m a . m! (∗ −2m 0

Substituting the values of a1, a2, ... in (6), we have

Created with Print2PDF. To remove this line, buy a license at: http://www.software602.com/

14.8

Hypergeometric Function ( (! −) 4 !− 5 ! −1! − ∃ /2 2 1# ∗ m x ∃ ...7 = a0 x y = a0 x1 # ∗ 61 ∃ x ∃ ) m ! (∗ −m) x ∗ 1 2 ∗ 1∗ ∃ /2 − − − m & 0 m 8 9

y = a0x1 – ∗ F(!−; ∗−; x) = a0x1 – ∗F(! – ∗ + 1; 2 – ∗; x). Putting a0 = 1, we have y = x1 – ∗ F(! – ∗ + 1; 2 – ∗, x) which is called the confluent hypergeometric function of the second kind. Thus the general solution of the confluent hypergeometric differential equation is given by y = AF(!; ∗; x) + Bx1 – ∗ F(! – ∗ + 1; 2 – ∗; x), where ∗ > 0. 14.14. Differentiation of hypergeometric confluent functions. or

Show that

d F(!; ∋; x ) = ! F(! ∃ / ; ∋ ∃ / ; x) and deduce that dx ∋

n (!)n (i) d n F(! ; ∋ ; x) = F(! ∃ n ; ∋ ∃ n ; x ) (∋)n dx

(ii)

F(!; ∋; x) =

Proof. By definition, we have

LM d N dx

n n

OP Q

F(! ; ∋ ; x )

= x&0

(!)n . (∋)n

(!)

(

r ) (∋) r x . r&0 r

Now the proof is similar to that of Art. 14.8. 14.15. Integral representation for confluent hypergeometric function.

or

F(!; ∋; x) =

%(∋) %(!2 %(∋ # !)

F(!; ∋; x) =

1 B (! , ∋ # ! )

1

3 (1 # t )

∋ # ! #1 ! #1

t

0

1

3 (1 # t )

∋ # ! #1 ! # 1

0

e xt dt

[Purvanchal 2006]

e xt dt , where ∋ > ! > 0.

t

Proof. By definition, we have ( (!)n x n %(! ∃ n) %(∋) x n , by Art. 14.1 & ) %(∋ ∃ n) n ! n & 0 (∋)n n ! n & 0 %(!) (

F(!; ∋; x) = ) =

( ( % (∋ # ! ) % (! ∃ n ) x n %(∋) % (∋ ) = ) ) % (! )% (∋ # ! ) n & 0 % (∋ # ! ∃ ! ∃ n ) n ! %(!) %(∋ # !) n & 0

< >≅

=

or

1 0

∋ # ! #1 ! ∃ n #1

t

, xn! , if ∋ > ! n

dt

= % ( p) % (q) 1 & B ( p, q) & 30 (1 # t ) p # 1 t q # 1 dt , if p Η 0, q Η 0? % ( p ∃ q) Α

( ( xt ) n = % (∋) 1 ∋ # ! #1 ! #1 < (1 # t ) t ) 3 > ? dt % (!) % (∋ # !) 0 ≅ n & 0 n! Α

0

+3 (1 # t)

F (!; ∋; x ) =

%(∋) %(!)%(∋ # !)

F (!; ∋; x ) =

1 B(!Ι ∋ # !)

z

z

1

0

1

0

(1 # t)

∋ # ! # 1 ! # 1 xt

t

e dt

∋ # ! # 1 ! # 1 xt

(1 # t )

t

e dt

...(1) ...(2)

< = %(!) % (∋ # !) 1 % (∋) >≅ B(!, ∋ # !) & % (! ∃ ∋ # !) ; B (!, ∋ # !) & % (!) % (∋ # !) ?Α

Thus (1) and (2) are the required results.

Created with Print2PDF. To remove this line, buy a license at: http://www.software602.com/

Hypergeometric Function

14.9 x

14.16. Theorem (Kummer’s Relation). Show that F(!; ∋; x) = e F(∋ – !; ∋; –x). Proof. From Art. 14.15, we know that F(! ; ∋ ; x) =

%(∋) 1 ∋ # ! # 1 ! # 1 xt t e dt . 3 (1 # t ) % (!) %(∋ # !) 0

...(1)

Replacing ! by ∋ – ! and x by –x in (1), we get F(∋ – !; ∋; –x) =

=

%(∋) 1 (1 # t )∋ # (∋ # !2 # 1 t ∋ # ! # 1 e # xt dt 3 0 % (∋ # !) %[∋ # (∋ # !)]

% (∋) % (∋) 1 0 ! #1 ! # 1 ∋ # ! # 1 # xt ∋ # ! # 1 # x (1 # u ) e dt = e (#du ) 3 (1 # t ) t 3 u (1 # u ) % (∋ # !) % (!) 0 % (∋ # !) % (!) 1

(putting 1 – t = u so that dt = – du and t = 1 – u) =

% (∋) % (∋)e # x 1 1 #x (1 # t )∋ # ! # 1 t ! # 1 e xt dt (1 # u )∋ # ! # 1 u ! # 1 e xu du = e 3 3 0 0 % ( ! ) % ( ∋ # ! ) % (∋ # !) % (!)

= e–xF(!; ∋; x), by (1) 0 F(∋ – !; ∋; –x) = e–x F(!; ∋; x) so that F(!; ∋; x) = ex(∋ – !; ∋; –x). 14.17. Contiguous hypergeometric functions. Definitions. According to Gauss, the function F(!−, ∋− ; ∗− ; x) is said to be contiguous to F(!, ∋ ; ∗ ; x) when it is increased or decreased by one and only one of the parameters !, ∋, ∗ by unity. According to above definition, there exist six hypergeometric functions contiguous to F(!, ∋ ; ∗ ; x). These are denoted and defined as given below : F!+ = F(! + 1, ∋ ; ∗ ; x), F∋+ = F(!, ∋ + 1 ; ∗ ; x), F∗+ = F(!, ∋ ; ∗ + 1 ; x) F!– = F(! – 1, ∋ ; ∗ ; x), F∋– = F(!, ∋ – 1 ; ∗ ; x), F∗– = F(!, ∋ ; ∗ –1 ; x). 14.18. To prove the contiguity relationship (! – ∋)F(!, ∋ ; ∗ ; x) = !F(! + 1, ∋ ; ∗ ; x) – ∋F(!, ∋ + 1 ; ∗ ; x) or (! – ∋)F(!, ∋, ∗, x) = !F!+ – ∋F∋+. Proof. We have, by definition 14.4, !F(! + 1, ∋ ; ∗ ; x) – ∋F(!, ∋ + 1 ; ∗ ; x) ( ! (! ∃ 1) (∋) ( (!) ∋(∋ ∃ /) ( (! ) (∋ ∃ /) (! ∃ 1) r (∋)r r r r r r r r r r x # ) x #∋ ) x = ) (∗ )r r ! ( ∗ )r r ! ( ∗ )r r ! (∗ )r r ! r&0 r&0 r&0 r &0 (

= !)

( (! ) 1∋2 (∋ ∃ r ) (! ∃ r ) (!)r (∋)r r r r r r x # ) x (∗ )r r ! ( ∗ )r r ! r&0 r&0 (

= )

[ !(! + 1)r = !(! + 1)(! + 2)...(! + 1 – r)(! + 1 + r – 1), by Art. 14.1 = [!(! + 1)(! + 2)...(! + r – 1)](! + r) = (! + r)(!)r, by Art. 14.1 again Similarly, ∋(∋ + 1)r = (∋ + r)(∋)r] (

=

)

[(! ∃ r) # (∋ ∃ r)]

r&0

(!)r 1∋2r r x = ( ! # ∋) (∗ )r r !

(

(! ) r 1∋2 r r x & (! # ∋) F ( ! , ∋ ; ∗ ; x ) ( ∗ )r r ! r&0

)

14.19. Contiguity relationship for confluent hypergeometric functions (! – ∋)xF(! ; ∋ + 1 ; x) + ∋(! + ∋ – 1)F(! ; ∋ ; x) – ∋(∋ – 1)F(! ; ∋ – 1 ; x) = 0. Proof. Proceed as explained in Art. 14.18.

Created with Print2PDF. To remove this line, buy a license at: http://www.software602.com/

14.10

Hypergeometric Function

14.20. A SOLVED EXAMPLES Ex. 1. Prove that (i) ex = 1F1(! ; ! ; x). (ii) (1 – x)–! = 2F1(!, ∋ ; ∋ ; x). (iii) (1 – x)–1 = F(1, 1 ; 1 ; x), | x | < 1. (iv) (1 + x)n = F(–n, 1 ; 1 ; –x). (v) ln(1 + x) = loge (1 + x) = x 2F1(1, 1 ; 2 ; –x). (vi) log (1 – x) = –x 2F1(1, 1 ; 2 ; x). (vii) log

1

1∃ x = 2 x F 1/ 2, 1 ; 3 / 2 ; x 2 1# x

2=

[Kanpur 2008, 09, Lucknow 2010] [Kanpur 2004]

41/ 2,1; 5 2 ; x7 . &F6 8 3/ 2 9

[Kanpur 2005, 10]

(viii) sin–1 x = xF(1/2; 1/2; 3/2; x2). (ix) tan–1 x = x F(1/2; 1; 3/2; –x2). Sol. (i) We have, by definition 14.3

[Kanpur 2006]

!(! ∃ 1) x 2 ; ∋ ; x) = 1 ∃ ! . x ∃ . ∃ ... ad.inf. ∋ 1! ∋(∋ ∃ 1) 2 ! Replacing ∋ by ! in (1), we have 1F1(!

2 x x F (! ; ! ; x) = 1 ∃ ∃ ∃ ... 1 1 1! 2 ! (ii) We have, by definition 14.4

or

2F1(!, ∋ ; ∗ ; x) = 1 ∃

1F1(!

...(1)

; ! ; x) = ex.

! ∋ x !(! ∃ 1) ∋(∋ ∃ 1) x 2 . ∃ ∃ ... ∋ 1! ∗ ( ∗ ∃ 1) 2!

...(1)

Replacing ∗ by ∋ in (1), we have 2F1(!,

∋ ; ∋ ; x) = 1 ∃

2 2 !∋ x !(! ∃ 1)∋(∋ ∃ 1) x 2 . ∃ ∃ ... = 1 ∃ !. x ∃ !(! ∃ 1) x ∃ !(! ∃ 1)(! ∃ 2) x ∃ ... ∋ 1! ∋(∋ ∃ 1) 2! 1! 2! 3!

( #! )( #! # 1) ( #!21#! # /21#! # Χ2 ( # x) 2 ∃ ( # x ) 3 ∃ ... 2! 3! = (1 – x)–!, by the binomial theorem. (iii) and (iv). Procced like part (ii) above.

= 1 ∃ ( #!21# x 2 ∃

(v) We have,

2F1(!, ∋ ; ∗ ; x) = 1 ∃

! ∋ x !(! ∃ 1) ∋(∋ ∃ 1) x 2 ∃ . ∃ ... ∗ 1! ∗ ( ∗ ∃ 1) 2!

...(1)

Replacing !, ∋, ∗, x by 1, 1, 2 and –x respectively in (1), we get 1 1 ( # x ) 1 2 1 2 ( # x ) 2 1 2 3 1 2 3 ( # x )3 ∃ ∃ ∃ ... 2 1! 2 3 2! 2 3 4 3! Multiplying both sides of the above equation by x, we get 2F1(1,

1 ; 2 ; 1 – x) = 1 ∃

2

3

4

x 2F1(1, 1 ; 2 ; – x) = x # x ∃ x # x ∃ ... ad. inf. = log (1 + x). 2 3 4 (vi) and (vii). Proceed like part (v) above (viii) We have,

F(!, ∋ ; ∗ ; x) = 1 ∃

! ∋ x !(! ∃ 1) ∋(∋ ∃ 1) x 2 ∃ ∃ ... ∗ 1! ∗ ( ∗ ∃ 1) 2!

...(1)

Replacing !, ∋, ∗, x by 1/2, 1/2, 3/2 and x2 respectively in (1), we get

Created with Print2PDF. To remove this line, buy a license at: http://www.software602.com/

Hypergeometric Function

14.11

FH

IK

(1/ 2) : (1/ 2) x 2 (1/ 2) : (3/ 2) : (1/ 2) : (3/ 2) x 4 ∃ ∃ ... F 1 , 1 ; 3 ; x2 = 1 ∃ 2 2 2 (3/ 2) 1! (3/ 2) : (5 / 2) 2! 0

FH

IK

x3 x5 x7 ∃ 12 32 ∃ 12 33 52 ∃ ... = sin–1 x. xF 1 , 1 ; 3 ; x = x ∃ 12 2 2 2 3! 5! 7!

! ∋ x !(! ∃ 1) ∋(∋ ∃ 1) x 2 ∃ ∃ ... ∗ 1! ∗ ( ∗ ∃ 1) 2!

F(!, ∋ ; ∗ ; x) = 1 ∃

(ix) We have,

...(1)

Replacing !, ∋, ∗ and x by 1/2, 1, 3/2 and –x2 respectively in (1), we have

FH

IK

2 2 2 F 1 , 1 ; 3 ; # x 2 = 1 ∃ (1/ 2) : 1 (# x ) ∃ (1/ 2) : (3 / 2) : 1 : 2 (# x ) ∃ ... 2 2 (3 / 2) 1! (3 / 2) : (5 / 2) 2!

FH

IK

FH

so that

IK

3 5 2 4 F 1 , 1 ; 3 ; # x 2 = 1 # x ∃ x # ... ( ; xF 1 , 1 ; 3 ; # x 2 = x # x ∃ x ∃ ... ( = tan–1 x. 3 5 2 2 3 5 2 2

Ex. 2. Show that lim 2 F1 (1, a; 1; x / a ) & e x . a Ε(

2

1 a < x = 1 2 a ( a ∃ 1) < x = 2 F1 (1, a; 1; x / a ) & 1 ∃ > ?∃ > ? ∃ .... 1 1≅aΑ 1 2 1 2 ≅aΑ

Sol. By definition,

or

2 F1 (1,

a; 1; x / a ) & 1 ∃

x < 1 = x 2 < 1 =< 2 = x 3 ∃ >1 ∃ ? ∃ > 1 ∃ ?> 1 ∃ ? ∃ ... 1! ≅ a Α 2! ≅ a Α≅ a Α 3!

lim 2 F1 (1, a; 1; x / a ) & 1 ∃ x /1!∃ x 2 / 2!∃ x 3 / 3!∃ ... & e x

0

a Ε(

Ex. 3. Show than

2 F1 ( a , 1;

Sol. F (a, 1; a; x) & 1 ∃

a; x ) & (1 # x) #1 .

a 1 a (a ∃ 1) 1 2 2 a (a ∃ 1) (a ∃ 2) 1 2 3 3 x∃ x ∃ x ∃ ... 1 a 1 2 a (a ∃ 1) 1 2 3 a (a ∃ 1) (a ∃ 2)

= 1 + x + x2 + x3 + .... = (1 – x)–1 ab 4d 5 Ex. 4. Show that 6 2 F1 (a, b; c; x ) 7 & . c 8 dx 9 x &c

Sol. By definition,

0

2 F1 ( a , b; c;

x) & 1 ∃

ab a (a ∃ 1) b (b ∃ 1) 2 x∃ x ∃ ... 1 c 1 2 c ( c ∃ 1)

d ab a (a ∃ 1) b (b ∃ 1) d ab 1∃ 2 x ∃ ... ; 4 2 F1 (a, b; c; x ) 5 & 2 F1 ( a, b; c; x) & 0 ∃ 6 dx 7 dx c 2 c (c ∃ 1) 8 9 x&0 c n Ex. 5. Show that 1 – n x 2F1(1 – n, 1; 2; x) = (1 – x) , where n is any natural number.

Sol. Using formula F (!, ∋; ∗ ; x ) & 1 ∃

!∋ !(! ∃ 1) ∋(∋ ∃ 1) 2 x∃ x ∃ ..., we get 1 ∗ 1 2 ∗ ( y ∃ 1)

1 – n x 2F1 (1 – n, 1; 2; x)

Created with Print2PDF. To remove this line, buy a license at: http://www.software602.com/

14.12

Hypergeometric Function

ϑ (1 # n ) .1 Κ (1 # n) (2 # n) 1 2 2 (1 # n) (2 # n) ...( n # 1 # n ) 1 2 ...( n # 1) n #1 & 1 # nx Λ1 ∃ x∃ x ∃ x ∃ 0Μ 1 2 1 2 2 3 1 2 .... ( n # 1) 2 3 .... n Ν Ο

(n # 1) (n # 2) 2 (#1)n #1 (n # 1) (n # 2) Πϑ n # 1 & 1 # nx Λ1 ∃ x∃ x ∃ ... ∃ 2! 3! n! ΠΝ

& 1 # nx ∃

21

ΠΚ x n #1 Μ ΟΠ

n(n # 1) 2 n(n # 1)(n # 2) 3 (#1) n n! n x # x ∃ ... ∃ x 2! 3! n!

= 1 + nc1 (–x) + nc2 (–x)2 + nc3 (–x)3 + ... + ncn (–x)n = (1 – x)n Ex. 6. Show that lim

a , b Ε(

2 F1 ( a , b; 1/ 2;

x2 / 4ab) & cosh x

Sol. Using formula 2 F1 (!, ∋; ∗; x) ∃ 1 ∃

!∋ ! (! ∃ 1) ∋ (∋ ∃ 1) 2 x∃ x ∃ ...., we get 1 ∗ 1 2 ∗ ( ∗ ∃ 1) 2

2 2 F1 ( a , b;1/ 2; x / 4ab ) & 1 ∃

< x2 = ab x2 a(a ∃ 1) b(b ∃ 1) : ∃ : >> ? ∃ ... 1: (1/ 2) 4ab 1: 2 : (1/ 2) : (3/ 2) ≅ 4ab ?Α

& 1 ∃ x2 / 2 ∃ (1 ∃ 1/ a) (1 ∃ 1/ b) : ( x 4 / 24) ∃ .... 0

lim

a , b Ε(

Ex. 7. Show that

2 F1 ( a , b;1/ 2;

2 F1 ( a # 1,

x2 / 4ab) & 1 ∃ x 2 / 2! ∃ x 4 / 4! ∃ ... & cosh x

b # 1, c; x) # F (a, b # 1); c; x ) & ( x / c ) : (1 # b) 2 F1 (a, b; c ∃ 1; x)

Sol. Using formula 2 F1 (!, ∋; ∗; x ) & 1 ∃

!∋ !(! ∃ 1) ∋(∋ ∃ 1) 2 x∃ x ∃ ..., we get 1 ∗ 1 2 ∗ ( ∗ ∃ 1)

F (a # 1, b # 1; c; x ) & 1 ∃ (a # 1) (b # 1) : ( x / c ) ∃

and

(a # 1) a (b # 1) b 2 (a # 1) a (a ∃ 1) (b # 1) b (b ∃ 1) 3 x ∃ x ∃ .... 1 2 c (c ∃ 1) 1 2 3 c (c ∃ 1) (c ∃ 2)

... (1)

F(a, b – 1; c; x) = 1 + a(b – 1) × (x/c) ∃

a(a ∃ 1) (b # 1) b 2 a (a ∃ 1) (a ∃ 2) (b # 1) b (b ∃ 1) 3 x ∃ x ∃ ... 1 2 c (c ∃ 1) 1 2 3 c (c ∃ 1) (c ∃ 2)

... (2)

Subtracting (2) from (1), F(a – 1, b – 1; c; x) – F(a, b – 1; c; x) &

(a # 1 # a) (b # 1) (a # 1 # a # 1) a(b # 1)b 2 (a # 1 # a # 2) a (a ∃ 1) (b # 1) b(b ∃ 1) 3 x∃ x ∃ x ∃ ... c 1 2 c (c ∃ 1) 1 2 3 c (c ∃ 1) (c ∃ 2)

&

Κ 1# b ϑ 2ab 3a(a ∃ 1) b(b ∃ 1) 2 x Λ1 ∃ x∃ x ∃ ...Μ = (x/c) × (1 – b) F (a, b; c + 1; x) 2 1 c Ν 1 2 (c ∃ 1) 1 2 3 (c ∃ 1) (c ∃ 2) Ο Ex. 8. Find the third derivative of 2F1 (2, 3; 1; x) w.r.t. ‘x’ (b) Find the fourth derivative of the following hypergeonetre functions w.r.t ‘x’: (i) 2F1 (2, 1; 4; x) (ii) 2F1 (2, –2; 5; x).

Created with Print2PDF. To remove this line, buy a license at: http://www.software602.com/

Hypergeometric Function

14.13

dn

Sol. (a) By Art. 14.8,

0

dx d3 dx

F (!, n 2 1

F (2, 3; 1; 3 2 1

∋; ∗; x) &

x) &

(!)n (∋)n F (! ∃ n, ∋ ∃ n; ∗ ∃ n; x) (∗ ) n

(2)3 (3)3 F (2 ∃ 3, 3 ∃ 3; 1 ∃ 3; x) (1)3

But

(! )n & ! (! ∃ 1) (! ∃ 2)...(! ∃ n # 1), by Art. 14.1

0 (1)3 = 1 2 3,

(2)3 = 2 × (2 + 1) × (2 + 2) = 24;

... (1)

... (2)

(3)3 = 3 × (3 + 1) × (3 + 2) = 60

d3

24 : 60 2 F1 (2, 3; 1; x) & 2 F1 (5, 6; 4; x) & 240 2 F1 (5, 6; 4; x ) 6 dx3 (b) Proceed as in part (a) Ans. (i) (24/7) × F(6, 5; 8; x) (ii) 0 Ex. 9. Find the solutions of the following equations :

Hence, (2) yields

(i) x(1 # x ) y −− ∃ (3/ 2 # 2 x) y − ∃ 2 y & 0 about x = 0 (ii) ( x # x2 ) y−− ∃ (3 / 2 # 2 x) y− # ( y / 4) & 0 about x = 0

(KU Kurukshetra 2004) (KU Kurukshetra 2004)

(iii) 8 x(1 # x ) y −− ∃ (4 # 14 x) y − # y & 0 about x = 0 (iv) 4 x (1 # x) y −− ∃ y − ∃ 8 y & 0 about x = 0 Sol. In what follows, we shall use the following results: x(1 # x ) y −− ∃ {∗ # (! ∃ ∋ ∃ 1) x} y − # !∋y & 0

I. Hypergeometric equation is given by II. General solution of hypergeometric equation is

y & a 2 F1 (!, ∋; ∗; x) ∃ b x ∗#1 2 F1 (1 # ∗ ∃ !, 1 # ∗ ∃ ∋; 2 # ∗; x), where a and b are arbitrary constants III. 2 F1 (!, ∋; ∗; x ) & 1 ∃

! ∋ ! (! ∃ 1) ∋(∋ ∃ 1) 2 x∃ x ∃ .... 1 ∗ 1 2 ∗ ( ∗ ∃ 1)

Part (i) Re-writing the given equation, we have, x(1 # x ) y −− ∃ (3 / 2 # 2 x) y − ∃ 2 y & 0

... (1)

Comparing (1) with x(1 # x ) y −− ∃ {∗ # (! ∃ ∋ ∃ 1) x} y − # !∋y & 0, we have ∗ & 3/ 2, ! ∃ ∋ ∃ 1 & 2 and

!∋ & #2 . Solving these, ! & 2, ∋ & #1, ∗ & 3/ 2. Here ∗ is not an integer. The general solution of (1) is y & au ∃ bv, where u & 2 F1 (!, ∋; ∗; x ) & F (2, # 1; 3 / 2; x) & 1 ∃

2 : (#1) 2 : 3 : (#1) : 0 4x x∃ x 2 ∃ ... & 1 # 1: (3/ 2) 1: 2 : (3/ 2) : (5 / 2) 3

and v & x1#∗ 2 F1 (! ∃ 1 # ∗ , ∋ ∃ 1 # ∗; 2 # ∗; x)

& x1#(3/2)2 F1 (2 ∃ 1 # 3/ 2, # 1 ∃ 1 # 3 / 2; 2 # 3/ 2; x) & x #1/ 22 F1 (3 / 2, # 3 / 2;1/ 2; x) Hence the general solution of (1) is given by y = a(1 – 4x/3) + b x–1/2 2F1(3/2, – 3/2; 1/2; x) (ii) Given

x(1 # x ) y −− ∃ (3 / 2 # 2 x ) y − # (1/ 4) : y & 0

... (1)

Comparing (1) with x(1 # x ) y −− ∃ {∗ # (! ∃ ∋ ∃ 1) x} y − # !∋y & 0, we have

Created with Print2PDF. To remove this line, buy a license at: http://www.software602.com/

14.14

Hypergeometric Function

! & 1/ 2, ∋ & 1/ 2, ∗ & 3/ 2. Hence ∗ is not an integer. The general solution of (1) is y & au & bv, u & 2 F1 (!, ∋; ∗; x ) & 2 F1 (1/ 2, 1/ 2; 3 / 2; x )

where

and

v & x1#∗ 2 F1 (! ∃ 1 # ∗ , ∋ ∃ 1 # ∗; 2 # ∗; x) & x1# 3 / 2 2 F1 (1 / 2 ∃ 1 # 3 / 2, 1 / 2 ∃ 1 # 3 / 2; 2 # 3 / 2; x )
1 ∃ ≅

= 0:0 1 x ∃ ... ? & 1 : (1 / 2) x Α

General solution of (1) is y & a 2 F1 (1/ 2,1/ 2; 3 / 2; x) ∃ b / x , a, b being arbitrary constants (iii) Ans. y = a (1 – x)–1/4 + bx1/2 2F1(1, 3/4; 3/2; x) (iv) Ans. y = a (1 – 8x + 32x2/5) + bx3/4 2F1(7/4 – 5/4; 7/4; x) Ex. 10. Solve the Legendre equation (1 – x2) (d2y/dx2) – 2x (dy/dx) + n(n + 1)y = 0 by changing it to a hypergeometre equation. (MDU Rohtak 2004) Sol. Given (1 – x2) (d2y/dx2) – 2x (dy/dx) + n(n + 1)y = 0 ... (1) Let z = x2 ... (2) dy dy dy dz & 2x & dx dz dx dz

Now,

and

d2y dx 2

&

... (3)

2 d < dy = d < dy = dy d < dy = & 2 dy ∃ 2 x d < dy = dz & 2 dy ∃ (2 x)2 d y & 2 x & 2 ∃ 2 x > ? > ? > ? > ? dz dz ≅ dz Α dx dz dz 2 dx ≅ dx Α dx ≅ dz Α dz dx ≅ dz Α

Substituing the above values in (1),

< dy d2y = dy (1 # x 2 ) > 2 ∃ 4 x 2 2 ? # (2 x) 2 ∃ n(n ∃ 1) y & 0 > dz ? dz dz Α ≅

or

< dy d2z = dy (1 # z ) > 2 ∃ 4 z 2 ? # 4 z ∃ n(n ∃ 1) y & 0 > dz ? dz dz Α ≅

or

4 z (1 # z )(d 2 y / dz 2 ) ∃ (2 # 6 z )(dy / dz ) ∃ n(n ∃ 1) y & 0 z (1 # z )

or

d2y

< 1 3 = dy n ( n ∃ 1) ∃ > # z? ∃ y&0 ≅ 2 2 Α dz 4 dz

... (4)

2

Refer Ex. 9 for hypergeometric equation and its solution. Comparing (4) with hypergeometre

z (1 # z ) (d 2 y / dz 2 ) ∃ {∗ # (! ∃ ∋ ∃ 1) z} (dy / dz ) # !∋ y & 0,

equation

∗ & 1/ 2,

we have,

Solving these,

! ∃ ∋ ∃ 1 & 3/ 2 ! & (n ∃ 1) / 2,

∋ & #(n / 2)

... (5)

!∋ & #(1/ 4) : n(n ∃ 1)

and and

∗ & 1/ 2

Note that here ∗ is not an integer. The general solution of (5) is given by

y & a 2 F1 (!, ∋; ∗ ; z ) ∃ b z ∗ #12 F1 (1 # ∗ ∃ !, 1 # ∗ ∃ ∋; 2 # ∗ ; z ) Hence the required solution of (1) is given by

1 n 1 1 = < n ∃1 n 1 = < n ∃1 y & a 2 F1 > , # ; ; z ? ∃ b z1/ 2 2 F1 > ∃1# , # ∃1 # ; 2 # ; z ? 2 2 Α 2 2 2 2 Α ≅ 2 ≅ 2 n ∃1 n 1 = < n ∃ 2 n #1 3 2 = , # ; ; x 2 ? ∃ bx 2 F1 > ,# ; ; x ? , where a and b are arbitrary constants. 2 2 2 2 2 ≅ Α ≅ 2 Α

< or y & a 2 F1 >

Created with Print2PDF. To remove this line, buy a license at: http://www.software602.com/

Hypergeometric Function

14.15

2 Θ

Ex. 11. Show that (i)

(ii)

2 Θ

3

Θ/2

0

3

1

2

2

1/ 2

(1 # x sin Ρ)

0

& 2 F1 (1/ 2, 1/ 2;1; x 2 ), | x | Σ 1

(1 # x 2 sin Ρ)1/ 2 d Ρ & 2 F1 ( #1/ 2, 1/ 2; 1; x 2 ), | x | Σ 1 2 F1 (!, ∋;

Sol. By Art. 14.9,

3t

so that



Θ/2

∋#1

0

%( ∗ ) %(∋) %( ∗ # ∋)

∗; x ) &

1

3t

∋#1

0

(1 # t ) ∗#∋#1 (1 # xt )#! dt

% (∋) % (∗ # ∋2 : 2 F1 (!, ∋; ∗ ; x ) %(∗ )

(1 # t )∗ #∋#1 (1 # xt )#! dt &

... (A)

Part (i) Putting t & sin 2 Ρ and dt & 2sin Ρ cos Ρ d Ρ, we have 2 Θ

3



Θ/2 2

2

1/ 2

(1 # x sin Ρ)

0

&

2 Θ

3

1

1

0

2 1/ 2

(1 # x t ) 1

3

1

1

dt

(1 # x 2 t )1/ 2 2 t1/ 2 (1 # t )1/ 2

0

1 1 1# #1 # 2 (1 # x 2 t ) 2 dt

#1

&

1 Θ

&

1 %(1/ 2)%(1 # 1/ 2) 2 2 : 2 F1 (1/ 2; 1/ 2; 1; x ), using result (A) taking ∋ & 1/ 2, ∗ & 1, ! & 1/ 2 , x = x Θ %(1)

3

1

0

t #1/ 2 (1 # t )#1/ 2 (1 # x 2t ) #1/ 2 dt &

&

1 Θ

dt 2 & 2 sin Ρ cos Ρ Θ

3

1

0

t 2 (1 # t )

1 Θ: Θ : : 2 F1 (1/ 2,1/ 2, 1; x 2 ) & 2 F1 (1/ 2, 1/ 2; 1; x 2 ) Θ 1

(ii) Putting t & sin 2 Ρ and dt & 2sin Ρ cos Ρ d Ρ, we have 2 Θ

3

Θ/2

0

(1 # x 2 sin 2 Ρ)1/ 2 d Ρ &

2 Θ

1

3 (1 # x t )

2 1/ 2

0

1

1

3 (1 # x t )

&

1 %(1/ 2)%(1 # 1/ 2) 2 2 : 2 F1 ( #1/ 2, 1/ 2; 1; x ) & 2 F1 ( #1/ 2, 1/ 2; 1; x ) Θ %(1)

3

0

3

1

0

t

1/ 2

(1 # t )1/ 2

1 1# #1 2 (1 # x 2 t )1/ 2 dt

#1

1 Θ

t #1/ 2 (1 # t )#1/ 2 (1 # x 2t )1/ 2 dt &

dt

2 1/ 2

0

&

1

1 Θ

dt 1 & 2sin Ρ cos Ρ Θ

t 2 (1 # t )

[using result (A), taking ∋ & 1/ 2, ∗ & 1, ! & #1/ 2, x & x 2 ] Ex. 12. Show that if | x | < 1 and | x/(1 – x) | < 1, then 2F1(!,

FG H

LM N

F(!, ∋ ; ∗ ; x) = (1 # x) #! F !, ∗ # ∋ ; ∗ ;

or

IJ K

∋ ; ∗ ; x) = (1 – x)–! 2 F1 !, ∗ # ∋ ; ∗ ; x x# 1

OP Q

#x . 1# x

Sol. By integral representation for the hypergeometric function (refer Art. 14.9), we have 2F1(!,

∋ ; ∗ ; x) =

%(∗ ) %(∋)%(∗ # ∋)

z

1

0

t∋ # 1(1 # t)∗ # ∋ # 1(1 # xt)#! dt.

...(1)

Created with Print2PDF. To remove this line, buy a license at: http://www.software602.com/

14.16

Hypergeometric Function

Putting u = 1 – t 2F1(!,

∋ ; ∗ ; x) =

so that

z

% (∗ ) % (∋) % ( ∗ # ∋)

=

%(∗ ) %(∋)%(∗ # ∋)

=

(1 # x ) %(∗ ) %(∋)%(∗ # ∋)

#!

dt = – du and t = 1 – u, (1) gives 0

1

z z

1

0

u

1

0

(1 # u) ∋ # 1 u ! # ∋ # 1 (1 # x ∃ xu) # ! ( #du)

∗ #∋ #1

u

∋ #1

(1 # u)

∗ #∋ #1

RS1 ∃ xu UV T 1# xW RS1 # x uUV du . T x #1 W

(1 # x )

∋ #1

(1 # u)

#!

#!

du

#!

...(2)

Replacing ∋ and x by ∗ – ∋ and x/(x – 1) in (1), we get
!, ∗



#∋ ; ∗ ;

z

x = % (∗ ) ? = x # 1 Α %(∗ # ∋)%[∗ # (∗ # ∋)]

%(∗ ) = %(∗ # ∋)%(∋)

Using (3), (2) reduces to

2F1(!,

z

1

0

u

1

0

t

∗ # ∋ #1

∗ #∋#1

(1 # t )

∋ #1

(1 # u)

RS1 # xt UV T x # 1W

RS1 # x uUV du . T x #1 W F I F G !, ∗ # ∋ ; ∗ ; x J x # 1K H

∋ ; ∗ ; x) = (1 # x)#!

FH

∗ # ( ∗ # ∋) # 1

#!

dt

#!

...(3)

2 1

IK

Ex. 13. Prove that Pn(x) = 2 F1 # n, n ∃ 1 ; / ; 1 # x 2 [Punjab 2005, Purvanchal 2005,06, Kanpur 2004, 06] Sol. Pn(x) =

n

1 d ( x 2 # 1)n , by Rodrigue’s formula n n n ! 2 dx

RS UV = 1#1) d LM(1 # x) . RS1 # 1 # x UV OP T 2 WQ T W n! dx N LM(1 # x) . R1 # nF 1 # x I ∃ n(n # 1) F 1 # x I # ...UOP , by the binomial theorem ST H 2 K 2! H 2 K VWPQ MN LM(1 # x) # n (1 # x) ∃ n(n # 1) (1 # x) ∃ ...OP 2 2! 2 N Q n

n

=

n n 1#1) d n (#1) d n 2 n n (1 ∃ x) n n (1 # x ) & n ! n (1 # x ) . n n ! 2 dx dx 2

=

1#1) d n n ! dx n

=

1#1) d n n ! dx n

=

5 1#1)n 4 n(n # 1) ( n ∃ 2) n n n ( n ∃ 1)! (1 # x) ∃ (#1)n : : (1 # x)2 ∃ ...7 6(#1) n ! # (#1) 2 n! 8 2 1! 2! 2! 2 9

n

n

n∃1

n∃2

2

LM N

2n

= (#1) n!

d n (a # bx )m & (#1)n b n . m ! (a # bx )m # n (m # n)! dx n

LMn! # n(n ∃ 1) n!(1 # x) ∃ n(n # 1) (n ∃ 2)(n ∃ 1)n!(1 # x) 2 2! 2 N

2

2

2

= 1∃

n

n

n

2

n

n

n

F H

OP Q

OP Q

∃ ...

I K

1# x (# n)( n ∃ 1) < 1 # x = ( #n)(# n ∃ 1)(n ∃ 1)(n ∃ 2) < 1 # x = >≅ ?∃ >≅ ? ∃ ... = 2 F1 # n, n ∃ 1 ; 1 ; 2 , by definition. 1 1! 2 Α 2 1 2! 2 Α

Created with Print2PDF. To remove this line, buy a license at: http://www.software602.com/

Hypergeometric Function

14.17

F H

I K

n #1 ; 1 ; # tan2 Ρ . Ex. 14. Show that Pn(cos Ρ) = cosn Ρ 2 F1 # n , # 2 2 Sol. From Laplace’s first integral for Pn(x), (Refer Art. 9.6 in chapter 9), we have

Pn(x) =

1 Θ

3

Θ

Τ

[ x Υ ( x 2 # 1)1/ 2 cos ς]n d ς.

...(1)

Let x = cos Ρ. Then, we have ( x2 # 1)1/ 2 & (cos 2 Ρ –1)1/ 2 & {(–1) : (1 – cos 2 Ρ)}1/ 2 = i sin Ρ. With these values and taking positive sign in (1), we get Pn(cos Ρ) =

1 Θ

3

Θ

Τ

(cos Ρ ∃ i sin Ρ cos ς)n d ς =

n

cos Ρ Θ

z

1

Τ

n

(1 ∃ i tan Ρ cos ς) dς

=

cos n Ρ Θ

3

=

cosn Ρ Θ

LM N



n(n # 1)(n # 2)(n # 3) 4 4 < i tan Ρ > 2 ≅ 321

=

n 3 1 Θ cos Ρ n(n # 1) 5 2 1 Θ n(n # 1)(n # 2)(n # 3) Θ# tan Ρ : 2 : : ∃ tan 2 Ρ : : : ∃ ...7 Θ 2 2 2 3 2 1 4 2 2 9

Θϑ

Λ1 ∃ i tan Ρ cos ς ∃ Ν

Τ

z

Θ

Τ

dς ∃ 0 ∃

n (n # 1) 2 Κ i tan 2 Ρ cos 2 ς ∃ ...Μd ς, by the binomial theorem 2! Ο

FG H

n(n # 1) 2 i tan 2 Ρ 2 2

3

Θ/ 2

0

z

Θ /2

0

IJ K

cos2 ς dς ∃ 0

ϑΠ0, if f (2a # x) & # f ( x) 5 2a = cos4 ς dς? ∃ ...7 , as 3 f ( x ) dx & Λ a Α 0 ΠΝ230 f ( x ) dx, if f (2a # x) & f ( x) 9

LM N

n = cos

LM FH # n IK F # n # 1I 2 H 2 K (# tan Ρ M1 # 1 1! N

2

FH

Ρ)

5 < n = < n = < n # 1= < n # 1 = ∃ 1? >≅ # ?Α >≅ # ∃ 1?Α >≅ # ? ># 7 Α (# tan 2 Ρ)2 2 2 2 Α≅ 2 ∃ ∃ ...7 12 2! 9

IK

n #1 n n 2 ; 1 ; tan Ρ , by definition. = cos Ρ 2 F1 # , # 2 2

EXERCISE 1. Show that 1F1(∋ ; ∗ ; x) = lim 2 F1(!, ∋ ; ∗ ; x / ∋) . ∋ Ε(

b g b g %b∗ / 2g%b∗ / 2 ∃ 1 / 2g F (!, 1 – ! ; ∗ ; 1/2) = . %b! / 2 ∃ ∗ / 2g%b1 / 2 # ! / 2 ∃ ∗ / 2g

2. Show that 2F1(!, ∋ ; ∋ – ! + 1 ; –1) =

%(1 ∃ ∋ # !)% 1 ∃ ∋ / 2 and deduce that %(1 ∃ ∋)% 1 ∃ ∋ / 2 # !

2 1

z

(

3. Evaluate the integral e #sx1F1(! ; ∋ ; x ) dx . 0

[Ans. (1/s)2 × F1(!, 1 ; ∋ ; s]

[Hint. Use Art. 14.15] 4. Prove that F(!, ∋ + 1 ; ∗ + 1 ; x) – F(!, ∋ ; ∗ ; x) =

!(∗ # ∋) xF(! ∃ 1, ∋ ∃ 1 ; ∗ ∃ 2 ; x ). ∗ (∗ ∃ 1)

Created with Print2PDF. To remove this line, buy a license at: http://www.software602.com/

14.18

Hypergeometric Function

5. The complete elliptic integral of the first kind is

z

Θ/2

K=

dς 2

2

(1 # k sin ς)

0

. Show that K =

FH

IK

Θ F 1 , 1 ; 1 ; k2 . 2 2 2

[Kanpur 2011]

6. The complete elliptic integral of the second kind is Θ/2

E= 3

FH

IK

(1 # k 2 sin 2 ς) d ς . Show that E = Θ F # 1 , 1 ; 1 ; k 2 , | k | < 1. 2

0

2 2

7. Prove the following relations : (i) F(! – 1, ∋ – 1 ; ∗ ; x) – F(!, ∋ – 1 ; ∗ ; x) =

(1 # ∋)x F(!, ∋ ; ∗ ∃ 1 ; x ) ∗

(ii) !F(! + 1, ∋ ; ∗ ; x) – (∗ – 1)F(!, ∋ ; ∗ – 1 ; x) = (! + 1 – ∗)F(!, ∋ ; ∗ ; x). 8. Prove that F(!, ∋; ∗; 1/2) = 2! F(!, ∗ – ∋ ; ∗ ; –1). 9. Show that (i) ex – 1 = xF(1 ; 2 ; x). (ii) (1 + x/!)ex = F(! + 1 ; ! ; x). 10. The incomplete Gamma function is defined by the equation ∗(!, x) =

z

Θ #t ! # t

0

e t

dt , ! > 0.

–1 !

Prove that ∗(!, x) = ! x F(! ; ! + 1 ; –x). 11. Prove that following relations : (i) ∋F(! ; ∋ ; x) = ∋F(! – 1 ; ∋ ; x) + xF(! ; ∋ + 1 ; x). (ii) !F(! + 1 ; ∋ ; x) – (∋ – 1)F(! ; ∋ – 1 ; x) = (! – ∋ + 1)F(! ; ∋ ; x). 12. Prove the following relations : !∋x

(i) F(!, ∋ ; ∗ ; x) – F(!, ∋ ; ∗ – 1 ; x) = # ∗ (∗ # 1) F(! ∃ 1, ∋ ∃ 1 ; ∗ ∃ 1 ; x) (ii) F(! + 1; ∗ ; x) – F(! ; ∗ ; x) = 1 x / ∗ 2 : F (! ∃ 1, ∗ ∃ 1; x) . 13. Hypergeometric function 2F1(!, ∋ ; ∗ ; x) = 1 ∃

! ∋ !(! ∃ 1). ∋(∋ ∃ 1) 2 x∃ x ∃ ... is the 1 ∗ 1 2 ∗ (∗ ∃ 1)

solution of the differential equation x(1 – x)y∆ + [∗ – (! + ∋ + 1)x]y – !∋y = 0. Show that

LM N

OP Q

d (a) dx 2 F1(!, ∋ ; ∗ ; x)

d

x&0

=

!∋ . ∗

d

i

(b) 2 F1 !, ∋ ; 12 ! ∃ 12 ∋ ∃ 12 ; 12 =

c h%c ∃ ! ∃ ∋h . %c ∃ !h%c ∃ ∋h %

1 2 1 2

1 2 1 2

1 2

1 2

1 2 1 2

i

(c) 2 F1 !, ∋ ; ∗ ; 12 = 2! 2F1(!, ∗ – ∋ ; ∗ ; –1). 14. Prove that (a) Pn(cos Ρ) = 2F1(–n ; n + 1 ; 1 ; sin2 Ρ/2). (b) Pn(cos Ρ) = (–1)n 2F1(n + 1, –n ; 1 ; cos2 Ρ/2). (c) Pn(x) =

F x # 1I H 2K

n 2 F1

FG #n, # n ; 1 ; x ∃ 1IJ . x # 1K H

[Purvanchal 2007]

15. Show that

FH

IK

(a) H2n(x) = (#1)n (2n !) 1F1 #n ; 1 ; x 2 . n!

2

n (b) H2n + 1(x) = (#1)

2(2n ∃ 1)! x n!

1F1

FH #n ; 3 ; x IK . 2 2

16. Show that Ln(x) = n! 1F1(–n ; 1 ; x). 17. Prove that, for |x| < a, 2F1 (a, b, c, x) = (1 – x)c–a–b 2F1 (c – a, c – b, c1, x). [Lucknow 2010] 18. State the confluent hyper geometric equation and explain its solution.

[Lucknow 2010]

Created with Print2PDF. To remove this line, buy a license at: http://www.software602.com/

15 Orthogonal Sets of Functions And Strum Liouville Problem 15.1. Orthogonality. Two functions f(x) and g(x) defined on some interval a ! x ! b are said to

#

be orthogonal on a ! x ! b if

b

a

f ( x) g (x) dx

0. 1/ 2

∃ b % || f (x) || = & f 2 ( x) dx ∋ ( a ) 15.2. Orthogonal set of functions. Let {fn(x)}, where n = 1, 2, 3, ... be a set of functions defined on some interval a ! x ! b. Then the set {fn(x)} is said to be an orthogonal set of functions

#

The norm || f (x) || of f (x) is defined by

#

on the interval a ! x ! b if

b

a

f m (x) f n (x) dx

0 , whenever m ∗ n..

15.3. Orthonormal set of functions. Let {fn(x)}, where n = 1, 2, 3, ... be a set of functions defined on some interval a ! x ! b. Then the set {fn(x)} is said to be orthonormal on a ! x ! b if they are orthogonal on a ! x ! b and all have norm 1. Thus, set {fn(x)} is orthonormal on a ! x ! b, if

#

b

a

+

f m ( x) f n ( x) dx = 0, when m ∗ n 1, when m n

#

i.e.,

b

a

f m ( x) f n ( x) dx = ,mn,

+

,mn = Kronecker delta = 0, when m ∗ n 1, when m n 15.4. Orthogonality with respect to a weight function. Let p(x) > 0. Then two functions f (x) and g(x) defined on some interval a ! x ! b are said to be orthogonal on a ! x ! b with respect where

#

to weight function p(x), if

b

a

p( x) f (x) g ( x) dx

0. 1/ 2

∃ b % || f (x) || = & p( x) f 2 ( x) dx ∋ ( a ) 15.5. Orthogonal set of functions with respect to a weight function. Let {fn(x)}, where n =1, 2, 3, ... be a set of functions defined on some interval a ! x ! b. Then the set {fn(x)} is said to be orthogonal on a ! x ! b with respect to weight function p(x) > 0, if Then, the norm || f (x) || of f (x) is defined by

#

b

a

p( x) f m (x) f n ( x) dx

#

0, whenever m ∗ n.

15.6. Orthonormal set of functions with respect to a weight function. Let {fn(x)}, where n = 1, 2, 3, ... be a set of functions defined on some interval a ! x ! b. Then the set {fn(x)} is said to be orthonormal with respect to a weight function p(x) > 0 if they are orthogonal with respect to weight function p(x) on a ! x ! b and all have norm 1. Thus, set {fn(x)} is orthonormal with respect to weight function p(x), if 15.1

Created with Print2PDF. To remove this line, buy a license at: http://www.software602.com/

15.2

Orthogonal Sets of Functions and Strum Liouville Problem

#

b

a

p( x) f m ( x) f n ( x) dx

when m ∗ n +1,0, when m n

, mn

Remark 1. Terms defined in Art. 15.1, 15.2 and 15.3 are particular cases of Art. 15.4, 15.5 and 15.6 respectively for p(x) = 1. 15.7. Working rule for getting orthonormal set {fn(x)} of functions corresponding to a known orthogonal set {fn(x)}, where n = 1, 2, 3, ..., where none of the functions fn(x) have zero norm. Divide each function fn(x) by its norm || fn(x) || and get a new function −n(x) = fn(x)/|| fn(x) ||. Then, we have . 0 02

1/ 2

b || −n(x) || = ∃& # − 2n ( x ) dx %∋

(

)

a

b

#

a

2 / ∃ fn (x) % & || f ( x ) || ∋ dx 1 13 ( n )

1/ 2

=

∃ 1 & || f n ( x ) || (

#

% f n2 ( x ) dx ∋ )

b

a

1/ 2

1 4 || f n ( x) || 1, || f n ( x) ||

showing that norm of −n(x) = 1. Hence the set {−n(x)} i.e., {fn(x)/|| fn(x) ||} is an orthonormal set of functions. 15.8. Gram–Schmidt process of Orthonormalization. [Kanpur 2010] Let {fn(x)}, where n = 1, 2, 3,... be a set of a linearly independent functions for each of which norm || fn(x) || exists and is non–zero. Then we wish to obtain an orthonormal set {−n(x)}, where n = 1, 2, 3, ... such that

#

b

a

− m ( x) − n ( x) dx

+

0, when m ∗ n 1, when m n

...(1)

We select f1(x) and obtain −1(x) = f1(x)/|| f1(x) || We next choose f2(x) and let F2(x) = f2 + c −1, where c is chosen in such a manner so that F2 may be orthogonal to −1, i.e.,

#

b

or

#

b

a

a

F2 −1 dx

0

f 2−1 dx 5 c

6 c= 7

#

b

a

b

# − dx a

2 1

0

or

#

b

or

#

b

a

a

f 2−1 dx . With this value of c, (3) gives

...(2) ...(3)

( f 2 5 c −1) −1 dx

f 2−1 dx 5 c

0 , using (3)

0, by (1)

F2 = f2 – −1

#

b

a

f 2−1 dx

...(4)

We now take −2(x) = F2 /|| F2 || ...(5) Now choose f3 and let F3(x) = f3 + c1−1 + c2−2, ...(6) where c1 and c2 are chosen in such a manner so that F2 may be orthogonal to −1 and −2, i.e.,

#

b

#

b

a

or

a

6

#

b

( f 3 5 c1−1 5 c2−2 )−1 dx

and

f 3−1 dx 5 c1

0

and

#

f 3−1 dx

and

c2 = 7

c1 = 7

#

b

a

F3 = f3 7 −1

a b

a

#

b

( f3 5 c1−1 5 c2−2 )−2 dx f 3 −2 dx 5 c2

#

b

a

0 , using (1)

f3−2 dx

#

b

...(7)

f3− 2 dx

...(8)

So we take −3(x) = F3/|| F3 || By continuing the above process, the nth normalized function −n is given by −n = Fn/|| Fn ||,

...(9)

Using (7), (6) gives

where

Fn = f n 7 −1

#

b

a

f n−1 dx 7 −2

#

b

a

a

f 3−1 dx 7 −2

0

f n−2 dx 7 ... 7 −n 71

#

b

a

a

f n −n 71 dx

...(10) ...(11)

Created with Print2PDF. To remove this line, buy a license at: http://www.software602.com/

Orthogonal Sets of Functions and Strum Liouville Problem

15.3

Above process will fail if and only if at some stage Fr = 0 for 1 ! r ! n. But, Fr = 0 8 f r 7 −1

#

b

a

f r −1 dx 7 −2

#

b

a

f r −2 dx 7 ... 7 −r 71

#

b

a

f r −r 71 dx

0

8 fr is a linear combination of −1, −2, ..., −r – 1 8 fr is a linear combination of f1, f2, ..., fr – 1 (by our construction of new functions −1, −2, ..., −r – 1) 8 f1, f2, ..., fr is a linearly dependent set for 1 ! r ! n. But this a contradiction because {fn(x)}, where n = 1, 2, 3, ..., is linearly independent set and its every subset would also be linearly independent. Hence Fr ∗ 0 for 1 ! r ! n and so we would always get a set {−n(x)}, where n = 1, 2, 3, ... which would be orthonormal. Remark. Sometimes orthonormalization is required with respect to a weight function

#

p(x) > 0. Then (1) takes the form Also,

#

b

a

F2−1 dx

b

a

0 would take the form

#

b

a

when m ∗ n +1,0, when m n

p( x) −m ( x) − n ( x) dx

p( x) F2−2 dx

0 etc. Rest of the procedure is similar..

15.9. Illustrative Solved Examples Ex. 1. Show that the set of functions {sin (n9x/c)}, n = 1, 2, 3, ... is orthogonal on the interval (0, c) and find the corresponding orthonormal set. Sol. Here the given functions are fn(x) = sin (n9x/c), n = 1, 2, 3, ... For m ∗ n, we have

#

c

0

f m ( x) f n ( x) dx

c

# sin 0

m9x n9x sin dx = 1 c c 2

# +cos c

0

:

(m 7 n)9x (m 5 n)9x 7 cos dx c c c

(m 7 n)9x (m 5 n)9x / . c c = 10 sin 7 sin 1 2 2 9(m 7 n) c 9(m 5 n) c 30

0,

showing that the given set of functions is orthogonal.

∃ Norm of fn(x) = || fn(x) || = & (

1/ 2

% sin 2 (n9x / c) dx ∋ 0 )

#

c

.1 02 2

1/ 2

/ {1 7 cos (2n9x / c)}1 0 3

#

c

c

.{x / 2 7 (c / 4 n9) sin(2 n9x / c )}1/ 2 / 2 30

( c / 2)1/ 2 .

Let −n(x) = fn(x)/|| fn(x) || = (2/c)1/2 sin(n9x/c) Hence the required orthonormal set is given by {−n(x)} i.e., {(2/c)1/2 sin (n9x/c)} where n = 1, 2, 3, ... Ex. 2.(a) Show that the set of functions {cos nx}, n = 0, 1, 2, 3, ... is orthogonal on the interval –9 ! x ! 9, and find the corresponding orthonormal set of functions. (b) Show that the set of functions {cos nx}, n = 0, 1, 2, 3, ... is an orthogonal set of functions on 0 ! x ! 9, and find the corresponding orthonormal set of functions. Sol. (a) Here the given functions are fn(x) = cos nx, n = 0, 1, 2, ... For m ∗ n, we have

#

9

#

9

79

=

0

f m ( x) f n (x) dx

#

9

79

cos mx cos nx dx

{cos (m 5 n) x 5 cos (m 7 n) x} dx

2

#

9

0

cos mx cos nx dx 9

. sin (m 5 n) x sin (m 7 n) x / 5 02 m 5 n m 7 n 13 0

0,

showing that the given set of functions is orthogonal.

Created with Print2PDF. To remove this line, buy a license at: http://www.software602.com/

15.4

Orthogonal Sets of Functions and Strum Liouville Problem

Norm of fn(x), i.e., || fn(x) ||, for n ∗ 0 1/ 2

9 = ∃&# fn2 (x) dx%∋

(

∃ & (

1/2

∃ 9 2 % ∃ &2 cos nx dx∋ = & ( 0 ) (

% 2 cos nxdx∋ 79 )

#

)

79

1/ 2

9

#

#

9

0

1/ 2

. || fn(x) || = || 1 || = 0 2

Again, for n = 0,

.+ x 5 (1/ 2n) sin 2nx:1/ 2 / 2 30

1/ 2

9

/ 2 (1) dx 1 79 3

#

9

29

∃1/ 29 , if n 0 & ((cos nx) ; 9 , if n ∗ 0

f n ( x) || f n ( x) ||

−n(x) =

Thus,

9

% (1 5 cos 2nx) dx ∋ )

Hence, the required orthonormal set is 1/(29)1/ 2 , (cos x) / 9, (cos 2x) / 9, (cos 3x) / 9 ,... (b) Proceed as in part (a), Ans. 1/ 9, (2 / 9)1/ 2 cos x, (2 / 9)1/ 2 cos 2x, (2 / 9)1/ 2 cos 3x,... Ex. 3. (a) Show that the functions sin x, sin 2x, sin 3x, ..., 1, cos x, cos 2x, cos 3x, .... constitute an orthogonal set on the interval (–9, 9). Normalize the set. [Lucknow 2010] Sol. For m ∗ n, we have the following results :

#

(i)

9

79

#

= (ii)

0

#

9

#

9

79

2

9

0

sin mx sin nx dx , as sin mx sin nx is an even function 9

. sin (m 7 n) x sin (m 5 n) x / 7 02 m 7 n m 5 n 13 0

{cos (m 7 n ) x 7 cos (m 5 n) x} dx

#

9

(iv)

#

9

79

#

9

79

0

sin mx cos nx dx

0, as sin mx cos nx is an odd function of x.

cos mx cos nx dx

2

#

9

0

cos mx cos nx dx, as cos mx cos nx is an even function of x 9

. sin (m 5 n)x sin (m 7 n) x / {cos (m 5 n)x 5 cos (m 7 n) x} dx 0 5 m 7 n 13 0 0 2 m5n

=

(v)

9

79

(iii)

#

sin mx sin nx dx

0

0, as sin mx is an odd function.

1. sin mx dx

#

1. cos nx dx

2

9

0

9

cos nx dx

(2 / n) > < sin nx =

0

0

Relations (i), (ii), (iii), (iv) and (v) together show that the given set of functions is orthogonal. 1/2

∃9 % Now, || sin nx || = & sin2 nxdx∋ ( 79 )

#

∃ || cos nx || = & (

#

and

1/ 2

% 2 cos nx dx ∋ 79 ) 9

1/2

1/2

∃ 9 % ∃ 9 2 % &2 sin nx dx∋ = &( 0 (1 7 cos 2nx) dx∋) ( 0 )

#

1/ 2

∃ 9 2 % &2 cos nx dx ∋ ( 0 )

#

∃ || 1 || = & ( Hence the required orthonormal set is

= 9

∃ & (

#

#

1/ 2

% (1 5 cos 2nx) dx ∋ )

9

0

1/ 2

% 12 dx∋ 79 )

#

+< x= :

9 1/ 2 79

1/ 2

9 ?∃. 1 / ?% &0 x 7 sin 2nx1 ∋ 30 ?) ?(2 2n

9

1/ 2

9 ?∃ . 1 / ?% & 0 x 5 2n sin 2nx 1 ∋ 3 0 )? (? 2

9

29

(sin x) / 9, (sin 2x) / 9 ,...., 1/ 29 , (cos x) / 9, (cos 2x) / 9 , .... Ex. 3. (b) Show that the functions 1, cos (2n9x/T), sin (2n9x/T), n = 1, 2, 3, .... are orthogonal on the interval –T/2 ! x ! T/2, and find the corresponding orthonormal set.

+

:

Sol. Do as in Ex. 3(a). Ans. 1 / T , (2 / T )1/ 2 cos (2 n 9x / T ), (2 / T )1/ 2 sin (2 n 9 x / T ) , n = 1, 2, 3, ...

Created with Print2PDF. To remove this line, buy a license at: http://www.software602.com/

Orthogonal Sets of Functions and Strum Liouville Problem

15.5

Ex. 4. (a) Show that the functions f1(x) = 1, f2(x) = x are orthogonal on the interval (–1, 1) and determine the constants A and B so that the function f3(x) = 1 + Ax + Bx2 is orthogonal to both f1 and f2 on the interval (–1, 1). [Meerut 2007; Kanpur 2011] 3 (b) Show that the functions f1(x) = 4 and f2(x) = x are orthogonal on the interval (–2, 2) and determine constants A and B so that the function f3(x) = 1 + Ax + Bx2 is orthogonal to both f1 and f2.

#

Sol. (a) Here

1

71

f1( x) f 2 ( x) dx

#

1

71

1

. x 2 / 2/ 2 3 71

x dx

0,

showing that f1(x) and f2(x) are orthgonal on the interval (–1, 1). If f3(x) is orthogonal to both f1(x) and f2(x) on the interval (–1, 1), then by definition,

#

1

or

#

1

or

. x 5 A x2 5 B x3 / 2 3 31 71 20

f1( x) f3 ( x) dx

71

71

0

2

(1 5 Ax 5 Bx ) dx

#

and

1

71

1

f 2 ( x) f 3 ( x) dx

0

0

and

#

0

and

. 1 x 2 5 1 Ax 3 5 1 Bx 4 / 3 4 20 2 31 71

71

1

2

x(1 5 Ax 5 Bx ) dx

0 1

0

2 + (2/3)×B = 0 and (2/3)×A = 0 so that A = 0 and B = –3. So f3 is orthogonal to both f1 and f2 if A = 0 and B = –3. (b) Proceed as in part (a). Ans. A = 0 and B = –3/4. Ex. 5. Given that f1(x) = a0, f2(x) = b0 + b1 x and f3(x) = c0 + c1 x + c2 x2. Determine the constants a 0 , b 0 , c 0 , b1, c1 and c2 so that the given functions form an orthonormal set on the interval –1 ! x ! 1. Sol. Since f1, f2 and f3 form an orthonormal set on –1 ! x ! 1, we have or

1

#

(i)

71

f1 f 2 dx 0

(ii)

(iv) || f1 || ∗ 0 Now, (i) 8

#

1

71

71

f 2 f 3 dx

1

71

1

71

f3 f1 dx

0,

(vi) || f3 || ∗ 0. 1

1 0 8 . a0b0 x 5 a0b1x2 / 2 20 31 71

a0 (b0 5 b1x) dx

#

#

(iii)

0

2

(b0 5 b1x) (c0 5 c1 x 5 c2 x ) dx

0

8

a0 b0

1

...(1)

b0(3c0 + c2) + b1c1 = 0

...(2)

0 1

.b c x 5 1 b c x 2 5 1 b c x 3 5 1 b c x 2 5 1 b c x 3 5 1 b c x 4 / 2 0 1 3 0 2 2 1 0 3 11 4 1 2 31 71 20 0 0

or

2b0c0 + (2/3)×b0c2 + (2/3)×b1c1 = 0 (iii) 8

or

1

(v) || f2 || ∗ 0

(ii) 8

or

#

#

1

71

2

a0 (c0 5 c1 x 5 c2 x ) dx

2a0c0 + (2/3)×a0c2 = 0

. 0 2 Now, (1) and (4) give (iv) 8

#

1

2 3 0 8 .a0c0 x 5 1 a0c1x 5 1 a0c2 x / 2 3 20 31 71

a0(3c0 + c2) = 0

or 1/ 2

/ a02 dx 1 71 3 1

or

0

∗ 0 8 (2a02 )1/ 2 ∗ 0 8 a0 ∗ 0 b0 = 0.

0

...(3) ...(4) ...(5)

Created with Print2PDF. To remove this line, buy a license at: http://www.software602.com/

15.6

Orthogonal Sets of Functions and Strum Liouville Problem

Again, since b0 ∗ 0, so by (2),

. 0 2

6 (v) 8

b1c1 = 0 1/ 2

/ (b0 5 b1x) 2 dx 1 71 3

#

1

Thus, [(2/3)×b21]1/2 ∗ 0 6 from (6), we get

. Finally, (vi) 8 0 2

+

6

1/ 2

/ b12 x 2dx 1 71 3

#

1

∗ 0, using (5) b1 ∗ 0 b1 ∗ 0

so that c1 = 0, 1/ 2

/ (c0 5 c1x 5 c1x 2 ) 2 dx 1 71 3

#

. ∗080 2

...(6)

1

as

∗ 0 8 .0 2

1/ 2

/ (c0 5 c2 x 2 )2 dx 1 71 3

#

...(7) ...(8)

1

:

1

.c 2 x 5 (2 / 3) > c c x3 5 (1/ 5) > c 2 x5 / 0 2 2 2 0 3 71

1/ 2

∗0,

∗0

{2c02+ (4/3)×c0c2 + (2/5)×c22}1/2 ∗ 0 or {c02 + (2/3)×c0c2 + (1/5)×c22}1/2 ∗ 0 ...(9) From (3) and (5), we get 3c0 + c2 = 0 or c2 = –3c0 ...(10) Using (10), (9) can be re–written as {c02 – 2c02 + (9/5)×c02}1/2 ∗ 0 so that c0 ∗ 0 ...(11) From (4), (5), (7), (8), (10) and (11), we find that b0 = 0, c1 = 0, c2 = –3c0 and a0, c0 can take arbitrary real values. Ex. 6. Show that the functions 1 – x, 1 – 2x + x2/2 and 1 – 3x + 3x2/2 – x3/6 are orthogonal with respect to e–x on 0 ! x < ≅. Determine the corresponding orthonormal functions. Sol. Let f1(x) = 1 – x, f2(x) = 1 – 2x + x2/2 and f3(x) = 1 – 3x + 3x2/2 – x3/6 Here the given weight function = p(x) = e–x. or

#

Now,



#

p(x) f1 f 2 dx

0



0

=

#

7x

#

2

e (1 7 x) (1 7 2x 5 x / 2) dx =



0

#

7x 0

e x dx 7 3



0

7x

e x dx 5

5 2

#



0



0

7x

2

3

e (1 7 3x 5 5x / 2 7 x / 2) dx

7x 2

e x dx 7

1 2

#



0

7x 3

e x dx

...(1)

To evaluate integrals involved in (1), we make use of the following result of Gamma function,

#



#



0

Using (2), (1) 8 Next,

#



0

p( x) f 2 f 3 dx

0

7x n

n! , if n is a non–negative integer

e x dx

p( x) f1 f 2 dx

#



7x

2



0

#

Thus,



0

#



0

p( x) f 3 f1 dx

0! 7 3 4 1! 5 (5 / 2) 4 2! 7 (1/ 2) 4 3! 1 7 3 5 5 7 3 0 ...(3) 2

3

e (1 7 2 x 5 x / 2) (1 7 3x 5 3x / 2 7 x / 6) dx

0

= # e7x (1 7 5x 5 8x2 714x3 / 3 5 13x4 /12 7 x5 /12) dx

Again,

...(2)

#



0

7x

0! 7 5 4 1! 5 8 4 2! – (14 / 3) 4 3! 5 (13 /12) 4 4! 7 (1/12) 4 5!, using (2)

p( x) f 2 f 3 dx 1 7 5 5 16 7 28 5 26 7 10 2

0

...(4)

3

e (1 7 3x 5 3x / 2 7 x / 6) (1 7 x) dx

Created with Print2PDF. To remove this line, buy a license at: http://www.software602.com/

Orthogonal Sets of Functions and Strum Liouville Problem

#

=



0

7x

2

3

4

e (1 7 4x 5 9x / 2 7 5x /3 5 x /6) dx

#

Thus,



0

15.7

0! – 4 41! 5 (9 / 2) 4 2! – (5 / 3) 4 3! 5 (1/ 6) 4 4!, using (2)

p( x) f1 f 3 dx 1 7 4 5 9 7 10 5 4

...(5)

0

From (3), (4) and (5) we find that f1, f2, f3 are orthogonal with respect to weight function p(x) on 0 ! x < ≅. Also, we have . || f1 || = 0 2

#



0

. || f2 || = 0 2

#

1/ 2

/ 7x 2 e (1 7 x) dx 1 3 ≅

0

. =0 2

#

1/ 2



/ 7x 2 e (1 7 2x 5 x ) dx 1 3

0

1/ 2

/ e7 x (1 7 2x 5 x 2 /2)2 dx 1 3

. = 0 2



#

0

= [0! – 2·1! + 2!]1/2 = 1 1/ 2

/ e7 x (1 7 4 x 5 5x 2 7 2x3 5 x4 / 4) dx 1 3

[0! 7 4 41! 5 5 4 2! – 2 4 3! 5 (1/ 4) 4 4!]1/ 2 = (1 – 4 + 10 – 12 + 6)1/2 = 1 . || f3 || = 0 2

. = 0 2

#



0

#



0

1/ 2

/ 7x 2 3 2 e (1 7 3x 5 3x / 2 7 x /6) dx 1 3

1/ 2

/ e 7 x (1 7 6 x 5 12x 2 7 28x 3 / 3 5 13x 4 / 4 7 x 5 / 2 5 x 6 / 36) dx 1 3

[0! 7 6 41! 5 12 4 2! – (28/ 3) 4 3! 5 (13/ 4) 4 4! 7 (1/ 2) 4 5! 5 (1/ 36) 4 6!]1/ 2 = (1 – 6 + 24 – 56 + 78 – 60 + 20)

1/2

=1

Since norm of each of the functions f1, f2 and f3 is unity, it follows that the given set of functions is an orthonormal set. Ex. 7. With help of 1, x, x2 construct three functions −0, −1 and −2 which are orthogonal over –1 ! x ! 1. Sol. We take −0 (x) = 1 ...(1) Next, choose −1 = x + c −0(x) = x + c ...(2) Let −1 be orthogonal to −0 so that

#

1

71

−0−1 dx

#

or

0

1

71

( x 5 c) dx

0

1

. 1 x 2 5 cx / = 0 02 2 13 71

or

giving c = 0. Hence (2) gives −1 = x. Next, we take −2 = x2 + c1−0 + c2−1 = x2 + c1 + c2 x Let −2 be orthogonal to both −0 and −1 so that

#

1

#

1

71

i.e.,

71

−2−0 dx 2

0

( x 5 c1 5 c2 x) dx

0 1

and

#

1

and

#

1

71

71

− 2−1 dx

...(3) ...(4)

0

2

( x 5 c1 5 c2 x) x dx

0

1

4 2 3 .(1/ 3) > x 3 5 c1x 5 (1/ 2) > c2 x 2 / 0 and .(1/ 4) > x 5 (1/ 2) > c1x 5 (1/ 3) > c2 x / 0 2 3 71 2 371 i.e., (2/3) + 2c1 = 0 and c2 × (2/3) = 0 so that c1 = –1/3, c2 = 0 6 (4) becomes −2(x) = x2 – (1/3). So the required functions are −0 = 1, −1 = x, −2 = x2 – (1/3).

i.e.,

Created with Print2PDF. To remove this line, buy a license at: http://www.software602.com/

15.8

Orthogonal Sets of Functions and Strum Liouville Problem 2

Ex. 8. With the help of 1, x, x construct three functions −0, −1 and −2 which are orthogonal with respect to e–x over 0 ! x < ≅. Sol. We take −0(x) = 1 ...(1) Next, choose −1 = x + c −0(x) = x + c ...(2) –x Let −1 be orthogonal to −0 with respect to e . Then, we have

#



7x

e ( x 5 c) dx

0

0

#

or

#

We know that



0



0

7x n

e x dx

7x

e x dx 5 c



0

7x

e dx

0

...(3)

n = 0, 1, 2, 3, .....

where

n !,

#

Using (4), (3) gives 1! + c × 0! = 0 or 1+c=0 or Hence (2) reduces to −1 = x – 1 Finally, take −2 = x2 + c1−0 + c2−1 = x2 + c1 + c2(x – 1) Let −2 be orthogonal to −0 and −1 with respect to e–x. Then, we have

#



#



0

and or

7x

e −2−0 dx

0

7x

e −2−1 dx

0

i.e.,

#

0

i.e.,

#

0

#

7x

2

7x

2

e

7x

+x

3

c = –1. ...(5) ...(6)

e ( x 5 c2 x 5 c1 7 c2 ) dx

:

2

5 (c2 7 1) x 5 (c1 7 2c2 ) x 5 (c2 7 c1) dx

...(7)

0

e (x 5 c2 x 5 c1 7 c2 ) ( x 7 1) dx

2! + c2 × 1! + (c1 – c2) × 0! = 0 so that



0



0

Using (4), (7) gives 2 + c2 + c1 – c2 = 0 Now, by (8),



...(4)

0

c1 = –2

...(8) ...(9)

0

3! + (c2 – 1) × 2! + (c1 – 2c2) × 1! + (c2 – c1) × 0! = 0, using (4) 6 + 2(c2 – 1) + c1 – 2c2 + c2 – c1 = 0 so that c2 = –4. Since c1 = –2, and c2 = –4, so (6) gives −2 = x2 – 4x + 2. So required functions are −0(x) = 1, −1(x) = x – 1 and −2(x) = x2 – 4x + 2. Ex. 9. Given the set of functions 1, x, x2, x3, ...,. Obtain from these a set of functions which are mutually orthonormal in (–1, 1). [Kanpur 2009] Sol. Let {−n(x)} be the required orthonormal set of function so that or or

#

1

71

+1,0, ifif mm ∗ nn

−n −m dx

−1(x) = f1(x)/|| f1(x) ||.

Step 1. Choose f1(x) = 1 and take

. Now, || f1(x) || = 0 2

1/ 2

/ f12 ( x) dx 1 71 3

#

1

...(1)

. 0 2

1/ 2

/ 12 dx 1 71 3

#

1

Step 2. Choose f2(x) = x and take a function

2 . So

−1 ( x )

1

...(2)

2

g2(x) = f2(x) + c −1(x) = x + (c / 2)

...(3)

Let g2(x) and −1(x) be orthogonal on the interval (–1, 1) 6 or

#

1

71

g 2 ( x) −1( x) dx 1

. x 2 / 2 5 cx / 2 / 2 3 71

0

or

0

so that

#

1

71

(x 5 c / 2) 4 (1/ 2) dx

0

c=0

Created with Print2PDF. To remove this line, buy a license at: http://www.software602.com/

Orthogonal Sets of Functions and Strum Liouville Problem

15.9

g2(x) = x

Hence, by (3), . || g2(x) || = 0 2

Now,

1/ 2

/ 2 g2 (x) dx 1 71 3

#

1

. 0 2

Α 23 Β

1/ 2

1/ 2

/ 2 x dx 1 71 3

#

1

6 −2(x) = g2(x)/|| g2(x) || = (3/2)1/2·x 2 Step 3. Choose f3(x) = x and take a function g3(x) = f3(x) + c1 −1(x) + c2 −2(x) = x2 + c1 −1 + c2 −2 Let g3(x) be orthgonal to −1(x) and −2(x) so that (i)

#

1

71

g3 ( x) −1( x) dx

#

(i) 8

1

2

71

( x 5 c1−1 5 c2− 2 ) −1 dx

#

or

1

71

0

#

or

1

#

Next, from (ii),

71

#

or

1

2

x 4 (1/ 2) dx 5 c1 41 5 c2 4 0

1/ 2

(3 / 2)

1/ 2

x > (3/ 2)

1

. x 4 / 4 / 5 c2 2 3 71

0 or

71

2

#

. = 01 29

1/ 2

.1 09 2

1

71

2

−1 dx 5 c2

#

1

71

c1 = 7

#

1

71

x 2−2 dx 5 c1

#

1

71

Α

2 /3

Β

−1− 2 dx 5 c2

ΑΒ 7 2

1/ 2

0

...(6)

#

1

71

− 22 dx

0

c2 = 0

1/ 2

/ 2 2 (3x 7 1) dx 1 71 3

#

1/ 2

1

2

(3x 7 1) / 3

1/ 2

1 1 ∃? . 9 5 3 / %? &0 x 7 2 x 5 x 1 ∋ 3 ?( 2 5 371 )?

6 −3(x) = g3(x)/|| g3(x) || = (1/2) × (5/2)1/2 × (3x2 – 1) Proceeding like wise, we obtain −4(x) =

−2−1 dx

0 , using (1) and (2)

1

/ 4 2 (9 x 7 6x 5 1) dx 1 71 3

#

#

0

or

/ 2 g3 ( x) dx 1 71 3 1

g3 ( x) −2 ( x) dx

x −1 dx 5 c1

g3(x) = x 2 7 ( 2 / 3) > (1/ 2) . || g3(x) || = 0 2

1

...(5)

> x dx 5 0 5 c2 >1 0 , using (1) and (4)

0

6 (5) gives Now,

#

or

(x 2 5 c1−1 5 c2−2 ) −2 dx

2

71

1

71

(2/3) × (1/ 2) + c1 = 0

or

or

(ii)

0

...(4)

Χ 3 ∆ 4 Ε 5 x 7 3x Φ , 2 Γ Η

−5(x) =

ΑΒ 9 2

1/ 2

2 Χ 4 ∆ . Ε 5 x 7 30 x 5 3 Φ 8 Γ Η

2 2 3 5

...(7)

...(8)

and so on. The required orthonormal set of functions {−n(x)} is given by (2), (4), (7), (8) and so on.

EXERCISE 15(A) 1. Show that the functions sin x, sin 2x, sin 3x, ... are orthogonal on the interval (0, 9). 2. Show that the functions 1, cos 2x, cos 4x, cos 6x, ... are orthogonal on interval 0 ! x ! 9, and find the corresponding orthonormal set.

Ans. {1/ 9 , (2/9)1/2 cos 2nx}, n = 1, 2, 3, ...

Created with Print2PDF. To remove this line, buy a license at: http://www.software602.com/

15.10

Orthogonal Sets of Functions and Strum Liouville Problem

3. Show that the functions sin 9x, sin 29x, sin 39x, ... form an orthogonal set on the interval –1 ! x < 1 and obtain the corresponding orthonormal set. Ans. sin 9x, sin 29x, sin 39x, ... i.e., the given set itself. 4. Show that the functions 1, cos 9x, sin 9x, cos 29x, sin 29x, ... form an orthogonal set on –2 ! x ! 2 and find the corresponding orthonormal set. Ans. 1/2, (cos 9x)/2, (sin 9x)/2, (cos 29x)/2, (sin 29x)/2, ... 5. Show that each of the following set is orthogonal on the given interval and find the corresponding orthonormal sets : (i) 1, cos (29x/c), cos (49x/c), cos (69x/c), ..., 0 ! x ! c Ans. {1 / c , (2 / c )1 / 2 cos (2 n 9 x / c )} , n = 1, 2, 3, ... (ii) 1, cos 2x, sin 2x, cos 4x, sin 4x, ..., –9/2 ! x ! 9/2 Ans. 1/ 9 , (2/9)1/2 cos 2x, (2/9)1/2 sin 2x, ... 15.10. Strum–Liouville equation. Strum–Liouville problem. Eigen (or characteristic) functions and eigen (or characteristic) values. [Meerut 2010; Ravishankar 1998; Himanchal 2010] Definitions. A differential equation of the form [r(x) yΙ]Ι + [q(x) + ϑ p(x)]y = 0 ... (1) is known as Strum–Liouville equation. We assume that the functions p, q, r and rΙ in (1) are continuous in a ! x ! b and p(x) > 0. Here ϑ is a parameter independent of x. Equation (1) is considered on some interval a ! x ! b, satisfying boundary conditions at the two end points a and b, a1 y(a) + a2 yΙ(a) = 0 and b1 y(b) + b2 yΙ(b) = 0 ...(2) with the real constants a1, a2, b1, b2. Suppose that a1, a2 in (2) are not both zero and so are b1, b2. The boundary value problem consisting of (1) and (2) is called a Strum–Liouville problem. Clearly y = 0 is always a solution of Strum–Liouville problem for any value of the parameter ϑ. y = 0 known as a trivial solution is of no practical use. The non–zero solutions of the Strum– Liouville problem given by (1) and (2) are called the eigenfunctions of the problem and the values of ϑ for which such solutions exist, are called eigenvalues of the problem. Remark. A special case of (1) and (2). Let p = r = 1 and q = 0 in (1). Also, let a1 = b1 = 1 and a2 = b2 = 0 in (2). Then (1) and (2) reduce to yΙΙ + ϑy = 0 with y(a) = 0, y(b) = 0. This is the simplest form of Strum–Liouville problem. 15.11. Orthogonality of eigenfunctions Theorem. Suppose that the functions p(x), q(x), r(x) and rΙ(x) in the Strum–Liouville equation [r(x) yΙ]Ι + {q(x) + ϑp(x)}y = 0 are real valued and continuous and p(x) > 0 on the interval a ! x ! b. Let ym(x) and yn(x) be eigenfunctions of the Strum–Liouville problem (given by the above Strum Liouville equation and boundary conditions a1 y(a) + a2 yΙ(a) = 0 and b1 y(b) + b2 yΙ(b) = 0) that correspond to different eigenvalues ϑm and ϑn respectively. Then ym, yn are orthogonal on that interval with respect to the weight function p(x). Prove that eigenfunctions corresponding to different eigenvalues are orthogonal with respect to some weight function. [Himanchal 2009] Proof. Consider the following Strum–Lioville problem : [r(x) yΙ]Ι + [q(x) + ϑ p(x)]y = 0 ...(1) a1 y(a) + a2 yΙ(a) = 0 ...(2a) b1 y(b) + b2 yΙ(b) = 0, ...(2b) where p, q, r and rΙ are real valued and continuous and p(x) > 0 on a ! x ! b. Let a1, a2 in (2a) be given constants, not both zero and so be b1, b2 in (2b). Let ym and yn be eigenfunctions of the above Strum–Liouville problem that correspond to different eigenvalues ϑm and ϑn. Then, by definition of eigen functions, ym and yn both satisfy (1).

Created with Print2PDF. To remove this line, buy a license at: http://www.software602.com/

Orthogonal Sets of Functions and Strum Liouville Problem

15.11

Hence (ryΙm)Ι + (q + ϑm p) ym = 0 ...(3) and (ryΙn)Ι + (q + ϑn p) yn = 0 ...(4) Multiplying (3) by yn and (4) by ym and subtracting, we get (ryΙm)Ι yn – (ryΙn)Ι ym + (ϑm – ϑn)p ymyn = 0 or (ϑm – ϑn)p ymyn = (ryΙn)Ι ym – (ryΙm)Ι yn d (ϑm – ϑn)p ymyn = dx {(ryΙn ) ym 7 (ryΙm ) yn},

or

...(5)

which can be verified by performing the indicated differentiation of the expression in brackets on R.H.S. of (5). Since r(x) and rΙ(x) are continuous by assumption and ym, yn are solutions of (1), it follows that the expression within brackets on R.H.S. of (5) is continuous on a ! x ! b. Integrating both sides of (5) over x from a to b, we thus obtain (ϑ m 7 ϑ n )

or (ϑ m 7 ϑ n )

#

b

a

#

b

a

b

p ym yn dx = 2. r ( yΙn ym 7 yΙm yn ) 3/ a

p ym yn dx = r(b) {yΙn(b) ym(b) – yΙm(b) yn(b)}– r(a) {yΙn(a) ym(a) – yΙm(a) yn(a)} ...(6)

We now have to consider several cases depending on whether r(x) vanishes or does not vanish at a or b. Case I. Let r(a) = r(b) = 0. Then (6) reduces to

(ϑ m 7 ϑ n )

#

b

a

p ym yn dx

0

...(7)

Cae II. Let r(b) = 0 but r(a) ∗ 0. Then (6) reduces to (ϑ m 7 ϑ n )

#

b

a

p ym yn dx

Ι (a) yn (a)} 7r(a){yΙn (a) ym (a) 7 ym

...(8)

Since ym and yn both satisfy (2a), we have a1 ym(a) + a2 yΙm(a) = 0 ...(9) and a1 yn(a) + a2 yΙn(a) = 0 ...(10) Let a2 ∗ 0. Multiplying (10) by ym(a) and (9) by yn(a) and then subtracting, we get a2{yΙn(a) ym(a) – yΙm(a) yn(a)} = 0 Since a2 ∗ 0, so yΙn(a) ym(a) – yΙm(a) yn(a) = 0 ...(11) Using (11), (8) reduces to (7). If a2 = 0, then let a1 ∗ 0. Now, multiplying (9) by yΙn(a) and (10) by yΙm(a) and then subtracting, we get a1{yΙn(a) ym(a) – yΙm(a) yn(a)} = 0 Since a1 ∗ 0, so yΙn(a) ym(a) – yΙm(a) yn(a) = 0 Hence as before (8) reduces to (7). Case III. Let r(a) = 0 but r(b) ∗ 0. Then (6) reduces to (ϑ m 7 ϑ n )

#

b

a

p ym yn dx

Ι (b) yn (b)} r (b){ yΙn (b) ym (b) 7 ym

...(12)

Since ym and yn both satisfy (2b), we have b1 ym(b) + b2 yΙm(b) = 0 ...(13) and b1 yn(b) + b2 yΙn(b) = 0 ...(14) Let b2 ∗ 0. Multiplying (14) by ym(b) and (13) by yn(b) and then subtracting, we get b2{yΙn(b) ym(b) – yΙm(b) yn(b)} = 0 Since b2 ∗ 0, so yΙn(b) ym(b) – yΙm(b) yn(b) = 0 ...(15) Using (15), (12) reduces to (7). If b2 = 0, then let b1 ∗ 0. Now, multiplying (13) by yΙn(b) and (14) by yΙm(b) and then subtracting, we get b1{yΙn(b) ym(b) – yΙm(b) yn(b)} = 0

Created with Print2PDF. To remove this line, buy a license at: http://www.software602.com/

15.12

Orthogonal Sets of Functions and Strum Liouville Problem

Since b1 ∗ 0, so yΙn(b) ym(b) – yΙm(b) yn(b) = 0 Hence, as before, (12) reduces to (7). Case IV. Let r(a) ∗ 0 and r(b) ∗ 0. There is no loss of generality by assuming that a2 ∗ 0 and b2 ∗ 0. Then, proceeding as in cases II and III, relations (11) and (15) can be proved. Then, using (11) and (15), (6) reduces to (7). Case V. Let r(a) = r(b). Proceed as in case IV to show that (6) reduces to (7). From the above discussion, we see that in all situations, we get (7). Since ϑm and ϑn are

#

different, (7) reduces to

b

a

p( x) ym ( x) yn (x) dx

0,

showing that ym(x) and yn(x) are orthogonal with respect to weight function p(x). 15.12. Reality of eigenvalues [Meerut 2010; Purvanchal 2005; Lucknow 2010] Theorem. To prove that all eigenvalues of Strum Liouville problem are real. Proof. Consider the following Strum–Liouville problem : [r(x) yΙ]Ι + [q(x) + ϑ p(x)]y = 0 ...(1) a1 y(a) + a2 yΙ(a) = 0 ...(2a) b1 y(b) + b2 yΙ(b) = 0, ...(2b) where p, q, r and rΙ are real valued and continuous and p(x) > 0 on a ! x ! b. Let a1, a2 in (2a) be given constants, not both zero, and so be b1, b2 in (2b). Let y(x) be an eigenfunction corresponding to an eigenvalue ϑ = Κ + iΛ, where Κ, Λ are real constants. This eigenfunction y(x) satisfies (1), (2a) and (2b) and may be a complex valued function. Taking the complex conjugates of all the terms in (1), (2a) and (2b), we get [r(x) y Ι ]Ι + [q(x) + ϑ p(x)] y = 0

...(3)

a1 y (a) + a2 y Ι (a) = 0

...(4a)

b1 y (b) + b2 y Ι (b) = 0

...(4b)

The above equations (3), (4a) and (4b) show that y ( x) is the eigenfunction corresponding to the eigenvalue ϑ = Κ – iΛ. Multiplying (1) by y and (3) by y and subtracting, we get Χ ∆Ι (r y Ι)Ι y 7 Ε r y Ι Φ y 5 (ϑ 7 ϑ) p y y Γ Η

0

or

(ϑ 7 ϑ) p y y = Α r y Ι Β Ι y 7 (r y Ι)Ι y

d ...(5) (ϑ 7 ϑ) p y y = dx {(r y Ι) y 7 (r y Ι) y} , which can be verified by performing the indicated differentiation of the expression in brackets on R.H.S. of (5). Integrating both sides of (5) w.r.t. ‘x’ from a to b, we thus obtain

or

(ϑ 7 ϑ)

or

#

(ϑ 7 ϑ)

b

a

#

b

a

p y y dx = [r ( y Ι y 7 y Ι y )]ba

p y y dx = r(b){y Ι(b) y(b) 7 y Ι(b) y (b)} 7r(a){y Ι(a) y(a) 7 y Ι(a) y (a)}

...(6)

We now have to consider several cases depending on whether r(x) vanishes or does not vanish at a or b. Case I. Let r(a) = r(b) = 0. Then (6) reduces to

(ϑ 7 ϑ)

#

b

a

p y y dx = 0

...(7)

Created with Print2PDF. To remove this line, buy a license at: http://www.software602.com/

Orthogonal Sets of Functions and Strum Liouville Problem

15.13

Case II. Let r(b) = 0 but r(a) ∗ 0. Then (6) reduces to

#

(ϑ 7 ϑ)

b

a

p y y dx = 7r(a){ y Ι(a) y(a) 7 y Ι(a) y (a)}

...(8)

Consider relations (2a) and (4a). Let a2 ∗ 0. Multiplying (4a) by y(a) and (2a) by y (a) and a2{ y Ι(a) y(a) 7 y Ι(a) y (a)} = 0

then subtracting, we get

Since a2 ∗ 0, so y Ι(a) y(a) 7 y Ι(a) y (a) 0 . ...(9) Using (9), (8) reduces to (7). If a2 = 0, then assume that a1 ∗ 0. Now, multiplying (2a) by y Ι(a) and (4a) by y Ι(a) and then subtracting, we get a1{y Ι(a) y(a) 7 y Ι(a) y (a)} 0 Since a1 ∗ 0, so

y Ι(a) y(a) 7 y Ι(a) y (a)

0

Hence as before (8) reduces to (7). Case III. Let r(a) = 0 but r(b) ∗ 0. Then (6) reduces to

#

(ϑ 7 ϑ)

b

a

p y y dx

r (b){ y Ι(b) y(b) 7 y Ι(b) y (b)}

...(10)

Consider relations (2b) and (4b). Let b2 ∗ 0. Multiplying (4b) by y(b) and (2b) by y (b) and b2{ y Ι(b) y(b) 7 y Ι(b) y (b)} 0 .

then subtracting, we get b2 ∗ 0,

Since

y Ι(b) y(b) 7 y Ι(b) y (b)

so

0

...(11)

Using (11), (10) reduces to (7). If b2 = 0, then assume that b1 ∗ 0. Now, multiplying (2b) by y Ι(b) and (4b) by yΙ(b) and then subtracting, we get b1{ y Ι(b) y(b) 7 y Ι(b) y (b)} 0 Since b1 ∗ 0, so

y Ι (b) y(b) 7 y Ι(b) y (b)

0

Hence as before (10) reduces to (7). Case IV. Let r(a) ∗ 0 and r(b) ∗ 0. There is no loss of generality by assuming that a2 ∗ 0 and b2 ∗ 0. Then proceeding as in cases II and III, relations (9) and (11) can be proved. Then, using (9) and (11), (6) reduces to (7). Case V. Let r(a) = r(b). Proceed as in case IV to show that (6) reduces to (7). From the above discussion, we see that in all situations we get (7). Now, ϑ = Κ + iΛ 8 ϑ = Κ – iΛ

and

ϑ – ϑ = (Κ + iΛ) – (Κ – iΛ) = 2iΛ.

hence

2

Again y y = |y| , where |y| stands for modulus of y. Then, (7) reduces to

#

2i Λ

b

a

Since

2

p( x) | y( x) | dx

#

b

a

0

or

Λ

#

b

a

2

p( x) | y( x) | dx

0

...(12)

2

p( x) | y( x) | dx has a positive value in the given interval a ! x ! b, (12) reduces to

Λ = 0 and hence ϑ Κ 5 ΜΛ ΚΝ which is real. Since ϑ is an arbitrary eigenvalue, it follows that eigenvalues of Strum–Liouville problem are all real. 15.13. SOLVED EXAMPLES Ex. 1. Find the eigenvalues and the corresponding eigenfunctions of X ΙΙ + ϑX = 0, X(0) = 0 and XΙ(L) = 0. [Himanchal 2009; Jiwaji 2004; Meerut 2006; Ravishakar 2005]

Created with Print2PDF. To remove this line, buy a license at: http://www.software602.com/

15.14

Orthogonal Sets of Functions and Strum Liouville Problem

Sol. Given XΙΙ + ϑX = 0 ...(1) with boundary conditions X(0) = 0 and XΙ(L) = 0 ...(2) Case I. Let ϑ = 0. Then solution of (1) is X(x) = Ax + B. ...(3) From (3), XΙ(x) = A ...(4) Replacing x by 0 in (3) and using (2), we get B = 0. Again, replacing x by L in (4) and using (2), we get A = 0. With A = 0, B = 0, (3) reduces to X(x) = 0. Since X(x) ∗ 0, so there is no eigen function corresponding to ϑ = 0. Case II. Let ϑ = – Ο2, where Ο ∗ 0 (i.e., ϑ is negative). So (1) gives XΙΙ – Ο2X = 0 whose solution is X(x) = AeΟx + Be–Οx ...(5) From (5), XΙ(x) = AΟeΟx – BΟe–Οx ...(6) Using (2), (5) and (6) reduces to 0=A+B and AΟeΟL – BΟe–ΟL = 0 ...(7) Solving (7), we find A = B = 0. Hence (5) gives X(x) = 0. Since X(x) ∗ 0, so there is no eigen function corresponding to ϑ = –Ο2. Case III. Let ϑ = Ο2 where Ο ∗ 0 (i.e., ϑ is positive). So (1) gives X ΙΙ + Ο2X = 0 whose solution is X(x) = A cos Οx + B sin Οx ...(8) From (8), X Ι(x) = –AΟ sin Οx + BΟ cos Οx ...(9) Using (2), (8) and (9) reduce to 0=A and 0 = BΟ cos ΟL Thus A = 0 and B cos ΟL = 0, as Ο∗0 ...(10) Now consider B cos ΟL = 0 ...(11) If B = 0, then with A = 0, (8) reduces to X(x) = 0, which is not an eigen function. So B ∗ 0 for the existence of eigen functions. Since B ∗ 0, (11) gives cos ΟL = 0 so that ΟL = (2n – 1)9/2, n = 1, 2, 3, ... 6 Ο = (2n – 1)9/2L ...(12) Using A = 0 and (12), (8) reduces to X(x) = B sin {(2n – 1)9/2L}, n = 1, 2, 3, ... and then ϑ = Ο2 = (2n – 1)292/4L2, n = 1, 2, 3, ... So required eigenfunctions Xn(x) with corresponding eigenvalues ϑn are Xn(x) = Bn sin {(2n – 1)9/2L}, and ϑn = (2n – 1)292/4L2, n = 1, 2, 3, ... Note. We can take Bn = 1 while writing eigenfunctions. Ex. 2. Find the eigenvalues and eigenfunctions of the Strum–Liouville problem XΙΙ + ϑX = 0, XΙ(0) = 0, XΙ(L) = 0. (Kanpur 2009; Meerut 1995) Sol. Given XΙΙ + ϑX = 0 ...(1) with boundary conditions XΙ(0) = 0 and XΙ(L) = 0 ...(2) Case I. Let ϑ = 0. Then solution of (1) is X(x) = Ax + B ...(3) From (3), XΙ(x) = A ...(4) Using (2), (4) gives 0=A and 0=A These gives A = 0, while B is arbitrary. Hence (3) reduces to X(x) = B which is non–zero. Hence taking B = 1, X(x) = 1 is an eigenfunction with ϑ = 0 as the corresponding eigenvalue. Case II. Let X = –Ο2, where Ο ∗ 0. Then (11) reduces to XΙΙ – Ο2X = 0 whose solution is X (x) = AeΟx + Be–Οx ...(5) Οx –Οx From (5), XΙ(x) = AΟe – BΟe ...(6) Using (2), (6) reduces to 0 = AΟ – BΟ and AΟeΟL – BΟe–ΟL = 0 i.e., A–B=0 and AeΟL – Be–ΟL = 0, as Ο ∗ 0. Solving these equations, A = B = 0. So (5) gives X(x) = 0, which is not an eigenfunction.

Created with Print2PDF. To remove this line, buy a license at: http://www.software602.com/

Orthogonal Sets of Functions and Strum Liouville Problem 2

15.15 2

Case III. Let ϑ = Ο , where Ο ∗ 0. Then (1) reduces to XΙΙ + Ο X = 0 whose solution is X(x) = A cos Οx + B sin Οx ...(7) From (7), X Ι(x) = –AΟ sin Οx + BΟ cos Οx ...(8) Using (2), (8) reduces to 0 = BΟ and 0 = –AΟ sin ΟL + BΟ cos ΟL i.e., B=0 and A sin ΟL = 0, as Ο∗0 ...(9) We now consider A sin ΟL = 0. ...(10) If A = 0, then with B = 0, (8) reduces to X(x) = 0, which is not an eigenfunction. So we take A ∗ 0 for the existence of eigenfunctions. Since A ∗ 0, (10) reduce to sin ΟL = 0 so that ΟL = n9, n = 1, 2, 3, ... 6 Ο = (n9/L), n = 1, 2, 3, ... ...(11) (n = 0 is omited as n = 0 8 Ο = 0 which is contrary to our assumption Ο ∗ 0). Using B = 0 and (11), (7) reduces to X(x) = A cos (n9x/L), n = 1, 2, 3, ... and then ϑ = Ο2 = n292/L2, n = 1, 2, 3, ... Hence the required eigenfunctions Xn(x) with the corresponding eigenvalues ϑn are given by (taking A = 1). Xn(x) = cos (n9x/L), ϑn = n292/L2, n = 0, 1, 2, 3, ... Ex. 3. Find all the eigenvalues and eigenfunctions of the Strum–Liouville problem yΙΙ + ϑy = 0 with y(0) + yΙ(0) = 0 and y(1) + yΙ(1) = 0. Sol. Given yΙΙ + ϑy = 0 ...(1) with boundary conditions y(0) + yΙ(0) = 0 ...(2) 6 y(1) + yΙ(1) = 0 ...(3) Case I. Let ϑ = 0. Then solution of (1) is y(x) = Ax + B ...(4) From (4), yΙ(x) = A ...(5) From (4) and (5), y(0) = B, yΙ(0) = A. With these values, (2) gives B+A=0 ...(6) From (4) and (5), y(1) = A + B and yΙ(1) = A. With these values (3) gives 2A + B = 0 ...(7) Solving (6) and (7), A = B = 0. Hence (4) reduces to X(x) = 0, which is not an eigenfunction and so ϑ =0 is not an eigenvalue. Case II. Let ϑ = –Ο2, where Ο ∗ 0. Then (1) reduces to yΙΙ – Ο2y = 0 whose solution is y(x) = AeΟx + Be–Οx ...(8) Οx –Οx From (8), yΙ(x) = AΟe – BΟe ...(9) From (8) and (9), y(0) = A + B and yΙ(0) = Ο(A – B) ...(10) Using (10), (2) reduces to A + B + Ο(A – B) = 0 i.e., A(1 + Ο) + B(1 – Ο) = 0 ...(11) Again, from (8) and (9), we have y(1) = AeΟ + Be–Ο and yΙ(1) = Ο(AeΟ – Be–Ο) ...(12) Ο –Ο Using (12), (3) reduces to Ae + Be + Ο(AeΟ – Be–Ο) = 0 Ο i.e., Ae (1 + Ο) + Be–Ο(1 – Ο) = 0 ...(13) We now use the theory of determinants for solving (11) and (13). For non–trivial solution of these equations, we must have

15 Ο 17 Ο eΟ (1 5 Ο) e 7Ο (1 7 Ο)

0 8

(1 + Ο) (1 – Ο) (e–Ο – eΟ) = 0,

giving

Ο = –1 and Ο = 1.

When Ο = –1, (11) and (13) give B = 0, while A will be arbitrary. So (8) reduces to y(x) = Ae–x and the corresponding eigenvalue is given by ϑ = –Ο2 = –(–1)2 = –1.

Created with Print2PDF. To remove this line, buy a license at: http://www.software602.com/

15.16

Orthogonal Sets of Functions and Strum Liouville Problem

Next when Ο = 1, (11) and (13) give A = 0, while B will be arbitrary. So (8) reduces to y(x) = Be– and the corresponding eigenvalue is given by ϑ = –Ο2 = –12 = –1. Taking A = B = 1, y(x) = e–x is an eigen function and ϑ = –1 is the corresponding eigenvalue. Case III. Let ϑ = Ο2, where Ο ∗ 0. Then (1) reduces to yΙΙ + Ο2y = 0 whose solution is y(x) = A cos Οx + B sin Οx. ...(14) From (14), yΙ(x) = –AΟ sin Οx + BΟ cos Οx ...(15) From (14) and (15), y(0) = A and yΙ(0) = BΟ. ...(16) Using (16), (2) reduces to A + BΟ = 0 ...(17) Again, from (14) and (15), we have y(1) = A cos Ο + B sin Ο and yΙ(1) = –AΟ sin Ο + BΟ cos Ο ...(18) Using (18), (3) reduces to A cos Ο + B sin Ο – AΟ sin Ο + BΟ cos Ο = 0. ...(19) From (17), A = –BΟ. With this value of A, (19) gives –BΟ cos Ο + B sin Ο + BΟ2 sin Ο + BΟ cos Ο = 0 or B(1 + Ο2) sin Ο = 0 or B sin Ο = 0, as (Ο2 + 1) ∗ 0. ...(20) If B = 0, then (17) gives A = 0. Hence (14) reduces to y(x) = 0, which is not an eigen function. So take B ∗ 0. Then (20) gives sin Ο = 0 so that Ο = n9, n = 1, 2, 3, ... ...(21) (Here we omit n = 0, for n = 0 gives Ο = 0 so that ϑ = Ο2 = 0 which has been considered in case I) Using A = –BΟ = –Bn9 and (21), (14) reduces to y(x) = B(sin n9x – n9 cos n9x) n = 1, 2, 3, ... and then ϑ = Ο2 = n292, n = 1, 2, 3, ... Hence the required eigenfunctions yn(x) with the corresponding eigenvalues ϑn are given by (taking B = 1) yn(x) = sin n9x – n9 cos n9x, n = 1, 2, 3, ... and ϑn = n292, n = 1, 2, 3, ... Ex. 4. Find all the eigenvalues and eigenfunctions of 4(e–xyΙ)Ι + (1 + ϑ)e –x y = 0, y(0) = 0, y(1) = 0. Sol. Re–writing the given equation, we have 4(e–xyΠ – e–xyΙ) + (1 + ϑ)e–xy = 0 or 4yΠ – 4yΙ + (1 + ϑ)y = 0 ...(1) Also, given y(0) = 0 and y(1) = 0 ...(2) Case I. Let ϑ = 0. Then (1) reduces to 4yΙΙ – 4yΙ + y = 0 i.e., (4D2 – 4D + 1)y = 0, where D = d/dx. ...(3) 2 Here auxiliary equation is 4D – 4D + 1 = 0 i.e., (2D – 1)2 = 0. This gives D = 1/2, 1/2. Hence solution of (3) is y(x) = (A + Bx)ex/2 ...(4) Using (2), (4) reduces to 0=A and 0 = (A + B)e1/2 These give A = B = 0. So (4) reduces to y(x) = 0, which is not an eigenfunction. Case II. Let ϑ = –Ο2 where Ο ∗ 0. Then (1) reduces to 4yΙΙ – 4yΙ + (1 – Ο2)y = 0 i.e., (4D2 – 4D + 1 – Ο2)y = 0. ...(5) 2 2 Here auxiliary equation is 4D – 4D + 1 – Ο = 0 giving x

D [4 Θ {16 –16(1 – µ2 )}1/ 2 ] 8 1/ 2 Θ µ / 2 Hence solution of (5) is y = Ae(1/2 + Ο/2)x + Be(1/2 – Ο/2)x ...(6) 1/2 + Ο/2 1/2 – Ο/2 Using (2), (6) reduces to 0 = A + B, 0 = Ae + Be These give A = B = 0. So (6) reduces to y(x) = 0, which is not an eigenfunction. Case III. Let ϑ = Ο2, where Ο ∗ 0. Then (1) reduces to 4yΙΙ – 4yΙ + (1 + Ο2)y = 0 i.e., (4D2 – 4D + 1 + Ο2)y = 0 ...(7) 2 Here auxiliary equation is 4D – 4D + 1 + Ο2 = 0

Created with Print2PDF. To remove this line, buy a license at: http://www.software602.com/

Orthogonal Sets of Functions and Strum Liouville Problem

giving

15.17

D [4 Θ {16 7 16(1 5 Ο 2 )}1/ 2 ]/ 8 1/ 2 Θ i(Ο / 2)

Hence solution of (7) is given by y(x) = ex/2{A cos (Οx/2) + B sin (Οx/2)} ...(8) Using (2), (8) reduces to 0=A and 0 = e1/2{cos (Ο/2) + B sin (Ο/2)} i.e., A = 0 and B sin (Ο/2) = 0 ...(9) Consider B sin (Ο/2) = 0 ...(10) If B = 0, then with A = 0, (8) reduces to y(x) = 0, which is not an eigen function. So we take B ∗ 0 for the existence of eigen function. As B ∗ 0, (10) gives sin (Ο/2) = 0 so that Ο/2 = n9 or Ο = 2n9, n = 1, 2, 3, ... ...(11) (n = 0 is not being considered here as n = 0 8 Ο = 0 which is contrary to our assumption.) Using A = 0 and (11), (8) reduces to y(x) = Bex/2 sin n9x and ϑ = Ο2 = 4n292, n = 1, 2, 3, .... Hence the required eigenfunctions yn(x) with the corresponding eigenvalues ϑn are given by (taking B = 1) yn(x) = ex/2 sin n9x, ϑn = 4n292, n = 1, 2, 3, ... Ex. 5. For the eigen–value problem given below, obtain the set of orthogonal eigenfunctions in the interval (0, 2c) : X ΙΙ + ϑX = 0, X(0) = X(2c), X Ι(0) = XΙ(2c). Sol. Given X ΙΙ + ϑX = 0 ...(1) with boundary conditions X(0) = X(2c) ...(2) and X Ι(0) = X Ι(2c) ...(3) Case I. Let ϑ = 0. Then solution of (1) is X(x) = Ax + B ...(4) From (4), XΙ(x) = A ...(5) From (4), X(0) = B and X(2c) = 2cA + B. So (2) reduces to B = 2cA + B and hence A = 0. Next, from (5), XΙ(0) = X Ι(2c) = A. So (3) gives A = A. Hence corresponding to the eigen value ϑ = 0, the eigenfunction is X(x) = B or X(x) = 1, taking B = 1. 2 2 Case II. Let ϑ = –Ο , where Ο ∗ 0. Then (1) becomes XΙΙ – Ο X = 0 whose solution is X(x) = AeΟx + Be–Οx ...(6) From (6), XΙ(x) = AΟeΟx – BΟe–Οx ...(7) 2Οc –2Οc From (6), X(0) = A + B and XΙ(2c) = Ae + Be . So (2) gives A + B = Ae2Οc + Be–2Οc or A(1 – e2Οc) + B(1 – e–2Οc) = 0 ...(8) From (7), XΙ(0) = Ο(A – B) and X Ι(2c) = Ο(Ae2Οc – Be–2Οc) 6 (3) gives Ο(A – B) = Ο(Ae2Οc – Be–2Οc) or A(1 – e2Οc) – B(1 – e–2Οc) = 0 ...(9) Solving (8) and (9), A = B = 0. So (6) reduces to X(x) = 0, which is not an eigen function. So there is no eigenfunction corresponding to ϑ = –Ο2. Case III. Let ϑ = Ο2, where Ο ∗ 0. Then (1) becomes XΙΙ + Ο2X = 0 whose solution is X(x) = A cos Οx + B sin Οx ...(10) From (10), XΙ(x) = –AΟ sin Οx + BΟ cos Οx ...(11) From (10), X(0) = A and X(2c) = A cos 2Οc + B sin 2Οc 6 (2) gives A = A cos 2Οc + B sin 2Οc or A(1 – cos 2Οc) – B sin 2Οc = 0 ...(12) From (11), XΙ(0) = BΟ and X Ι(2c) = Ο(–A sin 2Οc + B cos 2Οc) 6 (3) gives BΟ = Ο(–A sin 2Οc + B cos 2Οc) or A sin 2cΟ + B(1 – cos 2cΟ) = 0 ...(13) For non–trival solution of (12) and (13), we must have 1 7 cos 2Οc sin 2Οc

7 sin 2Οc 1 7 cos 2Οc

0

Created with Print2PDF. To remove this line, buy a license at: http://www.software602.com/

15.18

Orthogonal Sets of Functions and Strum Liouville Problem 2

2

(1 – cos 2Οc) + sin 2Οc = 0 or cos 2Οc = 1 = cos 0 6 2Οc = 2n9 or Ο = n9/c, n = 1, 2, 3, ... ...(14) (n = 0 is omitted here as n = 0 8 Ο = 0, which is contrary to our assumption Ο ∗ 0. ) With this value of Ο, (10) becomes X(x) = A cos (n9x/c) + B sin (n9x/c) ...(15) Taking A = 1 and B = 0 in (15), the eigenfunctions are given by X(x) = cos (n9x/c). Again taking A = 0 and B = 1 in (15), the eigenfunctions are given by X(x) = sin (n9x/c). or

Note that

#

2c

0

1 4 cos

n9x dx c

0,

#

2c

0

1 4 sin

n9x dx c

0,

#

2c

0

sin

n9x m9x cos dx c c

0, for m ∗ n ... (16)

In view of (16) the required eigenfunctions which are orthogonal on (0, 2c) are given by {1, cos (n9x/c), sin (n9x/c)}, (n = 1, 2, 3, ...) Ex. 6. Find the eigenvalues and eigenfunctions of [xyΙ(x)]Ι + (ϑ/x) y(x) = 0, yΙ(1) = yΙ(e29) = 0 [Kanpur 2009; Himanchal 2009] Sol. Re–writing the given equation, xyΙΙ + yΙ + (ϑ/x)y = 0 or x2yΙΙ + xyΙ + ϑy = 0 i.e., (x2D2 + xD + ϑ)y = 0, where D Ρ d/dx ...(1) This is a homogeneous differential equation. To solve it, we take x = ez so that z = log x ...(2) We know that, if D1 Ρ d/dz Ρ x(d/dx), then xD = D1 and x2D2 = D1(D1 – 1) ...(3) Using (3), (1) reduces to {D1(D1 – 1) + D1 + ϑ}y = 0 or (D12 + ϑ)y = 0 ...(4) Also, given that yΙ(1) = 0 and yΙ(e29) = 0, ...(5) 2 Case I. Let ϑ = 0. Then solution of (4) i.e., D1 y = 0 is y = Az + B or y = A log x + B, using (2) ...(6) From (6), yΙ(x) = A/x. ...(7) Using (5), (7) reduces to 0=A and 0 = A/e29 These give A = 0, while B may be taken as arbitrary. With these values and taking B = 1, (6) gives y(x) = 1 as the eigenfunction corresponding to eigenvalue ϑ = 0. Case II. Let ϑ = –Ο2, where Ο ∗ 0. Then solution of (4) is y = AeΟz + Be–Οz = A(ez)Ο + B(ez)–Ο or y(x) = AxΟ + Bx–ΟΝ as x = ez ...(8) Ο– 1 From (8), yΙ(x) = AΟx – BΟx–Ο – 1 ...(9) 29(Ο – 1) –29(Ο + 1) Using (5), (9) reduces to 0 = AΟ – BΟ and 0 = AΟe – BΟe i.e., A–B=0 and Ae29Ο – Be–29Ο = 0, as Ο ∗ 0, giving A = B = 0. So (8) reduces to y(x) = 0, which is not an eigenfunction. Case III. Let ϑ = Ο2, where Ο ∗ 0. Then solution of (4) is y = A cos Οz + B sin Οz, i.e., y(x) = A cos (Ο log x) + B sin (Ο log x), by (2) ...(10) From (10), yΙ(x) = –(AΟ/x) sin (Ο log x) + (BΟ/x) cos (Ο log x). ...(11) 29 Using (5), (11) reduces to (noting that log 1 = 0 and log e = 29) 0 = BΟ and 0 = –(AΟ/e29) sin 29Ο + (BΟ/e29) cos 29Ο, i.e., B=0 and A sin 29Ο = 0, as Ο ∗ 0. ...(12) Consider A sin 29Ο = 0 ...(13) If A = 0, then with B = 0, (10) reduces to y(x) = 0, which is not an eigenfunction. So we take A ∗ 0 for the existence of eigenfunctions. Since A ∗ 0, (13) gives sin 29Ο = 0 so that 29Ο = n9, n = 1, 2, 3, ... Thus, Ο = n/2, n = 1, 2, 3, .. ...(14) n = 0 is not being considered because n = 0 8 Ο = 0 8 ϑ = 0, which has already been considered in case I. Using B = 0 and (14), (10) reduces to y(x) = A cos {(n/2) × log x}, with ϑ = Ο2 = n2/4, n = 1, 2, 3, ....

Created with Print2PDF. To remove this line, buy a license at: http://www.software602.com/

Orthogonal Sets of Functions and Strum Liouville Problem

15.19

So the required eigenfunctions yn(x) (taking A=1) with the corresponding eigenvalues ϑn are yn(x) = cos {(n/2) × log x}, n = 1, 2, 3, ..., ϑn = n2/4, n = 1, 2, 3, ...; and y(x) = 1 with ϑ = 0. Ex. 7. Find all the eigenvalues and eigenfunctions of the Strum–Liouville problem (x3yΙ)Ι + ϑxy = 0, y(1) = 0, y(e) = 0. Sol. Re–writing the given equation x3yΙΙ + 3x2yΙ + ϑxy = 0 or x2yΙΙ + 3xyΙ + ϑy = 0 or (x2D2 + 3xD + ϑ)y = 0, D Ρ d/dx ...(1) This is a homogeneous differential equation. To solve it, we take x = ez so that z = log x ...(2) We know that, if D1 Ρ d/dz Ρ x(d/dx), then xD = D1 and x2D2 = D1(D1 – 1) ...(3) Using (3), (1) reduces to {D1(D1 – 1) + 3D1 + ϑ}y = 0 or (D12 + 2D1 + ϑ)y = 0 ...(4) Also, given that y(1) = 0 and y(e) = 0 ...(5) Case I. Let ϑ = 0. Then (4) reduces to D1(D1 + 2)y = 0 whose solution is y = Ae0.z + Be–2z or y(x) = A + B/x2 ...(6) Putting x = 1 in (6) and using (5), A+B=0 ...(7) Putting x = e in (6) and using (5), A + B/e2 = 0 ...(8) Solving (7) and (8), we get A = B = 0 and so (6) gives y(x) = 0, which is not an eigenfunction. Case II. Let ϑ = –Ο2, where µ ∗ 0. Then (4) reduces to (D12 + 2D1 – Ο2)y = 0 ...(9) whose auxiliary equation is D12 + 2D1 – Ο2 = 0 Solving it, D1 = {–2 ± (4 + 4Ο2)1/2}/2 = –1 ± (1 + Ο2)1/2 or D1 = –1 ± k, where k = (1 + Ο2)1/2 ...(10) (–1 + k)z (–1 – k)z Hence solution of (9) is y(x) = Ae + Be or y(x) = Ax(–1 + k) + Bx(–1 – k) or y(x) = x–1(Axk + B/xk) ...(11) Putting x = 1 in (11) and using (5), A+B=0 ...(12) Putting x = e in (11) and using (5), Aek – 1 + Be–k – 1 = 0 ...(13) Solving (12) and (13), we get A = B = 0. Hence (11) gives y(x) = 0, which is not an eigenfunction. Case III. Let ϑ = Ο2, where Ο ∗ 0. Then (4) reduces to (D12 + 2D1 + Ο2)y = 0 ...(14) 2 2 whose auxiliary equation is D1 + 2D1 + Ο = 0 ...(15) 2 1/2 2 1/2 Solving it, D1 = {–2 ± (4 – 4Ο ) }/2 = –1 ± i(Ο – 1) or D1 = –1 ± ip, where p = (Ο2 – 1)1/2 and Ο2 > 1 ...(16) –z Hence solution of (14) is y(x) = e (A cos pz + B sin pz) i.e., y(x) = (1/x) × {A cos (p log x) + B sin (p log x)} ...(17) Putting x = 1 in (17) and using (5), we get A = 0. Putting x = e and A = 0 in (17) and using (5), we get (B/e) sin (p) = 0. But we must take B ∗ 0 for non–trivial solution (and hence for getting eigenfunction). 6 sin p = 0 so that p = n9 or (Ο2 – 1)1/2 = n9, by (16) 2 2 2 2 2 2 or Ο –1=n9 or Ο =1+n9 or ϑ = 1 + n292 Hence the required eigenfunctions are yn(x) = (1/x) × sin (n9 log x) (taking B = 1) and the corresponding eigenvalues are ϑn = n292, where n = 1, 2, 3, ... Ex. 8. Find the eigenvalues and the corresponding eigenfunctions of the boundary value problem : yΙΙ + 2yΙ + (1 + ϑ)y = 0, y(0) = 0, yΙ(a) = 0 Sol. Given (D2 + 2D + 1 + ϑ)y = 0, D Ρ d/dx ...(1) with y(0) = 0, yΙ(a) = 0 ...(2) Case I. Let ϑ = 0. Then solution of (1) is y(x) = (A + Bx)e–x ...(3) –x –x From (3), yΙ(x) = Be – (A + Bx)e ...(4) Using (2), (3) and (4) reduce to 0=A and Be–a – (A + Ba)e–a = 0,

Created with Print2PDF. To remove this line, buy a license at: http://www.software602.com/

15.20

Orthogonal Sets of Functions and Strum Liouville Problem –a

A=0 and Be (1 – a) = 0. These give A = B = 0. So (3) reduces to y(x) = 0, which is not an eigenfunction. Case II. Let ϑ = –Ο2, where Ο ∗ 0. Then (1) reduces to [D2 + 2D + 1 – Ο2]y = 0 ...(5) whose auxiliary equation is D2 + 2D + (1 – Ο2) = 0. This gives D = [–2 ± {4 – 4(1 – Ο2)}1/2]/2 = – 1 ± µ Hence solution of (5) is y(x) = Ae(–1 + Ο)x + Be–(1 + Ο)x ...(6) so that yΙ(x) = A(–1 + Ο) e(–1 + Ο)x – B(1 + Ο)e–(1 + Ο)x ...(7) Using (2), (6) and (7) reduce to 0=A+B and 0 = A(–1 + Ο) e(–1 + Ο)a – B(1 + Ο)e–(1 + Ο)a, i.e., A+B=0 and A(Ο – 1)eΟa – B(Ο + 1)e–Οa = 0. These give A = B = 0. So (6) reduces to y(x) = 0, which is not an eigenfunction. Case III. Let ϑ = Ο2, where Ο ∗ 0. Then (1) reduces to (D2 + 2D + 1 + Ο2)y = 0 ...(8) 2 2 whose auxiliary equation is D + 2D + 1 + Ο = 0. This gives D = [–2 ± {4 – 4(1 + Ο2)}1/2]/2 = –1 ± iΟ. Hence solution of (8) is y(x) = e–x (A cos Οx + B sin Οx) ...(9) –x –x From (9), yΙ(x) = –e (A cos Οx + B sin Οx) + e (–AΟ sin Οx + BΟ cos Οx) ...(10) Using (2), (9) and (10) reduce to 0 = A, 0 = –e–a (A cos Οa + B sin Οa) + Οe–a (–A sin Οa + B cos Οa) 6 A=0 and B(Ο cos Οa – sin Οa) = 0 ...(11) Consider B(Ο cos Οa – sin Οa) = 0 ...(12) If B = 0, then with A = 0, (9) reduces to y(x) = 0, which is not an eigenfunction. So we take B ∗ 0 for the existence of eigenfunctions. Since B ∗ 0, (12) reduces to Ο cos Οa – sin Οa = 0 or tan Οa = Ο, ...(13) which is a trigonometrical equation in Ο. Let Οn (n = 1, 2, 3, ...) be positive roots of (13). With A = 0, (9) reduces to y(x) = Be–x sin Οx Hence the required eigenfunctions yn(x) with the corresponding eigenvalues ϑn are given by (taking B = 1) yn(x) = e–x sin Οn x and ϑn = Οn2, n = 1, 2, 3, ..., where Οn (n = 1, 2, 3, ...) are positive roots of (13). i.e.,

EXERCISE 15(B) 1. For the Strum–Liouville problem XΠ + ϑX = 0, X(0) = 0, X(9) = 0, obtain the eigenfunctions and the corresponding eigenvalues. [Nagpur 2005; Bilaspur 2004; Bhopal 2004; Meerut 2005, 11; Ravishaker 2004; Vikram 2004; Lucknow] Ans. Xn(x) = sin nx, ϑn = n2, n = 1, 2, 3, ... 2. Find all eigenvalues and eigenfunctions of the Strum–Liouville problem X ΙΙ + ϑX = 0, X(0) = 0, X Ι(9/2) = 0. [Jabalpur 2004] Ans. Xn(x) = sin (2n – 1)x, ϑn = (2n – 1)2, n = 1, 2, 3, ... 3. Find all eigenvalues and eigenfunctions of the problem XΙΙ + ϑX = 0, X Ι(–9)= 0, XΙ(9) = 0 Ans. Xn = cos[(n/2) × (9 + x)], n = 0, 1, 2, ..., ϑn = n2/4, n = 1, 2, 3, ... 4. Find the eigenvalues and the corresponding eigenfunctions of the boundary value problem yΙΙ + ϑy = 0, y(0) + 9 yΙ(0) = 0, y(9) = 0. Ans. yn(x) = sin Οnx – 9Οn cos Οnx, ϑn = Οn2, n = 1, 2, 3, ... where Οn are +ve roots of tan Ο9 = Ο9; y(x) = x – 9 is eigenfunction corresponding to eigenvalue ϑ = 0. 5. Find all the eigenvalues and eigenfunctions of Strum-Liouville problem :

yΙΙ 5 ϑ 2 y

0, y Ι(0)

yΙ(l )

0, 0 ! x ! l

[Nagpur 2005]

Created with Print2PDF. To remove this line, buy a license at: http://www.software602.com/

Orthogonal Sets of Functions and Strum Liouville Problem

15.21

Hint. Refer case III of Ex. 2 Art 15.13. Here take X = y, Ο ϑn

n9 / l and the corresponding eigenfunctions

yn

ϑ and L = l. Then eigenvalues

cos(n9x / l ), n 1, 2, 3,...

6. Find the eigenvalues and eigenfunctions of the Strum-Liouville problem ΙΙ y 5 ϑy 0, y Ι(0) 0, y Ι(9) 0. [CDVU 2004; M.D.U. Rohtak 2005] [Hint : This is a particular case of Ex. 2 of Art 15.13. Here X = y and L cos nx and eigenvalues ϑ n

Ans. Eigenfunctions: yn ( x )

9.

n2 , where n = 0, 1, 2, ....]

7. Find the eigenvalues and the corresponding eigenfunctions of the eigen value problem XΙΙ + ϑX = 0, X(0) = 0, X(1) – XΙ(1) = 0. Ans. Xn = sin Οnx, ϑn = Οn2, n = 1, 2, 3, ... where Οn are positive roots of tan Ο = Ο. Again X(x) = x is eigen function corresponding to the eigen value ϑ = 0. 8. Find the eigenvalues and eigenfunctions of the Strum Liouville problem yΙΙ + ϑy = 0, y(0) = y(9) = 0. Ans. ϑn = n2, yn = sin n9, n = 1, 2, ..., ... 9. Solve the Strum–Liouville problem (xX Ι)Ι + ϑ(1/x)X = 0, X Ι(1) = 0, X(b) = 0 (b > 1) and Ans. ϑn =

normalize the eigen functions.

∃ (2n 7 1)9 log x % (2n 7 1)9 , Xn = cos & ∋ , n = 1, 2, 3, ... 2 log b 2 log b ( )

10. Find the eigenvalues and eigenfunctions of y ΙΙ 7 4ϑy Ι 5 4ϑ 2 y 0, y Ι(0) 0, y (2) 5 2 y Ι(2) 0. [Purvanchal 2006] 15.14. Orthogonality of Legendre polynomials We know that Legendre’s differential equation is (1 – x2)yΙΙ – 2xyΙ + n(n + 1)y = 0 2 which can be re–written in the form [(1 – x )yΙ]Ι + ϑy = 0, where ϑ = n(n + 1) ...(1) and is therefore in the form of Strum–Liouville equation [r(x) yΙ]Ι + [q(x) + ϑ p(x)]y = 0 ...(2) Comparing (1) with (2), here r(x) = 1 – x2, q(x) = 0 and p(x) = 1. Since r(–1) = r(1) = 0, we need no boundary conditions to form a Strum–Liouville problem. Further, we know that Pn(x) for n = 0, 1, 2, 3, ... are solutions of (1) and so they are eigenfunctions. Again they have continuous derivatives and hence it follows that Legendre polynomials Pn(x), n = 0, 1, 2, ... are orthogonal on the interval –1 ! x ! 1 with respect to the weight function p(x) = 1,

#

i.e.,

1

71

Pm (x) Pn ( x) dx

0 , when

m∗n

Remark. For alternative proof, refer Art. 9.8 of chapter 9. 15.15. Orthogonality of Bessel functions The Bessel functions Jn(u) with fixed integer n Σ 0 satisfies Bessel’s equation (refer equation (1) of Art. 11.1 of chapter 11, by taking u as independent variable in place of x) u2(d2y/du2) + u(dy/du) + (u2 – n2)y = 0 6

u2

d 2 J n (u) du

2

and

d J n (u) 5 (u 2 7 n 2 ) J n (u) = 0 du

u = ϑx, where ϑ is a constant.

Let Then,

5u

d J n (u) d J n (ϑx) dx 4 = du dx du

d 2 J n (u) du 2

Χ d J n (u ) ∆ = d Ε Φ du Γ du Η

1 Ι J (ϑx) , using (2) ϑ n

...(1) ...(2) ...(3)

d Χ J nΙ (ϑx) ∆ , by (3) Ε Φ du Γ ϑ Η

Created with Print2PDF. To remove this line, buy a license at: http://www.software602.com/

15.22

Orthogonal Sets of Functions and Strum Liouville Problem

=

d Χ J nΙ (ϑ x) ∆ 4 dx dx ΕΓ ϑ ΦΗ du

1 J ΙΙ(ϑx) , by (2) n ϑ2

...(4)

Using (2), (3) and (4), (1) reduces to x2 J nΙΙ (ϑx) + x J nΙ (ϑx) + (ϑ2x2 – n2) Jn(ϑx) = 0 which can be re–written in the form

[x J nΙ (ϑx)]Ι + (–n2/x + ϑ2x) Jn(ϑx) = 0

...(5)

and is therefore in the form of Strum–Liouville equation for each fixed n [r(x) yΙ]Ι + [q(x) + ϑ p(x)]y = 0 ...(6) 2 2 Comparing (5) with (6), here r(x) = x, q(x) = –n /x and p(x) = x and the parameter is ϑ in place of ϑ. Since r(x) = 0 for x = 0, it follows from Art. 15.11 that solutions of (5) on an interval 0 ! x ! a satisfying the boundary condition Jn(ϑa) = 0, (n fixed) ...(7) form an orthogonal set with respect to the weight function p(x) = x. We know that Jn(u) has infinitely many real zeros, say, u = Κ1 < Κ2 < .... Hence (7) gives ϑa = Κi and so ϑ = ϑi = Κi/a, i = 1, 2, 3, ... ...(8) Further J nΙ (ϑx) is continuous at x = 0. Hence for each fixed non–negative integer n the sequence of Bessel functions of the first kind Jn(ϑ1x), Jn(ϑ2x), Jn(ϑ3x), ..., with ϑi as in (8), forms an orthogonal set on the interval 0 ! x ! a with respect to the weight function p(x) = x, that is

#

a

0

x J n (ϑi x) J n (ϑ j x) dx

0 , for i ∗ j.

Remark. From the above discussion, it follows that we obtain infinitely many orthogonal sets, each corresponding to one of the fixed values of n. 15.16. Orthogonality on an infinite interval (i) Orthogonality of Hermite Polynomial Hn(x). Hermite polynomials are orthogonal over ]–≅, ≅[ with respect to the weight function e 7 x

#



7≅

2

2

/2

e7 x H m ( x) H n ( x) dx

, that is, 0,

m ∗ n.

for

For proof, refer Art. 12.7 of chapter 12. (ii) Orthogonality of Laguerre’s polynomial Ln(x). Laguerre polynomials are orthogonal over [0, ≅[ with respect to the weight function e–x, that is,

#



0

7x

e Lm ( x) Ln ( x) dx

0,

for

m ∗ n.

For proof, refer Art. 13.6 of chapter 13. 15.17. Orthogonal expansion or genralized Fourier series Let {yn(x)}, n = 0, 1, 2, 3, ... be an orthogonal set of functions with respect to weight function p(x) on the interval a ! x ! b and f(x) a function that can be represented by a convergent series ≅

f(x) = C0 y0(x) + C1 y1(x) + C2 y2(x) + ... =

ΤC

n

yn ( x)

...(1)

n 0

This is called an orthogonal expansion or generalised Fourier series. Coefficients C0, C1, C2, C3, ... are called Fourier Constants. Since {yn(x)} is an orthogonal set with respect to weight function p(x), we get

Created with Print2PDF. To remove this line, buy a license at: http://www.software602.com/

Orthogonal Sets of Functions and Strum Liouville Problem

#

b

a

0, when m ∗ n.

p( x) yn ( x) ym (x) dx

#

|| yn(x) || = Norm of yn(x) =

Again,

15.23

b

a

...(2)

2

p( x) yn ( x) dx

...(3)

Multiply both sides of (1) by p(x) yn(x) (n fixed). Then integrating over a ! x ! b and assuming that term–by–term integration is permissible, we get

#

b

a

p(x) f ( x) yn (x) dx

C0

#

5 ... Cn 71

#

or

b

a

a

p( x) y0 ( x) yn ( x) dx 5 C1

a

p(x) yn 71(x) yn (x) 5 Cn

1 2 || yn ( x) ||

#

b

a

#

b

a

#

b

a

p( x) y1( x) yn ( x) dx

#

2

p(x) yn ( x) dx 5 Cn51

b

a

p( x) yn 51( x) yn (x) dx 5 ...

2

p( x) f ( x) yn ( x) dx

Cn =

or

b

#

b

Cn || yn (x) || , using (2) and (3)

p(x) f ( x) yn ( x) dx, n = 0, 1, 2, 3, ...

...(4)

We now discuss two particular cases of the generalised Fourier series. Particular Case I. Fourier–Legendre Series. We know that the set {Pn(x)}, n = 0, 1, 2, 3, ... of Legendre polynomials is an orthogonal set on the interval –1 ! x ! 1. Hence, we get ≅

f(x) =

ΤC

n Pn ( x)

–1 ! x ! 1

,

...(i)

n 0

Cn =

where

|| Pn(x) ||2 =

But,

1

1 2 || Pn ( x) ||

#

1

#

71

f ( x) Pn ( x) dx,

2

71

Pn ( x) dx

Α Β#

n = 0, 1, 2, ...

...(ii)

2 , by Art 9.8, chapter 9. 2n 5 1 1

f ( x) Pn ( x) dx Cn = n 5 1 ...(iii) 2 71 Expansion (i) of f(x) in a series of Legendre’s polynomials is known as Fourier–Legendre series. The constants Cn are given by (iii). Note : For altenative proof refer, Art. 9.15, of chapter 9. Particular Case II : Fourier–Bessel series. From Art. 15.15, we know that for a fixed n, the set of Bessel functions of first kind Jn(ϑ1x), Jn(ϑ2x), Jn(ϑ3x), ..., with ϑi given by ϑi = Κi/a, where i = 1, 2, 3, ..., (n fixed) ...(i) form an orthogonal set on the internal 0 ! x ! a with respect to the weight function p(x) = x.

So (ii) becomes



6

Τ C J (ϑ x)

f(x) =

i

n

...(ii)

i

i 1

where

But,

Ci =

1 2 || J n (ϑi x) ||

#

a

0

x f ( x) J n (ϑi x) dx , i = 1, 2, 3, ...

|| Jn(ϑix) ||2 =

#

a

0

2

x J n (ϑi x) dx

...(iii)

a2 2 J (aϑ ) , by Art. 11.10 2 n51 i

Created with Print2PDF. To remove this line, buy a license at: http://www.software602.com/

15.24

Orthogonal Sets of Functions and Strum Liouville Problem

6 (iii) becomes,

Ci =

2 2 2

a J n 51(aϑi )

#

a

0

x f ( x) J n (ϑ i x) dx , n = 1, 2, 3, ... ...(iv)

Expansion (ii) of f(x) in a series of Bessel functions is known as Fourier–Bessel series. The constants Ci are given by (iv). Note. For alternative proof refer, Art. 11.11, of chapter 11. Ex. 1. Obtain the formal expansion of the function f(x) = 9x – x2, 0 ! x ! 9, in the series of orthonormal characteristic functions {−n} of Strun–Liouville problem yΙΙ +ϑy = 0, y(0) = y(9) = 0. Sol. Proceed as usual to show that −n(x) = (2/9)1/2 sin nx. Then the orthogonal expansion or generalised Fourier series of f(x) is given by (refer Art. 15.17) ≅

Τ

f(x) =

Cn −n ( x) ,

#

Cn =

where

0

n 1

|| −n(x) || = 1,

noting that

=

#

Cn =

Now,

ΑΒ

1/ 2

2 9

9

0

p(x) = 1, 2

1/ 2

(9x 7 x ) (2 / 9)

Α

9

f (x) − n ( x) dx ,

...(1)

b = 9.

a = 0, sin nx dx

Β

9

. Χ 1 ∆ Χ1 ∆/ 2 1 0(9x 7 x ) 7 n cos nx 7 Α 9 7 2x Β Ε 7 2 sin nx Φ 5 (72) Ε 3 cos nx Φ 1 Γ n Η Γn Η 30 2

(By chain rule of integration by parts)

ΑΒ

2 = 9

1/ 2

Χ ∆ 4 Ε 23 Φ (1 7 cos n9) Γn Η

∃(2 / 9)1/ 2 > (4 / n 3 ), if n is odd & 0, if n is even (

Hence the required expansion of f(x) is given by ≅

Α9Β

9x – x2 = Τ 2 n 1

1/ 2

4

ΑΒ

4 4 2 (2n 7 1)3 9

1/ 2

i.e.,

sin (2n 7 1) x,

9x 7 x

2

8 9



sin (2 n 7 1) x 3 1 (2 n 7 1)

Τ n

,0!x!9

Ex. 2. Obtain the formal expansion of f(x) = log x, 1 ! x ! e29 in a seies of orthogonal eigen functions of Strum–Liouville problem [xyΙ]Ι + (l/x)y = 0, y(0) = y(e29) = 0. Ans. log x = 7 8 9

Sol. Do as in Ex. 1.



Τ (2n 17 1) n 1

2

+

:

cos 2n 7 1 log | x | 2

OBJECTIVE PROBLEMS ON CHAPTER 15 Write (a), (b), (c) or (d) whichever is correct 1. For the Strum-Liouville problem (1 5 x2 ) yΙΙ 5 2 xyΙ 5 ϑx2 y 0, the eigenvalues, ϑ , satisfy (a) ϑ Σ 0 (b) ϑ Υ 0

0 with yΙ(1)

0 and

yΙ(10)

(c) ϑ ∗ 0

(d) ϑ ! 0

[GATE 2003]

2. Let n be non-negative integer. The eigenvalues of the Strum-Liouville problem y ΙΙ 5 ϑy with boundary conditions y (0) (a) n

y (29), y Ι(0)

(b) n2 92

0

y Ι(29) are (c) n9

(d) n2

[GATE 2002]

Created with Print2PDF. To remove this line, buy a license at: http://www.software602.com/

Orthogonal Sets of Functions and Strum Liouville Problem

15.25

3.The eigenvalues of the Strum-Liouville problem y ΙΙ 5 ϑy are

(a) n2/4

(c) (2n 7 1)2 / 4

(b) (2n 7 1)2 92 / 4

0, 0 ! x ! 9, y (0) (d) n 2 9 2 / 4

4. The eigenvalues of the boundary value problem xΙΙ 5 ϑx (a) ϑ 5 tan ϑ9

satisfy

(c)

ϑ 5 tan ϑ9

ϑ 5 tan ϑ9

0

(d) ϑ 5 tan ϑ9

0.

(b)

0 0

0, x (0)

0, yΙ(9)

0

[GATE 2001]

0, x (9) 5 xΙ(9)

0

[GATE 2000]

ANSWERS 1. (a)

2. (d)

3. (c)

4. (b)

MISCELLANEOUS PROBLEM ON CHAPTER 15 1. Show that the set {1, cos (2n9;T)x, n = 1, 2, 3, ... in orthogonal set of functions on an interval 0 ! x ! T . [Lucknow 2010] 2. Find eigenfunctions of the system uΙΙ + ϑu = 0, – 9 ! x ! 9 with the boundary condition u (– 9) = u (9), uΙ (– 9) = uΙ (9). Ans. {1, cos nx, sin nx}, n = 1, 2, 3 .... [Nagpur 2010] 3. Prove that the set of eigenfunctions of strum-Liouville problem y(0) = 0, y(1) = 0, q(x) > 0, form a set of orthogonal functions 4. Define norm of a function. Hints. Refer Art. 15.1 and 15.4.

d dx

dy % ∃ & p( x ) ∋ 5 ϑ q ( x ) 0, dx ) ( [Himanchal 2008] [Meerut 2011]

Created with Print2PDF. To remove this line, buy a license at: http://www.software602.com/

MISCELLANEOUS PROBLEMS BASED ON THIS PART OF THE BOOK Ex. 1. The value of Pn (– 1) is (a) 1 (b) 0 (c) – 1 Sol. Ans. (d). Refer part (ii) of Ex. 2, page 9.6 Ex. 2. The value of P0 (x) is (a) 0 (b) ! (c) 1 Sol. Ans. (c). Refer Art. 9.2 Ex. 3. Rodrigue’s formula for Pn(x) is (a) Pn ( x )

dn

1 n

n

dn

1

( x 2 # 1)n (b) Pn ( x )

n

n

(d) (–1)n

[Agra 2008]

(d) None of these

[Agra 2007]

( x 2 # 1)# n (c) Pn ( x)

2 n! dx 2 n ! dx (d) None of these Sol. Ans. (a). Refer Art-9.14 Ex. 4. Statement for the following linked questions 14 (i) and 14 (ii). Let n % 3 be an integer. Let y be the polynomial solution of

(1 – x2) y&& – 2xy& + n(n – 1) y = 0 4. (i): Then the degree of y is (a) n (b) n – 1 4. (ii): If I =



1

#1

(a) I ( 0 , J ( 0

y ( x ) x n #3 dx

(b) I ( 0, J

J=

0 (c) I



1

#1

n

( x2 ∃1)n

y(1) = 1.

(d) greater than n + 1

y ( x) xn dx, then

0, J ( 0 (d) I = 0, J = 0

Sol: 4. (i): Ans. (a). Refer Ex. 2 (i), page 9.6 4. (ii): Ans. (c). Refer Ex. 5, page 9.29 5. Using Rodrigue’s formula, find value of P4 (x) at x = 1. [Hint. Do as in Ex. 1, page 9.37 to get P4 ( x)

n

2 n! dx [Agra 2007]

satisfying

(c) less than n – 1 and

1 dn

[GATE 2008]

[Kanpur 2008]

(35 x 4 # 30 x 2 ∃ 3) / 8 and so the value of

P4 ( x ) at x 1 is (35 – 30 + 3)/8, i.e. 1. 6. Show that the condition of integrability of (i) Pdx + Qdy + Rdz = 0 implies the orthoganality of any pair of intersecting curves of the families (ii) (dx)/P = (dy)/Q = (dz)/R and (iii)

dx )Q / )z # )R / )y

dy )R / )x # )P / )z

dz . Hence show that the curves of (iii) all lie )P / )y # )Q / )x

on the surfaces of (i). Verify this conclusion for P = ny – mz, Q = lz – nx, R = mx – ly.

[I.A.S. 2001]

7. If f 1, f 2, f 3 are homogeneous functions of the same degree in x, y and z and if xf1 + yf2 + zf3 = 0, then show that the equation f1dx + f2dy + f3dz = 0 is integrable. [Himanchal 2000; Kerla 2001, 07, Pune 2010] 8. If the equation x1 dx1 + x2 dx2 + ..... + xn dxn = 0 has integrating factor, show that it has infinitely many. If n = 2, prove that the equation always has an integrating factor. [Himanchal 2002; I.A.S. 2002; Calicut 2004, 05; Osmania 2000, 06; U.P.(P.C.S.) 2003] 9. xdx + ydy + zdz = 0 is the first order differential equation of (a) sphere (b) ellipsoid (c) right circular cone (d) hyperboloid. [I.A.S. (Prel) 2001] M.1

Created with Print2PDF. To remove this line, buy a license at: http://www.software602.com/

M.2

Miscellaneous problems based on this part of the book

Sol. Ans. (a). Integrating the given differential equation, x2/2 + y2/2 + z2/2 = a2/2, where a /2 is an arbitrary constant of integration. Thus, we get x2 + y2 + z2 = a2, which is a sphere. 2

1 ∗(a )

10. Show that 1 F1 (a ; b; z )



!

0

e #t t a #1

0 F1 ( #;

b; z t ) dt

[Kanpur 2009]

Sol. Using Art. 14.3 for the value of 0F1 (–; b; zt), we get R.H.S. =

=

1 ! # t a #1 + ! ( zt )r , e t −1 . dt ∗ (a ) ∋0 / r 0 (b)r r !0

! #t a ∃ r # 1 1 ! zr e t dt 1 ∋ 0 ∗ (a) r 0 (b)r r !

1 ∗ (a)

!

zr

1 (b)

r 0

r r!

∗ (a ∃ r )

[Using definition of Gamma function 6.2 page 6.1] !

(a )r z r = 1 r 0 (b ) r r !

F (a; b; z), using art-14.3

11. Apply the method of Frobenius to the equation xy” + 2y& + xy = 0 to derive its general solution y = c0 {(cos x)/x} + c1 {(sin x)/x} [Delhi B.Sc. (Hons.) II 2011, Nagpur 1996] Sol. Given xy&& + 2y& + xy = 0 ... (1) !

Let the series solution of (1) be

y=

1 cm xkem, where c0 ( 0

!

k ∃m #1 y& = 1 (k ∃ m)cm x and y&& =

From (2),

... (2)

m 0

m 0

!

1 cm (k ∃ m)(k ∃ m # 1) xk ∃m#2

... (3)

m 0

Substituting the values of y, y& and y&& given by (2) and (3) in (1), we get !

!

!

m 0

m 0

m 0

!

!

!

m 0

m 0

m 0

x 1 cm (k ∃ m)(k ∃ m # 1) x k ∃ m #2 ∃ 2 1 cm (k ∃ m) x k ∃ m#1 ∃ x 1 cm x k ∃m

or

or

or

1 cm (k ∃ m)(k ∃ m # 1) xk ∃ m #1 ∃ 2 1 cm (k ∃ m) xk ∃ m #1 ∃ 1 cm x k ∃ m∃1 !

!

m 0

m 0

1 cm (k ∃ m){(k ∃ m # 1) ∃ 2}xk ∃ m #1 ∃ 1 cm xk ∃ m ∃1 !

!

m 0

m 0

1 cm (k ∃ m) (k ∃ m ∃ 1) xk ∃m#1 ∃ 1 cm xk ∃m∃1

0

0

0

0

... (4)

Equating to zero the hcoefficient of the smallest power of x, namely xk–1, the above identity (4) yields the indicial equation c0 k (k + 1) = 0

or

k (k + 1) = 0,

as c0 ( 0

so that

which are uequal and differ by an integer. Next, equations the coefficient of x recurrence relation. cm ( k ∃ m) (k ∃ m ∃ 1) ∃ cm# 2

0

so that cm

k = 0 or k = – 1y,, k + m–1

#{1/ (k ∃ m)(k ∃ m ∃ 1)}cm #2

, we arrive at the ... (5)

Created with Print2PDF. To remove this line, buy a license at: http://www.software602.com/

Miscellaneous problems based on this part of the book

M.3

k

Finally, we equate to zero the coefficient of x in the identity (4) and get c1 (k ∃ 1) (k ∃ 2)

0.

... (6)

If we take k = –1, (6) shows that c1 is ideterminate. With k = –1 and using (5), we now proceed to express c2, c4, c6 in terms of c0 and c3, c5, c7, ... in terms of c1 if c1 is assume to be finite. Setting k = – 1 in (5), we have c m = –{1/m (m – 1)} cm–2 ... (7) Putting m = 3, 5, 7, ... in (7) by turn, we obtain 1 1 c c3 c1 # 1 , c5 = # 5 2 4 32 2 3! Putting m = 2, 4, 6, ... in (7) by turn, we obtain

c3 = #

#

1 c1 12 2 2 3 2 4 2 5

c1 and so on 5!

c c0 1 1 1 c0 # 0 , c4 = c2 c0 and so on 2 21 2! 423 12 2 2 32 4 4! With k = –1 and substituting the above values of c2, c3, c4, c5, ... in (1) the required solution

c2 = #

is or or

y = x–1 (c0 + c1x + c2x2 + c3x3 + ...) = x–1 (c0 + c2x2 + c4x4 + ...) + x–1 (c1x + c3x3 + c5x5 + ...) y = c0x–1(1 – x2/2! + x4/4! ...) + c1x–1 (x – x3/3! + x5/5! – ...) y = c0 {(cos x)/x} + c1 {(sin x)/x}, [ cos x = 1– x2/2! + x4/4! – ..., sin x = x – x3/3! + x5/5 – ....] !

Ex. 12. If y

x2 y&& # xy& # 3(1 ∃ x 2 ) y (a) 1 and 3

1 cm xr ∃ m

m 0

is assumed to be a solution of the differential equation

0 , then the value of r are (b) –1 and 3

(c) 1 and – 3

(d) –1 and –3

Hint. Ans. (b). As usual find indicial equation r # 2r # 3 0 giving r 2

[GATE 2012]

#1,3

Created with Print2PDF. To remove this line, buy a license at: http://www.software602.com/

M.4

Miscellaneous problems based on this part of the book

Created with Print2PDF. To remove this line, buy a license at: http://www.software602.com/

PARTIAL DIFFERENTIAL EQUATIONS WHERE

IS

WHAT

CONTENTS 1.

Origin of partial differential equations

1.3 – 1.20

2.

Linear partial differential equations of order one

2.1 – 2.40

3.

Non-linear partial differential equations of order one

3.1 – 3.84

4.

Homogeneous linear partial differential equations with constant coefficients

4.1 – 4.34

Non-homogeneous partial differential equations with constant coefficients

5.1 – 5.30

Partial differential equations reducible to equations with constant coefficients

6.1 – 6.11

Partial differential equations of order two with variable coefficients

7.1 – 7.14

Classification of partial differential equations Reduction to canonical (or normal) form. Riemann method

8.1 – 8.50

Mong’s methods

9.1 – 9.50

5. 6. 7. 8.

9. 10.

Transport equations

10.1 – 10.5

Created with Print2PDF. To remove this line, buy a license at: http://www.software602.com/

1 Origin of Partial Differential Equations 1.1 INTRODUCTION Partial differential equations arise in geometry, physics and applied mathematics when the number of independent variables in the problem under consideration is two or more. Under such a situation, any dependent variable will be a function of more than one variable and hence it possesses not ordinary derivatives with respect to a single variable but partial derivatives with respect to several independent variables. In the present part of the book, we propose to study various methods to solve partial differential equations. 1.2 PARTIAL DIFFERENTIAL EQUATION (P.D.E.) [Delhi Maths (H) 2001] Definition. An equation containing one or more partial derivatives of an unknown frunction of two or more independent variables is known as a partial differential equation. For examples of partial differential equations we list the following: !z / !x !z / !y # z xy

... (1)

(!z / !x) 2 ! 3 z / !y3 # 2 x(!z / !x)

... (2)

z (!z / !x ) !z / !y # x

... (3)

!u / !x !u / !y !u / !z # xyz

... (4)

! 2 z / !x2 # (1 !z / !y)1/ 2

... (5)

y (!z / !x) 2



%

(!z / !y ) 2 # z (!z / !y )

... (6)

1.3 ORDER OF A PARTIAL DIFFERENTIAL EQUATION [Delhi Maths (H) 2001] Definition. The order of a partial differential equation is defined as the order of the highest partial derivative occuring in the partial differential equation. In Art. 1.2, equations (1), (3), (4) and (6) are of the first order, (5) is of the second order and (2) is of the third order. 1.4 DEGREE OF A PARTIAL DIFFERENTIAL EQUATION [Delhi Maths (H) 2001] The degree of a partial differential equation is the degree of the highest order derivative which occurs in it after the equation has been rationalised, i.e., made free from radicals and fractions so far as derivatives are concerned. In 1.2, equations (1), (2), (3) and (4) are of first degree while equations (5) and (6) are of second degree. 1.5 LINEAR AND NON-LINEAR PARTIAL DIFFERENTIAL EQUATIONS Definitions. A partial differential equation is said to be linear if the dependent variable and its partial derivatives occur only in the first degree and are not multiplied. A partial differential equation which is not linear is called a non-linear partial differential equation. In Art. 1.2, equations (1) and (4) are linear while equations (2), (3), (5) and (6) are nonlinear. 1.6 NOTATIONS When we consider the case of two independent variables we usually assume them to be x and y and assume z to be the dependent variable. We adopt the following notations throughout the study of partial differential equations

p # !z / !x,

q # !z / !y,

r # ! 2 z / !x2 ,

s # ! 2 z / !x!y

and

t # ! 2 z / !y 2

1.3

Created with Print2PDF. To remove this line, buy a license at: http://www.software602.com/

1.4

Origin of partial differential equations

In case there are n independent variables, we take them to be x1, x2....., xn and z is then regarded as the dependent variable. In this case we use the following notations : p1 ! !z/!x1,

p2 ! !z/!x2,

p3 # !z / !x3 ,

and

pn ! !z/!xn.

Sometimes the partial differntiations are also denoted by making use of suffixes. Thus we write

u x # !u / !x,

u y # !u / !y,

u xx # ! 2 u / !x 2 ,

u xy # ! 2 u / !x!y

and so on.

1.7 Classification of first order partial differential equations into linear, semi-linear, quasi-linear and non-linear equations with examples. [Delhi Maths (H) 2001; 2004] Linear equation. A first order equation f (x, y, z, p, q) = 0 is known as linear if it is linear in p, q and z, that is, if given equation is of the form P(x, y) p + Q(x, y) q ! R(x, y) z + S(x, y). For examples, yx2p + xy2q ! xyz + x2y3 and p + q ! z + xy are both first order linear partial differential equations. Semi-linear equation. A first order partial differential equation f (x, y, z, p, q) ! 0 is known as a semi-linear equation, if it is linear in p and q and the coefficients of p and q are functions of x and y only i.e. if the given equation is of the form P(x, y) p + Q(x, y) q ! R(x, y, z) For examples, xyp + x2yq ! x2y2z2 and yp + xq ! (x2z2/y2) are both first order semi-linear partial differential equations. Quasi-linear equation. A first order partial differential equation f(x, y, z, p, q) = 0 is known as quasi-linear equation, if it is linear in p and q, i.e., if the given equation is of the form P(x, y, z) p + Q(x, y, z) q ! R(x, y, z) For examples, x2zp + y2zp ! xy and (x2 – yz) p + (y2 – zx) q ! z2 – xy are first order quasi-linear partial differential equations. Non-linear equation. A first order partial differential equation f(x, y, z, p, q) ! 0 which does not come under the above three types, in known as a non-liner equation. For examples, p2 + q2 ! 1, pq!z and x2 p2 + y2 q2 ! z2 are all non-linear partial differential equations. 1.8 Origin of partial differential equations. We shall now examine the interesting question of how partial differential equations arise. We show that such equations can be formed by the elimination of arbitrary constants or arbitrary functions. 1.9 Rule I. Derivation of a partial differential equation by the elimination of arbitrary constants. Consider an equation F(x, y, z, a, b) ! 0, ...(1) where a and b denote arbitrary constants. Let z be regarded as function of two independent variables x and y. Differentiating (1) with respect to x and y partially in turn, we get !F / !x

p(!F / !z ) # 0

and

!F / !y q(!F / !z ) # 0

..(2)

Eliminating two constants a and b from three equations of (1) and (2), we shall obtain an equation of the form f(x, y, z, p, q) ! 0, ...(3) which is partial differential equation of the first order. In a similar manner it can be shown that if there are more arbitrary constants than the number of independent variables, the above procedure of elimination will give rise to partial differential equations of higher order than the first. Working rule for solving problems: For the given relation F(x, y, z, a, b) ! 0 involving variables x, y, z and arbitrary constants a, b, the relation is differentiated partially with respect to independent variables x and y. Finally arbitrary constants a and b are eliminated from the relations

Created with Print2PDF. To remove this line, buy a license at: http://www.software602.com/

Origin of partial differential equations

1.5

F(x, y, z, a, b) ! 0, and !F / !y # 0. !F / !x # 0 The equation free from a and b will be the required partial differential equation. Three situations may arise : Situation I. When the number of arbitrary constants is less than the number of independent variables, then the elimination of arbitrary constants usually gives rise to more than one partial differential equation of order one. For example, consider z ! ax + y, ... (1) where a is the only arbitrary constant and x, y are two independent variables. Differentiating (1) partially w.r.t. ‘x’, we get

!z / !x # a

... (2)

Differentiating (1) partially w.r.t. ‘y’, we get

!z / !y # 1

... (3)

z # x(!z / !x ) y

... (4)

Eliminating a between (1) and (2) yields

Since (3) does not contain arbitrary constant, so (3) is also partial differential under consideration. Thus, we get two partial differential equations (3) and (4). Situation II. When the number of arbitrary constants is equal to the number of independent variables, then the elimination of arbitrary constants shall give rise to a unique partial differential equation of order one. Example: Eliminate a and b from az + b ! a2x + y ... (1) Differentiating (1) partially w.r.t ‘x’ and ‘y’, we have

a(!z / !x) # a 2

a(!z / !y ) # 1

... (2)

... (3)

(!z / !x) (!z / !y ) # 1,

Eliminating a from (2) and (3), we have

which is the unique partial differential equation of order one. Situation III. When the number of arbitrary constants is greater than the number of independent variables, then the elimination of arbitrary constants leads to a partial differential equation of order usually greater than one. Example: Eliminate a, b and c from z = ax + by + cxy ... (1) Differentiating (1) partially w.r.t., ‘x’ and ‘y’, we have !z / !x # a c y

! 2 z / !y 2 # 0

! 2 z / !x 2 # 0,

From (2) and (3),

! 2 z / !x!y # c

and Now, (2) and (3)

∋ or

!z / !y # b c x

... (2)

&

x(!z / !x ) # ax cxy x(!z / !x )

and

... (3) ... (4) ... (5)

y (!z / !y ) # by cxy

y (!z / !y ) # ax by cxy cxy

x(!z / !x) y(!z / !y) # z xy(! 2 z / !x!y ), using (1) and (5)

... (6)

Thus, we get three partial differential equations given by (4) and (6), which are all of order two. 1.10 SOLVED EXAMPLES BASED ON RULE I OF ART 1.9 Ex. 1. Find a partial differential equation by eliminating a and b from z ! ax + by + a2 + b2. Sol. Given z ! ax + by + a2 + b2. ...(1) Differentiating (1) partially with respect to x and y, we get !z/!x ! a and !z/!y ! b.

Created with Print2PDF. To remove this line, buy a license at: http://www.software602.com/

1.6

Origin of partial differential equations

Substituting these values of a and b in (1) we see that the arbitrary constants a and b are eliminated and we obtain, z ! x(!z/!x) + y(!z/!y) + (!z/!x)2 + (!z/!y)2, which is the required partial differential equation. Ex. 2. Eliminate arbitrary constants a and b from z ! (x ( a)2 + (y ( b)2 to form the partial differential equation. [Jiwaji 1999; Banglore 1995] Sol. Given z ! (x ( a)2 + (y ( b)2. ...(1) Differentiating (1) partially with respect to a and b, we get !z/!x ! 2(x – a) and !z/!y ! 2(y – b). Squatring and adding these equations, we have (!z/!x)2 + (!z/!y)2 ! 4(x ( a)2 + 4(y ( b)2 ! 4 [(x ( a)2 + (y ( b)2] or (!z/!x)2 + (dz/!y)2 ! 4z, using (1). Ex. 3. Form partial differential equations by eliminating arbitrary constants a and b from the following relations : (a) z ! a(x + y) + b. (b) z ! ax + by + ab. [Bhopal 2010, Rewa 1996] 2 2 (c) z ! ax + a y + b. [Agra 2010] (d) z ! (x + a) (y + b). [Madurai Kamraj 2008] Sol. (a) Given z ! a(x + y) + b ...(1) Differentiating (1) partially with respect to x and y, we get !z/!x ! a and !z/!y ! a. Eliminating a between these, we get !z/!x ! !z/!y, which is the required partial differential equation. (b) Given z ! ax + by + ab. ...(1) Differentiating (1) partially with respect to x and y, we get !z/!x ! a and !z/!y ! b ....(2) Substituting the values of a and b from (2) in (1), we get z ! x(!z/!x) + y(!z/!y) + (!z/dx)(!z/!y), which is the required partial differential equation. (c) Try yourself. Ans. !z/!y ! 2y(!z/!x)2. (d) Try yourself. Ans. z ! (!z/!y) (!z/!x). y 2 2y Ex. 4. Eliminate a and b from z ! axe + (1/2) × a e + b. [Meerut 2006] Sol. Given z ! axey + (1/2) × a2ey + b. ...(1) Differentiating (1) partially with respect to x and y, we get !z/!x ! aey ...(2) y 2 2y and !z/!y ! axe + a e ! x(aey) + (aey)2. ...(3) y Substituting the value of ae from (2) in (3), we get !z/!y ! x(!z/!x) + (!z/!x)2. Ex. 5(a). Form the partial differential equation by eliminating h and k from the equation (x ( h)2 + (y ( k)2 + z2 ! )2. [Gulbarga 2005; I.A.S. 1996] Sol. Given (x ( h)2 + (y ( k)2 + z2 ! )2. ...(1) Differentiating (1) partially with respect to x and y, we get 2(x – h) + 2z(!z/!x) ! 0 or (x – h) ! –z(!z/!x) ...(2) and 2(y – k) + 2z(!z/!y) ! 0 or (y – k) ! –z(!z/!y). ...(3) Substituting the values of (x ( h) and (y ( k) from (2) and (3) in (1) gives z2(!z/!x)2 + z2(!z/!y)2 + z2 ! )2 or z2[(!z/!x)2 + (!z/!y)2 + 1] ! )2, which is the required partial differential equation.

Created with Print2PDF. To remove this line, buy a license at: http://www.software602.com/

Origin of partial differential equations

1.7

Ex. 5(b). Find the differential equation of all spheres of radius ), having centre in the xyplane. [M.D.U. Rohtak 2005; I.A.S. 1996, K.U. Kurukshetra 2005] Sol. From the coordinate geometry of three-dimensions, the equation of any sphere of radius ), having centre (h, k, 0) in the xy-plane is given by (x – h)2 + ( y – k)2 + (z – 0)2 ! )2 or (x – h)2 + (y – k)2 + z2 ! )2, ...(1) where h and k are arbitray constants. Now, proceed exactly in the same way as in Ex. 5(a). Ex. 6. Form the differential equation by eliminating a and b from z ! (x2 + a) (y2 + b). [Madras 2005; Sagar 1997, I.A.S. 1997] 2 Sol. Given z ! (x + a) (y2 + b). ...(1) Differentiating (1) partially with respect to x and y, we get !z/!x ! 2x(y2 + b) or (y2 + b) ! (1/2x) × (!z/!x) ...(2) 2 2 and !z/!y ! 2y(x + a) or (x + a) ! (1/2y) × (!z/!y). ...(3) Substituting the values of (y2 + b) and (x2 + a) from (2) and (3) in (1) gives z ! (1/2y) × (!z/!y) × (1/2x) × (!z/!x) or 4xyz ! (!z/!x)(!z/!y), which the required partial differential equation. Ex. 7. Form differential equation by eliminating constants A and p from z ! A ept sin px. Sol. Given z ! A ept sin px. ...(1) Differentiating (1) partially with respect to x and t, we get !z/!x ! Ap ept cos px ...(2) !z/!t ! Ap ept sin px. ...(3) Differentiating (2) and (3) partially with respect to x and t respectively gives !2z/!x2 ! ( Ap2ept sin px. ...(4) !2z/!t2 ! Ap2ept sin px. ...(5) 2 2 2 2 Adding (4) and (5), ! z/!x + ! z/!t ! 0, which is the required partial differential equation. Ex. 8. Find the differential equation of the set of all right circular cones whose axes coincide with z-axix. [I.A.S. 1998] Sol. The general equation of the set of all right circular cones whose axes coincide with zaxis, having semi-vertical angle ∗ and vertex at (0, 0, c) is given by

x2

y 2 # ( z ( c) 2 tan 2 ∗,

... (1)

in which both the constants c and ∗ are arbitrary. Differentiating (1) partially, w.r.t. x and y, we get

2 x # 2( z ( c) (!z / !x) tan 2 ∗

and

2 y # 2( z ( c) (!z / !y ) tan 2 ∗

& y( z ( c) (!z / !x) tan 2 ∗ # xy

and

x( z ( c) (!z / !y) tan 2 ∗ # xy

&

y( z ( c) (!z / !x) tan 2 ∗ # x( z ( c) (!z / !y) tan 2 ∗

Thus, y (!z / !x) # x (!z / !y ), which is the required partial differential equation. Ex. 9. Show that the differential equation of all cones which have their vertex at the origin is px + qy ! z. Verify that yz + zx + xy ! 0 is a surface satisfying the above equation. [I.A.S. 1979, 2009] Sol. The equation of any cone with vertex at origin is ax2 + by2 + cz2 + 2fyz + 2gzx + 2hxy ! 0, ...(1) where a, b, c, f, g, h are parameters. Differentiating (1) partially w.r.t. ‘x’ and ‘y’ by turn, we have (noting that p ! !z/!x and q ! !z/!y) 2ax + 2czp + 2fyp + 2g(px + z) + 2hy ! 0 or ax + gz + hy + p(cz + gx + fy) ! 0 ...(2)

Created with Print2PDF. To remove this line, buy a license at: http://www.software602.com/

1.8

Origin of partial differential equations

and 2by + 2czq + 2f(yq + z) + 2gxq + 2hx ! 0 or by + fz + hx + q(cz + fy + gx) ! 0. ...(3) Multiplying (2) by x and (3) by y and adding, we have (ax2 + by2 + gzx + fyz + 2hxy) + (cz + fy + gx) (px + qy) ! 0. – (cz2 + fyz + gxz) + (cz + fy + gx) (px + qy) ! 0, using (1) or (cz + fy + gx) (px + qy – z) ! 0 or px + qy – z ! 0, ...(4) which is required partial differential equation. Second Part : Given surface is yz + zx + xy ! 0 ...(5) Differentiating (5) partially w.r.t. ‘x’ and ‘y’ by turn, we get yp + px + z + y ! 0 and z + qy + xq + x ! 0. ...(6) Solving (6) for p and q, p ! ( (z + y)/(x + y) and q ! ( (z + x)/(x + y). 2 ( xy yz zx ) x ( z y) y ( z x ) (z!( ! 0, using (5) ( x y x y x y Hence (5) is a surface satisfying (4). Ex. 10. Form partial differential equations by eliminating arbitrary constants a and b from the following relations: (a) 2z ! x2/a2 + y2/b2 [Nagpur 1995; M.D.U. Rohtak 2006] 2 (b) 2z ! (ax + y) + b [Nagpur 1996; Delhi Maths (G) 2006; Pune 2010] Sol. (a) Given 2z ! x2/a2 + y2/b2 ... (1) Differentiating (1) partially w.r.t. ‘x’ and ‘y’, we get



px + qy ( z ! (

2 + !z / !y , # 2 y / b2

... (3)

From (2) and (3), p ! x/a2, q ! y/b2 a2 ! x/p, & 2 2 Substituting these values of a and b in (1), we get 2z ! px + qy, which is the required partial differential equation (b) Given 2z ! (ax + y)2 + b Differentiating (1) partially w.r.t. ‘x’ and ‘y’, we get 2p ! 2a(ax + y) ... (2) 2q ! 2(ax + y)

b2 ! y/q

2(!z / !x) # 2 x / a 2

... (2)

... (1) ... (3)

where p # !z / !x and q # !z / !y. Dividing (2) by (3) yields p/q ! a. px + qy ! q2. [I.A.S. 1998] ... (1)

Substituting this value of a in (3), we get q ! (p/q) x + y or Ex. 11. Eliminate a, b and c from z = a(x + y) + b(x – y) + abt + c Sol. Given z = a(x + y) + b(x – y) + abt + c Differentiating (1) partially w.r.t. ‘x’, ‘y’ and ‘t’, we get !z / !x # a b ... (2) We have the identity:



!z / !y # a ( b ... (3) 2

!z / !t # ab ... (4) 2

(a + b) – (a – b) ! 4ab

(!z / !x) 2 ( (!z / !y) 2 # 4(!z / !t ), using (2), (3) and (4)

Ex. 12. Form the partial differential equation by eliminating the arbitrary constants a and b from log (az – 1) ! x + ay + b. [I.A.S. 2002] Sol. (a) Given log (az – 1) ! x + ay + b ... (1) Differentiating (1) partially w.r.t. ‘x’, we get

a !z #1 az ( 1 !x

... (2)

Differentiating (1) partially w.r.t. ‘y’, we get

a !z #a az ( 1 !y

... (3)

Created with Print2PDF. To remove this line, buy a license at: http://www.software602.com/

Origin of partial differential equations

!z !y

az ( 1 #

From (3),

1.9

a#

so that

1 (!z / !y ) z

... (4)

Putting the above values of az – 1 and a in (2), we have 1 (!z / !y ) !z #1 z (!z / !y ) !x

− !z . !z !z #z . /1 0 ! y ! x ! y 1 2 2 2 2 2 Ex. 13. Find a partial differential equation by eliminating a, b, c, from x /a + y /b + z2/c2!1. [Bhopal 2004; Jabalpur 2000, 03, Jiwaji 2000, Vikram 2002, 04; Ravishanker 2010] Sol. Given x2/a2 + y2/b2 + z2/c2 ! 1. ... (1) Differentiating (1) partially with respect to x and y, we get 2x

2 z dz #0 c 2 dx

a2

and

or

c2 x a2 z

or

dz #0 dx

!z 2 x !z or c 2 y b2 z # 0. #0 2 ! y b c !y Differentiating (2) with respect to x and (3) with respect to y, we have 2y 2

c2

− !z . a2 / 0 1 !x 2

2

a2 z

!2 z !x 2

#0

− !z . b2 / 0 1 !x 2

c2

... ( 4)

2

b2 z

!2 z !y 2

# 0.

c 2 # ( (a 2 z / x) 3 (!z / !x) Putting this value of c in (4) and dividing by a2, we obtain From (2),

... (2) ... (3)

... (5) ... (6)

2

(

or

z !z − !z . / 0 x !x 1 !x 2

2

z

!2 z !x 2

#0

zx

or

!2 z !x 2

2

!z − !z . x / 0 ( z # 0. !x 1 !x 2

... (7)

2

!2 z

− !z . !z y/ 0 ( z # 0. !y 1 !y 2

Similarly, from (3) and (5),

zy

Differentiating (2) partially w.r.t. y,

0 a 2 (!z / !y ) (!z / !x) z (! 2 z / !x!y ) # 0

!y 2



(!z / !x) (!z / !y) z (! 2 z / !x!y ) # 0

... (8)

%

... (9)

(7), (8) and (9) are three possible forms of the required partial differential equations. Ex. 14. Find the partial differential equation of all planes which are at a constant distance ‘a’ from the origin. Sol. Let lx + my + nz ! a ... (1) be the equation of the given plane where l, m, n are direction colines of the normal to the plane so that l2 + m2 + n2 ! 1, l, m, n being parameters ... (2) Differentiating (1) partially w.r.t. ‘x’ and ‘y’, we have l + np ! 0 ...(3) m + nq ! 0, ... (4) where p # !z / !x and q # !z / !y. From (3) and (4), l ! – np and m ! – nq. Substituting these values in (2), we have n2(p2 + q2 + 1) ! 1 so that n ! (p2 + q2 + 1)–1/2 ... (5) 2 2 –1/2 and m ! – nq ! – q(p2 + q2 + 1)–1/2 ... (6) ∋ l ! – np ! – p(p + q + 1) Substituting the values of l, m, n given by (5) and (6) in (1), we get

Created with Print2PDF. To remove this line, buy a license at: http://www.software602.com/

1.10

Origin of partial differential equations 2

2

–1/2

– px (p + q + 1) – qy (p + q + 1) + z(p2 + q2 + 1)–1/2 ! a or z ! px + qy + a (p2 + q2 + 1)1/2, which is the required partial differential equation. Ex. 15. Show that the partial differential equation obtained by eliminating the arbitrary constants a and c from z ! ax + g(a) y + c, where g (a) is an arbitrary function of a, is free of the variables x, y, z. Sol. Differentiating z ! ax + g(a) y + c partially w.r.t. ‘x’ and ‘y’ yields p ! a and q ! g(a). Eliminating a between them leads to q ! g(p) or f(p, q) ! 0, where f is an arbitrary function of p and q. Clearly, the resulting partial differential equation contains p and q but none of the variables x, y, z. Ex. 16. Show that the partial differential equation obtained by eliminating the arbitrary constants a and b from z ! ax + by + f (a, b) is given by z ! px + qy + f(p, q). Sol. Differentiating z ! ax + by + f (a, b) ... (1) partially with respect to ‘x’ and ‘y’, we get p!a and q!b ... (2) Eliminating a and b from (1) and (2) yields z ! px + qy + f (p, q) Ex. 17. Form a partial differential equation by eliminating a, b and c from the relation ax2 + by2 + cz2 = 1. [Mysore 2004] 2 2 2 Sol. Given ax + by + cz ! 1. ... (1) Differentiating (1) partially w.r.t. ‘x’ and ‘y’, we have 2ax 2cz (!z / !x ) # 0

2

2

–1/2

2by 2cz (!z / !y ) # 0

... (2)

... (3)

Differentiating (2) partially w.r.t. ‘y’, we get



%

0 2c (!z / !y ) (!z / !x ) z (! 2 z / !y !x # 0

or

(!z / !x) (!z / !y) z (! 2 z / !x!y ) # 0 , ... (4)

since c is an arbitrary constant. (4) is the desired partial differential equation. Again, differentiating partially (2) w.r.t. x and (3) w.r.t. y, we get



%

2a 2c (!z / !x) 2



z (! 2 z / !x 2 ) # 0 ... (5)

2b 2c (!z / !y )2

%

z (! 2 z / !y 2 ) # 0 ... (6)

From (2), a # ((cz / x) 3 (!z / !x). Putting this in (5), we get



( (cz / x ) 3 (!z / ! x ) c (!z / !x ) 2

%

z (! 2 z / !x 2 # 0 or zx(!2 z / !x2 ) x(!z / !x)2 ( z(!z / !x) # 0 ... (7)

Similarly, from (3) and (6), we get

zy(! 2 z / !y 2 ) y (!z / !y)2 ( z (!z / !y ) # 0 . ... (8)

(4), (7) and (8) are three possible forms of the required partial differential equations.

EXERCISE 1 (A) Eliminate the arbitrary constants indicated in brackets from the following equations and form corresponding partial differential equations. Ans. ! 2 z / !x 2

1. z ! A ept sin px, (p and A). 2

2. z # A e( p t cos px, (p and A) (Sagar 1999; Ranchi 2010) 3. z ! ax3 + by3; (a, b)

Ans. ! 2 z / !x 2 # dz / !t

Ans. x(!z / !x)

4. 4z ! [ax + (y/a) + b]2; (a, b). (Delhi B.A. (Prog) II 2011) 5. z ! ax2 + bxy + cy2, (a, b,c)

! 2 z / !t 2 # 0.

y (!z / !y ) # 3z

Ans. z # (dz / !x) (!z / !y )

Ans. x2 (! 2 z / !x2 ) 2 xy(! 2 z / !x!y ) y 2 (! 2 z / !y 2 ) # 2 z

6. z2 ! ax3 + by3 + ab, (a, b) Ans. 9 x2 y 2 z # 6 x3 y 2 (!z / !x) 6 x2 y3 (!z / !y) 4 z (!z / !x) (!z / !y)

Created with Print2PDF. To remove this line, buy a license at: http://www.software602.com/

Origin of partial differential equations

1.11

ax 2

!z !z b , ( a, b) Ans. x 2 y # 2( z ( x 2 / y )2 !x !y y2 y 8. Find the differential equation of the family of spheres of radius 4 with centres on the xy7. e1/{ z (( x

2

/ y )}

#

Ans. ( x ( y) 2 [(!z / !x)2

plane.

(!z / !y)2 1] # 16(!z / !x ( !z / !y) 2

9. Find the P.D.E of planes having equal x and y intercepts. Ans. p – q ! 0 10. Find the partial differential equation of the family of spheres of radius 7 with centres on the plane x – y ! 0. Ans. (p2 + q2 + 1) (x – y)2 ! 49 (p – q) 11. Find the partial differential equation of all spheres whose centres lie on z-axis. Ans. x q – y p ! 0 1.11 Rule II. Derivation of partial differential equation by the elimination of arbitrary function 4 from the equation 4 (u, v) ! 0, where u and v are functions of x, y and z. [Meerut 1995] Proof. Given 4(u, v) ! 0. ...(1) We treat z as dependent variable and x and y as independent variables so that !z/!x ! p, !z/!y ! q, !y/!x ! 0 and !x/!y ! 0. Differentiating (1) partially with respect to x, we get

FG H

!4 !u !x !u !y !u !x !x !y !x

or or

IJ + !4 FG !v !x !v !y !v !z IJ ! 0 K !v H !x !x !y !x !z !x K !4 F !u !u !4 F !v !v p I p I !0 !u H !x !z K !v H !x !z K !4 !4 ! ( F !v p !v I F !u p !u I . H K H K !u !v !u !z !z !x

!x !z Similarly, differentiating (1) partially w.r.t. ‘y’, we get

FG H

!x

IJ FG K H

!z

IJ K

!4 !4 !v q !v !u q !u ! ( !u !v !y !z !y !z Eliminating 4 with the help of (3) and (4), we get !u !u !v p !v !u p !u ! !v q !v q !y !z !y !z !x !z !x !z

or

FH FG !u H !y

IK FH I q !u J F !v !z K H !x

IK FGH p !v I ! FG !u !z K H !x

...(3)

...(4)

IJ FG IJ K H K !u I F !v !v I p JG q J !z K H !y !z K

Pp + Qq ! R, ...(5) ! u ! v ! u ! v ( where P ! !u !v ( !u !v , Q! , R ! !u !v ( !u !v . !z !x !x !z !y !z !z !y !x !y !y !x Thus we obtain a linear partial differntial equation of first order and of first degree in p and q. Note. If the given equation between x, y, z contains two arbitrary functions, then in general, their elimination gives rise to equations of higher order. or

1.12 SOLVED EXAMPLES BASED ON RULE II OF ART. 1.11. Ex. 1. Form a partial differential equation by eliminating the arbitrary function 4 from 4(x + y + z, x2 + y2 ( z2) ! 0. What is the order of this partial differential equation ? [Bilaspur 2003; Indore 2003; Jiwaji 2003; Vikram 2001] Sol. Given 4(x + y + z, x2 + y2 ( z2) ! 0. ...(1) Let u!x+y+z and v ! x2 + y2 ( z2. ...(2)

Created with Print2PDF. To remove this line, buy a license at: http://www.software602.com/

1.12

Origin of partial differential equations

Then (1) becomes 4(u, v) ! 0. Differentiating (3) w.r.t., ‘x’ partially, we get

FH

!4 !u !u p !u !x !z

IK

FH

!4 !v !v p !v !x !z

...(3)

IK ! 0.

!u ! 1, !x From (4) and (5),

!u !v !v !u ! 1, ! 1, ! 2x, ! (2z, !z !x !z !y (!4/!u)(1 + p) + 2(!4/!v)(x ( pz) ! 0 (!4/!u)/(!4/!v) ! (2(x ( pz)/(1 + p). Again, differentiating (3) w.r.t., ‘y’ partially, we get

From (2), or

or or

!4 − !u !u . !4 − !v !v . q 0 q 0 !0 / / !u 1 !y !z 2 !v 1 !y !z 2 (!4/!u)(1 + q) + 2(!4/!v)(y ( zq) ! 0, by (5) (!4/!u)/(!4/!v) ! (2(y ( qz)/(1 + q).

...(4) !v ! 2y. ..(5) !y

...(6)

...(7)

Eliminating 4 from (6) and (7), we obtain (x ( pz)/(1 + p) ! (y ( qz)/(1 + q) or (1 + q)(x ( pz) ! (1 + p) (y ( qz) (y + z)p ( (x + z)q ! x ( y, which is the desired partial differential equation of first order. Ex. 2. Form a partial differential equation by eliminating the arbitrary function f from the equation x + y + z ! f(x2 + y2 + z2). (Kanpur 2011) Sol. Given x + y + z ! f(x2 + y2 + z2). ...(1) Differentiating partially w.r.t. ‘x’ and ‘y’, (1) gives 1 + p ! f 5(x2 + y2 + z2).(2x + 2zp). ...(2) and 1 + q ! f 5(x2 + y2 + z2).(2y + 2zq). ...(3) 2 2 2 Eliminating f 5(x + y + z ) from (2) and (3), we obtain (1 + p)/(2x + 2zp) ! (1 + q)/(2y + 2zq) or (1 + p) (y + zq) ! (1 + q) (x + zp) or (y ( z)p + (z ( x)q ! x ( y, which is the required partial differential equations. Ex. 3. Eliminate the arbitrary functions f and F from y ! f(x ( at) + F(x + at). (Sagar 1997; Vikram 1995; Jabalpur 2002) Sol. Given y ! f(x ( at) + F(x + at). ...(1) From (1), !y/!x ! f 5(x ( at) + F5(x + at) and hence !2y/!x2 ! f 6(x ( at) + F6(x + at). ...(2) Also, !y/!t ! f 5(x ( at) . ((a) + F 5(x + at) . (a) and hence !2y/!t2 ! f 6(x ( at) . ((a)2 + F 6(x + at) . (a)2 or !2y/!t2 ! a2[ f 6(x ( at) + F 6(x + at)]. ...(3) 2 2 2 2 2 Then, (2) and (3) & ! y/!t ! a (! y/!x ). Ex. 4. Eliminate arbitrary function f from (i) z ! f(x2 ( y2). [Bilaspur 1996; Sagar 1996; Bangalore 1995] 2 2 (ii) z ! f(x + y ). [Meerut 1995; Pune 2010] Sol. (i) Given z ! f (x2 ( y2). ...(1) Differentiating (1) partially with respect to x and y, we get !z/!x ! f 5(x2 ( y2) × 2x so that f 5(x2 ( y2) ! (1/2x) × (!z/!x) ...(2) 2 2 and !z/!y ! f 5(x ( y ) × ((2y) so that f 5(x2 ( y2) ! ( (1/2y) × (!z/!y). ...(3) Eliminating f 5(x2 ( y2) between (2) and (3), we have or

Created with Print2PDF. To remove this line, buy a license at: http://www.software602.com/

Origin of partial differential equations

1.13

1 !z # ( 1 !z y !z x !z ! 0. or !x !y 2 x !x 2 y !y (ii) Proceed as in part (1). Ans. y(!z/!x) ( x(!z/!y) ! 0 Ex. 5. Form a partial differential equation by eliminating the function f from (i) z ! f(y/x). [Sagar 2000] (ii) z ! xn f(y/x). Sol. Given z ! f(y/x). ...(1) Differentiating (1) partially with respect to x and y, we get !z/!x ! f 5(y/x) × ((y/x2) or f 5(y/x) ! ((x2/y) × (!z/!x) ...(2) and !z/!y ! f 5(y/x) × (1/x) or f 5(y/x) ! x(!z/!y). ...(3) Eliminating f 5(y/x) between (2) and (3), we have 2 – x !z # x !z y !x !y

or

x

!z !z +y ! 0. !x !y

which is the required partial differential equation. (ii) Given z ! xn f(y/x). ...(1) Differentiating (1) partially with respect to x and y, we get !z/!x ! n xn ( 1 f(y/x) + xn f 5(y/x) × ((y/x2) ...(2) and !z/!y ! xn f 5(y/x) × (1/x). ...(3) Multiplying both sides of (2) by x, we have x(!z/!x) ! n xnf (y/x) ( yxn ( 1 f 5(y/x). ...(4) Multiplying both sides of (3) by y, we have y(!z/!y) ! y xn ( 1 f 5(y/x). ...(5) Adding (4) and (5), x(!z/!x) + y(!z/!y) ! n xn f (y/x) or x(!z/!x) + y(!z/!y) ! nz, by (1) Ex. 6. Form a partial differential equation by eliminating the function 4 from lx + my + nz ! 4(x2 + y2 + z2). [Ravishankar 2003; Vikram 2003] Sol. Given lx + my + nz ! 4(x2 + y2 + z2). ...(1) Differentiating (1) partially with respect to x and y, we get l + n(!z/!x) ! 45(x2 + y2 + z2) × {2x + 2z(!z/!x)} ...(2) and m + n(!z/!y) ! 45(x2 + y2 + z2) × {2y + 2z(!z/!y)} ...(3) Dividing (2) by (3), we get

b g b g

b b

g g

2{x z !z !x } l n !z !x ! 2 {y z !z !y } m n !z !y

(ny ( mz)(!z/!x) + (lz ( nx) (!z/!y) ! mx ( ly, which is the required partial differential equation. Ex. 7. Form partial differential eqn. by eliminating the function f from z ! eax + by f(ax ( by). Sol. Given z ! eax + by f(ax ( by). ...(1) Differentiating (1) partially with respect to x and y, we get !z/!x ! eax + by a f 5(ax ( by) + a eax + by f(ax ( by) ...(2) ax + by ax + by and !z/!y ! e {(b f 5(ax ( by)} + b e f(ax ( by). ...(3) Multiplying (2) by b and (3) by a and adding, we get b(!z/!x) + a(!z/!y) ! 2ab eax + by f (ax ( by) or b(!z/!x) + a(!z/!y) ! 2abz, by (1) Ex. 8. Form a partial differential equation by eliminating the arbitrary functions f and F from z ! f(x + iy) + F(x ( iy), where i2 ! (1. [Bilaspur 2004; Jiwaji 1998; Meerut 2010] Sol. Given z ! f(x + iy) + F(x ( iy). ...(1) Differentiating (1) partially with respect to x and y, we get !z/!x ! f 5(x + iy) + F 5(x ( iy) ...(2) and !z/!y ! i f 5(x + iy) ( iF5(x ( iy). ...(3) or

Created with Print2PDF. To remove this line, buy a license at: http://www.software602.com/

1.14

Origin of partial differential equations

Differentiating (2) and (3) partial w.r.t. x and y respectively, we get !2z/!x2 ! f 6(x + iy) + F6(x ( iy) ...(4) 2 2 2 and ! z/!y ! i f 6(x + iy) + i2F 6(x ( iy) ! ({f 6(x + iy) + F 6(x + iy)}. ...(5) 2 2 2 2 Adding (4) and (5), ! z/!x + ! z/!y ! 0, which is the required equation. Ex. 9. Form partial differential equation by eliminating arbitrary functions f and g from z ! f(x2 ( y) + g(x2 + y). [Nagpur 1996 ; I.A.S. 1996; Kanpur 2011] Sol. Given z ! f(x2 ( y) + g(x2 + y). ...(1) Differentiating (1) partially with respect to x and y, we get !z/!x ! 2xf 5(x2 ( y) + 2xg5(x2 + y) ! 2x{f 5(x2 ( y) + g5(x2 + y)}. ...(2) and !z/!y ! (f 5(x2 ( y) + g5(x2 + y). ...(3) Differentiating (2) and (3) w.r.t. x and y respectively, we get !2z/!x2 ! 2{f 5(x2 ( y) + g5(x2 + y)} + 4x2{f 6(x2 ( y) + g6(x2 + y)} ...(4) and !2z/!y2 ! f 6(x2 ( y) + g6(x2 + y). ...(5) 2 2 Again, (2) & f 5(x ( y) + g5(x + y) ! (1/2x) × (!z/!x). ...(6) Substituting the values of f 6(x2 ( y) + g6(x2 + y) and f 5(x2 ( y) + g5(x2 + y) from (5) and (6) in (4), we have

FH IK

2 2 2 1 !z !2 z 2! z or x ! z2 # ! z 4 x 3 ! z2 , 2 2 ! 2 × 2x !x + 4x !y !x !x !x !y which is the required partial differential equation. Ex. 10. Find the differential equation of all surfaces of revolution having z-axis as the axis of rotation. [I.A.S. 1997] Sol. From coordinate geometry of three dimensions, equation of any surface of revolution having z-axis as the axis of rotation may be taken as z ! 4[(x2 + y2)1/2], where 4 is an arbitrary function. ...(1) Differentating (1) partially with respect to x and y, we get !z/!x ! 45[(x2 + y2)1/2] × (1/2) × (x2 + y2)(1/2 × 2x ...(2) and !z/!y ! 45[(x2 + y2)1/2] × (1/2) × (x2 + y2)(1/2 × 2y. ...(3)

!z !x x or y !z # x !z . # !z !y y !x !y Ex. 11. Form a partial differential equation by eliminating the arbitrary functions f and g from z ! y f(x) + x g(y). (Guwahati 2007) Sol. Given z ! y f(x) + x g(y). ...(1) Differentiating (1) partially w.r.t. ‘x’ and ‘y’, we get !z/!x ! y f 5(x) + g(y) ...(2) !z/!y ! f(x) + x g 5(y). ...(3) Differentiating (3) with respect to x, !2z/!x!y ! f 5(x) + g 5(y). ...(4)

Dividing (2) by (3),

1 7 !z 8 ( g ( y) : y 9; !x < Substituting these values in (4), we have

From (2) and (3),

f 5(x) !

LM N

and

g 5( y ) #

8 1 7 !z 9 ( f ( x ): . x ; !y
), y0 (>) and z0 (>) are functions which, together with their first derivatives, aree continuous in the interval I defined by >1 ? > ? >2 . (b) And if f (x, y, z, p, q) is a continuous function of x, y, z, p and q in a certain region U of the xyzpq space, then it is required to establish the existence of a function ≅( x, y) with the following properties : (i) ≅( x, y) and its partial derivatives with respect to x and y are continuous functions of x and y in a region R of the xy space. (ii) For all values of x and y lying in R, the point {x, y, ≅( x, y), ≅ x ( x, y), ≅ y ( x, y) } lies in U and f [ x, y, ≅( x, y), ≅ x ( x, y), ≅ y ( x, y)] = 0. x (iii) For all > belonging to the interval I, the point {x0 (>), y0 (>)} belongs to the region R,

and ≅{x0 (>), y0 (>)} # z0 Stated geometrically, what we wish to prove is that there exists a surface z # ≅( x, y) which passes through the curve C whose parametric equations are given by x # x0 (> ), y # y0 (>), z # z0 (>) and at every point of which the *direction (p, q, – 1) of the normal is such that f ( x, y, z, p, q) # 0 Problem 1. State the properties of ≅( x, y) if there exists a surface z # ≅( x, y) which passes through the curve C with parametric equations x # x0 (>), y # y0 (>), z # z0 (>) and at every point of which the direction ( p, q, (1) of the normal is such that f ( x, y, z, p, z) # 0 . (Delhi B.Sc. (H) 2002) Sol. Hint. Refer conditions (i), (ii) and (iii) of the above Art. 1.13 Problem 2. Solve the Cauchy’s problem for zp q # 1 , when the initial data curve is x0 # >, y0 # >, z0 # > / 2, 0 Α > Α 1 . [Bangalore 2003; I.A.S. 2004] Sol. Given Given inital data curve From (1), and

f ( x, y, z, p, q) # zp q ( 1 # 0 x0 # >,

y0 # >,

... (1)

z0 # > / 2,

!f / !p # z,

0 Α > Α1

... (2)

!f / !q # 1 ,

!f dx 0 !f dy0 1 ( # 1 3 1 ( z 3 1 # 1 ( > = 0 , for 0 Α > Α 1 . !q d > !p d> 2 Now, we have the following ordinary differential equations : *Let be the equation of the given surface Let

z # ≅( x, y)

... (1)

F( x, y, z) # ≅( x, y) ( z .

... (2)

!F !4 !z !F !4 !z !F # # # p, # # # q, # (1 From (1) and (2), !x !x !x !y !y !y !z Since ΒF is normal to the surface F ( x, y, z) # 0, !F / !x, !F / !y , !F / !z i.e., p, q – 1 are direction ratios of the normal to F (x, y, z) = 0 or z # ≅ ( x, y) .

Created with Print2PDF. To remove this line, buy a license at: http://www.software602.com/

1.18

Origin of partial differential equations

dx !f # , dt !p

dy !f dz !z dx !z dy # and # dt !q dt !x dt !y dt dy / dt # 1 or dx / dt # z , and dz / dt # p(!f / !p) q (!f / !q) # pz q # 1, by (1) Integrating (3) and (4), y # t C1 and z # t C2

x(>, 0) # >, y(>,0) # > From (2), at t # 0, y#t > Using (6), (5) reduces to

Then, from (3) and (7), dx / dt # t > / 2 so that x # (1/ 2) 3 t

... (5)

z(>,0) # > /2 ... (6)

and and 2

... (3) ... (4)

z # t >/2

(1/ 2) 3 >t C3

Using (6), (8) reduces to x # (1/ 2) 3 t 2 (1/ 2) 3 >t > Solving y # t > with (9) for > and t in terms of x and y, we get

... (7) ... (8) ... (9)

x ( ( y2 / 2) 1 ( ( y / 2) Putting these values in z # t > / 2 , the required solution passing through the initial data curve is z # {2( y ( x ) x ( y2 / 2}/(2 ( y) . t#

y(x 1 ( ( y / 2)

>#

and

OBJECTIVE PROBLEMS ON CHAPTER 1 Indicate the correct answer by writing (a), (b), (c) or (d) 1. Equation p tan y + q tan x ! sec2 z is of order (a) 1 (b) 2 (c) 0 (d) none of these 2

2

2

[Agra 2005, 2008]

2

2. Equation ! z / !x ( 2(! z / !x!y) (!z / !y ) # 0 is of order (a) 1 (b) 2 (c) 3 (d) none of these [Agra 2005, 2006] 3. The equation (2x + 3y)p + 4xq – 8pq ! x + y is (a) linear (b) non-linear (c) quasi-linear (d) semi-linear [Agra 2005, 06] 4. ( x y ( z ) (!z / !x ) (3x 2 y ) (!z / !y ) 2 z # x y is (a) linear (b) quasi-linear (c) semi-linear Answers 1. (a)

2. (b)

3. (b)

(d) non-linear

4. (b)

MISCELLANEOUS EXAMPLES ON CHAPTER 1 Ex.1. Formulate a partial differential equation by eliminating arbitrary constants a and b from the equation ( x a)2 ( y b) 2 z 2 # 1 . Examine whether the partial differential equation is linear or non-linear. Also, find its order and degreee. [Delhi Maths (H) 2008] Hint. Proceed as in Ex. 5(a), page 1.6 with ) ! 1. Thus we get the partial differential



%

2 2 2 equation z (!z / !x) (!z / !y) 1 # 1 , which is non-linear partial differential equation of order

one and degree two. Ex. 2. Eliminate arbitrary constants a and b from the following equations : (i) ax2 + by2 + z2 ! 1 (Delhi B.A. (Prog.) II 2010) (ii) z ! ax + (1 – a) y + b (Lucknow 2010) Ans. (i) z (z – px – qy) ! 1

(ii) p + q ! 1, where p # !z / !x, q # !z / !y

Ex. 3. (i) Eliminate the arbitrary function 4 from p + x – y = 4 (q – x + y) (Ranchi 2010)

Created with Print2PDF. To remove this line, buy a license at: http://www.software602.com/

Origin of partial differential equations

1.19

(ii) State true or false with justification. Eliminating arbitrary function f from z = f (x2 + y2), we get first order non-linear partial differential equation. (Pune 2010) Ans. (i) (1 ! 2 z / !x2 ) (1 ! 2 z / !y 2 ) # (! 2 z / !x!y ( 1)2 (ii) False. see Ex. 4 (ii), page 1.21. Ex. 4. (i) Obtain the partial differential equation by eliminating arbitrary function of f and g from the equation v ! {f (r – at) + g (r + at)}/r (Nagpur 2010) Ans. Given v ! (1/r) × {f (r – at) + g (r + at)} ...(1) (1) & !v / !t ! (1/r) × {–a f 5(r – at) + ag5 (r + at)} ! – (a/r) × {f 5(r – at) – g5 (r + at)} 2

...( 2)

(2) & !2 v / !t 2 ! – (a/r)×{–af 55 (r – at) –ag55 (r + at)} ! (a /r)×{f 55(r – at)+g55 (r + at)}

...(3)

(1) & !v / !r ! (1/r)×{(f 5 (r – at) + g5 (r + at)} – (1/r2)×{f (r – at) + g (r + at)}

...(4)

2

(4) & !2 v / !r 2 ! (1/r)×{f 55 (r – at) + g55 (r + at)} – (1/r ) × {f 5 (r – at) + g5 (r + at)} ! –(1/r2) ×{f 5 (r – at) + g5 (r + at)} + (2/r3) × {f (r – at) + g (r + at)} ! (1/a2) × (! 2v / !t 2 ) – (2/r2) × {f5 (r – at)} + g5 (r + at)} + (2/r2) × v, using (1) and (3) ! (1/a2) × (! 2v / !t 2 ) – (2/r) × [!v / !r (1/ r 2 ) × {f (r – at)} + g (r + at)} + (2/r2)× v [Since from (4), (1/r)×{f5 (r – at) + g5 (r + at)} ! !v / !r +(1/r2)× {f (r – at) + g (r + at)}] Thus, !2 v / !r 2 ! (1/a2) × (! 2v / !r 2 ) –(2/r) × {!v / !r (1/ r) 3 v} +(2/r2)× v, using (1) or

2 2 2 2 !2 v / !r 2 ! (1/a ) × (! v / !t ) – (2/r) × (! / !r ), which is the required equation

Created with Print2PDF. To remove this line, buy a license at: http://www.software602.com/

2 Linear Partial differential equations of order one 2.1. LAGRANGE’S EQUATION A quasi–linear partial differential equation of order one is of the form Pp + Qq = R, where P, Q and R are functions of x, y, z. Such a partial differential equation is known as Lagrange equation. For Example xyp + yzq = zx is a Lagrange equation. 2.2. Lagrange’s method of solving Pp + Qq = R, when P, Q and R are functions of x, y, z (Delhi Maths (H) 2009; Meerut 2003; Poona 2003, 10; Lucknow 2010) Theorem. The general solution of Lagrange equation Pp + Qq = R, ... (1) !(u, v)

is

0

... (2)

where ! is an arbitrary function and u(x, y, z) = c1 are two independent solutions of

v(x, y, z) = c2

and

... (3)

(dx)/P = (dy)/Q = (dz)/R ... (4) Here, c1 and c2 are arbitrary constants and at least one of u, v must contain z. Also recall that u and v are said to be independent if u/v is not merely a constant. Proof. Differentiating (2) partially w.r.t. ‘x’ and ‘y’, we get

and

#! ∃ #u #u % #! ∃ #v #v % ∋) & p (∗ & ∋) & p (∗ #u #x #z #v #x #z

0

... (5)

#! ∃ #u #u % #! ∃ #v #v % &q ( & ∋ &q ( ∋ #u ) #y #z ∗ #v ) #y #z ∗

0

... (6)

Eliminating #! / #u and #! / #v between (5) and (6), we have #u / #x & p ( #u / #z ) #v / #x & p (#v / #z ) #u / #y & q (#u / #z )

#v / #y & q (#v / #z )

0

or

#u % ∃ #v #v % ∃ #u #u % ∃ #v #v % ∃ #u & p (∋ & q ( + ∋ & q (∋ & p ( ∋ #z ∗ ) #y #z ∗ ) #y #z ∗ ) #x #z ∗ ) #x

0

or

∃ #u #v #u #v % #u #v #u #v ∃ #u #v #u #v % ∋) #z #y + #y #z ∗( p & ∋) #x #z + #z #x (∗ q & #x #y + #y #x

0

∃ #u #v #u #v % #u #v #u #v ∃ #u #v #u #v % + + + ∋ ( p&∋ (q # y # z # z # y # z # x # x # z #x #y #y #x ) ∗ ) ∗ Hence (2) is a solution of the equation (7) Taking the differentials of u(x, y, z) = c1 and v(x, y, z) = c2, we get

,

(#u / #x )dx & (#u / #y )dy & (#u / #z )dz 2.1

0

... (7)

... (8)

Created with Print2PDF. To remove this line, buy a license at: http://www.software602.com/

2.2

Linear Partial differential equations of order one

(#v / #x)dx & (#v / #y )dy & (#v / #z )dz

and

0

... (9)

Since u and v are independent functions, solving (8) and (9) for the ratios dx : dy : dz, gives dx

dy

#u #v #u #v #u #v #u #v + + #y #z #z #y #z #x #x #z Comparing (4) and (10), we obtain #u #v #u #v + #y #z #z #y P

#u #v #u #v + #z #x #x #z Q

dz #u #v #u #v + #x #y #y #x #u #v #u #v + #x #y #y #x R

... (10)

k , say

#u #v #u #v #u #v #u #v #u #v #u #v + kP, + kR + kQ and #y #z #z #y #x #y #y #x #z #x #x #z Substituting these values in (7), we get k(Pp + Qq) = kR or Pp + Qq = R, which is the given equation (1). Therefore, if u(x, y, z) = c1 and v(x, y, z) = c2 are two independent solutions of the system of



differential equations (dx)/P = (dy)/Q = (dz)/R, then !(u, v)

0 is a solution of Pp + Qq = R,

! being an arbitrary function. This is what we wished to prove. Note. Equations (4) are called Lagrange’s auxillary (or subsidiary) equations for (1). 2.3. Working Rule for solving Pp + Qq = R by Lagrange’s method. [Delhi Maths Hons. 1998] Step 1. Put the given linear partial differential equation of the first order in the standard form Pp + Qq = R. ...(1) Step 2. Write down Lagrange’s auxiliary equations for (1) namely, (dx)/P = (dy)/Q = (dz)/R ...(2) Step 3. Solve (2) by using the well known methods (refer Art. 2.5, 2.7, 2.9 and 2.11). Let u(x, y, z) = c1 and v(x, y, z) = c2 be two independent solutions of (2). Step 4. The general solution (or integral) of (1) is then written in one of the following three equivalent forms : !(u, v) = 0, u = !(v) or v = !(u), ! being an arbitrary function. 2.4. Examples based on working rule 2.3. In what follows we shall discuss four rules for getting two independent solutions of (dx)/P = (dy)/Q = (dz)/R. Accordingly, we have four types of problems based on Pp + Qq = R. 2.5. Type 1 based on Rule I for solving (dx)/P = (dy)/Q = (dz)/R. ...(1) Suppose that one of the variables is either absent or cancels out from any two fractions of given equations (1). Then an integral can be obtained by the usual methods. The same procedure can be repeated with another set of two fractions of given equations (1). 2.6. SOLVED EXAMPLES BASED ON ART. 2.5 Ex. 1. Solve (y2z/x)p + xzq = y2. Sol. Given (y2z/x)p + xzq = y2. The Lagrange’s auxiliary equations for (1) are Taking the first two fractions of (2), we have x2zdx = y2zdy or

[Indore 2004; Sagar 1994] ...(1) dy dz dx = . ..(2) xz y2 ( y2 z x) 3x2dx – 3y2dy = 0,

...(3)

Created with Print2PDF. To remove this line, buy a license at: http://www.software602.com/

Linear Partial differential equations of order one 3

2.3

3

Integrating (3), x – y = c1, c1 being an arbitrary constant ...(4) Next, taking the first and the last fractions of (2), we get xy2dx = y2zdz or 2xdx – 2zdz = 0. ...(5) 2 2 Integrating (5), x – z = c2, c2 being an arbitrary constant ...(6) From (4) and (6), the required general integral is !(x3 – y3, x2 – z2) = 0, ! being an arbitrary function. Ex. 2. Solve (i) a(p + q) = z. [Bangalore 1997] (ii) 2p + 3q = 1. [Bangalore 1995] Sol. (i) Given ap + aq = z. ...(1) The Lagrange’s auxiliary equation for (1) are (dx)/a = (dy)/a = (dz)/1. ...(2) Taking the first two members of (1), dx – dy = 0. ...(3) Integrating (3), x – y = c1, c1 being an arbitrary constant ...(4) Taking the last two members of (1), dy – adz = 0. ...(5) Integrating (5), y – az = c2, c2 being an arbitrary constant. ...(6) From (4) and (6), the required solution is given by !(x – y, y – az) = 0, ! being an arbitrary function. Ex. 3. Solve p tan x + q tan y = tan z. [Madras 2005 ; Kanpur 2007] Sol. Given (tan x)p + (tan y)q = tan z. ...(1) The Lagrange’s auxiliary equations for (1) are

dy dx dz . ...(2) tan x tan y tan z cot x dx – cot y dy = 0. (sin x)/(sin y) = c1. ...(3) cot y dy – cot z dz = 0. (sin y)/(sin z) = c2. ...(4)

Taking the first two fractions of (2), Integrating, log sin x – log sin y = log c1 or Taking the last two fractions of (2), Integrating, log sin y – log sin z = log c2 or From (3) and (4), the required general solution is sin x/sin y = !(sin y/sin z), ! being an arbitrary function. Ex. 4. Solve zp = –x. Sol. Given zp + 0.q = –x. ...(1) The Lagrange’s subsidiary equations for (1) are (dx)/z = (dy)/0 = (dz)/(–x) ...(2) Taking the first and the last members of (2), we get – xdx = zdz or 2xdx + 2zdz = 0. ...(3) Integrating (3), x2 + z2 = c1, c1 being an arbitrary constant. ...(4) Next, the second fraction of (2) implies that dy = 0 giving y = c2 ...(3) 2 2 From (4) and (5), the required solution is x + z = !(y), ! being an arbitrary function. Ex. 5. Solve y2p – xyq = x(z – 2y) [Delhi Maths Hons. 1995, Delhi Meths(G) 2006] dx = dy + xy y2

dz . x( z + 2 y)

...(1)

Taking the first two fractions of (1) and re–writing, we get 2xdx + 2ydy = 0 so that Now, taking the last two fractions of (1) and re–writing, we get

x2 + y2 = c1.

...(2)

z + 2y dz =– dy y

dz & 1 z = 2 dy y

...(3)

Sol. Here Lagrange’s auxiliary equations are

which is linear in z and y. Its I.F. = e

or

z

(1/ y ) dy

e log y = y. Hence solution of (3) is

Created with Print2PDF. To remove this line, buy a license at: http://www.software602.com/

2.4

Linear Partial differential equations of order one

z

zy – y2 = c2. ...(4) 2 2 2 Hence !(x + y , zy – y ) = 0 is the desired solution, where ! is an arbitrary function. Ex. 6. Solve (x2 + 2y2)p – xyq = xz [K.U. Kurukshetra 2005] Sol. The Lagrange’s auxiliary equation for the given equation are z . y = 2 ydy & c2

or

dx 2

x & 2y

2

dy + xy

dz xz

... (1)

Taking the last two fractions of (2) and re–writing, we get (1/y) dy + (1/z)dz = 0 so that log y + log z = log c1 Taking the first two fractions of (1), we have

dx dy

x2 & 2 y 2 + xy

2x

or

or dx ∃ 2 % 2 &∋ (x dy ∋) y 2 (∗

yz = c1 ... (2)

+4 y

... (3)

y2x2 + y4 = c2

... (4)

Putting x2 = v and 2x(dx/dy) = dv/dx, (3) yields dv/dx + (2/y) v = – 4y, which is a linear equation. e.

Its integrating factor

yv2

(2 / y ) dy

. /(+ 4 y) xy 0 dy & c 2

y 2 and hence its solution is

e 2log y

or

2

From (2) and (4), the required solution is !( yz, y 2 x 2 & y 4 )

0, ! being an arbitrary function.

EXERCISE 2 (A) Solve the following partial differential equations 1. (–a + x)p + (–b + y)q = (–c + z). 2. xp + yq = z

(Kanpur 2011)

3. p + q = 1 4. x2p + y2p = z2

Ans. !{( x + a) /( y + b), ( y + b) /( z + c )} 0 Ans. !( x / z , y / z )

0

Ans. !( x + y , x + z )

0

[Bilaspur 2001, Jabalpur 2000, Sagar 2000, Vikram 1999] Ans. !(1/ x + 1/ y, 1/ y + 1/ z )

0

Ans. !(1/ x + 1/ y , 1/ y & 1/ z )

0

Ans. !( x + y , z & cos x)

0

7. yzp + 2xq = xy [Nagpur 1996]

Ans. !( x 2 + z 2 , y 2 + 4 z )

0

8. xp + yq = z [Bangalore 1995]

Ans. !( x / y, x / z )

0

5. x2p + y2q + z2 = 0 6. #z / #x & #z / #y

sin x [Meerut 1995]

9. yzp + zxq = xy

[M.S. Univ. T.N. 2007, Lucknow 2010, Revishankar 2004] Ans. !( x 2 + y 2 , x2 + z 2 ) 0

10. zp = x 11. y2p2 + x2q2 = x2y2z2

Ans. !( y, x2 + z 2 )

0

Ans. !( x3 + y3 , y 3 & 3z +1 ) 0

2.7. Type 2 based on Rule II for solving (dx)/P = (dy)/Q = (dz)/R. ...(1) Suppose that one integral of (1) is known by using rule I explained in Art 2.5 and suppose also that another integral cannot be obtained by using rule I of Art. 2.5. Then one integral known to

Created with Print2PDF. To remove this line, buy a license at: http://www.software602.com/

Linear Partial differential equations of order one

2.5

us is used to find another integral as shown in the following solved examples. Note that in the second integral, the constant of integration of first integral should be removed later on. 2.8. SOLVED EXAMPLES BASED ON ART. 2.7 Ex. 1. Solve p + 3q = 5z + tan (y – 3x). [Agra 2006; Meerut 2003; Indore 2002; Ravishankar 2003] Sol. Given p + 3q = 5z + tan (y – 3x). ...(1) dx = dy dz . ...(2) 1 3 5z & tan( y + 3x ) Taking the first two fractions, dy – 3dx = 0. ...(3) Integrating (3), y – 3x = c1, c1 being an arbitrary constant. ...(4) dx dz Using (4), from (2) we get . ...(5) 1 5z & tan c1 Integrating (5), x – (1/5) × log (5z + tan c1) = (1/5) × c2, c2 being an arbitrary constant. 5x – log [5z + tan (y – 3x)] = c2, using (4) ...(6) From (4) and (6), the required general integral is 5x – log [5z + tan (y – 3x)] = !(y – 3x), where ! is an arbitrary function. Ex. 2. Solve z(z2 + xy) (px – qy) = x4. Sol. Given xz(z2 + xy)p – yz(z2 + xy)q = x4. ...(1)

The Lagrange’s subsidary equations for (1) are

or

dy dx = xz(z 2 & xy) + yz(z 2 & xy)

The Lagrange’s subsidiary equations for (1) are

dz . ...(2) x4

Cancelling z(z2 + xy), the first two fractions give (1/ x) dx

+(1/ y ) dy

Integrating (3),

log x + log y = log c1

or

0.

xy = c1.

or dx xz(z 2 & c1)

Using (4), from (2) we get or

(1/ x) dx & (1/ y ) dy

or

...(3) ...(4)

dz x4

x3dx = z(z2 + c1)dz or x3dx – (z3 + c1z)dz = 0. ...(5) 4 4 2 4 4 2 Integrating (5), x /4 – z /4 – (c1z )/2) = c2/4 or x – z – 2c1z = c2 x4 – z4 – 2xy z2 = c2, using (4) ...(6) From (4) and (6), the required general integral is !(xy, x4 – z4 – 2xy z2) = 0, ! being an arbitrary function. Ex. 3. Solve xyp + y2q = zxy – 2x2. [Garhwal 2005] 2 2 Sol. Given xyp + y q = zxy – 2x . ...(1) dy dz dx = The Lagrange’s subsidiary equations for (1) are . ...(2) y 2 zxy + 2 x 2 xy Taking the first two fractions of (2), we have (dx)/xy = (dy)/y2 or (1/x)dx – (1/y)dy = 0 ...(3) Integrating (3), log x – log y = log c1 or x/y = c1. ...(4) From (4), x = c1y. Hence from second and third fractions of (2), we get dy y2

dz c1zy 2 + 2c12 y 2

or

c1dy –

dz = 0. z + 2c12

...(5)

Integrating (5), c1y – log (z – 2c12) = c2 or x – log [z – 2(x2/y2)] = c2, using (4). ...(6) From (4) and (6), the required general solution is x – log [z – 2(x2/y2)] = !(x/y), ! being an arbitrary function.

Created with Print2PDF. To remove this line, buy a license at: http://www.software602.com/

2.6

Linear Partial differential equations of order one

Ex. 4. Solve xzp + yzq = xy. [Bhopal 1996; Jabalpur 1999; Jiwaji 2000; Punjab 2005; Agra 2007; Ravishanker 1996; Vikram 2000] Sol. Given xzp + yzq = xy. ...(1) dx = dy dz . ...(2) xz yz xy Taking the first two fractions of (2), (1/x)dx – (1/y)dy = 0 ...(3) Integrating (3), log x – log y = log c1 or x/y = c1. ...(4) From (4), x = c1y. Hence, from second and third fractions of (2), we get (1/yz)dy = (1/c1y2)dz or 2c1y dy – 2z dz = 0. ...(5) Integrating (5), c1y2 – z2 = c2 or xy – z2 = c2, using (4). ...(6) 2 From (4) and (6), the required solution is !(xy – z , x/y) = 0, ! being an arbitrary function. Ex. 5. Solve py + qx = xyz2 (x2 – y2). Sol. Given py + qx = xyz2 (x2 – y2). ...(1)

The Lagrange’s subsidiary equations for (1) are

dx = dy x y

The Lagrange’s auxiliary equations for (1) are

or

or

dz . xyz ( x 2 + y 2 ) 2

...(2)

Taking the first two fractions of (2), 2xdx – 2ydy = 0. ...(3) 2 2 Integrating. x – y = c1, c1 being an arbitrary constant. ...(4) Using (4), the last two fractions of (2) give (dy)/x = (dz)/(xyz2c1) or 2c1y dy – 2z–2dz = 0. ...(5) 2 Integrating (5), c1 y + (2/z) = c2, c2 being an arbitrary constant. y2 (x2 – y2) + (2/z) = c2, using (4). ...(6) From (4) and (6), the required general solution is y2 (x2 – y2) + (2/z) = !(x2 – y2), where ! is an arbitrary function. Ex. 6. Solve xp – yq = xy [Madras 2005] Sol. The Lagrange’s auxiliary equations for the given equation are (dx)/x = (dy)/(–y) = (dz)/(xy) ... (1) Taking the first two fractions of (1), (1/x)dx + (1/y)dy = 0 Intergrating, log x + log y = c1 so that xy = c1 ... (2) Using (2), (1) yields (1/x)dx = (1/c1) dz so that log x – log c2 = z/c1 log (x/c2) = z/c1 or log (x/c2)= z/(xy), by (2) z/(xy) –z/(xy) Thus, x/c2 = e or xe = c2, c2 being an arbitrary constant. ... (3) From (2) and (3), the required solution is x e+ z /( xy )

!( xy), ! being an arbitrary function

Ex. 7. Solve p + 3q = z + cot (y – 3x). Sol. The Lagrange’s auxiliary equation for the given equation are dx 1

dy 3

dz z & cot ( y + 3x)

... (1)

Taking the first two fractions of (1), dy – 3 dx = 0 Taking the first and last fraction of (1), we have dx

dz z & cot ( y + 3 x)

Intergrating,

[M.D.U Rohtak 2006]

or

so that

dx

y – 3x = c1 ... (2)

dz , using (2) z & cot c1

x = log | z + cot c1| + c2, c1 and c2 being an arbitrary constants.

Created with Print2PDF. To remove this line, buy a license at: http://www.software602.com/

Linear Partial differential equations of order one

or

2.7

x – log | z + cot (y – 3x)| = c2, using (2) From (2) and (3), the required general solution is

... (3)

x + log | z & cot ( y + 3x ) | !( y + 3x ), ! being an arbitrary function. Ex. 8. Solve px (z – 2y2) = (z – qy) (z – y2 – 2x3)[Delhi B.Sc. II 2008; Delhi B.A. II 2010] Sol. Re–writing the given equation, we have x(z – 2y2) p + y (z – y2 – 2x3) q = z(z – y2 – 2x3) ... (1) The Lagrange’s subsidiary equations for (1) are dx

dy 2

dz

2

3

x( z + 2 y ) y ( z + y + 2 x ) Taking the last two fraction, we get Integrating, log z = log y + log a where a is an arbitrary constant. Using (3), (2) yields dx

... (2)

z( z + y 2 + 2 x3 ) (1/z)dz = (1/y)dy or z/y = a

... (3)

dy

so that (ay – y2 – 2x3) dx + x (2y – a) dy = 0 ... (4) x(ay + 2 y ) y (ay + y 2 + 2 x 3 ) Comparing (4) with Mdx + Ndy = 0, here M = ay – y2 – 2x3 and N = x (2y – a). Then 2

#M / #y

a + 2 y and #N / #x 1 ∃ #M #N % + ∋ ( N ) #y #x ∗

2 y + a. Now, we have

1 1 2(a + 2 y ) x(2 y + a )

2 + , which is a function of x alone. x

Hence, by usual rule, integrating factor of (1) e . e+2log x e x x+2 Multiplying (4) by x–2, we get exact equation (ayx–2 – y2x–2 – 2x)dx + x–1(2y – a) dy = 0 By the usual rule of solving an exact equation, its solution is ( +2 / x) dx

. /(ay + y ) x 2

or or

+2

0

.

+ 2 x dx & x+1 (2 y + a)dy

+2

=b

(Treating y as constant) (Integrating terms free from x) 2 2 (ay – y ) × (–1/x) – x = b or (y2 – ax)/x – x2 = b 2 3 (y – ax – x )/x = b, where b is an arbitrary constant. From (3) and (5), required solution is ( y 2 + ax + x3 ) / x

... (5)

!( z / y ), ! being an arbitrary function

EXERCISE 2 (B) Solved the following differential equations: 1. p – 2q = 3x2 sin (y + 2x).

Ans. x2 sin( y & 2 x) + z Ans. x + ( x & y ) log z

2. p – q = z/(x + y). 3. xy2p – y3q + axz = 0.

2

5. (a) z(p – q) = z + (x + y) . (Meerut 2011) (b) z ( p & q) z 2 & ( x + y )2 6. p – 2q = 3x2 sin (y + 2x). 7. p – q = z/(x + y).

!( x & y )

Ans. log z & (ax / 3 y 2 )

4. (x2 – y2 – z2)p + 2xyq = 2xz. 2

!( y & 2 x)

Ans. ( x2 & y 2 & z 2 ) / z

!( xy ) !( y / z )

Ans. e2 y [ z 2 & ( x & y )2 ] !( x & y) Ans. e2 y [ z 2 & ( x + y) 2 ] !( x + y) Ans. x3 sin( y & 2 x) + z Ans. x + ( x & y ) log z

!( y & 2 x) !( x & y )

Created with Print2PDF. To remove this line, buy a license at: http://www.software602.com/

2.8

Linear Partial differential equations of order one

Ans. 2 x( x & y) + z 2

8. zp – zq = x + y. 9. xyp + y2q + 2x2 – xyz = 0.

! ( x & y)

Ans. x + log | z + (2 x / y ) | !( x / y )

2.9. Type 3 based on Rule III for solving (dx)/P = (dy)/Q = (dz)/R. ...(1) Let P1, Q1 and R1 be functions of x, y and z. Then, by a well–known principle of algebra, ( P1dx & Q1dy & R1dz ) /( P1 P & Q1Q & R1 R) . ...(2)

each fraction in (1) will be equal to

If P1P + Q1Q + R1R = 0, then we know that the numerator of (2) is also zero. This gives P1dx + Q1dy + R1dz = 0 which can be integrated to give u1(x, y, z) = c1. This method may be repeated to get another integral u2(x, y, z) = c2. P1, Q1, R1 are called multipliers. As a special case, these can be constants also. Sometimes only one integral is possible by use of multipliers. In such cases second integral should be obtained by using rule I of Art. 2.5 or rule II of Art. 2.7 as the case may be. 2.10. SOLVED EXAMPLES BASED ON ART. 2.9 Ex.1. Solve {(b – c)/a}yzp + {(c – a)/b}zxq = {(a – b)/c}xy. Sol. Given {(b – c)/a}yzp + {(c – a)/b}zxq = {(a – b)/c}xy. b dy a dx = (c + a)zx (b + c ) yz

The Lagrange’s subsidiary equations of (1) are

...(1) c dz . ...(2) (a + b)xy

Choosing x, y, z as multipliers, each fraction for (2) =

a xdx & by dy & cz dz xyz [( b + c ) & ( c + a ) & ( a + b ) ]

=

ax dx & by dy & cz dz . 0

or 2axdx + 2bydy + 2czdz = 0. , ax dx + by dy + cz dz = 0 Integrating, ax2 + by2 + cz2 = c1, c1 being an arbitrary constant. Again, choosing ax, by, cz as multipliers, each fraction of (2) =

...(3)

a 2 xdx & b 2 ydy & c 2 zdz a 2 xdx & b 2 ydy & c 2 zdz = . xyz [a (b + c) & b(c + a) & c( a + b)] 0

2 2 2 or 2a2xdx + 2b2ydy + 2c2zdz = 0. , a xdx + b ydy + c zdz = 0 Integrating, a2x2 + b2y2 + c2z2 = c2, c2 being an arbitrary constant. From (3) and (4), the required general solution is given by

...(4)

! (ax2 + by2 + cz2, a2x2 + b2y2 + c2z2) = 0, where ! is an arbitrary function. Ex. 2. Solve z(x + y)p + z(x – y)q = x2 + y2. Sol. Given z(x + y)p + z(x – y)q = x2 + y2.

...(1)

dx = dy z( x & y) z( x + y)

The Langrange’s subsidiary equations for (1) are

dz . x 2 & y2

...(2)

Choosing x, –y, –z, as multipliers, each fraction =

x dx + y dy + z dz 2

2

xz ( x & y ) + yz ( x + y ) + z ( x + y )

=

x dx + y dy + z dz . 0

or 2x dx – 2y dy – 2z dz = 0. , x dx – y dy – z dz Integrating, x2 – y2 – z2 = c1, c1 being an arbitrary constant. Again, choosing y, x, –z as multipliers, each fraction = ,

y dx & x dy + z dz 2

2

yz ( x & y ) & xz ( x + y ) + z ( x & y )

y dx + x dy – z dz = 0

or

=

...(3)

y dx & x dy + z dz . 0

2d(xy) – 2zdz = 0.

Created with Print2PDF. To remove this line, buy a license at: http://www.software602.com/

Linear Partial differential equations of order one

2.9

2

Integrating, 2xy – z = c2, c2 being an arbitrary constant. From (3) and (4), the required general solution is given by

...(4)

! (x2 – y2 – z2, 2xy – z2) = 0, ! being an arbitrary function. 1

Ex. 3. Solve (mz – ny)p + (nx – lz)q = ly – mx. [Patna 2003; Madras 2005; Delhi Maths Hons. 9 9 1 ; Bhopal 2004; Meerut 2008, 10; Sagar 2002; I.A.S. 1977; Kanpur 2005,

06] Sol. The Lagrange’s auxiliary equations for the given equation are dx = dy dz . mz + ny nx + lz ly + mx Choosing x, y, z as multipliers, each fraction of (1)

...(1)

xdx & ydy & zdz xdx & ydy & zdz = x(mz + ny) & y(nx + lz) & z(ly + mx ) 0 or 2xdx + 2ydy + 2zdz = 0 , xdx + ydy + zdz = 0 Integrating, x2 + y2 + z2 = c1, c1 being an arbitrary constant. Again, choosing l, m, n as multipliers, each fraction of (1)

=

=

...(2)

ldx & mdy & n dz ldx & mdy & n dz = . l (mx + ny ) & m(nx + lz ) & n(ly + mx ) 0

so that , ldx + mdy + n dz = 0 From (2) and (3), the required general solution is given by

l x + m y + n z = c2.

...(3)

! (x2 + y2 + z2, l x + m y + n z) = 0, ! being an arbitrary function. Ex. 4. Solve x(y2 – z2)q – y(z2 + x2)q = z(x2 + y2). Sol. The lagrange’s auxiliary equations for the given equation are dy dx dz = . x( y 2 + z 2 ) + y(z 2 & x 2 ) z( x 2 & y 2 ) Choosing x, y, z, as multipliers, each fraction of (1) =

xdx & ydy & zdz 2

2

2

2

2

2

2

2

x ( y + z ) + y (z & x ) & z ( x & y )

so that − xdx + ydy + zdz = 0 Choosing 1/x, –1/y, –1/z as multipliers, each fraction of (1) =

2

=

...(1)

xdx & ydy & zdz 0

x2 + y2 + z2 = c1.

...(2)

a1 / xfdx + a1 / yfdy + a1 / zfdz = a1 / xfdx + a1 / yfdy + a1 / zfdz y2 + z 2 & z 2 & x 2 + ( x 2 & y 2 )

− (1/x)dx – (1/y)dy – (1/z)dz = 0 log {x/(yz)} = log c2 −

so that



0

log x – log y – log z = log c2 x/yz = c2.

2 2 2 , The required solution is ! (x + y + z , x/yz) = 0, ! being an arbitrary function. 2 Ex. 5. Solve (y – zx)p + (x + yz)q = x + y2. Sol. The Lagrange’s auxiliary equations for the given equation are dx dy dz = . x & yz x 2 & y 2 y + zx Choosing x, –y, z as multipliers, each fraction of (1)

...(3)

...(1)

Created with Print2PDF. To remove this line, buy a license at: http://www.software602.com/

2.10

Linear Partial differential equations of order one

xdx + ydy & zdz xdx + ydy & zdz = 0 x( y + zx) + y ( x & yz) & z( x 2 & y 2 ) 2xdx – 2ydy + 2zdz = 0 so that x2 – y2 + z2 = c1. − Choosing y, x, –1 as multipliers, each fraction of (1)

=

=



d(xy) – dz = 0

ydx & xdy + dz d( xy) + dz 2 2 = 0 y( y + zx ) & x( x & yz) + ( x & y ) so that xy – z = c2.

...(2)

...(3)

, From (2) and (3) solution is ! (x – y + z , xy – z) = 0, ! being an arbitrary function. Ex. 6. Solve x(y2 + z)p – y(x2 + z)q = z(x2 – y2). [I.A.S. 2004; Agra 2005 ; Delhi Maths (H) 2006; M.S. Univ. T.N. 2007; Indore 2003; Meerut 2009; Purvanchal 2007] Sol. Here Lagrange’s subsidiary equations for given equation are dy dx dz = . ...(1) x( y 2 & z ) + y( x 2 & z) z( x 2 + y2 ) Choosing 1/x, 1/y, 1/z as multipliers, each fraction of (1) (1 / x)dx & (1 / y)dy & (1 / z)dz (1 / x)dx & (1 / y)dy & (1 / z)dz = = 2 2 2 2 0 y & z + ( x & z) & x + y so that log x + log y + log z = log c1 − (1/x)dx + (1/y)dy + (1/z)dz = 0 log (xyz) = log c1 xyz = c1. ...(2) − − Choosing x, y, –1 as multipliers, each fraction of (1) xdx & ydy + dz xdx & ydy + dz = 2 2 2 2 2 2 = 0 x ( y & z) + y ( x & z) + z( x + y ) so that x2 + y2 – 2z = c2. ...(3) − x dx + y dy – z dz = 0 2

2

2

2 2 , From (2) and (3), solution is ! (x + y – 2z, xyz) = 0, ! is being an arbitrary function. Ex. 7. Solve (x + 2z)q + (4zx – y)q = 2x2 + y. [Meerut 2005]

dx = dy x & 2z 4zx + y

Sol. Here Lagrange’s auxiliary equations are

dz . 2x2 & y

Choosing y, x, –2z as multipliers, each fraction of (1) ydx & xdy + 2 zdz d( xy) + 2zdz = = 0 y( x & 2 z) & x(4 zx + y) + 2 z(2 x 2 & y) so that xy – z2 = c1. − d(xy) – 2zdz = 0 Choosing 2x, –1, –1 as multipliers, each fraction of (1) 2 xdx + dy + dz 2 xdx + dy + dz = = 0 2 x( x & 2 z) + ( 4zx + y) + (2 x 2 & y) so that x2 – y – z = c2. − 2xdx – dy – dz = 0 2

...(1)

...(2)

...(3)

2

, From (2) and (3), solution is ! (xy – z , x – y – z) = 0, ! being an arbitrary function. Ex. 8. Solve (z2 – 2yz – y2)p + (xy + zx)q = xy – zx. [Ranchi 2010; Meerut 1994] If the solution of the above equation represents a sphere, what will be the coordinates of its centre. Sol. Here Lagrange’s auxiliary equations for given equation are dy dx dz = . ...(1) x( y & z) x(y + z) z 2 + 2 yz + y 2

Created with Print2PDF. To remove this line, buy a license at: http://www.software602.com/

Linear Partial differential equations of order one

2.11

Taking the last two fractions of (1), we have (y – z)dy = (y + z)dz or 2ydy – 2zdz – 2(zdy + ydz) = 0. Integrating, y2 – z2 – 2yz = c1, c1 being an arbitrary constant. Choosing x, y, z as multipliers, each fractrion of (1) xdx & ydy & zdz xdx & ydy & zdz = = 2 2 0 x( z + 2 yz + y ) & xy( y & z) & xz( y + z) 2 so that x + y2 + z2 = c2. − 2xdx + 2ydy + 2zdz = 0

...(2)

...(3)

From (2) and (3), solution is ! (y2 – z2 – 2yz, x2 + y2 + z2) = 0, ! being an arbitrary function. From the solution of the given equation, it follows that if it represents a sphere, then its centre must be at (0,0,0), i.e., origin. Ex. 9. Solve (y3x – 2x4)p + (2y4 – x3y)q = 9z(x2 – y3). [Jabalpur 2004; M.S. Univ. T.N. 2007] Sol. Here Lagrange’s auxiliary equations for the given equation are given by dy dx dz = . ...(1) 2 y 4 + x 3 y 9z( x 3 + y3 ) y3 x + 2 x 4 Taking first two fractions of (1), we have (2y4 – x3y)dx = (y3x – 2x4)dy Dividing both sides by x3y3 gives or

F 1 dy + 2y dxI & FG 1 dx + 2x dyIJ = 0 Hx x K Hy y K 2

3

2

3

FG 2y + 1 IJ dx = FG 1 Hx y K Hx 3

or

2

2

∃ y d∋ 2 )x

+

IJ K

2x dy y3

% (&d ∗

∃ x % ∋∋ 2 (( = 0. )y ∗

Integrating, (y/x2) + (x/y2) = c1, c1 being an arbitrary constant. ...(2) Choosing 1/x, 1/y, 1/3z as multipliers, each fraction of (1) (1 / x )dx & (1 / y)dy & (1 / 3z)dz (1 / x)dx & (1 / y)dy & (1 / 3z)dz = 3 3 3 3 3 3 = 0 ( y + 2 x ) & (2 y + x ) & 3( x + y ) (1/x)dx + (1/y)dy + (1/3z)dz = 0 so that log x + log y + (1/3) × log z = log c2 − 1/3 log (xy z ) = log c2 xyz1/3 = c2. ...(3) − − From (2) and (3) solution is ! (xyz1/3, y/x2 + x/y2) = 0, ! being an arbitrary function. Ex. 10. Solve x2p + y2q = nxy. [Ravishankar 1998; Bhopal 1998; Jabalpur 2002] Sol. Here Lagrange’s auxiliary equations are (dx)/x2 = (dy)/y2 = (dz)/nxy ... (1) Taking the first two fractions of (1), we get x–2dx – y–2dy = 0. Integrating, –1/x + 1/y = – c1 so that (y – x)/xy = c1. ...(2) Choosing 1/x, –1/y, c1/n as multipliers, each fraction of (2) =

(1 / x )dx + (1 / y )dy & (c1 / n)dz (1 / x )dx + (1 / y )dy & (c1 / n)dz = , by (2) x + y & c1 xy x+ y& y+x

(1 / x )dx & (1 / y)dy & (c1 / n)dz 1 dx + 1 dy & c1 dz so that = 0. x y n 0 Integrating, log x – log y + (c1/n) z = (c1/n)c2, c2 being an arbitrary constant. z – (n/c1) (log y – log x) = c2 or z – (n/c1)log (y/x) = c2 =

or or

z–

nxy y log = c2, using (2). x y+x

...(3)

Created with Print2PDF. To remove this line, buy a license at: http://www.software602.com/

2.12

Linear Partial differential equations of order one

From (2) and (3), the required general solution is !

FG y + x , z + nxy log y IJ = 0, H xy y + x x K

! being an arbitrary function.

Ex. 11. Solve (x – y)p + (x + y)q = 2xz.

dx = dy x+y x&y

Sol. Here the Lagrange’s subsidiary equations are Taking the first two fractions of (1), Let From (3),

y/x = v

Using (3) and (4), (2) gives

or

1+ v dv = dx x 1 & v2 Integrating,

or or

...(2) ...(3) ...(4)

1& v 2 1+ v

FG 2 H1& v

IJ K

2v 2dx dv x 1 & v2 –1 2 2tan v – log (1 + v ) = 2 log x – log c1 log x2 – log (1 + v2) – log c1 = 2 tan–1 v

or

log {x2(1 + v2)/c1} = 2 tan–1v

2

+

2 tan x2(1 + v2) = c1e

or

x2[1 + (y2/x2)] = c1e2 tan

or or

...(1)

x & y 1 & ( y / x) dy = . dx x + y 1 + ( y / x) i.e., y = xv. (dy/dx) = v + x(dv/dx). dv 1 + v v+x = 1+ v dx

1 & v + v(1 + v) 1+ v x dv = –v = 1+ v dx 1+ v

or

dz . 2 xz

+1

(y / x)

v

, as v = y/x by (3)

+2 tan +1( y / x )

(x2 + y2) e = c1, c1 being an arbitrary constant. Choosing 1, 1, –1/z as multipliers, each fraction of (1) =

+1

...(5)

dx & dy + (1 / z)dz dx & dy + (1/ z )dz = 0 ( x + y ) & ( x & y ) + (1/ z ) 1 (2 xz )

so that − dx + dy – (1/z)dz = 0 From (5) and (6), the required general solution is

x + y – log z = c2.

...(6)

+1

! (x + y – log z, (x2 + y2) e +2 tan ( y / x) ) = 0, where ! is an arbitrary function. Ex. 12. Solve y2p + x2q = x2y2z2. Sol. Here Lagrange’s auxiliary equations are (dx)/y2 = (dy)/x2 = (dz)/x2y2z2. ...(1) Taking the first two fractions of (1), we have 3x2dx – 3y2dy = 0 so that x3 – y3 = c1. ...(2) Choosing x2, y2, –2/z2 as multipliers, each fraction of (1)

{x2 dx & y 2 dy + (2 / z 2 )dz}/ 0

so that 3x2dx + 3y2dy – (6/z2)dz = 0. 3 Integrating, x + y3 + (6/z) = c2, c2 being an arbitrary constant. From (2) and (3), the required general solution is

...(3)

! [x3 – y3, x3 + y3 + (6/z)] = 0, ! being an arbitrary function. Ex. 13. Solve (3x + y – z)p + (x + y – z)q = 2(z – y). [Bangalore 1992] Sol. Here Lagrange’s auxiliary equations are

dy dx = x&y+z 3x & y + z

dz 2(z + y )

...(1)

Created with Print2PDF. To remove this line, buy a license at: http://www.software602.com/

Linear Partial differential equations of order one

2.13

{dx + 3 dy + dz}/ 0

Choosing 1, –3, 1 as multipliers, each ratio of (1)

so that dx – 3dy – dz = 0. Integrating, x – 3y – z = c1, c1 being an arbitrary constant. From (2), z = c1 – x + 3y. Substituting the above value of z, the first two fractions of (2) reduce to dy dx = 3x & y + (c1 + x & 3y) x & y + (c1 + x & 3y) Let u =4y + c1

dy dx = . 2 x & 4y & c1 4 y & c1 dy = (1/4) × du.

or

so that 1 2x & u or dx + 1 x = 1 , which is linear.. 4 u 4 du 2u

(1 / 4)du dx dx = or u du 2x & u

Then, (3) −

+1/ 2 + (1/ 2 u ) du e +(1/ 2)log u e log (u ) u +1/ 2 Integrating factor of (5) = e .

Hence solution of (5) is

or



2x + u = c2 u

or

...(2) ...(3)

1 = u

z

1 1 du & c 4 u

...(3) ...(4) ...(5)

1/ u .

1 u &c 2 2 2 x + (4 y & c1) = c2, by (4) 4 y & c1

or

2 x + 4 y + (x + 3y + z) = c2, using (2) 4 y & x + 3y + z From (2) and (6), the required general solution is

x+ y&z

or

2

x& y+z

c2 ...(6)

3

! x + 3 y + z , ( x + y & z ) / x & y + z = 0, ! being an arbitrary function. 2

2

2

2

2

2

Ex. 14. Solve x(x + 3y )p – y(3x + y )q = 2z(y – x ). [Delhi Maths Hons 95, 2000] Sol. Here the Lagrange’s auxiliary equations for the given equation are dy dx dz . ...(1) 2 2 2 2 = + y(3x & y ) 2 z( y 2 + x 2 ) x( x & 3y ) Choosing 1/x, 1/y, –1/z as multipliers, each fraction of (1) (1 / x)dx & (1 / y)dy + (1 / z)dz so that 0 Integrating, log x + log y – log z = log c1

1 dx & 1 dy + 1 dz = 0. x y z so that (xy)/z = c1.

=

y (3 x 2 & y 2 ) dy = + dx x( x 2 & 3 y 2 )

Taking the first two ratios of (1), Put

y/x = v

or

Using (4), (3) reduces to or

x

or

v+x

so that

dv 3+ v 2 =–v dx 1 & 3v 2

dv 4(1 + v 2 )v =– dx 1 & 3v 2

or

2

(dy/dx) = v + x(dv/dx). or

x

LM N

...(3) ...(4)

OP Q

3+ v2 dv &1 =–v dx 1 & 3v2 1 + 3v2

dv = 0 4 dx + x v (1 & v 2 )

% (( d v , on resolving into partial fractions ∗ 4 log x + log v + log (1 + v2) or x4v(1 + v2) = c24

4 Integrating,

y = xv

∃ y % 3& 2 y x3 . +∋ ( ) x ∗ 1 & 3 2 y x 32

...(2)

dx ∃ 1 2v &∋ & x ∋) v 1 & v 2

Created with Print2PDF. To remove this line, buy a license at: http://www.software602.com/

2.14

Linear Partial differential equations of order one

x4 ( y / x)[1 & ( y / x)2 ] c24

or

or

z( x2 & y 2 )

or

,

From (2) and (5) solution is ! (z(x2 + y2), xy/z) = 0, ! being an arbitrary function.

c24 / c1

xy ( x 2 & y 2 ) c24

or

c1 z ( x 2 & y 2 ) c24 , by (2)

z ( x2 & y 2 ) c2 , where c2

Ex. 15. Solve (y – z)p + (z – x)q = x – y.

c24 / c1.

... (5)

[Agra 2010; Delhi Maths Hons. 1992]

dx = dy y+z z+x

Sol. Here the Lagrange’s auxiliary equations are

dz . x+y

...(1)

Choosing 1, 1, 1 as multipliers, each fraction of (1) dx & dy & dz dx & dy & dz = = . 0 ( y + z) & (z + x ) & ( x + y) so that x + y + z = c1. , dx + dy + dz = 0 Choosing x, y, z as multipliers, each fraction of (1) x dx & y dy & z dz x ( y + z ) & y ( z + x) & z ( x + y )

, 2x dx + 2y dy + 2z dz = 0

x dx & y dy & z dz 0

x2 + y2 + z2 = c2

so that

, From (2) and (3) solution is !( x & y & z, x2 & y 2 & z 2 )

...(2)

...(3)

0, ! being an arbitrary of function.

Ex. 16. Solve the general solution of the equation (y + zx)p – (x + yz)q + y2 – x2 = 0. [Delhi B.Sc. (Prog) II 2011; GATE 2001; Delhi Math Hons. 1997, 98] Sol. Given (y + zx)p – (x + yz)q = x2 – y2. ...(1) dy dx = y & zx +( x & yz)

Here the Lagrange’s auxiliary equations are

dz . ...(2) x + y2 2

Choosing x, y, –z as multipliers, each fraction of (2) xdx & ydy + zdz xdx & ydy + zdz = = 0 x( y & zx) + y( x & yz) + z( x 2 + y 2 ) so that 2xdx + 2ydy – 2zdz = 0. , xdx + ydy – zdz = 0 Integrating, x2 + y2 – z2 = c1, c1 being an arbitrary constant. Choosing y, x, 1 as multipliers, each fraction of (2) ydx & xdy & dz ydx & xdy & dz = 2 2 = 0 y( y & zx ) + x( x & yz) & x + y or d(xy) + dz = 0. , ydx + xdy + dz = 0 Integrating, xy + z = c2, c2 being an arbitrary constant.

...(3)

...(4)

2 2 2 , The required solution is ! (x + y – z , xy + z) = 0, ! being an arbitrary function. Ex. 17. Solve x(y – z) p + y (z – x)q = z(x – y), i.e., {(y – z)/(yz)}p + {(z – x)/(zx)}q = (x – y)/(xy). [Delhi B.A (Prog) II 2010; I.A.S. 2005, M.S. Univ. T.N. 2007; Vikram 2003] Sol. Given x(y – z)p + y(z – x)q = z(x – y) ... (1)

The Lagrange’s auxiliary equations for (1) are

dx x( y + z )

dy y ( z + x)

dz z( x + y )

... (2)

Choosing 1/x, 1/y, 1/z as multipliers each fraction of (1) (1/ x )dx & (1/ y )dy & (1/ z )dz ( y + z) & ( z + x) & ( x + y )

(1/ x )dx & (1/ y )dy & (1/ z )dz 0

Created with Print2PDF. To remove this line, buy a license at: http://www.software602.com/

Linear Partial differential equations of order one

2.15

so that − (1/x)dx + (1/y)dy + (1/z)dz = 0 log (xyz) = c or , 1 Choosing 1, 1, 1 as multipliers, each fraction of (1)

log x + log y + log z = log c1 xyz = c1 ... (3)

dx & dy & dz ( xy + xz ) & ( yz + yx ) & ( zx + zy ) dx + dy + dz = 0 so that −

dx & dy & dz 0

x + y + z = c2

... (4)

From (3) and (4), solution is !( x & y & z , xyz ) 0, ! being an arbitrary function. Ex. 18. Solve 2y(z – 3)p + (2x – z)q = y(2x – 3) [Delhi Math (H) 1999] Sol. The Lagrange’s auxiliary equations for given equation are dx dy dz ... (1) 2 y ( z + 3) 2 x + z y (2 x + 3) Taking the first and third fractions, (2x – 3)dx = 2(z – 3)dz. Integrating, x2 – 3x = z2 – 6z + C1 or x2 – 3x – z2 + 6z = C1 ... (2) Choosing 1, 2y, –2 as multipliers, each fraction of (1) dx & 2 ydy + 2dz 2 y ( z + 3) & 2 y (2 x + z ) + 2 y (2 x + 3) dx + 2ydy – 2dz = 0 so that ,

dx & 2 ydy + 2dz 0 x + y2 – 2z = C2

From (2) and (3), solution is !( x 2 + 3x + z 2 & 6 z , x & y 2 + 2 z ) function. Ex. 19. Solve x2 (#z / #x) & y 2 (#z / #y)

( x & y) z. 2

x p + y q = (x + y)z

The Lagrange’s auxiliary equations for (1) are

dx

dy

x2

y2

(1/ x )dx & (1/ y )dy + (1/ z )dz x & y + (x & y)

... (1)

dz ( x & y) z

... (2)

(1/x2)dx – (1/y2)dy = 0. 1/y – 1/x = C1 ... (3)

Taking the first two fractions of (2), Integrating, –(1/x) + (1/y) = C1 or Choosing 1/x, 1/y, –1/z as multipliers, each fraction of (2)

, (1/ x)dx & (1/ y )dy + (1/ z )dz

0, ! being an arbitrary [Delhi Maths (H) 2001]

2

Sol. Re-writing the given equation

... (3)

(1/ x )dx & (1/ y )dy + (1/ z )dz 0

0

xy/z = C2

so that

... (4)

From (3) and (4), solution is 5(1/ y + 1/ x, xy / z ) 0, 5 being an arbitrary function. Ex. 20. Solve z(x + 2y) p – z(y + 2x) q = y2 – x2 [Vikram 1999] Sol. The Lagrange’s subsidiary equations are

dx z( x & 2 y )

dy + z ( y & 2 x)

dz 2

y + x2

... (1)

Taking the first two fraction of (1), we have (y + 2x) dx + (x + 2y) dy = 0 or 2xdx + 2y dy + d (xy) = 0 Integrating, x2 + y2 + xy = C1, C1 being an arbitrary constant ... (2) Choosing x, y, z as multipliers, each fraction of (1) xdx & ydy & zdz 2

2

2

2

( x z & 2 xyz ) + ( y z & 2 xyz ) & ( zy + zx )

xdx & ydy & zdz 0

Created with Print2PDF. To remove this line, buy a license at: http://www.software602.com/

2.16

Linear Partial differential equations of order one

− 2x dx + 2y dy + 2z dz = 0

x2 + y2 + z2 = C2

so that

From (2) and (3), solution is !( x 2 & y 2 & z 2 , x2 & y 2 & xy)

... (3)

0, ! being an arbitrary function

EXERCISE 2(C) Solve the following partial differential equations: 1. x(y2 – z2)p + y(z2 – x2)q = z(x2 – y2)

Ans. !( x2 & y 2 & z 2 , xyz )

0

[Mysore 2004, Delhi B.Sc. (Prog). II 2007, M.S. Unit. T.N. 2007] 2. z(xp – yq) = y2 – x2

Ans. !( x 2 & y 2 & z 2 , xy)

0

Ans. !( x2 & y 2 & z 2 , y / z )

0

4. yp – xq = 2x – 3y [M.S. Univ. T.N. 2007]

Ans. !( x 2 & y 2 , 3x & 2 y & z )

0

5. x2(y – z)p + y2(z – x)q = z2(x – y)

Ans. !( xyz , 1/ x & 1/ y & 1/ z )

0

3. ( y 2 & z 2 ) p + xyq & xz

0 [I.A.S. 1990]

[Meerut 2007, Bilaspur 2004, Rewa 2003] 2.11. Type 4 based on Rule IV for solving (dx)/P = (dy)/Q = (dz)/R. ...(1) Let P1, Q1 and R1 be functions of x, y and z. Then, by a well–known principle of algebra, ( P1dx & Q1dy & R1dz ) /( P1 P & Q1Q & R1 R ) .

each fraction of (1) will be equal to

...(2)

Suppose the numerator of (2) is exact differential of the denominator of (2). Then (2) can be combined with a suitable fraction in (1) to give an integral. However, in some problems, another set of multipliers P2, Q2 and R2 are so chosen that the fraction ( P2 dx & Q2 dy & R2 dz ) /( P2 P & Q2 Q & R2 R )

...(3)

is such that its numerrator is exact differential of denominator. Fractions (2) and (3) are then combined to given an integral. This method may be repeated in some problems to get another integral. Sometimes only one integral is possible by using the above rule IV. In such cases second integral should be obtained by using rule 1 of Art. 2.5 or rule 2 of Art. 2.7 or rule 3 of Art. 2.9. 2.12. SOLVED EXAMPLES BASED IN ART. 2.11 Ex. 1. Solve (y + z)p + (z + x)q = x + y. [Indore 2000; Jabalpur 2000, Jiwaji 2002, Kanpur 2008; Purvanchal 2007, Ravishankar 2002, 2005; Delhi BA (Prog.) II 2011] Sol. Here the Lagrange’s auxiliary equations are

dx = dy y&z z& x

Choosing 1, –1, 0 as multipliers, each fraction of (1) =

dx + dy ( y & z) + ( z & x )

Again, choosing 0, 1, –1 as multipliers, each fraction of (1) =

dz . x&y

...(1)

d( x + y ) . +( x + y)

dy + dz ( z & x ) + ( x & y)

...(2)

d ( y + z) . ...(3) +( y + z )

Finally, choosing 1, 1, 1 as multipliers, each fraction of (1) dx & dy & dz d ( x & y & z) = . ( y & z) & (z & x ) & ( x & y) 2( x & y & z) d( x + y) d ( y + z) d ( x & y & z) (2), (3) and (4) = . − +( x + y ) + ( y + z) 2 ( x & y & z)

...(4) ...(5)

d( x + y ) d( y + z) = . x+y y+z log (x – y) = log (y – z) + log c1, c1 being an arbitrary constant.

Taking the first two fractions of (5), Integrating,

Created with Print2PDF. To remove this line, buy a license at: http://www.software602.com/

Linear Partial differential equations of order one

or

log {(x – y)/(y – z)} = log c1

2.17

(x – y)/(y – z) = c1.

or

Taking the first and the third fractions of (5),

d( x + y ) d ( x & y & z ) + =0 x& y&z ( x + y) or (x – y)2 (x + y + z) = c2.

...(6)

2

Integrating, 2 log (x – y) + log (x + y + z) = log c2 From (6) and (7), the required general solution is

...(7)

! [(x – y)2(x + y + z), (x – y)/(y – z)] = 0, ! being an arbitrary function. Ex. 2. Solve y2(x – y)p + x2(y – x)q = z(x2 + y2) [Delhi Maths Hons 1997; Nagpur 2010] Sol. Here the Lagrange’s auxiliary equations for the given equation are dy dz dx = . ...(1) 2 2 2 + x ( x + y) z ( x & y 2 ) y ( x + y) Taking the first two fractions of (1), x2dx = –y2dy or 3x2dx + 3y2dy = 0. Integrating, x3 + y3 = c1, c1 being an arbitrary as constant. ...(2) Choosing 1, –1, 0 as multipliers, each fraction of (1) dx + dy dx + dy . y 2 ( x + y) & x 2 ( x + y) ( x + y) ( x 2 & y 2 ) Combining the third fraction of (1) with fraction (3), we get

=

...(3)

d(x + y) dz + = 0. x+y z

dx + dy dz = or ( x + y)( x 2 & y2 ) z ( x 2 & y2 ) Integrating, log (x – y) – log z = log c2

(x – y)/z = c2.

or

...(4)

From (3) and (4), solution is ! (x + y , (x – y)/z) = 0, ! being an arbitrary function. 3

3

Ex. 3. Solve (x2 – y2 – z2)p + 2xyq = 2xz or (y2 + z2 – x2)p – 2xyq = –2xz. [Bangalore 1993, I.A.S. 1973; P.C.S. (U.P.) 1991; Bhopal 2010] Sol. Here the Lagrange’s auxiliary equations for the given equation are dy dx dz . ...(1) 2 2 2 = + 2 xy +2 xz y &z +x Taking the last two fractions of (1), we have (1/y)dy = (1/z)dz so that (1/y)dy – (1/z)dz = 0. Integrating, log y – log z = log c1 or y/z = c1. ...(2) Choosing x, y, z as mnultipliers, each fraction of (1) =

x dx & y dy & z dz

x dx & y dy & z dz

xy 2 & xz 2 + x3 + 2 xy 2 + 2 xz 2

+x (x2 & y 2 & z2 )

.

...(3)

Combining the third fraction of (1) with fraction (3), we have x dx & y dy & z dz 2

2

2

+x (x & y & z )

= 2

dz + 2 xz 2

2 x dx & 2 y dy & 2 z dz

or

2

2

x &y &z

2

Integrating, log (x + y + z ) – log z = log c2

or

2

+

dz = 0. z

(x2 + y2 + z2)/z = c2.

... (4)

From (2) and (4) solution is ! (y/z, (x + y + z )/z) = 0, ! being an arbitrary function. Ex. 4. Solve (1+ y)p + (1 + x)q = z. [M.S. Univ. T.N. 2007; Kanpur 2011] 2

2

2

Sol. Here the Lagrange’s auxiliary equations are Taking the first two fractions of (1), we have (1 + x)dx = (1 + y)dy or

dx = dy 1& y 1& x

dz . z

...(1)

2(1 + x)dx – 2(1 + y)dy = 0.

Created with Print2PDF. To remove this line, buy a license at: http://www.software602.com/

2.18

Linear Partial differential equations of order one 2

2

Integrating, (1 + x) – (1 + y) = c1, c1 being an arbitrary constant.

...(2)

dx & dy d(2 & x & y) = . 1& y & 1& x 2&x&y Combining the last fraction of (1) with fraction (3), we get

=

Taking 1, 1, 0 as multipliers, each fraction of (1)

d(2 & x & y) dz = or z 2&x&y Integrating, log (2 + x + y) – log z = log c2 or From (2) and (4), the required general solution is given by

...(3)

d(2 & x & y) dz + = 0. 2&x&y z

(2 + x + y)/z = c2. ...(4)

! [(1 + x)2 – (1 + y2), (2 + x + y)/z] = 0, ! being an arbitrary function. Ex. 5. Find the general integral of xzp + yzq = xy. Sol. Here the Lagrange’s auxiliary equations are From the first two fractions of (1), Integrating, log x = log y + log c1

(dx)/xz = (dy)/yz = (dz)/xy ... (1) (1/x)dx = (1/y)dy. or x/y = c1. ...(2)

(1 / x)dx & (1 / y)dy ydx & xdy = ...(3) 2 xyz (1 / x)xz & (1 / y)yz Combining the last fraction of (1) with fraction (3), we have

Choosing 1/x, 1/y, 0 as multipliers, each fraction of (1) = ydx & xdy = dz xy 2 xyz Integrating,

or

ydx + xdy = 2zdz

d(xy) = 2zdz

or

or

d(xy) – 2zdz = 0

xy – z2 = c2, c2 being an arbitrary constant.

...(4)

From (2) and (4) solution is ! (x/y, xy – z ) = 0, ! being an arbitrary function. 2

Ex. 6. Solve (x2 – yz)p + (y2 – zx)q = z2 – xy. Delhi Math (H) 2005, 11, M.D.U. Rohtak 2005; Agra 2008, 09; Guwahati 2007; Meerut 2006; Sagar 2000; Ravishankar 2000; Lucknow 2010] dx = dy y 2 + zx x 2 + yz Choosing 1, –1, 0 and 0, 1, –1 as multipliers in turn, each fraction of (1)

Sol. Here the Lagrange’s auxiliary equations are

dz . ...(1) z 2 + xy

dy + dz dx + dy = 2 ( y + z)( y & z & x) x + y & z( x + y) dx + dy d( x + y) d( y + z) dy + dz + so that = or = 0. ( x + y)( x & y & z) x+y y+ z ( y + z)( y & z & x) Integrating, log (x – y) – log (y – z) = log c2 or (x – y)/(y – z) = c1. ...(2) Choosing x, y, z as multipliers, each fraction of (1)

=

2

xdx & ydy & zdz xdx & ydy & zdz = . 3 3 x & y & z + 3xyz ( x & y & z)( x 2 & y 2 & z 2 + xy + yz + zx) Again, choosing 1, 1, 1 as multipliers, each fraction of (1)

=

3

dx & dy & dz . x 2 & y 2 & z 2 + xy + yz + zx xdx & ydy & zdz From (3) and (4), = dx + dy + dz x&y&z 2(x + y + z) d(x + y + z) – (2xdx + 2ydy + 2zdz) = 0. Integrating, (x + y + z)2 – (x2 + y2 + z2) = 2c2 2 2 (x + y + z2 + 2xy + 2yz + 2zx) – (x2 + y2 + z2) = 2c2 xy + yz + zx = c2, c2 being an arbitrary constant.

=

or or or

...(3)

...(4)

...(5)

Created with Print2PDF. To remove this line, buy a license at: http://www.software602.com/

Linear Partial differential equations of order one

2.19

From (2) and (5), the required general solution is given by

! [xy + yz + zx, (x – y)/(y – z)] = 0, ! being an arbitrary function. Ex. 7. Solve (x2 – y2 – yz)p + (x2 – y2 – zx)q = z(x – y). Sol. Here Lagrange’s auxiliary equations for the given equation are dy dx dz = 2 2 x + y + zx z( x + y) x 2 + y2 + yz Choosing 1, –1, 0 as multipliers, each fraction of (1) dx + dy dx + dy = 2 2 = . 2 2 z ( x + y) ( x + y + yz) + ( x + y + zx ) Choosing x, –y, 0 as multipliers each fraction of (1) xdx + ydy xdx + ydy = = . x( x 2 + y 2 + yz) + y( x 2 + y2 + zx ) ( x + y)( x 2 + y 2 ) From (1), (2), (3) we have dx + dy xdx + ydy = z( x + y) ( x + y)( x 2 + y 2 )

dz z ( x + y)

dz z

or

Integrating, log (x – y ) – 2 log z = c2

...(3)

z – x + y = c1

...(5)

d ( x 2 + y 2 ) / ( x 2 + y 2 ) + (2 / z) dz

Again , taking the first and third fractions of (4), 2

...(2)

2 xdx + 2 ydy dx + dy = . ...(4) z 2( x 2 + y 2 )

Taking the first two fractions of (4), we have dz = dx – dy so that 2

...(1)

2

or

2

2

(x – y )/z = c2.

0 ...(6)

From (5) and (6), solution is ! (z – x + y, (x + y )/z ) = 0, ! being an arbitrary function. 2

2

2

Ex. 8. Solve (x2 + y2 + yz)p + (x2 + y2 – xz)q = z(x + y). dy dx = x 2 & y2 & yz x 2 & y2 + xz Choosing 1, –1, 0 as multipliers, each fraction of (1) dx + dy dx + dy = 2 2 = . z( x & y) ( x & y & yz) + ( x 2 & y 2 + xz) Choosing x, y, 0 as multipliers, each fraction of (1)

Sol. Here the Lagrange’s, auxiliary equations are

dz . z( x & y)

...(2)

xdx & ydy xdx & ydy = . x( x 2 & y 2 & yz) & y( x 2 & y2 + xz) ( x & y)( x 2 & y 2 ) From (1), (2) and (3), we have dx + dy xdx & ydy xdx & ydy dz dx + dy dz = or = 2 2 . z z z( x & y) z(x & y) ( x & y)( x 2 & y 2 ) x &y Taking the first two fractions of (4), we have dz = dx – dy or dz – dx + dy = 0. Integrating, z – x + y = c1, c1 being an arbitrary constant. Taking the first and third fractions of (4), we have

=

2 xdx & 2 ydy 2

x &y

2

Integrating,

dz =2 z

d (x2 & y2 )

or 2

2

x &y

2

log (x + y ) – 2log z = log c2

or

2

+2

...(1)

...(3)

...(4)

...(5)

dz = 0. z

(x2 + y2)/z2 = c2.

...(6)

From (5) and (6), solution is ! (z – x + y, (x + y )/z ) = 0, ! being an arbitrary function. 2

2

2

Created with Print2PDF. To remove this line, buy a license at: http://www.software602.com/

2.20

Linear Partial differential equations of order one

Ex. 9. Solve cos (x + y)p + sin (x + y)q = z. [Garhwal 2010, Vikram 1998; Meerut 2007; Delhi Maths (H) 2007; Rajasthan 1994; Delhi B.A./B.Sc. (Prog.) Maths 2007] dy dx = cos (x & y) sin( x & y)

Sol. Here the Lagrange’s auxiliary equations are

dz . z

...(1)

Choosing 1, 1, 0 as multipliers, each fraction of (1) dx & dy d( x & y) = = . cos( x & y) & sin ( x & y) cos( x & y) & sin ( x & y) =

Choosing 1, –1, 0 as multipliers, each fraction of (1)

dx + dy . cos( x & y) + sin (x & y)

...(3)

dx + dy d( x & y) = . ...(4) cos( x & y) & sin ( x & y) cos( x & y) + sin (x & y)

dz z

From (1), (2) and (3),

...(2)

d( x & y) dz = . ...(5) cos( x & y) & sin ( x & y) z

Taking the first two fractions of (4), Putting x + y = t so that d(x + y) = dt, (5) reduces to dz z

dt = cos t & sin t

2

{d

1

dt 2 cos t & 1

i

d

i }

dt = 2 sin(6 / 4)cos t & cos(6 / 4)sin t

l

q

dt 2 sin (t & 6 / 4)

( 2 / z )dz = cosec (t & 6 / 4) dt.

Thus, Integrating,

FH IK or F x & y & 6 I = c . as t = x + y cot H K

1 6 2 log z = log tan 2 t & 4 + log c1,

or

z

2

2

z

2

= c1 tan

FH t & 6 IK 2 8 ...(6)

1

8

cos (x & y) + sin ( x & y) d(x + y). ...(7) cos ( x & y) & sin ( x & y) On R.H.S. of (7), putting x + y = t, so that d(x + y) = dt, (7) reduces to cos t + sin t dx – dy = cos t & sin t dt. so that x – y = log (sin t + cos t) – log c2 (sin t + cos t)/c2 = ex – y or e–(x – y) (sin t + cos t) = c2 y–x e [sin (x + y) + cos (x + y)] = c2, as t = x + y. ...(8) From (6) and (8), the required general solution is

Taking the last two fraction of (4),

or or

=

2 sin t

LM N

! z

2

cot

dx – dy =

FG x & y & 6 IJ , e lsin ( x & y) & cos( x & y)qOP = 0, where ! is an arbitrary function. H 2 8K Q y+x

Ex. 10. Solve cos (x + y)p + sin (x + y)q = z + (1/z). Sol. Do like Ex. 9.

LM N

Ans. ! (z2 & 1)1/

2

F H

I K

[Delhi B.A. (Prog.) 2011]

qOPQ = 0

l

x & y y+ x tan 36 + ,e cos( x & y) & sin ( x & y) 8 2

Ex. 11. Solve xp + yq = z – a ( x 2 & y 2 & z 2 ) . Sol. Here the lagrange’s auxiliary equations are

[Meerut 1997; Jiwaji 1997; Rawa 1999] dx = dy x y

dz 2

z + a ( x & y2 & z 2 )

.

...(1)

Taking the first two fractions of (1), we have

Created with Print2PDF. To remove this line, buy a license at: http://www.software602.com/

Linear Partial differential equations of order one

(1/x)dx = (1/y)dy Integrating,

2.21

or log x – log y = log c1

(1/x)dx – (1/y)dy = 0. x/y = c1.

or

Choosing x, y, z as multipliers, each fraction of (1) =

xdx & ydy & zdz 2

2

x & y & z 2 + az ( x 2 & y 2 & z 2 )

Combining first and third fractions of (1) with fraction (3), we get xdx & ydy & zdz dx dz = 2 2 2 . x z + a ( x 2 & y2 & z2 ) x & y & z + az ( x 2 & y 2 & z 2 ) Putting x2 + y2 + z2 = t2 so that dx dz tdt = 2 x z + at t + azt

...(2) ...(3)

...(4)

xdx + ydy + zdz = tdt, (4) gives dx dz dt or = . x z + at t + az

...(5)

dz & dt d(z & t) = . ...(6) (z & t ) + a (t & z) (1 + a)(z & t ) Combining the first fraction of (5) with fraction (6), we get

Choosing 0, 1, 1 as multipliers, each fraction of (5) =

d( z & t ) dx d(z & t ) dx or (1 – a) + = 0. x z&t x (1 + a)(z & t ) Integrating, (1 – a) log x – log (z + t) = log c2, c2 being an arbitrary constant.

or

x a +1 = c2 z &t

or

x a +1 2

2

2

z & (x & y & z )

= c2,

as

t = (x2 + y2 + z2)1/2

...(7)

From (2) and (7), the required general solution is

! [xa – 1/{z + ( x 2 & y 2 & z 2 ) }, x/y] = 0, ! being an arbitrary function. Ex. 12. Solve (x3 + 3xy2)p + (y3 + 3x2y)q = 2z(x2 + y2).

[I.A.S. 1993]

dy dx = 3 x 3 & 3xy2 y & 3x 2 y

dz . 2 z( x 2 & y 2 )

...(1)

Choosing 1, 1, 0 as multipliers, each fraction of (1) =

dx & dy d( x & y) = . x 3 & 3xy 2 & 3x 2 y & y 3 ( x & y)3

...(2)

Choosing 1, –1, 0 as multipliers, each fraction of (1) =

dx + dy d(x + y) = . x & 3xy 2 + y 3 + 3x 2 y (x + y)3

...(3)

Sol. Here the Lagrange’s subsidiary equations are

3

(x + y)–3 d(x + y) = (x – y)–3 d(x – y) u–3du – v–3dv = 0, on putting u = x + y and v = x – y. –2 –2 Integrating, u /(–2) – v /(–2) = c1/2 or v–2 – u–2 = c1 –2 –2 (x – y) – (x + y) = c1, as u = x + y and v = x – y. ...(4) Choosing 1/x, 1/y, 0 as multipliers, each fraction of (1) From (2) and (3),

or or

=

21 x3 1 ( x

3

21 x 3 dx & 21 y 3 dy & 3xy 2 ) & 21 y 3 1 ( y 3 & 3x 2 y )

=

b1 xgdx & b1 ygdy . 4 ( x 2 & y2 )

...(5)

Combining the last fraction of (1) with fraction (5), we have

b g b g

1 x dx & 1 y dy dz = 2z ( x 2 & y2 ) 4( x 2 & y2 )

Integrating,

log x + log y – 2 log z = log c2

dx & dy + 2 dz = 0. x y z

or or

(xy)/z2 = c2.

...(6)

Created with Print2PDF. To remove this line, buy a license at: http://www.software602.com/

2.22

Linear Partial differential equations of order one

From (4) and (6), the required general solution is given by

! [(x – y)–2 – (x + y)–2, (xy)/z2] = 0, ! being an arbitrary function. Ex. 13. Solve p + q = x + y + z. [Bhopal 2010, Bilaspur 2000, 02; I.A.S. 1975; Gulberge 2005] Sol. Here Lagrange’s auxiliary equations are Taking the first two fractions of (1),

dx – dy = 0

Choosing 1, 1, 1 as multipliers, each fraction of (1) =

dy dx dz = . ...(1) 1 1 x&y&z so that x – y = c1. ...(2)

d(2 & x & y & z) dx & dy & dz = 2& x & y&z 1 & 1 & ( x & y & z)

Combining the first fraction of (1) with fraction (3), d (2 & x & y & z ) /(2 & x & y & z )

... (3)

dx.

Integrating, log (2 + x + y + z) – log c2 = x or (2 + x + y + z)/c2 = ex –x or e (2 + x + y + z) = c2, c2 being arbitrary function ...(4) From (2) and (4), the required general solution is ![x – y, e–x(2 + x + y + z)] = 0, ! being an arbitrary function. Ex. 14. Solve (2x2 + y2 + z2 – 2yz – zx – xy)p + (x2 + 2y2 + z2 – yz – 2zx – xy)q = x2 + y2 2 + 2z – yz – zx – 2xy. [Meerut 1996 ; I.A.S. 1992] Sol. Here Lagrange’s auxiliary equations are dy dx dz = 2 = . ...(1) 2 x 2 & y 2 & z 2 + 2 yz + zx + xy x & 2 y 2 & z 2 + yz + 2 zx + xy x 2 & y 2 & 2z 2 + yz + zx + 2 xy Choosing 1, –1, 0 ; 0, 1, –1 and –1, 0, 1 as multipliers in turn, each fraction of (1) dx + dy dy + dz dz + dx = 2 2 = 2 2 2 x + y + yz & zx y + z + zx & xy z + x 2 + xy & yz dx + dy dy + dz dz + dx = . ...(2) , ( x + y) ( x & y & z) ( y + z) ( x & y & z) (z + x) ( x & y & z) Taking the first two fractions of (2), we have (dx – dy)/(x – y) – (dy – dz)/(y – z) = 0. Integrating, log (x – y) – log (y – z) = log c1 or (x – y)/(y – z) = c1. ...(3) Taking the last two fractions of (2), (dy – dz)/(y – z) – (dz – dx)/(z – x) = 0. Integrating, log (y – z) – log (z – x) = log c2 or (y – z)/(z – x) = c2. ...(4) From (3) and (4), the required general solution is ![(x – y)/(y – z), (y – z)/(z – x)] = 0, ! being an arbitrary function. Ex. 15. Find the general solution of the partial differential equation px(x + y) – qy(x + y) + (x – y) (2x + 2y + z) = 0. [Delhi B.Sc. II (Prog) 2009; Delhi Maths Hons. 2006, 09, 11] Sol. Given x(x + y)p – y(x + y)q = – (x – y) (2x + 2y + z). ...(1) dy dz dx = . ...(2) + y (x & y) + ( x + y) (2 x & 2 y & z) x ( x & y) Taking the first two fractions, (1/x)dx = – (1/y)dy or (1/x)dx + (1/y)dy = 0. Integrating, log x + log y = log c1 or xy = c1. ...(3) Again, each fraction of (2) dx & dy dx & dy & dz = = x( x & y) + y( x & y) x( x & y) + y( x & y) + ( x + y) (2 x & 2 y & z) dx & dy dx & dy & dz = = ( x + y)( x & y) ( x + y)( x & y) + ( x + y)(2 x & 2 y & z)

Lagrange’s auxiliary equations are

Created with Print2PDF. To remove this line, buy a license at: http://www.software602.com/

Linear Partial differential equations of order one

2.23

dx & dy dx & dy & dz dx & dy & dz + = ( x & y) x & y + (2 x & 2 y & z ) x&y& z

Thus,

dx & dy & dz dx & dy + = 0, so that log (x + y) + log (x + y + z) = log c2 x& y&z x&y (x + y) (x + y + z) = c2, c2 being an arbitrary constant. ...(4) From (3) and (4), solution is ![xy, (x + y) (x + y + z)] = 0, ! being an arbitrary function.

Thus, or

Ex. 16. Solve {my( x & y) + nz 2} (#z / #x) + {lx( x & y) + nz2 } (#z / #y)

(lx + my)z [I.A.S. 2001]

Sol. Re-writing the given equation, {my( x & y) + nz2 } p + {lx( x & y) + nz 2}q Lagrange’s auxiliary equations for (1) are Each fraction of (2) =

dx & dy (my + lx )( x & y)

dx

dy

my( x & y) + nz

2

dz +(my + lx )z

+lx( x & y) & nz

2

2

lx 2 & my2 & nz 2

From (3) and (4), solution is 5( xz & yz, lx2 & my2 & nz2 )

dx

dy

dz

y (z + y2 + 2 x 2 )

z( z + y 2 + 2 x 2 )

2

2

y/z

so that d ( z + y2 )

+2 ydy & dz 2

z(z + y2 + 2 x2 ) ... (1)

x ( z + 2 y2 )

Taking the last two fractions, (1/ y) dy + (1/ z) dz 0 Taking 0, – 2y, 1 as multipliers, each fraction of (2) =

...(4)

C2

[Delhi Maths (H) 2002]

x(z + 2 y2 ) p & y( z + y2 + 2 x 2 )q

Lagrange’s auxiliary equations for (1) are

2

2

2

( z + 2 y )( z + y2 + 2 x 2 )

+2 y (z + y + 2 x ) & z( z + y + 2 x )

dz z

0, 5 being an arbitrary function.

Ex. 17. Solve px ( z + 2 y2 ) (z + qy) ( z + y2 + 2 x 2 ) . Sol. Re-writing the given equation

+

lx dx & my dy & nz dz 0

=

so that

0

...(2)

(x + y)z = C1 ... (3)

or

lx my ( x & y) + lx nz + mylx( x & y) & mynz & nz (lx + my)

, 2lx dx & 2my dy & 2nzdz

d ( x & y) x&y

so that

lxdx & mydy & nzdz 2

dz (lx + my)z

2

Integrating, log (x + y) = – log z + log C1 Taking lx, my, nz as multipliers, each fraction of (2) =

(lx + my)z ...(1)

C1

...(2) ...(3)

... (4)

Combining fraction (4) with first fraction of (2), we get dx

d ( z + y2 )

x (z + 2y 2 )

( z + 2 y2 ) ( z + y2 + 2 x 2 )

du/dx = (u – 2x2)/x, taking z + y2

or

(du / dx) + (1/ x) u

or

whose I.F. = e u.

1 x

.

.

+ (1/ x ) dx

d ( z + y2 ) dx

or

z + y2 + 2 x 2 x

... (5)

u

+2x which is an ordinary linear differential equation

e+ log x

∃1% (+2 x ) ∋ ( dx & C2 )x∗

elog x

+1

x +1 1/ x and solution is or

z + y2 x

+2 x & C2 , using (5)

Created with Print2PDF. To remove this line, buy a license at: http://www.software602.com/

2.24

or

Linear Partial differential equations of order one

or ( z + y2 ) / x & 2 x C2 From (3) and (6), the required general solution of (1)

5( y / z, ( z + y2 + 2 x 2 ) / x)

( z + y2 & 2 x2 ) / x C2

... (6)

0 , 5 being an arbitrary function.

Ex. 18. Solve px( z + 2 y2 ) (z + qy) ( z + y2 + 2 x 3 ) .

[I.A.S. 2006] Ans. 5( y / z, (z + y2 & x 3 ) / x)

Sol. Do like Ex. 17,

0

For another method of solution, refer solved Ex. 8 of Art. 2.8. Ex. 19. Solve x (z & 2a) p & ( xz & 2 yz & 2ay)q

z( z & a) .

Sol. The Lagrange’s auxiliary equations for given equation are dx x ( z & 2 a)

dx & dy 2( x & y)( z & a)

Each fraction of (1) = Integrating,

dy xz & 2 yz & 2ay dz z( z & a)

log (x + y) = 2 log z + log C1

Taking the first and third ratios of (4),

dz z( z & a)

d (x & y) x& y

or

( x & y ) / z2

or

z & 2a dz z( z & a)

dx x

... (1)

Integrating, log x = 2 log z – log (z + a) + log C2

dx x

or

x(z & a) / z2

or

2 dz z ...(2)

C1

1 % ∃2 ∋ z + z & a ( dz ) ∗

... (3)

C2

From (2) and (3), solution is 5 {( x & y) / z2 , x( z & a) / z2 )} 0 . ! being an arbitrary function. Ex. 20. Solve 2 x ( y & z2 ) p & y(2 y & z 2 )q

z3

[Delhi Maths (Hans.) 2007]

Sol. The Lagrange’s auxiliary equations for the given equation are dx

dy 2

Each fraction of (1) =

2 x( y & z )

y(2 y & z )

dx

z dy & y dz 2

2 x( y & z )

, (1/ x) dx & (1/ yz) d ( yz)

Its I.F. = e. or

(1/ z ) dz

elog z

+ y+1z + 2z +1

2 yz( y & z2 )

2 yz( y & z )

dy dz

y(2 y & z2 ) z

3

x /( yz) C1

2 y2 z

3

&

y dy 1 +1 or y +2 + y z dz z

...(2)

2 z3

...(3)

du / dz in (3), we get

2 / z 3 , which is an ordinary linear equation.

z and solution is u z

+ C2

d ( yz)

so that

u and (1/ y2 ) 1 (dy / dz)

(du / dz) & (1/ z) u

...(1)

z3

2

0

From the last two fractions of (1), Putting + y+1

dz 2

or

From (3) and (4), solution is 5 ( x / yz, z / y + 2 / z) Ex. 21. xp + zq + y = 0. Sol. Given equation is

. (2 / z ) z dz + C 3

2

+2 z +1 + C2

z / y + 2 / z C2

... (4)

0 , ! being arbitrary function. [M.D.U. Rohtak 2004] xp + zq = – y

Created with Print2PDF. To remove this line, buy a license at: http://www.software602.com/

Linear Partial differential equations of order one

2.25

dx x

Its Lagrange’s auxiliary equation are Taking the last two fractions of (2), 2ydy + 2zdz = 0 Choosing 0, z, –y as multipliers, each fraction of (1) 2

z &y

2

1 & ( y / z)

2

dz +y

... (1)

y2 + z2 = C1 ... (2)

so that

(1/ z )dy + ( y / z 2 )dz

zdy + ydz

dy z

d ( y / z)

... (3)

1 & ( y / z)2

Combining the first faction of (1) with fraction (3), we get dx x

d ( y / z) 1& ( y / z)

dx y% ∃ + d ∋ tan +1 ( x z∗ )

or

2

0

Integrating, log | x | – tan–1 (y/z) = C2, c2 being an arbitrary constant. From (2) and (4), the required general solution is

log | x | + tan +1 ( y / z ) Ex. 2

22.

Find

2

x (#z / #x) & y (#z / #y) Sol. Let p

the

general

!( y 2 & z 2 ), ! being an arbitrary function.

solution

( x & y) z.

#z / #x and q

... (4)

of

the

differential

equation

[Delhi B.A./B.Sc. (Prog.) Maths 2007) #z / #y. Then, the given equation takes the form

x2p = y2q = z (x + y) The Lagrange’s auxiliary equations for (1) are (dx)/x2 = (dy)/y2 = (dz)/z (x + y) Taking the first two fractions of (2), (1/x2)dx – (1/y2)dy = 0 Integrating, – (1/x) + (1/y) = c1 or (x – y)/xy = c1 dx + dy

Chossing 1, –1, 0 as multipliers, each fraction of (2)

... (1) ... (2) ... (3) ... (4)

x2 + y 2

Combining the last fraction of (2) with fraction (4), we have

dx + dy ( x + y )( x & y )

dz z( x & y)

dx + dy dz + x+ y z

or

Integrating, log (x – y) – log z = sin c2 From (5), x – y = c2 z using (6), (3) becomes (c2z)/xy = a or From (5) and (7), the required solution is

or

0

(x – y)/z = c2

... (5) ... (6) (xy)/z = c2/c1 = c3 say ... (7)

! 2 ( x, y ) / z , ( x + y ) / z 3

0.

EXERCISE 2(D) Solve the following partial differential equations: 1. (x2 + y2)p + 2xy q = z(x + y) 2. {y (x + y) + az} p + {x (x + y) – az}q = z(x + y) 3. (y2 + yz + z2)p + (z2 + zx + x2)q = x2 + xy + y2

Ans. ( x & y ) / z Ans. ( x & y) / z

2

! y /( x 2 + y 2 )

3

!( x2 + y 2 + 2az )

∃ y+z x+z % , ( Ans. ! ∋ ) x+ y x+ y∗

0

Created with Print2PDF. To remove this line, buy a license at: http://www.software602.com/

2.26

Linear Partial differential equations of order one

2.13. Miscellaneous Examples on Pp + Qq = R Ex. 1. Solve (x + y – z) (p – q) + a (px – qy + x – y) = 0. Sol. Let u=x+y and Then and

#z #z #u & #z #v #z & #z = , using (1) #x #u #x #v #x #u #v #z #z #u #z #v #z #z q = #y = #u #y & #v #y #u + #v , using (1)

p=

p+q

From (2) and (3), we get and or

v = x – y.

2(#z / #v ).

#z & x #z + y #z & y #z px – qy = x #u #v #u #v #z #z #z #z px – qy = (x – y) + (x + y) =v + u ,using (1) #u #v #u #v Using (1), (4) and (5), the given equation reduces to

FH

...(2) ...(3) ...(4)

...(5)

IK

#z + a v #z & u #z & v = 0 #v #u #v or av(#z/#u) + (2u – 2z + au)(#z/#v) = – av, which is Lagrange’s linear equation. Its Lagrange’s auxiliary equations are

2(u – z)

...(1)

...(6)

du dv dz . = ...(7) av 2u + 2z & au +av Taking the first and third fractions of (7), we have du + dz = 0 so that u + z = c1. ...(8) Considering the first two fractions of (7) and eliminating z with help of (8), we have dv du = or avdv = (4u – 2c1 + au)du. 2u + 2(c1 + u) & au av Integrating, (1/2) × av2 = 2u2 – 2c1u + (1/2) × au2 + c2/2 2 2 av = 4u – 4u(u + z) + au2 + c2, or av2 + 4uz – au2 = c2 (using (8) ....(9) From (8) and (9), the required general solution is given by !(u + z, av2 + 4uz – au2) = 0, where ! is an arbitrary function and u and v are given by (1). Ex. 2 (a). Find the surface whose tangent planes cut off an intercept of constant length k from the axis of z. (b) Formulate partial differential equation for surfaces whose tangent planes form a tetrahedrom of constant volume with the coordinate planes. [I.A.S. 2005] Sol. (a) We know that the equation of the tangent plane at point (x, y, z) to a surface is given by p(X – x) + q(Y – y) = Z – z, ...(1) where X, Y, Z denote current coodrinates of any point on the plane (1). Since (1) cuts an intercept k on the z–axis, it follows that (1) must pass through the point (0, 0, k). Hence putting X = 0, Y = 0 and Z = k in (1), we obtain px + qy = z – k, ...(2) which is well known Lagrange’s linear equation. For (2), the Lagrange’s auxiliary equations are (dx)/x = (dy)/y = (dz)/(z – x). ...(3) Taking the first two fractions of (3), (1/x)dx – (1/y)dy = 0. so that x/y = c1. ...(4) Again, taking the first and third fraction of (3), [1/(z – k)]dz – (1/x)dx = 0 Integrating, log (z – k) – log x = log c2 or (z – k)/x = c2. ...(5) From (4) and (5), the required surface (solution) is given by ![y/x, (z – k)/x] = 0, ! being an arbitrary function. (b) Left as an exercise.

Created with Print2PDF. To remove this line, buy a license at: http://www.software602.com/

Linear Partial differential equations of order one

2.27

EXERCISE 2 (E) Solve the following partial differential equations : 1. p – qy log y = z log y. Ans. !(yz, ex log y) = 0 2. (p + q)(x + y) = 1. Ans. !(y – x, e–2zy + x) = 0 2 2 3. x p + y q = x + y. Ans. ![(1/y) – (1/x), e–z(x – y)] = 0 4. (x2 + 2y2)p – xyq = xz. Ans. !(x2y2 + y4, yz) = 0 2 5. px – qy = (z – xy) . Ans. ![xy, xe1/(z – xy)] = 0 2 2 2 6. zp + zq = z + (x – y) . Ans. log [z + (x – y)2] – 2x = !(x – y). 7. x(yn – zn)p + y(zn – xn)q = z(xn – yn). Ans. xn + yn + zn = !(xyz). 2 2 2 8. (xz + y )p + (yz – 2x )q + 2xy + z = 0. Ans. !(yz + x2, 2xz – y2) = 0. 9. xyp + y(2x – y)q = 2xz. Ans. !(xy – x2, z/xy) = 0. 2.14. Integral surfaces passing through a given curve. In the last article we obtained general integral of Pp + Qq = R. We shall now present two methods of using such a general solution for getting the integral surface which passes through a given curve. Method I. Let Pp + Qq = R ...(1) be the given equation. Let its auxiliary equations give the following two independent solutions u (x, y, z) = c1 and v (x, y, z) = c2. ...(2) Suppose we wish to obtain the integral surface which passes through the curve whose equation in parametric form is given by x = x(t), y = (t), z = z(t), ...(3) where t is a parameter. Then (2) may be expressed as u[x(t), y(t), z(t)] = c1 and v[x(t), y(t), z(t)] = c2. ...(4) We eliminate single parameter t from the equations of (4) and get a relation involving c1 and c2. Finally, we replace c1 and c2 with help of (2) and obtain the required integral surface. 2.15. SOLVED EXAMPLES BASED ON ART. 2.14. Ex. 1. Find the integral surface of the linear partial differential equation x(y2 + z)p – y(x2 + z)q = (x2 – y2)z which contains the straight line x + y = 0, z = 1. [Delhi 2008; Pune 2010] 2 2 2 2 Sol. Given x(y + z)p – y(x + z)q = (x – y )z. ...(1) dy dz dx = . ...(2) + y( x 2 & z) ( x 2 + y 2 )z x( y 2 & z ) Proceed as in solved Ex. 6, Art. 2.10 and show that xyz = c1 and x2 + y2 – 2z = c2. ...(3) Taking t as parameter, the given equation of the straight line x + y = 0, z = 1 can be put in parametric form x = t, y = –t, z = 1. ...(4) Using (4), (3) may be re–written as – t2 = c1 and 2t2 – 2 = c2. ... (5) Eliminating t from the equations of (5), we have 2(–c1) – 2 = c2 or 2c1 + c2 + 2 = 0. ... (6) Putting values of c1 and c2 from (3) in (6), the desired integral surface is 2xyz + x2 + y2 – 2z + 2 = 0. Ex. 2. Find the equation of the integral surface of the differential equation 2y(z – 3)p + (2x – z)q = y(2x – 3), which pass through the circle z = 0, x2 + y2 = 2x. [Meerut 2007] Sol. Given equation is 2y(z – 3)p + (2x – z)q = y(2x – 3). ...(1) Given circle is x2 + y2 = 2x, z = 0. ...(2)

Lagrange’s auxiliary equations of (1) are

Lagrange’s auxiliary equations for (1) are

dy dx = 2 y(z + 3) 2x + z

dz . y(2 x + 3)

...(3)

Created with Print2PDF. To remove this line, buy a license at: http://www.software602.com/

2.28

Linear Partial differential equations of order one

Taking the first and third fractions of (3), (2x – 3)dx – 2(z – 3)dz = 0. 2 2 Integrating, x – 3x – z + 6z = c1, c1 being an arbitrary constant. ...(4) Choosing 1/2, y, –1 as multipliers, each fraction of (3) (1 / 2)dx & ydy + dz (1 / 2)dx & ydy + dz = = y(z + 3) & y(2 x + z) + y(2 x + 3) 0 Hence (1/2)dx + ydy – dz = 0 or dx + 2ydy – 2dz = 0. 2 Integrating, x + y – 2z = c2, c2 being an arbitrary constant. ...(5) Now, the parametric equations of given circle (2) are x = t, y = (2t – t2)1/2, z = 0. ...(6) Substituting these values in (4) and (5), we have t2 – 3t = c1 and 3t – t2 = c2. ...(7) Eliminating t from the above equations (7), we have c1 + c2 = 0. ...(8) Substituting the values of c1 and c2 from (4) and (5) in (8), the desired integral surface is x2 – 3x – z2 + 6z + x + y2 – 2z = 0 or x2 + y2 – z2 – 2x + 4z = 0. Method II. Let Pp + Qq = R ...(1) be the given equation. Let is Lagrange’s auxiliary equations give the following two independent integrals u(x, y, z) = c1 and v(x, y, z) = c2. ...(2) Suppose we wish to obtain the integral surface passing though the curve which is determined by the following two equations !(x, y, z) = 0 and 7(x, y, z) = 0. ...(3) We eliminate x, y, z from four equations of (2) and (3) and obtain a relation between c1 and c2. Finally, replace c1 by u(x, y, z) and c2 by v(x, y, z) in that relation and obtain the desired integral surface. Ex. 3. Find the integral surface of the partial differential equation (x – y)p + (y – x – z)q = z through the circle z = 1, x2 + y2 = 1. (Nagpur 2002) Sol. Given (x – y)p + (y – x – z)q = z. ...(1) dy dz . dx = y+x+z z x+y Choosing 1, 1, 1 as multipliers, each fraction on (2) = (dx + dy + dz)/0 so that x + y + z = c1. , dx + dy + dz = 0 Taking the last two fractions of (2) and using (3) we get

Lagrange’s auxiliary equations for (1) are

dy y + (c1 + y)

dz z

or

...(2) ...(3)

2dy 2dz + = 0. 2 y + c1 z

Integrating it, log (2y – c1) –2 log z = log c2 or (2y – c1)/z2 = c2 or (2y – x – y – z)/z2 = c2 or (y – x – z)/z2 = c2. ....(4) The given curve is given by z=1 and x2 + y2 = 1. ...(5) Putting z = 1 in (3) and (4), we get x + y = c1 – 1 and y – x = c2 + 1. ...(6) But 2(x2 + y2) = (x + y)2 + (y – x)2. ...(7) Using (5) and (6), (7) becomes 2 = (c1 – 1)2 + (c2 + 1)2 or c12 + c22 – 2c1 + 2c2 = 0. ...(8) Putting the values of c1 and c2 from (3) and (4) in (8), required integral surface is (x + y + z)2 + (y – x – z)2/z4 – 2(x + y + z) + 2(y – x – z)/z2 = 0 or z4(x + y + z)2 + (y – x – z)2 – 2z4(x + y + z) + 2z2(y – x – z) = 0. Ex. 4. Find the equation of the integral surface of the differential equation (x 2 – yz)p 2 + (y – zx)q = z2 – xy which passes through the line x = 1, y = 0.

Created with Print2PDF. To remove this line, buy a license at: http://www.software602.com/

Linear Partial differential equations of order one 2

2.29 2

2

Sol. Given (x – yz)p + (y – zx)q = z – xy. ...(1) Proceed as in solved Ex. 6, Art. 2.12 and show that (x – y)/(y – z) = c1 ...(2) and xy + yz + zx = c2. ...(3) The given curve is represented by x=1 and y = 0. ...(4) Using (4) in (2) and (3), we obtain –1/z = c1 and z = c2 so that (–1/z) × z = c1c2 or c1c2 + 1 = 0. ...(5) Putting the values of c1 and c2 from (2) and (3) in (5), the required integral surface is [(x – y)/(y – z)] (xy + yz + zx) + 1 = 0 or (x – y) (xy + yz + zx) + y – z = 0 Ex. 5. Find the equation of surface satisfying 4yzp + q + 2y = 0 and passing through y2 + z2 = 1, x + z = 2. [I.A.S. 1997] Sol. Given 4yzp + q = –2y. ...(1) Given curve is given by y2 + z2 = 1, and x + z = 2. ...(2)

dx = dy dz . ...(3) 4 yz 1 +2 y Taking the first and third fractions of (3), dx + 2zdz = 0 so that x + z2 = c1. ...(4) Taking the last two fractions of (3), dz + 2ydy = 0 so that z + y2 = c2. ...(5) 2 2 Adding (4) and (5), (y + z ) + (x + z) = c1 + c2 or 1 + 2 = c1 + c2, using (2) ...(6) Putting the values of c1 and c2 from (4) and (5) in (6), the equation of the required suface is given by 3 = x + z2 + z + y2 or y2 + z2 + x + z – 3 = 0. Ex. 6. Find the general integral of the partial differential equation (2xy – 1)p + (z – 2x2)q = 2(x – yz) and also the particular integral which passes through the line x = 1, y = 0. [I.A.S. 2008] Sol. Given (2xy – 1)p + (z – 2x2)q = 2(x – yz). ...(1) Given line is given by x=1 and y = 0. ...(2) The Lagrange’s auxiliary equations for (1) are

dz dx = dy . ...(3) 2 xy + 1 z + 2 x 2 2 x + 2 yz Taking z, 1, x as multipliers, each fraction of (3) = (zdx + dy + x dz)/0 so that zdx + dy + xdz = 0 or d(xz) + dy = 0 Integrating, xz + y = c1. ...(4) Again, taking x, y, 1/2 as multipliers, each fraction of (3) = {xdx + ydy + (1/2)dz}/0 so that x dx + ydy + (1/2) × dz = 0 or 2xdx + 2ydy + dz = 0 Integrating, x2 + y2 + z = c2. ...(5) Since the required curve given by (4) and (5) passes through the line (2), so putting x = 1 and y = 0 in (4) and (5), we get z = c1 and 1 + z = c2 so that 1 + c1 = c2. ...(6) Substituting the values of c1 and c2 from (4) and (5) in (6), the eqution of the required surface is given by 1 + xz + y = x2 + y2 + z or x2 + y2 + z – xz – y = 1. Ex. 7. Find the integral surface of x2p + y2q + z2 = 0, p = #z/#x, q = #z/#y which passes through the hyperbola xy = x + y, z = 1. [I.A.S. 1994, 2009] Sol. Given x2p + y2q + z2 = 0 or x2p + y2q = –z2. ...(1) Given curve is given by xy = x + y and z = 1. ...(2) 2 2 2 Here Lagrange’s auxiliary equations for (1) are (dx)/x = (dy)/y = (dz)/(–z ). ...(3)

Lagrange’s auxiliary equations of (1) are

Created with Print2PDF. To remove this line, buy a license at: http://www.software602.com/

2.30

Linear Partial differential equations of order one

Taking the first and third fractions of (1), Integrating, – (1/x) – (1/z) = –c1 Taking the second and third fractions of (1), Integrating, – (1/y) – (1/z) = –c2

x–2dx + z–2dz = 0. 1/x + 1/z = c1. y–2dy + z–2dz = 0. 1/y + 1/z = c2.

or or

1&1&2 = c1 + c2 x y z

Adding (4) and (5),

...(4) ...(5)

x&y 2 & = c1 + c2 xy z c1 + c2 = 3. ...(6)

or

(xy)/(xy) + 2 = c1 + c2, using (2) or Substituting the values of c1 and c2 from (4) and (5) in (6), we get 1/x + 1/z + 1/y + 1/z = 3 or yz + 2xy + xz = 3xyz. Ex. 6. Find the integral surface of the linear first order partial differential equation yp & xq z + 1 which passes through the curve z x2 & y2 & z, y 2x

or

yp & xq

Sol. Given equation is and the given curve is given by

z +1 and dx dy dz y x z +1

z = x2 + y2 + 1

Lagrange’s auxiliary equations for (1) are

2 ydy + 2 xdx

Taking the first two fractions,

y = 2x

... (3)

0

Integrating, it, y2 + x 2 C1 , C1 being an arbitrary constant Taking the first and the last fractions of (3) and using (4), we get dx

dz so that z +1 ( x & C1 ) log(z – 1) – log (x + y) = log C2, by (4) 2

or

... (1) ... (2)

... (4)

log( z + 1) + log{x & ( x 2 & C1 )1/ 2 } log C2

1/ 2

(z – 1)/(x + y) = C2

or

The parametric form of the given curve (2) is

x

Substituting these values in (4) and (5), we get

3t 2

t,

y

2t,

5t & 1 ... (6)

z

5t / 3 C2

and

C1

... (5)

2

... (7)

Eliminating t from the above equations (7), we get 5 C1 / 3 3 C2 ... (8) Substituting the values of C1 and C2 from (4) and (5) in (8), the required surface is given by

5( y2 + x2 )1/ 2 / 3 3 ( z + 1) /( x & y) . Ex. 7. Find the integral surface of the partial differential equation (x –y)y2p + (y – x)x2q = (x2 + y2)z passing through the curve xz a3 , y 0 . ( x + y) y2 p & ( y + x ) x 2 q ( x 2 & y2 ) z xz = a3 and

Sol. Given equation is and the given curve is given by

dx

Lagrange’s auxiliary equations for (1) are Each fraction of (3) =

( x + y) y

dx + dy 2

dz 2

( x + y) ( y & x )

2

( y + x )x

dz 2

2

( x & y2 ) z

so that

2

( x & y )z

3x 2 dx & 3y2 dy

Taking the first two fractions,

x3 & y3

z

t,

... (3) 0

... (4)

0

C2 , C2 being an arbitrary constant.

The parameteric form of the given curve (2) is

... (1) ... (2)

d ( x + y) dz + x+y z

( x + y) / z C1 , C1 being an arbitrary contant

Integrating it,

Integrating it,

dy 2

y=0

x

a 3 / t,

... (5)

y

0 ... (6)

Created with Print2PDF. To remove this line, buy a license at: http://www.software602.com/

Linear Partial differential equations of order one

2.31

Substituting these values in (4) and (5), we get

a3 / t 2

C1

(a3 / t )3

and

C2

Squaring both sides of (8),

(a3 / C1 )3

or

t6

so that

t2

a3 / C1

... (7)

so that

t3

a9 / C2

... (8)

a18 / C22

a18 / C22 ,

(t 2 )3

or since

t2

a18 / C22

a3 / C1 , by (7)

or ... (9) a9 / C13 a18 / C22 , C22 a9 C13 Substituting the values of C1 and C2 from (4) and (5) in (9), the required integral surface of (1) is given by or

( x3 & y3 )2

a9 ( x + y)3 / z3

z 3 ( x3 & y3 )2

or

a9 ( x + y)3 .

EXERCISE 2(F) 1. Find particular integrals of the following partial differential equations to represent surfaces passing through the given curves : (i) p + q = 1 ; x = 0, y2 = z. Ans. (y – x)2 = z – x. (ii) xp + yq = z ; x + y = 1, yz = 1. Ans. yz = (x + y)2. 2 (iii) (y – z)p + (z – x)q = x – y ; z = 0, y = 2x Ans. 5(x + y + z) = 9(x2 + y2 + z2). (iv) x(y – z)p + y(z – x)q = z(x – y) ; x = y ; x = y = z. Ans. (x + y + z)3 = 27xyz. (v) yp – 2xyq = 2xz ; x = t, y = t2, z = t3. Ans. (x2 + y2)5 = 32y2z2. 2 2 2 2 2 (vi) (y – z) [2xyp + (x – y )q] + z(x – y ) = 0 ; x = t , y = 0, z = t3. Ans. x3 – 3xy2 = z2 – 2yz. 2. Find the general solution of the equation 2x(y + z2)p + y(2y + z2)q = z2 and deduce that 2 yz(z + yz – 2y) = x2 is a solution. 3. Find the general solution of x(z + 2a)p + (xz + 2yz + 2ay)q = z(z + a). Find also the integral surfaces which pass through the curves : (i) y = 0, z2 = 4ax. (ii) y = 0, z3 + x(z + a)2 = 0. 4. Solve xp + yq = z. Find a solution representing a surface meeting the parabola Ans. General solution ! (x/2, y/2) = 0 ; surface y2 = 4xz.

y2 = 4x, z = 1.

2.16. SURFACES ORTHOGONAL TO A GIVEN SYSTEM OF SURFACES Let f(x, y, z) = C ...(1) represents a system of surfaces where C is parameter. Suppose we wish to obtain a system of surfaces which cut each of (1) at right angles. Then the direction ratios of the normal at the point (x, y, z) to (1) which passes through that point are #f/#x, #f/#y, #f/#z. Let the surface z = !(x, y) ...(2) cuts each surface of (1) at right angles. Then the normal at (x, y, z) to (2) has direction ratios #z/#x, #z/#y, –1 i.e., p, q, –1. Since normals at (x, y, z) to (1) and (2) are at right angles, we have p(#f / #x ) & q(#f / #y ) + (#f / #z )

0

or

p(#f / #x) & q(#f / #y )

#f / #z

...(3)

which is of the form Pp + Qq = R. Conversely, we easily verify that any solution of (3) is orthogonal to every surface of (1). 2.17.SOLVED EXAMPLES BASED ON ART. 2.16. Ex. 1. Find the surface which intersects the surfaces of the system z(x + y) = c(3z + 1) orthogonally and which passes through the circle x2 + y2 = 1, z = 1. [I.A.S. 1999] Sol. The given system of surfaces is

f ( x, y, z ) 8 / z ( x & y0 / (3z & 1) C.

...(1)

Created with Print2PDF. To remove this line, buy a license at: http://www.software602.com/

2.32

or

Linear Partial differential equations of order one

(3z & 1) + z 1 3 x& y #f #f z #f z , = , = ( x & y) . , #x = 2 3 z & 1 #z 3z & 1 #y (3z & 1) (3z & 1)2 The required orthogonal surface is solution of #f #f #f z p& z q = x & y = or p &q #z 3z & 1 3z & 1 #x #y (3z & 1)2 z(3z + 1)p + z(3z + 1)q = x + y. ...(2)

dy dz dx = . ...(3) z(3z & 1) x & y z(3z & 1) Taking the first two fractions of (3), we get dx – dy = 0 so that x – y = C1. ...(4) Choosing x, y, –z(3z + 1) as multipliers, each fraction of (3) = [xdx + ydy – z(3z + 1)dz]/0 xdx + ydy – 3z2dz – zdz = 0 or 2xdx + 2ydy – 6z2dz – 2zdz = 0 , 2 2 3 2 Integrating, x + y – 2z – z = C2, C2 being an arbitrary constant. ...(5) Hence any surface which is orthogonal to (I) has equation of the form x2 + y2 – 2z3 – z2 = !(x – y), ! being an arbitrary function ...(6) In order to get the desired surface passing through the circle x2 + y2 = 1, z = 1 we must choose !(x – y) = –2. Thus, the required particular surface is x2 + y2 – 2z3 – z2 = –2. Ex. 2. Write down the system of equations for obtaining the general equation of surfaces orthogonal to the family given by x( x2 & y2 & z2 ) C1 y2 . [I.A.S. 2001]

Lagrange’s auxiliary equations for (2) are

x( x2 & y2 & z2 ) / y2

Sol. Given family of surfaces is 2

2

2

C1

2

Let ... (1) f ( x, y, z) x( x & y & z ) / y C1 Then the surfaces orthogonal to the system (1) are the surfaces generated by the integral curves of the equations dx dy dz dx dy dz or 2 2 2 2 2 2 3 #f / #x #f / #y #f / #z (3x & y & z ) / y +2 x ( x & z ) / y 2 x / y2 z or

dx 2

dy 2

2

2

dz 2 xyz

2

y(3x & y & z ) +2 x ( x & z ) Taking x, y, z as multipliers, each fraction of (2) xdx & ydy & zdz

=

2

2

2

2

xy( x & y & z )

2

2

xdx & ydy & zdz

... (3)

xy(3x & y & z ) + 2 xy( x & z ) & 2 xyz xy( x 2 & y 2 & z 2 ) Combining this fraction (3) with the last fraction of (2), we get xdx & ydy & zdz

2

... (2)

dz 2 xyz

2

2 xdx & 2 ydy & 2 zdz

or

2

x &y &z

Integrating, log( x2 & y2 & z2 ) log z & log C2 Taking 4x, 2y, 0 as multipliers, each fraction of (2) =

or

4 xdx & 2 ydy

2

2

4 xy(2 x & y )

Integrating,

dz 2 xyz

log(2 x 2 & y2 )

2

2

2

dz z

( x2 & y2 & z2 ) / z C2 ...(4) 4 xdx & 2 ydy

2

... (5)

4 xy(3x & y & z ) + 4 xy( x & y ) 4 xy(2 x 2 & y 2 ) Combining this fraction (5) with the last fraction of (2), we get 4 xdx & 2 ydy

2

2

2

4 xdx & 2 ydy

or

2 log z & log C3

2

2x & y

or

2

(2x 2 & y2 ) / y2

2dz z

C3

... (6)

Created with Print2PDF. To remove this line, buy a license at: http://www.software602.com/

Linear Partial differential equations of order one

2.33

From (4) and (5), the required general equation of the surfaces which are orthogonal to the given family of surfaces (1) is of the form ( x2 & y2 & z2 ) / z

x 2 & y2 & z 2

or

! {(2 x2 & y2 ) / z 2} , i.e.,

z ! {(2 x 2 & y2 ) / z2 } , where ! is an arbitrary function.

Ex. 3. Find the surface which is orthogonal to the one parameter system z which passes through the hyperbola x 2 + y2

a2 , z

Sol. The given system of surfaces is #f #x

+

z(3x 2 y & y3 ) 3

3

( x y & xy )

0

f ( x, y, z) #f #y

, 2

+

z /( x 3 y & xy3 ) C

z(3y 2 x & x 3 ) 3

3 2

( x y & xy )

+

z(3x 2 y & y3 ) 3

3 2

( x y & xy )

p+

z(3y2 x & x 3 ) 3

#f #z

,

3 2

( x y & xy )

q

dx 2

(3x & y ) / x

Taking the first two fractions of (3),

2

x y & xy 3

1 x y & xy3

+( x 2 & y2 ) / z

dy 2

1 3

3

{(3x 2 & y2 ) / x}p & {(3y2 & x 2 ) / y}q Lagrange’s auxiliary equations for (2) are

or

... (1)

p(#f / #x) & q(#f / #y) #f / #z

The required orthogonal surface is solution of or

cxy( x 2 & y2 )

... (2)

dz 2

(3y & x ) / y

2 xdx + 2 ydy

... (3)

2

+( x & y 2 ) / z

0

x 2 + y2

so that

C1

Choosing x, y, 4z as multipliers, each fraction of (3) = ( xdx & ydy & 4zdz) / 0 so that , 2 xdx & 2 ydy & 8zdz 0 Hence any surface which is orthogonal to (1) is of the form

x 2 & y2 & 4 z 2

5 ( x2 + y2 ) , 5 being an arbitrary function.

For the particular surface passing through the hyperbola x 2 + y2 2

2

5( x + y )

4

2

2

2

x 2 & y2 & 4 z 2

a2 , z

C2 ... (4)

0 we must take

2 2

a ( x & y ) /( x + y ) . Hence, the required surface is given by

( x2 & y2 & 4z2 )2 ( x 2 + y2 )2 a4 ( x 2 & y2 ) Ex. 4. Find the equation of the system of surfaces which cut orthogonally the cones of the system x2 + y2 + z2 = cxy. 2.18 (a). Geometrical description of the solutions of Pp + Qq = R and of the system of equations dx/P = dy/Q = dz/R and to establish relationship between the two. [G.N.D.U. Amritsar 1998; Meerut 1997; Kanpur 1996] Proof. Consider Pp + Qq = R. ....(1) and (dx)/P = (dy)/Q = (dz)/R, ...(2) where P, Q and R are functions of x. Let z = !(x, y) ...(3) represent the solution of (1). Then (3) represents a surface whose normal at any point (x, y, z) has direction ratios #z/#x, #z/#y, –1 i.e., p, q, –1. Also we know that the simultaneous equations (2) represent a family of curves such that the tangent at any point has direction ratios P, Q, R. Rewriting (1), we have Pp + Qq + R(–1) = 0, ...(4) showing that the normal to surface (3) at any point is perpendicular to the member of family of curves (2) through that point. Hence the member must touch the surface at that point. Since this

Created with Print2PDF. To remove this line, buy a license at: http://www.software602.com/

2.34

Linear Partial differential equations of order one

holds for each point on (3), we conclude that the curves (2) lie completely on the surface (3) whose differential equation is (1). 2.18 (b). Another geomertical interpretation of Lagrange’s equation Pp + Qq = R. To show that the surfaces represented by Pp + Qq = R are orthogonal to the surfaces represented by Pdx + Qdy + Rdz = 0. We know that the curves whose equations are solutions of (dx)/P = (dy)/Q = (dz)/R ...(1) are orthogonal to the system of the surfaces whose equation satisfies Pdx + Qdy + Rdz = 0. ...(2) Again from Art 2.18 (a) the curves of (1) lie completely on the surface represented by Pp + Qq = R. ...(3) Hence we conclude that surfaces represented by (2) and (3) are orthogonal. 2.19.SOLVED EXAMPLES BASED ON ART 2.18(a) AND ART. 2.18 (b) Ex. 1. Find the family orthogonal to ! [z(x + y)2, x2 – y2] = 0. Sol. Given ![z(x + y)2, x2 – y2] = 0. 2 Let u = z(x + y) and v = x2 – y2 Then (1) becomes !(u, v) = 0. Differentiating (3) w.r.t. x and y partially by turn, we get #! #u #! #v #u #v &p & &p =0 #u #x #z #v #x #z and

and

FG H

FH

IJ K

IK

FH

FG #v & q #v IJ = 0. H #y #z K

#! #u #! & q #u & #u #y #z #v From (2), (#u/#x) = 2z(x + y), (#v/#x) = 2x, Putting these values in (4) and (5), we get

IK

(#u/#y) = 2z(x + y), (#v/#y) = –2y,

...(1) ...(2) ...(3) ...(4) ...(5)

(#u/#z) = (x + y)2, (#v/#z) = 0.

(#! / #u) 2 z( x & y) & p( x & y)2 & (#! / #v) (2 x & 0) = 0

...(6)

(#! / #u ) 2 z( x & y) & q( x & y)2 & (#! / #v ) (+2 y & 0) = 0

...(7)

Evaluating the values of +

#! / #u from (6) and (7) and then equating these, we get #! / #v

+2 y #! / #u 2x = #! / #v 2 z( x & y) & p( x & y)2 2 z( x & y) & q( x & y)2 or x(x + y)[2z + q(x + y)] = –y(x + y)[2z + p(x + y)] or 2xz + qx(x + y) + 2yz + py(x + y) = 0 or py(x + y) + qx(x + y) = –2z(x + y) or py + qx = –2z ...(8) which is differential equation of the family of surfaces given by (1). So the differential equation of the family of surfaces orthogonal to (8) is given by [use Art. 2.18 (b)] ydx + xdy – 2zdz = 0 or d(xy) – 2zdz = 0. ...(9) Integrating (9), xy – z2 = C, which is the desired family of orthogonal surfaces, C being parameter Ex. 2. Find the family of surfaces orthogonal to the family of surfaces given by the differential equation (y + z)p + (z + x)q = x + y. Sol. Let P = y + z, Q=z+x and R = x + y. ...(1) Then, the given differential equation can be written as Pp + Qq = R. ...(2) Now, the differential equation of the family of surfaces orthogonal to the given family is +

Created with Print2PDF. To remove this line, buy a license at: http://www.software602.com/

Linear Partial differential equations of order one

2.35

Pdx + Qdy + Rdz = 0 or (y + z)dx + (z + x)dy + (x + y)dz = 0 or (ydx + xdy) + (ydz + zdy) + (zdx + xdz) = 0. Integrating, xy + yz + zx = C, which is the required family of surfaces, C being a parameter. 2.20. The linear partial differential equation with n independent variables and its solution. Let x1, x2 ..., xn be the n independent variables and let p1 = #z/#x1, p2 = #z/#x2, ..., pn = #z/#xn, where z is the dependent variable. Consider the general linear partial differential equation with n independent variables P1 p1 + P2 p2 + .. + Pn pn = R, ...(1) where P1, P2,.., Pn are functions of x1, x2..., xn. Let u1 = c1, u2 = c2, ..., un = cn be any n independent integrals of the auxiliary equations (dx1)/P1 = (dx2)/P2 = ... = (dxn)/Pn. ...(2) Then the general solution of (1) is given by !(u1, u2, ...un) = 0. ...(3) Note that the above procedure is generalization of Lagrange’s method. 2.21.SOLVED EXAMPLES BASED ON ART. 2.20 Ex. 1. Solve x2x3 p1 + x3x1 p2 + x1x2 p3 + x1x2x3 = 0. Sol. Re–writing the given equation in standard form, we have x2x3 p1 + x3x1 p2 + x1x2 p3 = –x1x2x3.

dx1 x2 x3

The auxiliary equations for (2) are

dx2 dx3 = x3 x1 x1x2

...(2)

dz . + x1x2 x3

...(3)

Taking the first and the fourth fractions of (3), x1dx1 + dz = 0 so that x12 + 2z = c1. ...(4) Taking 1st and 2nd fractions of (3), x1dx1 = x2dx2 so that x12 – x22 = c2. ...(5) Finally, 2nd and 3rd fractions of (3) give x2dx2 = x3dx3 so that x22 – x32 = c3. ...(6) Hence the required general integral is !(x12 + 2z, x12 – x22, x22 – x32) = 0, ! being an arbitrary function. xz Ex. 2. Solve x #z & y #z & t #z = az + t #x #y #t Sol. Here auxiliary equations for the given equation are dx dy dt dz . = ...(1) x y t az & xy t From the first two fractions of (1), (1/x)dx – (1/y)dy = 0 so that x/y = C1. ...(2) From the first and third fractions of (1), (1/x)dx – (1/t)dt = 0 so that x/t = C2. ...(3) Dividing (3) by (2), we have y/t = C2/C1. ...(4) Taking the first and third fractions of (1) and using (4), we get dx x

dz az & (C2 C1)x

or

FH IK FG IJ H K

dz dx

az & (C2 C1 ) x x

dz a + z = C2 , which is linear.. dx x C1

or

...(5)

+a + ( a / x ) dx e + a log x elog x = x–a and so solution of (5) is given by I.F. of (5) = e .

z

C2 +a C2 x1 + a y x1+ a zx–a = C3 + C x dx C3 & C 1 + a C3 & t 1 + a , using (4) 1 1

,

zx–a –

y x1+ a = C3, C3 being an arbitrary constant. t 1+ a

...(6)

Created with Print2PDF. To remove this line, buy a license at: http://www.software602.com/

2.36

Linear Partial differential equations of order one

From (2), (3) and (6), the required general solution is !

FG x , x , zx Hy t

+a

+

IJ K

y x1+ a = 0, ! being an arbitrary function. t 1+ a

Ex. 3. Solve x(#u/#x) + y(#u/#y) + z(#u/#z) = xyz. [Bhopal 1995, 98; I.A.S. Sol. Here the auxiliary equations for the given equation are dx dy dz du . = x y z xyz Taking the first two fractions of (1), (1/x)dx – (1/y)dy = 0. Integrating it, log x – log y = log C1 or x/y = C1. Taking the first and third fractions of (1), (1/x)dx – (1/z)dz = 0 Integrating it, log x – log z = log C2 or x/z = C2.

1999] ...(1) ...(2) ...(3)

yzdx & zxdy & xydz d( xyz) = . ...(4) xyz & xyz & xyz 3 xyz Combining the fourth fraction of (1) with fraction (4), we get

Choosing yz, zx, xy as multipliers, each fraction of (1) =

du d( xyz) or d(xyz) – 3du = 0 so that xyz – 3u = C3. ...(5) xyz 3 xyz From (2), (3) and (5), the required general solution is !(x/y, x/z, xyz – 3u) = 0, ! being an arbitrary function. Ex. 4. Solve (y + z + w) (#w/#x) + (z + x + w) (#w/#y) + (x + y + w) (#w/#z) = (x + y + z). [Ravishanker 2004 ; I.A.S. 1995; Indore 1998, Kanpur 2004] Sol. Here the auxiliary eqautions of the given equation are dy dx dz dw = . ...(1) y&z&w z& x &w x & y&w x & y& z

Each fraction of (1) =

dw + dx + (w + x )

dw + dy dw + dz = +(w + y) + (w + z )

dw & dx & dy & dz . 3 (w & x & y & z )

...(2)

dw & dx & dy & dz dw + dz & = 0. 3(w & x & y & z) w+ x Integrating, (1/3) × log (w + x + y + z) + log (w – x) = log C1 or (w + x + y + z)1/3 (w – x) = C1. ...(3) 1/3 Similarly, (w + x + y + z) (w – y) = C2. ...(4) and (w + x + y + z)1/3 (x – z) = C3. ...(5) From (3), (4) and (5), the required general solution is ![(w + x + y + z)1/3 (w – x), (w + x + y + z)1/3 (w – y), (w + x + y + z)1/3 (w – z)] = 0, where ! is an arbitrary function.

Taking the first and the fourth fractions of (2),

Ans. ! (ze +4 x1 , ze +4 x2 , ze +4 x3 ) = 0

Ex. 5. Solve p1 + p2 + p3 = 4z.

Ex. 6. Solve x2 x3 p1 + x3 x1 p2 + x1x2 p3 + x1x2x3 = 0. Sol. Putting the given equation in standard form, we have x2 x3 p1 + x3 x1 p2 + x1 x2 p3 = –x1 x2 x3. Here the auxiliary equations for (1) are

dx1 x2 x3

dx3 dx2 = x3 x1 x1x2

Taking the first and second fractions of (2), we have 2x1dx1 – 2x2dx2 = 0 so that

dz . + x1x2 x3 x12 – x22 = C1.

...(1) ...(2)

...(3)

Created with Print2PDF. To remove this line, buy a license at: http://www.software602.com/

Linear Partial differential equations of order one

2.37

Taking the first and third fractions of (2), we have 2x1dx1 – 2x3dx3 = 0 so that x12 – x32 = C2. Taking the first and fourth fractions of (2), we have 2x1dx1 + 2dz = 0 so that x12 + 2z = C3. From (3), (4) and (5), the required general solution is !(x12 – x22, x12 – x32, x12 + 2z) = 0, ! being an arbitrary function.

...(4) ...(5)

Ex. 7. Solve p1 + x1p2 + x1x2p3 = x1x2x3 z . Ans. ! (x12 – 2x2, x22 – 2x3, x32 – 4 z ) = 0 Ex. 8. Solve (x3 – x2)p1 + x2 p2 – x3 p3 + x22 – (x2x1 + x2x3) = 0. Sol. Re–writing the given equation in the standard form, we get (x3 – x2)p1 + x2 p2 – x3 p3 = x2 x1 + x2 x3 – x22. ...(1) Here the auxiliary equations for (1) are

dx1 x3 + x2

dx2 dx = 3 x2 + x3

dz . x2 x1 & x2 x3 + x22

...(2)

Taking the second and the third fractions of (2), we have (1/x2)dx + (1/x3)dx2 = 0 so that log x2 + log x3 = log C1 or x2x3 = C1....(3) dx1 & dx2 & dx3 dx1 & dx2 & dx3 Each fraction of (2) = . ( x3 + x2 ) & x2 + x3 0 so that x1 + x2 + x3 = C2. ...(4) , dx1 + dx2 + dx3 = 0 d( x1 x2 ) Each fraction of (2) = x2 dx1 & x1dx2 . ...(5) x2 ( x3 + x2 ) & x1 x2 x1 x2 & x2 x3 + x22 Combining the last fraction of (2) with fraction (5), we have d(x1 x2 ) dz = or dz – d(x1x2) = 0. x1 x2 & x2 x3 + x22 x1 x2 & x2 x3 + x22 Integrating, z – x1x2 = C3, C3 being an arbitrary constant. ...(6) From (3), (4) and (5), the required general solution is !(x2 x3, x1 + x2 + x3, z – x1x2) = 0, ! being an arbitrary function. Ex. 9. If u is a function of x, y and z which satisfies (y – z)(#u/#x) + (z – x) (#u/#y) + (x – y) (#u/#z) = 0, show that u contains x, y, z only in combinations of x + y + z and x2 + y2 + z2. (Nagpur 2002, 05) Sol. Here auxiliary equations for given equation are Each fraction of (1) =

dx y+z

dy dz = z+x x+y

xdx & ydy & zdz dx & dy & dz = x( y + z) & y(z + x ) & z( x + y) ( y + z) & (z + x ) & ( x + y)

xdx & ydy & zdz du dx & dy & dz = . 0 0 0 2xdx + 2ydy + 2zdz = 0 , dx + dy + dz = 0, Integrating, x + y + z = C1, x2 + y2 + z2 = C2 Hence the required general solution is u = f(x + y + z, x2 + y2 + z2), f being an arbitrary function.

du . 0

...(1)

du 0

=

and and

du = 0 u = C3.

Ex. 10. Prove that if x13 & x23 & x33 1 when z = 0, the solution of the equation (s – x1) p1 + (s – x2)p2 + (s – x2)p2 + (s – x3)p3 = s – z can be given in the form s3{(x1 – z)3 + (x2 – z)3 + (x3 – z)3}4 = (x1 + x2 + x3 – 3z)3, where s = x1 + x2 + x3 + z and pi #z / #xi , i 1, 2, 3. [I.A.S. 2000] Sol. Given where

(s + x1 ) p1 & (s + x 2 ) p2 & (s + x3 ) p3

s

x1 & x2 & x 3 & z

s+z

... (1) ... (2)

Created with Print2PDF. To remove this line, buy a license at: http://www.software602.com/

2.38

Linear Partial differential equations of order one

The auxiliary equations for (2) are

dx1 s + x1

dx1 x2 & x3 & z

dx3 x1 & x2 & z

or

dx2 x3 & x1 & z

dx2 s + x2

dx3 s + x3

dz , using (2) x1 & x 2 & x3

dx1 & dx2 & dx3 + 3dz 2( x1 & x2 & x3 ) & 3z + 3( x1 & x 2 & x3 )

Each fraction of (3) =

dz s+z

... (3)

d ( x1 & x2 & x3 + 3z) +( x1 & x 2 & x3 + 3z)

... (4)

Again, each fraction of (3) =

dx1 & dx 2 & dx3 & dz 3( x1 & x2 & x3 & z)

d ( x1 & x2 & x 3 & z) 3( x1 & x2 & x3 & z)

... (5)

Then, (4) and (5) give

d ( x1 & x 2 & x3 + 3z) +( x1 & x 2 & x3 + 3z)

d ( x1 & x 2 & x3 & z) 3( x1 & x2 & x3 & z)

... (5)

d ( x1 & x 2 & x3 & z) d( x1 & x2 & x3 + 3z) &3 x1 & x 2 & x3 & z x1 & x2 & x3 + 3z

or

log( x1 & x2 & x3 & z) & 3log( x1 & x2 & x3 + 3z)

Integrating,

3

( x1 & x2 & x3 & z) ( x1 & x2 & x3 + 3z)

or

0

x13

Given that

&

x23

&

x33

log a

a, where a is an arbitrary constant. z=0

when

1

... (6) ... (7)

Hence (6) gives a ( x1 & x2 & x3 )4 . Then (6) reduces to

( x1 & x2 & x3 & z) ( x1 & x2 & x3 + 3z)3 Now, each fraction of (3) =

dx1 + dz +( x1 + z)

d ( x1 + z)3

+3( x1 + z)3

3( x1 + z)3

d ( x 2 + z )3

d ( x3 + z)3

+3( x2 + z)3 Using (9) and (10), we find that each fraction of (3)

+3( x3 + z)3

... (9) ...(10)

d( x3 + z)3

d[( x1 + z)3 & ( x2 + z)3 & ( x3 + z)3 ]

+3( x1 + z)3 +3( x2 + z)3 +3( x3 + z)3 Then, from (4) and (11), we have

+3[( x1 + z)3 & ( x2 + z)3 & ( x3 + z)3 ]

=

d ( x1 + z)3

d ( x 2 + z )3

3d ( x1 & x 2 & x3 + 3z) ( x1 & x2 & x3 + 3z)

Integrating it,

[( x1 + z)3 & ( x2 + z)3 & ( x3 + z)3 ]

3log ( x1 & x2 & x3 + 3z) & log b

( x1 + z)3 & ( x2 + z)3 & ( x3 + z)3

log {( x1 + z)3 & ( x2 + z)3 & ( x3 + z)3}

b ( x1 & x2 & x3 + 3z)3 where b is an arbitrary constant. ...(12) x13 & x23 & x33

1 b ( x1 & x2 & x3 )3 , using (7)

b ( x1 & x2 & x3 )3 so that

b 1/( x1 & x2 & x 3 )3

, (12) − ( x1 + z)3 & ( x2 + z)3 & ( x3 + z)3 ( x1 & x2 & x3 + 3z)3 /( x1 & x2 & x3 )3 Raising both sides of (8) to power 3, we have ( x1 & x2 & x3 & z)3 ( x1 & x2 & x3 + 3z)9 ( x1 & x2 & x3 )12 Raising both sides of (13) to power 4, we have {( x1 + z)3 & ( x2 + z)3 & ( x3 + z)3}4 ( x1 & x2 & x3 + 3z)12 /( x1 & x2 & x3 )12 Multiplying the corresponding sides of (14) and (15), we have

( x1 & x2 & x3 & z)3 {( x1 + z)3 & ( x2 + z)3 & ( x3 + z)3}4 or

... (11)

d[( x1 + z)3 & ( x2 + z)3 & ( x3 + z)3 ]

Putting z = 0, (12) gives or

... (8)

3( x1 + z)2 d ( x1 + z)

By symmetry, each fraction of (3) is also =

or

( x1 & x2 & x3 )4

s3 {( x1 + z)3 & ( x2 + z)3 & ( x3 + z)3 }4

... (13) ... (14) ... (15)

( x1 & x2 & x3 + 3z)3

( x1 & x2 & x3 + 3z)3 , using (2)

Created with Print2PDF. To remove this line, buy a license at: http://www.software602.com/

Linear Partial differential equations of order one

2.39

EXERCISE 2 (H) Ex. 1. Solve p2 + p3 = 1 + p1. Ans. !(x1 + x2, x1 + x3, x1 + z) = 0 Ex. 2. Solve zx2x3p1 + zx3x1p2 + zx1x2p3 = x1x2x3. Ans. !(x12 – x22, x12 – x22, x12 – z2) = 0] Ex. 3. Solve x1p1 + 2x2p2 + 3x3p3 + 4x4p4 = 0. Ans. !(x12/x2, x13/x3, x14/x4, z) = 0 1/2 Ex. 4. Solve p1 + p2 + p3 {1 – (z – x1 – x2 – x3) } = 3. Ans. ![z – 3x1, z – 3x2, z + 6(z – x1 – x2 – x3)1/2] = 0 Ex. 5. p1 + p2 + p3 {1 + (z + x1 + x2 + x3)1/2} + 3 = 0. Ans. ![z + 3x1, z + 3x2, z + 6(z + x1 + x2 + x3)1/2] = 0 Ex. 6. x1 p1 + x2 p2 + x3 p3 = az + (x1x2)/x3. [Delhi Maths (H) 1998] [Hint. This is same as Ex. 2 of Art. 2.21. Here x = x1, y = x2, t = x3, #z / #x #z / #y

#z / #x2

p2 , #z / #t

#z / #x3

#z / #x

p1 ,

p3 ]

OBJECTIVE PROBLEMS ON CHAPTER 2 Select correct answer by writing (a), (b), (c) or (d). 1. The equation Pp + Qq = R is known as (a) Charpit’s equation (b) Lagrange’ equation (c) Bernoulli’s equation (d) Clairaut’s equation. [Agra 2005, 06, 08] 2. The Lagrange’s auxiliary equations for the partial differential equation Pp + Qq = R are (a) (dx)/P = (dy)/Q = (dz)/R (b) (dx)/P = (dy)/Q (c) (dx)/P = (dz)/R. (d) none of these. [Garhwal 2005] 3. The general solution of (y – z) p + (z – x) q = x – y is (a) !( x & y & z, x2 & y 2 & z 2 ) (c) !( xyz , x2 & y 2 & z 2 )

0

0, (b) !( xyz , x & y & z )

0

(d) !( x2 + y 2 + z 2 , x + y + z )

0 [M.S. Univ. T.N. 2007]

[Hint : Refer Ex. 15, Art 2.10] 4. Subsidiary equations for equation (y2z/x) + zxy = y2 are (a) (dx)/y2z = (dy)/(zx) = (dz)/y2 (b) (dx)/x2 = (dy)/y2 = (dz)/zx (c) (dx)/x2 = (dy)/y2 = (dz)/zx (d) (dx)/(1/x2) = (dy)/(1/y2) = (dz)/(1/zx) [Kanpur 2004] 5. The general solution of the linear partial differential equation Pp & Qq R is (a) !(u , v) 1 (b) !(u , v ) +1 (c) !(u , v ) 0 Answers. 1. (b) 2. (a) 3. (a)

(d) None of these 4. (d) 5. (c)

[Agra 2007]

MISCELLANEOUS EXAMPLES ON CHAPTER 2 Ex. 1. Tranform the equation yzx – xzy = 0 into one in polar coordinates and thereby show that the solution of the given equation represents surfaces of revolution. (I.A.S. 2007) Sol. Let x " r cos # and y " r sin #

$

$ 2r (&r / &x ) " 2 x, 2r (&r / &y ) " 2 y

$

&# 1 " &x 1 % y 2 / x 2 Now,

r sin # sin # ' y( )+ ! 2 *, " ! 2 " ! r , x r zx "

r 2 " x 2 % y 2 and # " tan !1 ( y / x) ... (1) &r / &x " cos # , &r / &y " sin #

&# 1 " &y 1 % y 2 / x 3

' 1 ( r cos # cos # " )+ *, " x r r2

... (2) ... (3)

&z &z &r &z &# &z sin # &z " % " cos # ! , using (2) and (3) ...(4) &x &r &x &# &x &r r &#

Created with Print2PDF. To remove this line, buy a license at: http://www.software602.com/

2.40

Linear Partial differential equations of order one

and

zy "

&z &z &r &z &# &z cos # &z " % " sin # % using (2) and (3) ...(5) &y &r &y &# &y &r r &#

Using (1), (4) and (5), the given equation yzx – xzy = 0 reduce to

&z sin # &z ( &z cos # &z ( ' ' r sin # ) cos # ! *, ! r cos # )+ sin # % * "0 + &r r &# &r r &# ,

or

&z " 0 ... (6) &#

Integrating (6) w.r.t. ‘9’, z = f (r), where f is an arbitrary function .... (7) Clearly (7) represents surfaces of revolution, as required. Ex.2. Solve (y + z ) p – (x + z) q = x – y (Agra 2010) Ans. 5( x & y & z , x 2 & y 2 + z 2 )

Hint. Do like Ex. 15, page 2.14.

Ex. 3. The integral surface satisfying equation y(#z / #x) + x(#z / #y) through the curve x = 1 – t, y = 1 + t, z = 1 + t2 is (a) z

xy & ( x2 + y 2 ) / 2

(c) z

xy & ( x 2 + y 2 ) 2 / 4

(b) z (d) z

0

x 2 & y 2 and passing

xy & ( x 2 + y 2 ) 2 / 8 xy & ( x 2 + y 2 )2 /16

(GATE 2009]

Ex. 4. Find the partial differential equation whose surfaces are orthogonal to the surface z (x + y) = 3z + 1 [Pune 2010] Ans. z (p + q) = x + y – 3 Ex. 5. if u (x, y, z) = c1 and v (x, y, z) = c2 are integral curves of (dx)/P = (dy)/Q = (dz)/R, then show that F (u, v) = 0 is general solution of Pp + Qq = R, where F is an arbitrary function. [Pune 2010]

Created with Print2PDF. To remove this line, buy a license at: http://www.software602.com/

3 Non–linear partial differential equations of order one 3.1. Explanation of terms : complete integral (or complete solution), particular integral, singular integral (or singular solution), and general integral (or general solution) as applied to solutions of first order partial differential equations [I.A.S. 1995; Meerut 1997; Delhi Maths Hons. 1995] A solution or integral of a differential equation is a relation between the variables, by means of which and the derivatives obtained there from the equation is satisfied. Let us now discuss various classes of integrals of a partial differential equation of order one. Complete Integral (C. I.). or complete solution (C.S.) [Sagar 1995] Let us consider a relation !(x, y, z, a, b) = 0 ...(1) in which x, y, z are variables such that z is dependent on x and y. Differentiating (1) partially w.r.t x and y respectively, we obtain

! ! ! ! # p =0 and ...(2) # q = 0, x z y z Since there are two arbitrary constants (namely a and b) connected by the above three equations, these can be eliminated and there will appear a relation of the form f(x, y, z, p, q) = 0, ...(3) which is a partial differential equation of order one. Suppose, now, that (1) has been derived from (3), by using some method; then the integral (1), which has as many arbitrary constants as there are independent variables, is called the complete integral of (3). Particular Integral : A particular integral of (3) is obtained by giving particular values to a and b in (1) which is the complete integral of (3). Singular Integral (S.I.) or singular solution (S.S.) [Delhi 2009; Sagar 1995] We know that the locus of all the points whose co–ordinates along with the values of p and q satisfy (3), represent the doubly infinite system of surfaces given by (1). The system is doubly infinite, since there are two constants a and b and each of these can take an infinite number of values. Since the envelope of all the surfaces given by (1) is touched at each of its points by some one of these surfaces, the coordinates of any point on the envelope along with the values of p and q belonging to the envelope at that point must also satisfy (3). Hence we conclude that the equation of the envelope is a solution of (3). The envelope of the surfaces given by (2) is obtained by eliminating a and b between the equations !(x, y, z, a, b) = 0,

!/ a = 0

and

! / b = 0. ...(4)

The relation between x, y, and z so obtained is called the singular integral. In general, it is distinct from the complete integral. However, in exceptional cases it may be contained in the 3.1

Created with Print2PDF. To remove this line, buy a license at: http://www.software602.com/

3.2

Non-Linear Partial Differential Equations of Order One

complete integral, that is, singular integral may be obtained by giving particular values to the constants in the complete integral. Since other relations may appear in the process of getting the singular integral, it is necessary to test that the equation of singular integral satisfies the given differential equation. General Integral (G.I.) or General Solution (G.S.). Assume that in (1), one of the constants is a function of the other, say b = F(a), then (1) becomes !(x, y, z, a, F(a)) = 0. ...(5) Now (5) represents one of the families of surfaces given by the system (1). As before, the equation of the envelope of the family of surfaces given by (5) must also satisfy (3). Again the equation so obtained will be distinct from that of the envelope of the surfaces, and it is not a particular integral. It is known as the general integral and is obtained by eliminating a between !(x, y, z, a F(a)) = 0

and

! / a = 0.

...(6)

Since other relations may appear in the process of getting the singular integral, it is necessary to test that the equation of general integral satisfies the given differential equation. Important Note. While solving a non–linear equation, we must not only obtain the complete integral but should also find the singular and general integrals. In absence of details of singular and general integrals, merely the complete solution is considered to be incomplete solution of the given partial differential equation. However, for reason of space, we have found complete integral only in some problems. The students are advised to find singular and general integrals also for such problems. Note that there is always a simple routine method for the same. Also, if you are asked to find complete integral of a given equation, then you need not give singular and general integrals. Again, if examiner wants singular integral/general integral, then you must find them. 3.2. Geometrical interpretation of three types of integrals of f(x, y, z, p, q) = 0. (i) Complete integral. A complete integral, being a relation between x, y and z represents equation of a surface. Since it involves two arbitrary parameters, it belongs to a double infinite system of surfaces or to a single infinite system of family of surfaces. (ii) General integral. Let a complete solution of f(x, y, z, p, q) = 0 be !(x, y, z, a, b) = 0. ...(1) A general integral is obtained by eliminating ‘a’ between (1) and the equations b = ∃(a) ...(2) ( !/ a) + ( !/ b) ∃%(a) = 0. ...(3) where ∃ is an arbitrary function. The operation of elimination is equivalent to selecting from the system of families of surfaces a representative family and finding the envelope. Equations (1), (2) and (3) together represent a curve drawn on the surface of the family whose parameter is ‘a’ whereas the equation obtained by eliminating ‘a’ between them is the envelope of the family. It follows that the envelope touches the surface represented by (1) and (2) along the curve represented by (1), (2) and (3). This curve is known as characteristic of the envelope and the general integral thus represents the envelope of a family of surfaces considered as composed of its characteristics. (iii) Singular Integral. The singular integral is obtained by eliminating ‘a’ and ‘b’ between equation (1) and

!/ a = 0

...(4)

! / b = 0.

...(5)

Created with Print2PDF. To remove this line, buy a license at: http://www.software602.com/

Non-Linear Partial Differential Equations of Order One

3.3

The operation of elimination is equivalent to finding the envelope of all the surfaces included in the complete integral. (1), (4) and (5) give the point of contact of the particular surfaces represented by (1) with the general envelope. It follows that the singular integral represents the general envelope of all surfaces included in the complete integral. 3.3. Method of getting singular integral directly from the partial differential equation of first order. Let the given partial differential equation be f(x, y, z, p, q) = 0, ...(1) whose complete integral is of the form !(x, y, z, a, b) = 0, ...(2) where ‘a’ and ‘b’ are arbitrary constants. The singular integral of (1) is obtained by eliminating ‘a’ and ‘b’ between equation (2) !/ a = 0 ...(3) and !/ b = 0. ...(4) The values of z, p, q derived from (2) when substituted in (1) will reduce it into an identity and the substitution of the values of p and q (but not of z) will in general render (1) equivalent to the integral equation. By using this substitution p and q are replaced by functions of x, y, z, a and b in (1). It follows that the singular integral is given by (1) and the equations obtained on differentiating (1) partially w.r.t. ‘a’ and ‘b’, namely the equations f p f q # =0 p a q a

If

f & 0 and p

...(5)

f & 0, (5) and (6) hold if q

f p f q = 0. ...(6) # p b q b

p q p q = 0, ∋ a b b a

showing that there exists a functional relation between p and q which does not contain a and b explicitly. Let this functional relation be ∃(p, q) = 0. ...(7) If both the constants a and b occur in p and q (which does nolt always happen), then (7) shows that one of them is a function of the other and the equations using them give general integral which is not now required. Equations (5) and (6) are also true if f/ p = 0 ...(8) and f/ q = 0. ... (9) Elimination of p and q from (1), (7) and (8) will yield a relation between x, y, z free from ‘a’ and ‘b’. If this relation satisfies the given differential equation (1), it must be the singular integral. 3.4. COMPATIBLE SYSTEM OF FIRST–ORDER EQUATIONS [Delhi Maths (H) 2007; Pune 2010] Consider first order partial differential equations f(x, y, z, p, q) = 0 ...(1) and g(x, y, z, p, q) = 0. ...(2) Equations (1) and (2) are known as compatible when every solution of one is also a solution of the other. To find condition for (1) and (2) to be compatible. [Delhi 2008; Pune 2011] Let

J = Jacobian of f and g (

( f , g) / ( p, q ) & 0.

...(3)

Then (1) and (2) can be solved to obtain the explicit expressions for p and q given by p = !(x, y, z) and q = ∃(x, y, z). ...(4) The condition that the pair of equations (1) and (2) should be compatible reduces then to the condition that the system of equations (4) should be completely integrable, i.e., that the equation dz = pdx + qdy or ! dx + ∃dy – dz = 0, using (4) ...(5)

Created with Print2PDF. To remove this line, buy a license at: http://www.software602.com/

3.4

Non-Linear Partial Differential Equations of Order One

should be integrable. (5) is integrable if* ) ! ∃∗ !∗ ) ∃ ∗ ) !+ ∋ 0 , # ∃ + 0 ∋ , # ( ∋1) + ∋ , =0 z. x. − z . − − y

∃ ∃ ! ! #! = #∃ . ...(6) y z x z Substituting from equations (4) in (1) and differentiating w.r.t. ‘x’ and ‘z’ respectively, we get f f ! f ∃ # # =0 ...(7) x p x q x f f ! f ∃ and # # = 0. ...(8) z p z q z which is equivalent to

F ! # ! ! I # f F ∃ # ! ∃ I = 0. H x zK q H x z K !I ∃I g g gF ! gF ∃ #! # #! # #! = 0. x z pH x zK qH x zK ∃ ∃ 1 R ( f , g) # ! ( f , g) U #! = S V. J T (x, p) (z, p) W x z

f f f #! # x z p

From (7) and (8), Similarly (2) yields Solving (9) and (10),

...(9) ...(10) ...(11)

Again, substituting from equations (4) in (1) and differentiating w.r.t. ‘y’ and ‘z’ and proceeding ! ! #∃ =∋1 y z J

RS T

UV W

( f , g) ( f , g) #∃ ...(12) (y, q) (z, q) Substituting from equations (11) and (12) in (1) and replacing !, ∃ by p, q respectively, we obtain

as before, we obtain

1 J

RS T

UV W

( f , g) ( f , g) #p = –1 (x, p) (z, p) J

RS T

( f , g) ( f , g) #q ( y, q) ( z, q)

[f, g] (

where

UV W

or

[f, g] = 0,

( f , g) ( f , g) ( f , g) ( f , g) #p # #q ( x, p) ( z, p) ( y, q) ( z, q)

...(13) ...(14)

3.5. A PARTICULAR CASE OF ART. 3.4. To show that first order partial differential equations p = P(x, y) and q = Q(x, y) are compatible if and only if P / y / Q / x. Proof. Given Since

[Delhi Maths (H) 2009; Pune 2010]

z / x / p / P ( x, y )

and

z / z / q / Q ( x, y )

dz / ( z / x )dx # ( z / y )dy / pdx # qdy ,

... (1) ... (2)

it follows that the given partial differential equations (1) are compatible if and only if the single differential equation dz = Pdx + Qdy ... (3) is integrable. Since P and Q are functions of two variables x and y, hence Pdx + Qdy is an exact differential if and only if P / y / Q / x. Therefore (3) is integrable if and only if P / y / Q / x Remark 1. If P / y / Q / x, then the system of two given partial differential equations (1) is compatible and hence these will possess a common solution. ) Q

*Pdx + Qdy + Rdz = 0 is integrable if P +

− z



R∗

) P Q∗ ) R P∗ , # Q+ ∋ , # R+ ∋ , / 0. x z x. − . − y

y.

Created with Print2PDF. To remove this line, buy a license at: http://www.software602.com/

Non-Linear Partial Differential Equations of Order One

3.5

Remark 2. If P / y & Q / x, then the system of two given partial differential equations (1) is not compatible and hence these equations possess no solution. 3.6. SOLVED EXAMPLES BASED ON ART. 3.4 AND ART. 3.5. Ex. 1. (a) Show that the differential equations z / x / 5x ∋ 7 y and z / y / 6 x # 8 y aree not compatible. (b) z / x / 5 x ∋ 7 y , z / y / 6 x # 8 y possess (i) common solution (ii) No common solution (iii) No solution (iv) None of these. Point out correct choice. [Agra 2005, 06] 2 2 (c)Show that the differential equations p = x – ay, q = y – ax are compactible and find their common solution. (d) Show that the differential equations

z / x / ( x # y )2 ,

z / y / x2 # 2 xy ∋ y 2 aree

compatible and solve them (e)Show that p = x – y/(x2 + y2), q = y + x/(x2 + y2) are compatible and find their solution. (f) Show that p = 1 + ex/y, q = ex/y(1 – x/y) are compatible and find their solution. Sol. (a) Given dz/dx = p = 5x – 7y and dz/dy = q = 6x + 8y ...(1) Comparing (1) with p = P(x, y) and q = Q(x, y) ... (2) here p = 5x – 7y and Q = 6x + 8y ... (3) P / y / Q / x. Hence the

We know that p = P(x, y) and q = Q(x, y) are compatible if system (1) is compatible if P / y / Q / x. From (3), P / y / ∋7 and Q / x / 6 and so P / y & Q / x

Therefore, the given system (1) is not compatible. (b) Ans. (iii) As in part (a), the given system is not compatible. Hence the given equations have no solution (refer Art. 3.5). (c)We known that the system of equations p = P(x, y), q = Q(x, y) ... (1) is compatible if and only if Comparing p = x2 – ay, with (1), here P = x2 – ay,

P / y / Q / x. and and

q = y2 – ax Q = y2 – ax

... (2) ... (3)

From (3), P / y / ∋a / Q / dx and so equations (2) are compatible To find the common solution of (2). Substituting the values of p and q given by (2) in dz = pdx + qdy, we get dz = (x2 – ay) dx + (y2 – ax) dy = x2dx + y2dy – a d(xy) Integrating, z = (x3 + y3)/3 – axy + c, c being an arbitrary contant ... (4) (4) is the required common solution of the given equation (2). (d) We know that the system of equations p = P(x, y), q = Q(x, y) ...(1) is compatible if and only if Comparing with (1), here

z / x / p / ( x # y )2 , P = (x + y)2 = x2 + 2xy + y2

P / y / Q / x.

and and

z / y / q / x2 # 2 xy ∋ y 2 ... (2) Q = x2 + 2xy – y2

... (3)

From (3), P / y / 2 x # 2 y / Q / y and hence equations (2) are compatible. The find the common solution of (2). Substituting the values of p and q given by (2) in dz = pdx + qdy, we get dz = (x2 + 2xy + y2)dx + (x2 + 2xy – y2) dy

... (4)

Created with Print2PDF. To remove this line, buy a license at: http://www.software602.com/

3.6

Non-Linear Partial Differential Equations of Order One

Integrating (4) and noting that R.H.S. of (4) must an exact differential, we have, by method of solving an exact equation

0

z / ( x 2 # 2 xy # y 2 )dx # (Treating y as a constant)

0

( x 2 # 2 xy ∋ y 2 )

#c

(Integrating terms free from x )

z / x3 / 3 # x2 y # y 2 x ∋ y3 / 3 # c, c being an arbitrary constant

or

p = P(x, y),

(e)We know that the system of equation

P / y / Q / x.

is compatible if and only if 2

2

p = x – y/(x + y ), P = x – y/(x2 + y2)

Comparing with (1), here

q = Q(x, y) ... (1)

From (3),

and

q = y + x/(x2 + y2) ... (2) Q = y + x/(x2 + y2) ... (3)

and and

P 11 ( x 2 # y 2 ) ∋ 2 y 1 y y 2 ∋ x2 / 0∋ / 2 2 2 2 y (x ∋ y ) ( x # y 2 )2

... (4)

Q 11 ( x 2 # y 2 ) ∋ 2 x 1 x y 2 ∋ x2 / 0# / x ( x 2 # y 2 )2 ( x 2 ∋ y 2 )2

... (5)

(4) and (5) 2 P/ y / Q/ x The system (2) is compatible. 2 To find the solution of the system (2). Substituting the values of p and q given by (2) in dz = pdx + qdy, we get dz = {x – y/(x2 + y2)}dx + {y + x/(x2 + y2)}dy ... (6) Integrating (6) and noting that R.H.S. of (6) must be an exact differential, we obtain z/

0 3 x ∋ y /( x

2

4

# y 2 ) dx #

(Treating y as a constant)

0 3 y # x /( x

2

4

# y 2 ) dy # c

(Integrating terms free from x )

or z = x2/2 – y × (1/y) × tan–1 (x/y) + y2/2 + c = (x2 + y2)/2 – tan–1 (x/y) + c, which is the required solution, c being an arbitrary constant. (f) We know that the system of equations p = P(x, y), and q = Q(x, y) ... (1) P / y / Q / x.

is compatible if and only if x/y

p=1+e P = 1 + ex/y

Comparing with (1), here (3) 2

and and

q = ex/y(1 – x/y) Q = ex/y (1 – x/y)

P / y / 0 # e x / y (∋ x / y 2 ) / ∋( x / y 2 ) e x / y

... (4)

Q / x / e x / y 5 (1/ y) 5 (1 ∋ x / y) # e x / y 5 (∋1/ y ) / ∋( x / y 2 )e x / y

and

... (2) ... (3)

... (5)

P/ y / Q/ x The system (2) is compatible. 2 2 To find the solution of (2). Substituting the values of p and q given by (2) in dz = pdx + qdy, we get dz = (1 + ex/y)dx + ex/y(1 – x/y)dy ... (6) Integrating (6) and noting that R.H.S. of (6) must be an exact differential, we obtain (4) and (5)

z/ or

0

(1 # e x / y )dx (Treating y as constant)

#

0

e x / y (1 ∋ x / y ) dy

#c

(Integrating terms free from x )

z = x + y ex/y + c, c being an arbitrary constant.

Created with Print2PDF. To remove this line, buy a license at: http://www.software602.com/

Non-Linear Partial Differential Equations of Order One

3.7

Ex. 2. Show that the equations xp = yq and z(xp + yq) = 2xy are compatible and solve them. [Delhi Maths (Hons) 2005, 07, 11] Sol. Let f(x, y, z, p, q) = xp – yq = 0 ...(1) and g(x, y, z, p, q) = z(xp + yq) – 2xy = 0 ...(2)

6

and 6 [f, g] =

( f , g) / ( x, p)

f x g x

p x f p = zp ∋2y xz = 2xy, g p

( f , g) / (z, p)

f z g z

0 x f p = xp # yq xz = – x2p – xyq, g p

( f , g) / ( y, q)

f y g y

f q g q =

( f , g) / ( z, q)

f z g z

0 ∋y f q = xp # yq zy = xyp + y2q. g q

∋q

∋y

zq ∋ 2 x

zy

= –2xy

( f , g) ( f , g) ( f , g) ( f , g) #p # #q = 2xy – x2p2 – xyqp – 2xy + xypq + y2q2 ( x, p) ( z, p) ( y, q) ( z, q)

= – xp(xp + yq) + yq(xp + yq) = – (xp – yq) (xp + yq) = 0, using (1) Hence (1) and (2) are compatible. Solving (1) and (2) for p and q, p = y/z and q = x/z. ... (3) Using (3) in dz = pdx + qdy, we have dz = (y/z)dx + (x/z)dy or z dz = d(xy). Integrating, z2/2 = xy + c/2 or z2 = 2xy + c, where c is an arbitrary constant. Ex. 3. Show that the equations xp – yq = x and x2p + q = xz are compatible and find their solution. [Delhi B.Sc. II (Prog) 2009; Delhi Maths Hons. 2007] Sol. Let f(x, y, z, p, q) = xp – yq – x = 0. ...(1) and g(x, y, z, p, q) = x2p + q – xz = 0. ...(2)

6

( f , g) f x / g x ( x, p)

Similarly,

( f , g) = x2, ( z, p)

f p = p ∋1 2 xp ∋ z g p

x 2 x 2 = (p – 1)x – x(2xp – z). ( f , g) = –xy. ( z, q )

( f , g) = – q, ( y, q )

( f , g) ( f , g) ( f , g) ( f , g) #p # #q = (p – 1)x2 – x(2xp – z) – px2 – q – xyq ( x, p) (z, p) (y, q) (z, q) = – x2 + zx – q – xyq = –x2 + x2p – qxy, by (2) = x(– x + xp – yq) = 0, by (1) Hence (1) and (2) are compatible. Solving (1) and (2) for p and q, p = (1 + yz)/(1 + xy) and q = x(z – x)/(1 + xy). ...(3) Using (3) in dz = pdx + qdy, dz = [(1 + yz)/(1 + xy)] dx + [x(z – x)/(1 + xy)]dy (1 + xy)dz = (1 + yz)dx + x(z – x)dy or (1 + xy)dz – z(ydx + xdy) = dx – x2dy

6 [f, g] =

or or

dx ∋ x 2 dy (dx x 2 ) ∋ dy (1 # xy) dz ∋ z d ( xy) / = 2 (1 # xy) (1 # xy)2 ( y # 1 x )2

Integrating it, or

or

) z ∗ ∋d ( y #1 x) d+ . , = 1 # xy ( y # 1 x )2 − .

1 z = or #c 1# xy ( y # 1 x) z – x = c(1 + xy), c being an arbitrary constant.

z / x #c 1 # xy 1 # xy

Created with Print2PDF. To remove this line, buy a license at: http://www.software602.com/

3.8

Non-Linear Partial Differential Equations of Order One

Ex. 4. Show that the equation z = px + qy is compatible with any equation f(x, y, z, p, q) = 0 which is homogeneous in x, y, z. [Delhi Maths, Hons. 2001, 06, 10] Sol. Given that differential equation f(x, y, z, p, q) = 0 ...(1) is homogeneous in x, y, z. Then, clearly f(x, y, z, p, q) will be a homogeneous function in variables x, y, z ; say of degree n. Then, by Euler’s theorem on homogeneous function, we have x( f / x) # y ( f / y) # z ( f / z ) / nf We take Then, using (3), we have ( f , g) / ( x, p ) ( f , g) / ( z, p ) ( f , g) / ( y, q ) ( f , g) / ( z, q )

and

6

[ f , g] /

so that x( f / x ) # z ( f / z ) / 0, by (1)

g(x, y, z, p, q) = px + qy – z = 0

f/ x

f/ p

g/ x

g/ p

f/ z

f/ p

g/ z

g/ p

f/ y

f/ q

g/ y

g/ q

f/ z

f / q

g/ z

g/ q

/

/

/

/

f/ x

f / p

p

x

f/ z

f / p

∋1

x

f/ y

f/ q

q

y

f/ z

f/ q

∋1

y

/x

... (3) f f ∋p , x p

/x

f f # , z p

/y

f f ∋q y q

/y

... (2)

f f # z q

( f , g) ( f , g) ( f , g) ( f , q) #p # #q ( x, p ) ( z, p ) ( y, q ) ( z, q )

/x

) f ) f f f f∗ f f f∗ ∋ p # p+ x # , # y ∋ q # q+ y # , x p − z p. y q − z q.

/x

f f f f f f # y # 7 px # qy 8 / x # y # z , using (3) x y y x y z

= 0, using (2) Hence, the differential equation z = px + qy is compatible with any differential equation f(x, y, z, p, q) that is homogeneous in x, y, z. Ex. 5. If u1 = u/ x, u2 = u/ y, u3 = u/ z, show that the equations f(x, y, z, u1, u2, u3) = 0 and g(x, y, z, u1, u2, u3) = 0 are compatible if

( f , g) ( f , g) ( f , g) # # = 0. (Delhi Maths (H) 2004) ( x, u1) (y, u2 ) (z, u3)

Sol. Treating z as constant, given equations are compatible if ( f , g) ( f , g) ( f , g) ( f , g) #u # #u = 0. ...(1) ( x, u1) 1 (u, u1) ( y, u2 ) 2 (u, u2 ) Since f and g do not contain u, we have f/ u = 0 and g/ u = 0. ...(2) ( f , g) ( f , g) =0 and = 0. ...(3) 6 (u, u1) (u, u2 ) ( f , g) ( f , g) # (1) reduces to = 0. ...(4) 6 ( x, u1) ( y, u2 ) Similarly treating x and y constant respectively, given equtions are compatible if

Created with Print2PDF. To remove this line, buy a license at: http://www.software602.com/

Non-Linear Partial Differential Equations of Order One

3.9

( f , g) ( f , g) ( f , g) ( f , g) # =0 ...(5) # = 0. ...(6) ( y, u2 ) (z, u3 ) ( x, u1 ) (z, u3 ) We know that the given equations are compatible when they remain compatible even when any variable is taken as constant, i.e., (4), (5) and (6) hold simultaneously. Hence adding (4), (5) and (6), the required condition for given equations to be compatible is ( f , g) ( f , g) ( f , g) # # = 0. ( x, u1 ) (y, u2 ) (z, u3 )

Ex. 6. Show that the equations f ( x, y, p, q) / 0 , g( x, y, p, q) / 0 are compatible if

( f , g) / ( x, p) # ( f , g) / ( y, q) / 0 Verify that the equations p / P( x, y), q / Q ( x, y) are compatible if P / y / Q / x . Sol. We know that

f ( x, y, z, p, q) / 0

g( x, y, z, p, q) / 0

and

( f , g) ( f , g) ( f , g) ( f , g) #p # #q /0 ( x , p) ( z, p) ( y, q) (z, q)

are compatible if

... (1) ... (2)

First part: Comparing the given equations f ( x, y, p, q) / 0 and g( x, y, p, q) / 0 with (1), we find that z is absent in given equations and so

f / z/0

f/ z ( f , g) / ( z, p) g/ z

Now, and

g/ z / 0

and f/ p g/ p

/

0

f/ p

0

g/ p

f/ z f/ q 0 ( f , g) / / g/ z g/ q 0 (z, q) Substituting these values in (2), the required condition is

... (3)

/0

f/ q /0 g/ q

( f , g) / ( x, p) # ( f , g) / ( y, q) / 0 Second Part. Let so

f / P( x, y) ∋ p

g / Q( x, y) ∋ q

and

... (4)

Comparing (4) with (1), we find that z and q are absent in f and z and p are absent in g and f / z / 0, f / q / 0, g/ z / 0 and g/ p / 0 ... (5)

6

( f , g) / ( x, p)

f/ x

f/ p

g/ x

g/ p

f/ z ( f , g) / ( z, p) g/ z f/ y ( f , g) / ( y, q) g/ y

f/ p g/ p f/ q g/ q

/ / /

P/ x

∋1

Q/ x

0

0 ∋1 0

0

/

Q x

/∋

P y

/0

P/ y

0

Q / y ∋1

f/ z f/ q 0 0 ( f , g) / / /0 ( z, q ) g / z g / q 0 ∋1 Substituting these values in (2), the required condition is or Q/ x ∋ P/ y / 0

P/ y / Q/ x .

Ex. 7. Show that p2 # q2 / 1 and ( p2 # q2 ) x / pz are compatible and solve them. Hint. Proceed as in solved Ex. 1 Ans. z 2 / x 2 # ( y # c)2 .

Created with Print2PDF. To remove this line, buy a license at: http://www.software602.com/

3.10

Non-Linear Partial Differential Equations of Order One

Ex. 8. Solve completey the simultaneous equations: z = px + qy and 2xy(p2 + q2) = z(yp + xq). [Delhi Math (H) 2006, 10] z / px # qy Sol. Given ... (1)

2 xy( p2 # q2 ) ∋ z( yp # xq) / 0

and

... (2)

Let f ( x, y, z, p, q) / 2xy ( p2 # q2 ) ∋ z ( yp # xq) = z2 {2(x / z) ( y / z) ( p2 # q2 ) ∋ (y / z) p ∋ (x / z)q} , showing that f ( x, y, z, p, q) is homogeneous in x, y, z. We know that (refer solved example 4) the equation z / px # qy is compatible with any equation f ( x, y, z, p, q) / 0 which is homogenous in x, y, z. Hence (1) and (2) are compatible.

q / ( z ∋ px) / y

From (1), we have

... (3)

2 x( x2 # y2 ) p2 ∋ z(3x 2 # y2 ) p # xz 2 / 0

Using (3), (2) gives

(2 xp ∋ z) {( x2 # y2 ) p ∋ xz} / 0

or so that Using (4), (3) gives

p / z / 2 x , xz /( x 2 # y2 )

... (4)

q / z / 2 y , yz /( x 2 # y2 )

... (5)

Using the corresponding values p / z / 2x, q / z / 2 y in dz / px # qdy , we get dz = (z/2x)dx + (z/2y)dy

or

2(1/ z)dz / (1/ x )dx # (1/ y)dy

2 log z / log x # log y # log C1

Integrating,

or

z 2 / C1xy ...(6)

Similarly, using the corresponding values p / xz /( x2 # y2 ) and q / yz /( x2 # y2 ) in dz = pdx + qdy, we get dz /

xzdx 2

x #y

Integrating,

2

#

yzdy 2

x #y

2

2 dz 2( xdx # ydy) / z x 2 # y2

or

2 log z / log( x 2 # y2 ) # log C2

or

z 2 / C2 ( x 2 # y2 )

... (7)

(6) and (7) give two two common solutions of (1) and (2)

EXERCISE 3(A) 1. Show that z / x / 7 x # / 8 y ∋ 1 and z / y / 9 x # 11y ∋ 2 are not compatible. 2. Show that the partial differential equations p = 6x – 4y + 1 and q = 4x + 6y + 1 do not possess any common solution. 3. Show that the following system of partial differential equations are compatible and hence solve them (i) p = 6x + 3y, q = 3x – 4y Ans. z = 3x2 + 3xy – 2y2 + c (ii) p = ax + hy + g, q = hx + by + f Ans. z = (ax2 + by2)/2 + hxy + gx + fy + c (iii) z / x / y (2ax # by ), z / y / x(ax # 2by ) (iv) p = x4 – 2xy2 + y4, q = 4xy3 – 2x2y – sin y (v) p = (ey + 1) cos x, q = ey sin x (vi) p = y (1 + 1/x) + cos y, q = x + log x – x sin y

Ans. z = ax2y + bxy + c Ans. z = x5/5 – x2y2 + xy4 + cos y + c Ans. z = (ey + 1) sin x + c Ans. z = y (x + log x) + x cos y + c

Created with Print2PDF. To remove this line, buy a license at: http://www.software602.com/

Non-Linear Partial Differential Equations of Order One 2

3.11

2

2

(vii) p / y 2 e xy # 4 x3 , q / 2 xy e xy ∋ 3 y 2

Ans. z / e xy # x 4 ∋ y 3 # c

(viii) p = sin x cos y + e3x, q = cos x sin y + tan y Ans. x = (1/3) × e3x – cos x cos y + log sec x + c 3.7. Charpit’s method.* (General method of solving partial differential equations of order one but of any degree. [Agra 2003; Delhi Maths (H) 2000, 05, 06, 08-11; Kanpur 1998; Meeerut 2003, 05; Nagpur 2002, 04, 06, 08; Rohilkhand 2001, 04] Let the given partial equation differential of first order and non–linear in p and q be f(x, y, z, p, q) = 0. ...(1) We know that dz = p dx + q dy. ...(2) The next step consists in finding another relation F(x, y, z, p, q) = 0 ...(3) such that when the values of p and q obtained by solving (1) and (3), are substituted in (2), it becomes integrable. The integration of (2) will give the complete integral of (1). In order to obtain (3), differentiate partially (1) and (3) with respect to x and y and get f f f p f q # p# # = 0, ... (4) x z p x q x F F F p F q # p# # = 0, ... (5) x z p x q x f f f p f q # q# # =0 ... (6) y z p y q y F F F p F q # q# # and = 0. ... (7) y z p y q y Eliminating p/ x from (4) and (5), we get

FG H

or

FG H

f f f q # p# x z q x

IJ FG K H

f F F f ∋ # x p x p

IJ K

FG F # F p # F qIJ f = 0 H x z q xK p fI JpK p # )+− qf Fp ∋ Fq pf .∗, qx = 0.

F ∋ p

f F F ∋ z p z

Similarly, eliminating q/ y from (6) and (7), we get

FG H

...(8)

IJ K

) f F F f ∗ ) f F F f ∗ f F F f p ∋ ∋ ...(9) + ,#+ , q # p q ∋ p q y = 0. y q y q z q z q − . − . Since q/ x = 2z/ x y = p/ y, the last term in (8) is the same as that in (9), except for a minus sign and hence they cancel on adding (8) and (9). Therefore, adding (8) and (9) and rearranging the terms, we obtain

F H

I K

FG H

IJ K

FG H

IJ K

FG IJ H K

FG IJ H K

f f F f f f f F # ∋ f F# ∋ f F = 0. ...(10) #p # # q F # ∋p ∋ q p x q y x z p y z q p q z This is a linear equation of the first order to obtain the desired function F. As in Art 2.20 of chapter 2, integral of (10) is obtained by solving the auxiliary equations dp = ( f ( f x ) # p ( f z)

dq y) # q ( f

z)

#

∋ p( f

dz p) ∋ q ( f

q)

=

dy dx = / dF . ...(11) ∋ f p ∋ f q 0

*This is general method for solving equations with two independent variables. Since the solution by this method is generally more complicated, this method is applied to solve equations which cannot be reduced to any of the standard forms which will be discussed later on. Thus, Charpit’s method is used in two situations (i) When you are asked to solve a problem by Charpit’s method (ii) when the given equation is not of any four standard forms given in Articles 3.10, 3.12, 3.14 and 3.17.

Created with Print2PDF. To remove this line, buy a license at: http://www.software602.com/

3.12

Non-Linear Partial Differential Equations of Order One

Since any of the integrals of (11) will satisfy (10), an integral of (11) which involves p or q (or both) will serve along with the given equation to find p and q. In practice, however, we shall select the simplest integral. Note. In what follows we shall use the following standard notations: f / x / fx ,

f / y / fy,

f / z / fz ,

f / p / f p,

f / q / f q.

Therefore, Charpit’s auxiliary equations (11) may be re–written as dp dq dz dx dy dF / / / / / fx # p fz f y # q f z ∋ p f p ∋ q fq ∋ f p ∋ fq 0

... (11)%

3.8A. WORKING RULE WHILE USING CHARPIT’S METHOD Step 1. Transfer all terms of the given equation to L.H.S. and denote the entire expression by f. Step 2. Write down the Charpit’s auxiliary equations (11) or (11)%. Step 3. Using the value of f in step 1 write down the values of f/ x, f/ y ..., i.e., fx, fy, ... etc. occuring in step 2 and put these in Charpit’s equations (11) or (11)%. Step 4. After simplifying the step 3, select two proper fractions so that the resulting integral may come out to be the simplest relation involving at least one of p and q. Step 5. The simplest relation of step 4 is solved along with the given equation to determine p and q. Put these values of p and q in dz = p dx + q dy which on integration gives the complete integral of the given equation. The Singular and General integrals may be obtained in the usual manner. Remark. Sometimes Charpit’s equations give rise to p = a and q = b, where a and b are constants. In such cases, putting p = a and q = b in the given equation will give the required complete integral. 3.8.B. SOLVED–EXAMPLES BASED ON ART. 3.8A. Ex. 1. Find a complete integral of z = px + qy + p2 + q2. [Bilaspur 2000l; Bhopal 1996, I.A.S. 1996; Indore 2000; Jabalpur 2000; K.U. Kurukshetra 2005; Ravishankar 2000; 04; Meerut 2010; Garhwal 2010] Sol. Let f(x, y, z, p, q) ( z – px – qy – p2 – q2 = 0 ... (1) Charpit’s auxiliary equations are From (1), fx = –p, fy = –q, Using (3), (2) reduces to

dp dq dz dx dy / / / / f x # p f z f y # q f z ∋ p f p ∋ q fq ∋ f p ∋ fq

... (2)

fz = 0,

... (3)

fp = –x – 2p

and

fq = –y – 2q

dp dq dz dx dy / / / / 0 0 p ( x # 2 p ) # q ( y # 2q ) x # 2 p y # 2 q

Taking the first fraction of (4), dp = 0 so that p=a Taking the second fraction of (4), dq = 0 so that q=b Putting p = a and q = b in (1), the required complete integral is z = ax + by + a2 + b2, a, b being arbitrary constants. Ex. 2. Find a complete integral of q = 3p2. [Agra 2 Sol. Here given equation is f(x, y, z, p, q) ( 3p – q = 0.

6 Charpit’s auxiliary equations are

... (4) ... (5) ... (6)

2006] ...(1)

dp dq dy dz / / dx / = f f f f f f f f #p #q ∋p ∋ q ∋ ∋ x z y z p q p q

Created with Print2PDF. To remove this line, buy a license at: http://www.software602.com/

Non-Linear Partial Differential Equations of Order One

3.13

dp dq dy dz dx / / / , using (1) = 2 ∋6 p # q ∋6 p 1 0 # p.0 0 # q .0

or

...(2)

Taking the first fraction of (1), dp = 0 so that p = a. Substituting this value of p in (1), we get q = 3a2. Putting these values of p and q in dz = pdx + qdy, we get dz = adx + 3a2dy so that z = ax + 3a2y + b, which is a complete integral, a and b being arbitrary constants. Ex. 3. Find the complete integral of zpq = p + q [Nagpur 2010; Meerut Sol. Let f(x, y, z, p, q) = zpq – p – q = 0 Here Charpit’s auxiliary equations are

...(3) ...(4)

2006] ... (1)

dp dq dz dx dy / / / / fx # p fz f y # q f z ∋ p f p ∋ q fq ∋ f p ∋ fq ... (2)

From (1), fx = 0, fy = 0, Using (3), (2) reduces to dp 2

p q

/

dq pq

2

/ ......

or

fz = pq,

fp = zq – 1

dp dq / p q

and

fq = zp – 1

... (3)

p = aq

... (4)

so that

Solving (1) and (2), p = (1 + a)/z and q = (1 + a)/az. dz = pdx + qdy = [(1 + a)/z]dx + [(1 + a)/az]dy or 2zdz = 2(1 + a) [dx + (1/a)dy] 6 2 Integrating, z = 2(1 + a) [x + (1/a)y] + b, a, b being arbitrary constants Ex. 4. Find a complete integral of p2 – y2q = y2 – x2. [M.D.U. Rohtak 2006] Sol. Here given equation is f(x, y, z, p, q) = p2 – y2q – y2 + x2 = 0. ...(1) Charpit’s auxiliary equations are

or

dp dq dy dz dx / / 2 , using (1) = ...(2) 2 / 2 x ∋2qy ∋ 2y ∋ 2 p ∋ p (2 p) ∋ q(∋ q ) y Taking the first and fourth fractions, pdp + xdx = 0 so that p2 + x2 = a2 ... (3) 2 2 1/2 Solving (1) and (3) for p and q, p = (a – x ) , q = a2y–2 – 1. dz = pdx + qdy = (a2 – x2)1/2dx + (a2y–2 –1)dy. 6 Integrating, z = (x/2) × (a2 – x2)1/2 + (a2/2) × sin–1 (x/a) – (a2/y) – y + b. Ex. 5. Find a complete integral of z2(p2z2 + q2) = 1. [I.A.S. 1997; Meerut 2007] Sol. Here given equation is f(x, y, z, p, q) = p2z4 + q2z2 – 1 = 0. ...(1)

Charpit’s auxiliary equations are or

dy dp dq dz / dx / = / f f f f f f f f ∋p ∋ q ∋ ∋ #p #q p q p q x z y z

dp dq dz dx dq / / / / fx # p fz f y # q f z ∋ p f p ∋ q fq ∋ f p ∋ fq

dp dq dy dz dx , by (1) ... (2) 2 4 2 2 = 2 = 2 3 2 = 4 / p (4 p z # 2 zq ) q (4 p z # 2 zq ) ∋2 p z ∋ 2q z ∋2 pz ∋2qz 2 Taking the first two fractions, (1/p)dp = (1/q)dq so that p = aq. 2 3

Solving (1) and (2) for p and q,

p=

a 2 2

1/ 2

z (a z # 1) 2 2 1/2 dz = pdx + qdy = (a dx + dy)/z (a z + 1) or 6

,

q=

1

. z (a z # 1)1/ 2 adx + dy = z(a2z2 + 1)1/2dz. 2 2

Created with Print2PDF. To remove this line, buy a license at: http://www.software602.com/

3.14

Non-Linear Partial Differential Equations of Order One

z

2 2 12 Integrating, ax + y = (a z # 1) . zdz . Putting a2z2 + 1 = t2 so that 2a2zdz = 2tdt, (3) becomes

ax + y =

z

(1 a 2 ) t . t dt

...(3)

ax + y + b = (1/3a2)t3, where t = (a2z2 + 1)1/2

or

or ax + y + b = (1/3a2) × (a2z2 + 1)3/2 or 9a4(ax + y + b)2 = (a2z2 + 1)3, which is a complete integral, a and b being arbitrary constants. Ex. 6. Find a complete integral of px + qy = pq. [Kurukshetra 2006 Rajasthan 2000, 01, Gulbarga 2005; Meerut 2002; Kanpur 2004; Jiwaji 2004; Rewa 2001; Vikram 2000, 03, 04; Bhopal 2010] Sol. Here given equation is f(x, y, z, p, q) ( px + qy – pq = 0. ...(1) dp dq dz dx dq / / / / fx # p fz f y # q f z ∋ p f p ∋ q fq ∋ f p ∋ fq dp dy dp dq dz or = , by (1) ...(2) / / / ∋( x ∋ q) ∋( y ∋ q) ∋ p( x ∋ q) ∋ q(y ∋ p) p # p . 0 q # q . 0 Taking the last two fractions of (2), (1/p)dp = (1/q)dq. Integrating, log p = log q + log a or p = aq. ...(3) Substituting this value of p in (1), we have aqx + qy – aq2 = 0 or aq = ax + y, as q & 0 ...(4) From (3) and (4), q = (ax + y)/a and p = ax + y. ...(5) 6 Putting these values of p and q in dz = pdx + qdy, we get dz = (ax + y)dx + [(ax + y)/a] dy or adz = (ax + y) (adx + dy) or adz = (ax + y) d(ax + y) = udu, where u = ax + y. Integrating, az = u2/2 + b = (ax + y)2/2 + b, which is a complete integral, a and b being arbitrary constants. Ex. 7. Find the complete integrals of following equations: (i) q = (z + px)2 [Indore 2004; Ravishanker 2005] (ii) p = (z + qy)2 [Meerut 2008, 09; Agra 2001; Delhi B.Sc. (Prog) 2008; Kurukshetra 2005] Sol. (i). Here given equations is f(x, y, z, p, q) = (z + px)2 – q = 0 ... (1)

Charpit’s auxiliary equations are

dp dq dz dx dq / / / / fx # p fz f y # q f z ∋ p f p ∋ q fq ∋ f p ∋ fq dq dy dp dz dx = = = = , by (1) ∋2 px(z # px) # q 0 2q(z # px) ∋2 x(z # px) 2 p(z # px) # 2 p (z # px ) Taking the second and fourth fractions, (1/q)dq = – (1/x)dx. Integrating, log q = log a – log x so that q = a/x. ...(2) Substituting the above value of q in (1), we have

Charpit’s auxiliary equations are

or

or or

(z + px)2 = a/x

or

6

dz = pdx + qdy =

px =

a

x –z

or

FG a ∋ z IJ dx + a dy, H x x xK x

p=

a x x ∋z x.

by (2) and (3)

a x–1/2dx – zdx + ady or xdz + zdx = d(xz) = a x–1/2dx + ady. Integrating, xz = 2 a x + ay + b, a, b being arbitrary constants xdz =

(ii) Sol. Do as in part (1).

...(3)

a x–1/2dx + ady

Ans. yz = ax +

ay + b.

Created with Print2PDF. To remove this line, buy a license at: http://www.software602.com/

Non-Linear Partial Differential Equations of Order One

3.15

2

Ex. 8. Find a complete integral of yzp – q = 0. Sol. Here f(x, y, z, p, q) = yzp2 – q = 0. dp dq dz dx dy / / / / fx # p fz f y # q f z ∋ p f p ∋ q fq ∋ f p ∋ fq

Charpit’s uxiliary equations are or or

dp dq dy dz dx / / , by (1) 2 / 2 2 = 2 ∋ 2 yzp 1 0 # p( yp ) zp # q( yp ) ∋2 yzp # q 3 Taking the first and fifth fractions, (1/yp ) dp = dy p–3dp = ydy or –2p–3dp = –2ydy. Integrating, p–2 = a2 – y2 so that p = 1/(a2 – y2)1/2. 2 Using (3), (1) 2 q = yzp q = yz/(a2 – y2). 2

dz = pdx + qdy =

6 ( a 2 ∋ y 2 )1/ 2 dz ∋

or

yzdy 2

( a ∋ y 2 )1/ 2

dx 2

(a ∋ y )

= dx

Integrating, z (a 2 ∋ y 2 )1/ 2 = x + b or

2 1/ 2

#

or

yzdy (a ∋ y 2 )

d [z (a 2 ∋ y 2 )1/ 2 ] = dx.

or

z2(a2 – y2) = (x + b)2, a, b being arbitrary constants. [I.A.S. 1994] ...(1)

dp dq dz dx dy / / / / fx # p fz f y # q f z ∋ pf p ∋ q f q ∋ f p ∋ f q

dp dq dy dz = = = dx 2 / . q(32 p 2 z # 18q 2 z # 8z ) ∋ p(32 pz 2 ) ∋ q(18qz 2 ) ∋ p(32 p2z # 18q2z # 8z) ∋32pz ∋18qz2

Taking the first and second fractions, (1/p)dp = (1/q)dq Solving (1) and (2) for p and q, we have q=

2(1 ∋ z 2 )1/ 2 2

and

1/ 2

z (16a # 9)

dz = pdx + qdy =

Hence,

2(1 ∋ z 2 )1/ 2 z (16a 2 # 9)1/ 2

so that

p=

p = aq 2a(1 ∋ z 2 )1/ 2

z (16a 2 # 9)1/ 2

Putting or Integrating,

2

1–z =t

... (2)

.

...(3)

(adx + dy), using (3)

(1/ 2) 5 (16a 2 # 9)1/ 2 (1 – z2)–1/2 (–2zdz) = – 2(adx + dy).

or

or

...(3) ...(4)

2

Ex. 9. Find a complete integral of 16p2z2 + 9q2z2+ 4z2 – 4 = 0. Sol. Given equation is f(x, y, z, p, q) = 16p2z2 + 9q2z2 + 4z2 – 4 = 0. Charpit’s auxiliary equations are

...(2)

...(4)

so that –2zdz = dt, (4) becomes 2 1/2 –1/2 (1/2) × (16a + 9) t dt = – 2 (adx + dy). (16a2 + 9)1/2 t1/2 = – 2(ax + y) + b, a, b being arbitrary constants.

2 (16 a 2 # 9)1/ 2 (1 ∋ z 2 ) + 2(ax + y) = b, as t = 1 – z .

Ex. 10(a). Find a complete integral of (p2 + q2)x = pz. [Agra 2003; Rajasthan 2005; Ravishankar 2001; Delhi Maths (Hons) 2004, 05] (b). Find the complete integral of the partial differential equation (p2 + q2)x = pz and deduce the solution which passes through the curve x = 0, z2 = 4y. [Meerut 2007] Sol. Let f(x, y, q, p, q) = (p2 + q2)x – pz = 0. ...(1)

Created with Print2PDF. To remove this line, buy a license at: http://www.software602.com/

3.16

Non-Linear Partial Differential Equations of Order One

dp dq dz dx dy / / / / fx # p fz f y # q f z ∋ p f p ∋ q fq ∋ f p ∋ fq

Charpit’s auxiliary equations are

giving dp/q2 = dq/(–pq), by (1) or 2pdp + 2qdq = 0. Integrating, p2 + q2 = a2, where a is an arbitrary constant. p = a2x/q

Solving (1) and (2), 6

dz = pdx + qdy =

q = (a/z) ×

and

2 2 2 a 2 xdx a (z ∋ a x ) dy # z z

or

...(2)

(z 2 ∋ a2 x 2 ) .

zdz ∋ a2 xdx (z 2 ∋ a2 x 2 )

...(3) = ady.

Putting z2 – a2x2 = t so that 2(zdz – a2xdx) = dt, we get

(1/ 2 t )dt = ady

(1/ 2) 5 t ∋1/ 2 = ady.

or

as t / z 2 ∋ a 2 x 2 (z 2 ∋ a2 x 2 ) = ay + b, 2 2 2 or z – a x = (ay + b) or z = a x + (ay + b)2. ... (4) (b) Proceeding as in part (a), (4) is the complete integral. The parametric equations of the given curve x = 0, z2 = 4y are given by x = 0, y = t2, z = 2t ... (5) Therefore the intersections of (1) and (2) are determined by 4t2 = (at2 + b)2 or a2t4 + 2(ab – 2)t2 + b2 = 0 ... (6) Equation (6) has equal roots if its discriminant = 0, i.e., if 4(ab – 2)2 – 4a2b2 = 0 or a2b2 = 1 so that b = 1/a Hence from (4), the appropriate one paremeter sub–system is given by z2 = a2x2 + (ay + 1/a)2 or a4(x2 + y2) + a2(2y – z2) + 1 = 0, which is a quatratic equation in parameter ‘a’. Therefore, this has for its envelope surface (2y – z2)2 – 4(x2 + y2) = 0 or (2y – z2)2 = 4(x2 + y2) ... (7) The desired solution is given by the function z defined by equation (7). Ex. 10(c). Find a complete, singular and general integrals of (p2 + q2)y = qz. [Guwahati 2007; Agra 2001; Bilaspur 1998; Delhi Maths (H) 2003, 05; Garhwal 2005; Meerut 2010, 11; K.V. Kurukshetra 2004; Kanpur 2005; Rohilkhand 2001; Pune 2010] Sol. Here the given equation is f(x, y, z, p, q) = (p2 + q2)y – qz = 0. ...(1) Integrating,

t1/2 = ay + b 2

or

2 2

Charpit’s auxiliary equations are or

2

dp dq dz dx dy / / / / fx # p fz f y # q f z ∋ pf p ∋ q f q ∋ f p ∋ f q

dp dq dz dx / dy / = = , by (1) ...(2) 2 2 ∋ pq p 2 ∋ 2 py ∋2qy # z ∋2 p y # qz ∋ 2q y Taking the first two fractions, we get 2pdp + 2qdq = 0 so that p2 + q2 = a ... (3) Using (3), (1) gives a2y = qz or q = a2y/z. Putting this value of q in (3), we get

a ( z 2 ∋ a2 y2 ) . z Now putting these values of p and q in dz = pdx + qdy, we have

p=

(a2 ∋ q2 ) =

a 2 y dy dz = a (z 2 ∋ a2 y 2 ) dx + dy z z

a 2 ∋ (a 4 y 2 / z 2 ) /

or

zdz ∋ a2 y dy (z 2 ∋ a2 y 2 )

= a dx.

Created with Print2PDF. To remove this line, buy a license at: http://www.software602.com/

Non-Linear Partial Differential Equations of Order One 2

2 2 1/2

3.17 2

2 2

2

Integrating, (z – a y ) = ax + b or z – a y = (ax + b) , ...(4) which is a required complete integral, a, b being arbitrary constants. Singular Integral. Differentiating (4) partially w.r.t. a and b, we have 0 = 2ay2 + 2 (ax + b)x ...(5) and 0 = 2(ax + b). ...(6) Eliminating a and b between (4), (5) and (6), we get z = 0 which clearly satisfies (1) and hence it is the singular integral. General Integral. Replacing b by !(a) in (4), we get z2 – a2y2 = [ax + !(a)]2. ...(7) Differentiating (7) partially w.r.t. a, –2ay2 = 2[ax + !(a)] . [x + !%(a)]. ...(8) General integral is obtained by eliminating a from (7) and (8). Ex. 11. Find a complete integral of p (1 + q2) + (b – z)q = 0. [Agra 1996] Sol. Here given equation is f(x, y, z, p, q) ( p(1 + q2) + (b – z)q = 0. ...(1) Charpit’s auxiliary equations are

dp dq dy dz dx / = / / f f f f f f f f #p #q ∋p ∋ q ∋ ∋ x z y z p q p q

dz dx dy dp dq / / = = , by (1) 2 2 pq p2 ∋ p (1 # q ) ∋ (b ∋ z)q ∋(q # 1) ∋2 pq ∋ (b ∋ z )

or

First two fractions give

(1/p)dp = (1/q)dq

so that

q = pc.

p = [c (z ∋ b) ∋ 1] c .

Putting q = pc in (1), we have

q = pc gives q = [c ( z ∋ b) ∋ 1] . 6 Putting these values of p and q in dz = pdx + qdy, we get dz = [c ( z ∋ b) ∋ 1]

FH dx # dyIK c

or

cdz = dx + c dy. [c (z ∋ b) ∋ 1]

Integrating, 2 [c ( z ∋ b) ∋ 1] = x + cy + a or 4{c(z – b) – 1} = (x + cy + a)2 which is a complete integral, a and c being arbitrary constants. Ex. 12. Find a complete and singular integrals of 2xz – px2 – 2q xy + pq = 0. [I.A.S. 1991, 93, 2007, 2008; Delhi Hons. 2001, 01, 05; Kanpur 2001, 03; Meerut 2005; Bhopal 2004, 10; Indore 1999; M.D.U. Rohtak 2004, Ravishanker 2004; Rajasthan 2000, 03, 05, 10] Sol. Here given equation is f (x, y, z, p, q) = 2xz – px – 2qxy + pq = 0. ...(1) Charpit’s auxiliary equations are

or

dy dp dq dz dx / / / = f f f f f f f f ∋p ∋ q ∋ ∋ #p #q p q p q x z y z

dp dq dy dz = 2dx / , by (1) / / 2z ∋ 2qy 0 x ∋ q 2 xy ∋ p px 2 # 2 xyq ∋ 2 pq The second fraction gives dq = 0 so that Putting q = a in (1), we get p = 2x(z – ay)/(x2 – a) Putting values p and q in dz = p dx + q dy, we get

dz =

2 x(z ∋ ay) dx + a dy x2 ∋ a

Integrating,

or

q=a

dz ∋ ady 2 xdx . / 2 z ∋ ay x ∋a

log (z – ay) = log (x2 – a) + log b

Created with Print2PDF. To remove this line, buy a license at: http://www.software602.com/

3.18

Non-Linear Partial Differential Equations of Order One 2

or z – ay = b(x – a) or z = ay + b(x2 – a), ...(2) which is the complete integral, a and b being arbitrary constants. Differentiating (2) partially with respect to a and b, we get 0=y–b and 0 = x2 – a. ...(3) Solving (3) for a and b, a = x2 and b = y. ...(4) Substituting the values of a and b given by (4) in (2), we get z = x2y, which is the required singular integral. Ex. 13. Find a complete integrals of the following partial differential equations: (i) q = px + p2. [Sagar 2003; Meerut 1994] (ii) q = – px + p2. Sol. (i) Here given equation is f(x, y, z, p ,q) ( q – px – p2 = 0. ... (1) dp dq dz dx dy / / / / fx # p fz f y # q f z ∋ pf p ∋ q f q ∋ f p ∋ f q

Charpit’s auxiliary equations are

dp dq dy dx dz / / , by (1) = = ∋p 0 ∋ p(∋ x ∋ 2 p) ∋ q ∋(∋ x ∋ 2 p) ∋1 dq = 0 so that

or The 2nd fraction gives Putting

q = a in (1) gives p2 + px – a = 0

so that

q = a.

1/ 2 p = (1/ 2) 5 9 ( x # 4 a )1/ 2 # 2 a log{x # ( x 2 # 4a )1/ 2 }? + ay + b, 4 2 ( x # 4 a )1/ 2 # 2 a log{x # ( x 2 # 4a )1/ 2 }? + ay + b. 4 2 ( x ∋ y ) # ( x ∋ y ) ∋ a / 2 ? ,, + < =. 4 2 2 2− 2 2 2 Ex. 17. Find a complete integral of p x + q y = z. [Gujarat 2005; K.U. Kurukshetra 2001; Meerut 2008; Agra 2004; I.A.S. 2004, 06 ; Delhi Maths Hons. 1997; Punjab 2001] Sol. Given equation is f (x, y, z, p, q) = p2x + q2y – z = 0. ...(1) z/

dp dq dz dx dy / / / ∋ f x # p fz f y # q f z ∋ p f p ∋ q f q ∋ f p ∋ fq

Charpit’s auxiliary equations are

dp dq dz dx / dy / = = , by (1) ∋ p # p2 ∋q # q2 ∋2 px ∋2qy ∋2( p2 x # q2 y)

or

Now, each fraction in (2) d ( p 2 x)

or

2

∋2 p x

Integrating it, Form (1) and (3),

/

2 px dp # p 2 dx

=

2 px (∋ p # p 2 ) # p 2 (∋2 px )

d (q 2 y ) ∋2qy

=

2qy dq # q 2 dy 2qy (∋q # q 2 ) # q 2 (∋2qy ) d( p 2 x) d (q 2 y) . / 2 p2 x q y

i.e.,

log (p2x)= log (q2y) +log a aq2y + q2y = z

p2x = q2ya. ...(3) q = [z/(1 + a)]1/2. ...(4)

or or

ya R za U p = q F I /S H x K T(1 # a)x VW 12

Form (3) and (4),

...(2)

12

.

Putting the above values of p and q in dz = p dx + q dy, we get

RS za UV T(1 # a)x W

12

dz =

RS z UV T(1 # a)y W

12

dx #

dy

or

(1 + a)1/2z–1/2 dz =

ax ∋1/ 2 dx # y ∋1/ 2 dy .

(1 + a)1/2

z = a x # y # b , a, b being arbitrary constants. Ex. 18. Find a complete integral of 2z + p2 + qy + 2y2 = 0. [I.F.S. 2005; Meerut 2000; Rohilkhand 1993; Bilaspur 2004, M.D.U Rohtak 2005; Rawa 1999; Ranchi 2010] Sol. Given equation is f(x, y, z, p, q) = 2z + p2 + qy2 + 2y2 = 0. ... (1)

Integrating,

Charpit’s auxiliary equations are or

dp dq dz dp dq / / / / fx # p fz f y # q f z ∋ pf p ∋ q f q ∋ f p ∋ f q

dz dp dq dx / dy = = , by (1) / ∋ 2 p ∋y – p 5 (2 p) – qy 0 # 2 p ( q # 4 y ) # 2q Taking the first and fourth fractions, dp = – dx. Integrating, p=a–x or Using (2), (1) becomes 2z + (a – x)2 + qy + 2y2 = 0 q = –[2z + (x – a)2 + 2y2]/y. 6

p = –(x – a). ...(2) ...(3)

Created with Print2PDF. To remove this line, buy a license at: http://www.software602.com/

Non-Linear Partial Differential Equations of Order One

3.21 2

dz = p dx + q dy = – (x – a) dx – [{2z + (x – a) + 2y }/y]dy, by (2) and (3) Multiplying both sides by 2y2 and re–writing, we have 2y2dz = –2 (x – a)y2 dx – 4zydy – 2y(x – a)2dy – 4y3dy 2 2(y dz + 2zy dy) + [2(x – a)2y2dx + 2y(x – a)2dy] + 4y3dy = 0 2d(y2z) + d[y2(x – a)2] + 4y3dy = 0. 2 2 Integrating, 2y z + y (x – a)2 + y4 = b, a, b being arbitrary constants Ex. 19(a). Find a complete integral of 2(z + px + qy) = yp2. [Delhi B.A. (Prog.) II 2007, 10; CDLU 2004; Delhi Maths Hons. 1998, 2008] Sol. Given equation is f(x, y, z, p, q) = 2(z + px + qy) – yp2 = 0 ...(1) 6

or or

2

Charpit’s auxiliary equations are

dp dq dz dp dq / / / / fx # p fz f y # q f z ∋ pf p ∋ q f q ∋ f p ∋ f q

dz dp dq dy dx / / = = , by (1) 2 2 p # 2 p 2q ∋ p # 2q ∋(2 x ∋ 2 yp) ∋2 y ∋ p (2 x ∋ 2 yp) ∋ q 5 2 y

or

Taking the first and the last fractions, Integrating,

log p + 2 log y = log a

Solving (1) and (2) for p and q,

p=

dp dy / 4 p ∋2 y

dp dy #2 = 0. p y py2 = a. ...(2)

or or

a

z ax a2 q=– ∋ 3# 4. y y 2y

and

y2

LM N

OP Q

a z ax a2 dx # ∋ ∋ # dy y y3 2 y 4 y2 Multiplying both sides by y and re–arranging, we get 6

dz = p dx + q dy =

FG IJ H K

x a 2 ∋3 ) y dx ∋ x dy ∗ a 2 ∋ y dy = 0. (ydz + zdy) – a + dy = 0 or d(yz) – ad ∋ , y 2 − y2 . 2 y3 Integrating, yz – a(x/y) + (a2/4y2) = b, a, b being arbitrary constants. ... (3) Ex. 19(b). Find the complete integral, general integral and the singular integral of 2(z + xp + yq) = yp2 [Delhi B.Sc. (H) 1998, 2008] Sol. Proceed as in solved Ex. 19(a) to get the complete integral (3). General integral. Replacing b by !(a) in (3), we get yz – a(x/y) + (a2/4y2) = !(a) ... (4) Differentiating (4) partially w.r.t. ‘a’, –(x/y) + (a/2y2) = !%(a) ... (5) Then the general integral is obtained by eliminating a from (4) and (5). Singular integral. Differentiating (3) partially w.r.t. ‘a’ and ‘b’ by turn, we get –(x/y) + (a/2y2) = 0 ... (6) 0=1 ... (7) Relation (7) is absurd and hence there is no singular solution of the given equation. Ex. 20. Find a complete integral of z2 = pqxy. [Delhi B.A. (Prog) II 2010] [Delhi Maths (H) 2004 Jabalpur 2004; Meerut 2006; Lucknow 2010] Sol. The given equation is f(x, y, z, p, q) = z2 – pqxy = 0. ...(1) Charpits’s auxiliary equations are

dp dq dz dx dy / / / / fx # p fz f y # q f z ∋ p f p ∋ q fq ∋ f p ∋ fq

Created with Print2PDF. To remove this line, buy a license at: http://www.software602.com/

3.22

Non-Linear Partial Differential Equations of Order One

dp dq dy dz / = = dx / , by (1) ∋ pqy # 2 pz ∋ pqx # 2qz ∋ p (∋ qxy) ∋ q(∋ pxy) qxy pxy

or

Each fraction of (2) = or

y dq # q dy x dp # p dx = y (∋ pqx # 2qz ) # pqxy x (∋ pqy # 2 pz ) # pqxy

d(xp) d( yq) xdp # pdx ydq # qdy / / or xp yq . 2 pxz 2qyz Integrating, log (xp) = log (yq) + log a2 or xp = a2yq. ...(3) Solving (1) and (2) for p and q, p = (az)/x and q = z/(ay). or (1/z) dz = (a/x) dx + (1/ay) dy. 6 dz = p dx + q dy = (az/x) dx + (z/ay) dy Integrating, log z = a log x + (1/a) log y + log b or z = xay1/a b. Ex. 21. Using Charpit’s method, find three complete integrals of pq = px + qy. (Kanpur 2004; Meerut 2002; Rajasthan 2001) Sol. Here given equation is f(x, y, z, p, q) = pq – px – qy = 0. ...(1) dp dq dz dx dy / / / / fx # p fz f y # q f z ∋ p f p ∋ q fq ∋ f p ∋ fq

Charpit’s auxiliary equations are or

dp dq dz dx / dy , by (1) / = = ...(2) ∋ p (q ∋ x ) ∋ p( p ∋ y) ∋ p ∋q ∋(q ∋ x ) ∋( p ∋ y) To find first complete integral. Taking the first two fractions of (2), we get (1/p)dp = (1/q)dq so that log p = log q + log a or p = aq. ...(3) 2 Using (3), (1) 2 aq = q(ax + y) q = (ax + y)/a. ...(4) 2 Hence, from (3), we have p = ax + y. ...(5) 6 dz = p dx + q dy = (ax + y)dx + [(ax + y)/a]dy = (1/a)(ax + y)(a dx + y). Putting ax + y = t so that adx + dy = dt, we get dz = (1/a)× t dt so that z = (1/2a) × t2 + b or z = (1/2a) × (ax + y)2 + b, as t = ax + y. To find second complete integral. Taking the second and the fourth ratios in (2), we get dx/(q – x) = dq/q or q dx + x dq = q dq. 2 Integrating, qx = q /2 + a/2 or q2 – 2xq + a = 0.

6 q = [2x ± 2( x2 ∋ a)1/ 2 ]/2 Using (6),

(1) 2

6

q = x + ( x2 ∋ a)1/ 2 .

so that

...(6)

p[x + ( x 2 ∋ a )1/ 2 ] – px – y[x + ( x2 ∋ a)1/ 2 ] = 0

3

4

2 1/ 2 y. p = 1# x ( x ∋ a)

so that

or

...(2)

3

...(7)

4

2 1/ 2 dz = p dx + q dy = 1 # x ( x ∋ a ) ydx + [x + ( x 2 ∋ a )1/ 2 ]dy

9 xy dy : # ( x 2 ∋ a)1/ 2 dy ? or dz = d(xy) + d[y ( x 2 ∋ a )1/ 2 ]. dz = (y dx + x dy) + > 2 1/ 2 < ( x ∋ a) = 2 1/ 2 Integraing, z = xy + y ( x ∋ a) + b, a, b being arbitrary constants.

To find third complete integral. Taking the first and the fifth ratios of (2) and proceeding as above third complete integral is

z = xy + x ( y 2 ∋ a)1/ 2 + b.

Ex. 22. Find complete integral of xp + 3yq = 2(z – x2q2). Sol. Given equation is

[Delhi B.Sc. (Prog) II 2009; Delhi B.Sc. (Hons) II 2010; ] f(x, y, z, p, q) = xp + 3yq – 2z + 2x2q2 = 0. ...(1)

Created with Print2PDF. To remove this line, buy a license at: http://www.software602.com/

Non-Linear Partial Differential Equations of Order One

3.23

dp dq dz dx dy / / / / fx # p fz f y # q f z ∋ p f p ∋ q fq ∋ f p ∋ fq

Charpit’s auxiliary equations are

dz dp dq dy = = dx / , by (1) 2 / 2 q ∋ x ∋ p # 4 xq ∋3y ∋ 4 x 2 q ∋ p x ∋ q (3 y # 4 x q )

or (2) 2

dq dx / q ∋x

2

log q = log a – log x

2

Using (3), (1) 2

xp + 3y(a/x) – 2z + 2x2(a2/x2) = 0

6

dz = p dx + q dy =

2

qx = a

2

p=

RS2(z ∋ a ) ∋ 3ay UV dx + a dy T x x W x

q=

...(2) a . x

...(3)

2(z ∋ a2 ) 3ay ∋ 2 . ...(4) x x

2

or or

x2dz = 2x(z – a2)dx – 3ay dx + ax dy

or

x2dz – 2x(z – a2) dx = –3ay dx + ax dy

or

F I H K

) z ∋ a2 ∗ ay d + 2 , =d 3 + x , x x x x − . Integrating, (z – a2)/x2 = (ay)/x3 + b or z = a(a + y/x) + bx2. Ex. 23. Find complete integrals of the following equations : (i) (p2 + q2)n (qx – py) = 1. (ii) qx + py = (p2 – q2)n. Sol. (i) Given equation is f(x, y, z, p, q) = (p2 + q2)n (qx – py) – 1 = 0. ...(1) x 2 dz ∋ 2 x ( z ∋ a 2 )dx 4

= ∋

3ay dx 4

#

a dy

or

3

dp dq dz dx dy / / / / fx # p fz f y # q f z ∋ p f p ∋ q fq ∋ f p ∋ fq

Charpit’s auxiliary equations are or

2

dp dq dp dq / = ... or or pdp + qdq = 0. / q ∋p q ( p2 # q2 )n ∋ p( p2 # q2 )n Integrating, p2 + q2 = constant = (1/a2), say ...(2) 2n 2n Using (2), (1) 2 qx – py = a or qx = py + a . ...(3) Using (3), (2) 2 p2 + (p2y2 + a4n + 2a2nyp)/x2 = 1/a2 p2(x2 + y2) + 2a2nyp + {a4n – (x2/a2)} = 0 so that

p=

∋ ya 2n #

6

{a

d

4n 2

y ∋ ( x 2 # y 2 ) a 4n ∋ x 2 / a 2

i}

∋ ya 2 n # x

=

x 2 # y2

(3) 2

xa 2 n # y

q=

o( x

2

o( x

2

x2 # y2

t

t

# y 2 ) a 2 ∋ a 4n

...(4)

# y 2 ) a 2 ∋ a 4n

x2 # y2

.

...(5)

Substituting these values in dz = p dx + q dy, we have ) x dy ∋ y dx ∗ x dx # y dy dz = a2n + 2 ,# 2 x # y2 − x # y2 .

Integrating,

z + b = a2n tan–1

FG y IJ # 1 H xK 2

z

≅Β) x 2 # y 2 ∗ 4 n ΑΒ Χ+ , ∋a ∆ . 2 . ΒΦ ΕΒ− a

1 (ua ∋2 ∋ a 4 n )1/ 2 du , where u = x2 + y2. u

Created with Print2PDF. To remove this line, buy a license at: http://www.software602.com/

3.24

Non-Linear Partial Differential Equations of Order One

Part (ii). Proceed as in part (i). If u = x2 + y2, then complete integral is

z

x∋y 1 1 z + b = – 1 a2n log ∋ (a4n # a2 u) du . 2 x#y 2 u Ex. 24. Find complete integral of p2 + q2 – 2pq tanh 2y = sech2 2y. Sol. Given f(x, y, z, p, q) = p2 + q2 – 2pq tanh 2y – sech2 2y = 0.

...(1)

dp dq dz dx dy / / / / fx # p fz f y # q f z ∋ p f p ∋ q fq ∋ f p ∋ fq

Charpit’s auxiliary equations are

dp dq / = ····, by (1) 0 ∋ 4 pq sech 2 2 y # 4 sech 2 2 y tanh 2 y

or Then, Using (2),

first fraction 2 dp = 0 p = constant = a, say. 2 (1) 2 q2 – (2a tanh 2y)q + a2 – sech2 2y = 0

2

q = [2a tanh 2y ± 2 ( a 2 tanh 2 2 y ∋ a 2 # sec h 2 2 y) ]/2

2

q = a tanh 2y + (1 ∋ a 2 ) . sech 2y..

...(2)

...(3) 2

2

[Note that sech 2y = 1 – tanh 2y] Using (2) and (3), dz = p dx + q dy reduces to dz = a dx + {a tanh 2y + (1 ∋ a 2 ) sech 2y}dy Integrating,

z + b = ax + a log cosh 2y + 2

2

z z

(1 ∋ a )

2dy ∋2 y e #e 2y

2e2 y dy 1 # (e2 y )2

or

z + b = ax +

a log cosh 2y + 2

(1 ∋ a2 )

or

z + b = ax +

a log cosh 2y + 2

(1 ∋ a ) tan (e ) ,

∋1

2

2y

: / tan ∋1 t / tan ∋1 e 2 y ? ) = 2 2 2 Ex. 25. Find complete integral of the equation q = {(1 + p )/(1 + y )}x + yp(z – px) . Sol. Let f(x, y, z, p, q) = {(1 + p2)/(1 + y2)}x + yp(z – px2) – q = 0. ...(1)

! " #

on putting e2y = t

2e2y dy = dt,

and

or

2y 2

dp 2

/

dt

0 1# t

2

dp dq dz dx dy / / / / fx # p fz f y # q f z ∋ pf p ∋ q f q ∋ f p ∋ f q

Charpit’s auxiliary equations are or

2e 2 y dy

0 1 # (e

2

2

2

/

dy = ........., by (1) 1

{ (1 # p ) (1 # y )} ∋ 2 yp ( z ∋ px ) # 2 yp ( z ∋ px ) dp dy / so that tan–1 p – tan–1 y = constant = tan–1 a 2 1# p 1# y 2 (p – y)/(1 + py) = a p = (y + a)/(1 – ay). ...(2) 2 2

1 # a2 y( y # a) 2 2 x# 3 {z (1– ay) – x(y + a)} . (1 ∋ ay) (1 ∋ ay) Using (2) and (3), dz = p dx + q dy reduces to Using (2),

(1) 2

q=

LM N

...(3)

OP Q

2 dz = y # a dx # 1 # a 2 x # y( y # a3) {z(1 ∋ ay) ∋ x( y #a)}2 dy 1 ∋ ay (1 ∋ ay) (1 ∋ ay)

Created with Print2PDF. To remove this line, buy a license at: http://www.software602.com/

Non-Linear Partial Differential Equations of Order One

or dz = d

FG y # a xIJ # y(y # a) {z (1 ∋ ay) ∋ x(y #a)} dy H 1 ∋ ay K (1 ∋ ay) 2

3

or

u = x(y + a)/(1 – ay).

where (4) 2

3.25

dz ∋ du y( y # a) = dy. 2 1 ∋ ay (z ∋ u)

Integrating, b –

dz = du +

R|S |T

y( y # a) (z – u)2dy, ...(4) 1 ∋ ay ...(5)

U|V |W

2 d(z ∋ u) ∋1 ∋ 1 (1 # ay ) # a # 1 1 = dy. 2 a2 a 2 1 ∋ ay ( z ∋ u)

or

F GH

1 = – y – 1 y # ay 2 2 z ∋u a

2

I ∋ a #1 log (1 – ay), where u is given by (5). JK a 2

3

Ex. 26. Find complete integral of xp – yq = xq f (z – px – qy). Sol. Let F (x, y, z, p, q) = xp – yq – xq f (z – px – qy) = 0. Charpit’s auxiliary equations are

...(2)

dp dq dz dx dy / / / / F / x # p( F / z ) F / y # q( F / z ) ∋ p( F / p) ∋ q( F / q) ∋( F / p) ∋( F / q) dp dq = ·····, by (2) / p ∋ qf #xqpf % ∋ pqxf % ∋ q # xq 2 f % ∋ xq 2 f %

or

...(3)

x dp # y dq x dp # y dq / , by (2) xp ∋ yq ∋ qxf 0 x dp + y dq = 0 x dp + y dq + p dx + q dy = p dx + q dy 2 2 dz – d(xp) – d(yq) = 0, as dz = pdx + qdy 2 Integrating, z – xp – yq = constant = a, say ...(4) xp + yq = z – a. ...(5) 6 Using (4), (1) becomes x p – y q = x q f(a). ...(6) Subtracting (6) from (5), 2yq = z – a – xqf(a) q = (z – a)/{2y + xf(a)} ...(7) 2

Each ratio of (3) =

Using (7),

(5) 2

Using (7) and (8),

dz = p dx + q dy reduces to

p=

dz = (z – a) or

(z ∋ a){y # xf (a)} . x {2 y # xf (a)}

...(8)

LM {y # xf (a)} dx # dy OP N x {2y # xf (a)} 2y # xf (a) Q

2dz = 2 y dx # 2 xf (a )dx # 2 x dy / 2 d ( xy ) # 2 xf (a ) dx . x {2 y # xf ( a )} z∋a 2 xy # x 2 f (a ) Integrating, 2 log (z – a) = log {2xy + x2f (a)} + log b or (z – a)2 = b {2xy + x2f (a)}. 1/2 Ex. 27. Find a complete integral of px + qy = z(1 + pq) [Meerut 2001, 02; Kanpur 1995, I.A.S. 1992] Sol. Given f(x, y, z, p, q) = px + qy – z(1 + pq)1/2 = 0. ...(1) Charpit’s auxiliary equation are

or

dp dq dz dx dy / / / / fx # p fz f y # q f z ∋ pf p ∋ q f q ∋ f p ∋ f q

dp dq 1/ 2 / 1/ 2 = ......... so that p ∋ p (1 # pq) q ∋ q(1 # pq) log p = log a + log q 2 2 Using (2), (1) 2 q(ax + y) = z (1 + aq2)1/2 or

dp dq / , by (1) p q

p = aq. ...(2) q2 [(ax + y)2 – az2] = z2.

Created with Print2PDF. To remove this line, buy a license at: http://www.software602.com/

3.26

Non-Linear Partial Differential Equations of Order One

az z and p = aq = 2 2 1/ 2 . 2 2 1/ 2 [( ax # y) ∋ az ] [( ax # y) ∋ az ] Substituting these values in dz = p dx + q dy, we have

6 q=

dz =

z (a dx # dy ) {(ax # y )2 ∋ az 2 }

ax + y =

Let 6

dz = z

(3) 2

dz = z

or

au 2

(au ∋ az )

v+z

dv = (v 2 ∋1)1/ 2 . dz

.

... (3)

a dx + dy =

(u 2 ∋ z 2 ) du / / dz z

or

which is linear homogeneus equation. To solve it, we put u =v or u = vz z 6 (4) yields

{(ax # y )2 ∋ az 2 }

so that

adu 2

a dx # dy

RSF uI ∋1UV , TH z K W 2

a du. ... (4)

du = v + z dv . dz dz dv dz = 2 z (v ∋ 1)1/ 2 ∋ v

so that

or

(1/z)dz = – 9(v 2 ∋1)1/ 2 # v : d v , on rationalization. < =

or

2 9v 2 u ax # y 1/ 2 1 2 1/ 2 : v Integrating, log z = – > (v ∋1) ∋ log{ v # (v ∋1) ? ∋ # b , where, v / / 2 z 0; S.S. Does not exist; G.S. It is given by z ∋ ax ∋ y log a ∋ ∃(a ) / 0, – x – (y/a) – ∃%(a) / 0, 2 3

where ∃ is an arbitrary function. y + c, where a and c are arbitrary constants and a > 0;

–2/3

8. p q = 1 Ans. C.I. z = ax + a

S.S. Does not exist; G.S. It is given by z ∋ ax ∋ a ∋2 / 3 y ∋ ∃(a) / 0, ∋ x # (2 / 3) 5 a ∋5 / 3 y ∋ ∃(a) / 0, 2

2

9. p + p = q .

where ∃ is an arbitrary function. Ans. C.I. z = ax + (a + a) y + c, where a and c are arbitrary constants 2

1/2

and a Ν R ∋ (∋1, 0); S.S. Does not exist; G.S. It is given by z – ax – (a2 + a)1/2 y – ∃(a ) / 0,

∋ x ∋ {(2a # 1) / 2(a 2 # a)1/ 2 } y ∋ ∃%(a) / 0, where ∃ is an arbitrary function. 10. p2 + 6p + 2q + 4 = 0.

C.I. z = ax – (2 + 3a + a2/2)y + c, where a and c are

arbitrary constants; S.S. Does not exist; G.S. It is given by z ∋ ax # (2 # 3a # a 2 / 2) y ∋ ∃(a) / 0, ∋ x # (a # 3) y ∋ ∃%(a) / 0, where ∃ is an arbitrary function. Find the complete integral (solution) of the following equations (Ex. 11—18). 11. zy2p = x(y2 + z2q2). Ans. z2 = ax2 ; y2(a – 1)1/2+ c, where a Κ 1 12. z2(p2/x2 + q2/y2) = 1. Ans. z2 = ax2 ; y2(1 – a2)1/2+ c, where ∋1 Μ a Μ 1 2 2 2 13. yp + x q = 2x y. Ans. (3z – ax3 – b)2 = 4(2 – a)y2 14. (1 – y2) xq2 – y2p = 0. Ans. (2z – ax2 – b)2 = a(1 – y2) 2 2 2 15. p y(1 + x ) = qx . Ans. z = a(1 + x2)1/2+ (1/2) × a2y2 + c 4 2 2 2 16. x p + y zq – z = 0. Ans. xy log z = ay + (a2 – 1)x + bxy

Created with Print2PDF. To remove this line, buy a license at: http://www.software602.com/

3.44

Non-Linear Partial Differential Equations of Order One

17. p2 + q2 = z.

[Bangalore 1995] Ans. 2z1/2 = ax ; (1 – a2)1/2y + c, where ∋1 Μ a Μ 1 18. x2p2 + y2q2 = 4z2. Ans. log z = a log x + (4 – a2)1/2 + c, ∋2 Μ a Μ 2 3.12. Standard form II. Clairaut equation. [Meerut 2009; Nagpur 2002] A first order partial differential equation is said to be of Clariaut form if it can be written in the form z = px + qy + f(p, q). ...(1) Let F(x, y, z, p, q) ( px + qy + f(p, q) – z. ...(2) Charpit’s auxiliary equations are

dp dq dz dx dy / / / = F F F F F F F F #p #q ∋p ∋q ∋ ∋ x z y z p q p q

dy dp dq dz dx / = = = , by (1) ∋ px ∋ qy ∋ p ( f p) ∋ q( f q) ∋ x ∋ ( f p) ∋ y ∋ ( f q) 0 0 Then, first and second fractions 2 dp = 0 and dq = 0 2 p = a and q = b. Substituting these values in (1), the complete integral is z = ax + by + f(a, b) Remark 1. Observe that the complete integral of (1) is obtained by merely replacing p and q by a and b respectively. Singular and general integrals can be obtained by usual methods. Remark 2. Sometimes change of variables can be employed to transform a given equation to standard form II.

or

3.13. SOLVED EXAMPLES BASED ON ART. 3.12 Ex. 1. Solve z = px + qy + pq. [Ravishanker 1997; Bangalore 2005; Sagar 1995, 96] Sol. The complete integral is z = ax + by + ab, a, b being arbitrary constants ...(1) Singular integral. Differentiating (1) partially w.r.t. a and b, we have a=x+b and 0 = y + a. ...(2) Eliminating a and b between (1) and (2), we get z = – xy – xy + xy i.e., z = –xy, which is the required singular solution, for it satisfies the given equation. General Integral. Take b = !(a), where ! denotes an arbitrary function. Then (1) becomes z = a x + !(a) y + a ! (a). ...(3) Differentiating (3) partially w.r.t. a, 0 = x + !%(a)y + !(a) – a !%(a). ...(4) The general integral is obtained by eliminating a between (3) and (4). Ex. 2. Prove that complete integral of the equations (px + qy – z)2 = 1 + p2 + q2 2 2 2 1/2 is ax + by + cz = (a + b + c ) . [I.A.S. 1989] Sol. Re–writting the given equation, we have px + qy – z = ± (1 # p 2 # q 2 )

or

z = px + qy ± (1 # p 2 # q 2 )

which is of standard form II and so its complete integral is z = Ax + By ± (1 + A2 + B2)1/2. ...(1) To get the desired form of solution we take +ve sign in (1) and set A = –a/c and B = –b/c. Then (1) becomes z = – (ax + by)/c + (c2 + a2 + b2)1/2/c or ax + by + cz = (a2 + b2 + c2)1/2. Ex. 3. Solve z = px + qy + c (1 # p 2 # q 2 ) .

[I.A.S. 1989; Meerut 1998]

Sol. The complete integral of the given equation is z = ax + by + c (1 # a 2 # b 2 ) , a, b being arbitrary constants.

...(1)

Singular Integral. Differentiating (1) partially w.r.t. a and b, we get

Created with Print2PDF. To remove this line, buy a license at: http://www.software602.com/

Non-Linear Partial Differential Equations of Order One

0 = x + ac/ (1 # a 2 # b 2 ) 6 From (2) and (3),

3.45

0 = y + bc/ (1 # a 2 # b 2 ) . x2 + y2 = (a2c2 + b2c2)/(1 + a2 + b2).

...(2)

...(3)

a2 c2 # b 2c2 c2 / 1 # a 2 # b 2 1 # a2 # b2 2 2 1 + a + b = c2/(c2 – x2 – y2).

c2 – x2 – y2 = c2 –

6 so that

2

a= –

From (2),

...(4)

2

x (1 # a # b ) x , by (4) /∋ 2 2 2 c (c ∋ x ∋ y )

b = – y/ c 2 ∋ x 2 ∋ y 2 .

Similarly from (3) and (4), we obtain

Putting these values of a and b in (1), the singular solution is z= ∋ or

x

2

2

2

2

(c ∋ x ∋ y )



y 2

2 2

2

(c ∋ x ∋ y )

#

c 2

2 2

2

(c ∋ x ∋ y )

2

2

2

2 2

(c ∋ x ∋ y )

z = (c2 – x2 – y2)1/2 or z2 = c2 – x2 – y2 or x2 + y2 + z2 = c2. ...(5) We can easily verify that (1) is satisfied by (5). General Integral. Take b = !(a), where ! is an arbitrary function. Then, (1) yeilds z = ax + y!(a) + c[1 + a2 + {!(a)}2]1/2. ...(6) Differentiating both sides of (6) partially w.r.t. ‘a’, we get 0 = x + y!%(a) + (c/2) × [1 + a2 + {!(a)}2]–1/2 × [2a + 2!(a) !%(a)]. ...(7) Eliminating a from (6) and (7), we get the general integral. Ex. 4. Find the complete and singular integrals of the following equations: (i) z = px + qy + log (pq) [Indore 2004; K.U. Kurukshetra 2006] (ii) z = px + qy – 2 pq .

or

2

c ∋x ∋y

=

[Bangalore 1993; Lucknow 2010]

Sol. (i) The complete integral is z = ax + by + log (ab) z = ax + by + log a + log b, a, b being arbitrary constants Differentiating (1) partially with respect to a and b, we get 0 = x + (1/a) and 0 = y + (1/b) so that a = –1/x and b = –1/y. Eliminating a and b from (1) and (2), the required singular integral is z = – 1 – 1 + log (1/xy) or z = – 2 – log (xy). (ii) The complete integral is z = ax + by – 2 ab . Differentiating (1) partially with respect to a and b, we get 0=x–

2b 2 ab

Now, using (1) 6

and

0=y–

2a 2 ab

so that

x – z = x – (ax + by – 2 ab ) = x–z=

x=

...(2)

...(1)

b and y = a

a . ...(2) b

b b a –a –b + 2 ab , using (2) a a b

(b / a) .

Similarly, using (1) y – z = y – (ax + by – 2 ab ), =

...(1)

...(3) a –a b

y – z = (a / b) . 6 From (3) and (4), (x – z) (y – z) = 1, which is singular integral as it satisfies the given equation.

b –b a

a + 2 ab b

...(4)

Created with Print2PDF. To remove this line, buy a license at: http://www.software602.com/

3.46

Non-Linear Partial Differential Equations of Order One

Ex. 5. Prove that the complete integral of z = px + qy – 2p – 3q represents all possible planes through the point (2, 3, 0). Also find the envelope of all planes represented by the complete integral (i.e., find the singular integral). (M.D.U. Rohtak 2006) Sol. Given that z = px + qy – 2p – 3q, ...(1) which is of the form z = px + qy + f(p, q) and so its complete integral is z = ax + by – 2a – 3b, a, b being arbitrary constants ...(2) Since (2) is a linear equation in x, y, z, it follows that (2) represents planes for various values of a and b. Again putting x = 2, y = 3, z = 0 in (2), we have 0 = 2a + 3b – 2a – 3b i.e., 0 = 0, showing that coordinates of the point (2, 3, 0) satisfy (2). Hence the complete integral (2) of (1) represents all possible planes passing through the point (2, 3,0). Differentiating (2) partially with respect to a and b, we get 0=x–2 and 0=y–3 so that x = 2 and y = 3. Substituting these values in (2), we get z = 0 as the required envelope (i.e., singular integral). Ex. 6. Prove that the complete integral of z = px + qy + [pq/(pq – p – q)] represents all planes such that the algebraic sum of the intercepts on three coordinate axes is unity. Sol. Since the given equation is of the form z = px + qy + f(p, q), so its complete integral is z = ax + by + [ab/(ab – a – b)], a and b being arbitrary constants. ...(1) Since (2) is a linear equation in x, y, z, it follows that (1) represents planes for various values of a and b. We now rewrite (1) in the intercept form of a plane as follows : ax + by – z = ab/(a + b – ab) y x z or = 1. # # [b / (a # b ∋ ab)] [a / (a # b ∋ ab)] [∋ ab / ( a # b ∋ ab)] The algebric sum of the intercepts on three coordinate axes 6 b # a ∋ ab ( ∋ ab ) b a = # # / = 1, as required. a # b ∋ ab a # b ∋ ab a # b ∋ ab a # b ∋ ab Ex. 7. Show that the complete integral of the equation z = px + qy + (p2 + q2 + 1)1/2 represents all planes at unit distance from the origin. Sol. Given equation is of the form z = px + qy + f(p, q), so its complete integral is z = ax + by + (a2 + b2 + 1)1/2, a, b being an arbitrary constants. or ax + by – z + (a2 + b2 + 1)1/2 = 0. ...(1) Since (2) is a linear equation in x, y, z, it follows that (1) represents planes for various values of a and b. The perpendicular distance of (1) from origin (0, 0, 0) 2

=

2

a . 0 # b . 0 ∋ 0 # a # b #1 2

2

2

{a # b # ( ∋1) }

/

2

2

2

2

a # b #1 a # b #1

= 1, as required

Ex. 8. Find the complete integral of the following equations: (i) ( p # q) ( z ∋ px ∋ qy) / 1

[Pune 2010]

(ii) pqz / p2 ( xq # p2 ) # q2 ( yp # q2 )

[Delhi B.A. (Prog) II 2008, 10]

Sol. (i) Re–writing the given equation in the standard form z / px # qy # f ( p, q) , we get

z ∋ px ∋ qy / 1/( p # q)

or

z / px # qy # 1/( p # q)

Created with Print2PDF. To remove this line, buy a license at: http://www.software602.com/

Non-Linear Partial Differential Equations of Order One

3.47

6 Its complete integral is z / ax # by # 1/(a # b) , where a and b are arbitrary constants.

z / px # qy # ( p4 # q4 ) / pq ,

(ii) Dividing both sides of the given equation by pq,

z / ax # by # (a4 # b4 ) / ab , a, b being arbitrary constants.

Its complete integral is

2( y # zq) / q( xp # yq) .

Ex. 9. (a) Find the complete integral the equation

[Delhi Maths (H) 1999] Sol. Re–writing the given equation, we have

2zq / xpq # yq2 ∋ 2 y

z / (1/ 2) px # (1/ 2)qy ∋ ( y / q)

or

) 1 z∗ 2 ) 1 z∗ 1 ) 1 z∗ z / x2 + , # y + 2y y , ∋ 2 + 2 y y , − 2x x . − . − .

or Putting

2x dx / dX and

so that

2y dy / dY

z / X ( z / X ) # Y ( z / Y ) ∋ 1/{2 ( z / Y )}

∋1

... (1)

x 2 / X and y2 / Y ,

(1) gives

z / PX # QY ∋ (1/ 2Q ) ,

or

where P / z / X and Q / z / Y . The above equation is of the form z / PX # Qy # f (P, Q) and hence its complete integral is or z / ax 2 # by2 ∋ (1/ 2b) , a and b being arbitrary constants.

z / aX # bY ∋ (1/ 2b)

Ex. 9. (b) Find the complete integral of 2q(z ∋ px ∋ qy) / 1 # q2 . Sol. Re–writing the given equation in the form z / px # qy # f ( p, q) , we have

z ∋ px ∋ qy / (1 # q2 ) / 2q

z / px # qy # (1 # q2 ) / 2q ,

or

Its complete integral is z / ax # by # (1 # b2 ) / 2b , a and b being arbitrary constants. Ex. 10. Find the compelte integral of p2 x # q2 y / ( z ∋ 2 px ∋ 2qy)2 . Sol. Taking positive root, the given equation reduces to

z ∋ 2 px ∋ 2qy / ( p2 x # q2 y)1/ 2

or

z / 2 px # 2qy # ( p2 x # q2 y)1/ 2 1/ 2

z

1 z/ x # y # (1/ 2 x ) x (1/ 2 y ) y 2

or

Put

2 2 9 ∗ : ∗ ) z z >)+ #+ , ? >+− (1/ 2 x ) x ,,. + (1/ 2 y ) y , ? − . =?

(1/ 2 x ) dx / dX and (1/ 2 y )dy / dY

Using (2), (1) gives or

z

so that

x/X

... (1)

and

y / Y ... (2)

z / ( z / X ) X # ( z / Y )Y # (1/ 2) 5 {( z / X )2 # ( z / Y )2}1/ 2

z / PX # QY # (1/ 2) 5 ( P2 # Q2 )1/ 2 ,

where

P / z/ X

and

Q / z/ Y .

It is of the Clairaut’s form z / Px # Qy # f ( P, Q) and so its complete integral is given by

z / aX # bY # (1/ 2) 5 (a2 # b2 )1/ 2

or

z / a x # b y # (1/ 2) 5 (a2 # b2 )1/ 2

Created with Print2PDF. To remove this line, buy a license at: http://www.software602.com/

3.48

Non-Linear Partial Differential Equations of Order One

Ex. 11. Find a complete and the singular integral of 4xyz = pq + 2px2y + 2qxy2 Sol. The given equation can be rewritten as

Put so that

z∗ ) 1 z ∗) 1 z ∗ 2 ) 1 z ∗ 2) 1 z/+ ,# x + ,. ,+ ,# y + − 2x x . − 2 y y . − 2x x . − 2y y . 2x dx = dX and 2y dy = dY x2 = X and y2 = Y.

... (1) ... (2) ... (3)

z / ( z / X )( z / Y ) # X ( z / X ) # Y ( z / Y )

Using (2), (1) becomes

z = XP + YQ + PQ,

or

... (4)

where P / z / X and Q / z / Y . (4) is of the form z = XP + YQ + f(P, Q). z = aX + bY + ab, a, b being arbitrary constants. 6 Solution of (4) is or z = ax2 + by2 + ab, which is complete integral. ... (5) Differentiating (5) partially w.r.t a and b, we have 0 = x2 + b and 0 = y2 + b so that b = – x2 and a = – y2 .... (6) Eliminating a and b between (5) and (6), the required singular integral is z = – x2y2 – x2y2 + x2y2 or z = – x2y2. 2 2 Ex. 12. Find the complete and singular solutions of z = px + qy + p q . [Jabalpur 2000; Sagar 1995; Rewa 2003; Ravishankar 2004] Sol. Given z = px + qy + p2q2 ... (1) Since (1) is in Clairaut’s form, its complete solution is z = ax + by + a2b2, a, b being arbitrary constants ... (2) To find singular solution of (1). Differentiating (2) partially w.r.t. ‘a’ and ‘b’ successively, 0 = x + 2ab2 and 0 = y + 2a2b ... (3) 2 1/3 2 1/3 From (3), a = – (y /2x) and b = – (x /2y) ... (4) Substituting the values of a and b given by (4) in (2), we get z = – x(y2/2x)1/3 – y(x2/2y)1/3 + (x2y2/16)1/3 or z = – (3/4) × 41/3 x2/3 y2/3, which is the required singular solution of (1)

EXERCISE 3 (D) Solve the following partial differential equations : (1 – 9) 1. z = px + qy – 2p – 3q.

[M.D.U. Rohtak 2006]

Ans. C.I. z = ax + by – 2a – 3b; S.S. z = 0; G.S. It is given by z ∋ ax ∋ ∃(a) y # 2a # 3∃(a) / 0, x # ( y ∋ 3)∃%(a) ∋ 2 / 0 Ans. S.I. z = ax + by + 5ab; S.S. 5z + xy = 0

2. z = px + qy + 5pq.

G.S. z ∋ ax ∋ ∃(a) y ∋ 5a ∃(a) / 0, x # 5∃(a) # ( y # 5a) ∃ %(a) / 0 3. z = px + qy + p – q [Purvanchal 2007] Ans. S.I. z = ax + by + a2 – b2; 2

2

S.S. x2 – y2 + 4z = 0; G.S. z ∋ ax ∋ ∃(a) y ∋ a 2 # {∃(a)}2 / 0; x # 2a # { y ∋ 2∃(a)}∃%(a) / 0; 4. z = px + qy + (q/p) – p. [Madras 2005] Ans. C.I. z = ax + by + (b/a) – a; S.S. yz = 1 – x; G.S. It is given by z ∋ ax # ∃(a) y # (1/ a) 5 ∃(a) ∋ a, ∋ x # ∃%(a) y ∋ (1/ a 2 ) ∃ (a) # (1/ a ) 5 ∃%(a) / 0 5. z = px + qy + p/q Ans. C.I. = ax + by + a/b; S.S. xz + 4 = 0; G.S. z ∋ ax ∋ ∃(a) y ∋ a / ∃(a) / 0; x # ∃%(a) y # 1/ ∃(a) ∋ {a∃%(a)}/{∃(a )}2 / 0

Created with Print2PDF. To remove this line, buy a license at: http://www.software602.com/

Non-Linear Partial Differential Equations of Order One

3.49

6. z / px # qy # 2 pq

[Bangalore 1994]

Ans. C.I. z / ax # by # 2 ab ; S.S. (x – z) (y – z) = 1; G.S. z ∋ ax ∋ ∃(a) y ∋ 2 a∃(a ) / 0,

x # ∃%(a) # {∃(a) # a∃%(a)}/ 2 a∃(a) / 0 7. z / px # qy ∋ 2 pq .

Ans. C.I. z / ax # by ∋ 2 ab ; S.S. (x – z) (y – z) = 1;

G.S. z ∋ ax ∋ ∃(a) y # 2 a∃(a) / 0, x # ∃%(a) y ∋ {∃(a) # a∃%(a)}/ a∃(a ) / 0 8. z = px + qy + p2 + pq + q2. [Ranchi 2010] Ans. C.I. z = ax + by + a 2 + ab + b 2 ; S.S. x 2 + y 2 – xy + 3z = 0, G.S. z ∋ ax ∋ ∃(a) y ∋ a 2 ∋ a∃(a) ∋ {∃(a)}2 / 0, x # { y # a # 2 ∃ (a)}∃%(a) # 2a # ∃(a) / 0 9. z / px # qy # (Γp 2 # Οq 2 # 1)1/ 2 .

Ans. C.I. z / ax # by # (Γa 2 # Οb2 # 1)1/ 2 ;

S.S. x2 / Γ # y 2 / Ο # z 2 / 1; G.S. z ∋ ax ∋ ∃(a) y ∋ [aΓ 2 # Ο{∃(a)}2 # 1]1/ 2 / 0; x # ∃%(a) y

#{aΓ # Ο∃(a)∃ %(a )}/[aΓ2 # Ο(∃(a)}2 # 1]1/ 2 / 0 10. Find the complete integral of z = px + qy – sin (pq) [GATE 2003] Ans. z = ax + by – sin (ab) a, b being arbitrary constants. 11. Find the complete integral and singular integral of the differential equation z = px + qy + p2 – q2. Find also a developable surface belonging to the general integral of this differential equation. [I.A.S 1983] 2 2 Ans. Complete integral is z = ax + by + a – b ; singular integral is 4z = 3(x2 – y2) 3.14. Standard form III. Only p, q and z present. [Nagpur 2003; Delhi Maths (H) 2006] Under this standard form we consider differential equation of the form f(p, q, z) = 0. ...(1) dp dq dy dz dx / / / Charpit’s auxiliary equations are f f f f = f f f f #p #q ∋p ∋ q ∋ ∋ x z y z p q p q dp dq dz dx / dy , using (1) / = = ∋ p ( f p) ∋ q( f q) ∋ f p ∋ f q p ( f z) q ( f z) Taking the first two ratios, (1/p)dp = (1/q)dq Integrating, q = ap, a being an arbitrary constant. Now, dz = p dx + q dy = p dx + ap dy, using (2) or dz = p(dx + ady) = pd(x + ay) = p du, where u = x + ay. Now, (3) 2 p = dz/du and so by (2) q = ap = a(dz/du). or

Substituting these values of p and q in (1), we get

f

FH dz , a dz , zIK = 0, du du

...(2) ...(3) ...(4)

...(5)

which is an ordinary differential equation of first order. Solving (5), we get z as a function of u. Complete integral is then obtained by replacing u by (x + ay). 3.15. Working rule for solving equations of the form f(p,q , z) = 0. ...(1) Step I. Let u = x + ay, where a is an arbitrary constant. ...(2) Step II. Replace p and q by dz/du and a(dz/du) respectively in (1) and solve the resulting ordinary differential equation of first order by usual methods. Step III. Replace u by x + ay in the solution obtained in step II.

Created with Print2PDF. To remove this line, buy a license at: http://www.software602.com/

3.50

Non-Linear Partial Differential Equations of Order One

Remark 1. Sometimes change of variables can be employed to reduce a given equation in the standard form III. Remark 2. Singular and general integrals are obtined by well known methods. 3.16.SOLVED EXAMPLES BASED ON ART 3.15. Ex. 1. Find a complete integral of 9(p2z + q2) = 4. [Delhi Maths (H) 2006; Banglore 1995; I.A.S. 1988; Meerut 1996; Rohilkhand 1995] Sol. Given equation is 9(p2z + q2) = 4, ...(1) which is of the form f(p, q, z) = 0. Let u = x + ay, where a is an arbitrary constant. Now, replacing p and q by dz/du and a(dz/du) respectively in (1), we get

LM F I NH K

9 z dz du

2

FH IK OP = 4 Q

# a 2 dz du

FH dz IK du

2

or

2

=

4 . 9 ( z #a 2 )

du = ± (3/2) × (z + a2)1/2dz, separating variables u and z. Integrating, u + b = ± (3/2) × [(z + a2)3/2/(3/2)] or u + b = ± (z + a2)3/2 2 2 3 2 2 3 or (u + b) = (z + a ) or (x + ay + b) = (z + a ) , as u = x + ay which is a complete integral contaning two arbitrary constants a and b. Ex. 2. Find a complete integral of p2 = qz. [Bilaspur 1996; Sagar 2004] Sol. Given equation is p2 = qz, ...(1) which is of the form f(p, q, z) = 0. Let u = x + ay, where a is an arbitrary constant. Now, replacing p and q by dz/du and a(dz/du) respectively in (1), we get or

FH dz IK / FH a dz IK z du du 2

dz = az dz = a du. or du z Integrating, log z – log b = au or z = beau or z = bea(x + ay), which is a complete integral containing two arbitrary constants a and b. Ex. 3.(a) Find a complete integral of z = pq. [Meerut 1994] Sol. Given equation is z = pq, ... (1) which is of the form f(p, q, z) = 0. Let u = x + ay, where a is an arbitrary constant. Now, replacing p and q by dz/du and a(dz/du) respectively in (1), we get

z=a

FH dz IK du

or

2

or

dz z =± du a

± a z–1/2dz = du.

or

Integrating, ±2 az = u + b or 4(az) = (x + ay + b)2, as u = x + ay Ex. 3.(b) Find a complete integral of pq = 4z. Sol. Proceed as in Ex. 3.(a). Ans. (x + ay + b)2 = az 2 Ex. 4.(a) Find a complete integral of p(1 + q ) = q(z – Γ). [Meerut 1999; Bilaspur 2002; Jiwaji 2003; Ravishanker 2005 Rewa 1998, Vikram 2004] (b) Find a complete integral of p(1 + q2) = q(z – 1). [M.S. Univ. T.N. 2007] Sol. (a) Given equation is p(1 + q2) = q(z – Γ), ...(1) which is of the form f(p, q, z) = 0. Let u = x + ay, where a is an arbitrary constant. Now, replacing p and q by dz/du and a(dz/du) respectively in (1), we get

RS F I UV = a dz T H K W du

dz 1 # a dz du du or

2

{a (z ∋ Γ ) ∋ 1} dz = ; du a

(z – Γ)

or or

1 + a2

FH dz IK du

du = ±

2

= a(z – Γ)

adz . {a ( z ∋ Γ ) ∋ 1}

Created with Print2PDF. To remove this line, buy a license at: http://www.software602.com/

Non-Linear Partial Differential Equations of Order One

3.51

u + b = ± 2 {a ( z ∋ Γ ) ∋ 1} or (u + b)2 = 4{a(z – Γ) – 1}2 2 2 or (x + ay + b) = 4{a(z – Γ) – 1} , a and b being arbitrary constants. (b) Proceed as in part (a) by taking Γ / 1. Ex. 5.(a) Find a complete integral of pz = 1 + q2. [Meerut 1996] Sol. Given equation is pz = 1 + q2, ...(1) which is of the form f(p, q, z) = 0. Let u = x + ay, where a is an arbitrary constant. Now, replacing p and q by dz/du and a(dz/du) respectively in (1), we get Integrating,

z

FH IK

dz dz = 1 + a2 du du

2

z ; ( z 2 ∋ 4a 2 )1/ 2 6 dz = du 2a 2

[z

–z

dz + 1 = 0. du

du

z ( z 2 ∋ 4a 2 )1/ 2

or

4a 2

/

/

du 2a 2

du 2a 2

( z 2 ∋ 4a 2 )1/ 2 ] dz = 2du.

2 9z 2 : b 2 1/ 2 4 a log z # ( z 2 ∋ 4 a 2 )1/ 2 ? / 2u # > ( z ∋ 4a ) ∋ 2 2 0.

3

Ans. C.S. (1 + a )log z = 3a(x + ay) + b; S.S. Does not

exist. G.S. It is given by (1 # a )log z ∋ 3a( x # ay ) ∋ ∃(a) / 0, 3a 2 log z ∋ 3x ∋ 6ay ∋ ∃%(a) / 0 3

14. p2 + q2 = 4z.

Ans. C.I. 4(1+a2)z – (x + ay + b)2 = 0; S.S. z = 0; G.S. It is given by

(1 # a 2 ) z ∋ {x # ay # ∃(a)}2 / 0, az ∋ {x # ay # ∃( a)} 5 { y # ∃%(a)} / 0.

Created with Print2PDF. To remove this line, buy a license at: http://www.software602.com/

Non-Linear Partial Differential Equations of Order One

3.57

3.17. Standard form IV. Equation of the form f1(x, p) = f2(y, q). i.e., a form in which z does not appear and the terms containing x and p are on one side and those containing y and q on the other side. [Bhopal 2010; Ravishankar 1999] Let F(x, y, z, p, q) = f1(x, p) – f2(y, q) = 0. ...(1) Then Charpit’s auxiliary equations are dp F F #p x z

/

dq F F #q y z

dp dq / = f1 x ∋ f2 y ∋ p ( f1

or

=

dz dx dy / / F F F F ∋p ∋q ∋ ∋ p q p q

dz p ) # q( f 2

q)

=

dx dy / , by (1) ∋ f1 p f2 q

Taking the first and the fourth ratios, we have ( f1 / p) dp # ( f1 / x)dx = 0

df1 = 0.

or

f1 = a, a being an arbitrary constant. (1) f1(x, p) = f2(y, q) = a. ...(2) 2 6 Now, (2) 2 f1(x, p) = a and f2(y, q) = a. ...(3) From (3), on solving for p and q respectively, we get p = F1(x, a), say and q = F2(y, a), say ...(4) Substituting these values in dz = p dx + q dy, we get dz = F1(x, a) dx + F2(y, a) dy. Integrating,

Integrating,

z=

0 F ( x, a ) 1

0

dx # F2 ( y, a)dy # b,

which is a complete integral containing two arbitrary constants a and b. Remark 1. Sometimes change of variables can be employed to reduce a given equation in the standard form IV. Remark 2. Singular and general integral are obtained by well known methods. 3.18.SOLVED EXAMPLES BASED ON ART 3.17 Ex. 1. Find a complete integral of x(1 + y)p = y(1 + x)q. Sol. Separating p and x from q and y, the given equation reduces to (xp)/(1 + x) = (yq)/(1 + y) Equating each side to an arbitrary constant a, we have

F I H K

xp 1# x yq =a and =a so that p=a 1# x 1# y x Putting these values of p and q in dz = p dx + q dy, we get

FH

[Agra 1991]

q=a

and

IK

FG H

FG 1# y IJ . H yK

IJ K

a(1 # x ) a(1 # y) 1 1 dx # dy or dz = a # 1 dx # a # 1 dy. x y x y Integrating, z = a(log x + x) + a(log y + y) + b = a(log xy + x + y) + b, which is a complete integral containing two arbitrary constants a and b. Ex. 2. Find a complete integral of p – 3x2 = q2 – y. [Meerut 1996] Sol. Equating each side to an arbitrary constant a, we get p – 3x2 = a and q2 – y = a so that p = a + 3x2 and q = (a + y)1/2. Putting these values of p and q in dz = pdx + qdy, we get dz = (a + 3x2)dx + (a + y)1/2 dy so that z = ax + x3 + (2/3) × (a + y)3/2 + b.

dz =

Created with Print2PDF. To remove this line, buy a license at: http://www.software602.com/

3.58

Non-Linear Partial Differential Equations of Order One

Ex. 3. Find a complete integral of yp = 2yx + log q. [Ravishankar 2005] Sol. Rewritting the given equation, p = 2x + (1/y) log q or p – 2x = (1/y) log q. Equating each side to an arbitrary constant a, we get p – 2x = a and (1/y) log q = a so that p = a + 2x and q = eay. Putting these values of p and q in dz = p dx + q dy, we get dz = (a + 2x)dx + eaydy so that z = (ax + x2) + (1/a) × eay + b. Ex. 4. Find a complete integral of q = px + p2. [Agra 1995; Meerut 1994; Bilaspur 2004; Jabalpur 1998] Sol. Equating each side of the given equation to an arbitrary constant a, we have q=a and px + p2 = a or q = a and p2 + px – a = 0. q=a and p = [–x ± (x2 + 4a)1/2]/2. 6 Putting thse values of p and q in dz = p dx + q dy, we get dz = (1/2) × [–x ± (x2 + 4a)1/2]dx + a dy. z=–

Integrating,

or

2

LM N

}OPQ

{

x 1 x 2 2 ; ( x # 4a) # 2 a log x # ( x # 4a) + ay + b. 4 2 2

Ex. 5. Solve py + qx + pq = 0. [Kurukshetra 2004; I.A.S 1990] Sol. Given py + q(x + p) = 0. or p/(p + x) = –q/y. Equating each side to an arbitrary constant a, we get p/(p + x) = a and –q/y = a p = (xa)/(1 – a) and q = –ay. 2 Putting thse values of p and q in dz = p dx + q dy, we get dz = {a/(1 – a)} x dx – ay dy so that z = {a/(1 – a)}× (x2/2) – a × (y2/2) + b/2 2 2 2z = {a/(1 – a)}x – ay + b, a, b being arbitrary constants. Ex. 6. Find a complete integral of z2(p2 + q2) = x2 + y2, i.e., z2[( z/ x)2 + ( z/ y)2] = x2 + y2. [Agra 2006; Jabalpur 2004; Rewa 2002 Sagar 1999; Vikram 1996 Delhi Maths Hons 1990; I.A.S. 1989; Kanpur 1994; Meerut 2003] Sol. Given

z

2

FH z IK x

2

#z

2

FG z IJ H yK

2

= x2 + y2

FH z z IK # FG z z IJ x H yK 2

or

2

= x2 + y2. ...(1)

Let z dz = dz so that z2/2 = Z. ...(2) 2 2 2 2 2 2 2 Using (2), (1) becomes ( Z/ x) + ( Z/ y) = x + y or P + Q = x + y2, where P = Z/ x and Q = Z/ y. Separating P and x from Q and y, we get P2 – x2 = y2 – Q2. Equating each side of the above equation to an arbitrary constant a2, we get 2 P – x 2 = a2 and y2 – Q2 = a2 so that P = (a2 + x2)1/2 and Q = (y2 – a2)1/2. Putting these values of P and Q in dZ = P dx + Q dy, we have dZ = (a2 + x2)1/2 dx + (y2 – a2)1/2 dy. Integrating, Z = (x/2) × (a2 + x2)1/2 + (a2/2) × log{x + (a2 + x2)1/2} + (y/2) × (y2 – a2)1/2 – (a2/2) × log {y + (y2 – a2)1/2} + (b/2) or

z2 = x2 (a 2 # x2 )1/ 2 + a2 log [x + (a 2 # x2 )1/ 2 } + y ( y 2 ∋ a 2 )1/ 2 – a2log{y + ( y 2 ∋ a 2 )1/ 2 }+ b [ 2

From (2), Z = z2/2]

2

Ex. 7. Find a complete integral of z(p – q ) = x – y. [Bilaspur 2003; Indore 2002, 02; Jiwaji 2000; Bangalore 1995; I.A.S 1989]

Created with Print2PDF. To remove this line, buy a license at: http://www.software602.com/

Non-Linear Partial Differential Equations of Order One

3.59

( z z x )2 – ( z z y) 2 = x – y.

Sol. Re–writting the given equation,

...(1)

z dz = dZ Let so that (2/3) × z3/2 = Z. ...(2) 2 2 Using (2), (1) becomes ( Z/ x) – ( Z/ y) = x – y or P2 – Q2 = x – y, where P = Z/ x and Q = Z/ y. Separating P and x from Q and y, we get P2 – x = Q2 – y. ...(3) Equating each side to an arbitrary constant a, we get P2 – x = a and Q2 – y = a so that P = (x + a)1/2 and Q = (y + a)1/2 1/2 Putting these values of P and Q in dZ = P dx + Q dy, dZ = (x + a) dx + (y + a)1/2 dy. Integrating, Z = (2/3) × (x + a)3/2 + (2/3) × (y + b)3/2 + 2b/3 3/2 or (2/3) × z = (2/3) × (x + a)3/2 + (2/3) × (y + b)3/2 + 2b/3, as Z = (2/3) × z3/2 or z3/2 = (x + a)3/2 + (y + a)3/2 + b, a, b being arbitrary constants. Ex. 8. Find a complete integral of z(xp – yq) = y2 – x2. Sol. Re–writting the given equation, we have

xz z ∋ yz z = y2 – x2 x y

FG z IJ ∋ yFG z z IJ H xK H yK

or

x z

= y2 – x2.

...(1)

Let z dz = dZ so that z2/2 = Z. ...(2) 2 2 2 Using (2), (1) becomes x( Z/ x) – y( Z/ y) = y – x or xP – yQ = y – x2, where P = Z/ x and Q = Z/ y. Separating P and x from Q and y, we get xP + x2 = yQ + y2. Equating each side to an arbitrary constant a, we have xP + x2 = a and yQ + y2 = a so that P = a/x – x and Q = a/y – y. Putting these values of P and Q in dZ = P dx + Q dy, dZ = (a/x – x)dx + (a/y – y)dy. Integrating, Z = a log x – (x2/2) + a log y – (y2/2) + b/2 or z2/2 = a(log x + log y) – (x2 + y2 – b)/2 or z2 = 2a log (xy) – x2 – y2 + b. Ex. 9. Find a complete integral of p2 + q2 = z2(x + y). [Agra 2010; M.S. Univ. T.N. 2007]

F zI F zI Sol. Given H K # G J x H yK 2

FH 1 z IK # FG 1 z IJ z x H z yK

2

2

2

= z (x + y)

or

2

= x + y ....(1)

Let (1/z)dz = dZ so that log z = Z. ...(2) 2 2 2 2 Using (2), (1) becomes ( Z/ x) + ( Z/ y) = x + y or P + Q = x + y, where P = Z/ x and Q = Z/ y. Separating P and x from Q and y, we get P2 – x = y – Q2. Equating each side to an arbitrary constant a, we have P2 – x = a and y – Q2 = a so that P = (a + x)1/2 and Q = (y – a)1/2. Putting these values of P and Q in dZ = P dx + Q dy, dZ = (a + x)1/2dx + (y – a)1/2dy. Integrating, Z = (2/3) × [(a + x)3/2 + (y – a)3/2] + (2/3) × b 3/2 3/2 6 log z = (2/3) × [(a + x) + (y – a) + b] is a complete integral, using Z = log z Ex. 10. Find a complete integral of p2 + q2 = (x2 + y2)z. [Delhi Maths Hons. 1995] Sol. The given equation can be rewritten as 1 z

LMF z I # FG z IJ OP = x MNH x K H y K PQ 2

FG 1 z IJ # FG 1 z IJ = x H z x K H z yK 2

2

2

+ y2

or

2

2

+ y2.

...(1)

Created with Print2PDF. To remove this line, buy a license at: http://www.software602.com/

3.60

Non-Linear Partial Differential Equations of Order One

d1 z idz = dZ

Let

z–1/2dz = dZ

i.e., 2

2 z = Z. ...(2)

so that

Using (2), (1) becomes ( Z/ x) + ( Z/ y) = x + y or P + Q = x2 + y2, where P = Z/ x and Q = Z/ y. Separting P and x from Q and y, we get P2 – x2 = y2 – Q2. Equating each side to an arbitrary constant a2, we have P2 – x2 = a2 and y2 – Q2 = a2 so that P = (a2 + x2)1/2 and Q = (y2 – a2)1/2 2 2 1/2 Putting these values of P and Q in dZ = P dx + Q dy, dZ = (a + x ) dx + (y2 – a2)1/2dy. Z=

Integrating, or

2

2

2

2

2

2 2 x 2 y x y (x + a2)1/2+ a sinh–1 # (y2 – a2)1/2– a cosh–1 # b 2 a 2 2 2 a 2

4z1/2 = x(x2 + a2)1/2 + a2sinh–1(x/a) + y(y2 – a2)1/2 – a2cosh–1(y/a) + b, as Z / 2 z Ex. 11. Find a complete integral of (p2/x) – (q2/y) = (1/z) × [(1/x) + (1/y)]. [Delhi B.Sc. Hons. 1996] Sol. The given equation can be re–written as

z x

FH z IK x

2

∋z y

FG z IJ H yK

2

= 1#1 x y

or z dz = dZ

Let Using (2), (1) becomes

FH IK

FG IJ H K

1 x

FH

so that

z z x

IK

2

∋1 y

FG H

z z y

IJ K

2

= 1#1 x y

(2/3) × z3/2 = Z.

2

2

....(1) ...(2)

2

2 Q P 1 Z ∋1 Z = 1#1 ∋ or = 1#1, x y x y x y x x y y where P = Z/ x and Q = Z/ y. Separating P and x from Q and y, we get (P2 – 1)/x = (Q2 + 1)/y. Equating each side to an arbitrary constant a, we have (P2 – 1)/x = a and (Q2 + 1)/y = a so that P = (1 + ax)1/2 and Q = (ay – 1)1/2. Putting these values of P and Q in dZ = P dx + Q dy, dZ = (1 + ax)1/2dx + (ay – 1)1/2dy. Integrating, Z = (2/3a) × (1 + ax)3/2 + (2/3a) × (ay – 1)3/2 + (2/3a) × b or az3/2 = (1 + ax)3/2 + (ay – 1)3/2 + b, as Z = (2/3) × z3/2. Ex. 12. Find a complete integral of yzp2 = q. [M.S. Univ. T.N. 2007]

Sol. Given

yz2

FH z IK x

2

=z

z y

FG z IJ / FG z z IJ . ...(1) H xK H yK 2

or

y z

Let z dz = dZ so that z2/2 = Z. ...(2) 2 Using (2), (1) becomes y( Z/ x) = Z/ y or yP2 = Q, ...(3) where P = Z/ x and Q = Z/ y. Separating P from y and Q, we get P2 = Q/y = a2, (say) ; a being an arbitrary constant. Hence P=a and Q = ya2. 2 2 2 Then, dZ = P dx + Q dy reduces to dZ = a dx + ya dy so that Z = ax + (a /y )/2 + b/2 or z2/2 = ax + (a2y2)/2 + b/2 or z2 = 2ax + a2y + b. Ex. 13. Find a complete integral of zpy2 = x(y2 + z2q2). Sol. Given

) z∗ ) z∗ y2 + z , / x y2 # x + z , − x. − y.

2

...(1)

Let z dz = dZ so that z2/2 = Z. ...(2) 2 2 2 Using (2), (1) becomes y ( Z/ x) = xy + x ( Z/ y) or y2P = x(y2 + Q2), where P = Z/ x and Q = Z/ y. Separating P and x from, Q and y, we get

Created with Print2PDF. To remove this line, buy a license at: http://www.software602.com/

Non-Linear Partial Differential Equations of Order One

3.61 2

2

2

P/x = (y + Q )/y . Equating each side to an arbitrary constant a, we get P/x = a and 1 + (Q2/y2) = a so that P = ax and Q = ± (a – 1)1/2y. 1/2 dZ = P dx + Q dy = ax dx ± (a – 1) y dy 6 Integrating, Z = (ax2/2) ± (a – 1)1/2(y2/2) + b/2 or z2 = ax2 ± (a – 1)1/2y2 + b, as Z = z2/2. Ex. 14. Find the complete integral of the partial differential equation

2 p2q2 # 3x 2 y2 / 8 x2 q2 ( x 2 # y2 ) Sol. Re–writing the given equation, we have 2q2 ( p2 ∋ 4x4 ) / x2 y2 (8q2 ∋ 3)

[I.A.S. 2001]

( p2 ∋ 4 x4 ) / x2 / y2 (8q2 ∋ 3) / 2q2 / 4a2 , say

or

where a is an arbitrary constant. Then, p2 / 4x 2 (a2 # x2 )

p / 2 x(a2 # x2 )1/ 2

so that

8q2 ( y2 ∋ a2 ) / 3y2

and

q / (3/ 2)1/ 2 5 ( y / 2) 5 ( y2 ∋ a2 )∋1/2

and

Substituting these values in dz / p dx # q dy , we get

dz / 2 x(a2 # x 2 )1/2 dx # (3/ 2)1/ 2 5 ( y / 2) 5 ( y2 ∋ a2 )∋1/ 2 dy

0

0

Integrating, z / 2 x (a 2 # x 2 )1/ 2 dx # (3 / 2)1/ 2 5 (1/ 2) 5 y ( y 2 ∋ a 2 )∋1/ 2 dy # b i.e.,

Put

2

2

2

2

x + a = u and y – a = v so that 2xdx = du and xdx = (1/2) × du and ydy = (1/2) × dv. Then (1) reduces to

0

... (1)

2ydy = dv ... (2)

0

z / u1/ 2 du # (3/ 2)1/ 2 5 (1/ 4) 5 v –1/ 2 d v # b

z / (2 / 3) 5 u 3 / 2 # (3/ 2)1/ 2 5 (1/ 4) 5 2v1/ 2 # b

or

or z / (2 / 3) 5 ( x 2 # a 2 )3/ 2 # (3/ 2)1/ 2 5 (1/ 2) 5 ( y 2 ∋ a 2 )1/ 2 # b, which is the required complete integral containing a and b as arbitrary constants. Ex. 15. Find the complete integral of the partial differential equation p2q2 + x 2y 2 2 2 2 = x q (x + y2) [Delhi Maths (H) 2002; Agra 2005] Sol. Re–writing, p2 / x 2 # y2 / q2 / x 2 # y2

2

p / x( x 2 # a2 )1/ 2 ,

or

( p2 / x 2 ) ∋ x 2 / y2 ∋ ( y2 / q2 ) / a2 , say q = y/(y2 – a2)1/2

and

6 dz / p dx # q dy

dz / x( x 2 # a2 )1/ 2 dx # y( y2 ∋ a2 )∋1/ 2 dy

becomes

z / (1/ 3) 5 ( x2 # a2 )1/ 2 # ( y2 ∋ a2 )1/ 2 # b ,

Integrating,

which is complete integral with a and b as arbitrary constants. Ex. 16. Find the complete integral of (1 ∋ x 2 )yp2 # x2 q / 0

( x2 ∋ 1) p2 / x2 / q / y / a2 , say

Sol. Re–writing, we have

6 p / ax /( x2 ∋ 1)1/ 2 and q / a2 y . Hence dz / p dx # q dy becomes

dz / ax( x 2 ∋ 1)∋1/ 2 dx # a2 ydy

so that

z / a( x 2 ∋ 1)1/ 2 # (a2 y2 ) / 2 # b .

Ex. 17. Find the the complete integral of p # q ∋ 2 px ∋ 2qy # 1 / 0 .

p ∋ 2 px / 2qy ∋ q ∋ 1 / a, say

Sol. Re–writing,

6

p/

a , 1 ∋ 2x

q/

a#1 2y ∋ 1

and so

dz / p dx # q dy /

a dx (a # 1)dy # 1 ∋ 2x 2y ∋ 1

Created with Print2PDF. To remove this line, buy a license at: http://www.software602.com/

3.62

Non-Linear Partial Differential Equations of Order One

z / ∋(a / 2) 5 log | 1 ∋ 2 x | #(1/ 2) 5 (a # 1)log | 2 y # 1 | #b .

Integrating,

Ex. 18. Find the complete integral of 2 x( z2 q2 # 1) / pz

2 x {( z z / y)2 # 1} / (z z / x)

Sol. Re–writing the given equation, we have

... (1)

2

Putting z dz / dZ so that z / 2 / Z , (1) reduces to or 2 x{( Z / y)2 # 1} / Z / x where P / Z / x and Q / Z / y . Re–writing (2), we have ( P / 2 x ) ∋ 1 / Q2 / a2 , say

P / 2 x(1 # a2 ),

so that

6 d Z / P dx # Q dy

... (2)

Q=a 2

dZ / 2 x(1 # a )dx # a dy

becomes

Integrating, Z / (1 # a2 ) x2 # ay # b

2 x (Q2 # 1) / P ,

or

z 2 / 2 / (1 # a2 ) x # ay # b .

EXERCISE 3 (F) Find a complete integral of the following equations (1 – 9) 1(a). p2 = q + x. Ans. z = (2/3) × (a + x)3/2 + ay + b. 2 2 2 (b). p y (1 + x ) = qx . [Delhi B.A (Prog) II 2011] Ans. z = a(1 + x2)1/2 +(a2y/2) + b. 2 2 2. p + q = x + y. [Agra 2009; Meerut 2007] Ans. 3z = 2(x + a)3/2 + 2(y – a)3/2 + b. 3. p2 + q2 = x2 + y2. [Jiwaji 1999; Ravishankar 2003] Ans. 2z = x(x2 + a2)1/2 + a2sinh–1(x/a) + y(y2 – a2)1/2 – a2cosh–1(y/a) + b. 4. pey = qex. [Jiwaji 1996] Ans. z = aex + aey + b. 5. p1/3 – q1/3 = 3x – 3y. Ans. z = 3x3 – 3ax2 + a2x + 2y4 – 4ay3 + 3a2y2 – a3y + b. 2 6. q = 2yp . Ans. z = ax + a2y2 + b. 2 3 2 2 2 2 1/2 2 –1 7. p – y q = x – y . Ans. 2z = x(x + a ) + a sinh (x/a) – (a2/2)+ log y2 + b. 8. z2(p2 + q2) = x2 + e2y. [Delhi Maths (H) 2005] Ans. z2 = x(x2 + a)1/2 + a sinh–1(x/ a ) + 2(e2y – a)1/2 – a tan–1 {(e2y – a)/a}1/2 + b 9. p + q = px + qy. [Bangalore 1996] Ans. z = –a log (1 – x) + a log (y – 1) + b. Solve the following partial differential equations: (10 – 17) 10. pq = xy Ans. C.I. 2z = ax2 + y2/a + b; S.S. Does not exist G.S. 2z ∋ax2 ∋ y2 / a ∋∃(a) / 0,

x2 ∋ y 2 / a 2 # ∃%(a) / 0 11.

p # q / 2 x. Ans. C.I. z = (2x – a)3/6 + a2y + b; S.S. Does not exit; G.S.

z ∋ (2 x ∋ a)3 / 6 ∋ a 2 y ∋ ∃(a ) / 0, (2 x ∋ a)2 / 2 ∋ 2ay ∋ ∃%(a) / 0 12. q (p – cos x) = cos y. Ans. z = ax + sin x + (1/a) × sin y + b; S.S. Does not exist G.S. z ∋ ax ∋ (1/ a) 5 sin y ∋ ∃(a) / 0, ∋ x ∋ (1/ a 2 ) 5 sin y # ∃%(a) / 0 13. q = xyp2 Ans. C.I. 2 z / 4 ax # ay 2 # b; S.S. Does not exist; G.S. 2z ∋4 ax ∋ay2 ∋∃(a) / 0, 2 (x / a) # y2 #∃%(a) / 0

14. x2p2 = q2y. 2

Ans. C.I. z / a log x # 2 ay # b; S.S. Does not exist. 2

15. p – q = x + y .

G.S. z ∋ a log x ∋ 2 ay ∋ ∃(a) / 0, log x # 2 y # 2 a ∃%(a) / 0 Ans. C.I. z = (x3 – y3)/3 + a(x + y) + b; S.S. Does not exit; G.S. z ∋ ( x3 ∋ y3 ) / 3 ∋ a( x # y) ∋ ∃(a) / 0, x # y # ∃%(a) / 0

16. p 2 ∋ x / q 2 ∋ y

Ans. C.I. 3z = 2(x + a)3/2 + 2(y + a)3/2 + b; S.S. Does not exist

Created with Print2PDF. To remove this line, buy a license at: http://www.software602.com/

Non-Linear Partial Differential Equations of Order One

3.63

G.S. 3 z ∋ 2( x # a)3/ 2 ∋ 2( y # a)3/ 2 ∋ ∃(a) / 0, 3( x # a)1/ 2 # 3( y # a)1/ 2 # ∃%(a) / 0 17. px + q = p2.

3

4

2 2 1/ 2 Ans. C.I. z / (1/ 4) 5 x # x ( x # 4a ) + a log {x + (x2 + 4a2)1/2} ay

+ b; S.S. Does not exist G.S. z ∋ (1/ 4) 5 {x 2 # x( x2 # 4a)1/ 2 } ∋ a log{x # ( x2 # 4a)1/ 2 } ∋ ay ∋∃( a) / 0, (x/2) × (x2 + 4a)–1/2 + log {x + (x2 + 4a)1/2} + (2a)/[{x + (x2 + 4a)1/2} × (x2 + 4a)] # y # ∃%(a) / 0 3.19. JACOBI’S METHOD [Himachel 2005; Meerut 2005, 06, 08; Pune 2010] This method is used for solving partial differential equations involving three or more independent variables. The central idea of Jacobi’s method is almost the same as that of Charpit’s method for two independent variables. We begin with the case of three independent variables. The results arrived at are, however, general and will be used with suitable modification for the case of four independent variables and so on. Let p1 = z/ x1, p2 = z/ x2 and p3 = z/ x3. Consider a partial differential equation f(x1, x2, x3, p1, p2, p3) = 0, ...(1) where the dependent variable z does not occur except by its partial differential coefficients with respect to the three independent variables x1, x2, x3. The main idea in Jacobi’s method is to get two additional partial differential equations of the first order F1(x1, x2, x3, p1, p2, p3) = a1 ...(2) and F2(x1, x2, x3, p1, p2, p3) = a2, ...(3) where a1 and a2 are two arbitrary constants such that (1), (2) and (3) can be solved for p1, p2, p3 in terms of x1, x2, x3 which when substituted in dz = p1dx1 + p2dx2 + p3dx3, ...(4) makes it integrable, for which the conditions are p2/ x1 = p1/ x2, p3/ x2 = p2/ x3, and p1/ x3 = p3/ x1 ...(5) Differentiating (1) and (2) partially, w.r.t. x1, we have

and

f f p1 f p2 f p3 =0 # # # x1 p1 x1 p2 x1 p3 x1

...(6)

F1 F p F p F p # 1 1 # 1 2 # 1 3 = 0. x1 p1 x1 p2 x1 p3 x1

...(7)

Eliminating p1/ x1 from (6) and (7), we have ) f F1 ) f F1 f F1 ∗ p3 f F1 ∗ ) f F1 f F1 ∗ p2 +− x p ∋ p x ,. # +− p p ∋ p p ,. x + +− p p ∋ p p ,. x = 0. ...(8) 3 1 1 3 1 1 1 1 1 2 1 1 2 1 Similarly, differentiating (1) and (2) partially w.r.t. x2 and then eliminating p2/ x2 from the resulting equations, we have ) f F1 ) f F1 f F1 ∗ p3 f F1 ∗ ) f F1 f F1 ∗ p1 +− x p ∋ p x ,. # +− p p ∋ p p ,. x + +− p p ∋ p p ,. x = 0. ...(9) 3 2 2 3 2 2 2 2 2 1 2 2 1 2 Again, differentiating (1) and (2) partially w.r.t. x3 and then eliminating p3/ x3 from the resulting equation, we have ) f +− x 3

F1 f F1 ∗ ) f ∋ # p3 p3 x3 ,. +− p1

F1 f F1 ∗ p1 ) f F1 f F1 ∗ p2 ∋ ∋ + + = 0. ...(10) , p3 p3 p1 . x3 − p2 p3 p3 p2 ,. x3 Adding (8), (9) and (10) and using the relations (5), we have

Created with Print2PDF. To remove this line, buy a license at: http://www.software602.com/

3.64

Non-Linear Partial Differential Equations of Order One

! #f #F1 #f #F1 ! #f #F1 #f #F1 ! #f #F1 #f #F1 %∋ #x #p ∃ #p #x &( + %∋ #x #p ∃ #p #x &( + %∋ #x #p ∃ #p #x &( = 0. ...(11) 3 3 3 3 1 1 1 1 2 2 2 2

The L.H.S. of (11) is generally denoted by (f, F1). Then, (11) becomes 3

(f, F1) =

F #f

∗ GH #x r )1

r

#F1 #f #F1 ∃ #pr #pr #x r

IJ = 0. K

...(11)+

Starting with (1) and (3) in place of (1) and (2) and proceeding as above, we have a similar 3

relation

(f, F2) =

F #f

∗ GH #x r )1

#F2 #f #F2 ∃ # pr # pr # x r

r

IJ = 0. K

...(12)

Again, starting with (2) and (3) in place of (1) and (2) and proceeding as above, we again

F #F

3

have a similar relation

(F1, F2) =

∗ GH #x r )1

#F2 #F1 #F2 ∃ # pr # pr # x r

1 r

IJ = 0. K

...(13)

(11) [or (11)+ ] and (12) are linear equtions of first order with x1, x2, x3, p1, p2, p3 as independent variables and F1, F2 as dependent variables respectively. For both of these equations, Lagrange’s auxiliary equations are dp3 dx3 dp1 dx1 dp2 dx2 ) ) = ) = , ...(14) #f #x3 ∃ #f #p3 #f #x1 ∃ #f #p1 #f #x2 ∃ #f #p2 which are known as Jacobi’s auxiliary equations. We try to find two independent integrals F1(x1, x2, x3, p1, p2, p3) = a1 and F2(x1, x2, x3, p1, p2, p3) = a2 with help of (14). If these relations satisfy (13), these are the required two additional relations (2) and (3). We now solve (1), (2) and (3) for p1, p2, p3 in terms of x1, x2, x3. Substituting these values in (4) and then integrating the resulting equation, we shall obtain a complete integral of the given equation containing three arbitrary constants of integration. 3.20. Working rules for solving partial differential equations with three or more independent variable. Jacobi’s method Step 1 : Suppose the given equation with three independent variables is f(x1, x2, x3, p1, p2, p3) = 0. ...(1) Step II. We write Jacobi’s auxiliary equations

dp1 dx1 dp2 dx2 dp3 dx3 ) ) ) = = . #f #x1 ∃ #f #p1 #f #x2 ∃ #f #p2 #f #x3 ∃ #f #p3 Solving these equation we obtain two additional equations F1(x1, x2, x3, p1, p2, p3) = a1 ...(2) F2(x1, x2, x3, p1, p2, p3) = a2. ...(3) where a1 and a2 are arbitrary constants. While obtaining (2) and (3), try to select simple equations so that later on solutions of (1), (2) and (3) may be as easy as possible. Step III. Verify that relations (2) and (3) satisfy the condition 3

(F1, F2) =

F #F

∗ GH #x r )1

1 r

#F2 #F1 #F2 ∃ # pr # pr # x r

IJ = 0. K

...(4)

If (4) is satisfied then solve (1), (2) and (3) for p1, p2, p3 in terms of x1, x2, x3. Their substitution in dz = p1dx1 + p2dx2 + p3dx3 and subsequent integration leads to a complete integral of the given equation. Remark 1. Sometime, change of variables can be employed to reduce the given equation in

Created with Print2PDF. To remove this line, buy a license at: http://www.software602.com/

Non-Linear Partial Differential Equations of Order One

3.65

a form solvable by Jacobian method. Remark 2. While solving a partial differential equation with four independent variables, we modify the above working rule as follows : Step I. Suppose the given equation with four independent variables is f(x1, x2, x3, x4, p1, p2, p3, p4) = 0. ...(1) Step II. We write Jacobi’s auxiliary equations

dp1 dx1 dp2 dx2 dp3 dx3 dp4 dx4 ) ) ) ) = = = #f #x1 ∃ #f #p1 #f #x2 ∃ #f #p2 #f #x3 ∃ #f #p3 #f #x4 ∃ #f #p4 Solving these equations we obtain three additional equations F1(x1, x2, x3, x4, p1, p2, p3, p4) = a1, ...(2) F2(x1, x2, x3, x4, p1, p2, p3, p4) = a2, ...(3) and F3(x1, x2, x3, x4, p1, p2, p3, p4) = a3, ...(4) where a1, a2 and a3 are arbitrary constants. Step IV. Verify that relations (2), (3) and (4) satisfy following three conditions: 4

(F1, F2) =

F #F

∗ GH #x

1

r )1

r

#F2 #F1 #F2 ∃ #pr #pr #xr

IJ = 0, K

4

(F2, F3) =

...(4)

F #F

∗ GH #x r )1

2 r

#F3 #F2 #F3 ∃ #pr #pr #x r

IJ = 0 ...(5) K

4

(F3, F1) =

and

! #F3 #F1 #F3 #F1 ∃ & = 0. #pr #xr ( r #pr r )1

∗ %∋ #x

...(6)

If (4), (5) and (6) are satisfied, then solve (1), (2), (3) and (4) for p1, p2, p3 and p4 in terms of x1, x2, x3 and x4. Their substitution in dz = p1dx1 + p2dx2 + p3dx3 + p4dx4 and subsequent integration leads to a complete integral of the given equation. 3.21 SOLVED EXAMPLES BASED ON ART 3.20. Ex. 1. Find a complete integral of p13 + p22 + p3 = 1. Sol. Let the given equation be rewritten as

[I.A.S. 1997; Meerut 2006]

3 2 f(x1, x2, x3, p1, p2, p3) = p1 , p2 , p3 ∃ 1 ) 0 . − Jacobi’s auxiliary equations are

...(1)

dp3 dx3 dp1 dx1 dp2 dx2 = ) ) ) ) #f #x1 ∃ #f #p1 #f #x2 ∃ #f #p2 #f #x3 ∃ #f #p3

dp dx dp1 dx1 dp dx2 ) = 2 ) = 3 ) 3 , using (1) 0 ∃ 2 p2 0 ∃3 p12 0 ∃1

or

From first and third fractions, dp1 = 0 and dp2 = 0 so that F1(x1, x2, x3, p1, p2, p3) = p1 = a1. − Here and F2(x1, x2, x3, p1, p2, p3) = p2 = a2. 3

(F1, F2) =

Now,

r )1

or

(F1, F2) =

F #F

∗ GH #x

1 r

#F2 #F1 #F2 ∃ # pr # pr # x r

p1 = a1

and

p2 = a2. ...(2) ...(3)

IJ K

#F1 #F2 #F1 #F2 #F1 #F2 #F1 #F2 #F1 #F2 #F1 #F2 ∃ , ∃ , ∃ #x1 #p1 #p1 #x1 #x2 #p2 #p2 #x2 #x3 #p3 #p3 #x3

(F1, F2) = (0)(0) – (1)(0) + (0)(1) – (0) (0) + (0)(0) – (0)(0) = 0, by (3) and (4). Thus, we have verified that for relations (2) and (3), (F1, F2) = 0. Hence (2) and (3) may be taken as additional equations. or

Created with Print2PDF. To remove this line, buy a license at: http://www.software602.com/

3.66

Non-Linear Partial Differential Equations of Order One

Solving (1), (2) and (3) for p1, p2, p3, p1 = a1, p2 = a2, p3 = 1 – a13 – a22. Putting these values in dz = p1dx1 + p2dx2 + p3dx3, we have dz = a1dx1 + a2dx2 + (1 – a13 – a22)dx3. Integrating, z = a1x1 + a2x2 + (1 – a13 – a22) x3 + a3, which is a complete integral of given equation containing three arbitrary constants a1, a2, and a3. Ex. 2. Find a complete integral of x32p12p22p32 + p12p22 – p32 = 0. [Delhi Maths (H) 2006] Sol. Let f(x1, x2, x3, p1, p2, p3) = x32p12p22p32 + p12p22 – p32 = 0. ...(1) Jacobi’s auxiliary equations are − dp3 dx3 dp1 dx1 dp2 dx2 ) ) ) = = #f #x3 ∃ #f #p3 #f #x1 ∃ #f #p1 #f #x2 ∃ #f #p2 or

dp1 dx1 dp dx2 ) = 2) = ....., by (1) 2 2 2 2 2 0 0 ∃ (2 p2 x3 p12 p32 , 2 p2 p12 ) ∃ (2 p1 x 3 p2 p3 , 2 p1 p2 )

From first and third fractions, dp1 = 0 and dp2 = 0 so that p1 = a1 and F1(x1, x2, x3, p1, p2, p3) = p1 = a1, − Here and F2(x1, x2, x3, p1, p2, p3) = p2 = a2. As in Ex. 1, verify that for relations (2) and (3), (F1, F2) = 0. Hence (2) and (3) may be taken as the additional equations. Solving (1), (2) and (3) for p1, p2, p3, we have p1 = a1, p2 = a2, p3 ) .a1a2

p2 = a2. ...(2) ...(3)

(1 ∃ a12 a22 x32 ) .

Putting these values in dz = p1dx1 + p2dx2 + p3dx3, we get

{

dz = a1dx1 + a2dx2 ± a1a 2

}

(1 ∃ a12 a 22 x 32 ) dx , whose integration gives 3

z = a1x1 + a2x2 ± sin–1(a1a2x3) + a3, a1, a2, a3 being arbitrary constants. Ex. 3. Find a complete integral of p1x1 + p2x2 = p32. [Meerut 2007] Sol. Let f(x1, x2, x3, p1, p2, p3) = p1x1 + p2x2 – p32 = 0. ...(1) Jacobi’s auxiliary equations are − dp1 dx1 dp2 dx2 dp3 dx3 ) ) ) = = #f #x1 ∃ #f #p1 #f #x2 ∃ #f #p2 #f #x3 ∃ #f #p3 or

dp1 dx1 dp dx dp dx ) = 2 ) 2 = 3 ) 3 , using (1) p1 ∃ x1 p2 ∃ x2 0 2 p3

...(2)

Taking the first two fractions of (2), (1/x1)dx + (1/p1)dp1 = 0. / log x1 + log p1 = log a1. and let F1(x1, x2, x3, p1, p2, p3) = x1p1 = a1. ...(3) − x1p1 = a1 Taking the third and fourth fractions of (2), (1/x2)dx2 + (1/p2)dp2 = 0. x p = a and let F (x , x , x − 2 2 2 2 1 2 3, p1, p2, p3) = x2p2 = a2. ...(4) As in Ex. 1, verify that for relations (3) and (4), (F1, F2) = 0. Solving (1), (3) and (4) for p1, p2, p3, p1 = a1/x1, p2 = a2/x2 and p3 = (a1 + a2)1/2. Putting these values in dz = p1dx1 + p2dx2 + p3dx3, we have dz = (a1/x1)dx1 + (a2/x2)dx2 + (a1 + a2)1/2 dx3. Integrating, z = a1 log x1 + a2 log x2 + x3 (a1 + a2)1/2 + a3. Ex. 4. Find complete integral of 2p1x1x3 + 3p2x32 + p22p3 = 0. [I.A.S. 1998, Meerut 1999] Sol. Let f(x1, x2, x3, p1, p2, p3) = 2p1x1x3 + 3p2x32 + p22p3 = 0. ...(1)

Created with Print2PDF. To remove this line, buy a license at: http://www.software602.com/

Non-Linear Partial Differential Equations of Order One

3.67

Jacobi’s auxiliary equations are



dp1 dx1 dp3 dx3 dp2 dx2 ) ) ) = = #f #x1 ∃ #f #p1 #f #x2 ∃ #f #p2 #f #x3 ∃ #f #p3

or

or

dp1 dx1 dp3 dx dp dx 2 ) ) 32 , by (1) = 2) = ...(2) 2 2 p1 x 3 ∃ 2 x1 x 3 0 2 p x , 6 p x ∃3 x 3 ∃ 2 p2 p3 ∃ p2 1 1 2 3 Taking the first two fractions of (2), (1/p1)dp1 + (1/x1)dx1 = 0. so p1x1 = a1 Let F1(x1, x2, x3, p1, p2, p3) = p1x1 = a1. ...(3) From the third fraction of (2), dp2 = 0 so that p2 = a2. Let F2(x1, x2, x3, p1, p2, p3) = p2 = a2. ...(4) As in Ex. 1, verify that for relations (3) and (4), (F1, F2) = 0. Solving (1), (3) and (4) for p1, p2, p3, p1 = a1/x1, p2 = a2 , p3 = – (2a1x3 + 3a2x32)/a22. Putting these values in dz = p1dx1 + p2dx2 + p3dx2, we have dz = (a1/x1)dx1 + a2dx2 – {(2a1x3 + 3a2x32)/a22}dx3, whose integration gives z = a1 log x1 + a2x2 – (a1x32 + a2x33)/a22 + a3. which is required complete integral Ex. 5. Find a complete integral of p3x3 (p1 + p2) + x1 + x2 = 0. Sol. Given f(x1, x2, x3, p1, p2, p3) = p3x3(p1 + p2) + x1 + x2 = 0. ...(1) Jacobi’s auxiliary equations are − dp1 dx1 dp3 dx3 dp2 dx2 ) ) = = ) #f #x1 ∃ #f #p1 #f #x2 ∃ #f #p2 #f #x3 ∃ #f #p3 dp1 dx1 dp dx 2 dp3 dx3 ) ) = 2) = , by (1) ...(2) 1 p3 x3 1 ∃ p3 x3 p3 ( p2 , p3 ) ∃ x3 ( p1 , p2 ) Taking the two fractions of (2), dp1 – dp2 = 0 so p1 – p2 = a1 Let F1(x1, x2, x3, p1, p2, p3) = p1 – p2 = a1. ...(3) Taking the fifth and sixth fractions of (2), (1/p3)dp3 + (1/x3)dx3 = 0 giving p3x3 = a3 Let F2(x1, x2, x3, p1, p2, p3) = p3x3 = a2. ...(4) 3

Now, (F1, F2) =

=

FG #F H #x

1 1

F #F

∗ GH #x r )1

1 r

#F2 #F1 #F2 ∃ #p1 #p1 #x1

#F2 #F1 #F2 ∃ # pr # pr # x r

IJ + FG #F K H #x

1 2

IJ K

IJ FG K H

#F2 #F1 #F2 #F1 #F2 #F1 #F2 ∃ , ∃ #p2 #p2 #x 2 #x 3 #p3 #p3 #x 3

IJ K

= (0)(0) – (1)(0) + (0)(0) – (–1)(0) + (0)(x3) – (0)(p3) = 0 by (3) and (4) Thus, we have verified that for the relations (3) and (4), (F1, F2) = 0. From (1) and (4), a2(p1 + p2) + x1 + x2 = 0 or p1 + p2 = –(x1 + x2)/a2. ...(5) Solving (3) and (5),

p1 =

a1 x1 , x 2 ∃ 2 2 a2

and

p2 = –

a1 x ,x ∃ 1 2. 2 2a2

...(6)

Again, from (4), p3 = a2/x3. ...(7) Putting the values of p1, p2, p3 given by (6) and (7) in dz = p1dx1 + p2dx2 + p3dx3, we have dz = Integrating,

a1 a2 (x , x ) (dx1 – dx2) – 1 2 (dx1 + dx2) + dx . 2 x3 3 2 a2

z = (a1/2) × (x1 – x2) – (1/4a2) × (x1 + x2)2 + a2 log x3 + a3.

Created with Print2PDF. To remove this line, buy a license at: http://www.software602.com/

3.68

Non-Linear Partial Differential Equations of Order One

Ex. 6. Find a complete integral of (p1 + x1)2 + (p2 + x2)2 + (p3 + x3)2 = 3(x1 + x2 + x3). Sol. Let the given partial differential equation be re–written as f(x1, x3, x3, p1, p2, p3) = (p1 + x1)2 + (p2 + x2)2 + (p3 + x3)2 – 3(x1 + x2 + x3) = 0. ...(1) Jacobi’s auxiliary equations are − dp1 dx1 dp3 dx3 dp2 dx2 ) ) ) = = , giving #f #x1 ∃ #f #p1 #f #x2 ∃ #f #p2 #f #x3 ∃ #f #p3

dp3 dx3 dp2 dx2 dp1 dx1 ) ) ) = = . ...(2) 2( p1 , x1) ∃ 3 ∃2( p1 , x1 ) 2( p2 , x2 ) ∃ 3 ∃ 2( p2 , x2 ) 2 ( p3 , x3 ) ∃ 3 ∃2( p3 , x3 ) dp1 , dx1 dp2 , dx 2 dp3 , dx3 ) ) ...(3) ∃3 ∃3 ∃3 Then (3) / dp1 + dx1 = dp2 + dx2 and dp3 + dx3 = dp2 + dx2 Integrating, p1 + x1 = p2 + x2 + a1 and p3 + x3 = p2 + x2 + a2, where a1 and a2 are arbitrary constants Let F1(x1, x2, x3, p1, p2, p3) = x1 + p1 – x2 – p2 = a1. ...(4) and F2(x1, x2, x2, p1, p2, p3) = x3 + p3 – x2 – p2 = a2. ...(5) As in Ex. 1, verify that for relations (4) and (5), the condition (F1, F2) = 0 is satisfied. Hence (4) and (5) may be taken as two additional equations. With help of (4) and (5), (1) reduces to (x2 + p2 + a1)2 + (x2 + p2)2 + (x2 + p2 + a2)2 = 3(x1 + x2 + x3) or 3(p2 + x2)2 + 2(p2 + x2) (a1 + a2) + a12 + a22 – 3(x1 + x2 + x3) = 0. Each fraction of (2) =

p2 , x2 ) 0 ∃2( a1 , a2 ) . [4( a1 , a2 ) 2 ∃ 12{a12 , a22 ∃ 3( x1 , x2 , x3 )} 1 6 42 53



2 2 p2 = – x2 + 02 ∃ ( a1 , a2 ) . {9( x1 , x2 , x3 ) ∃ 2 a1 ∃ 2a2 , 2a1a2 } 13 3 4 5 For sake of simplification, we take a1 = 3c1 and a2 = 3c2. Then, we get

/

p2 = – x2 – (c1 + c2) ± {( x1 , x 2 , x 3 ) ∃ 2 c12 ∃ 2c22 , 2c1c2 } .

− From (4), /

p1 = x2 + p2 + 3c1 – x1

p1 = – x1 + 2c1 – c2 ± {( x1 , x 2 , x 3 ) ∃ 2 c12 ∃ 2c22 , 2c1c2 } ,by (6)

Again, from (5), /

...(6)

p3 = x2 + p2 + 3c2 – x3

p3 = – x3 + 2c2 – c1 ± {( x1 , x 2 , x 3 ) ∃ 2 c12 ∃ 2c22 , 2c1c2 } , by (6)

Substituting these values in dz = p1dx1 + p2dx2 + p3dx3, we get dz = – (x1dx + x2dx2 + x3dx3) + [(2c1 – c2)dx1 – (c1 + c2)dx2 + (2c2 – c1)dx3]

. ( x1 , x2 , x3 ∃ 2c12 ∃ 2c22 , 2c1c2 )1/ 2 (dx1 + dx2 + dx3). 2 2 2 Integrating, z = – (1/2) × ( x1 , x 2 , x 3 ) + (2c1 – c2) x1 – (c1 + c2)x2 + (2c2 – c1)x3

± (2/3) × (x1 + x2 + x3 – 2c12 ∃ 2c22 , 2c1c2 )3/2 + c3, which is a complete integral containing c1, c2, c3 as arbitrary constants. Ex. 7. Find a complete integral of (x2 + x3) (p2 + p3)2 + zp1 = 0. [Delhi B.Sc. (Hons) III 2011] Sol. Given (x2 + x3)(p2 + p3)2 + zp1 = 0. ...(1) Since the dependent variable z is involved, the given equation (1) is not in the standard form. We shall first reduce it in the standard form and then proceed as usual. Re–writting (1), we have

Created with Print2PDF. To remove this line, buy a license at: http://www.software602.com/

Non-Linear Partial Differential Equations of Order One

(x2 + x3)

FG 1 #z , 1 #z IJ H z #x z #x K 2

3

2

3.69

, 1 #z = 0. z #x1

...(2)

Let (1/z)dz = dZ so that Z = log z. Then, (2) / (x2 + x3) (#Z/#x2 + #Z/#x3)2 + #Z/#x1 = 0. Let P1 = #Z/#x1, P2 = #Z/#x2, P3 = #Z/#x3. Then (4) becomes (x2 + x3)(P2 + P3)2 + P1 = 0. So here f(x1, x2, x3, P1, P2, P3) 6 (x2 + x3)(P2 + P.3)2 + P1 = 0. Jacobi’s auxiliary equations take the form

...(3) ...(4)

...(5)

dP1 dx1 dP2 dx2 dP3 dx3 = = ) ) ) #f #x1 ∃ #f #P1 #f #x2 ∃ #f #P2 #f #x3 ∃ #f #P3

or

dx1 dP1 dx 2 dP2 dx3 dP3 ) = = . ...(6) ) ) 2 ∃1 0 ∃2( x 2 , x 3 ) ( P2 , P3 ) ( P , P ) ∃2 ( x2 , x3 )( P2 , P3 ) ( P2 , P3 )2 2 3 Taking second ratio of (6), we have dP1 = 0 / P1 = – a1. Let F1(x1, x2, x3, P1, P2, P3) = P1 = –a1. ...(7) Taking the fourth and sixth ratios in (6), we get dP2 = dP3 / P2 – P3 = a2. Let F2(x1, x2, x3, P1, P2, P3) = P2 – P3 = a2. ...(8) Using (7), (5) / P2 + P3 = ± {a1/(x2 + x3)}1/2. ...(9) Solving (8) and (9) for P2 and P3, we have

P2 =

LM MN

FG H

a1 1 a . 2 2 x2 , x3

IJ K

1/ 2

OP PQ

P3 =

and

Using (7) and (10), dZ = P1dx1 + P2dx2 + P3dx3 becomes dZ = –a1dx1 + or

LM MN

OP PQ

LM F MN GH

a1 1 . 2 x2 , x3

LM MN

IJ K

1/ 2

OP PQ

∃ a2 . ...(10)

OP PQ

a1 a1 1 a . dx 2 , 1 . ∃ a2 dx 3 2 1 / 2 2 2 ( x 2 , x 3 )1/ 2 ( x2 , x3 )

dZ = – a1dx1 + (1/2) × a2dx2 – (1/2) × a2dx3 ± (1/ 2) 7 a1 (x2 + x3)–1/2(dx2 + dx3). Integrating and noting that dZ = (1/z)dz, complete integral is given by log z = – a1x1 + (a2/2) × (x2 – x3) ± a1 (x2 + x3)1/2 + a3.

Ex. 8. Find a complete integral of p1p2p3 = z3x1x2x3. [Meerut 1998] i.e., (#z/#x1)(#z/#x2) (#z/#x3) = z3x1x2x3. [Delhi Maths (H) 2000, 10; I.A.S. 1995] Sol. Given p1p2p3 = z3x1x2x3 or (#z/#x1)(#z/#x2) (#z/#x3) = z3x1x2x3. ...(1) Since the dependent variable z is involved, the given equation (1) is not in the standard form. We shall first reduce it in the standard form and then proceed as usual. Re–writting (1) we have

FG 1 #z IJ FG 1 #z IJ FG 1 #z IJ = x x x . H z #x K H z #x K H z #x K 1

2

3

1 2 3

...(2)

Let (1/z)dz = dZ so that log z = Z. Then (2) becomes (#Z/#x1)(#Z/#x2) (#Z/#x3) = x1x2x3 or P1P2P3 = x1x2x3. Here f(x , x , x , P , P , P ) 6 P P P – x x x = 0. ...(3) − 1 2 3 1 2 3 1 2 3 1 2 3 − Jacobi’s auxilliary equations are dP3 dx3 dP1 dx1 dP2 dx2 = = ) ) ) #f #x3 ∃ #f #P3 #f #x1 ∃ #f #P1 #f #x2 ∃ #f #P2

Created with Print2PDF. To remove this line, buy a license at: http://www.software602.com/

3.70

or

Non-Linear Partial Differential Equations of Order One

dP1 dx1 dP2 dx2 dP3 dx3 ) ) ) = = , by (3) ∃ x 2 x3 ∃ P2 P3 ∃ x1 x3 ∃ P1 P3 ∃ x1 x2 ∃ P1 P2 Since from (3), P2P3 = (x1 x2 x3)/P1, hence first and second fractions give dP1 dx1 = ∃ x 2 x3 ∃( x1 x2 x3 / P1 )

dP1 dx1 ) . P1 x1

or

Integrating, log P1 = log x1 + log a1 or P1 = a1x1. Thus, here we have F1(x1, x2, x3, P1, P2, P3) 6 P1 – a1x1 = 0. ...(4) Similarly, F2(x1, x2, x3, P1, P2, P3) 6 P2 – a2x2 = 0. ...(5) As in Ex. 1, verify that for (4) and (5) the condition (F1, F2) = 0 is satisfied. Hence (4) and (5) can be taken as two additional equations. Solving (3), (4) and (5) for P1, P2, P3, we have P1 = a1x1, P2 = a2x2 and P3 = x3/(a1a2). Putting these values in dZ = P1dx1 + P2dx2 + P3dx3, we have dZ = a1x1dx + a2x2dx2 + {x3/(a1a2)}dx3. Integrating, Z = (1/2) × a1x12 + (1/2) × a2x22 + {1/(2a1a2)}x32 + a3/2 or 2 log z = a1x12 + a2x22 + {1/(a1a2)}x32 + a3, as Z = log z Ex. 9. Find a complete integral of p12 + p2p3 – z(p2 + p3) = 0. [Delhi Maths (H) 2009] Sol. Given equation is p12 + p2p3 – z(p2 + p3) = 0. ...(1) Since the dependent variable z is involved, the given equation (1) is not in the standard form. We shall first reduce it in the standard form and then proceed as usual. Dividing each term by z2, (1) can be re–written as

FG 1 #z IJ , FG 1 #z IJ FG 1 #z IJ ∃ FG 1 #z IJ ∃ FG 1 #z IJ H z #x K H z #x K H z #x K H z #x K H z #x K 2

1

2

3

2

3

= 0.

Let (1/z)dz = dZ so that log z = Z. 2 Using (3), (2) becomes P1 + P2P3 – P2 – P3 = 0, Let us write f(x1, x2, x3, P1, P2, P3) = P12 + P2P3 – P2 – P3 = 0. Jacobi’s auxiliary equations are −

...(2) ...(3) ...(4) ...(5)

dP3 dx3 dP1 dx1 dP2 dx2 ) = = ) ) # f # x ∃ # f #P3 #f #x1 ∃ #f #P1 #f #x2 ∃ #f #P2 3 dP1 dx1 dP dx2 dP dx3 ) = 2) = 3) , by (5) 0 ∃2 P1 0 ∃ P3 , 1 0 ∃ P2 , 1 Taking the third and fifth fractions, dP2 = 0 and dP3 = 0 so that P2 = a1 and P3 = a2. Let F1(x1, x2, x3, P1, P2, P3) = P2 = a1. ...(6) and F3(x1, x2, x3, P1, P2, P3) = P3 = a2. ...(7) As in Ex. 1, verify that for (6) and (7), the condition (F1, F2) = 0 is satisfied. Hence (6) and (7) can be taken as two additional equations. Solving (4), (6) and (7) for P1, P2, P3, we have P2 = a1, P3 = a2, P1 = (a1 + a2 – a1a2)1/2. Putting these values in dZ = P1dx1 + P2dx2 + P3dx3, we have dZ = (a1 + a2 – a1a2)1/2dx1 + a1dx2 + a2dx3. Integrating, Z = (a1 + a2 – a1a2)1/2x1 + a1x2 + a2x3 + a3. Then, the complete integral is log z = (a1 + a2 – a1a2)1/2x1 + a1x2 + a2x3 + a3, using (3). Ex. 10. Find a complete integral of 2x1x3zp1p3 + x2 p2 = 0. Sol. Given equation is 2x1x3zp1p3 + x2 p2 = 0. ...(1) or

Created with Print2PDF. To remove this line, buy a license at: http://www.software602.com/

Non-Linear Partial Differential Equations of Order One

3.71

Since the dependent variable z is involved, the given equation (1) is not in the standard form. We shall first reduce it in the standard form and then proceed as usual. Multiplying each term by z, (1) can be re–written as

FG H

#z 2x1x3 z #x 1

IJ FG z #z IJ , x FG z #z IJ = 0. K H #x K H #x K 3

2

...(2)

2

Let zdz = dZ so that z2/2 = Z. Using (3), (2) becomes 2x1x3P1P3 + x2P2 = 0, where P1 = #Z/#x1, P2 = #Z/#x2 and P3 = #Z/#x3. We re–write (4) as f(x1, x2, x3, P1, P2, P3) = 2x1x3P1P3 + x2P2 = 0. − Jacobi’s auxilairy equations are

...(3) ...(4) ...(5)

dP3 dx3 dP1 dx1 dP2 dx2 ) = = ) ) #f #x3 ∃ #f #P3 #f #x1 ∃ #f #P1 #f #x2 ∃ #f #P2 dP1 dx1 dP dx dP3 dx3 ) = 2) 2 = , by (5) ) 2 x3 P1 P3 ∃2 x1 x3 P3 P2 ∃ x2 2 x1 P1 P3 ∃2 x1 x 3 P2

or

...(6)

Taking the first and second fractions of (6) and simplifying, we get (1/P1)dP1 + (1/x1)dx1 = 0 so that log P1 + log x1 = log a1 or P1x1 = a1 So here F1(x1, x2, x3, P1, P2, P3) = P1x1 = a1. ...(7) Taking the fifth and sixth fractions of (6) and simplifying, we get (1/P3)dP3 + (1/x3)dx3 = 0 so that log P3 + log x3 = log a3 or P3x3 = a2 So here F2(x1, x2, x3, P1, P2, P3) = P3x3 = a2. ...(8) As in Ex. 1, verify that for (7) and (8), the condition (F1, F2) = 0 is satisfied. Hence (7) and (8) can be taken as additional equations. Solving (5), (7) and (8) for P1, P2, P3, we have P1 = a1/x, P3 = a2/x3, P2 = – (2a, a2)/x2. Putting these values in dZ = P1dx + P2dx2 + P3dx3, we have dZ = (a1/x1)dx1 – {(2a1a2)/x2}dx2 + (a2/x3)dx3. Integrating, Z = a1log x1 – 2a1a2 log x2 + a2 log x3 + a3 2 or z /2 = a1 log x1 – 2a1a2 log x2 + a2 log x3 + a3, by (3). Ex. 11. Find a complete integral of p1p2p3 + p43x1x2x3x43 = 0. Sol. [In the present problem we have four independent variables in places of three. According we shall use modified working as explained in remark 2 of Art 3.20] The given equation can be written as f(x1, x2, x3, x4, p1, p2, p3, p4) = p1p2p3 + p43x1x2x3x43 = 0. ...(1) Jacobi’s auxiliary equations are −

dp1 dx1 dp2 dx2 dp3 dx3 dp4 dx 4 ) = ) = = , giving ) ) #f #x1 ∃ #f #p1 #f #x2 ∃ #f #p2 #f #x3 ∃ #f #p3 #f #x4 ∃ #f #p4 dp1 p43 x2 x3 x43

)

dp dx 2 dp4 dx 4 dp dx3 dx1 = 3 2 3) = 3 3 3) = ) 3 2 3 ∃ p2 p3 p 4 x1 x 3 x 4 ∃ p1 p3 p4 x1 x 2 x 4 ∃ p1 p2 3 p4 x1 x 2 x 3 x 4 ∃3 p4 x1 x 2 x 3 x 43

Since from (1), p43x2x3x43 = –p1p2p3/x1, the first two fractions give dp1 dx1 = ∃( p1 p2 p3 / x1 ) ∃ p2 p3

or

dp1 dx1 ) . p1 x1

Created with Print2PDF. To remove this line, buy a license at: http://www.software602.com/

3.72

Non-Linear Partial Differential Equations of Order One

log p1 = log x1 + log a1 or p1 = a1x1. F1(x1, x2, x3, x4 p1, p2, p3, p4) = p1 – a1x1 = 0. F2(x1, x2, x3, x4, p1, p2, p3, p4) = p2 – a2x2 = 0 and F1(x1, x2, x3, x4, p1, p2, p3, p4) = p3 – a3x3 = 0. With these values of F1, F2 and F3, we can verify that Integrating, Let Similarly,

4

(F1, F2) =

F #F #F ∃ #F

∗ GH #x r )1

1

#F2 # pr # x r

2

1

# pr

r

...(2) ...(3) ...(4)

IJ = 0. K

Similarly, we see that (F2, F3) = 0 and (F3, F1) = 0. Hence (2), (3) and (4) can be taken as the three desired additional equations. Now solving (1), (2) (3) and (4) for p1, p2, p3 and p4, we get p1 = a1x1,

p2 = a2x2,

p3 = a3x3

p4 ) (a1a2 a3 )1/ 3 x4 .

and

Putting these in dz = p1dx1 + p2dx2 + p3dx3 + p4dx4 and integrating the desired complete integral is z = (1/2) × (a1x12 + a2x22 + a3x32) – (a1a2a3)1/2 log x4 + a4/2 or 2z = a1x12 + a2x22 + a3x32 – 2(a1a2a3)1/2 log x4 + a4, Ex. 12. Find a complete integral by Jacobi’s method of the equation 2 x 2 y(#u / #x)2 (#u / #z)

) x 2 (#u / #y) , 2 y(#u / #x)2 . Sol. Let

x ) x1 ,

[Delhi Maths (H) 2001] y ) x2 ,

z ) x3 ,

#u / #x ) p1 ,

#u / #y ) p2 ,

and #u / #z ) p3

2 x12 x2 p12 p3 ) x12 p2 , 2 x2 p12

Then given equation becomes

Dividing by x12 x2 , 2 p12 p3 ) ( p2 / x2 ) , (2 p12 / x12 ) , which can be written as

f ( x1, x2 , x3 , p1, p2 , p3 ) ) 2 p12 ( p3 ∃ 1/ x12 ) ∃ p2 / x2 ) 0 − Jacobi’s auxiliary equations are dp3 dx3 dp1 dx1 dp2 dx2 ) ) ) ) ) #f / #x1 ∃#f / #p1 #f / #x2 ∃#f / #p2 #f / #x3 ∃#f / #p3 dp1

or

4 p12

/ x13

)

dx1 ∃4 p1 ( p3 ∃ 1/ x12 )

Taking the fifth fraction,

)

dp2 p2 / x22

dp3 ) 0



log p2 ) log x2 , log(2a22 )

Here

and

dx2 dp dx3 , by (1) ) 3 ) 1/ x2 0 ∃2 p12

p3 ) a1

so that (1/ p2 ) dp2 ) (1/ x2 ) dx2

Taking the second and fourth fractions, Integrating,

)

... (1)

or

p2 / x2 ) 2a22

F1 ( x1 , x2 , x3 , p1, p2 , p3 ) ) p3 ) a1

... (2)

F2 ( x1, x2 , x3 , p1 , p2 , p3 ) ) p2 / x2 ) 2a22

... (3)

3

( F1 , F2 ) )

Now,

or

(F1 , F2 ) )

! #F1 #F2 #F1 #F2 ∃ & #pr #xr ( r #pr r )1

∗ %∋ #x

#F1 #F2 #F1 #F2 #F1 #F2 #F1 #F2 #F1 #F2 #F1 #F2 ∃ , ∃ , ∃ #x1 #p1 #p1 #x1 #x2 #p2 #p2 #x2 #x3 #p3 #p3 #x3

= (0) (0) ∃ (0) (0) , (0) (0) ∃ (0) (0) , (0) (0) ∃ (1) (0) ) 0 Hence (2) and (3) may be taken as additional equations.

Created with Print2PDF. To remove this line, buy a license at: http://www.software602.com/

Non-Linear Partial Differential Equations of Order One

3.73

p1 ) a2 x1 /(a1x12 ∃ 1)1/ 2 ,

Solving (1), (2) and (3) for p1, p2, p3,

p2 ) 2a22 x2 ,

p3 ) a1

Putting these in du ) p1dx1 , p2 dx2 , p3 dx3 ) a2 x1 (a1x12 ∃ 1)∃1/ 2 dx1 , 2a22 x2 dx2 , a1dx3 .

u ) (a2 / a1 ) 7 (a1x12 ∃ 1)1/ 2 , a22 x22 , a1x3 , a3 ,

Integrating,

which is the complete integral with a1 , a2 , a3 as arbitrary constants. Ex. 13. Show that a complete integral of the equation f (#u / #x, #u / #y , #u / #z ) ) 0 is u ) ax , by , 8(a, b) z , c, where a, b and c are arbitrary constants and f (a, b, 8) ) 0 (b)Find

a

complete

integral

) (#u / #x) (#u / #y ) (#u / #z ).

the

equation

#u / #x , #u / #y , #u / #z

[Allahabad 2004, 06; Meerut 2004, 06; Purvanchal 2003] #u / #x ) p1 ,

Sol. (a) Let

of

#u / #y ) p2

and

#u / #z ) p3 .

f ( p1 , p2 , p3 ) ) 0

Then gievn equation becomes

... (1)

We shall now proceed as in Ex. 1, Art. 3.21. Here Jacobi’s auxiliary equations are given by dp3 dp1 dx dp2 dy dz ) ) ) ) ) #f / #x ∃#f / #p1 #f / #y ∃#f / #p2 #f / #z ∃#f / #p3 dp1 dp2 ) , using (1) dp1 = 0 and / 0 0 Integrating, p1 = a, p2 = b, a and b being arbitrary constants Putting p1 = a and p2 = b in (1), f(a, b, p3) = 0 so that

/

p3 = a function of a, b ) 8(a, b), say

dp2 = 0 ... (2) ... (3)

du ) (#u / #x ) dx , (#u / #y )dy , (#u / #z )dz ) p1dx , p2 dy , p3 dz

Now, we have

du ) a dx , b dy , 8(a, b) dz, by (2) and (3)

or

u ) ax , by , 8 (a , b) z , c,

Integrating,

... (4)

where c is an arbitrary constant and a, b, 8 are connected by relation f (a, b, 8 (a, b)) ) 0, by (1), (2) and (3) (b) Given Let

#u / #x , #u / #y , #u / #z ∃ (#u / #x) (#u / #y ) (#u / #z ) ) 0 p1 ) #u / #x,

p2 ) #u / #y

and

From (v),

... (i)

p3 ) #u / #z. Then, (i) gives

p1 + p2 + p3 – p1 p2 p3 = 0 Comparing (ii) with (1) of part (a), here f(p1, p2, p3) = p1 + p2 + p3 – p1 p2 p3 Hence required complete integral is given by (4) and (5) of part (a) i.e.,

where

... (5)

... (ii) ... (iii)

u ) a x , b y , 8 (a, b) z , c,

... (iv)

a , b , 8 (a , b) ∃ ab 8(a, b) ) 0

... (v)

8(a, b) ) (a , b) /(ab ∃ 1)

... (vi)

From (iv) and (vi), u = ax + by + {(a + b)/(ab – 1)} + c, which is the required complete integral of (i), a, b, c being arbitrary constants.

Created with Print2PDF. To remove this line, buy a license at: http://www.software602.com/

3.74

Non-Linear Partial Differential Equations of Order One

EXERCISE 3(G) Find the complete integral of the following equation: (1 – 5) 1. f ( p1 , p2 , p3 ) ) 0

Ans. z ) a1 x1 , a2 x2 , a3 x3 , a4 , where f (a1 , a2 , a3 ) ) 0

2. p1 , p2 , p3 ∃ p1 p2 p3 ) 0 Ans. z ) a1 x1 , a2 x2 , a3 x3 , a4 , where a1 , a2 , a3 ∃ a1a2 a3 ) 0 Ans. z ) ∃(a12 , a a22 ) x1∃1 , a1x2 , a2 a3 x3 , a3

3. p1x12 ∃ p22 ∃ ap32 ) 0 4. x3 ( x3 , p3 ) ) p12 , p22

Ans. z ) a1x1 , a2 x2 , (a12 , a22 ) log x3 ∃ x32 / 2 , a3

5. x3 , 2 p3 ∃ ( p1 , p32 ) ) 0

Ans. z ) a1x1 , a2 x2 , (a1 , a2 )2 7 ( x3 / 2) ∃ ( x32 / 4) , a3

6. x1 , p12 , x2 , p22 ∃ x3 p32 ) 0

Ans. z ) 2 (a1x1 )1/2 , 2 (a2 x2 )1/2 , 2 {(a1 , a2 ) x3}1/2 , a3

7. Show how to solve, by Jacobi method, a partial differential equation of the type f ( x, #u / #x, #u / #z ) ) g ( y , #u / #y, #u / #z ) and illustrate the method by finding a complete integral of equation 2 x 2 y(#u / #x)2 (#u / #z ) ) x2 (#u / #y) , 2 y(#u / #x)2 .

[Meerut 2005]

Ans. u = (ax2 – b)1/2 + ay2 + (z/b) + c

Sol. Try yourself

8. Prove that an equation of the ‘‘Clairaut’’ form

x(#u / #x) , y (#u / #y ) , z (#u / #z )

) f (#u / #x, #u / #y , #u / #z ) is always solvable by Jacobi’s method. Hence solve (#u / #x , #u / #y , #u / #z ) {x (#u / #x) , y (#u / #y ) , z (#u / #z )} ) 1 3.22. Jacobi’s method for solving a non-linear first order partial differential equation in two independent variables. [Delhi Maths (H) 1997; Amaravati 2001; Himanchal 2003, 05]

F( x , y, z, p, q) ) 0

Let

... (1)

be the non-linear first order equation in two independent variables x, y. Then we know that a solution of (1) is of the form

u( x, y, z) ) 0

... (2)

showing that u can be treated as a dependent variable and x, y, z as three independent variables. Differentiating (2) partially w.r.t. ‘x’ and ‘y’, respectively, we get

or

#u #u #z , )0 #x #z #x

and

#u #u #z , )0 #y #z #y

p1 , p3 p ) 0

and

p2 , p3 q ) 0

... (3)

where p ) #z / #x, q ) #z / #y, p1 ) #u / #x ) #u / #x1 , p2 ) #u / #y ) #u / #x2 , p3 ) #u / #z ) #u / #x3 by taking From (3),

x ) x1 ,

y ) y2 p ) ∃ ( p1 / p3 )

and and

z ) x3

... (4)

q ) ∃ ( p2 / p3 )

... (5)

Using (4) and (5), (1) reduces to f ( x1 , x2 , x3 , p1 , p2 , p3 ) ) 0 ... (6) We now solve (6) by Jacobi’s method as usual (refer Art. 3.20) to get the complete integral of (6). Finally, putting x1 ) x, x2 ) y, x3 ) z , we obtain solution of (6) containing original variables x, y, z and new dependent variable u. The solution so obtained will contain three arbitrary constants a1 , a2 , a3 (say). However, for the given equation in the form (1), we need only two arbitrary constants in the final solution. The required solution u = 0 of (1) is obtained by making different choices of our third arbitrary constant.

Created with Print2PDF. To remove this line, buy a license at: http://www.software602.com/

Non-Linear Partial Differential Equations of Order One

3.75

Ex. 1. Solve p2 x , q2 y ) z by Jacobi’s method.

[Nagpur 2002; Himanchal 2003, 05]

Sol. Given

p2 x , q2 y ) z .

Let a solution of (1) be of the form

u( x, y, z) ) 0

... (2)

So treating u as dependent variable and x, y, z as three independent variables, differentiation of (2) partially w.r.t ‘x’ and ‘y’ respectively gives #u #u #z , )0 #x #z #x

so that

#u #u #z , )0 #y #z #y

and

p ) ∃ p1 / p3

p1 , p3 p ) 0

i.e.

and

p2 + p3 q = 0

q ) ∃ p2 / p3

and

.... (3)

where p1 ) #u / #x ) #u / #x1 , p2 ) #u / #y ) #u / #x2 , p3 ) #u / #z ) #u / #x3 , p ) #z / #x, q ) #z / #y by taking

x ) x1 ,

y ) x2

Using (3) and (4), (1) / Let

z ) x3

and

x1 ( p1 / p3 )2 , x2 ( p2 / p3 )2 ) x3 .

/

... (4)

x1 p12 , x2 p22 ∃ x3 p32 ) 0

f ( x1 , x2 , x3 , p1, p2 , p3 ) ) x1 p12 , x2 p22 ∃ x3 p32 ) 0

... (5)

Now, the Jacobi’s auxiliary equations are dp3 dx3 dp1 dx1 dp2 dx2 ) ) ) ) ) #f / #x1 ∃#f / #p1 #f / #x2 ∃#f / #p2 #f / #x3 ∃#f / #p3 dp1

or

p12

)

dx1 dp dx2 dp dx3 ) 22 ) ) 32 ) ∃2 p1 x1 ∃ 2 p x 2 p3 x3 , by (5) p2 ∃ p3 2 2

(2 / p1 ) dp1 , (1/ x ) dx ) 0 .

Taking the first two fractions, Integrating,

2 log p1 , log x1 ) log a1

so that

x1 p12 = a1

or

p1 ) (a1 / x1 )1/ 2

Similarly, the third and fourth fractions give

p2 ) (a2 / x2 )1/ 2

Substituting these values of p1 and p2 in (5), we get

p3 ) {(a1 , a2 ) / x3}1/ 2 .

Putting the above values of p1, p2 and p3 in du ) p1dx1 , p2 dx2 , p3 dx3 , we get

du ) a11 / 2 x1∃1/ 2 dx1 , a2 x2∃1/ 2 dx2 , (a1 , a2 )1 / 2 x3∃(1/ 2) dx3 . Integrating,

2 u ) 2(a1x1 )1/ 2 , (a2 x2 )1/ 2 , 2(a1 , a2 )1/ 2 x1/ 3 , a3

... (6)

Taking a2 ) 1 and using (4), the required solution u ) 0 is given by

2(a1x)1/ 2 , 2 y1/ 2 , 2(a1 , 1)1/ 2 z1/ 2 , a3 ) 0 , which is the complete integral containing two arbitrary constants a1 and a3. Ex. 2. Solve p2 + q2 = k2 by Jacobi’s method [Delhi B.A./B.Sc. (Prog) Maths 2007] Sol. Given p2 + q2 = k2 ... (1) Let a solution of (1) be of the form u(x, y, z) = 0 ... (2) So treating u as dependent variable and x, y, z as three independent variable, differentiation of (2) partially w.r.t. ‘x’ and ‘y’ respectively gives

Created with Print2PDF. To remove this line, buy a license at: http://www.software602.com/

3.76

Non-Linear Partial Differential Equations of Order One

#u #u #z , )0 #x #z #x

and

#u #u #z , )0 #y #z #y

p = – (p1/p3)

so that

i.e.,

p1 + p3 p = 0

and p2 + p3q = 0

q = – (p2/p3)

and

... (3)

where p1 ) #u / #x ) #u / #x1, p2 ) #u / #y ) #u / #x2 , p3 ) #u / #z ) #u / #x3 , p ) #z / #x, q ) #z / #y by taking

x = x1,

y = x2

Using (3) and (4), (1) reduces to

and

z = x3

p12 / p32 , p22 / p32 ) k 2

or

... (4)

p12 , p22 ) k 2 p32

f ( x1 , x2 , x3 , p1 , p2 , p3 ) ) p12 , p22 – k 2 p32 ) 0

Let

... (5)

Now, the Jacobi auxilliary equations are given by dp1 dx1 dp2 dx2 dp3 dx3 ) ) ) ) ) #f / #x1 ∃#f / #p1 #f / #x2 ∃#f / #p2 #f / #x3 ∃#f / #p3 dp dx dp1 dx1 dp dx2 ) ) 2 ) ) 3 ) 2 3 , using (5) 0 ∃2 p1 0 ∃2 p2 0 2k p3

or

From the first and third fractions of (5), dp1 = 0 and Integrating, p1 = a1 and p2 = a2, a1 and a2 being arbitrary constants With

p1 = a1

and

p2 = a2,

dp2 = 0

(5) gives p3 ) ( a12 , a22 )1/ 2 / k

Putting the above values of p1, p2 and p3 in du = p1dx1 + p2dx2 + p3dx3, we get

du ) a1dx1 , a2 dx2 , {(a12 , a22 )1/ 2 / k}dx3 Integrating,

u ) a1 x , a2 x2 , {(a12 , a22 )1/ 2 / k}x3 , a3

... (6)

Taking a2 = 1 and using (4), the required solution u = 0 is given by

a1 x , x2 , {(a12 , 1)1/ 2 / k}x3 , a3 ) 0, which is the complete integral of (1) containing two arbitrary constants a1 and a3. Ex. 3. Solve the following partial differential equations by Jacobi’s method: 2 (i) p ) (z , qy)

(ii) ( p2 , q2 ) x ) pz (iii) xpq + yq2 = 1 [Nagpur 2005] Hint. Proceed as in the above solved Ex. 1 3.23 Cauchy’s method of characteristics for solving non-linear partial differential equation i.e., ... (1) f ( x, y, z, #z / #x, #z / #y) ) 0 f ( x, y, z, p, q) ) 0 We know that the plane passing through the point P( x0 , y0 z0 ) with its normal parallel to the direction n whose direction ratios are p0 , q0 , ∃ 1 is uniquely given by the set of five numbers D( x0 , y0 , z0 , p0 , q0 ) and conversely any such set of five numbers defines a plane in three dimensional space. In view of this fact a set of five numbers D( x, y, z, p, q) is known as a plane element of a three dimensional space. As a special case a plane element ( x0 , y0 , z0 , p0 , q0 ) whose components satisfy (1) is known as an integral element of (1) at P. Solving (1) for q, suppose we get

Created with Print2PDF. To remove this line, buy a license at: http://www.software602.com/

Non-Linear Partial Differential Equations of Order One

3.77

q ) F( x, y, z, p) . which gives a value of q corresponding to known values of x, y, z and p. Then, keeping x0, y0 and z0 fixed and varying p, we shall arrive at a set of plane elements {x0 , y0 , z0 , p , G( x 0 , y0 , z0 , p) } which depend on the single parameter p. As p varies, we get a set of plane elements all of which pass through the point P. Hence the above mentioned set of plane elements envelop a cone with vertex P. The cone thus obtained is known as the elementary cone of (1) at the point P.

z ) g( x, y)

Consider a surface S with equation If the function g( x, y) and its first partial derivatives gx ( x, y) and gy ( x, y) are continuous in a certain region R of the xy-plane, then the tangent plane at each point of S determines a plane element of the form {x0, y0, g(x0, y0), gx ( x0 , y0 ) , g y ( x 0 , y 0 ) } which will be referred as the tangent element of the surface S at the point {x0 , y0 , g( x0 , y0 )] . Consider a curve C with parametric equations x ) x (t ) ,

y ) y(t ) ,

... (2) P0, q0, – 1

P0

z ) z( t ) ,

S P1

t being the parameter.

z(t ) ) g {x(t ), y(t )}

Then curve C lies on (2) provided

C

... (3) ... (4)

holds good for all values of t in the appropriate interval I. Let P0 be a point on curve C corresponding to t ) t 0 . Now, the direction ratios of the tangent line P0 P1 are x +(t0 ), y+(t 0 ) , z+(t0 ) where x +(t0 ), y+(t0 ) z+(t0 ) denote the values of dx / dt, dy / dt, dz / dt respectively at t ) t 0 This direction will be perpendicular to direction of normal n (with direction ratios p0 , q0 , ∃ 1 ) p0 x +(t0 ) , q0 y+(t0 ) , (∃1) z+(t0 ) ) 0

if

It follows that any set

z+(t0 ) ) p0 x +(t 0 ) , q0 y+(t0 )

or

{x(t), y(t), z(t ), p(t ), q(t )}

...(5)

of five real functions satisfying the condition that z+(t ) ) p(t ) x +(t ) , q(t ) y+(t ) ... (6) defines a strip at the point (x, y, z) of the curve C. When such a strip is also an integral element of (1), then the strip under consideration is known as an integral strip of (1). In other words, the set of functions (5) is known as an integral strip of (1) provided these satisty (6) and the following additional condition

f {x(t), y(t), z(t ), p(t ), q(t )} = 0,

for all t in I.

If at each point of the curve (3) touches a generator of the elementary cone, then the corresponding strip is known as a characteristic strip. Derivation of the equations determining a characteristic strip Clearly, the point ( x , dx, y , dy, z , dz) lies in the tangent plane to the elementary cone at P if

dz ) pdx , q dy

... (7)

where p, q satisfy (1). Differentiation (7) w.r.t. ‘p’, we get

0 ) dx , (dq / dp) dy

... (8)

Again, differentiating (1) partially w.r.t. ‘p’, we have

#f / #p , (#f / #q) (dq / dp) ) 0 Here,

#f / #p ) f p

f p , fq (dq / dp) ) 0

i.e., and

... (9)

#f / #q ) f q

Created with Print2PDF. To remove this line, buy a license at: http://www.software602.com/

3.78

Non-Linear Partial Differential Equations of Order One

Solving (7), (8) and (9) for the ratios of dy, dz to dx, we get dx dy dz ) ) fp fq p f p , q fq .

... (10)

Hence along a characteristic strip x +(t ), y+(t ) , z +(t ) will be proportional to f p , fq , p f p , q fq respectively. If the parameter t be selected satisfying the relations

x+(t) ) f p

y+(t) ) fq .

and

z +(t ) ) p f p , q fq

then, we have

Since along a characteristic strip p is a function of t, hence p+(t ) ) (#p / #x ) (dx / dt ) , (#p / #y) (dy / dt ) = (#p / #x ) (#f / #p) , (#p / #y) (#f / #q) , using (11) 1) p+(t ) ) (#p / #x ) (#f / #p) , (#q / #x ) (#f / #q)

Thus,

0 2 24 Now, differentiating (1) partially w.r.t. ‘x’, gives

... (12)

#p # ! #z # ! #z #q 1 ) ) % &) 3 % & #y #y ∋ #x ( #x ∋ #y ( #x 35 #f #f #f #p #f #q , p, , )0 #x #z #p #x #q #x

f x , p fz , p+( t ) ) 0 , using (12)

or

Hence on a characteristic strip,

p+(t ) ) ∃ fx ∃ p fz

... (13)

Similarly, we have

q+(t ) ) ∃ fy ∃ q fz

... (14)

Here

f x ) #f / #x,

f y ) #f / #y,

f z ) #f / #z

From (11), (13) and (14), we get the following system of five ordinary differential equations for the determination of the characteristic strip

x+(t) ) f p ,

y+(t ) ) fq ,

z+(t) ) p f p , q fq ,

p+(t ) ) ∃ fx ∃ p fz

and

q+(t ) ) ∃ fy ∃ q fz ...(15)

The above equations are called the characteristic equations of (1). In view of a well known result if the functions which are involved in (15) satisfy a Lipschitz condition, there exists a unique solution of (15) for given set of initial values of the variables. It follows that the characteristic strip is determined uniquely by any initial element ( x0 , y0 , z0 , p0 , q0 ) and any initial value to of t. Working rule for solving Cauchy’s problem. [Meerut 2005] Suppose we wish to find the integral surface of (1) which passes through a given curve with parametric equation

x ) f1 (9) , y ) f2 (9) ,

then in the solution

z ) f3 (9 ),

9 being the parameter ... (16)

x ) x ( p0 , q0 , x0 , y0 , t0 , t ) etc.

... (17)

of the characteristic equations (15), we shall assume that x0 ) f1 (9 ) ,

y0 ) f2 (9) ,

z0 ) f3 (9)

are the initial values of x, y, z respectively. Then the corresponding initial values of p0, q0 can be obtained by the following relations f3+ (9) ) p0 f1+ (9) , q0 f3+ (9 )

and

f { f1 (9), f2 (9), f3 (9), p0 , q0 } ) 0

Created with Print2PDF. To remove this line, buy a license at: http://www.software602.com/

Non-Linear Partial Differential Equations of Order One

3.79

When the above values of x0 , y0 , z0 , p0 , q0 and the appropriate value of t0 is substituted in (17), we shall be able to express x, y, z involving the two parameters t and 9 of the form x ) :1 (t, 9),

y ) : 2 (t , 9 )

z ) : 3 (t , 9 )

and

... (18)

which are known as characteristics of (1) Finally, by eliminating 9 and t from (18), we arrive at a relation of the form G ( x, y, z) = 0, which is the required equation of the integral surface of (1) passing through the given curve (16). 3.24 Some Theorems: Theorems 1. A necessary and sufficient condition that a surface be an integral surface of a partial differential equation is that at each point its tangent element should touch the elementary cone of the equation. Proof. Using geometrical considerations and Art 3.23, complete the proof yourself. Theorem II. Along every characteristic strip of the partial differential equation f (x, y, z, p, q) = 0 the function f (x, y, z, p, q) is a constant. Proof. Along a characteristic strip, we have d f {x(t ), y(t ), z(t ), p(t ), q(t )} = fx x +(t) , fy y +(t ) , fz z +(t ) , f p p+(t) , fq q +(t ) dt

= fx f p , fy fq , fz ( p f p , q fq ) ∃ f p ( fx , p fz ) ∃ fq ( fy , q fz ) = 0, using the characteristic equation (15) of Art. 3.23 showing that f (x, y, z, p, q) = K, a constant along the strip. Corollary to theorem II. If a characteristic strip contains at least one integral element of f ( x, y, z, p, q) ) 0 it is an integral strip of the equation f ( x, y, z, #z / #x, #z / #y) ) 0 Proof. Left as an exercise. 3.25 SOLVED EXAMPLES BASED ON ART. 3.23 Ex. 1. Find the characteristics of the equation pq ) z , and determine the integral surface which passes through the parabola x = 0, y2 = z. [Meerut 2005; I.A.S. 1999] Sol. Given equation is pq = z ... (1) We are to find its integral surface which passes through the given parabola given by x = 0, and y2 = z ... (2) Re-writing (2) in parametric form, we have x = 0,

y = 9,

z ) 92 ,

9 being a parameter

... (3)

Let the initial values x0 , y0 , z0 , p0 , q0 of x, y, z, p, q be taken as x0 ) x0 ( 9 ) ) 0 ,

z0 ) z0 (9) ) 9 2

y0 ) y0 (9) ) 9,

... (4A)

Let p0, q0 be the initial values of p, q corresponding to the initial values x0, y0, z0. Since initial values ( x0 , y0 , z0 , p, q0 ) satisfy (1), we have p0 q0 ) z0 ,

29 ) p0 7 0 , q0 7 1

Solving (5) and (6),

... (5)

z0+ (9) ) p0 x 0+ (9 ) , q0 y0+ (9 )

Also, we have so that

p0 q0 ) 9 2 , by (4A)

or

p0 ) 9 / 2

or

q0 ) 29, by (4A)

... (6)

q0 ) 29

... (4B)

and

Created with Print2PDF. To remove this line, buy a license at: http://www.software602.com/

3.80

Non-Linear Partial Differential Equations of Order One

Collecting relations (4A) and (4B) together, initial values of x0 , y0 , z0 , p0 , q0 are given by x0 ) 0 ,

z0 ) 9 2 ,

y0 ) 9,

p0 ) 9 / 2 ,

q0 ) 29

t ) t0 ) 0

when

... (7)

f ( x, y, z, p, q) ) pq ∃ z ) 0

Re-writing (1), let

... (8)

The usual characteristic equations of (8) are given by

and

dx / dt ) #f / #p ) q

... (9)

dy / dt ) #f / #q ) p

... (10)

dz / dt ) p(#f / #p) , q(#f / #q) ) 2 pq

... (11)

dp / dt ) ∃(#f / #x) ∃ p(#f / #z) ) p

... (12)

dq / dt ) ∃(#f / #y) ∃ q(#f / #z) ) q

... (13)

(dx / dt ) ∃ (dq / dt ) ) 0 ,

From (9) and (13),

x ∃ q ) C1 ,

so that

.... (14)

where C1 is an arbitrary constant. Using initial values (7), (14) gives x0 ∃ q0 ) C1

0 ∃ 29 ) C1

or

x ∃ q ) ∃29

C1 ) ∃29 , Then (14) becomes

or

x ) q ∃ 29 ,

or

(dy / dt) ∃ (dp / dt ) ) 0

From (10) and (12),

... (15)

y ∃ p ) C2 ,

so that

... (16)

where C2 is an arbitrary constant. Using initial values (7), (16) gives y0 ∃ p0 ) C2

9 ∃ (9 / 2) ) C2

or

y∃ p ) 9/2 From (12),

C2 ) 9 / 2 . Then (16) becomes

or

or

(1/ p) dp ) dt

so that

Using initial values (7), (18) gives

log p ∃ log C3 ) t

or

p0 ) C3e0

or

so that

Using initial values (7), (20) gives

log q ∃ log C4 ) t

q0 ) C4 e0

p ) C3 et

... (18) 9 / 2 ) C3 ... (19)

q ) C4 et ... (20)

or

or 29 ) C4

q ) 29 et

Hence (20) reduces to Using (21), (15) becomes

... (17)

p ) (9 / 2) 7 et

Hence (18) reduces to From (13), (1/q) dq ) dt

y ) p , (9 / 2)

x ) 29 et ∃ 29

Using (19), (17) becomes y ) (9 / 2) et , 9 / 2

... (21)

or

x ) 29 (et ∃ 1)

... (22)

or

y ) (9 / 2) 7 (et , 1)

... (23)

Substituting values of p and q from (19) and (21) in (11), we get

dz / dt ) 2{(9 / 2) 7 et } 7 {29 et } Integrating,

dz ) 29 2 e2t dt .

or

z ) 92 e2t , C5 , C5 being arbitrary constant

Using initial values (7), (24) gives z0 ) 92 e0 , C5

or

9 2 ) 92 , C5

.... (24) or

C5 ) 0

Created with Print2PDF. To remove this line, buy a license at: http://www.software602.com/

Non-Linear Partial Differential Equations of Order One

3.81

z ) 9 2 e2t

Then, (24) gives

z ) 9 2 ( et )2

or

... (25)

The required characteristics of (1)are given by (22), (23) and (25) To find the required integral surface of (1), we now proceed to eliminate two parameters t and t 9 from three equations (22), (23)and (25). Solving (22) and (23) for e and 9 , we have

et ) ( x , 4 y) /(4 y ∃ x)

9 ) (4 y ∃ x ) / 4

and t

Substituting these values of e and 9 in (25), we have

z ) {(4 y ∃ x)2 /16}7 {( x , 4 y) /(4y ∃ x)}2

16z ) (4 y , x)2 ,

or

which is the required integral surface of (1) passing through (2). Ex. 2. Find the solution of the equation z ) ( p2 , q2 ) / 2 + ( p ∃ x) (q ∃ y) which passes through the x-axis. [Himachal 1996; 2004; I.A.S. 2002]

z ) ( p2 , q2 ) / 2 , ( p ∃ x)(q ∃ y)

Sol. Given equation is

... (1)

We are to find its integral surface which passes through x-axis which is given by equations y=0 and z=0 ... (2) Re-writing (2) in parametric form, x ) 9,

y ) 0,

z ) 0, 9 being the parameter

... (3)

Let the initial values x0 , y0 , z0 , p0 , q0 of x, y, z, p, q be taken as x0 ) x0 ( 9 ) ) 9 ,

y0 ) y0 (9) ) 0 ,

z ) z0 (9) ) 0

... (4A)

Let p0 , q0 be the initial values of p, q corresponding to the initial values x0, y0, z0. Since initial values ( x0 , y0 , z0 , p0 , q0 ) satisfy (1), we have

z0 ) ( p02 , q02 ) / 2 , ( p0 ∃ x0 ) (q0 ∃ x0 )

0 ) ( p02 , q02 ) / 2 , q0 ( p0 ∃ 9), by (4A)

or

p02 , q02 , 2q0 p0 ∃ 2q0 9 ) 0

or

z0+ (9) ) p0 x0+ (9) , q0 y0+ (9)

Also, we have so that

... (5)

0 ) p0 7 1 , q0 7 0

Solving (5) and (6),

p0 ) 0

or

p0 ) 0 , by (4A)

and

q0 ) 29

... (6) ... (4B)

Collecting relations (4A) and (4B) together, initial values of x0 , y0 , z0 , p0 , q0 are given by x0 ) 9,

y0 ) 0,

z0 ) 0,

p0 ) 0,

q0 ) 29

when

t ) t0 ) 0

f ( x, y, z, p, q) ) ( p2 , q2 ) / 2 , pq ∃ py ∃ qx , xy ∃ z ) 0

Let

... (7) ... (8)

The usual characteristic equations of (8) are given by

dx / dt ) #f / #p ) p , q ∃ y

... (9)

dy / dt ) #f / #q ) q , p ∃ x

... (10)

dz / dt ) p (#f / #p) , q(#f / #q) ) p( p , q ∃ y) , q(q , p ∃ x ) , and

... (11)

dp / dt ) ∃(#f / #x) ∃ p(#f / #z) ) p , q ∃ y

... (12)

dq / dt ) ∃(#f / #y) ∃ q (#f / #z) ) p , q ∃ x

... (13)

From (9) and (12),

(dx / dt ) ∃ (dp / dt ) ) 0

so that

x ∃ p ) C1

where C1 is an arbitrary constant. Using initial conditions (7), (14) gives 9 ∃ 0 ) C1 or

... (14) C1 ) 9 .

Created with Print2PDF. To remove this line, buy a license at: http://www.software602.com/

3.82

Non-Linear Partial Differential Equations of Order One

x∃ p)9

Hence (14) reduces to

(dy / dt) ∃ (dq / dt) ) 0

From (10) and (13),

or

x ) p , 9 ... (15)

so that

y ∃ q ) C2 , ... (16)

where C2 is an arbitrary constant. 0 ∃ 29 ) C2

Using initial conditions (7), (16) gives

y ∃ q ) ∃29

Hence (16) reduces to

C2 ) ∃29 .

or

y ) q ∃ 29 ... (17)

or

d ( p , q ∃ x ) dp dq dx ) , ∃ ) p , q ∃ y , p , q ∃ x ∃ ( p , q ∃ y) , using (9), (12) and (13) dt dt dt dt d ( p , q ∃ x) d ( p , q ∃ x) or ) p,q∃ x ) dt . dt p, q∃ x

− or

log( p , q ∃ x ) ∃ log C3 ) t

Integrating,

p , q ∃ x ) C3 et ,

or

... (18)

where C3 is an arbitrary constant. Using initial conditions (7), (18) gives 0 , 29 ∃ 9 ) C3 or C3 ) 9 .

p , q ∃ x ) 9et

Hence (18) reduces to

... (19)

Now, d ( p , q ∃ y) ) dp , dq ∃ dy ) p , q ∃ y , p , q ∃ x ∃ (q , p ∃ x ) , by (10), (12) and (13) dt dt dt dt d ( p , q ∃ y) ) p, q∃ y dt

or

Integrating,

log( p , q ∃ y) ∃ log C4 ) t

or

d ( p , q ∃ y) ) dt . b,q∃y

or

p , q ∃ y ∃ C4 et

... (20)

where C4 is an arbitrary constant. Using initial conditions (7), (20) gives 0 , 29 ∃ 0 ) C4 or C4 ) 29 .

p , q ∃ y ) 29 et

Hence (20) reduces to From (9) and (21),

dx / dt ) 29 et so that

... (21)

x ) 29 et , C5

... (22)

where C5 is an arbitary constant. Using initial conditions (7), (22) gives 9 ) 29 , C5 or C5 ) ∃9 .

x ) 29 et ∃ 9

Hence (22) reduces to From (10) and (19),

or

dy / dt ) 9 et so that

x ) 9 (2et ∃ 1) y ) 9 et , C6

... (23) ... (24)

where C6 is an arbitrary constant. Using initial conditions (7), (24) gives 0 ) 9 , C6 or C6 ) ∃9 . Hence (24) reduces to

y ) 9et ∃ 9

or

y ) 9 (et ∃ 1) ... (25)

Substituting value of y from (17) in (12), we get

dp / dt ) p , q ∃ (q ∃ 29)

(dp / dt ) ∃ p ) 29, ... (26)

or

( ∃1)dt ) e ∃t and solution is which is a linear equation whose integrating factor = e ;

;

p e∃t ) (29) e∃t dt , C7 ) ∃29 e∃t , C3

p ) ∃29 , C3 et

or

... (27)

where C7 is an arbitrary constant. Using initial condition (7), (27) gives 0 ) ∃29 , C7 or C7 ) 29 . Hence (27) reduces to

p ) ∃29 , 29 et

or

p ) 29 (et ∃ 1) ... (28)

Substituting value of x from (15) in (13), we get

Created with Print2PDF. To remove this line, buy a license at: http://www.software602.com/

Non-Linear Partial Differential Equations of Order One

dq / dt ) p , q ∃ ( p , 9)

3.83

dq / dt ∃ q ) ∃9 ,

or

... (29)

( ∃1)dt ) e ∃t and solution is which is a linear equation whose integrating factor = e ;

;

q e∃ t ) (∃9) e∃t dt , C8 ) 9e∃ t , C8

q ) 9 , C8 et

or

... (30)

where C8 is an arbitrary constant. Using initial condition (7), (30) gives 29 ) 9 , C8 or C8 = 9 . Hence (30) reduces to

q ) 9 , 9 et

q ) 9 (1 , et ) ... (31)

or

Substitutions the values of p , q ∃ x and p , q ∃ y from (13) and (24) respectively in (1) gives

dz / dt ) p(29 et ) , q(9 et ) ) 29 (et ∃ 1) (29 et ) , 9(1 , et ) (9 et ) [on putting values of p and q with help of (28) and (31)] or

or

dz / dt ) 59 2 e2t ∃ 39 2 et

dz ) (59 2 e2t ∃ 39 2 et ) dt .

... (32) z ) (5/ 2) 7 9 2e2t ∃ 392 et , C9 where C9 is an arbitrary constant. Using initial conditions (7), namely z = 0 where t = 0, (32) gives Integrating,

0 = (5/2) × 92 ∃ 39 2 , C9 or C9 ) 392 ∃ (5/ 2)9 2 . Hence (32) reduces to

z ) (5 / 2) 7 9 2 (e2t ∃ 1) ∃ 39 2 (et ∃ 1) Solving (23) and (25) for 9 and et,

9 ) x ∃ 2y

and

... (33)

et ) ( x ∃ y) /( x ∃ 2 y) ... (34)

Eliminating 9 and et from (33) and (34), we have (

2 >! x ∃ y 2 ! x∃y z ) ( x ∃ 2 y) ?% ∃ 1& & ∃ 1≅ ∃ 3( x ∃ 2y) % 2 ∋ x ∃ 2y ( >Α∋ x ∃ 2 y ( >Β or or

z ) (5/ 2) 7 {( x ∃ y)2 ∃ ( x ∃ 2 y)2} ∃ 3 {( x ∃ 2y) ( x ∃ y) ∃ ( x ∃ 2y)2 }

z ) ( y / 2) 7 (4 x ∃ 3y) , on simplification. Ex. 3. Determine the characteristics of the equation z ) p2 ∃ q2 and find the integral surface

which passes through the parabola 4z , x 2 ) 0, y ) 0 .

[Himachal 2000, 05]

Sol. Do yourself, the required characteristics are x ) 29(2 ∃ e∃t ) , y ) 2 29 (e∃t ∃ 1), z ) ∃9 2 e∃2t , 9 being parameter. Solution is 4z , ( x , y 2)2 ) 0 . Ex. 4. Determine the characteristics of the equation p2 + q2 = 4z and find the solution of this equation which reduces to z = x2 + 1 when y = 0. Miscellaneous Problem on Chapter 3 1. Show that the envelope of the family of surfaces touch each member of the family at all points of its characteristics. [Meerut 2008] 2. Find a complete integral of the partial differential equation ( p 2 , q 2 ) x ) pz and deduce the surface solution which passes through the curve x = 0, z2 = 4 y. [Meerut 2007] 3. Solve p2y + p2yx2 = qx2 [Pune 2010] Ans. Complete integral is z = a (1 + x2)1/2 + (a2y2)/2 + b. 4. Given thta (x – a)2 + (y – b)2 + z2 = 1 is complete integral of z2 (1 + p2 + q2) = 1. Find its singular integral. [Pune 2010] Hint. Use definition on page 3.1. Ans. z2 = 1

Created with Print2PDF. To remove this line, buy a license at: http://www.software602.com/

4 Homogenous Linear Partial Differential Equations with Constant Coefficients 4.1. Homogeneous and Non–homogeneous linear equations with constant coefficients. A partial differential equation in which the dependent variable and its derivatives appear only in the first degree and are not multiplied together, their coefficients being constants or functions of x and y, is known as a linear partial differential equation. The general form of such an equation is

FG A H

IJ FG B K H

!n # 1z ∃ % !z !z ∃ M0 M1 & n #1 & ∋ ! x !y ! x ! y) !y ) ( + N0 z = f(x, y), ...(1) where the coefficients A0, A1, ... An, B0, B1, ... Bn – 1, M0, M1 and N0 are constants or functions of x and y. If A0, A1, ... An, B0, B1, ... Bn – 1, M0, M1 and N0 are all constants, then (1) is called a linear partial differential equation with constant coefficients. For convenience !/!x and !/!y will be denoted by D (or Dx) and D∗ (or Dy) respectively. Then (1) can be rewritten as [(A0Dn + A1Dn – 1D∗ + ... + AnD∗n) + (B0Dn – 1 + B1 Dn – 2D∗ + ... + Bn – 1D∗n – 1) + (M0D + M1D∗) + N0]z = f(x, y), ...(2) or, briefly, F(D, D∗)z = f(x, y). ...(3) When all the derivatives appearing in (1) are of the same order, then the resulting equation is called a linear homogeneous partial differential equation with constant coefficients and it is then of the form (A0Dn – 1 + A1Dn – 1D∗ + ... + AnD∗n)z = f(x, y). ...(4) On the other hand, when all the derivatives in (1) are not of the same order, then it is called a non–homogeneous linear partial differential equation with constant coefficients. In this chapter we propose to study the various methods of solving homogeneous linear partial differential equation with constant coefficients, namely, (4) 4.2. Solution of a homogeneous linear partial differential equation with constant coefficients, namely, (A0Dn – 1 + A1Dn – 1D∗ + ... + AnD∗n)z = f(x, y), ...(1) where A0, A1, ..., An are constants. (1) may rewritten as F(D, D∗)z = f(x, y), ...(2) where F(D, D∗) = A0Dn – 1 + A1Dn – 1D∗ + ... + AnD∗n. ...(3) As in the case of linear ordinary differential equation with constant coefficients, we start with the following basic theorems. Theorem I. If u is the complementary function and z∗ a particular integral of a linear partial differential equation F(D, D∗)z = f(x, y), then u + z∗ is a general solution of the equation. Proof. Given F(D, D∗)z = f(x, y). ...(1) The complementary function u of (1) is the most general solution of F(D, D∗)z = 0. ...(2) 4.1 0

!n z !x n

A1

!nz

n #1

...

n An ! zn !y

0

!n # 1z !x n # 1

B1

! n # 1z !x n # 2 !y

...

Bn # 1

Created with Print2PDF. To remove this line, buy a license at: http://www.software602.com/

4.2

Homogenous linear partial differential equations with constant coefficients

F(D, D∗)u = 0. ...(3) + Note that the complementary function must contain as many arbitrary constants as is the order of equation (2). Any solution z∗ of (1) is called a particular integral of (1). Note that particular integral does not contain any arbitrary constant. Thus, by definition, we have F(D, D∗)z∗ = f(x, y). ...(4) Adding (3) and (4), F(D, D∗)(u + z∗) = f(x, y), showing that u + z∗ is a solution of (1). Since (1) and (2) are of the same order, the general solution u + z∗ will contain as many arbitrary constants as the general solution of (1) requires. Theorem II. If u1, u2, ..., un are solutions of the homogeneous linear partial differential n

equation F(D, D ∗ )z = 0, then − cr ur is also a solution, where C1, C2, ..., Cr, ..., Cn are arbitraryy r ,1

constants. Proof. Given equation is We have

F(D, D∗)z = 0. F(D, D∗)(cr ur) = cr F(D, D∗)ur

and

F(D, D' ) − vr = − F(D, D' )vr

n

n

r ,1

r ,1

...(1) ...(2) ...(3)

for any set of functions vr. Using results (2) and (3), we get n

F ( D, D' )

n

n

− (c u ) = − F (D, D' )(c u ) , c − F (D, D' )u r

r ,1

r

r

r

r ,1

...(4)

r

r ,1

Since ur is solution of (1) for r = 1, 2, ..., n,

+

r

so

F(D, D∗)ur = 0

for r = 1, 2, 3, ..., n.

n

(4) gives F( D, D' ) − (cr ur ) = 0, which proves the required result. r ,1

Note. For convenience we shall denote complementary function by C.F. and particular integral by P.I. 4.3. Method of finding the complementary function (C.F.) of the linear homogeneous partial differential equation with constant coefficients, namely, F(D, D∗)z = f(x, y) i.e., (A0Dn + A1Dn – 1D∗ + ... + AnD∗n)z = f(x, y), ...(1) where A0, A1, ..., An are all constants. The complementary function of (1) is the general solution of (A0Dn + A1Dn – 1D∗ + ... + AnD∗n)z = 0. ...(2) or [(D – m1D∗)(D – m2D∗) ... (D – mnD∗)]z = 0. ...(3) where m1, m2, ..., mn are some constants. Clearly, the solution of any one of the equations (D – m1D∗)z = 0, (D – m2D∗)z = 0, ................., (D – mnD∗)z = 0 ...(4) is also a solution of (3). We now show that the general solution of (D – mD∗)z = 0 is z = .(y + mx), where . is an arbitrary function. We have, (D – mD∗)z = 0 or (!z/!x) – m(!z/!y) = 0 or p – mq = 0, ...(5) which is in Lagrange’s form Pp + Qq = R. Here Lagrange’s auxiliary equations for (5) are dy dz dx , = . # m 0 1

...(6)

Created with Print2PDF. To remove this line, buy a license at: http://www.software602.com/

Homogenous linear partial differential equations with constant coefficients

4.3

Taking the first two fractions of (6), dy + mdx = 0 so that y + mx = c1 ... (7) From the third fraction of (6), dz = 0 so that z = c2. ...(8) Hence from (7) and (8), the general solution of (5) is z = .(y + mx), where . is an arbitrary function. So, we assume that a solution of (2) is of the form z = .(y + mx). ...(9) From (9), Dz = !z/!x = m.∗(y + mx), D2z = !2z/!x2 = m2.∗∗(y + mx), ................................................... and Dnz = !nz/!xn = mn.(n)(y + mx). Again,

D∗z = !z/!y = .∗ (y + mx),

D∗2z = !2z/!y2 = .∗∗ (y + mx), ................................................... and D∗nz = !nz/!yn = .(n)(y + mx). Also, in general, DrD∗sz = !r + sz/!xr!ys = mr.(r + s)(y + mx). Substituting these values in (2) and simplifying, we get (A0mn + A1mn – 1 + A2mn – 2 + ... + An).(n)(y + mx) = 0, which is true if m is a root of the equation A0mn + A1mn – 1 + A2mn – 2 + ... + An = 0. ...(10) The equation (10) is known as the auxiliary equation (A.E.) and is obtained by putting D = m and D∗ = 1 in F(D, D∗) = 0. Let m1, m2, ..., mn be n roots of A.E. (10). Two cases arise. Case I. When m1, m2, m3, ... mn are distinct. Then the part of C.F. corresponding to m = mr is z = .r(y + mrx) for r = 1, 2, 3, ..., n. Since (2) is linear, the sum of the solutions is also a solution. C.F. of (2) = .1(y + m1 x) + .2(y + m2 x) + ... + .n(y + mn x), ...(11) + where .1, .2, ..., .n are arbitrary functions. Case II. Repeated roots. Let m be repeated root of (10) and so consider (D – mD∗)(D – mD∗)z = 0. ...(12) Let (D – mD∗)z = v. ...(13) Then, (12) / (D – mD∗)v = 0 or (!v/!x) – m(!v/!y) = 0, ...(14) which is in Lagrange’s form. Hence Lagrange’s auxiliary equations for (14) are dy dv dx = , . ...(15) 1 #m 0 As before, two independent integrals of (15) are y + mx = c3 and v = c4. v = .(y + mx) ...(16) + is a solution of (14), . being as arbitrary function. Using (16), (13) becomes (!z/!x) – m(!z/!y) = .(y + mx) ...(17) which is in Lagrange’s form. Its Lagrange’s auxiliary equations for (7) are dy dx dz , = ...(18) 1 #m .( y mx ) Taking the first two fractions of (18), dy + mdx = 0 so that y + mx = c5. ... (19) Taking the first and third fractions of (18) and using (19), we get (dx ) /1 , (dz ) / .(c5 ) Integrating, z – x .(c5) = c6

so that or

dz – .(c5)dx = 0. z – x .(y + mx) = c6, using (19). ...(20)

Created with Print2PDF. To remove this line, buy a license at: http://www.software602.com/

4.4

Homogenous linear partial differential equations with constant coefficients

From (19) and (20), the general solution of (12) is z – x.(y + mx) = 0(y + mx) or z = 0(y + mx) + x.(y + mx), ...(21) where . and 0 are arbitrary functions. (21) is a part of C.F. corresponding to the two times repeated root m. In general, if a root ‘m’ is repeated ‘r’ times, the corresponding part of C.F. is .1(y + m x) + x.2(y + mx) + x2.3(y + mx) + ... + xr – 1.r(y + mx). 4.4.A. Working rule for finding C.F. of linear homogeneous partial differential equation with constant coefficients Step 1. Put the given equation in standard form (A0Dn + A1Dn – 1D∗ + ... AnD∗n)z = f(x, y)....(1) Step 2. Replacing D by m and D∗ by 1 in the coefficients of z, we obtain auxiliary equation (A.E.) for (1) as A0mn + A1mn – 1 + ... + An = 0. ...(2) Step 3. Solve (2) for m. Two cases will arise : Case (i) Let m = m1, m2, ..., mn (different roots). Then C.F. = .1(y + m1x) + .2(y + m2 x) + ... + .n(y + mn x), where .1, .2 ..., .n are arbitrary functions. If in the above case (i), m = a 1/b 1, a 2/b 2, ..., a n/b n, then C.F. , .1 (b 1 y a 1 x) .2 (b 2 y a 2 x ) ... .n (b n y a n x) Further if m = –(a 1/b 1), –(a 2/b 2), ..., –(a n/b n), then C.F. , .1 (b 1 y # a 1 x ) .2 (b 2 y # a 2 x ) ... .n (b n y # a n x) Case (ii) Let m = m∗ (repeated n times). Then corresponding to these roots C.F. = .1(y + m∗ x) + x .2(y + m∗ x) + x2 .2 ( y m ∗ x) ... + xn – 1.n(y + m∗ x). In the above case (ii), if m = a/b (repeated n times), Then corresponding to these n roots, C.F. , .1 (by ax) x .2 (by ax) x 2.2 (by ax) ... x n#1.n (by ax) And, if m = –(a/b), (repeated n times), then C.F. , .1 (by # ax) x .2 (by # ax) x 2.2 (by # ax) ... xn .n (by # ax) Case (iii) Corresponding to a non–repeated factor D on L.H.S. of (1), the part of C.F. is taken as .(y). m Case (iv) Corresponding to a repeated factor D on L.H.S. of (1), the part of C.F. is taken as .1(y) + x .2(y) + x2.3(y) + ... + xm – 1.m(y). Case (v) Corresponding to a non–repeated factor D∗ on L.H.S. of (1), the part of C.F. is taken as .(x). m Case (vi) Corresponding to a repeated factor D∗ on L.H.S. of (1), the part of C.F. is taken as .1(x) + y .2(x) + y2.3(x) + ... + ym – 1.m(x). 4.4.B. Alternative working rule for finding C.F. Let the given partial differential equation be F(D, D∗)z = f(x, y). Factorize F(D, D∗) into linear factors of the form (bD – aD∗). Then we use the following results : (i) Corresponding to each non–repeated factor (bD – aD∗), the part of C.F. is taken as .(by + ax). (ii) Corresponding to a repeated factor (bD – aD∗)m, the part of C.F. is taken as .1(by + ax) + x.2(by + ax) + x2.3(by + ax) + ... + xm – 1.m(by + ax). (iii) Corresponding to a non–repeated factor D, part of C.F. is taken as .(y). (iv) Corresponding to a repeated factor Dm, the part of C.F. is taken as .1(y) + x .2(y) + x2.3(y) + ... + xm – 1.m(y). (v) Corresponding to a non–repeated factor D∗, part of C.F. is taken as .(x).

Created with Print2PDF. To remove this line, buy a license at: http://www.software602.com/

Homogenous linear partial differential equations with constant coefficients

4.5

m

(vi) Corresponding to a repeated factor D∗ , the part of C.F. is taken as .1(x) + y .2(x) + y2.3(x) + ... + ym – 1.m(x). 4.5. Solved examples based on articles 4.4A and 4.4B [Notations p = !z/!x, q = !z/!y, r = !2z/!x2, s = !2z/!x!y and t = !2z/!y2 will be used] Ex. 1. Solve (a) r = a2t. [I.A.S. 1987; Meerut 1991] (b)(!2z/!x2) – (!2z/!y2) = 0. (c) (D2 – 3aDD∗ + 2a2D∗2)z = 0. [Kanpur 2007; Meerut 2007] Sol. (a) Given equation is !2z/!x2 = a2(!2z/!y2) or (D2 – a2D∗2)z = 0. ...(1) 2 2 The auxiliary equation of (1) is m –a =0 so that m = a, –a. The general solution of (1) is z = C.F. = .1(y + ax) + .2(y – ax), + where .1 and .2 are arbitrary functions. (b) Proceed as in part (a). Ans. z = .1(y + x) + .2(y – x) (c) Proceed as in part (a). Ans. z = .1(y + ax) + .2(y + 2ax) Ex. 2. Solve (a) (D3 – 6D2D∗ + 11DD∗2 – 6D∗3)z = 0. [Agra 2005] 3 3 3 2 3 3 (b) (! z/!x ) – 7(! z/!x!y ) + 6(! z/!y ) = 0. [Bhopal 2010] (c) (D3 – 3D2D∗ + 2DD∗2)z = 0. [Meerut 2008; Lucknow 2010] Sol. (a) The auxiliary equation is m3 – 6m2 + 11m – 6 = 0 or (m – 1)(m – 2)(m – 3) = 0 so that m = 1, 2, 3. The general solution of the given equation is + z = .1(y + x) + .2(y + 2x) + .3(y + 3x), .1, .2, .3 being arbitrary functions. (b) The given equation can be written as (D3 – 7DD∗2 + 6D∗3)z = 0. ...(1) 3 Its auxiliary equation is m – 7m + 6 = 0 or (m – 1)(m – 2)(m + 3) = 0. Hence m = 1, 2, –3 and so the general solution of (1) is z = .1(y + x) + .2(y + 2x) + .3(y – 3x), .1, .2, .3 being arbitrary functions. (c)Proceed as above. Ans. z = .1(y) + .2(y + x) + .3(y + 2x). Ex. 3. (a) Solve 2r + 5s + 2t = 0. [Meerut 2011] 2 2 2 2 2 (b) 2(! z/!x ) – 3(! z/!x!y) – 2(! z/!y ) = 0. Sol. (a) Now, r = !2z/!x2 = D2z, s = !2z/Dx!y = DD∗z and t = !2z/!y2 = D∗2z. Hence the given equation can be re–written as (2D2 + 5 DD∗ + 2D∗2)y = 0. ...(1) 2 Its auxiliary equation is 2m + 5m + 2 = 0 or (2m + 1)(m + 2) = 0. So m = –1/2, –2 and hence the general solution of (1) is z = .1(2y – x) + .2(y – 2x), .1 and .2 being arbitrary functions. Alternative method : (1) can be re–written as (2D + D∗)(D + 2D∗) = 0. So by using the alternative working rule 4.4B, the general solution of (1) is z = .1(2y – x) + .2(y – 2x), .1 .2 being arbitrary functions. (b) Proceed as in part (a). Ans. z = .1(2y – x) + .2(y + 2x) Ex. 4. Solve (a) r + t + 2s = 0 [Kanpur 2009] (b) 25r – 40s + 16t = 0 [Bilaspur 1996; Jabalpur 2002; Sagar 2004]; (c) (4D2 + 12DD∗ + 9D∗2)z = 0. [Kanpur 2008; Indore 2004] Sol. (a) Here r = !2z/!x2 = D2z, t = !2z/!y2 = D∗2z, s = !2z/!x!y = DD∗z. So the given equation becomes (D2 + D∗2 + 2DD∗)z = 0. ...(1) 2 Its auxiliary equation is m + 1 + 2m = 0 or (m + 1)2 = 0 or m = –1, –1. So the general solution of (1) is z = .1(y – x) + x.2(y – x), .1 and .2 being arbitrary functions. (b) Here r = !2z/!x2 = D2z, s = !2z/!x!y = DD∗z, t = !2z/!y2 = D∗2z. So the given equation becomes (25D2 – 40DD∗ + 16D∗2)z = 0. ...(1)

Created with Print2PDF. To remove this line, buy a license at: http://www.software602.com/

4.6

Homogenous linear partial differential equations with constant coefficients

Its auxiliary equation is 25m2 – 40m + 16 = 0 or (5m – 4)2 = 0 so that m = 4/5, 4/5. Hence the general solution of (1) is z = .1(5y + 4x) + x.2(5y + 4x). Alternative method (1) may be written as (5D – 4D∗)2z = 0. Using result (ii) of Art 4.4B, the solution of (1) is z = .1(5y + 4x) + x.2(5y + 4x), .1, .2 being arbitrary functions. (c)Proceed as in part (b). Ans. z = .1(2y – 3x) + x.2(2y – 3x) 3 2 2 Ex. 5. Solve (a) (D – 4D D∗ + 4DD∗ )z = 0. [Bhopal 2000, 03] (b) (D4 – 2D3D∗ + 2DD∗3 – D∗4)z = 0. [Bilaspur 2004] (c) (D4 + D∗2 – 2D2D∗2)z = 0. (d) (D3 – 3D2D∗ + 3DD∗2 – D∗3)z = 0. Sol. (a) The auxiliary equation of the given equation is m3 – 4m2 + 4m = 0 or m(m – 2)2 = 0 so that m = 0, 2, 2 Hence the general solution of the given equation is z = .1(y) + .2(y + 2x) + x.3(y + 2x), .1, .2, .3 being arbitrary functions. (b) The auxiliary equation of the given equation is m4 – 2m3 + 2m – 1 = 0 or (m + 1)(m – 1)3 = 0 so that m = –1, 1, 1, 1. Hence the general solution of the given equation is z = .1(y – x) + .2(y + x) + x.2(y + x) + x2.3(y + x), where .1, .2, .3 and .4 are arbitrary functions. (c) Try yourself. Ans. z = .1(y + x) + x.2(y + x) + .3(y – x) + x.4(y – x) (d) Proceed as in part (a). Ans. z = .1(y + x) + x.2(y + x) + x2.3(y + x) 3 2 2 3 Ex. 6. Solve (a) (D D∗ + D D∗ )z = 0. (b) (D3D∗ – 4D2D∗2 + 4DD∗3)z = 0. Sol. (a) The given equation can be re–written as D2D∗2(D + D∗)z = 0. ...(1) Hence using the alternative method 4.4B for C.F., the general solution is z = .1(y) + x.2(y) + .3(x) + y.4(x) + .5(y – x), where .1, .2, .3, .4 and .5 are arbitrary functions. (b) The given equation can be re–written as DD∗(D2 – 4DD∗ + 4D∗2) = 0 Or DD∗(D – 2D∗)2 = 0. Using the working rule 4.4B, the required general solution is z = .1(y) + .2(x) + .3(y + 2x) + x.4(y + 2x), where .1, .2, .3 and .4 are arbitrary functions. Ex. 7. Solve (a) (!4z/!x4) – (!4z/!y4) = 0 (b) (D4 + D∗4)z = 0. 4 4 Sol. (a) Rewriting, the given equation is (D – D∗ )z = 0. ...(1) 4 2 2 Its auxiliary equation is m – 1 = 0 or (m – 1)(m + 1) = 0 / m = 1, –1 i, –i. Hence the general solution of (1) is z = .1(y – x) + .2(y + x) + .3(y + ix) + .4(y – ix), where .1, .2, .3 and .4 are arbitrary functions. (b) The auxiliary equation of the given equation is m4 + 1 = 0 or (m2 + 1)2 – 2m2 = 0 (m2 + 1)2 – ( m 2 )2 = 0

or so that

m

2

2m 1 = 0

or

or 2

m # 2m 1 = 0 /

(m2

1

2 m)( m2

m = ( # 1 1 i)

1 # 2 m) = 0

2 , (1 1 i)

2.

Let z1 =(# 1 i) 2 and z2 = (1 i) 2 , then, m = z1, z1, z2 , z2 , where z1 and z2 denote complex conjugates of z1 and z2 respectively.. Hence the general solution of the given equation is z = .1(y + z1x) + .2(y + z1x) + .3(y + z2x) + .4(y + z2 x), where .1, .2, .3, .4 are arbitrary functions. 4.6. Particular integral (P.I.) of homogeneous linear partial differential equation given by

F ( D, D ∗) y , f ( x, y )

... (1)

Created with Print2PDF. To remove this line, buy a license at: http://www.software602.com/

Homogenous linear partial differential equations with constant coefficients

4.7

The inverse operator1/ F (D , D∗) of the operator F ( D, D∗) is defined by the following identity % ∃ 1 F ( D, D∗) ∋ f ( x, y ) & , f ( x, y ) ( F ( D, D∗) ) +

Particular integral (P.I.) of (1) ,

1 f ( x, y ) F ( D, D∗)

In what follows we shall treat the symbolic functions of D and D∗ as we do for the symbolic functions of D alone in ordinary differential equations. Thus it will be factorized and resolved into partial fractions or expanded in an infinite series as the case may be. The reader is advised to note carefully the following results : (i) D, D2, ... will stand for differentiating partially with respect to x once, twice and so on. 2

! 4 5 2 5 2 x y , 12 x y . !x (ii) D∗, D∗2... will stand for differentiating partially with respect to y once, twice and so on.

For example,

Dx4y5 =

! 4 5 3 5 x y , 4x y ; !x

D2x4y5 =

2 D∗2x4y5 = ! 2 x 4 y5 , 20 x 4 y3 . !y (iii) 1/D, 1/D2, ... will stand for integrating partially with respect to x once, twice and so on.

For example,

! 4 5 4 4 4 5 D ∗ x y = !y x y , 5 x y ;

z

zz

z

zz

x5 y5 x 6 y5 1 4 5 1 4 5= 4 5 x y = x 4 y 5 dx , ; x y dx dx , x y D 30 D2 5 2 (iv) 1/D∗, 1/D∗ , ... will stand for integrating partially with respect to y once, twice and so on.

For example,

x 4 y6 1 4 5 1 x 4 y 5 = x 4 y5 dy dy , x 4 y 7 . x y = x 4 y 5 dy , ; D∗ 6 42 D∗2 4.7. Short methods of finding the P.I. in certain cases. Before taking up the general method for finding P.I. of F(D, D∗)z = f(x, y) we begin with cases when f(x, y) is in two special forms. The methods corresponding to these forms are much shorter than the general methods to be discussed in Art. 4.12. 4.8. A Short Method I. When f(x, y) is of the form f(ax + by). The method under consideration is based on the following theorem. Theorem I. If F(D, D∗) be homogeneous function of D and D∗ of degree n, then 1 .(ax by) 1 .(n)(ax by) = , F(a, b) F( D, D∗) provided F(a, b) 2 0, .(n) being the nth derivative of . w.r.t. ax + by as a whole. Proof. By direct differentiation, we have Dr.(ax + by) = ar.(r)(ax + by), s s (s) r s D∗ .(ax + by) = b . (ax + by) and D D∗ .(ax + by) = arbs.(r + s)(ax + by). Since F(D, D∗) is homogeneous function of degree n, so we have F(D, D∗).(ax + by) = F(a, b).(n)(ax + by). ...(1) Operating both sides of (1) by 1/F(D, D∗), we have 1 .3n4(ax by) . .(ax + by) = F(a, b) ...(2) F( D, D∗) Since F(a, b) 2 0, dividing both sides of (2) by F(a, b), we get 1 1 .(ax by) . .3n4(ax by) = ...(3) F( D, D∗) F(a, b) An important deduction from result (3) : Putting ax + by = v, (3) gives

For example,

Created with Print2PDF. To remove this line, buy a license at: http://www.software602.com/

4.8

Homogenous linear partial differential equations with constant coefficients

1 1 .3n4 (v) = ...(4) . ( v) . F( D, D∗) F ( a , b) Integrating both sides of (4) n times w.r.t. ‘v’, we have 1 1 .(v) = where v = ax + by. ... .(v) dv dv ... dv , F(D, D∗) F(a, b) Exceptional case when F(a, b) = 0. When F(a, b) = 0, then the above theorem does not hold good. In such a case the new method is based on the following theorem. Note that F(a, b) = 0 if and only if (bD – aD∗) is a factor F(D, D∗).

zz z

n

x 1 by) = n .(ax by) . n .( ax (bD # aD ∗) b n! Proof. Consider the equation (bD – aD∗)z = xr.(ax + by) bp – aq = xr.(ax + by).

Theorem II. or

...(1) ...(2)

dy dx dz = , . ...(3) b #a x r .(ax by)

Lagrange’s subsidiary equations for (2) are

Taking the first two fractions of (3), adx + bdy = 0 so that Taking the first and third members of (4) and using (4), we get

ax + by = c1

... (4)

r

dx = r dz b x .(c1 )

dz =

or

x .(c1) dx . b

r 1

z=

Integrating,

x .(c1) x r 1.(ax by) , by (4). , b(r 1) b(r 1)

...(5)

1 x r .(ax (bD # aD' )

...(6)

(5) is a solution of (1). z=

Now, from (1), From (5) and (6), Hence, if

by) .

r 1 1 x r .(ax by) = x .(ax (bD # aD' ) b(r 1) 1 z= .(ax by) , then we have (bD # aD' )n

z= =

LM N

1 1 x 0.(ax (bD # aD' )n # 1 (bD # aD' )

1 (bD # aD' )

n #1

x .( ax b

LM N

= =

1 1 x2 .(ax b (bD # aD' )n # 2 2b 1

1 2

2!b (bD # aD' )

1

n#2

1

n !b n (bD # aD' )n # n

...(7)

OP Q

by) , as x0 = 1

by ) , using (7) for r = 0

1 1 x.(ax = 1 b (bD # aD' )n # 2 (bD # aD∗)

=

by) .

OP Q

by)

by ) , using (7) for r = 1

x 2 .( ax

by )

x n .(ax

by ) ,

xn bn n !

. (ax

by )

[after repeated use of (7) for n – 2 times more]

Created with Print2PDF. To remove this line, buy a license at: http://www.software602.com/

Homogenous linear partial differential equations with constant coefficients

4.9

Working rule for finding particular integral where f(x, y) = .(ax + by). The following rules will used depending upon the situation in hand. Formula (i). When F(a, b) 2 0 and F(D, D∗) is a homogeneous function of degree n, then 1 1 P.I. = .(ax by) = ... f (v) dv dv ... dv , where v = ax + by F(a, b) F(D, D' ) Note that R.H.S. contains a multiple integral of nth order. Formula (ii). When F(a, b) = 0, we have

zz z

n 1 by) = xn .(ax by) . n .(ax (bD # aD' ) b n! 4.9. Solved Examples based on Short Method I of Art. 4.8 Ex. 1. Solve (D2 + 3DD∗ + 2D∗2)z = x + y. [I.A.S. 1986, Meerut 2005, 07, 09, 10] Sol. The auxiliary equation of the given equation is m2 + 3m + 2 = 0 giving m = –1, –2. C.F. = .1(y – x) + .2(y – 2x), .1, .2 being arbitrary functions. +

P.I. =

Now, =

D

2

1 3DD∗

1 2

1

3 5 1 5 1 2 5 12

2 D∗

2

(x

y)

6 6 v d v d v , where v = x +

y, using formula (i) of working rule

z

= (v 2 / 2) dv = (1/6) × (v3/6) = (1/36) × (x + y)3. Hence the required general solution is z = C.F. + P.I., i.e., z = .1(y – x) + .2(y – 2x) + (1/36) × (x + y)3. 2 Ex. 2. Solve (a) (2D – 5DD∗ + 2D∗2)z = 24(y – x). [Vikram 2000] (b) (!2V/!x2) + (!2V/!y2) = 12(x + y) [Nagpur 2010] 2 2 (c) (D + D∗ )z = 30(2x + y). [Bhopal 1996, Sagar 2004] (d) (!2z/!x2) + 2(!2z/!x!y) + (!2z/!y2) = 2x + 3y. [Kumaun 1992] (e) (D2 + 3DD∗ + 2D∗2)z = 2x + 3y. [Kurukshetra 2005] (f) r + s – 2t = (2x + y)1/2. [Lucknow 2010] Sol. (a) The auxiliary of the given equation is 2m2 – 5m + 2 = 0 giving m = 1/2, 2. C.F. = .1(2y + x) + .2(y + 2x), .1, .2 being arbitrary functions. 1 1 Now, P.I. = 2 2 24( y # x ) , 24 2 2 ( y # x) 2 D # 5DD∗ 2 D∗ 2 D # 5DD∗ 2 D∗

+

24

=

2 5 ( #1)2 # 5 5 (#1) 5 2 working rule

2 5 22

6 6 v d v d v , where v = y – x, using formula (i) of

6

2 3 3 = (24 / 20) 5 ( v / 2) d v , (6 / 5) 5 ( v / 6) , (1/ 5) 5 ( y # x) .

Hence the required general solution is z = .1(2y + x) + .2(y + 2x) + (y – x)3/5. (b) The given equation can be written as (D2 + D∗2)V = 12(x + y). ...(1) 2 Its auxiliary equation is m +1=0 so that m = ± i. C.F. = .1(y + ix) + .2(y – ix), .1, .2 being arbitrary functions. + 1 1 12 y) , 12 2 y) , 2 v dv dv Now, P.I. = 2 2 12( x 2 (x 2 D D∗ D D∗ 1 1

zz

Created with Print2PDF. To remove this line, buy a license at: http://www.software602.com/

4.10

Homogenous linear partial differential equations with constant coefficients

6

2 3 = 6 ( v / 2) d v , v , ( x

y )3 .

Hence the required general solution is V = .1(y + ix) + .2(y – ix) + (x + y)3. (c)Try yourself. Ans. z = .1(y + ix) + .2(y – ix) + (2x + y)3 (d)Try yourself. Ans. z = .1(y – x) + x.2(y – x) + (1/150) × (2x + 3y)3 (e)Proceed as in part (d). Ans. y = .1(y – x) + .2(y – 2x) + (1/240) × (2x + 3y)3 (f) Since r , !2 z / !x 2 , s , !2 z / !x!y, t , !2 z / !y 2 , the given equation can be re–written as or (D2 + DD∗ – 2D∗2)z = (2x + y)1/2. ! 2 z / !x2 ! 2 z / !x !y # 2 (! 2 z / !y 2 ) = (2x + y)1/2 2 Its auxiliary equation is m +m–2=0 so that m = 1, –2. C.F. = .1(y + x) + .2(y – 2x), .1, .2 being arbitrary functions. + P.I. =

D

2

1 (2 x DD' # 2 D'2

y)1/ 2 =

1 2

2

1 2 3/2 1 2 2 1 v d v , 5 5 v 5 / 2 , (2 x 4 3 4 3 5 15 Hence the required general solution is

=

6

66 v

1/ 2

2 5 1 # 2 5 12

d v d v , where v = 2x + y

y )5/ 2

z = .1(y + x) + .2(y – 2x) + (1/15)(2x + y)5/ 2 . 2 2 2x + 3y Ex. 3. Solve (a) (D + 2DD∗ + D∗ )z = e . [Bhopal 2010; Indore 1998; Jabalpur 1998; Purvanchal 2007, Sagar 1999; K.V. Kurkshetra 2005] (b) (D2 – 2DD∗ + D∗2)z = ex + 2y. [Bhopal 1997, 98, Kanpur 2005] (c) (D3 – 6D2D∗ + 11DD∗2 – 6D∗3)z = e5x + 6y. Sol. (a) Here auxiliary equation is m2 + 2m + 1 = 0 so that m = –1, –1. C.F. = .1(y – x) + x.2(y – x), .1, .2 being arbitrary functions. + 1 v 1 1 2 x 3y 2 x 3y P.I. = 2 = e dv dv , where v = 2x + 3y , 2 2 e 2 e D 2 DD' D' ( D D∗) (2 3)

zz

6

= (1/ 25) 5 e v d v , (1/ 25) 5 ev , (1/ 25) 5 e2 x

3y

Solution is z = C.F. + P.I. = .1(y – x) + x.2(y – x) + (1/25) × e2x + 3y. + (b) Proceed as in part (a). Ans. z = .1(y + x) + x.2(y + x) + ex + 2y (c) Here auxiliary equation is m3 – 6m2 + 11m – 6 = 0 giving m = 1, 2, 3. C.F. = . (y + x) + . (y + 2x) + . (y + 3x), . , . , . being arbitrary functions. + 1 2 3 1 2 3 1 1 P.I. = 3 e5x 6 y = e5 x 6 y ( D # D' )( D # 2 D' )( D # 3D' ) D # 6 D 2 D' 11DD' 2 # 6 D' 3 =

1 (5 # 6)(5 # 12)(5 # 18)

zz

zzze

v

dv dv dv , where v = 5x + 6y

z

1 1 1 1 e v dv dv , # e v dv , # e v , # e5x 6 y #91 91 91 91 Hence the required solution is z = .1(y + x) + .2(y + 2x) + .3(y + 3x) – (1/91) × e5x + 6y. Ex. 4. Solve (a) r – 2s + t = sin (2x + 3y). [Meerut 2007; Indore 2002; Vikram 1996;] 3 2 2 (b) (D – 4D D∗ + 4DD∗ )z = 2 sin (3x + 2y). [Kanpur 2008; I.A.S. 2006] (c) (D3 – 4D2D∗ + 4DD∗2)z = cos (2x + 3y). (d) (D3 – 3DD∗2 – 2D∗3)z = cos (x + 2y). [Delhi Maths Hons. 1992] =

Sol. (a) Since r , !2 z / !x 2 , s , !2 z / !x!y, t , !2 z / !t 2 , the given equation becomes

Created with Print2PDF. To remove this line, buy a license at: http://www.software602.com/

Homogenous linear partial differential equations with constant coefficients

4.11

(D2 – 2DD∗ + D∗2)z = sin (2x + 2y). 2 Its auxiliary equation is m – 2m + 1 = 0 so that m = 1, 1. C.F. = .1(y + x) + x .2(y + x), .1, .2 being arbitrary functions. +

! 2 z / !x2 # 2(! 2 z / !x!y) ! 2 z / !y 2 = sin (2x + 3y) or

P.I. =

1 2 sin (2 x ( D # D∗)

3y) ,

z

1 2 (2 # 3)

z z sin v dv dv , where v = 2x + 3y

= # cos v dv = –sin v = –sin (2x + 3y).

or

Hence the required general solution is z = .1(y + x) + x.2(y + x) – sin(2x + 3y). (b) The auxiliary equation of the given equation is m3 – 4m2 + 4m = 0. m(m2 – 4m + 4) = 0 or m(m – 2)2 = 0 so that m = 0, 2, 2. C.F. = .1(y) + .2(y + 2x) + x.3(y + 2x), .1, .2, .3 being arbitrary functions. + Now, P.I. =

1 3

D # 4 D D∗ 4 DD ∗2

= 25

2

2 sin (3 x 2 y )

1 33 # 4 5 32 5 2

4 5 3 5 22

6 6 6 sin v d v d v , where v = 3x + 2y

= (2 / 3) 5 6 6 (# cos v )d v d v , #(2 / 3) 5 6 sin v d v , (2 / 3) 5 cos v , (2 / 3) 5 cos(3 x 2 y ) The required general solution is z = .1(y) + .2(y + 2x) + x.3(y + 2x) + (2/3) × cos (3x + 2y). (c) Proceed as in part (b). Ans. z = .1(y) + .2(y + 2x) + x.3(y + 2x) – (1/32) × sin (2x + 3y) (d) Proceed as in part (b). Ans. z = .1(y – x) + x.2(y – x) + .3(y + 2x) + (1/27) × sin (x + 2y) Ex. 5. Solve !2z/!x2 + !2z/!y2 = cos mx cos ny. [Kanpur 2007; Nagpur 2010] Sol. Given equation can be written as (D2 + D∗2)z = cos mx cos ny. Its auxiliary equation is m2 + 1 = 0 so that m = ± i. C.F. = .1(y + ix) + .2(y – ix), .1 and .2 being arbitrary functions. + cos (mx ny) cos (mx # ny) P.I. = 2 1 2 cos mx cos ny , 2 1 2 2 D D' D D' = 1 2 1 2 cos (mx ny) 1 2 1 2 cos (mx # ny) 2D 2D D' D' 1 1 1 1 = cos v dv dv cos u du du , 2 m2 n2 2 m 2 (# n )2 where v = mx + ny and u = mx – ny

zz

= 1

1

2 m2

,#

=#

n2

z sin v dv

1 2(m

2

1

zz

n2 )

z

1 1 1 1 [ # cos v # cos u ] sin u du = 2 m2 n2 2 m 2 n2

[cos ( mx

ny )

cos ( mx # ny )] , as v = mx + ny, u = mx – ny

5 2 cos mx cos ny , # ( m

2

2 #1

cos mx cos ny . 2(m n ) Hence the required general solution is z = .1(y + ix) + .2(y – ix) – (m2 + n2)–1 cos mx cos ny. 2

2

Ex. 5. (b) Solve !2 z / !x2 Sol. Do like Ex. 5(a)

!2 z / !y 2 , cos mx sin ny.

n )

[Ravishankar 1999, 2001]

Ans. z , .1 ( y ix) .2 ( y # ix) (sin mx sin ny ) /(m 2

n2 )

Created with Print2PDF. To remove this line, buy a license at: http://www.software602.com/

4.12

Homogenous linear partial differential equations with constant coefficients

Ex. 6. Solve the following partial differential equations : (a) (D2 – 2DD∗ + D∗2)z = tan (y + x) or (D – D∗)2z = tan (y + x) [Jiwaji 1996] 2 2 2 (b) (D – 2aDD∗ + a D∗ )z = f(y + ax) or (D – aD∗)2z = f(y + ax). (c) 4r – 4s + t = 16 log (x + 2y). [Agra 2009; Meerut 2009; Ravishankar 2000] Sol. (a) Here auxiliary equation is (m – 1)2 = 0 so that m = 1, 1. C.F. = .1(y + x) + x.2(y + x), where .1 and .2 are arbitrary functions. + Now,

P.I. =

1

tan ( y

( D # D' )2

x) ,

x2 12 5 2!

tan ( y

x) ,

x2 tan ( y 2

x)

[Using formula (ii) of working rule with a = 1, b = 1, m = 2] Hence the required general solution is z = .1(y + x) + x.2(y + x) + (x2/2) × tan (y + x). (b) Here auxiliary equation is (m – a)2 = 0 so that m = a, a. C.F. = .1(y + ax) + x.2(y + ax), .1, .2 being arbitrary functions. + P.I. =

+

1 ( D # aD' )2

f (y

ax)

x2 12 5 2!

f (y

ax) ,

x2 f (y 2

ax) .

[using formula (ii) of working rule with a = a, b = 1, m = 2] General solution is z = .1(y + x) + x.2(y + x) + (x2/2) × f(y + ax).

(c) Since r , !2 z / !x 2 , s , !2 z / !x!y, t , !2 z / !y 2 , the given equation becomes 4(!2z/!x2) – 4(!2z/!x!y) + (!2z/!y2) = 16 log (x + 2y) or (4D2 – 4DD∗ + D∗2)z = 16 log (x + 2y) Its auxiliary equation is 4m2 – 4m + 1 = 0 so that m = 1/2. 1/2. C.F. = .1(2y + x) + x.2(2y + x), .1 and .2 being arbitrary functions. + Now,

P.I. =

1 (2 D # D' )

2

16 log ( x

2 y ) , 16 5

x2 2

2 5 2!

2 y ) = 2x2 log (x + 2y)

log ( x

[using formula (ii) of working rule with a = 1, b = 2, m = 2] z = .1(2y + x) + x.2(2y + x) + 2x2 log (x + 2y). + The required solution is Ex. 7. Solve the following partial differential equations : (a) (2D2 – 5DD∗ + 2D∗2)z = 5 sin (2x + y). [M.D.U. Rohtak 2005] (b) (D2 – 5DD∗ + 4D∗2)z = sin (4x + y). [Meerut 2006, 08] (c) (D3 – 2D2D∗ – DD∗2 + 2D∗3)z = ex + y. [Bhopal 2000, 03, Meerut 2007; Jabalpur 2004] (d) r – t = x – y. (e) 2r – s – 3t = 5ex/ey. [Indore 2003; Jiwaji 2003, Vikram 1998] (f) r + 5s + 6t = (y – 2x)–1

or

(!2 z / !x2 ) 5(!2 z / !x!y ) 6(!2 z / !y 2 ) , 1/( y # 2 x)

[Agra 2009; Indore 2000; I.A.S. 1991; Garhwal 2005] Sol. (a) Here auxiliary equation is 2m2 – 5m + 2 = 0 so that m = 2, 1/2. C.F. = .1(y + 2x) + .2(2y + x), .1, .2 being arbitrary functions. + Now, = 5

P.I. =

LM N

OP Q

1 1 1 5 sin (2 x y) , 5 sin (2 x y) ( D # 2 D∗) (2 D # D∗) 2 D # 5DD∗ 2 D∗ 2 2

1 1 sin v d v , where v = 2x + y, using formula (i) of working rule D # 2D ∗ (2 5 2) # 1

6

Created with Print2PDF. To remove this line, buy a license at: http://www.software602.com/

Homogenous linear partial differential equations with constant coefficients

4.13

5 1 5 1 5 x (# cos v) , # cos (2 x y ) , # 5 1 cos (2 x y ) 3 D # 2 D' 3 D # 2 D' 3 1 5 1! [Using formula (ii) with a = 2, b = 1, m = 2] The required general solution is z = . (y + 2x) + .2(2y + x) – (5x/3) × cos (2x + y). + 1 (b) Do as in part (a). Ans. z = .1(y + x) + .2(y + 4x) – (x/3) × cos (4x + y) (c) Here auxiliary equation is m3 – 2m2 – m + 2 = 0 or m2(m – 2) – (m – 2) = 0 2 (m – 1)(m – 2) = 0 so that m = 2, 1, –1. C.F. = .1(y + 2x) + .2(y + x) + .3(y – x), .1, .2, .3 being arbitrary functions. +

=

or

P.I. =

1 3

2

2

D # 2D D∗ # DD ∗

3

2 D∗

=

1 1 D # D ∗ 12 # (1 5 1) # (2 5 1)2

=

1 D # D∗

ex

6e

v

y

,

79 1 1 ex : ( D # D∗) 2 2 > 22

6

, (1/ 3) 5 sin v dv , #(1/ 3) 5 cos v , #(1/ 3) 5 cos ( x 2 y ) z , .1 ( y x) .2 ( y 2x ) # (1/ 3) 5 cos( x 2 y)

+ Solution is

Ex. 14. Solve ( D2 # DD∗ # 2 D∗2 )z , 2x 3y e3x

4y

.

[I.A.S. 2000]

2

m #m#2 , 0

Sol. The auxiliary equation

giving

m , 2, # 1 .

C.F. = .1 ( y 2 x ) .2 ( y # x ), .1 , .2 being arbitrary functions + P.I. corresponding to (2x + 3y) 1 1 = 2 (2 x 3y) , 2 v (dv)2 , where v , 2 x 3y D # DD∗ # 2 D∗2 2 # (2 5 3) # (2 5 32 )

66

= #

1 20

6

v2 1 % v3 ∃ 1 3 dv , # ∋ && , # (2 x 3y) ∋ 2 20 ( 2 5 3 ) 60

P.I. corresponding to e3x 4 y 1 1 = 2 e3x 4 y , 2 2 D # DD∗ # 2 D∗ 3 # (3 5 4) # (2 5 42 ) = #(1/ 35) 5 ev , #(1/ 35) 5 e3 x +

General solution is

4y

66 e (dv) v

2

, where v , 3x 4 y

.

z , .1 ( y 2 x) .2 ( y # x) # (1/ 60) 5 (2 x 3y)3 # (1/ 35) 5 e3x

4y

.

EXERCISE 4(A) Solve the following partial differential equations: 1. ( D 2 # DD∗ – 6 D∗2 ) z , cos(2 x

[Agra 2009, 10]

y)

Ans. z , .1 ( y 3x ) .2 ( y # 2 x ) – (1/ 4) 5 cos (2 x 2. r # 4 s 4t , e2 x

y ) .1 , .2 , being arbitrary functions.

y

[Agra 2010] 2

Ans. z , .1 ( y 2 x) x.2 ( y 2 x) ( x / 2) 5 e 3. ( D 3 – 4 D 2 D∗2

2x y

.1 , .2 being arbitrary functions

4 DD∗2 ) z , 6sin(3x 2 y)

Ans. z , .1 ( y ) .2 ( y 2 x ) x.3 ( y 2 x ) 2cos(3x 2 y ), .1 , .2 , .3 , being arbitrary functions. 4. ( D – 3D∗)2 (D 3D ∗) z , e3 x

y

[Agra 2005]

Ans. z , .1 ( y 3 x) x.2 ( y 3 x) .3 ( y – 3x ) ( x /12) 5 e3 x y , .1 , .2 , .3 , being arbitrary functions. m n 4.10. Short Method II. When f(x, y) is of the form x y or a rational integral algebraic function of x and y. Then the particular integral (P.I.) is evaluated by expanding the symbolic function 1/f (D, D ∗) in an infinite series of ascending powers of D or D ∗. In solved examples 1 and 2 of Art. 4.11, we have shown that P.I. obtained on expanding 1/f(D, D ∗) in ascending powers of D is different from that obtained on expanding 1/f(D, D ∗) in ascending powers of D ∗. Since to get the required general solution of given differential equation any P.I. is required, any of the two methods can be used. The difference in the two answers of P.I. is not material as it can be incorporated in the arbitrary functions occuring in C.F. of that given differential equation.

Created with Print2PDF. To remove this line, buy a license at: http://www.software602.com/

Homogenous linear partial differential equations with constant coefficients

4.19

Remark : If n < m, 1/f(D, D ∗) should be expanded in powers of D ∗/D whereas if m < n, 1/f(D, D ∗) should be expanded in powers of D/D ∗. 4.11 SOLVED EXAMPLES BASED ON SHORT METHOD II Ex. 1. Solve (D2 # a2D∗2)z = x or (!2z/!x2) # a2(!2z/!y2) = x. Sol. Here auxiliary equation is m2 # a2 = 0 so that + C.F. = .1(y + ax) + .2(y # ax), .1, .2 being arbitrary functions. Now, P.I. =

FG H

FG H

IJ K

2

IJ K

2 #1

1 1 1 1 # a 2 D∗ 2 2 2 x , 2 2 2 2 x , 2 2 D # a D∗ D [1 # a ( D∗ / D )] D D

m = a, #a. ...(1) x

3

= 12 1 a2 D∗2 ... x , 12 x , x . 6 D D D Alternatively, we can compute P.I. on follows :

...(2)

FG H

2

1 1 D P.I. = 2 1 2 2 x , 2 2 2 2 2 x ,# 2 2 1# 2 2 D # a D∗ # a D∗ [1 # ( D / a D∗ )] a D∗ a D∗

#1

x

% ∃ D2 1 1 xy 2 1 ... x , # x , # 5 . ∋ & 2 a 2 D ∗2 ( a 2 D ∗2 a 2 D ∗2 a2 ) Hence the required general solution is z = C.F. + P.I. that is, z = .1(y + ax) + .2(y # ax) + x3/6, using (1) and (2). z = .1(y + ax) + .2(y # ax) # (xy2)/(2a2), using (1) and (3). 1

= #

or

IJ K

...(3)

Ex. 2. Solve (D2 + 3DD∗ + 2D∗2)z = x + y, by expanding the particular integral in ascending powers of D as well as in ascending powers of D∗. [Bhopal 2000, 03; Indore 1999; Jiwaji 1995; Rewa, 2002, 03; I.A.S. 1994] Sol. Here auxiliary equation is m2 + 3m + 2 = 0 so that m = #2, #1. +C.F. = .1(y # 2x) + .2(y # x), .1, .2 being arbitrary functions. ...(1) Now, by expanding in ascending powers of D, we have 1 1 P.I. = 2 y) , ( x y) 2 (x 2 D 3DD∗ 2D∗ 2 D 3 D 2 D∗ 1 2 2 D∗ 2D∗ ? % D2 = Α1 ∋ 2 D ∗ 2 ΑΧ ( 2 D ∗ 2 1

FH

IK

3 D ∃≅ Β 2 D ∗ )& Β∆

LM FG N H

IJ OP KQ

y) ,

1 % 3 D 1# 2 ∋ 2 D∗ 2D ∗ (

#1

(x

FH

∃ ...& ( x )

y)

IK

y xy 2 y3 1 3 1 . x y # y , x # , # 2 2 4 24 2 D∗ 2 2 D∗ 2 Again, by expanding in ascending powers of D, P.I. of given equation is given by

=

P.I. =

D2

1 3DD∗

2 D∗ 2

(x

y) ,

LM FG N H

D2 1

1 3D ∗ D

2 D∗ 2 D2

IJ OP (x KQ

1 y) = 2 D

? % 3D ∗ Α1 ∋ ΑΧ ( D

...(2) #1

2D ∗ 2 ∃ ≅ Β (x D2 &) Β∆

y)

1 % 3D ∗ ∃ yx 2 x 3 1 1 # ... ( x y ) = ( x y # 3 x ) , # . ∋ & 2 3 D D2 ( D2 ) Hence the required general solution is given by z = C.F. + P.I., i.e., z = .1(y # 2x) + .2(y # x) + (1/4) × xy2 # (1/24) × y3, using (1) and (2) z = .1(y # 2x) + .2(y # x) + (1/2)× yx2 # (1/3) × x3, using (1) and (3). ,

or

Created with Print2PDF. To remove this line, buy a license at: http://www.software602.com/

4.20

Homogenous linear partial differential equations with constant coefficients 3

3

Ex. 3. Solve (! z/!x ) # (!3z/!y3) = x3y3

3

3

3 3

or (D # D∗ )z = x y . [I.A.S. 1997] Sol. Here auxiliary equation is m3 # 1 = 0 so that m = 1, Ε, Ε2, 2 where Ε and Ε are complex cube roots of unity. + C.F. = .1(y + x) + .2(y + Εx) + .3(y + Ε2x), .1, .2, .3 being arbitrary functions. Now, P.I. =

FG H

or

FG H

∗3 1 1 x 3 y3 , 3 x 3 y 3 , 13 1 # D 3 3 3 3 3 D # D∗ D [1 # ( D∗ / D )] D D

IJ K

F H

I K

IJ K

#1

x 3y3

6 ∃ ∗3 1 6x 3 = 1 % x 3 y 3 6 5 x = 13 1 D 3 ... x 3 y3 , 13 x 3 y3 3 3 ∋ 4 5 5 5 6 &) D D D D D ( 6 3 9 = (1/120)× x y + (1/10080) × x . Hence the required general solution is z = C.F. + P.I. 2 6 3 9 z = .1(y + x) + .2(y + Εx) + .3(y + Ε x) + (1/120) × x y + (1/10080) × x . Ex. 4. Solve r + (a + b)s + abt = xy. [Indore 1998; Vikram 1998, 2000; Rewa 1998] Sol. Given equation can be written as [D2 + (a + b)DD∗ + abD∗2] = xy. 2 Its auxiliary equation is m + (a + b)m + ab = 0 or (m + a)(m + b) = 0 so that m = #a, #b.

C.F. = .1(y # ax) + .2(y # bx), .1 , .2 being arbitrary functions.

+

Now, P.I. =

D2

1 b)DD∗

(a

abD∗2

xy ,

1

L D M1 N 2

(a

OP xy Q ...O xy PQ

∗ b) D D

∗2 ab D 2 D

LM D' D' O D' 1 L b) ab xy , 1 # (a b) P M D D D Q D N N a b a b U 1 R (a 1 R = 1 RSxy # D' ( xy)UV , D D T W D STxy # D xVW , D STxy # 2 #1

= 12 1 (a D

2

2

2

2

2

b) x 2 2

UV W

x3 a b x4 x3 y ( a b ) x 4 . # 5 , # 253 2 35 4 6 24 Required general solution z = .1(y # ax) + .2(y #bx) + (1/6) × x3y # (a + b) × (x4/24),

= y5

Ex. 5. Solve (a) (2 D 2 – 5 DD∗ 2 D ∗2 ) z , 24( y – x). (b) ( D 2 3DD∗ 2 D∗2 ) z , x

[Meerut 1996]

y.

(c) (! 2 z / !x2 ) 3(! 2 z / !x !y) 2(! 2 z / !y 2 ) , 2 x 3 y. (d) (! 2 z / !x 2 ) 3(! 2 z / !x !y) 2(! 2 z / !y 2 ) , 6( x

y).

(e) (! 2 z / !x 2 ) – (! 2 z / !y 2 ) , x – y. Sol. (a) Here auxiliary equation is

2m 2 – 5m 2 , 0

so that

m , 2, 1/ 2. +

Now, P.I. ,

C.F. = . 1 ( y 2 x) .2 (2 y x ), .1 , .2 being arbitrary functions. 1 2

2

2 D – 5 DD ∗ 2 D ∗

24( y – x ) , 24

1 % 5D ∗ 2 D ∋∋1 # ( 2D 2

D∗2 ∃ & D 2 &)

( y – x)

Created with Print2PDF. To remove this line, buy a license at: http://www.software602.com/

Homogenous linear partial differential equations with constant coefficients –1

D∗2 ∃ 12 % 5D ∗ 12 ∃ ( y – x) , 2 ∋ 1 ... & ( y – x) , 2 2 & & 2 D D ) D ( ) D

4.21

,

12 % 5D ∗ ∋1 – 2D D 2 ∋(

,

% x3 ∃ 5 x ∃ 12 % 3x ∃ % x2 ∃ 12 % 5 ∃ 12 % , y – x , y , 12 y 5 18 5 y – x 1 ∋ & ∋ & ∋ & ∋ 2 5 3& 2 ) D2 ( 2 ) ∋( 2 &) 2D ) D 2 ( ( ) D2 (

5 7 8 D∗( y – x ); :( y – x ) 2 D < =

z , .1 ( y 2 x) 0 2 (2 y x) 6 x 2 y 3x3 .

Hence the required general solution is (b)Try as in part (a).

Ans. z , .1 ( y – 2 x) .2 ( y – x) # (1/ 3) 5 x3 (1/ 2) 5 x 2 y

(c) Try yourself

Ans. z , .1 ( y – x) .2 ( y – 2 x) (3 / 2) 5 x2 y – (7 / 6) 5 x3 [Ans. z , .1 ( y – x) .2 ( y # 2 x) 3x2 y – 2 x3

(d) Try as in part (c).

Ans. z , .1 ( y x) .2 ( y # x) (1/ 6) 5 x3 – (1/ 2) 5 x2 y

(e) Try yourself.

Ex. 6. Solve ( D 2 – 6 DD ∗ 9 D∗2 ) z , 12 x 2 36 xy. [Meerut 1994, Bhuj 1999, Jabalpur 2003] ( D – 3D ∗) 2 z , 12( x 2

3 xy).

so that

m = 3, 3.

Sol. Re-writing the given equation, we get Its auxiliary equation is

2

(m – 3) = 0

C.F. = .1 ( y 3x) x.2 ( y 3x ), .1 , .2 being arbitrary functions.

+

Now, P.I.=

1 (D – 3D ∗) 2

12( x 2

3 xy) , 12

1 2

D (1 – 3 D∗ / D) 2

( x2

3 xy )

[Take D common as power of y is less than that of x] =

–2 12 % 3D∗ ∃ 12 % D∗ ∃ 2 1– ... & ( x 2 3xy ) & ( x 3xy ) , 2 ∋1 6 2 ∋ D ) D D ( D ( )

[Retain upto D ∗ as maximum power of y in ( x 2 3xy ) is one]

6 8 12 7 D ∗( x 2 3xy ); , 2 : x 2 3xy D = D
0, xy > 1 of the equation % 2 z / %x%y 1/( x ! y) (2 y ) /( x ! y ) on the hyperbola xy = 1. [Meerut 2007 Delhi Maths (H) 1999, 2006 Himanchal 1997, K.U. Kurukshatra 1999] Sol. Comparing the given equation with L(z) = f(x, y), we have a = b = c = 0 and f(x, y) = 1/(x + y). Hence the adjoint operator M of the operator L is given by M Α %2/%x%y. y A(1/ΤΛ Τ ) M N So Green’s function can be taken as w = 1. ...(1) P(ΣΛ Τ ) such that z = 0, p

y =Τ

In the present problem, the values of z and %z/%x (=p) are given by z = 0, %z/%x = (2y)/(x + y), ...(2) along the curve C, which is hyperbola. xy = 1. ...(3)

dxdy x = Σ

MΙ NΙ

O

Fig. 2.

C B (ΣΛ ΜΩΣ7 x

Created with Print2PDF. To remove this line, buy a license at: http://www.software602.com/

8.40

Classification of P.D.E Reduction to Canonical or normal forms Riemann Method

Then we wish to find the solution of given equation at the point P (Σ, Τ) agreeing with these boundary conditions. Through P we draw PA parallel to the x-axis and cutting xy = 1 in the point A and PB parallel to the y-axis and cutting xy = 1 in B. Then region enclosed by xy = 1, x = Σ, y = Τ is denoted by S. Now, we know that (refer equation (10) of Art 8.13.) [ z]P = [wz]A &

z

AB

z z

[z]P =

or

z

Now,

AB

z FGH zz z z RSTz

wz (ady & bdx) !

2y dx = 2 x!y

zz

AB

B

A

AB

2y dx ! x!y

IJ K

%w %z dy ! w dx ! %y %x

zz

S

wf dxdy

1 dxdy , by (1) and (2). x!y

S

xy dx 2 x ! xy 2

z

Σ

1/ Τ

...(4)

1 dx = 2 {tan&1 Σ & tan&1 (1/Τ)} 1! x2

...(5)

UV W

Τ 1 dy dx 1 dxdy = Σ , ...(6) x 1/ Τ y 1/ x x ! y S 1! x since to integrate over area bounded by PABP, we first integrate along the strip MNN Ι M Ι by fixing x and varying y from y = 1/x at M Ι to y = Τ at M and then integrate from A to P (keeping y fixed) by varying x from x = 1/Τ to x = Σ. Evaluating the double integral on R.H.S. of (6) by the usual rule,

and

zz z

S

=

Σ

1/ Τ

1 dxdy = x!y

z

Σ

Τ

log( x ! y) 1/ x dx

1/ Τ

z z

Σ

[log( x ! Τ) & log( x ! 1 / x)] dx

1/ Τ

2

{log( x ! Τ) & log(1 ! x ) ! log x } dx

F 1 & 2x ! 1 I dx GH x ! Τ 1 ! x x JK |R F I F I = Σ {log (Σ + Τ) & log (1 + Σ ) + log Σ)} & 1 Slog G 1 ! ΤJ & log G1 ! 1 J ! log 1 ΧΕ Τ |T H Τ K H Τ K ΤΓ 2 = {log( x ! Τ) & log(1 ! x ) ! log x } x

Σ

&

1/ Τ

Σ

1/ Τ

x

2

2

2



: 2 Τ ; &

ϑ

Σ

FG H

IJ K

Σ Τ (Σ ! Τ) &1 &1 &1 1 = Σ log Σ(Σ ! Τ7 = Σ log Σ(Σ ! Τ7 ! Τ log 2 & 2 tan Σ & tan 2 2 & 2 tan x & Τ log( x ! Τ)

...(7)

[z]P = Σ log Σ (Σ ! 2Τ) ! Τ log Τ (Σ ! 2Τ) . 1! Σ 1! Τ

...(8)

1! Σ

1/ Τ

Using (5) and (7), (4) reduces to

1! Σ

Τ

1! Τ

Replacing Σ and Τ by x and y respectively in (8), the value of z (i.e., solution of the given x( x ! y) y ( x ! y) equation) at any point (x, y) is given by z = x log . 2 ! y log 1! x 1 ! y2

Ex. 2. Prove that, for the equation (%2z/%x%y) + (z/4) = 0, the Green’s function is w (x, y ; Σ, Τ) = J0 ( x & Σ)(y & Τ) , wheree J0 (z) denotes Bessel’s function of the first kind of order zero. [Himanchal 1998, 2004, Kerala 2001; Kurukshetra 2000; Nagpur 2000, 03, 05 Delhi Maths Hons. 2004, 07, 09] Sol. Here L(z) = (%2z/%x%y) + (z/4) = 0. ...(1) 2 2 1 L Α % !a % !b % !c = % ! z . 3 a= 0, b = 0, c = z/4. ...(2) %x%y %x %y %x%y 4 * x

F 1 & 2x GH x ! Τ 1 ! x

2

!

1 x

I JK

2 x 2 x2 (x ! Τ7 & Τ (1 ! x ) & 1 Τ 2 & !1 = &2 !1 & ! 2 2 x!Τ x ! Τ 1 ! x2 x ! Τ 1! x 1! x

Created with Print2PDF. To remove this line, buy a license at: http://www.software602.com/

Classification of P.D.E Reduction to Canonical or normal forms Riemann Method

M Α (%2 / %x %y) ! (1/ 4)

So the adjoint operator M to the operator L is given by

w = J0 ( x & Σ)(y & Τ) .

Given,

0.

...(3) ...(4)

( y & Τ) %w J0Ι = %x 2 ( x & Σ)

From (4)

8.41

...(5)

( y & Τ) ( x & Σ) %2 w = 1 1 ∋ J 0ΙΙ J0Ι + %y%x 4 ( x & Σ)( y & Τ) 2 ( x & Σ) 2 ( y & Τ)

From (5),

R| S| T

U| V| W U| 1 R| 1 So (3) and (6) 3 Mw = S J ΙΙ! JΙ ! J V. 4| |W T ( x & Σ)( y & Τ) Now, Bessel’s eqution of order zero is given by %2 w 1 1 = J 0ΙΙ! J 0Ι . 4 %y%x ( x & Σ)( y & Τ)

or

0

x2yΞ + xyΙ + x2y = 0

0

or

...(6) ...(7)

0

yΞ + (1/x) × yΙ + y = 0.

...(8)

Mw = 0, by (7)

...(9)

Since y = J0{ ( x & Σ)(y & Τ) } is a solution of (8), we get

J0ΙΙ !

1 J0Ι ! J0 = 0 ( x & Σ)(y & Τ)

or

Again, (5) 3 (%w/%x) = 0 = bw when y = Τ, as b = 0 ...(10) Similarly, (%w/%y) = 0 = aw when x = Σ, as a = 0 ...(11) Finally, when x = Σ, y = Τ, w = J0(0) = 1. ...(12) Since w satisfies four properties (9), (10), (11) and (12) of a Green’s function, it follows that w must be a Green’s function of the given equation (1).

FG H

2 Ex. 3. Prove that for the equation % z ! 2

IJ K

%z ! %z = 0, the Green’s function is %y%x x ! y %x %y ( x ! y){2 xy ! (Σ & Τ) ( x ! y) ! 2ΣΤ} w(x, y ; Σ, Τ) = . 3 (Σ ! Τ) Hence find the solution of the differential equation which satisfies the conditions z = 0, %z/%x = 3x2 on y = x. [Bangalore 2003, Himanchel 2001; Kurukshetra 2004; Delhi Maths (H) 2001,05,11] 2 Sol. Compare the given equation with L(z) = f(x, y) where L Α % ! a % ! b % ! c , we find %y%x %x %y a = 2/(x + y), b = 2/(x + y), c = 0, f(x, y) = 0. ...(1) So the adjoint operator M to the operator L is given by

Mw Α w(x, y ; Σ, Τ) =

Given (3) 3

%w %x

(3) 3

%w %y

(5) 3

2

FG H

IJ K

FG H

IJ K

%2 w % 2 % 2 & w & w . %y%x %x x ! y %y x ! y

( x ! y ) { 2 xy ! ( Σ & Τ) ( x & y ) ! 2ΣΤ }

. ( Σ ! Τ) 3 2 xy ! (Σ & Τ)(x & y) ! 2ΣΤ ! 6 x ! y76Κ y ! Σ & Τ7 = . (Σ ! Τ)3 2 xy ! (Σ & Τ) (x & y) ! 2ΣΤ ! 6 x ! y76Κ y & Σ ! Τ7 = . 3 (Σ ! Τ)

% w 2 y ! Σ & Τ ! 2 x & Σ ! Τ ! 2 6 x ! y7 = %x %y (Σ ! Τ)3

4 ( x ! y) . (Σ ! Τ)3

...(2) ...(3) ...(4) ...(5) ..(6)

Created with Print2PDF. To remove this line, buy a license at: http://www.software602.com/

8.42

Classification of P.D.E Reduction to Canonical or normal forms Riemann Method

Using (6) and (3), (2) reduces to

RS T

UV W

RS T

UV W

4( x ! y) % 2 xy ! (Σ & Τ)( x & y) ! 2ΣΤ & 2 % 2 xy ! (Σ & Τ)( x & y) ! 2ΣΤ 3 & 2 %x %y 6Σ!Τ73 (Σ ! Τ) 6Σ!Τ73 4 ( x ! y) 2(2 y ! Σ & Τ) ! 2 (2 x & Σ ! Τ7 = & = 0. ...(7) (Σ ! Τ)3 6Σ ! Τ73 2 xΤ! (Σ & Τ)( x & Τ) ! 2ΣΤ ! ( x ! Τ)(2Τ! Σ& Τ7 %w At y = Τ, = , by (4) %x 6Σ!Τ73 2{ x (Σ ! Τ) ! Τ2 ! ΣΤ} = . ...(8) 6Σ ! Τ73 2 {2 xy ! (Σ & Τ)( x & y) ! 2ΣΤ} From (1) and (3), bw = . ...(9) 6Σ ! Τ73 So at y = Τ, (9) reduces to 2 { 2 xΤ! ( Σ & Τ) ( x & y ) ! 2 ΣΤ} 2 { x (Σ ! Τ) ! Τ2 ! ΣΤ} bw = = . ...(10) 6Σ ! Τ73 6Σ ! Τ73 From (8) and (10), %w/%x = bw when y = Τ. ...(11) Similarly, %w/%y = aw when x = Σ. ...(12)

Mw =

From (3), when x = Σ, y = Τ, we get

w=

(Σ ! Τ){ 2ΣΤ ! ( Σ & Τ) 2 ! 2Τ} 6Σ ! Τ73

= 1.

...(13)

Since w satisfies four properties (7), (11), (12) and (13) of a Green’s function, it follows that w must be a Green’s function of the given equation. To find the solution of the equation. In the present problem, the values of z and %z/%x (= p) are given by y=x z=0 and %z/%x = 3x2. ...(14) y B(ΣΛ Σ ) along the line AB, y = x. ...(15) Then we wish to find the solution of given equation at the x =Σ y =Τ point P(Σ, Τ) agreeing with these boundary conditions. Through A (Τ Λ Τ 7 P(ΣΛ Τ ) P we draw PA parallel to the x-axis and cutting y = x in the point A and PB parallel to the y-axis and cutting y = x in B. Then triangular region enclosed by straight lines y = x, y = Τ and x = Σ x O is denoted by S. Then we know that (refer equation (10) of Art Figure-3 8.13). [z]P = [wz]A &

ϑAB

: %w %z ; wz ( ady & bdx ) ! ϑ < z dy ! w dx = ! ϑϑ wf dxdy . AB > %y S %x ?

Now on line AB, from (3),

w=

4 x ( x 2 ! ΣΤ) , as y = x. (Σ ! Τ)2

..(16) ...(17)

Using (1), (14) and (17), (16) reduces to B 4 x ( x 2 ! ΣΤ) Σ 12 [z]P = ϑ A (Σ ! Τ)3 3x 2 dx (Σ ! Τ)3 ϑΤ ( x5 ! ΣΤx3 )dx Σ

=

+ 12 ∗ x 6 x4 + 12 ∗ Σ6 & Τ6 ΣΤ 4 ! ΣΤ − ! (Σ & Τ4 )− , 3 , 6 3 4 /Τ (Σ ! Τ) . 6 4 (Σ ! Τ) . /

Υ

ς

(Σ ! Τ)&3 2(Σ3 ! Τ3 ) (Σ3 & Τ3 ) ! 3ΣΤ (Σ2 & Τ2 )(Σ2 ! Τ2 )

Created with Print2PDF. To remove this line, buy a license at: http://www.software602.com/

Classification of P.D.E Reduction to Canonical or normal forms Riemann Method

Υ

8.43

ς

(Σ ! Τ)&3 2(Σ3 ! Τ3 ) (Σ & Τ) (Σ2 ! Τ2 ! ΣΤ) ! 3ΣΤ(Σ & Τ)(Σ ! Τ)(Σ2 ! Τ2 )

Υ

ς

(Σ ! Τ)&3 (Σ & Τ) 2(Σ 3 ! Τ3 ) (Σ 2 ! Τ2 ) ! 2ΣΤ(Σ3 ! Τ3 ) & 3ΣΤ (Σ ! Τ)(Σ2 ! Τ2 )

Υ

ς

(Σ ! Τ)&3 (Σ & Τ) 2(Σ3 ! Τ3 ) (Σ 2 ! Τ2 ) ! 6ΣΤ(Σ ! Τ) (Σ 2 ! Τ2 ) &3ΣΤ(Σ ! Τ)(Σ2 ! Τ2 ) ! 2ΣΤ( Σ3 ! Τ3 )

Υ

ς

Υ

ς

(Σ ! Τ)&3 ( Σ & Τ) ∗2( Σ2 ! Τ2 ) Σ 3 ! Τ3 ! 3ΣΤ (Σ ! Τ) &ΣΤ 3( Σ ! Τ) ( Σ2 ! Τ2 ) & 2 ( Σ3 ! Τ3 ) + . /

Υ

(Σ ! Τ)&3 (Σ & Τ) 2(Σ2 ! Τ2 )(Σ ! Τ)3 & ΣΤ(Σ ! Τ)3 or

[z]P

ς

(Σ & Τ)(2Σ2 ! 2Τ2 & ΣΤ)

2Σ3 ! 3ΣΤ2 & 3Σ2Τ & 2Τ3 .

....(18)

Replacing Σ and Τ by x and y respectively in (18), the value of z (i.e., solution of the given equation) at any point (x, y) is given by z = 2x3 + 3xy2 – 3x2y – 2y3. Ex.4. Obtain the solution of % 2 z / %x%y 1/( x ! y) such that z = 0, p = 2 y /( x ! y) on y = x. [Delhi Maths (H) 1998]

% 2 z / %x%y 1/( x ! y) ,

Sol. Here we are solve z=0

where

... (1)

p %z / %x = 2 y /( x ! y)

and

on

y=x

... (2)

Here the given curve C is straight line y = x. Then we wish to find the solution of (1) at P(Σ, Τ) agreeing with boundary conditions (2). Through P we draw PA parallel to the x-axis and cutting y = x at the point A and PB parallel to the y-axis and cutting y = x in B. The triangularregion enclosed by stright lines y = x, y = Τ, x = Σ is denoted by S (draw figure as shown in figure 3 of solved Ex. 3). Then we know that (refer particular case II of Art. 8.13). [z]P

[ z ]A !

Comparing (1) with % 2 z / %x %y

%z dx ! AB %x

ϑ

ϑϑ

S

f ( x, y) dx dy

... (3)

f ( x, y) 1/( x ! y) .

f ( x, y) , here

Since A lies on given curve AB and it is given that z = 0 on AB, hence

%z / %x

From (2),

2 y /( x ! y)

on

AB.

%z / %x 2 x /( x ! x ) = 1 so that Using the above facts, (3) reduces to [ z ]P

ϑ

Σ

Τ

dx !

ϑ

Σ

x

Β ∆ ΤΦ

ϑ

x y

Χ 1 dy Ε dx = Σ & Τ ! Τx!y Γ Σ

i.e.

y

[ z] A

x

y = x.

on

ϑ

Σ

Τ

∗. log( x ! y) +/

x

dx y Τ

0.

Σ&Τ!

ϑ

Σ

Τ

[log(2 x ) & log( x ! Τ)] dx

Σ Σ: 1 ∗ 2Σ Τ 2x + 1 ; & Τ log1 & dx = Σ & Τ ! , x log = x dx = Σ & Τ ! Σ log − & Τ< & Τ x!Τ x ! Τ /Τ Σ!Τ . > x x!Τ? 2Σ 2Σ 2Τ Σ = Σ & Τ ! Σ log & Τ Ψ log ( x ! Τ) ΖΤ Σ & Τ ! Σ log ! Τ log Σ! Τ Σ!Τ Σ!Τ Replacing Σ and Τ by x and y respectively in the above equation, the value of z (i.e., solution of (1) at any point (x, y) is given by z x & y ! x log {2x /( x ! y)} ! y log{2y /( x ! y)} .

ϑ

ϑ

Created with Print2PDF. To remove this line, buy a license at: http://www.software602.com/

8.44

Classification of P.D.E Reduction to Canonical or normal forms Riemann Method

8.15. Riemann-Volterra method for solving the Cauchy problem for the onedimensional wave equation The entire procedure of solution will become clear from the following solved examples. Ex. 1. Using Riemann-Volterra method, solve , 2 z / ,x2 ∃ ,2 z / ,y2 , when z, ,z / ,x , ,z / ,y are prescribed along a curve C in the xy-plane

, 2 z / ,x2 # , 2 z / ,y2 ∃ 0

Sol. Given

or

... (1)

r #t ∃ 0

x+y=

Comparing (1) with Rr & Ss & Tt & f ( x, y, z, p, p) ∃ 0 , here R = 1, S = 0 and T = – 1. Hence the > -quadratic equation R> 2 & S> & T ∃ 0 reduces to > 2 # 1 ∃ 0 so that > ∃ 1, # 1 . The corresponding characteristic equations of (1) are given by dy / dx & 1 ∃ 0 dy / dx # 1 ∃ 0 and y Integrating these, x + y = C1 and x – y = C2, ... (2) P(%Α ∋ ) ∋ which are characteristics of (1) and these are two families of %– = straight lines. Let P(%, ∋) be any point in xy-plane. We now y x– obtain characteristics of (1) passing through P. So putting x ∃ % A C and y ∃ ∋ in (2), we have C1 ∃ % & ∋ and C2 ∃ % # ∋ . Hence B the characteristics of (1) passing through P are given by x O x& y ∃ %&∋ x # y ∃ % # ∋, and ... (3) which have been shown by straight lines PB and PA respectively in the figure. Let the characteristics PA and PB cut the given curve C in A and B respectively. Let C? denote the closed curve PABP (which is made up of straight line PA, curve C (i.e. AB) and straight line BP). Let S be the region enclosed by C? . Integrating both sides of (1) over S, we have .≅ , 4 ,z 5 , 4 ,z 5 /≅ 4 ,2 z , 2 z 5 or 66 2 # 2 77 dx dy ∃ 0 0 6 7# 6 7 1dx dy ∃ 0 S ≅ ,x : ,x ; ,y : ,y ; ≅ S ,y ; 2 3 : ,x 4 ,z ,z 5 or dx & dy 7 ∃ 0 , by Green’s theorem.* C? 6 ,x ; : ,y %&∋

−−

−−



4 ,z 4 ,z 4 ,z ,z 5 ,z 5 ,z 5 dx & dy 7 & dx & dy7 & dx & dy 7 ∃ 0 6 6 6 C : ,y BP : ,y PA : ,y ,x ; ,x ; ,x ; Equation of BP is x & y ∃ % & ∋ and hence dx = – dy on BP. Similarly, equation of PA is x # y ∃ % # ∋ and hence dx ∃ dy on PA A. Using these facts in the above equation, we get

or





4 ,z 5 ,z dx & dy 7 & 6 ,x ; : ,y



4 ,z ,z 5 6 dx & dy 7 # , y , x ; :

C

or or



C



C



,z ,z 5 6 # dy # dx 7 + B : ,y ,x ;





P

B

P4

dz &



A

P



dz ∃ 0 , as dz ∃

A4

P

,z ,z 5 6 dy & dx 7 ∃ 0 ,x ; : ,y

,z ,z dx & dy ,x ,y

4 ,z ,z 5 6 dx & dy 7 – ( zP # zB ) & (z A # zP ) ∃ 0 ,x ; : ,y

* Green’s theorem. Let C ? be a closed curve bounding the region S on xy-plane and u ( x, y ), v ( x, y ) be differential functions in S and continusou on C ?, then



C?

(udx & vdy ) ∃

−−

S

4 ,v ,u 5 6: ,x – ,y 7; dx dy

Created with Print2PDF. To remove this line, buy a license at: http://www.software602.com/

Classification of P.D.E Reduction to Canonical or normal forms Riemann Method

8.45

4 ,z ,z 5 1 1 ... (4) ( zA & zB ) & 6 dx & dy 7 , ,x ; 2 2 C : ,y which is the required solution of (1) at any point P. Ex. 2. Solve one dimensional wave equation by Riemann Volterra method. [Kurukshetra 2001; Delhi Maths (H) 1996] Β



zP ∃

or Solve homogeneous one-dimensional wave equation ,2 z / ,x 2 ∃ (1/ c 2 ) Χ (,2 z / ,t 2 ) , when

z, ,z / ,x, ,z / ,t are prescribed along a curve C. Sol. Given Let y be a new variable such that

,2 z / ,x 2 ∃ (1/ c2 ) Χ (,2 z / ,t 2 ) y = ct

, 2 z / ,x2 # , 2 z / ,y2 ∃ 0

Then (i) becomes

... (i) ... (ii)

or

... (iii)

r #t ∃ 0

for which z, ,z / ,x and ,z / ,y are now prescribed along C. Proceed with (iii) as we did in solved Ex. 1 upto equation (4). Ex. 3. Find z (x, y) such that , 2 z / ,x2 ∃ ,2 z / ,y2 and z ∃ f ( x) and ,z / ,y ∃ g( x) on y = 0. [Kanpur 2003; Delhi Maths (H) 2000, 08] Sol. Given

, 2 z / ,x2 # , 2 z / ,y2 ∃ 0

z( x,0) ∃ f ( x) ,

where

i.e.,

(,z / ,y) y∃ 0 ∃ g( x)

and

i.e.,

or

r #t ∃ 0,

... (1)

z ∃ f ( x)

on

y=0

i.e. x – axis ... (2)

,z / ,y ∃ g( x)

on

y=0

i.e. x – axis ...(3)

Comparing (1) with Rr & Ss & Tt & f ( x, y, z, p, q) ∃ 0 , here R = 1, S = 0 and T = – 1. Hence the 2 2 > -quadratic equation R> & S > & T ∃ 0 reduces to > # 1 ∃ 0 so that > ∃ 1, # 1. The corresponding dy / dx & 1 ∃ 0 dy / dx # 1 ∃ 0 characteristic equations of (1) are given by and

y



P(%Α ∋ ) x+

Integrating these x & y ∃ C1 and x # y ∃ C2 ... (4) which are the characteristics of (1) and these are two families of straight lines. Let P(%, ∋) be any point in xy-plane. We now

%–

y=

%&

x–

y=

obtain characteristics of (1) passing through P. So putting x ∃ % and y ∃ ∋ in (4), we get C1 ∃ % & ∋ and C2 ∃ % # ∋ . Hence the characteristics of (1) passing through P are given by x O A (% – ∋ , 0) x& y ∃ %&∋ and x # y ∃ % # ∋ ... (5) B (% + ∋ , 0) which have been shown by straight lines PB and PA respectively in the figure. Let the characteristics PA and PB cut given curve (here y = 0 i.e. x-axis) in A (% # ∋, 0) and B (% & ∋, 0) respectively. Let C? denote the closed curve PA BP (which is made up of straight lines PA, AB and BP). Let S be the region enclosed by C? . Integrating both sides of (1) over S, we have ∋

−−

S

4 , 2 z ,2 z 5 66 2 # 2 77 dxdy ∃ 0 ,y ; : ,x

or



or



−−

or

S

.≅ , 4 ,z 5 , 4 ,z 5 /≅ 0 6 7 # 6 7 1 dx dy ∃ 0 ≅2 ,x : ,x ; ,y : ,y ; ≅3

4 ,z ,z 5 6 dx & dy 7 ∃ 0 , by Green’s theorem ,y ,x ;

C? :

4 ,z ,z 5 6 dx & dy 7 & ,y ,x ;

AB :



4 ,z ,z 5 6 dx & dy 7 + ,y ,x ;

BP :

4 ,z ,z 5 6 dx & dy 7 = 0 PA : ,y ,x ;



Created with Print2PDF. To remove this line, buy a license at: http://www.software602.com/

8.46

Classification of P.D.E Reduction to Canonical or normal forms Riemann Method

On AB (i.e., x-axis), y = 0 so that dy = 0. Also, from (3), ,z / ,y = g (x) on y = 0. On BP (i.e., x + y = % & ∋ ), dx = – dy. Similarly, on PA (i.e., x # y ∃ % # ∋ ), dx = dy. Using these facts, the above equation reduces to



B



B

A

or or

A



P

,z ,z dy & dx ∃ dz ,y ,x



g( x ) dx #



P4

4 ,z ,z 5 6 dy & dx 7 ∃ 0 ,x ; : ,y

,z ,z 5 6 # dy # dx 7 + B : ,y ,x ;

g( x )dx &

B

dz &



A

P

dz ∃ 0 ,

A

P

as

1 1 B ( zA & zB ) & g( x ) dx ... (6) A 2 2 A From (2), z = f (x) on y = 0 (i.e., x-axis). Since x-coordinates of A and B are % # ∋ and % & ∋



B

g( x ) # ( zP # zB ) & z A # zP ∃ 0



zP ∃

or

respectively, it follows that z A ∃ f (% # ∋) and zB ∃ f (% & ∋) . Hence (6) reduces to 1 1 %&∋ { f (% # ∋) & f (% & ∋)} & g( x ) dx ... (7) 2 2 %#∋ Replacing % and ∋ by x and y respectively in (7), the value of z (i.e., solution of (1) at any point P (x, y)) is given by



zP ∃

1 1 x&y { f ( x # y) & f ( x & y)} & g(u) du 2 2 x# y Ex. 4. Find the solution of one-dimensional non-homogeneous wave equation



z ( x, y) ∃

, 2 z / ,x2 # ,2 z / ,y2 & f ( x, y) ∃ 0 by Riemann-Vatterra method. [A.M.I.E. 2005; Delhi Maths (H) 1998, 2002; Kanpur 1998] Sol. Given

, 2 z / ,x2 # ,2 z / ,y2 & f ( x, y) ∃ 0

r # t & f ( x, y) ∃ 0 . ... (1)

or

Suppose that z, ,z / ,x and ,z / ,y are prescribed along a given curve C. Comparing (1) with Rr & Ss & Tt & f ( x, y, z, p, q) ∃ 0 , here R = 1, S = 0, T = – 1 and so > -quadratic equation R> 2 & S> & T ∃ 0 reduces to > 2 # 1 ∃ 0 giving > ∃ 1, # 1 . The corresponding characteristic equations dy/dx + 1 = 0 and dy/dx – 1 = 0 give, on integration, x + y = C1 and x – y = C2 ... (2) which are characteristics of (1). Draw figure as in solved Ex. 1. Let P(%, ∋) be any point in xyplane. Then characteristics of (1) passing through P(%, ∋) are given by

x& y ∃ %&∋ x#y ∃ %#∋ and ... (3) which have been shown by straight lines PB and PA respectively. Let the characteristics PA and PB cut the given curve C in A and B respectively. Let C? denote the closed curve PABP and let S denote the region enclosed by C? . Integrating both sides of (1) over S, we have ≅. , 4 ,z 5 , 4 ,z 5≅/ 0 6 7# 6 71 dx dy & ≅2 ,x : ,x ; ,y : ,y ;≅3

−−

S



or or



C

4 ,z ,z 5 dx & dy 7 + 6 C ? : ,y ,x ;

4 ,z ,z 5 6 dx & dy 7 & , y , x ; :

−−

4 ,z ,z 5 6 dx & dy 7 + BP : ,y ,x ;



S

−−

S

f ( x, y) dx dy ∃ 0

f ( x, y) dx dy = 0, by Green’s theorem 4 ,z ,z 5 6 dx & dy 7 + PA : ,y ,x ;



−−

S

f ( x, y ) dxdy ∃ 0

Created with Print2PDF. To remove this line, buy a license at: http://www.software602.com/

Classification of P.D.E Reduction to Canonical or normal forms Riemann Method

8.47

Equation of BP is x & y ∃ % & ∋ and so dx = – dy on BP. Similarly, equation of PA is x – y = % # ∋ and so dx = dy on PA A. Using these facts, the above equation reduces to



C

4 ,z ,z 5 6 dx & dy 7 & , y , x ; :



or

C

P

B

4 ,z ,z 5 6 # dy # dx 7 + , y , x ; :

4 ,z ,z 5 6 dx & dy 7 # , y , x ; :



P

B

dz &

− −

A P

A

P

4 ,z ,z 5 6: ,y dy & ,x dx7; &

−−

dz &



or

zP ∃

1 1 (z A & zB ) & 2 2

4 ,z

,z

5

−−

S

f ( x, y ) dx dy ∃ 0

f ( x, y) dx dy ∃ 0

S

1

− 6: ,y dx & ,x dy 7; & 2 −− C

−−

f ( x, y) dx dy ∃ 0

S

4 ,z ,z 5 6 dx & dy 7 # (zP # zB ) & zA # zP & , y , x ; :

or

C



S

f ( x, y) dx dy ∃ 0 ,

which is the required solution of (1) at any point P. Ex. 5. Solve , 2 z / ,x 2 # ,2 z / ,y2 ∃ 1 , when z (x, 0) = sin x, zy (x, 0) = x. Sol. Given

, 2 z / ,x2 # , 2 z / ,y2 # 1 ∃ 0

where

z( x, 0) ∃ sin x ,

and

zy (x, 0) = x,

i.e., i.e.,

or

r # t #1 ∃ 0

... (1)

z ∃ sin x

on

y ∃ 0 , i.e., x-axis ... (2)

,z / ,y ∃ x

on

y ∃ 0, i.e., x-axis ... (3)

Comparing (1) with Rr & Ss & Tt & f ( x, y, z, p, q) ∃ 0 , here R = 1, S = 0 and T = – 1. Hence the > -quadratic equation R> 2 & S> & T ∃ 0 reduces to > 2 # 1 ∃ 0 so that > ∃ 1, # 1 . The corresponding characteristic equations of (1) are dy/dx + 1 = 0 and dy/dx – 1 = 0 Integrating these, x + y = C1 and x – y = C2 ... (4) which are the characteristics of (1) and these are two families of straight lines Draw a figure as in solved Ex. 3. Let P(%, ∋) be any point in xy-plane. Putting x = %, y = ∋ in (4), we get C1 ∃ % & ∋ ,

C2 ∃ % # ∋ . Hence the characteristics of (1) passing through P are given by

x& y ∃ %&∋ x#y ∃ %#∋ and ... (5) which have been shown by straight lines PB and PA respectively in the figure. Let the characteristics PA and PB cut the given line y = 0 i.e., x-axis in A(% # ∋, 0) and B(% & ∋, 0) respectively. Let C? denote the closed curve PABP and let S denote the region enclosed by C? . Integrating both sides of (1) over S, we have

−−

S



or Now,

≅. , 4 ,z 5 , 4 ,z 5 ≅/ 0 6 7# 6 71 # 2≅ ,x : ,x ; ,y : ,y ; 3≅ 4 ,z ,z 5 6 dx & dy 7 # ,y ,x ;

C? :

−− dx dy ∃ 0 S

−− dx dy ∃ 0 , by Green’s theorem S

... (5)

−− dx dy = area of the triangle SAB = (1/2) × AB × perpendicular distance of P from AB S

= (1/2) × {% & ∋ # (% # ∋)} Χ ∋ ∃ ∋2 Hence (5) reduces to 4 ,z ,z 5 6 dx & dy 7 & AB : ,y ,x ;



4 ,z ,z 5 6 dx & dy 7 + BP : ,y ,x ;



4 ,z ,z 5 2 6 dx & dy 7 # ∋ ∃ 0 PA : ,y ,x ;



... (6)

Created with Print2PDF. To remove this line, buy a license at: http://www.software602.com/

8.48

Classification of P.D.E Reduction to Canonical or normal forms Riemann Method

On AB (i.e., x-axis), y = 0 so that dy = 0. Also from (3), ,z / ,y ∃ x on y = 0. On BP (i.e., x + y = % & ∋ ), dx = – dy. Similarly on PA (i.e., x – y = % # ∋ ), dx = dy. Using these facts, (6) reduces to



B

A

,z ,z 5 6 # dy # dx 7 + B : ,y ,x ;



P4



or or

x dx &

B

A

x dx #



P

B

dz &



A

P



A

P

4 ,z ,z 5 2 6 dy & dx 7 #∋ ∃ 0 , y , x : ;

dz # ∋2 ∃ 0 , as dz ∃

,z ,z dx & dy ,x ,y

1 1 B 1 ( zZ & zB ) & x dx # ∋2 ... (7) 2 2 A 2 A From (2), z = sin x on y = 0 (i.e., x-axis). Since x-coordinates of A and B are % # ∋ and % & ∋



B

x dx # ( zP # zB ) & zA # zP # ∋2 ∃ 0

or



zP ∃

respectively, it follows that z A ∃ sin(% # ∋) and zB ∃ sin(% & ∋) . Hence (7) reduces to 1 1 zP ∃ {sin (% # ∋) & sin (% & ∋)} + 2 2

or



%&∋

%#∋

x dx #

∋2 2

or

z P ∃ sin % cos ∋ #

1 ( 2 ∗%&∋ ∋2 x # 4 ) +%#∋ 2

zP ∃ sin % cos ∋ # (1/ 4) Χ {∆% & ∋)2 # (% # ∋)2 } # (1/ 2) Χ ∋2 zP ∃ sin % cos ∋ # %∋ # (∋2 / 2)

or

... (8)

Replacing % and ∋ by x and y respectively in (8), the value of z (i.e., solution of (1)) at any point (x, y) is given by z (x, y) = sin x cos y – xy – (y2/2). Ex. 6. A function z (x, y) satisfies the non-homogeneous equation ,2 z / ,x2 # ,2z / ,y2 & f ( x, y) ∃ 0 and the initial conditions z ∃ ,z / ,y ∃ 0 when y = 0. Show that (using Riemann - Volterra method) 1 f (u,v) du dv , z (x, y) = 2 Ε where Ε is the triangle cut off from the upper half of uv-plane by two characteristics through the point (x, y). [Delhi Maths (Hons) 2002, 07, 11; A.M.I.E. 2005; Amaravati 2003; Kanpur 1999; Rohilkhand 2004]

−−

Sol. Given , 2 z / ,x2 # ,2 z / ,y2 & f ( x, y) ∃ 0 where z (x, 0) = 0, i.e., z=0 and

(,z / ,y) y∃ 0 ∃ 0 ,

i.e.,

(,z / ,y) ∃ 0

or on

r # t & f ( x, y) ∃ 0 , ... (1) y = 0 (x-axis) ... (2)

on

y = 0 (x-axis) ... (3)

Comparing (1) with Rr & Ss & Tt & f ( x, y, z, p, q) ∃ 0 , here R = 1, S = 0 and T = – 1 and so -quadratic equation R> 2 & S> & T ∃ 0 reduces to > 2 # 1 ∃ 0 giving > ∃ 1, # 1 . The corresponding > characteristic equations are given by dy/dx + 1 = 0 and dy/dx – 1 = 0 Integrating these, x + y = c1 and x – y = c2, ... (4) which are characteristics of (1). Draw figure as in solved Ex. 3. Let P(%, ∋) be any point in xy-plane. We now obtain characteristics of (1) passing through P. So putting x ∃ % and y ∃ ∋ in (4), we get C1 ∃ % & ∋ and C2 ∃ % # ∋ . Hence the characteristics of (1) passing through P are given by

x& y ∃ %&∋ x#y ∃ %#∋ and ... (5) which have been shown by straight lines PB and PA respectively in the figure. Let the characteristics PA and PB cut y = 0 (i.e., x-axis) at A and B respectively. Let C? denote the closed curve PA BP and let Ε denote the triangular region enclosed by C? . Integrating both sides of (1) over Ε , we have

Created with Print2PDF. To remove this line, buy a license at: http://www.software602.com/

Classification of P.D.E Reduction to Canonical or normal forms Riemann Method

−−



or

4 , 4 ,z 5 , 4 ,z 5 5 6 6 7 # 6 7 7 dx dy & Ε 6 ,x : ,x ; ,y : ,y ; 7 : ;

4 ,z ,z 5 dx & dy 7 + 6 C ? : ,y ,x ;

4 ,z ,z 5 6 dx & dy 7 & AB : ,y ,x ;

−−

Ε

−−

Ε

8.49

f ( x, y) dx dy ∃ 0

f ( x, y) dx dy ∃ 0 , using Green’s theorem

4 ,z ,z 5 6 dx & dy 7 + BP : ,y ,x ;

4 ,z ,z 5 6 dx & dy 7 + Ε ( x, y) dx dy ∃ 0 ... (6) PA : ,y ,x ; On AB (i.e., x-axis), y = 0 so that dy = 0. Also, from (3), ,z / ,y ∃ 0 on y = 0. On BP (i.e., x & y ∃ % & ∋ ), dx = – dy. Similarly, on PA (i.e., x – y = % # ∋ ), dx = dy. Using these facts, (6) reduces to

or





P4

B

,z ,z 5 6 # dy # dx 7 + , y , x ; : #

or or





P

B

dz &



A

P

dz &

−−



A

P

Ε

−−



4 ,z ,z 5 6 dy & dx 7 & ,x ; : ,y

−− ( x, y) dxdy ∃ 0 Ε

f ( x, y)dxdy = 0, as dz ∃

– (zP – zB) + zA – zP +

−−

Ε

,z ,z dx & dy ,x ,y

f ( x, y) dx dy ∃ 0

... (7)

From (2), z = 0 on y = 0 (i.e., x-axis) and so zA = zB = 0, because A and B both lie on y = 0. Hence (7) becomes 2zP ∃

−−

Ε

f ( x, y) dxdy

z (x, y) =

or

which gives the value of z (i.e., solution of (1)) at any point (x, y)

1 2

−−

Ε

f (u, v) du dv ,

Ex.7. Find the solution of the non-homogeneous wave equation ,2z / ,x2 #(1/ c2)(,2z / ,t2) & f (x,t) ∃ 0 with initial conditions z (x, 0) = f(x), zt (x, t) = g (x). Sol. Let y be a new variable such that y=ct ... (1) Then the given problem may be re-written as

, 2 z / ,x2 # , 2 z / ,y2 & F( x, y) ∃ 0 with the modified initial conditions given below z (x, 0) = f (x), i.e., z = f (x)

or on

r # t & F ( x, y) ∃ 0 , y = 0,

i.e., x-axis

... (2) ... (3)

(,z / ,y)y∃ 0 = G (x), i.e., ,z / ,y ∃ G( x) on y = 0, i.e., x-axis ... (4) Here F (x, y) = f (x, t) and G (x) = (1/c) × g (x) ... (5) Comparing (2) with Rr & Ss & Tt & f ( x, y, z, p, q) ∃ 0 , here R = 1, S = 0 and T = – 1. Hence the 2 > -quadratic equation R> 2 & S> & T ∃ 0 reduces to > # 1 ∃ 0 so that > ∃ 1, # 1 . The corresponding characteristic equations of (1) are given by dy / dx & 1 ∃ 0 and dy/dx – 1 = 0 ... (6) Integrating these, x + y = C1 and x – y = C2, which are characteristics of (1). Draw a figure same as in Ex. 3. Let P(%, ∋) be any point in xy-plane. Then characteristics (2) passing through P(%, ∋) are x& y ∃ %&∋ x#y ∃ %#∋ and ... (7) which have been shown by straight lines PB and PA respectively in the figure. Here lines given by (7) cut x-axis (i.e., y = 0) in A (% # ∋, 0) and B (% & ∋, 0) respectively. Let C? denote the closed curve PABP and let S be the region enclosed by C? .

Created with Print2PDF. To remove this line, buy a license at: http://www.software602.com/

8.50

Classification of P.D.E Reduction to Canonical or normal forms Riemann Method

Integrating both sides of (2) over S, we have ≅. , 4 ,z 5

, 4 ,z 5 ≅/

−− 0≅2 ,x 6: ,x 7; # ,y 6: ,y 7;1≅3 S



or or

4 ,z ,z 5 6 dx & dy 7 + ,y ,x ;

C?:

4 ,z ,z 5 6 dx & dy 7 + AB : ,y ,x ;



dx dy &

−−

F ( x, y) dx dy ∃ 0

S

−− F(x, y) dx dy ∃ 0 , using Green’s theorem S

4 ,z ,z 5 dx & dy7 + 6 BP : ,y ,x ;



4 ,z ,z 5 6 dx & dy 7 + ,x ; : ,y



PA

−−

S

F( x, y) dx dy ∃ 0

... (8)

On AB (i.e., x-axis), y = 0 so that dy = 0. Also, from (4), ,z / ,y ∃ G( x) on y = 0. On BP (i.e., x + y = % & ∋ ), dx = – dy. Similarly, on PA (i.e., x – y = % # ∋ ), dx = dy. Using these facts, (8) reduces to



B

A

G( x ) dx &



P

B

4 ,z ,z 5 dy # dx 7 + 6# , y , x ; :



or

B

A



or

B

A



A4

,z ,z 5 6 dy & dx 7 + , y , x ; :

P

G( x )dx #



P

B

dz &



A

P

S

−− F (x, y) dx dy ∃ 0 −− F( x, y) dx dy = 0

dz &

G ( x ) dx # (zP # zB ) & z A # zP &

−− F(x, y) dx dy ∃ 0

S

S

1 1 1 B F ( x, y) dx dy ( zA & zB ) & G( x ) dx & ... (9) 2 2 A 2 S From (3), z = f (x) on y = 0 (i.e., x-axis). Since x-coordinates of A and B are % # ∋ and % & ∋ respectively, it follows that z A ∃ f (% # ∋) and zB ∃ f (% & ∋) . Hence (9) reduces to zP ∃

or

−−



1 1 %&∋ 1 z p ∃ { f (% – ∋) & f (% & ∋)} & G ( x) dx & F ( x, y) dx dy 2 2 %–∋ 2 s Replacing % and ∋ by x and y (= ct) respectively and using (5), (10) reduces to



1 1 z ( x, y ) ∃ { f ( x – y ) & f ( x & y )} & 2 2c

−−



x & ct x – ct

g (u )du &

1 2

−−

s

... (10)

f ( x, t )dx dt

Miscellaneous Problems on chapter 8 2

2

Ex. 1. , u / ,t ∃ c2 (, 2u / ,x2 ) is hyperbolic or parabolic. Classify it. Hint. See Art 8.1 Ans. Hyperbolic Ex. 2. The equation , 2 u / ,t 2 ∃ , 2 u / ,x 2 is (a) parabolic (b) hyperbolic (c) elliptic Sol. Ans. (b.) See Art 8.1.

(d) Nonw of these

Ex. 3. Classify and solve the following equation ,2 z / ,x2 ∃ x2 (, 2 z / ,y 2 ).

[Agra 2008]

[Agra 2007] [Bhopal 2010]

Created with Print2PDF. To remove this line, buy a license at: http://www.software602.com/

9 Monge’s Methods 9.1 INTRODUCTION The most general form of partial differential equation of order two is f(x, y, z, p, q, r, s, t) = 0. ...(1) It is only in special cases that (1) can be integrated. Some well known methods of solutions were given by Monge. His methods are applicable to a wide class (but not all) of equations of the form (1). Monge’s methods consists in finding one or two first integrals of the form u = !( v ), ...(2) where u and v are known functions of x, y, z, p and q and ! is an arbitrary function. In other words, Monge’s methods consists in obtaining relations of the form (2) such that equation (1) can be derived from (2) by eliminating the arbitrary function. A relation of the form (2) is known as an intermediate integral of (1). Every equation of the form (1) need not possess an intermediate integral. However, it has been shown that most general partial differential equations having (2) as an intermediate integral are of the following forms Rr + Ss + Tt = V and Rr + Ss + Tt + U(rt – s2) = V, ...(3) where R, S, T, U and V are functions of x, y, z, p and q. Even equations (3) need not always possess an intermediate integral. In what follows we shall assume that an intermediate integral of (3) exists. 9.2. MONGE’S METHOD OF INTERGRATING Rr + Ss + Tt = V. [Agra 2005; Delhi Maths (Hons) 2000, 02, 08, 09, 11; Garhwal 1994; Patna 2003; Kanpur 1997; Meerut 2000] Given Rr + Ss + Tt = V, ...(1) where R, S, T and V are functions of x, y, z, p and q. We know that p = z/ x, q = z/ y, r= s= Now, and

F zI = p , x H xK x x FG z IJ = q z = x H yK x y x 2

z 2

=

t=

2

and

s=

z

y

2

2

=

y

FG z IJ = q , %& H y K y &∋ ... (2) FH z IK = p &&(

z = y x y

x

y

dp = ( p / x ) dx # ( p / y )dy = rdx + sdy, using (2)

...(3)

dq = ( q / x)dx # ( q / y )dy = sdx + tdy, using (2)

...(4)

From (3) and (4), r = (dp ∃ s dy ) / dx and Substituting the values of r and s given by (5) in (1), we get

R

2

FH dp ∃ sdy IK + Ss + T FG dq ∃ sdx IJ = V dx H dy K

or

t = (dq ∃ s dx) / dy

...(5)

R(dp – sdy)dy + Ss dxdy + T(dq – sdx)dx = V dxdy

(Rdpdy + Tdqdx – Vdxdy) – s{R(dy)2 – Sdxdy + T(dx)2} = 0. ...(6) Clearly any relation between x, y, z, p and q which satisfies (6) must also satisfy the following two simultaneous equations or

9.1

Created with Print2PDF. To remove this line, buy a license at: http://www.software602.com/

9.2

Monge’s Methods

Rdpdy + Tdq dx – Vdxdy = 0. ...(7) 2 2 and (dy) – Sdxdy + T(dx) = 0. ...(8) The equations (7) and (8) are called Monge’s subsidiary equations and the relations which satisfy these equations are called intermediate integrals. Equation (8) being a quadratic, in general, it can be resolved into two equations, say dy – m1 dx = 0 ...(9) and dy – m2 dx = 0. ...(10) Now the following two cases arise : Case I. When m1 and m2 are distinct in (9) and (10). In this case (7) and (9), if necessary by using well known result dz = pdx + qdy, will give two integrals u1 = a and v 1 = b, where a and b are arbitrary constants. These give u1 = f1( v 1), ...(11). where f1 is an arbitrary function. It is called an intermediate integral of (1). Next, taking (7) and (10) as before, we get another intermediate integral of (1), say u2 = f2( v 2), where f2 is an arbitrary function. ...(12) Thus we have in this case two distinct intermediate integrals (11) and (12). Solving (11) and (12), we obtain values of p and q in terms of x, y and z. Now substituting these values of p and q in well known relation dz = pdx + qdy ...(13) and then integrating (13), we get the required complete integral of (1). Case II . When m1 = m2 i.e., (8) is a perfect square. As before, in this we get only one intermediate integral which is in Lagrange’s form Pp + Qq= R. ...(14) Solving (14) with help of Lagrange’s method (refer Art. 2.3, chapter 2), we get the required complete integral of (1). Remark 1. Usually while dealing with case I, we obtain second intermediate integral directly by using symmetry. However sometimes in absence of any symmetry, we find the complete integral with help of only one indetermediate integral. This is done with help of using Lagrange’s method. Remark 2. While obtaining an intermediate integral, remember to use the relation dx = pdx + qdy as explained below : (i) pdx + qdy + 2xdx = 0 can be re–written as dz + 2xdx = 0 so that z + x2 = c. (ii) xdp + ydq = dx can be re–written as xdp + ydq + pdx + qdy = dx + pdx + qdy or d(xp) + d(yq) = dx + dz so that xp + yq = x + z + c, on integration Remark 3. While integrating, we shall use the following types of calculations. In what follows, f and g are arbitrary functions and k and a are a constants. 1 2 2 (i) k f (t ) dt = g(t) (ii) k 1 f (t ) dt = g(t). (iii) k 2 f (t ) d(t ) = g(t2) t t k 1 1 =g 1 (iv) k f ( x # y) d ( x # y) = g(x + y). (v) k t 2 f 1 d 1 = 2 f t d t t t t 1t

z

z

z z

z

z

z

z

FH IK FH IK FH IK FH IK FH IK b g

k 2 2 k f (at 2 ) d (t 2 ) = 2 2 f (at ) d (at ) = g(at ) 2 (at ) t Proof of (vi). Putting at2 = u, and d(at2) = du we have k f (at 2 ) d (t 2 ) = k f (u) d (u) = g(u) = g(at2), as u = at2. u t2 Similarly, other results can be proved. In examination we shall not use substitution as explained above. With good practice, the students will be able to write direct results of integration very easily.

(vi)

z

z

Created with Print2PDF. To remove this line, buy a license at: http://www.software602.com/

Monge’s Methods

9.3

Important Note. For sake of convenience, we have divided all questions based on Rr + Ss + Tt = V in four types. We shall now discuss them one by one. 9.3. Type 1. When the given equation Rr + Ss + Tt = V leads to two distict intermediate intergrals and both of them are used to get the desired solution. Working rule for solving problems of type 1. Step 1. Write the given equation in the standard form Rr + Ss + Tt = V. Step 2. Substitute the values of R, S, T and V in the Monge’s subsidiary equations: R d p d y + T d q d x – V d x d y = 0 ...(1) R(dy)2 – Sdxdy + T(dx)2 = 0 ... (2) Step 3. Factorise (1) into two distinct factors. Step 4. Using one of the factors obtained in (1), (2) will lead to an intermediate integral. In general, the second intermediate integral can be obtained from the first one by inspection, taking advantage of symmetry. In absence of any symmetry, the second factor obtained in step 3 is used in (2) to arrive at second intermediate integral. You should use remark 2 of Art. 9.2 while finding intermediate integrals. Step 5. Solve the two intermediate integrals obtained in step 4 and get the values of p and q. Step 6. Substitute the values of p and q in dz = pdx + q dy and integrate to arrive at the required general solution. You should use remark 3 of Art. 9.2 while integrating dz = pdx + qdy. 9.4. SOLVED EXAMPLES BASED ON ART. 9.3. Ex. 1. (a) Solve r = a2t. [Agra 2008; Lucknow 2010; Patna 2003; Meerut 2008] (b) r = t. [Agra 2006] (c) Solve one-dimensions wave equation by Monge’s method:

2

y / dx 2 ) a 2 ( 2 y / t 2 ). [Meerut 2003]

Sol. (a) Given equation is r – a2t = 0. Comparing it with Rr + Ss + Tt = V, we have R = 1, S = 0, T = – a2, V = 0. Hence Monge’s subsidiary equations Rdpdy + Tdq dx – Vdxdy = 0 and R(dy)2 – S dxdy + T (dx)2 = 0 2 become dpdy – a dqdx = 0 ...(1) 2 2 2 and (dy) – a (dx) = 0. ...(2) Equation (2) may be factorised as (dy – adx) (dy + adx) = 0 Hence two systems of equations to be considered are dpdy – a2 dqdx = 0, dy – adx = 0. ...(3) and dpdy – a2 dqdx = 0, dy + adx = 0. ...(4) Integrating the second equation of (3), we get y – ax = c1. ...(5) Eliminating dy/dx between the equations of (3), we get dp – adq = 0 so that p – aq = c2. ...(6) Hence the intermediate integral corresponding to (3) is p – aq = !1(y – ax). ...(7) Similarly another intgermediate integral corresponding to (4) is p + aq = !2(y + ax)....(8) Here !1 and !2 are arbitrary functions. Solving (7) and (8) for p and q, we have p = (1/2) × {!2(y + ax) + !1(y – ax)} and q = (1/2a) × {!2(y + ax) – !1(y – ax)}. Substituting these values of p and q in dz = pdx + qdy, we get dz = (1/2) × {!2(y + ax) + !1(y – ax)}dx + (1/2a) × {!2(y + ax) – !1(y – ax)}dy = (1/2a) × !2(y + ax) (dy + adx) – (1/2a) × !1(y – ax)(dy – adx) Integrating, z = ∗2(y + ax) + ∗1(y – ax), ∗1, ∗2 being arbitrary functions.

Created with Print2PDF. To remove this line, buy a license at: http://www.software602.com/

9.4

Monge’s Methods

(b) This is a particular case of part (a). Here a = 1. (c) Refer part (a). Note that

2

y / x 2 ) r and

2

Ans. z = ∗2(y + x) + ∗1(y – x).

y / t2 ) t

Ex. 2. Solve r + (a + b)s + abt = xy. [Vikram 2003] Sol. Comparing the given equation with Rr + Ss + Tt = V, we have R = 1, S = a + b, T = ab, V = xy. The usual Monge’s subsidiary equations Rdpdy + Tqdx – Vdxdy = 0 and R(dy)2 – Sdxdy + T(dx)2 = 0. become dp dy + a b dq dx – xy dx dy = 0 ...(1) and (dy)2 – (a + b) dxdy + ab (dx)2 = 0. ...(2) Factorizing, (2) gives (dy – bdx) (dy – adx) = 0. Hence two systems to be considered are dp dy + ab dq dx – xy dx dy = 0, dy – b dx = 0. ...(3) and dp dy + ab dq dx – xy dx dy = 0, dy – a dx = 0. ...(4) Integrating the second equation of (3), y – bx = c1. ...(5) Eliminating dy/dx between the equations of (3), we get dp + a dq – xy dx = 0 or dp + a dq – x(c1 + bx) dx = 0, by (5) ...(6) Integrating (6), p + aq – (c1/2)x2 – (b/3)x3 = c2 or p + aq – (x2/2) (y – bx) – (b/3)x3 = c2, using (5) or p + aq – (1/2) × yx2 + (1/6) × bx3 = c2. ...(7) Using (5) and (7), the first intermediate integral corresponding to (3) is p + aq – (1/2) × yx2 + (1/6) × bx3 = !1(y – bx), !1 being an arbitrary function ...(8) Similarly, another intermediate integral corresponding to (4) is p + bq – (1/2) × yx2 + (1/6) × ax3 = !2(y – ax), !2 being an arbitrary functionJ ...(9) Solving (8) and (9) for p and q, we have p = (1/2) × x2y – (1/6) × (a + b)x3 + (a – b)–1 [a!2(y – ax) – b!1(y – ax)] and q = (1/6) × x3 + (a – b)–1 [!1(y – bx) – !2(y – ax)]. Substituting these values in dz = pdx + qdy, we get dz = (1/2) × x2ydx – (1/6) × (a + b)x3dx + (a – b)–1 [!2(y – bx)dx – !1(y – ax)dx] + (1/6) × x3dy + (a – b)–1 [!1(y – bx)dy – !2(y – ax)dy] 2 3 or dz = (1/6) × (3x ydx + x dy) – (1/6) × (a + b) x3dx – (b – a)–1 [!2(y – bx)dx – !1(y – ax)dx] – (b – a)–1 [(!1(y – bx)dy – !2(y – ax)dy] 3 or dz = (1/6) × d(x y) – (1/6) × (a + b) x3dx + (b – a)–1!2(y – ax) (dy – adx) – (b – a)–1!1(y – bx) (dy – bdx) or dz = (1/6) × d(x3y) – (1/6) × (a + b) x3dx + (b – a)–1 !2(y – ax) d(y – ax) – (b – a)–1 !1(y – bx) d(y – bx) Integrating, z = (1/6) × x3y – (1/24) × (a + b)x4 + ∗2(y – ax) + ∗1(y – bx), where ∗1 and ∗2 are arbitrary functions. Ex. 3. Solve r – t cos2 x + p tan x = 0. [K.U. Kurukshetra 2005; Meerut 1993] 2 Sol. Given r – t cos x = – p tan x ...(1) Comparing (1) with Rr + Ss + Tt = V, we find R = 1, S = 0, T = – cos2x and V = – p tan x. ...(2) Monge’s subsidiary equations are Rdp dy + Tdq dx – V dx dy = 0 ...(3) and R (dy)2 – S dx dy + T (dx)2 = 0 ...(4) Putting the values of R, S, T and V, (3) and (4) become dp dy – cos2 x dq dx + p tan x dx dy = 0 ...(5) and (dy)2 – cos2x (dx)2 = 0 ...(6)

Created with Print2PDF. To remove this line, buy a license at: http://www.software602.com/

Monge’s Methods

9.5

(dy – cos x dx) (dy + cos x dx) = 0 dy – cos x dx = 0 ...(7) or dy + cos x dx = 0 ...(8) Putting the value of dy from (7) in (5), we get dp cos x dx – cos2 x dq dx + p tan x dx cos x dx = 0 or dp – cos x dq + p tan x dx = 0 or sec x dp + p sec x tan x dx – dq = 0 or d (p sec x) – dq = 0. Integrating it, p sec – q = c1, c1 being an arbitrary constant ....(9) Integrating (7), y – sin x = c2, c2 being an arbitrary constant ...(10) From (9) and (10), one integral of (1) is p sec x – q = f(y – sin x). ...(11) In a similar manner, (8) and (5) give another integral of (1) p sec x + q = g(y + sin x). ...(12) Solving (11) and (12) for p and q, we find p = (f + g)/2 sec x = (1/2) × (f + g) cos x and q = (g – f)/2 ...(13) Now, dz = p dx + q dy or dz = (1/2) × (f + q) cos x dx + (1/2) × (g – f) dy, by (13) or dz = – (1/2) × f(y – sin x) (dy – cos x dx) + (1/2) × g(y + sin x) (dy + cos x dx) Integrating, z = F(y – sin x) + G(y + sin x), F and G being arbitrary functions. Ex. 4. Solve t – r sec4y = 2q tan y. [Delhi Maths Hons 1995; Kanpur 1995; Meerut 1995] Sol. Given t – r sec4y = 2q tan y. ...(1) Comparing (1) with Rr + Ss + Tt = V, R = – sec4y, S = 0, T = 1, V = 2q tan y. ...(2) Monge’s subsidiary equations are Rdp dy + T dq dx – V dx dy = 0 ...(3) 2 2 and R(dy) – S dxdy + T (dx) = 0 ...(4) Putting the values of R, S, T and V, (3) and (4) become –sec4y dp dy + dq dx – 2q tan y dx dy = 0 ...(5) 4 2 2 and –sec y (dy) + (dx) = 0. ...(6) Equation (6) may be factorised as (dx – sec2 y dy) (dx + sec2 y dy) = 0 so that dx – sec2 y dy = 0 ...(7) 2 or dx + sec y dy = 0. ...(8) Putting the value of dx from (7) in (5), we get –sec4y dp dy + dq sec2 y dy – 2q tan y dy × sec2 y dy = 0 or –dp + cos2 y dq – 2q sin y cos y dy = 0 or dp – (cos2 x dq – q × 2 sin y cos y dy) = 0 or dp – d(q cos2 y) = 0. 2 Integrating it, p – q cos y = c1, c1 being an arbitrary constant ...(9) Integrating (7), x – tan y = c2, being an arbitrary constant ...(10) From (9) and (10), one integral of (1) is p – q cos2 y = f(x – tan y). ...(11) Similarly, from (8) and (5) the other integral of (1) is p + q cos2 y = g(x + tan y). ...(12) Solving (11) and (12) for p and q, we find p = (f + g)/2 and q = (g – f)/(2 cos2 y) = (1/2) × (g – f) × sec2y ...(13) Now, we have dz = pdx + qdy or dz = (1/2) × (f + g)dx + (1/2) × (g – f) × sec2y dy, using (13) or dz = (1/2) × f(x – tan y) (dx – sec2y dy) + (1/2) × g(x + tan y) (dx + sec2y dy) or dz = (1/2) × f(x – tan y) d(x – tan y) + (1/2) × g(x + tan y) d(x + tan y). Integrating, z = F(x – tan y) + G(x + tan y), F, G being arbitrary functions. Ex. 5. Solve q(yq + z)r – p (2yq + z)s + yp2t + p2q = 0. [Delhi 2008] Sol. As usual, here Monge’s subsidiary equations are q(yq + z)dp dy + yp2dqdx + p2qdxdy = 0 ... (1) 2 2 2 and q(yq + z)(dy) + p(2yq + z)dxdy + yp (dx) = 0. ... (2) Equation (6) may be factoriesed as +

Created with Print2PDF. To remove this line, buy a license at: http://www.software602.com/

9.6

Monge’s Methods

On factorization, (2) gives (qdy + pdx) {(yq + z)dy + ypdx} = 0. Hence two systems to be considered are q(yq + z)dpdy + yp2dqdx + p2qdxdy = 0, qdy + pdx = 0 ... (3) 2 2 and q(yq + z)dpdy + yp dqdx + p q dxdy = 0, (yq + z)dy + ypdx = 0 ... (4) Using dz = pdx + qdy, the second equation of (3) reduces to dz = 0 so that z = c1. ... (5) From second equation of (3), qdy = – pdx. Hence first equation of (3) reduces to (yq + z)dp – ypdq – pqdy = 0 or (yq + z)dp – p d(yq) = 0 or (yq + z)dp – pd(yq+ z) = 0, as dz = 0, by (5) or

d ( yq # z ) dp ∃ )0 yq # z p

so that

log (yq + z) – log p = log c1

or

(yq + z)/p = c2, c2 being an arbitrary constant ... (6) From (5) and (6), the intermediate integral corresponding to (3) is (yq + z)/p = !1(z) or yq + z = p!1(z), ...(7) where !1 is an arbitrary function. Using dz = pdx + qdy, the second equation of (4) becomes y(qdy + pdx) + zdy = 0 or ydz + zdy = 0 or d(yz) = 0. Integrating it, yz = c3, c3 being an arbitrary constant ...(8) From second equation of (4), (yq + z)dy = – ypdx. Using this fact, first equation of (4) reduces to qdp – pdq – (pq/y)dy = 0 or – (1/p)dp + (1/q)dq + (1/y)dy = 0. Integrating, – log p + log q + log y = log c1 or (yq)/p = c2 ...(9) From (8) and (9), another intermediate integral corresponding to (4) is (qy)/p = !2(yz), where !2 is an arbitrary function. Solving (7) and (10) for p and q, we have Substituting these in dz = pdx + qdy,

p=

z , !1(z) ∃ !2 ( yz)

dz =

q=

...(10)

z!2 (yz) . y,!1(z) ∃ !2 ( yz)}

z {dx + (1/y) × !2(yz) dy} !1(z) ∃ !2 ( yz)

!1(z) dz ! ( yz) d( yz) zdy # ydz or = dx + 1 . z yz y Integrating, ∗1(z) = x + ∗2(yz), where ∗1 and ∗2 are arbitrary functions. Ex. 6. Solve (r – t)xy – s(x2 – y2) = qx – py. [Delhi Maths 2005, Kurukshetra 2005 (H)] Sol. Usual Monge’s auxiliary equations are xydpdy – xydqdx – (qx – py)dxdy = 0 ...(1) 2 2 2 2 and xy(dy) + (x – y ) dxdy – xy (dx) = 0. ...(2) On factorizing, (2)gives (xdy – ydx) (ydx + xdy) = 0. Hence, two systems to be considered are xydpdy – xydqdx – (qx – py) dxdy = 0, xdy – ydx = 0 ...(3) and xydpdy – xydqdx – (qx – py) dxdy = 0, ydx + xdy = 0. ...(4) Second equation of (3) gives y/z = c1, c1 being an arbitrary constant ...(5) Using second equation, first equation of (3) reduces to ydp – xdq – qdx + pdy = 0 or d(yp – xq) = 0 or

!1(z)dz = zdx + !2(yz)

Created with Print2PDF. To remove this line, buy a license at: http://www.software602.com/

Monge’s Methods

9.7

Integrating, yp – xq = c2, c2 being an arbitrary constant ...(6) From (5) and (6), intermediate integral corresponding to (3) is yp – xq = !1(y/x), where !1 is an arbitrary function. ...(7) 2 2 Second equation of (4) gives x + y = c3, c3 being arbitrary constant ...(8) Using second equation, first equation of (4) reduces to xdp + ydq + qdy + pdx = 0 or d(xp) + d(yq) = 0 Integrating, xp + yq = c4, c4 being an arbitrary constant ...(9) From (8) and (9), another intermediate integral corresponding to (4) is xp + yq = !2(x2 + y2), where !2 is an arbitrary function. Solving (7) and (10) for p and q, we have p=

RS F I T HK

UV W

y 1 y !1 # x !2 ( x 2 # y2 ) x x # y2 2

q=

and

1 x # y2 2

FH IK

LMRSy ! F y I # x ! (x NT H x K 1

2

2

FH

y 1 2 2 2 y !2 ( x # y ) ∃ x !1 x x #y 2

Substituting these values in dz = pdx + qdy, we get dz =

RS T

...(10)

UV RS W T

# y 2 ) dx # y !2 ( x 2 # y 2 ) ∃ x !1

b g F I b g H K

IK UV . W

FH y IK dyUVOP x WQ

ydx ∃ xdy y xdx # ydy !1 y x ! (x 2 # y2 ) 2 2 y 2 2 # 1 2 2 d(x # y ) . 2 d 2 2 !1 x # 2 2 !2( x # y ) or dz = – x 2 x # y2 1# y x x #y x #y Integrating, z = ∗1(y/x) + ∗2(x2 + y2), ∗1, ∗2 being arbitrary functions. Ex. 7. Solve (r – s) x = (t – s) y. (M.D.U Rohtak 2005) Sol. Usual Monge’s subsidiary equations are xdpdy – ydqdx = 0 ...(1) and x(dy)2 + (x – y) dxdy – y(dx)2 = 0. ...(2) Factorising, (2) − (xdy – ydx) (dy + dx) = 0. Hence two systems to be considered are xdpdy – ydqdx = 0, xdy – ydx = 0 ...(3) and xdpdy – ydqdx = 0, dy + dx = 0. ...(4) Integrating second equation of (3), y/x = c1, c1 being an arbitrary constant ...(5) Eliminating dy/dx between equations of (3), we get dp – dq = 0 so that p – q = c2, c2 being an arbitrary constant ...(6) Hence the intermediate integral corresponding to (3) is p – q = !1(y/x). ...(7) Integrating second equation of (4), x + y = c3, c3 being an arbitrary constant ...(8) Eliminating dy/dx between equations of (4), we get xdp + ydq = 0 or xdp + ydq + pdx + qdy = pdx + qdy or d(xp) + d(yq) – dz = 0, as dz = pdx + qdy. Integrating, xp + yq – z = c4, c4 being an arbitrary constant ...(9) Hence the intermediate integral corresponding to (4) is xp + yq – z = !2(x + y) or xp + yq = z + !2(x + y), ...(10) Solving (7) and (10) for p and q, we have

or

p=

dz =

RS T

FH

IK UV W

1 z # ! ( x # y) # y! y and 2 1 x#y x Substituting these values in dz = pdx + qdy, we have

dz =



1 x#y

q=

RS T

FH IK UV . W

y 1 z # !2 ( x # y) ∃ x !1 x#y x

LMRSz # ! (x # y) # y ! F y I UVdx # RSz # ! (x # y) ∃ x ! F y I UVdyOP H xKW T H xKW Q NT 2

1

2

1

! ( x # y) d (x # y) ( ydx ∃ xdy) !1 ( y / x ) ( x # y) dx ∃ zdx = 2 # 2 2 2 ( x # y) ( x # y) ( x # y)

Created with Print2PDF. To remove this line, buy a license at: http://www.software602.com/

9.8

Monge’s Methods



d

FG z IJ = ! (x # y) d(x + y) – ! (y / x) dF y I . H x # y K (x # y) 1 # (y / x ) H x K 2

2

1

2

Integrating, z/(x + y) = ∗2(x + y) + ∗1 (y/x), ∗1, ∗2 being arbitrary functions. 2 Ex. 8. Solve r + ka t – 2as = 0. Sol. Given r – 2as + ka2t = 0. ...(1) 2 Comparing (1) with Rr + Ss + Tt = V, we have R = 1, S = –2a, T = ka , V = 0. Hence the Monge’s subsidiary equations Rdp dy + Tdq dx – Vdx dy = 0 and R(dy)2 – S dx dy + T(dx)2 = 0 become dp dy + ka2 dq dx = 0 ...(2) 2 and (dy) + 2a dx dy + ka2 (dx)2 = 0. ...(3) From (3),

dy = [–2a dx . {4a2(dx)2 – 4ka2 (dx)2}1/2]/2 = – a dx ± a

dy + a {1 ± (1∃ k ) }dx = 0

or

or

dy + a (1 ± l) dx = 0, where

(1∃ k ) dx

l = (1∃ k ) .

Hence (3) reduces to the following two equations : dy + a(1 + l)dx = 0 ...(4) and dy + a(1 – l)dx = 0. ...(5) From (2) and (4) , eliminating dy, we have dp{–a (1 + l) dx} + ka2 dqdx = 0 or (1 + l)dp – ka dq = 0. Integrating it, (1 + l)p – kaq = c1, c1 being an arbitrary constant ...(6) Again, integrating (4), y + a(1 + l)x = c2, c2 being an arbitrary constant ...(7) From (6) and (7), first intermediate integral is (1 + l)p – kaq = f1{y + a(1 + l)x}, where f1 is an arbitrary function. ...(8) Similary, from (2) and (5), second intermediate intgegral is given by (replacing l by –l in (8) since (5) differs from (4) in having –l in place of l) (1 – l)p – kaq = f2{y + a(1 – l)x}, where f2 is an arbitrary function ...(9) Solving (8) and (9) for p and q, p = (1/2l) × [f1{y + a(1 + l)x} – f2{y + a(1 – l)x}] and q = (1/2akl) × [(1 – l)f1{y + a(1 + l)x} – (1 + l) f2{y + a(1 – l)x}]. Substituting these values of p and q in dz = pdx + qdy, we get dz = (1/2l) × [f1{y + a(1 + l)x} – f2{y + a(1 – l)x}]dx + (1/2akl) × [(1 – l) f1{y + a(1 + l)x} – (1 + l) f2{y + a(1 – l)x}]dy or dz = (1/2l) × [f1{y + a(1 + l)x} – f2{y + a(1 – l)x}]dx +

1 1/2 2 [ (1 – l) f1{y + a(1 + l)x} – (1 + l) f2{y + a(1 – l)x}]dy, as l = (1 – k) − k = 1 – 2al (1 ∃ l )

l2

LM N

b g

OP Q

dy 1 dy or dz = (1/2l) [dx f1 {y + a(1 + l)x} – dx f2{y + a(1 – l)x}] + 2al 1 # l f1{y # a 1 # l x} ∃ 1 ∃ l f2{y # (1 ∃ l)x}

=

1 1 f {y + a(1 + l)x}{dy + a(1 + l)dx} – f {y + a(1 – l)x} {dy + a(1 – l)dx} 2al (l # 1) 1 2al (1 ∃ l ) 2 1 1 f {y + a(1 + l)x} d{y + a(1 + l)x} – f {y + a(1 – l)x} d{y + a (1 – l)x}. 2al (l # 1) 1 2al (1 ∃ l) 2 Integrating, z = F1{y + a(1 + l)x} + F2{y + a (1 – l)x}, where F1 and F2 are arbitrary functions.

or dz =

Created with Print2PDF. To remove this line, buy a license at: http://www.software602.com/

Monge’s Methods

9.9 –2

–2

–3

–3

Ex. 9. Solve x r – y t = x p – y q. Sol. Comparing the given equation with Rr + Ss + Tt = V, we get R = x–2, S = 0, T = y–2, V = x–3p – y–3q. Then Monge’s subsidiary equations Rdpdy + Tdqdx + Vdxdy = 0 and R(dy)2 – Sdxdy + T(dx)2 = 0 become x–2dpdy + y–2dqdx – (x–3p – y–3q) dxdy = 0. ...(1) and x–2(dy)2 – y–2(dx)2 = 0. ...(2) 3 3 Multiplying both sides of (1) by x y , we get xy3dpdy – x3ydqdx – py3dxdy + qx3dxdy = 0. ...(3) Again, (2) − x2y2(y2dy2 – x2dx2) = 0 or x2y2(ydy + xdx) (ydy – xdx) = 0 Hence (2) is equivalent to the equations ydy + xdx = 0 i.e., ydy = –xdx ...(4) and ydy – xdx = 0. ...(5) Integrating (4), y2/2 + x2/2= c1/2 or x2 + y2 = c1. ...(6) 2 2 2 2 From (3), xy dp(ydy) – x ydq(xdx) – py dx(ydy) + qx dy(xdx) = 0 or xy2dp(–xdx) – x2ydq(xdx) – py2dx (–xdx) + qx2dy(xdx) = 0, using (4) or –xy2dp – x2ydq + py2dx + qx2dy = 0 or y2(xdp – pdx) + x2(ydq – qdy) = 0

FH IK FG IJ HK

xdp ∃ pdx ydq ∃ qdy p q #d =0 or d = 0. # 2 2 x y x y Integrating, (p/x) + (q/y) = c2, c2 being an arbitrary constant ...(7) From (6) and (7), an intermediate integral is (1/x)p + (1/y)q = f(x2 + y2), where f is an arbitrary function. ...(8) Similarly, from (3) and (5), another intermediate integral is (1/x)p – (1/y)q = g(x2 – y2), where g is an arbitrary function ...(9) Solving (8) and (9) for p and q, we obtain p = (x/2) × {f(x2 + y2) + g(x2 – y2)} and q = (y/2) × {f(x2 + y2) – g(x2 – y2)}. Substituting these values of p and q in dz = pdx + qdy, we get dz = (x/2) × {f(x2 + y2) + g(x2 – y2)}dx + (y/2) × {f(x2 + y2) – g(x2 – y2)}dy or dz = (1/4) × f(x2 + y2) (2xdx + 2ydy) + (1/4) × g(x2 – y2) (2xdx – 2ydy) ... (10) 2 Putting x + y2 = u, x2 – y2 = v so that 2xdx + 2ydy = du and 2xdx – 2ydy = dv, (10) gives dz = (1/4) × f(u) du + (1/4) × g(v)dv, ... (11) 2 2 2 2 Integrating (11), z = F(u) + G(v) = F(x + y ) + G(x – y ), where F and G are arbitrary functions. Ex. 10. Solve rx2 – 3s xy + 2t y2 + px + 2qy = x + 2y. Sol. Comparing the given equation with Rr + Ss + Tt = V, we get R = x2, S = –3xy, T = 2y2, V = x + 2y – px – 2qy. Hence Monge’s subsidiary equations are Rdpdy + Tdqdy – Vdxdy = 0 and R(dy)2 – Sdxdy + T(dx)2 = 0 become x2 dpdy + 2y2dqdx – (x + 2y – px – 2qy) dxdy = 0 ...(1) and x2(dy)2 + 3xy dxdy + 2y2(dx)2 = 0. ...(2) Here (2) − (xdy + 2ydx) (xdy + ydx) = 0. Hence (2) resolves into the following two equations xdy + 2ydx = 0 i.e., 2ydx = – xdy ...(3) and xdy + ydx = 0. ...(4) Re–writing (3), (1/y)dy + 2(1/x)dx = 0

or

Created with Print2PDF. To remove this line, buy a license at: http://www.software602.com/

9.10

Monge’s Methods 2

Integrating, log y + 2 log x = log c1 or yx = c1. ...(5) Re–writing (1), (xdp) (xdy) + ydq (2ydx) – dx(xdy) – dy (2ydx) + pdx (xdy) + qdy (2ydx) = 0 (xdp) (xdy) + ydq(–xdy) – dx(xdy) – dy(–xdy) + pdx(xdy) + qdy(–xdy) = 0, using (3) xdp – ydq – dx + dy + pdx – qdy = 0 (xdp + pdx) – (ydq + qdy) – dx + dy = 0 or d(xp) – d(yq) – dx + dy = 0. Integrating, xp – yq – x + y = c2, c2 being an arbitrary constant ...(6) From (5) and (6), an intermediate integral is xp – yq – x + y = f(x2y), where f is an arbitrary function. ...(7) Similarly from (1) and (4), another intermediate integral is xp – 2yq – x + 2y = g(xy), where g is an arbitrary function. ...(8) Solving (7) and (8) for p and q, we have p = (1/x) × {x + 2f(x2y) – g(xy)}, and q = (1/y) × {y + f(x2y) – g(xy)}. Substituting these values of p and q in dz = pdx + qdy, we get dz = (1/x) × {x + 2f(x2y) – g(xy)}dx + (1/y) × {y + f(x2y) – g(xy)}]dy

or or or

FG H

or

IJ K

FG H

2 1 dx dy dz = dx + dy + f(x2y) x dx # y dy – g(xy) x # y

or

IJ K

dz = dx + dy + f(x2y) d[log (x2y)] – g(xy) d[log (xy)]. Integrating, z = x + y + F(x2y) + G(xy), G, and F being arbitrary functions. Ex. 11. Find the general solution of the equation r + 4t = 8 xy, by Monge’s method. Find also the

particular solution for which z ) y2 and p ) 0 , when x = 0

[Delhi Maths (Hons) 2006, 09]

r # 4t ) 8xy

Sol. Given

... (1)

Comparing (1) with Rr # Ss # Tt ) V , here R = 1, S = 0, T = 4 and V = 8 xy. Hence Monge’s 2 2 Rdp dy # Tdq dx ∃ Vdxdy ) 0 and R(dy ) ∃ Sdx dy # T (dx ) = 0 become

subsidians equations

dpdy # 4 dqdx ∃ 8 xydxdy ) 0

... (2)

(dy)2 # 4 (dx) 2 ) 0

... (3)

and Re–writing (3),

dy2 ∃ 4i2 dx2 ) 0

(dy ∃ 2idx) (dy # 2idx) ) 0

or

so that

dy ∃ 2idx ) 0

or

dy ) 2idx

... (4)

and

dy # 2idx ) 0

or

dy ) ∃2idx

... (5)

y ∃ 2ix ) C1

... (6)

We first consider (4) and (2). Integrating (4), Using (4) and (6), (2) gives or

i dp # 2dq ∃ 8 xi (C1 # 2ix) ) 0, by (6) Integrating,

or

dp (2i dx) # 4 dq dx ∃ 8x (C1 # 2ix ) (2i dx) dx ) 0 or

idp # 2dq ∃ 8C1ix dx # 16x 2 dx ) 0

ip # 2q ∃ 4C1ix 2 # (16 / 3) x3 ) C2 , C2 being an arbitrary constant ip # 2q ∃ 4ix 2 ( y ∃ 2ix) # (16 / 3) / x3 ) C2 , by (6) 2

... (7) 3

From (6) and (7) first intermediate integral of (1) is ip + 2q – 4ix (y – 2ix) + (16/3)x = f(y – 2ix) or

ip # 2q ) (8 / 3) / x3 # 4ix 2 y # f ( y ∃ 2ix) , f being an abritrary function

... (8)

Similarly considering the pair (5) and (2), the second intermediate integral of (1) is

Created with Print2PDF. To remove this line, buy a license at: http://www.software602.com/

Monge’s Methods

9.11

ip ∃ 2q ) ∃(8 / 3) / x3 # 4ix 2 y # g ( y # 2ix) , g being an arbitrary function

... (9)

p ) {8ix 2 y # f ( y ∃ 2ix) # g( y # 2ix)}/ 2i

Solving (8) and (9) for p and q,

q ) {(16 / 3) / x3 # f ( y ∃ 2ix) ∃ g ( y # 2ix)}/ 4

and

Putting the above values of p and q in dz ) pdx # qdy , we get

dz ) (1/ 2i) /{8ix2 y # f ( y ∃ 2ix) # g( y # 2ix)}dx + (1/ 4) / {(16 / 3) / x3 # f ( y ∃ 2ix) ∃ g ( y ∃ 2ix)}dy = (4 / 3) / (3x 2 ydx # x3dy) # (1/ 4) / f ( y ∃ 2ix)d ( y ∃ 2ix) ∃ (1/ 4) / g ( y # 2ix)d ( y # 2ix) +

dz ) (4 / 3) / d ( x 3 y) # (1/ 4) / f ( y ∃ 2ix ) d ( y ∃ 2ix) ∃ (1/ 4) / g( y # 2ix)d( y # 2ix)

z ) (4 / 3) / x 3 y # F ( y ∃ 2ix) # G ( y # 2ix), which is the general solution of (1) containing F and G as arbitrary functions To find particular solution of (1) Given conditions are Integrating,

z ) y2 From (11),

and

p ) z/ x ) 0

z / y ) 2y

when x ) 0 x=0

when

... (10)

... (11) ... (12)

Differentiating (10) partially w.r.t. ‘x’ and ‘y’, we get

and

z / x ) 4x 2 y ∃ 2i F 0( y ∃ 2ix) # 2i G0( y # 2ix )

... (13)

z / y ) (4 / 3) / x3 # F 0( y ∃ 2ix) # G 0( y # 2ix)

... (14)

Using (11) and (12), (10), (13) and (14) reduce to

and

F ( y) # G ( y) ) y2

... (15)

F 0( y) ∃ G0( y) ) 0

... (16)

F 0( y) # G 0( y) ) 2 y

... (17)

From (16) and (17),

F 0( y) ) y

and

G 0( y) ) y

Integrating these,

F ( y) ) y2 / 2

and

G( y) ) y2 / 2 ... (18)

and

G( y # 2ix ) ) ( y # 2ix)2 / 2

which also satisfy (15). From (18),

F( y ∃ 2ix) ) ( y ∃ 2ix)2 / 2

Putting these values in (10), the required particular solution is

z ) (4 / 3) / x 3 y # ( y ∃ 2ix)2 / 2 # ( y # 2ix)2 / 2

or

z = (4/3) × x3y + y2 – 4x2.

9.5. Type 2. When the given equation Rr + Ss + Tt = V leads to two distinct intermediate integrals and only one is employed to get the desired solution. Working rule for solving problems of type 2. Step 1. Write the given equation in the standard form Rr + Ss + Tt = V. Step 2. Substitute the values of R, S, T and V in the Monge’s subsidiary equations Rdpdy + Tdqdx – Vdxdy = 0 ... (1) R(dy)2 – Sdxdy + T(dx)2 = 0 ... (2) Step 3. Factorise (1) into two distinct factors. Step 4. Take one of the factors of step 3 and use (2) to get an intermediate integral. Don’t find second intermediate integral as we did in type 1. If required use remark 1 of Art. 9.2.

Created with Print2PDF. To remove this line, buy a license at: http://www.software602.com/

9.12

Monge’s Methods

Step 5. Re–write the intermediate integral of the step 4 in the form of Lagrange equation, namely, Pp + Qq = R (refer chapter 2). Using the well known Lagrange’s method we arrive at the desired general solution of the given equation. 9.6 SOLVED EXAMPLES BASED ON ART. 9.5. Ex. 1. Solve (r – s)y + (s – t)x + q – p = 0. Sol. The given can be written as yr + s(x – y) – tx = p – q. ...(1) Comparing (1) with Rr + Ss + Tt = V, R = y, S = x – y, T = –x and V = p – q. Hence Monge’s subsidiary equations Rdpdy + Tdqdx – Vdxdy = 0 and R(dy)2 – Sdxdy + T(dx)2 = 0 become ydpdy – xdqdx + (q – p)dxdy = 0 ...(1) and y(dy)2 – (x – y) dxdy – x (dx)2 = 0. ...(2) Re–writing (2), (dy + dx) (ydy – xdx) = 0. so that dy + dx = 0 or dy = – dx ...(3) and ydy – xdx = 0. ...(4) Using (3), (1) becomes – ydpdx – xdqdx + q dx(– dx) – p dxdy = 0 or ydp + xdq + qdx + pdy = 0 or (ydp + pdy) + (xdq + qdx) = 0 or d(yp) + d(xq) = 0 so that yp + xq = c1. ...(5) Integrating (3), x + y = c2, c2 being an arbitrary constant ...(6) From (5) and (6), one intermediate integral is yp + xq = f(x + y), ...(7) which is of the Lagrange’s form and so its subsidiary equations are dx dy dz = = . ...(8) y x f ( x # y) From first and second fractions of (8), 2xdx – 2ydy = 0. Integrating, x2 – y2 = a, a being an arbitrary constant ...(9) Taking first and third fractions of (8), we get dz dx = dx dz or , as (9) − y = (x2 – a)1/2 2 1/ 2 = f ( x # y) y ( x ∃ a) f x # ( x 2 ∃ a)1/ 2 dz = f[x + (x2 – a2)1/2] (x2 – a2)–1/2dx

or Put

2

x + (x – a)

1/2

=v

so that

2

...(10) 1/ 2

1 # x ( x ∃ a)

dx = dv

...(11)

x # ( x 2 ∃ a)1/ 2 dx dx = dv or = dv , using (11) 1) v ( x 2 ∃ a)1/ 2 ( x 2 ∃ a)1/ 2 Then, (10) reduces to dz – (1/v) f(v)dv = 0. Integrating, z – F(v) = b or z – F[x + (x2 – a)1/2] = b, by (11) 2 or z – F(x + y) = b, as y = (x – a)1/2, by (9) ...(12) 2 2 From (9) and (12), the required general solution is z – F(x + y) = G(x – y ) or z = F(x + y) + G(x2 – y2), where F and G are arbitrary functions. Ex. 2. Solve : q(1 + q)r – (p + q + 2pq)s + p(1 + p)t = 0. [Meerut 1994; I.A.S. 1974] Sol. Comparing the given equation with Rr + Ss + Tt = V, we find R = q(1 + q), S = – (p + q + 2pq), T = p(1 + p), V=0 ...(1) Monge’s subsidiary equations are Rdpdy + Tdq dx – Vdxdy = 0 ...(2) and R(dy)2 – Sdxdy + T(dx)2 = 0 ...(3) Using (1), (2) and (3) become (q + q2)dpdy + (p + p2)dqdx = 0 ...(4) and (q + q2) (dy)2 + (p + q + 2pq)dxdy + (p + p2) (dx)2 = 0. ...(5)

or

Created with Print2PDF. To remove this line, buy a license at: http://www.software602.com/

Monge’s Methods

9.13

In order to factorise (5), we re–write it as q(1 + q)(dy)2 + (p + pq)dxdy + (q + pq)dxdy + p(1 + p)(dx)2 = 0 or q(1 + q)(dy)2 + p(1 + q)dxdy + q(1 + p)dxdy + p(1 + p)(dx)2 = 0 or (1 + q)dy(qdy + pdx) + (1 + p)dx(qdy + pdx) = 0 or (qdy + pdx) [(1 + q)dy + (1 + p)dx] = 0. ... (6) Then, from (6), we get qdy + pdx = 0 i.e., qdy = –pdx ...(7) and (1 + q)dy + (1 + p)dx = 0. ...(8) Keeping (7) in view, (4) may be re–written as (1 + q)dp (qdy) – (1 + p)dq (–pdx) = 0 From (7), qdy and (–pdx) are equivalent. Hence dividing each term of the above equation by qdy, or its equivalent (–pdx), we get (1 + q)dp – (1 + p)dq = 0 or dp/(1 + p) – dq/(1 + q) = 0. Integrating it, log (1 + p) – log (1 + q) = log c1 or (1 + p)/(1 + q) = c1. ...(9) Using dz = pdx + qdy, (7) becomes dz = 0 so that z = c2. ...(10) From (9) and (10), one intermediate integral of (1) is given by (1 + p)/(1 + q) = f(z) or p – f(z)q = f(z) – 1, ...(11) which is of the form Pp + Qq = R. Here Lagrange’s auxiliary equations for (11) are dy dx = = dz . ...(12) 1 ∃ f ( z) f (z) ∃1 dx # dy # dz dx # dy # dz Choosing 1, 1, 1 as multipliers, each fraction in (12) = = 1 ∃ f ( z ) # f (z ) ∃ 1 0 dx + dy + dz = 0 so that x + y + z = c2. ...(13) + From first and third fractions in (12), we get dx – [f(z) – 1]–1 dz = 0. Integrating it, x + F(z) = c4, c4 being an arbitrary constant ...(14) From (13) and (14), the required general solution is x + F(z) = G(x + y + z), F, G being arbitrary functions. Ex. 3. Solve (x – y) (xr – xs – ys + yt) = (x + y)(p – q). [Delhi Maths (H) 97, 2000; Meerut 1999; Garhwal 1996] Sol. Given (x – y)xr – (x2 – y2)s + (x – y)yt = (x + y)(p – q) ...(1) Comparing (1) with Rr + Ss + Tt = V, we find R = x(x – y), S = –(x2 – y2), T = y(x – y), V = (x + y)(p – q). ...(2) Monge’s subsidiary equations are Rdpdy + Tdqdx – Vdxdy = 0 ...(3) 2 2 and R(dy) – Sdxdy + T(dx) = 0. ...(4) Using (2), (3) and (4) become x(x – y)dpdy + y(x – y)dqdx – (x + y)(p – q)dxdy = 0 ...(5) 2 2 2 2 and x(x – y)(dy) + (x – y )dxdy + y(x – y)(dx) = 0. ...(6) Since x2 – y2 = (x – y)(x + y), dividing (6) by (x – y) gives xdy2 + (x + y)dxdy + ydx2 = 0 or (xdy + ydx) (dx + dy) = 0 Thus we get xdy + ydx = 0 or xdy = – ydx ...(7) and dx + dy = 0. ...(8) Keeping (7) in view, (5) may be rewritten as (x – y)dp (xdy) – (x – y) dq (–ydx) – (p – q) dx (xdy) + (p – q)dy(– ydx) = 0. From (7), x dy and (–y dx) are equal. So dividing each term of the above equation by x dy, or its equivalent (–y dx), we get (x – y) dp – (x – y)dq – (p – q) dx + (p – q)dy = 0 or (x – y) (dp – dq) – (p – q) (dx – dy) = 0

Created with Print2PDF. To remove this line, buy a license at: http://www.software602.com/

9.14

Monge’s Methods

dp ∃ dq dx ∃ dy p∃q ∃ =0 so that = c1 ...(9) p∃q x∃y x∃y Integrating (7), xy = c2, c2 being an arbitrary constant ...(10) From (9) and (10), one intermediate integral of (10) is (p – q)/(x – y) = f(xy) or p – q = (x – y)f(xy) ...(11) which is of the form Pp + Qq = R. Its Lagrange’s auxiliary equations are

or

dy dx dz = = . ∃1 1 ( x ∃ y) f ( xy) Taking the first two fractions of (12), we get dx + dy = 0 so that x + y = c3, c3 being an arbitrary constant

...(12) ...(13)

y f ( xy) dx # x f ( xy) dy # dz 0 so that f(xy) × (ydx + x dy) + dz = 0 or f(xy) × d(xy) + dz = 0. Integrating it, F(xy) + z = c4, c4 being an arbitrary constant ...(14) From (13) and (14), the required general solution is F(xy) + z = G(x + y), where F and G are arbitrary functions. Ex. 4. xy (t – r) + (x2 – y2) (s – 2) = py – qx. [Delhi Maths (H) 2001] Sol. Given – xyr + (x2 – y2) s + xyt = py – qx + 2(x2 – y2). ...(1) Comparing (1) with Rr + Ss + Tt = V, we find R = –xy, S = x2 – y2, T = xy, V = py – qx + 2(x2 – y2). ...(2) Monge’s subsidiary equations are Rdp dy + Tdq dx – V dx dy = 0 ...(3) 2 2 and R(dy) – Sdxdy + T(dx) = 0. ...(4) Using (2), (3) and (4) become – xy dp dy + xydqdx – [py – qx + 2(x2 – y2)]dxdy = 0 ...(5) 2 2 2 2 and – xy (dy) – (x – y )dxdy + xy(dx) = 0. ...(6) From (6), xy(dy)2 + x2dxdy – y2dxdy – xy(dx)2 = 0 or xdy(ydy + xdx) – ydx (ydy + xdx) = 0 or (xdy – ydx) (ydy + xdx) = 0. So, we get xdx + ydy = 0, i.e., xdx = –ydy ...(7) and xdy – ydx = 0. ...(8) Keeping (7) in view, (5) may be re–written as xdp(–ydy) + ydq (xdx) + pdx (–ydy) + qdy(xdx) – 2xdy(xdx) – 2ydx(–ydy) = 0. From (7), xdx and (–ydy) are equivalent. So dividing each term of the above equation by xdx, or its equivalent (–ydy), we get xdp + ydq + pdx + qdy – 2xdy – 2ydx = 0 or (xdp + pdx) + (ydq + qdy) – 2(xdy + ydx) = 0. Integrating it, xp + yq – 2xy = c1, being an arbitrary constant ...(9) Integrating (7), x2/2 + y2/2 = c2/2 or x2 + y2 = c2. ...(10) From (9) and (10), one intgegral of (1) is xp + qy – 2xy = f(x2 + y2) or xp + yq = 2xy + f(x2 + y2), ...(11) which is of the form Pp + Qq = R. So Lagrange’s auxiliary equations for (11)are

Taking y f(xy), x f(xy), 1 as multipliers, each fraction of (12) =

dz dx = dy = 2 2 . y x 2 xy # f ( x # y ) Taking the first two fractions in (12), we get log y – log x = log c3 or y/x = c3 or

..(12) y = xc3

...(13)

Created with Print2PDF. To remove this line, buy a license at: http://www.software602.com/

Monge’s Methods

9.15

Taking the first and the last fractions in (12) and using y = xc3 in it, we get dz = (1/x) × [2c3x2 + f(x2 + x2c32)]dx or dx = 2c3xdx + (1/x) × f{(1 + c32)x2}dx 2 or dz = 2c3xdx + (1/2x ) × f{(1 + c32)x2}d (x2). 2 Integrating z – 2c3(x /2) + F{(1 + c32)x2} = c4 or z – (y/x)x2 + F{(1 + y2/x2)x2} = c4, by (13) or z – xy + F(x2 + y2) = c4, c4 being an arbitrary constant ...(14) From (13) and (14), the required general solution is z – xy + F(x2 + y2) = G(y/x), where F and G are arbitrary functions. 2 2 Ex. 5. Solve x r – y t – 2xp + 2z = 0. Sol. Given x2r – y2t = 2xp – 2z. ...(1) 2 2 Comparing (1) with Rr + Ss + Tt = V, R=x, S = 0, T = –y , V = 2xp – 2z. Hence the usual Monge’s subsidiary equations Rdpdy + Tdqdx – Vdxdy = 0 and R(dy)2 – Sdxdy + T(dx)2 = 0 become x2dpdy – y2dqdx – (2xp – 2z)dxdy = 0 ...(2) and x2(dy)2 – y2(dx)2 = 0. ...(3) On factorizing, (3) − (xdy – ydx) (xdy + ydx) = 0 Thus, we have xdy – ydx = 0 i.e., xdy = ydx. ...(4) and xdy + ydx = 0. ...(5) Re–writing (2), xdp (xdy) – ydq(ydx) – 2(xp – z) (xdy)(1/x)dx = 0 or xdp(xdy) – ydq(xdy) – 2(xp – z) (xdy) (1/x)dx = 0, using (4) or xdp – ydq – 2(xp – z) (1/x)dx = 0 or xdp – dz + pdx + qdy – ydq – 2(xp – z) (1/x)dx = 0 as dz = pdx + qdy − –dz + pdx + qdy = 0 or d(xp – z) – d(yq) + 2qdy – 2(xp – z) (1/x)dx = 0 or d(xp – yq – z) + 2qy(1/x)dx – 2(xp – z) (1/x)dx = 0, as from (4), dy = (y/x)dx or

or

d(xp ∃ yq ∃ z) 2dx ∃ xp ∃ yq ∃ z x = 0. Integrating, log (xp – yq – z) – 2 log x = log c1 or (xp – yq – z)/x2 = c1. ...(6) From (4), (1/y)dy – (1/x)dx = 0 so that log y – log x = log c2 y/x = c2, c2 being an arbitrary constant ...(7) From (6) and (7), an intermediate integral is (xp – yq – z) /x2 = !1(y/x) or xp – yq = z + x2!1(y/x). ...(8)

d(xp – yq – z) – 2 (xp – yq – z) (1/x) dx = 0

or

dy dx dz = = . 2 x ∃y z # x !1(y / x )

Lagrange’s auxiliary equations for (8) are From the first two ratios of (9), we get (1/x) dx + (1/y) dy = 0 so that Taking the second and third ratios of (9), we get

FH IK

xy = c3.

F I GH JK

...(9)

...(10)

c2 y2 y dz # z = – x 2 = – 33 !1 , by (10) !1 c3 dy y y x y

Its I.F = e(1/y)dy = y and so solution is

or

zy +

c33 / 2 2

z

zy = –

F c I ! Fy IF c I dFy I = c GH y JK GH c JK GH y JK GH c JK 3 2

2

3

2

1

3

z

2

3

4

F I GH JK

2 c3 y ! dy + c4 1 y2 c32

or

FG IJ = c H K

2 zy + c33/2 ∗1 y c3

4

Created with Print2PDF. To remove this line, buy a license at: http://www.software602.com/

9.16

Monge’s Methods 3/2

zy + (xy) ∗1(y/x) = c4, using (10). ...(11) From (10) and (11), the required general solution is zy + (xy)3/2 ∗1(y/x) = ∗2(xy), where ∗1 and ∗2 are arbitrary functions. Ex. 6. Solve (r – t)xy – s(x2 – y2) = qx – py. Sol. Given xyr – (x2 – y2)s – xyt = qx – py. ...(1) Hence the usual Monge’s subsidiary equations Rdpdy + Tdqdx – Vdxdy = 0 and R(dy)2 – Sdxdy + T(dx)2 = 0 become xy dpdy – xy dqdx – (qx – py) dxdy = 0 ...(2) and xy (dy)2 + (x2 – y2) dxdy – xy (dx)2 = 0. ...(3) Now, (3) − (xdx + ydy) (xdy – ydx) = 0 Hence, xdx + ydy = 0 i.e., xdx = – ydy ...(4) and xdy – ydx = 0 ...(5) Re–writing (2), (xdp) (ydy) – ydq(xdx) – qdy(xdx) + pdx (ydy) = 0 or (xdp) (ydy) – ydq(–ydy) – qdy(–ydy) + pdx(ydy) = 0, using (4) or xdp + ydq + qdy + pdx = 0 or d(xp) + d(yq) = 0. Integrating, xp + yq = c1, c1 being an arbitrary constant ...(6) 2 2 2 2 Integrating (4) x /2 + y /2 = c2/2 or x + y = c2. ...(7) From (6) and (7), are intremediate integral is xp + yq = f(x2 + y2), f being an arbitrary function. ...(8) or

Lagrange’s subsidiary equations for (8) are Taking the first and second fractions of (9), Integrating, log y – log x = log a where a is an arbitrary constant. Taking the first and third fraction of (9), we get dx = dz 2 2 x f (x # y )

or Putting

or

dy dx dz = = 2 2 . x y f (x # y ) (1/y)dy – (1/x)dx = 0. or y/x = a,

...(9) ...(10)

dz dx = , using (10) 2 x f (x # a 2 x2 )

dz = (1/x) × f[x2 (1 + a2)] dx = (1/x2) × f[x2 (1 + a2)] xdx. ...(11) 2 2 2 x (1 + a ) = v and 2x(1+ a )dx = dv, (11) gives

1# a2 1 12 1 f(v) × f(v)dv. 2 dv = 3 5 2v 46 v 2(1 # a ) Integrating, z = F(v) + b or z – F[x2 (1 + a2)] = b 2 2 2 2 or z – F(x + x a ) = b or z – F(x + y2) = b, using (10). ...(12) Here b is an arbitrary constant. From (10) and (12), general solution of (1) is z – F(x2 + y2) = G(y/x) or z = F(x2 + y2) + G(y/x), where F and G are arbitrary functions. Ex. 7. Solve 2xr – (x + 2y)s + yt = [(x + 2y) (2p – q)]/(x – 2y) Sol. Comparing the given equation with Rr + Ss + Tr = V, we have R = 2x, S = –(x + 2y), T = y, V = [(x – 2y) (2p – q)]/(x – 2y). Hence the usual Monge’s subsidiary equations Rdpdy + Tdqdx – Vdxdy = 0 and R(dy)2 – Sdxdy + T(dx)2 = 0 dz =

become

2xdpdy + ydqdx –

x # 2y (2p – q) dxdy = 0 x ∃ 2y

...(1)

Created with Print2PDF. To remove this line, buy a license at: http://www.software602.com/

Monge’s Methods

9.17 2

2

2x(dy) + (x + 2y)dxdy + y(dx) = 0. ...(2) The equation (2) can be resolved into the following two equations xdy + ydx = 0 i.e., xdy = – ydx ...(3) and dx + 2ydy = 0. ...(4) 2p ∃ q Re–writing (1), 2dp(xdy) + dq(ydx) – [(xdy)dx + 2(ydx)dy] x ∃ 2y 2p ∃ q or 2dp(–ydx) + dq(ydx) – {(–ydx)dx + 2(ydx)dy} = 0 using (3) x ∃ 2y 2dp ∃ dq dx ∃ 2dy 2p ∃ q ∃ or –2dp + dq – (–dx + 2dy) = 0 or = 0. 2p ∃ q x ∃ 2y x ∃ 2y Intergrating, log (2p – q) – log (x – 2y) = log c1 or (2p – q)/(x – 2y) = c1. ...(5) Re–writing (3), (1/y)dy + (1/x)dx = 0 so that log x + log y = log c2 xy = c , c being an arbitrary constant ...(6) + 2 2 From (5) and (6), an intermediate integral is (2p – q)/(x – 2y) = f(xy) or 2p – q = (x – 2y) f(xy), ...(7) where f is an arbitrary function. The equation (7) is of Lagrange’s form Pp + Qq = R. So Lagrange’s, subsidiary equation for (7) are and

dz dx dy = = . ∃1 ( x ∃ 2 y) f ( xy) 2 Taking the first and second fractions of (8), dx + 2dy = 0. Integrating, x + 2y = a, a being an arbitrary constant Taking y f(xy), x f(xy), 1 as multipliers, each fraction of (8) =

...(8) ...(9)

y f ( xy) dx # x f ( xy) dy # dz f ( xy)( ydx # x dy) # dz = 0 2 y f ( xy) ∃ x f ( xy) # ( x ∃ 2 y) f ( xy)

This − f(xy) d(xy) + dz, as ydx + xdy = d(xy) Integrating, F(xy) + z = b, b being an arbitrary constant. ...(10) From (9) and (10), the required complete integral is F(xy) + z = G(x + y), F and G being arbitrary functions. Ex. 8. Solve xr + (x + y)s + yt + p + q = 0 by Monge’s method. Sol. Given xr + (x + y)s + yt = –(p + q) ...(1) Comparing (1) with Rr + Sr + Tt = V, here R = x, S = x + y, T = y and V = –(p + q). Hence Monge’s subsidiary equations Rdpdy + Tdqdx – Vdxdy = 0 and R(dy)2 – Sdxdy + T(dx)2 = 0 become xdpdy + ydqdx + (p + q)dxdy = 0 ...(2) and x(dy)2 – (x + y)dxdy + y(dx)2 = 0 ...(3) Re–writing (3), (xdy – ydx) (dy – dx) = 0 so that xdy – ydx = 0 ...(4) and dy – dx = 0 i.e., dy = dx ...(5) For the required solution, we consider relation (5) only. Integrating (5), x – y = c1, being an arbitrary constant ... (6) Using (5), (2) becomes xdpdx + ydqdx + (p + q)(dx)2 = 0 or xdp + ydq + pdx + qdx = 0, on dividing by dx (as dx 7 0) or (xdp + pdx) + (ydq + qdx) = 0 or (xdp + pdx) + (ydq + qdy) = 0 by(5) or d(xp) + d(yq) = 0 so that xp + yq = c2. ...(7)

Created with Print2PDF. To remove this line, buy a license at: http://www.software602.com/

9.18

Monge’s Methods

From (6) and (7), one intermediate integral of (1) is xp + yq = f(x – y), f being an arbitrary function which is of Lagrange’s form. Its Lagrange’s auxiliary equations are

...(8)

dx dy dz ) ) x y f ( x ∃ y)

...(9)

(1/ x)dx ∃ (1/ y)dy ) 0

Taking the first two fractions of (9), Integrating,

log x ∃ log y ) log c3

Now,

each fraction of (9) =

x/y = c3

or dx ∃ dy d ( x ∃ y) ) x∃y x∃y

...(10) ...(11)

Combining this fraction with last fraction of (9), we get dz d ( x ∃ y) ) f ( x ∃ y) x∃y

Integrating, or

or

dz )

f ( x ∃ y) f (u)du d ( x ∃ y) ) , if x∃y u

z ) F (u) # c4 ) F ( x ∃ y) # c4 ,

F (u ) ) 8

where

z – F(x – y) = c4, c4 being an arbitrary constant From (10) and (12), the required solution is

z ∃ F ( x ∃ y) ) G( x / y)

u=x–y 1 f (u ) du u ...(12)

z ) G( x / y) # F ( x ∃ y) ,

or

where F and G are arbitrary functions. Ex. 9. Solve rq2 – 2pqs + p2t = pt – qs by Monge’s method. [Delhi Maths (Hons) 2002] Sol. Given q2r – q (2p – 1)s + p(p – 1)t = 0 ...(1) Comparing (1) with Rr + Ss + Tt = V, here R = q2, S = – q(2p – 1), T = p(p – 1), V = 0. Hence Monge’s subsidiary equations Rdpdy + Tdqdx – Vdxdy = 0 and R(dy)2 – S dxdy + T(dx)2 = 0 become 2 q dpdy + p(p – 1)dqdx = 0 ...(2) and q2(dy)2 + q(2p – 1)dxdy + p (p – 1) (dx)2 = 0 ...(3) Re–writing (3), (qdy + pdx) {qdy + (p – 1)dx} = 0 so that qdy + pdx = 0 i.e., qdy = –pdx ...(4) and qdy + (p – 1) dx = 0 ...(5) For the required solution, we consider relation (4) only. Since dz = pdx + qdy, (4) reduces to dz = 0 and so z = c1 ...(6) Re–writing (2), (qdp) (qdy) + (p – 1) dq(pdx) = 0 or (qdp) (–pdx) + (p – 1) dq(pdx) = 0, since from (4), qdy = –pdx or –qdp + (p – 1)dq = 0 or {1/(p – 1)}dp – {1/q}dq = 0 Integrating,

log( p ∃ 1) ∃ log q ) log c2

( p ∃ 1) / q ) c2 ...(7)

or

From (6) and (7), one intermediate integral of (1) is (p – 1)/q = f(z) or which is of Lagrange’s form. Its Lagrange’s auxiliary equations are dx dy dz ) ) 1 ∃ f ( z) 1 From the first and the last fractions of (9), dx – dz = 0

p – qf(z) = 1,

...(8)

...(9) so that

x – z = c3

...(10)

Created with Print2PDF. To remove this line, buy a license at: http://www.software602.com/

Monge’s Methods

9.19

dy – f(z)dz = 0

From the last two fractions of (9), Integrating,

y – F(z) = c4,

F(z) = 8 f(z)dz

where

...(11)

From (10) and (11), the required solution is y – F(z) = G(x – z) or y = F(z) + G(x – z), where F, G are arbitrary functions. Ex. 10. Solve e2y(r – p) = e2x(t – q) by Monge’s method. Sol. Given e2yr – e2xt = pe2y – qe2x ...(1) Comparing (1) with Rr + Ss + Tt = V, here R = e2y, S = 0, T = –e2x and V = pe2y – qe2x. Hence Monge’s subsidiary equations Rdpdy + Tdqdx – Vdxdy = 0 and R(dy)2 – Sdxdy + T(dx)2 = 0 2y 2x 2y 2x become e dxdy – e dqdx – (pe – qe )dxdy = 0 ...(2) and e2y(dy)2 – e2x(dx)2 = 0 ...(3) y x y x From (3), (e dy – e dx) (e dy + e dx) = 0 so that eydy – exdx = 0 , that is, exdx = eydy ...(4) y x and e dy + e dx = 0 ...(5) For the required solution, we consider relation (4) only. Integrating (4), ex – ey = c1, c1 being arbitrary constant ...(6) y y x x y y x x Rewriting (2), (e dp)(e dy) – (e dq)(e dx) – p(e dy)(e dx) + q(e dx)(e dy) = 0 or (eydp)(exdx) – (exdq)(exdx) – p(exdx)(eydx) + q(xxdx)(eydy) = 0, by (4) y x y or e dp – e dq – pe dx + qexdy = 0 or {d(eyp) – peydy} – {d(exq) – qexdx} = peydx – qexdy or d(eyp) – d(exq) = pey(dx + dy) – qex(dx + dy) or d(eyp – exq) = (eyp – exq) (dx + dy) d (e y p ∃ e x q ) ) d ( x # y) ey p ∃ ex q

or

log(e y p ∃ e x q) ∃ log c2 ) x # y

Integrating, or

(e y p ∃ e x q ) / c2 ) e x # y

or

(eyp – exq)/ex+y = c2, c2 being an arbitrary constant From (6) and (7), one intermediate integral of (1) is ( e y p ∃ e x q ) / e x # y ) f (e x ∃ e y )

...(7)

eyp – exq = ex + yf(ex – ey)

or

which is of Lagrange’s form. Its Lagrange’s auxiliary equations are dx dy dz ) x ) x#y x y e ∃e e f (e ∃ e y )

From the first two fractions of (8),

e x dx # e y dy ) 0

...(8) e x # e y ) c3 ...(9)

so that

Taking the first and third fraction of (8) and noting that e y ) c3 ∃ e x from (9), we get dx dz ) x y x y e e e f (e ∃ c3 # e x )

or

dz ∃ (1/ 2) / f (2ex ∃ c3 )d (2e x ∃ c3 ) ) 0

Integrating, or

z – F(u) = c3,

z ∃ F (2e x ∃ c3 ) ) c4

dz ) e x f (2ex ∃ c3 )dx

or or

x dz ∃ (1/ 2) / f (u)du ) 0 , taking u = 2e – c3

F (u) ) 8 (1/ 2) / f (u )du

where or

z ∃ F (e x ∃ ey ) ) c4 , by (9) x

or

y

x

...(10) y

From (9) and (10), the required solution is z – F(e – e ) = G(e + e ) z = F(ex – ey) + G(ex + ey), where F, G are arbitrary functions.

Created with Print2PDF. To remove this line, buy a license at: http://www.software602.com/

9.20

Monge’s Methods 2

2

Ex. 11. Solve x r – y t = xp – yq by Monge’s method. Sol. Given x2r – y2t = xp – yq ...(1) Comparing (1) with Rr + Ss + Tt = V, here R = x2, S = 0, T = –y2 and V = xp – yq. Hence Monge’s subsidiary equations Rdpdy + Tdqdx – Vdxdy = 0 and R(dy)2 – Sdxdy + T(dx)2 = 0 become 2 2 x dpdy – y dqdx – (xp – yq)dxdy = 0 ...(2) 2 2 2 2 and x (dy) – y (dx) = 0 ...(3) Re–writing (3), (xdy – ydx) (xdy + ydx) = 0 so that xdy – ydx = 0 that is, xdy = ydx ...(4) and xdy + ydx = 0 ...(5) From (4), so that y/x = c ...(6) (1/ y)dy ∃ (1/ x )dx ) 0 1 For the required solution, we consider relation (4) only.

( xdp)( xdy) ∃ ( ydq)( ydx) ∃ ( pdx)( xdy) # (qdy)( ydx) ) 0

Re–writing (2),

( xdp)( ydx) ∃ ( ydq)( ydx ) ∃ ( pdx)( ydx) # (qdy)( ydx) ) 0 , by (4)

or or

xdp ∃ ydq ∃ pdx # qdy ) 0

or

{d(xp) ∃ pdx} ∃ {d(yq) ∃ qdy} ∃ pdx # qdy ) 0

or

d( xp ∃ yq) ∃ 2 pdx # 2qdy ) 0

or

d ( xp ∃ yq) ∃ 2 p dx # 2( y / x)dx = 0, by (4)

or

d ( xp ∃ yq) ∃ (2 / x)( xp ∃ yq)dx ) 0

d ( xp ∃ yq) 2dx ∃ )0 xp ∃ yq x

or

log( xp ∃ yq) ∃ 2 log x ) c2

Integrating,

( xp ∃ yq ) / x 2 ) c2

or

...(7)

From (6) and (7), one intermediate integral of (1) is ( xp ∃ yq ) / x 2 ) f ( y / x )

xp ∃ yq ) x 2 f ( x / y)

or

...(8)

which is of Lagrange’s form. Its Lagrange’s auxiliary equations are dx dy dz ) ) 2 x ∃ y x f (y / x)

or

Taking the first two ratios of (9), (1/x) dx + (1/y) dy = 0 so that xy = c3, c3 being an arbitrary constant Taking the first and last fractions of (9), we get

dz ) x f ( y / x)dx

...(9) log x # log y ) c3 ...(10)

dz ) x f ( c3 / x 2 ) , since by (10), y ) c3 / x

or

c3 2c3 1 x 4 2 1 c3 21 2c3 2 1 c32 2 f (t )dt , putting 2 ) t and ∃ 3 dx ) dt + z ) 8 3∃ 4 f 3 2 43 ∃ 3 4 dx ) 8 3 ∃ 2 4 x x 5 2c3 6 5 x 65 x 6 5 2c3t 6 c3 2

8

f (t )

or

z)∃

or

z ∃ c3F(c3 / x 2 ) ) c4

t2

dt # c4 ) c3 F (t ) # c4 ,

where or

F (t ) ) ∃

1 2

8

f (t ) t2

dt

z ∃ xy F ( y / x ) ) c4 , by (10) ... (11)

From (10) and (11), the required solution is

z ∃ xy F( y / x) ) G( xy) or

or

z ) x 2 ( y / x) F ( y / x) # G( xy)

z ) x2 H ( y / x) # G( xy) where H ( y / x) ) ( y / x ) F ( y / x) and H, G are arbitrary functions.

Created with Print2PDF. To remove this line, buy a license at: http://www.software602.com/

Monge’s Methods

9.21 2

2

Ex. 12. Solve 2x r – 5xys + 2y t + 2(px + qy) = 0. and hence find the surface satisfying the above equation and touching the hyperbolic paraboloid z = x2 – y2 along its section by the plane y = 1. [Meerut 2001, I.A.S. 1978, Ranchi 2010] Sol. Given 2x2r – 5xys + 2y2t = –2(px + qy). ...(1) 2 2 Comparing (1) with Rr + Ss + Tt = V, R = 2x , S = –5xy, T = 2y , V = –2 (px + qy) Hence Monge’s subsidiary equations Rdpdy + Tdqdx – Vdxdy = 0 and R(dy)2 – S dxdy + T(dx)2 = 0. become 2x2dpdy + 2y2dqdx + 2(px + qy)dxdy = 0. ...(2) and 2x2(dy)2 + 5xydxdy + 2y2(dx)2 = 0. ...(3) Re–writing (3), (xdy + 2ydy) (2xdy + ydx) = 0. so that xdy + 2ydx = 0, i.e., xdy = – 2ydx ...(4) and 2xdy + ydx = 0. ...(5) Keeping (4) in view, (2) may be re–written as 2xdp(xdy) – ydq (– 2ydx) + 2pdx (xdy) – qdy (– 2ydx) = 0. or 2xdp(xdy) – ydq (xdy) + 2pdx (xdy) – qdy (xdy) = 0, using (4) or 2xdp – ydq + 2pdx – qdy = 0 or 2(xdp + pdx) – (ydq + qdy) = 0 or 2d(xp) – d(yq) = 0 so that 2xp – yq = c1. ...(6) From (4), (1/y)dy + 2(1/x)dx = 0 so that log y + 2 log x = log c2 or log y + log x2 = log c2 or x2y = c2. ...(7) From (6) and (7), one intermediate integral is 2xp – yq = f(x2y), f being an arbitrary function. ...(8) which is of Lagrange’s form. Hence Lagrange’s subsidiary equations are dy dx = = dz2 . ...(9) 2x ∃y f ( x y) Taking the first two fractions of (9), 2(1/y)dy + (1/x)dx = 0. Integrating, 2 log y + log x = log a or y2x = a or x = a/y2. ...(10) Taking the second and third fractions of (9) and using (10), we get dy dz = 2 3 ∃y f (a / y )

F I GH JK

2 dz + 1 f a 3 dy = 0. y y

or

...(11)

Putting (a2/y3) = v so that – (3a2/y4) dy = d v , (11) gives dz + or or

or

FG H

IJ K

1 y4 f( v ) × ∃ 2 d v = 0 y 3a

or

dz –

f ( v) 3(a 2 / y 3 )

dv = 0

dz – (1/3 v ) × f( v ) dv = 0, as v = a2/y3. Integrating, z – F( v ) = b or z – F(a2/y3) = b, b being an arbitrary constant. 2 z – F(x y) = b, as y2x = a. ...(12) From (10) and (12), the required complete solution is z – F(x2y) = G(xy2), F and G being arbitrary fucntions. z = F(x2y) + G(xy2). ...(13) 2 2 Second Part. The given surface is z=x –y. ...(14) (13) − p = z/ x = 2xy F 0 (x2y) + y2G 0 (xy2) and q = z/ y = x2F 0 (x2y) + 2xyG0(xy2). ...(15) From (14), p = z/ x = 2x and q = x/ y = –2y. ...(16)

Created with Print2PDF. To remove this line, buy a license at: http://www.software602.com/

9.22

Monge’s Methods

Since (13) and (14) touch each other along their section by the plane y = 1, the values of p and q given by (15) and (16) at any point on y = 1 must be equal Thus, 2xyF 0 (x2y) + y2G0(xy2) = 2x, where y = 1 ...(17) 2 2 2 and x F 0 (x y) + 2xy G0(xy ) = –2y, where y = 1. ...(18) 2 From (17), 2xF 0 (x ) + G0(x) = 2x. ...(19) From (18), x2F 0 (x2) + 2xG0(x) = –2. ...(20) 2 Solving (19) and (20) for F 0 (x ) and G0(x), we have F0(x2) = (4/3) + (2/3) × (1/x2). ...(21) and G0(x) = – (2/3)× x – (4/3) × (1/x). ...(22) 2 (21) − F0(u) = (4/3) + (2/3) × (1/u), on putting x = u Integrating, This −

F(u) = (4/3) × u + (2/3) × log u + c1, c1 being an arbitrary constant F(x2y) = (4/3) × x2y + (2/3) × log (x2y) + c1.

...(23)

Integrating (22), G(x) = – (2/3) (x2/2) – (4/3) log x + c2, being an arbitrary constant This − G(xy2) = – (1/3) × x2y4 – (4/3) × log (xy2) + c2. ...(24)

or or or or or

Putting values of F(x2y) and G(xy2) given by (23) and (24) in (13), we get z = (4/3) × x2y + (2/3) × log (x2y) + c1 – (1/3) × x2y4 – (4/3) × log (xy2) + c2 z = (4/3) × x2y – (1/3)× x2y4 + (2/3) × [log (x2y) – 2 log (xy2)] + c, taking c1 + c2 = c z = (4/3)× x2y – (1/3) × x2y4 + (2/3) × [log (x2y) – log (xy2)2] z = (4/3) × x2y – (1/3) × x2y4 + (2/3) × [log {(x2y)/(x2y4)} + c z = (4/3) × x2y – (1/3) × x2y4 + (2/3) × log y–3 + c z = (4/3) × x2y – (1/3) × x2y4 – 2 log y + c. ...(25) Now at the point of contact of (14) and (25), the values of z must be the same and hence x2 – y2 = (4/3) × x2y – (1/3) × x2y4 – 2 log y + c, where y = 1 x2 – 1 = (4/3) × x2 – (1/3) × x2 + c, putting y = 1 −

x2 – 1 = x2 + c − c = –1. − Putting c = –1 in (25), the required surface is z = (4/3) × x2y – (1/3) × x2y4 – 2 log y – 1 or 3z = 4x2y – x2y4 – 6 log y – 3. 9.7. Type 3. When the given equation Rr + Ss + Tt = V leads to two identical intermediate intergrals. Working rule for solving problems of type 3 Step 1. Write the given equation in the standard form Rr + Ss + Tt = V. Step 2. Substitute the values of R, S, T and V in the Monge’s subsidiary equations Rpdy + Tdqdx – Vdx dy = 0 ... (1) R(dy)2 – S dxdy + T(dx)2 = 0 ... (2) Step 3. R.H.S. of (2) reduces to a perfect square and hence it gives only one distinct factor in place of two as in type 1 and type 2. Step 4. Start with the only one factor of step 3 and use (2) to get an intermediate integral. Step 5. Re–write the intermediate integral of the step 4 in the form of Pp + Qq = R and use Lagrange’s method to obtain the required general solution of the given equation. 9.8. Solved examples based on Art 9.7 Ex. 1. Solve : (1 + q)2r – 2(1 + p + q + pq)s + (1 + p)2t = 0 [Meerut 2002, Delhi Maths (H) 1999 2007, 10; Rohailkhand 1997; Kanpur 1994]

Created with Print2PDF. To remove this line, buy a license at: http://www.software602.com/

Monge’s Methods

9.23

Sol. Comparing the given equation with Rr + Ss + Tt = V, ...(1) 2 2 R = (1 + q) , S = –2(1 + p + q + pq), T = (1 + p) , V = 0. ...(2) Monge’s subsidiary equations are Rdpdy + Tdqdx – Vdxdy = 0 ...(3) 2 2 and R(dy) – Sdxdy + T(dx) = 0. ...(4) Using (2), (3) and (4) become (1 + q)2dpdy + (1 + p)2dqdx = 0 ...(5) 2 2 2 2 and (1 + q) (dy) + 2(1 + p + q + pq)dxdy + (1 + p) (dx) = 0. ...(6) Since 1 + p + q + pq = (1 + p) (1 + q), (6) becomes [(1 + q)dy + (1 + p)dx]2 = 0 so that (1 + q)dy + (1 + p)dx = 0 or (1 + q)dy = –(1 + p)dx. ...(7) Keeping (7) in view, (5) may be re–written as (1 + q)dp {(1 + q)dy} – (1 + p)dq {– (1 + p)dx} = 0. ...(8) Dividing each term of (8) by (1 + q)dy, or its equivalent –(1 + p)dx, we get (1 + q)dp – (1 + p)dq = 0 or dp/(1 + p) – dq/(1+ q) = 0. Integrating it, (1+ p)/(1 + q) = c1, c1 being an arbitrary constant ...(9) From (7), dx + dy + pdx + qdy = 0 or dx + dy + dz = 0, as dz = pdx + qdy Integrating it, x + y + z = c2, c2 being an arbitrary constant ...(10) From (9) and (10), one intermediate integral of (1) is (1 + p)/(1 + q) = F(x + y + z) or 1 + p = (1 + q) F(x + y + z) or p – q F(x + y + z) = F (x + y + z) – 1, ...(11) which is of the form Pp + Qq = R. So Lagrange’s auxiliary equations are dy dx dz = = ...(12) 1 ∃ F( x # y # z) F( x # y # z) ∃1 Choosing 1, 1, 1 as multipliers, each fraction of (12) = (dx + dy + dz)/0 so that dx + dy + dz = 0 giving x + y + z = c2 ... (13) Using (13) and taking the first two fractions of (12), we have dx = – dy/F(c2) or dy + F(c2)dx = 0. Integrating it, y + xF(c2) = c3 or y + x F(x + y + z) = c3 ...(14) From (13) and (14), the required general solution is y + x F(x + y + z) = G(x + y + z), F, G being arbitrary functions. 2 Ex. 2. Solve y r + 2xys + x2t + px + qy = 0. [Bilaspur 2004] Sol. Given y2r + 2xys + x2t = – (px + qy). ...(1) Comparing (1) with Rr + Ss + Tt = V, here R = y2, S = 2xy, T = x2, V = – (px + qy). ...(2) Monge’s subsidiary equations are Rdpdy + Tdqdx + Vdxdy = 0 ...(3) and R(dy)2 – Sdxdy + T(dx)2 = 0. ...(4) Using (2), (3) and (4) become y2dpdy + x2dqdx + (px + qy) dxdy = 0 ...(5) and y2(dy)2 – 2xydxdy + x2(dx)2 = 0. ...(6) 2 From (6), (xdx – ydy) = 0 so that xdx – ydy = 0 or xdx = ydy. ...(7) Keeping (7) in view, (5) may be re–written as ydp (ydy) + xdq (xdx) + pdy (xdx) + qdx (ydy) = 0. ...(8) Dividing each term of (8) by xdx, or its equivalent ydy, we get ydp + xdq + pdy + qdx = 0 or (ydp + pdy) + (xdq + qdx) = 0 Integrating it, yp + xq = c1, being an arbitrary constant ...(9) Integrating (7), x2/2 – y2/2 = c2/2 or x2 – y2 = c2. ...(10)

Created with Print2PDF. To remove this line, buy a license at: http://www.software602.com/

9.24

Monge’s Methods 2

From (9) and (10), one intermediate integral of (1) is yp + xq = F(x – y2), which is of the form Pp + Qq = R. Its Lagrange’s auxiliary equations are dx = dy = dz 2 2 . x y F( x ∃ y ) From the first two fractions of (2), xdx – ydy = 0 so that x2 – y2 = c2. Taking the last two fractions and using (13), we get dy dz = F(c2 ) ( y 2 # c2 )1/ 2

or

Integrating,

z – F(c2) log [y +(y2 + c2)1/2] = c3

dz – F(c2)

...(11) ...(12) ...(13)

dy = 0. ( y 2 # c2 )1/ 2

z – F(x2 – y2) log [y + ( y 2 # x 2 ∃ y 2 ) ] = c3, using (13)

or

z – F(x2 – y2) log (x + y) = c3, c3 being an arbitrary constant ...(14) From (13) and (14), the required general solution is z – F(x2 – y2) log (x + y) = G(x2 – y2), F, G being arbitrary functions. Ex. 3(a). Obtain the integral of q2r – 2pqs + p2t = 0 in the form y + xf(z) = F(z). [Delhi Maths Hons. 1999, 2007; Meerut 1994, 95; Nagpur 2005] (b) Show also that this solution represents a surface generated by straight lines that are parallel to a fixed plane. Sol. (a) Given q2r – 2pqs + p2t = 0. ...(1) As ususal Monge’s subsidiary equations are q2dpdy + p2dp dx = 0 ...(2) 2 2 2 2 2 and q (dy) + 2pqdxdy + p (dx) = 0 or (qdy + pdx) = 0. ...(3) From (3), we have qdy + pdx = 0 or qdy = –pdx. ...(4) In view of (4), (2) may be re–written as qdp (qdy) – pdq (–pdx) = 0. ...(5) Dividing each term of (5) by qdy, or its equivalent (–pdx), we find qdp – pdq = 0 or (1/p)dp – (1/q) dp = 0. Integrating it, p/q = c1, c1 being an arbitrary constant ...(6) From (4), dz = 0, (as dz = pdx + qdy) so that z = c2. ...(7) From (6) and (7), one integral of (1) is p/q = f(z) or p – f(z)q = 0, ...(8) which is of the form Pp + Qq = R. Here f is an arbitrary function. Its Lagrange’s auxiliary equations dx = dy = dz . are ...(9) ∃ f ( z) 0 1 The last fraction in (9) gives dz = 0 so that z = c2 ...(10) From the first two fractions in (9) and (10), we find or

dy dx = 1 ∃ f (c2 )

or

dy + f(c2)dx = 0.

Integrating, y + xf(c2) = c3 or y + xf(z) = c3, by (10). ...(11) From (10) and (11), the required integral is y + xf(z) = F(z). ...(12) Part (b). Let z = k, k being an arbitrary constant. Then (12) is the locus of the straight lines given by the intersection of the planes z=k and y + xf(k) – F(k) = 0. ...(13) Clearly the lines are parallel to the plane z = 0 (which is a fixed plane) because these lie on the plane z = k for different values of k. Ex. 4. Solve y2r – 2ys + t = p + 6y. [Agra 1993; Bhopal 2004; Vikram 2004; Meerut 2009; Delhi Maths Hons 1994, 98, 2006, 09, 10]

Created with Print2PDF. To remove this line, buy a license at: http://www.software602.com/

Monge’s Methods

Sol. As usual Monge’s subsidiary equations are y2dpdy + dqdx – (p + 6y)dxdy = 0 2 2 and y (dy) + 2ydydx + (dx)2 = 0 or (ydy + dx)2 = 0. From (2), ydy + dx = 0 or dx = –ydy. Putting the value of dx from (3) in (1), we find y2dpdy + dq(–ydy) – (p + 6y) dy (–ydy) = 0 or ydp – dq + (p + 6y) dy = 0 or (ydp + pdy) – dq + 6ydy = 0. Integrating it, yp – q + 3y2 = c1, c1 being an arbitrary constant Integrating (4), y2/2 + x = c2/2 or y2 + 2x = c2. From (5) and (6), one integral of (1) is yp – q + 3y2 = F(y2 + 2x) or yp – q = F(y2 + 2x) – 3y2, which is of the form Pp + Qq = R. Its Lagrange’s auxiliary equations are

9.25

...(1) ...(2) ...(3)

...(4) ...(6) ...(7)

dz dx = dy = ...(8) 2 2 . ∃1 y F( y # 2 x) ∃ 3y From the first two fractions of (8), 2ydy + 2dx = 0 so that y2 + 2x = c2. ...(9) Taking the last two fractions of (8) and using (9), dz + [F(c2) – 3y2]dy = 0. 3 Integrating, z + yF(c2) – y = c2 or z + yF(y2 + 2x) – y3 = c3. ...(10) From (9) and (10), the required general solution is z + yF(y2 + 2x) – y3 = G(y2 + 2x), F, G being arbitrary functions. Ex. 5. Solve (b + cq)2r – 2(b + cq) (a + cp)s + (a + cp)2t = 0 Sol. Usual Monge’s subsidiary equations are (b + cq)2 dpdy + (a + cp)2 dqdx = 0. ...(1) and (b + cq)2 (dy)2 + 2(b + cq) (a + cp) dxdy + (a + cp)2 (dx)2 = 0. ...(2) 2 (2) − {(b + cq)dy + (a + cp)dx} = 0 ...(3) or (b + cq)dy + (a + cp)dx = 0 or adx + bdy + c(pdx + qdy) = 0 or adx + bdy + cdz = 0, as dz = pdx + qdy. Integrating, ax + by + cz = c1, c3 being an arbitrary constant ...(4) From (3), (b + cq)dy = –(a + cp)dx. So (1) reduces to (b + cq)dp – (a + cp)dq = 0 dp dq a # cp ∃ or =0 so that = c2 ...(5) a # cp b # cq b # cq So the intermediate integral of the given equation is (a + cp)/(b + cq) = !1(ax + by + cz) or cp – c!1(ax + by + cz)q = – a + b !1(ax + by + cz). ...(6) Lagrange’s auxiliary equations are dy dx dz = = . ...(7) c ∃c !1(ax # by # cz) ∃a # b !1(ax # by # cz) Using a, b, c as multipliers, each fraction of (7) = (adx + bdy + cdz)/0 adx + bdy + cdz = 0 so that ax + by + cz = c3. ...(8) + Using (8) and taking the first two ratios of (7), we get dx = – dy/!1(c3) or dy + !1(c3)dx = 0. Integrating, y + x!1(c3) = c4 or y + x!1(ax + by + cz) = c4. ...(9) From (8) and (9), the required solution is y + x!1(ax + by + cz) = !2(ax + by + cz), !1, !2 being arbitrary functions. Ex. 6. Solve x2r – 2xs + t + q = 0. [K.U. Kurukshetra 2004; Ravishankar 2005] Sol. Usual Monge’s subsidiary equations are x2dpdy + dqdx + qdxdy = 0 ...(1) 2 2 2 and x (dy) + 2xdxdy + (dx) = 0. ...(2)

Created with Print2PDF. To remove this line, buy a license at: http://www.software602.com/

9.26

Monge’s Methods 2

(2) − (3) − Using (3), (1) reduces to

(xdy + dx) = 0 − xdy + dx = 0 (dx)/x + dy = 0 − y + log x = c1. 2 x dpdy + dq (–x dy) + q(– x dy)dy = 0

Now,

or

dp –

FG dq ∃ qdx IJ = 0 Hx x K

FH

d p∃

or

2

...(3) ...(4)

IK

q = 0. x

Integrating, p – (q/x) = c2, c2 being an arbitrary constant From (4) and (5), the intermediate integral of the given equation is p – (q/x) – !1(y + log x) or xp – q = x!1(y + log x). Lagrange’s auxiliary equations for (6) are Taking the first two fractions of (7),

...(5) ...(6)

dz dx = dy = . ∃1 x !1( y # log x) x (1/x)dx + dy = 0 − y + log x = c3.

Using (8), first and third fractions of (7) give

...(7) ...(8)

dx = dz = − z – x!1(c3) = c4 x x !1(c3 )

z – x!1(y + log x) = c4, c4 being an arbitrary constant ...(9) From (8) and (9) the required solution is z – x!1(y + log x) = !2(y + log x), !1, !2 being arbitrary functions. Ex. 7. Solve (y – x) (q2r – 2pqs + p2t) = (p + q)2 (p – q). Sol. The usual Monge’s subsidiary equations are (y – x) (q2dpdy + p2dqdx) – (p + q)2 (p – q)dxdy = 0 ...(1) 2 2 2 2 and q (dy) + 2pqdxdy + p (dx) = 0. ...(2) (2) − (qdy + pdx)2 = 0 or qdy + pdx = 0. ...(3) dz = pdx + qdy and (3) dz = 0 − z = c1. ...(4) − 2 2 Using (3), (1) reduces to (y – x) (qdp – pdq) – (p – q ) (dx – dy) = 0 or

q2d

or

FG pIJ –(p H qK

2

– q2)

b g

d x∃y =0 y∃x

or

b g # d b p / qg

d x∃ y x∃y

( p / q) 2 ∃ 1

=0

p∃q 1 ( p / q) ∃ 1 1 log = log c2 or (x – y)2 = c2. ...(5) 2 p#q ( p / q) # 1 2 From (4) and (5), the intermediate integral of the given equation is p∃q (x – y)2 = !1(z) or (x – y)2(p – q) = (p + q)!1(z) p#q

Integrating,

or

log (x – y) +

p{(x – y)2 – !1(z)} – q{(x – y)2 + !1(z)} = 0. Here Lagrange’s subsidiary equation for (6) are dz dy dx = = . 2 2 0 ( x ∃ y) ∃ !1(z) ∃ {( x ∃ y) # !1(z)}

Now, the third fraction of (7) − where ‘a’ is an arbitrary constant. Now, each fraction of (7) =

dz = 0

dx ∃ dy dx # dy = ∃ 2!1( z) 2( x ∃ y)2

so that − d(x + y) = – !9(a)

...(6) ...(7) z = a, ...(8) d ( x ∃ y) , by (8). ( x ∃ y) 2

Integrating it, x + y – !1(a) (x – y)–1 = b or x + y – !1(z) (x – y)–1 = b, using (8). ...(9) From (8) and (9), the required general solution is x + y – (x – y)–1 !1(z) = !2(z), !1, !2 being arbitrary functions.

Created with Print2PDF. To remove this line, buy a license at: http://www.software602.com/

Monge’s Methods

9.27 2

2

Ex. 8. Solve x r + 2xys + y t = 0. [Meerut 2003, Garhwal 1993; Delhi Maths (H) 2001] Sol. Comparing the given equation with Rr + Ss + Tt = V, we get R = x2, S = 2xy, T = y2. Hence the usual Monge’s subsidiary equations Rdpdy + Tdqdx – Vdxdy = 0 and R(dy)2 – S dxdy + T(dx)2 = 0 become x2dpdy + y2dqdx = 0 ...(1) 2 2 and x (dy) – 2xydxdy + y2(dx)2 = 0. ...(2) 2 Now, (2) gives (xdy – ydx) = 0 so that xdy – ydx = 0. ...(3) Re–writing (1), (xdp) (xdy) + (ydx) (ydq) = 0 or (xdp)(xdy) + (xdy) (ydq) = 0 [ from (3), ydx = xdy] or xdp + ydq = 0 or xdp + ydq + pdx + qdy = pdx + qdy or d(xp) + d(yq) – dz = 0, as dz = pdx + qdy. Integrating (1) xp + yq – z = c1, c1 being an arbitrary constant ...(4) Now (3) gives (1/y)dy – (1/x)dx = 0. Integrating, log y – log x = log c2 or y/x = c2. ...(5) From (4) and (5), the intermediate integral of the given equation is xp + yq – z = f(y/x) or xp + yq = z + f(y/x), ...(6) where f is an arbitrary function. Lagrange’s subsidiary equation for (6) are dy dz dx = = . ...(7) z # f ( y / x) x y Taking the first two fractions of (7), (1/y)dy – (1/x)dx = 0. Integrating, log y – log x = log a so that y/x = a. ...(8) dz ∃ dy = 0. z # f (a) y Integrating it, log [z + f(a)] – log y = log b, b being an arbitrary constant so that [z + f(a)]/y = b or [z + f(y/x)]/y = b, using (8) ...(9) From (8) and (9), the required solution is [z + f(y/x)]/y = g(y/x) or z = yg(y/x) – f(y/x), where f and g are arbitrary functions. Ex. 9. Solve r – 2s + t = sin (2x + 3y). Sol. Comparing the given equation with Rr + Ss + Tt = V, we have R = 1, S = –2, T = 1, V = sin (2x + 3y). So Monge’s subsidiary equations Rdpdy + Tdqdx – Vdxdy = 0 and R(dy)2 – Sdxdy + T(dx)2 = 0 become dpdy + dqdx – sin (2x + 3y)dxdy = 0. ...(1) and (dy)2 + 2 dxdy + (dx)2 = 0. ...(2) Now, (2) gives (dy + dx)2 = 0 so that dy + dx = 0. ...(3) From (3), dy = – dx. Then, (1) becomes – dpdx + dqdx + sin (2x + 3y)dxdy = 0 or dp – dq + sin (2x + 3y)dy = 0, as dx 7 0. ...(4) Now, integrating (3), x + y = c1, c1 being an arbitrary constant ...(5) From (4), dp – dq + sin [2(x + y) + y]dy = 0 or dp – dq + sin (2c1 + y)dy = 0, using (5). Integrating, p – q – cos (2c1 + y) = c2 or p – q – cos(2x + 3y) = c2, as c1 = x + y ...(6) From (5) and (6), an intermediate integral is p – q – cos (2x + 3y) = f(x + y) or p – q = cos (2x + 3y) + f(x + y), ...(7) where f is an arbitrary function. Its Lagrange’s auxiliary equations are

Taking the last two fractions of (7) and using (8), we get

Created with Print2PDF. To remove this line, buy a license at: http://www.software602.com/

9.28

Monge’s Methods

dy dx dz = = . 1 ∃1 cos(2 x # 3y) # f ( x # y) Taking the first two fractions of (8), dx + dy = 0 so that Taking the last two fractions of (8) and using (9), we get

dz dy = ∃1 cos(2a # y ) # f (a )

...(8) x+y=a

... (9)

dz + [cos (2a + y) + f(a)]dy = 0.

or

z + sin (2a + y) + y f(a) = b, b being an arbitrary constant z + sin (2x + 3y) + y f(x + y) = b, using (9). ...(10) From (9) and (10) the required complete integral is z + sin (2x + 3y) + y f(x + y) = g(x + y), f and g being an arbitrary functions. Ex. 10. Solve q2r – 2pqs + p2t = pq2. [I.A.S. 1986] Sol. Comparing the given equation with Rr + Ss + Tt = V, we have R = q2, S = –2pq, T = p2, V = pq2. The Monge’s subsidiary equations Rdpdy + Tdqdx – Vdxdy = 0 and R(dy)2 – Sdxdy + T(dx)2 = 0 become q2dpdy + p2dqdx – pq2dxdy = 0 ...(1) and q2(dy)2 + 2pqdxdy + p2(dx)2 = 0. ...(2) 2 Re–writing (2), (qdy + pdx) = 0 so that pdx + qdy = 0. ...(3) Since dz = pdx + qdy, (3) − dz = 0 so that z = c1. ...(4) Re–writing (1), (qdy)(qdp) + (pdx)(pdq) – (qdy)(pqdx) = 0 or (qdy)(qdp) – (qdy)(pdq) – (qdy)(pqdx) = 0, as from (3), pdx = –qdy or qdp – pdq – pqdx = 0 or (1/p)dp – (1/q)dq = dx. Integrating, log p – log q – log c2 = x or p/(c2q) = ex –x or (p/q)e = c2, c2 being an arbitrary constant ...(5) From (4) and (5), the intermediate integral of the given equation is (p/q)e–x = f(z) or px–x – f(z)q = 0. ...(6) Integrating it,

or

dx = dy = dz . ∃ f ( z) 0 e∃ x The last fraction of (7) − dz = 0 so that z = a. Taking the first fractions of (7) and using (8), we get Lagrange’s auxiliary equations for (6) are

...(7) ...(8)

dx = dy or exf(a)dx + dy = 0. ∃ f (a ) e∃ x Integrating, exf(a) + y = b or exf(z) + y = b, as from (8), a = z From (8) and (9), the required complete integral is exf(z) + y = g(z), where f and g are arbitrary functions.

...(9)

Ex. 11. Solve q2r ∃ 2q(1 # p)s # (1 # p)2 t ) 0 by Monge’s method.

q2r ∃ 2q (1 # p)s # (1 # p)2 t ) 0

Sol. Given

Comparing (1) with Rr # Ss # Tt ) V , here

R ) q2 ,

S ) ∃2q(1 # p)

... (1) and

T ) (1 # p)2 .

Hence Monge’s subsidiary equations

Rdpdy # Tdqdx ∃ Vdx dy ) 0

and

R(dy)2 ∃ Sdxdy # T (dx)2 ) 0 become

Created with Print2PDF. To remove this line, buy a license at: http://www.software602.com/

Monge’s Methods

9.29

and

... (2)

q2 (dy)2 # 2q(1 # p)dxdy # (1 # p)2 (dx )2 ) 0

... (3)

{qdy # (1 # p)dx}2 ) 0

Rewriting (3), From (4),

q2 dpdy # (1 # p)2 dqdx ) 0

dx # ( pdx # qdy) ) 0

dx # dz ) 0 ,

or

Re–writing (2),

... (5)

(qdy) (qdp) # [(1 # p) dx}/ {(1 # p)dq} ) 0

(qdy) (qdp) # (∃qdy) [(1 # p) dq] ) 0, using (4)

or

qdp ∃ (1 # p) dq ) 0

{1/(1 # p)}dp ∃ (1/ q)dq ) 0

or

Integrating, log(1 # p) ∃ log q ) log C2 or From (5) and (6), the intermediate integral of (1) is

(1 # p) / q ) C2

... (6)

(1 # p) / q ) f ( x # z) p ∃ q f ( x # z) ) ∃1 or which is of Lagrange’s form. Its Lagrange’s auxiliary equations are

... (7)

dx dy dz ) ) 1 ∃ f ( x # z ) ∃1

... (8)

Taking the first and last ratios, dx + dz = 0 − Using (9) and taking the first two ratios of (8), we get dy # f (C3 )dx ) 0 or

... (4)

dz ) pdx # qdy

as

x # z ) C1 , C1 being an arbitrary constant

Integrating,

or

qdy # (1 # p) dx ) 0

or

x # z ) C3

... (9)

y # xF (C3 ) ) C4

so that

y # x f ( x # z) ) C4 , using (9) From (9) and (10), the required general solution is

... (10)

y # xf ( x # z) ) g( x # z) , f, g are arbitrary functions Ex. 12. Solve ( x ∃ y) ( x 2 ∃ 2 xys # y2 t ) ) 2 xy ( p ∃ q) .

[Delhi B.Sc. (Hons) 2011]

x 2 ( x ∃ y)r ∃ 2 xy( x ∃ y)s # y2 ( x ∃ y)t ) 2 xy( p ∃ q)

Sol. Given

... (1)

Comparing (1) with Rr # Ss # Tt ) V , here R ) x 2 ( x ∃ y), S = –2xy (x – y), T = y2 (x – y) and V = 2xy (p – q). Hence Monge’s subsidiars equations

Rdpdy # Tdqdx ∃ Vdxdy ) 0

and

... (2)

( x ∃ y){x2 (dy)2 # 2 xy dx dy # y 2 (dy) 2 } ) 0

... (3)

ydx ) ∃ xdy ... (4)

( xdy # ydx)2 ) 0

so that

From (4),

(1/ x)dx # (1/ y)dy ) 0

so that

Re–writing (2),

x( x ∃ y) dp ( xdy) ∃ 2( p ∃ q)( xdy) ( ydx) # y( x ∃ y) dq( ydx) ) 0

Since x 7 y , (3) gives

or

become

x 2 ( x ∃ y) dpdy ∃ 2 xy ( p ∃ q) dxdy # y2 ( x ∃ y)dqdx ) 0 and

or

R(dy)2 ∃ S dyxy # T (dx)2 ) 0

xy ) C1

... (5)

x( x ∃ y) dp ( xdy) ∃ 2( p ∃ q) ( xdy) ( ydx) # y( x ∃ y)dq(∃ xdy), by(4) x( x ∃ y)dp ∃ 2 ( p ∃ q) ( ydx) ∃ y( x ∃ y) dq ) 0

Created with Print2PDF. To remove this line, buy a license at: http://www.software602.com/

9.30

Monge’s Methods

( x ∃ y) ( xdp ∃ ydq) ) 2 y ( p ∃ q)dx

or

xdp ∃ ydq ) {2 y( p ∃ q)dx}/( x ∃ y)

or

or

( xdp # pdx) ∃ ( ydq # qdy) ) {2 y( p ∃ q) dx}/( x ∃ y) # pdx ∃ qdy

or

d( xp) ∃ d ( yq) ) {2( p ∃ q) ydx # ( x ∃ y) p dx ∃ ( x ∃ y) qdy}/( x ∃ y) ( x ∃ y)d ( xp ∃ yq) ) 2 pydx ∃ 2qydx # xpdx ∃ ypdx ∃ xqdy # yqdy

or

= pydx ∃ 2qydx # xpdx # qydx # yqdy = ∃ pxdy ∃ qydx # xpdx # yqdy, by (4) +

or

( x ∃ y) d( xp ∃ yq) ) xp(dx ∃ dy) ∃ yq(dx ∃ dy) ) ( xp ∃ yq)(dx ∃ dy) d ( xp ∃ yq) dx ∃ dy ) xp ∃ yq x∃y

Integrating,

d ( xp ∃ yq) d ( x ∃ y) ∃ ) 0. xp ∃ yq x∃y

or

log ( xp ∃ yq) ∃ log( x ∃ y) ) log C2

or

( xp ∃ yq) /( x ∃ y) ) C2

... (6)

From (5) and (6), the intermediate integral of the given equation is (xp – yq)/(x – y) = f (xy)

xp ∃ yq ) ( x ∃ y) f ( xy) ,

or

which is of Lagrange’s form. Its auxiliary equations are Taking the first two fractions, Now, or

(1/ x) dx # (1/ y)dy ) 0

each fraction of (8) =

dz ) f ( xy) d ( x # y)

From (9) and (10), the required solution is or

so that

xy ) C3

... (8) ... (9)

dx # dy dz ) x∃y ( x ∃ y) f ( xy)

dz ) f (C3 ) d ( x # y) , by (9)

or

Integrating, z ∃ ( x # y) f (C3 ) ) C4

dx dy dz ) ) x ∃ y ( x ∃ y) f ( xy)

... (7)

or

z ∃ ( x # y) f ( xy) ) C4 ... (10)

z ∃ ( x # y) f ( xy) ) g( xy)

z ) ( x # y) f ( xy) # g( xy) , f and g being arbitrary functions.

9.9 Type 4. When the given equation Rr + Ss + Tt = V fails to yield an intermediate integral as in cases 1, 2 and 3. Working rule for solving problems of type 4. Suppose the R.H.S. of R(dy)2 – Sdxdy + T(dx)2 = 0 neither gives two factors nor a perfect square (as in Types 1, 2 and 3 above). In such cases factors dx, dy, p, 1 + p etc. are cancelled as the case may be and an integral of given equation is obtained as usual. This integral is then integrated by methods explained in chapter 7. 9.10 SOLVED EXAMPLES BASED ON ART 9.9 Ex. 1. Solve (q + 1)s = (p + 1)t. [Agra 2009] Sol. Given (q + 1)s – (p + 1)t = 0. ...(1) Comparing (1) with Rr + Ss + Tt = V, we find R = 0, S = (q + 1). T = –(p + 1), V = 0....(2) Monge’s subsidiary equations are Rdpdy + Tdqdx – Vdxdy = 0. ...(3) and R(dy)2 – Sdxdy + T(dx)2 = 0. ...(4) Using (2), (3) and (4) become – (p + 1)dqdx = 0 ...(5) and –(q + 1)dxdy – (p + 1)(dx)2 = 0. ...(6) Dividing (5) by – (p + 1)dx, we obtain dq = 0. ...(7) and dividing (6) by – dx we get (q + 1) + (p + 1)dx = 0. ...(8) From (8), dx + dy + pdx + qdy = 0 or dx + dy + dz = 0, as dz = pdx + qdy

Created with Print2PDF. To remove this line, buy a license at: http://www.software602.com/

Monge’s Methods

9.31

Integrating it, x + y + z = c1, being an arbitrary constant ...(9) Integrating (7), q = c2, c2 being an arbitrary constant ...(10) From (9) and (10), an integral of (1) is q = f(x + y + z) or z/ y = f(x + y + z) ...(11) Integrating (11) partially w.r.t. y (treating x as constant), we find z = F(x + y + z) + G(x), F, G being arbitrary functions. Ex. 2. Solve pq = x(ps – qr). [Delhi. Maths (H) 2002, 08] Sol. Given xqr – xps + 0.t = –pq. ...(1) Comparing (1) with Rr + Ss + Tt = V, R = xq, S = xp, T = o and V = –pq Monge’s subsidiary equations Rdp dy + T dq dx – V dx dy = 0 and R(dy)2 – Sdxdy + T(dx)2 = 0 become xqdpdy + pqdxdy = 0. ...(2) and xq(dy)2 + xpdxdy = 0. ...(3) Dividing (2) by qdy we get xdp + pdx = 0 ...(4) and dividing (3) by xdy, we get qdy + pdx = 0. ...(5) Using dz = pdx + qdy, (5) gives dz = 0 so that z = c1 ...(6) Integrating (4), xp = c2, c2 being an arbitrary constant ...(7) From (6) and (7), one integral of (1) is 1 z z 1 xp = f(z) or x = f(z) or = . x f (z ) x x Integrating it partially w.r.t. x, F(z) = log x + G(y), F, G being arbitrary functions. Ex. 3. Solve pt – sqs = q3 [MDU Rohtak 2004; Ravishankar 2004; Delhi Maths (H) 2005; Meerut 2005; 06 ; Rohilkhand 1994] Sol. Given pt – qs = q3 ... (1) Comparing (1) with Rr + Ss + Tt = V, here R = 0, S = –q, T = p, V = q3. 2 2 + Monge’s subsidiary equations Rdpdy + Tdqdx – Vdxdy = 0, R(dy) – Sdxdy + T(dx) = 0 3 become pdqdx – q dxdy = 0 ...(2) and qdxdy + p(dx)2 = 0. ...(3) 3 Dividing (2) by dx, we get pdq – q dy = 0 ...(4) and dividing (3) by dx, we get pdx + qdy = 0. ...(5) From (5), dy = – (pdx)/q. Putting this value of dy into (4) gives pdq – q3(pdx/q) = 0 or (1/q2)dq + dx = 0. Integrating it, –1/q + x = C1, C1 being an arbitrary constant ...(6) Using dz = pdx + qdy, (5) gives dz = 0 so that z = C2. ...(7) From (6) and (7), one integral of (1) is – 1 + x = f(z) q

or

z y = x – f(z), as q ) , z y

Integrating with respect to z partially (treat x as constant), we obtain y = xz – F(z) + G(x), F, G being arbitrary functions, where F ( z ) ) 2

Ex. 4. Solve z(qs – pt) = pq . Sol. Given The usual Monge’s subsidiary equations are and Dividing (2) by – pdx , we get

z

f ( z ) dz .

[Delhi Maths (H) 1998; 2004, 11] zqs – zpt = pq2. ...(1) – zpdqdx – pq2dxdy = 0 ...(2) 2 – zqdxdy – zp(dx) = 0. ...(3) zdq + q2dy = 0 ...(4)

Created with Print2PDF. To remove this line, buy a license at: http://www.software602.com/

9.32

Monge’s Methods

and dividing (3) by –z dx we get Using dz = pdx + qdy, (5) gives dz = 0 Using (6) in (4), C1dq + q2dy = 0 Integrating it, –1/q + y/C1 = C2 From (6) and (7), one integral of (1) is

or or

qdy + pdx = 0. so that z = C1. (1/q2)dq + (1/C1)dy = 0. –1/q + y/z = C2, by (6)

...(5) ...(6) ...(7)

y 1 y y ∃ y = – f(z), or as q ) ∃ 1 # = f(z) z z z q z which is linear in variables y and z (treating x as constant). Its integrating factor (I.F.) = e–(1/z)dz = e–log z = z –1 and so its solution is

z

yz–1 = – z ∃1 f (z) dz + G(x)

or

yz–1 = F(z) + G(x),

where

F (z) )

z

f ( z ) dz

or y = zF(z) + zG(x) or y = H(z) + zG(x), where H(z)[= zF(z)] and G(x) are arbitrary functions. Ex. 5. Solve 2yq + y2t = 1. Sol. Given equation is 0.r + 0.s + y2.t = 1 – 2yq. ...(1) Comparing (1) with Rr + Ss + Tt = V, here R = 0, S = 0, T = y2, V = 1 – 2yq. Hence the usual subsidiary equations Rdpdy + Tdqdx – Vdxdy = 0 and R(dy)2 – Sdxdy + T(dx)2 = 0 2 become y dqdx – (1 – 2yq)dxdy = 0 ...(2) and y2(dx)2 = 0. ...(3) From (3), dx = 0 so that x = c1. ...(4) From (2), y2dq + 2yq dy – dy = 0 or d(y2q) – dy = 0. 2 Integrating it, y q – y = c2, c2 being an arbitrary constant ...(5) From (4) and (5), an intermediate integral is y2q – y = f(x) or y2( z/ y) – y = f(x)

z / y ) 1/ y # (1/ y 2 ) / f ( x)

or

...(6)

Integrating (6) w.r. t. y, treating x as constant, we get z = log y – (1/y) f(x) + g(x) or yz = y log y – f(x) + y g(x), where f and g being arbitrary functions. Ex. 6. Solve (ex – 1) (qr – ps) = pqex. Sol. Given q(ex – 1)r – p(ex – 1)s = pqex. ...(1) Comparing (1) with Rr + Ss + Tt = V, R = q(ex – 1), S = –p(ex – 1), T = 0, V = pqex. Then the usual Monge’s subsidiary equations Rdpdy + Tdqdx – Vdxdy = 0 and R(dy)2 – Sdxdy + T(dx)2 = 0 become q(ex – 1)dpdy – pqexdxdy = 0 ...(2) and q(ex – 1)(dy)2 + p(ex – 1)dxdy = 0. ...(3) Now, (3) qdy + pdx = 0 dz = 0, as dz = pdx + qdy. − − Integrating, z = c1, c1 being an arbitrary constant ...(4) Again, from (2),

(ex – 1)dp – pexdx = 0

or

dp ex ∃ x dx = 0 p e ∃1

Integrating, log p – log (ex – 1) = log c2 or p/(ex – 1) = c2. ...(5) x From (4) and (5), an intermediate integral is p/(e – 1) = f(z), f being an arbitrary function

Created with Print2PDF. To remove this line, buy a license at: http://www.software602.com/

Monge’s Methods

or

9.33

1 z = ex – 1. f (z ) x

z = (ex – 1)f(z), or x Integrating w.r.t. ‘x’, treating y as constant, we get F(z) = ex – x + G(y) or

F and G being arbitrary functions, where

x = ex + G(y) – F(z),

z b1 f (z)gdz = F(z).

Miscellaneous problems based on types 1, 2, 3 and 4 Solve the following partial differential equations by using Monge’s method: 1. x2r – y2t = xy. Ans. z = xy log x + x F (y/x) + G(xy) 2 2 2 2 2. (1 + pq + q )r + s(q – p ) – (1 + pq + p )t = 0 Ans. z{2 + (x + y)}1/2 = F(x + y) + G(x – y) 3. q (1 + q)r – (1 + 2q) (1 + p)s + (1 + p)2t = 0 Ans. x = F(x + y + z) + G(x + z) 4. x2r – y2t – xp + yq = xy. Ans. z = (xy/4) × {(log x)2 – (log y)2} + xyF(x/y) + G(xy) 9.11. Monge’s Method of integrating the equation Rr + Ss + Tt + U(rt – s2) = V, where r, s, t have their usual meaning and R, S, T, U, V are functions of x, y, z. Given Rr + Ss + Tt + U(rt – s2) = V. ...(1) dp = ( p / x ) dx + ( p / y ) dy = rdx + sdy

We have

dq = ( q / x ) dx +

and

:

q / y ; dy = sdx + tdy

which give r = (dp – sdy)/dx and t = (dq – sdx)/dy. Putting these values in (1) and simplifying, we get (Rdpdy + Tdqdx – Udpdq – Vdxdy) – s{R(dy)2 – Sdxdy + T(dx)2 + Udpdx + Udqdy} = 0. Hence the usual Monge’s subsidiary equations are L < Rdpdy + Tdqdx + Udpdq – Vdxdy = 0 ...(2) and M < R(dy)2 – Sdxdy + T(dx)2 + Udpdx + Udqdy = 0. ...(3) We cannot factorise M as we did before (see Art 9.1), on account of the presence of the additional terms, Udpdx + Udqdy. Hence let us factorise M + =L, where = is some multiplier to be determined later. Now, we have M + =L < R(dy)2 + T(dx)2 – (S + =V)dxdy + Udpdx + Udqdy + =Rdpdy + =Tdqdx + =Udpdq = 0. ...(4) Factorising L.H.S. of (4), let k and m be constants such that

FH

IK

1 = M + =L < (Rdy + mTdx + kUdp) dy # dx # dq = 0. ...(5) m k Comparing coefficients in (4) and (5), we get R/m + mT = – (S + =V), ...(6) k=m and R=/k = U. ...(7) Now, the two relations of (7) give m = R=u Putting this valus of m in (6) and simplifying, we get =2(UV + RT) + =US + U2 = 0, ...(8) which is quadratic in =. Let =1 and =2 be its roots.

When = ) =1 , (7) Hence (5) gives or



R=1 / k ) U



k ) R=1 / U



FG Rdy # R= Tdx # R= dpIJ FGdy # U dx # U dqIJ = 0 H U K H R= R K 1

1

m ) R=1 / U

1

(Udy + =1Tdx + =1Udp) (Udx + =1Rdy + =1Udq) = 0. Similarly for = = =2, (5) gives (Udy + =2Tdx + =2 Udp) (Udx + =2Rdy + =2Udq) = 0.

...(9) ...(10)

Created with Print2PDF. To remove this line, buy a license at: http://www.software602.com/

9.34

Monge’s Methods

Now one factor of (9) is combined with one factor of (10) to give an intermediate integral. Exactly similarly, the other pair will give rise to another intermediate integral. In this connection remember that we must combine first factor of (9) with the second factor of (10) and similarly the second factor of (9) with the first factor of (10). Thus for the desired solution the proper method is to combine the factors in the following manner : Udy + =1Tdx + =1Udp = 0, Udx + =2Rdy + =2Udq = 0 ...(11) Udy + =2Tdx + =2Udp = 0, Udx + =1Rdy + =1Udq = 0 ...(12) Let equations (11) give two integrals u1 = c and v 1 = d1 so that one intermediate integral is u1 = f1( v 1), f1 being an arbitrary function ...(13) Similarly, (12) gives second intermediate integral u2 = f2( v 2), ...(14) where f2 is an arbitrary function We now solve (13) and (14) for p and q and substitute in dz = pdx + qdy, which after integration gives the desired general solution. Remark 1. There are in all four ways of combining factors of (9) and (10). By combining the first factors in these equations, we would get u dy = 0 on substraction (after dividing equations by =1 and =2 respectively) and this would not produce any solution. Similarly, combining the second factors in these equations would give u dx = 0 and hence would produce no solution. Hence for getting integrals of the given equation we must proceed as explained in (11) and (12). Remark 2. In what follows we shall use the following two results of equation a=2 + b= + c = 0 (i) a = b = 0, i.e., the coefficients of =2 and = both equal to zero imply that both roots of the equatin are equal to > (ii) a = 0 but b 7 0, i.e., the coefficient of =2 is zero but that of = is non–zero imply that one root of the equation is > and the other is –c/b. Remark 3. When the two values of = are equal, we shall have only one intermediate integral u1 = f(v1) and proceed as explained in solved examples of type 1 based on Rr + Ss + Tt + U(rt – s2) = V given below. An integral of a more general form can be obtained by taking the arbitrary function occuring in the intermediate integral to be linear. Let u1 = mv1 + n, where m and n are some constants. Then integrating it by Lagrange’s method we find the solution of the given equation. 9.12. Type 1: When the roots of = –quadratic (8) of Art 9.11 are identical. Solved examples of type 1 based on Rr + Ss + Tt + U(rt – s2) = V Ex. 1. Solve 5r + 6s + 3t + 2(rt – s2) + 3 = 0. [I.A.S. 1973 ; Meerut 1998] 2 Sol. Given equation 5r + 6s + 3t + 2(rt – s ) = –3. ...(1) Comparing the given equation with Rr + Ss + Tt + U(rt – s2) = V, we have R = 5, S = 6, T = 3, U = 2 and V = –3. Hence the =–quadratic =2(UV + RT) + =SU + U2 = 0 2 2 becomes 9= + 12= + 4 = 0 or (3= + 2) = 0 so that =1 = =2 = –2/3. There is only one intermediate integral given by the equations Udy + =1Tdx + =1Udp = 0 and Udx + =2Rdy + =2Udq = 0 or 2dy + (–2/3) × 3dx + (– 2/3) × 2dp = 0 and 2dx + (–2/3) × 5dy + (–2/3) × 2dq = 0 or 3dy – 3dx – 2dp = 0 and 3dx – 5dy – 2dq = 0. Integrating, 3y – 3x – 2p = c1 and 3x – 5y – 2q = c2. ...(2) Hence here the only intermediate integral is 3y – 3x – 2p = f(3x – 5y – 2q), where f is an arbitrary function. ...(3) Solving the two equations of (2) for p and q, we have p = (1/2) × (3y – 3x – c1) and q = (1/2) × (3x – 5y – c2).

Created with Print2PDF. To remove this line, buy a license at: http://www.software602.com/

Monge’s Methods

9.35

Putting these values of p and q in dz = pdx + qdy, we have dz = (1/2) × (3y – 3x – c1)dx + (1/2) × (3x – 5y – c2)dy or 2dz = 3(ydx + xdy) – 3xdx – 5ydy – c1dx – c2dy. Integrating, 2z = 3xy – (3x2/2) – (5y2/2) – c1x – c2y + c3, which is the required complete integral, c1, c2 and c3 being arbitrary constants. Alternative solution. An integral of a more general form can be obtained by supposing the arbitrary function f occuring in the intermediate integral (3) to be linear, giving 3y – 3x – 2p = m(3x – 5y – 2q) + n, where m and n are arbitrary constants. ...(4) Re–writing (4), 2p – 2mq = 3y – 3x + 5my – 3mx – n. ...(5) Lagrange’s auxiliary equations for (5) are

dy dx dz = = . 2 ∃2m 3y ∃ 3x # 5my ∃ 3mx ∃ n

Taking the first two fractions of (6), we have dy + mdx = 0 so that

y + mx = a.

3xdx # 5ydy # 2dz 6 x ∃ 10 my # 6 y ∃ 6 x # 10my ∃ 6mx ∃ 2n Hence taking first fraction of (6) and fraction (8), we have

Now,

each fraction of (6) =

3xdx # 5ydy # 2dz dx = 6y ∃ 6mx ∃ 2n 2

...(6) ...(7) ...(8)

3xdx # 5ydy # 2dz 3y ∃ 3mx ∃ n or 3xdx + 5ydy + 2dz = (3y – 3mx – n)dx or 2dz + 3xdx + 5ydy = {3(a – mx) – 3mx – n}dx, using (7) or 2dz + 3xdx + 5ydy = (3a – 6mx – n)dx . Integrating, 2z + (3x2/2) + (5y2/2) = 3ax – 3mx2 – nx + b/2 or 4z + 3x2 + 5y2 = 6x(y + mx) – 6mx2 – 2xn + b, using (7) or 4z – 6xy + 3x2 + 5y2 + 2nx = b. ...(9) From (7) and (9), the required general solution is 4z – 6xy + 3x2 + 2nx = !(y + mx), where ! is an arbitrary function and m and n are arbitrary constants. Ex. 2. Solve 3r + 4s + t + (rt – s2) = 1. Sol. Comparing the given equation with Rr + Ss + Tt + U(rt – s2) = V, we get R = 3, S = 4, T = 1, U = 1, V = 1. Then, =–quadratic =2(UV + RT) + =SU + U2 = 0 2 becomes 4= + 4= + 9 = 0 or (2= + 1)2 = 0 so that =1 = =2 = –1/2. There is only one intermediate integral given by the equations Udy + =1Tdx + =1Udp = 0 and Udx + =2Rdy + =2Udq = 0 or dy + (–1/2) × dx + (–1/2) × dp = 0 and dx + (–1/2) × 3dy + (–1/2) × dq = 0 or –2dy + dx + dp = 0 and 3dy – 2dx + dq = 0. ... (1) Integrating, –2y + x + p = c1 and 3y – 2x + q = c2. ...(2) Hence the only intermediate integral is –2y + x + p = f(3y – 2x + q), where f is an arbitrary function. ...(3) Solving (2) for p and q, p = 2y – x + c1 and q = –3y + 2x + c2. Putting these values of p and q in dz = pdx + qdy, we get dz = (2y – x + c1)dx + (–3y + 2x + c2)dy or dz = 2(ydx + xdy) – xdx – 3ydy + c1dx + c2dy. Integrating, z = 2xy – (x2/2) – (3y2/2) + c1x + c2y + c3, which is the required complete integral, c1, c2, c3 being arbitrary constants.

or

dx =

Created with Print2PDF. To remove this line, buy a license at: http://www.software602.com/

9.36

Monge’s Methods

Alternative solution. In order to get the more general solution, we assume the arbitrary function ! in (3) to be linear. Thus, we take –2y + x + p = m(3y – 2x + q) + n, m, n being arbitrary constants or p – mq = 2y – x + 3my – 2mx + n. ...(4) dy dx dz = = . ...(5) 1 2y ∃ x #3my ∃2mx # n ∃m Taking the first two fractions of (5), dy + mdx = 0 so that y + mx = a. ...(6)

Lagrange’s auxiliary equations for (4) are

each fraction of (5) =

Now,

xdx # 3ydy # dz x ∃ 3my # 2 y ∃ x # 3my ∃ 2mx # n

...(7)

xdx # 3ydy # dz dx = 2y ∃ 2mx # n 1

Taking the first fraction of (5) and the fraction (7), we have

xdx + 3ydy + dz = (2y – 2mx + n)dx xdx + 3ydy + dz = 2(a – mx)dx – 2mxdx + ndx, using (6) Integrating, (x2/2) + (3y2/2) + z = 2ax – mx2 – mx2 + nx + b/2 or x2 + 3y2 + 2z – 2x(y + mx) + 2mx2 – nx = b, using (6) ...(8) 2 2 From (6) and (8), the required general solution is x + 3y + 2z – 2xy – nx = !(y + mx), where ! is an arbitrary function and m and n are arbitrary constants. Ex. 3. Solve (q2 – 1)zr – 2pqzs + (p2 – 1)zt + z2(rt – s2) = p2 + q2 – 1. Sol. Comparing the given equation with Rr + Ss + Tt + U(rt – s2) = V, we have R = z(q2 – 1), S = –2pqz, T = z(p2 – 1), U = z2 and V = p2 + q2 – 1. Hence the =–quadratic =2(UV + RT) + =US + U2 = 0 becomes p2q2=2 – 2pqz + z2 = 0 or (pq= – z)2 = 0 so that =1 = =2 = z/pq. There is only one intermediate integral given by equations Udy + =1Tdx + =1Udp = 0 and Udx + =2Rdy + =2Udq = 0 or or

2

2

2

2

z ( p ∃ 1) z (q ∃ 1) z3 z3 dx + dp = 0 and z2dx + dy + dq = 0 pq pq pq pq or pqdy + (p2 – 1)dx + zdp = 0 and pqdx + (q2 – 1)dy + zdq = 0 or p(qdy + pdx) – dx + zdp = 0 and q(pdx + qdy) – dy + zdq = 0 or pdz + zdp – dx = 0 and qdz + zdq – dy = 0, as dz = pdx + qdy or d(pz) – dx = 0 and d(qz) – dy = 0. Integrating, pz – x = c1 and qz – y = c2. ...(1) Hence the only intermediate integral is pz – x = f(qz – y), f being an arbitrary function. ...(2) Solving (1) for p and q, p = (c1 + x)/z and q = (c2 + y)/z. Putting these values of p and q in dz = pdx + qdy, we get dz = (1/z) × (c1 + x)dx + (1/z) × (c2 + y)dy or zdz = (c1 + x)dx + (c2 + y)dy. Integrating, (1/2) × z2 = (1/2) × (c1 + x)2 + (1/2) × (c2 + y)2 + (1/2) × c30. or z2 = x2 + y2 + 2c1x + zc2y + c3, where c3 = c12 + c22 + c30 which is the complete integral, c1, c2, c3 being arbitrary constants. Alternative solution. To find the more general solution, we take the arbitrary function f in (2) to be linear. So, let pz – x = m(qz – y) + n, m, n being arbitrary constants. or pz – mqz = x – my + n. ...(3)

or

z2dy +

dz dx = dy = . ∃ mz x ∃ my # n z Taking the first two fractions of (4), dy + mdx = 0 so that y + mx = a. Lagrange’s auxiliary equation for (3) are

...(4) ...(5)

Created with Print2PDF. To remove this line, buy a license at: http://www.software602.com/

Monge’s Methods

Now,

9.37

each fraction of (4) =

(∃ x / z )dx ∃ ( y / z )dy # dz . z / (∃ x / z ) ∃ mz / (∃ y / z ) # x ∃ my # n

...(6)

∃( x / z)dx ∃ ( y / z)dy # dz dx = n z or – xdx – ydy + zdz = ndx or – 2zdz + 2xdx + 2ydy + 2ndx = 0. Integrating, –z2 + x2 + y2 + 2nx = b, b being an arbitrary constant ...(7) From (5) and (7), the required general solution is –z2 + x2 + y2 + 2nx = !(y + mx), where ! is an arbitrary function and m, n are arbitrary constants. Ex. 4. Solve 2s + (rt – s2) = 1. [Garwhal 1995; Meerut 2000] Sol. Comparing the given equation with the equation Rr + Ss + Tt + U(rt – s2) = V, we get R = 0, S = 2, T = 0, U = 1, V = 1, so =–quardratic =2 (UV + RT) + =SU + U2 = 0 2 becomes = + 2= + 1 = 0 so that =1 = =2 = –1. Since we have equal values of l, there would be only one intermediate integral given by Udy + =1Tdx + =1Udp = 0 and Udx + =2Rdy + =2Udq = 0 or d y – dp = 0 and dx – dq = 0, using (1) which give y – p = c1, and x – q = c2. Solving these for p and q, p = y – c1 and q = x – c2. dz = pdx + qdy = (y – c )dx + (x – c )dy = (ydx + xdy) – c + 1 2 1dx – c2dy, or dz = d(xy) – c1dx – c2dy. Integrating, z = xy – c1x – c2y + c3, which is solution, c1, c2, c3 being arbitrary constants. Ex. 5. z(1 + q2)r – 2pqzs + z(1 + p2)t + z2(s2 – rt) + 1 + p2 + q2 = 0. Sol. Comparing the give equation with Rr + Ss + Tt + U(rt – s2) = V, we get R = z(1 + q2), S = –2pqz, T = z(1 + p2), U = z2 and V = –(1 + p2 + q2). ... (1) 2 2 Hence =–quadratic i.e. = (RT + UV) + =US + U = 0 gives =2(p2q2) – 2=zpq + z2 = 0 or (=pq – z)2 = 0. Thus here we obtain =1 = =2 = z/pq. Hence there would be only one intermediate integral which is given by Udy + =1Tdx + =1Udp = 0. ...(2)

Taking the first fraction of (4) and fraction (6),

Udx + =2Rdy + =2Udq = 0 ...(3) 2 Using (1), (2) becomes pq dy + (1 + p )dx + zdp = 0 ...(4) Using (1), (3) becomes pqdx + (1 + q2)dy + zdq = 0 ...(5) Now from (4), p(pdx + qdy) + dx + zdp = 0 or pdz + dx + zdp = 0, as dz = pdx + qdy or d(zp) + dx = 0 so that zp + x = c1. ...(6) Similarly (5) gives zq + y = c2, c2 being an arbitrary constant ...(7) Solving (6) and (7), we get p = (c1 – x)/z and q = (c2 – y)/z. + dz = pdx + qdy = {(c1 – x)/z}dx + {(c2 – y)/z}dy or zdz = c1dx + c2dy – (xdx + ydy). Integrating, (1/2) × z2 = c1x + c2y – (x2 + y2)/2 + c3/2 or z2 = 2c1x + 2c2y – x2 – y2 + c3, which is complete integral, c1, c2, c3 being arbitrary constants. Ex. 6. Solve 2r + tex – (rt – s2) = 2ex. Sol. Comparing the given equation with Rr + Ss + Tt + U(rt – s2) = V, we get R = 2, S = 0, T = e x, U = –1 and V = 2ex. ... (1) and

Created with Print2PDF. To remove this line, buy a license at: http://www.software602.com/

9.38

Monge’s Methods 2

2

2

x

x

Hence the =–quadratic = (UV + RT) + =SU + U = 0 gives = (2e – 2e ) + := × 0) + 1 = 0. Since the coefficient of =2 and = in the above quadratic vanish, it follows from the theory of equations that its both the roots must be infinite. Thus =1 = =2 = >. Since the two roots are equal there would be only one intermediate integral which is given by Udy + =1Tdx + =1Udp = 0 and Udx + =2Rdy + =2Udq = 0, i.e., by (U/=1)dy + Tdx + Udq = 0 and (U/=2)dx + Rdy + Udq = 0, i.e., by exdx – dp = 0 using (1) and 2dy – dq = 0, using (1) x Integrating these e – p = c1 and 2y – q = c2. x Solving these, p = e – c1 and q = 2y – c2. x Now, dz = pdx + qdy = (e – c1)dx + (2y – c2)dy. Integrating, z = ex – c1x + y2 – c2y + c3, which is complete integral, c1, c2, c3 being arbitrary constants. Ex. 7. Solve r + t – (rt – s2) = 1. Sol. Comparing the given equation with Rr + Ss + Tt + U(rt – s2) = V, R = 1, S = 0, T = 1, U = –1, V = 1. ...(1) 2 2 2 So =–quadratic = (UV + RT) + =US + U = 0 becomes (0 × = ) + (0 × =; + 1 = 0. Since the coeffieicnts of both =2 and = are zero, so both roots of this quadratic are equal to >. So λ1 = λ 2 = >

or or

Now, the only one intermediate integral is given by equations Udy + =9Tdx + =9Udp = 0 and =9Rdy + Udx + =9Udq = 0 On dividing each term by = 9as =9 is infinite, the above equations become (1/=9) × Udy + Tdx + Udp = 0 and Rdy + (1/=9) × Udx + Udq = 0 Tdx + Udp = 0,

as

=1 ) >

and

Rdy + Udq = 0, as =9 = >

dx – dp = 0 and dy – dq = 0, using (1) Integrating, p – x = c1 and q – y = c2. ...(2) Solving (2) for p and q, p = x + c1 and q = y + c2. Putting these values of p and q in dz = pdx + qdy, we get dz = (x + c1)dx + (y + c2)dy Integrating, z = x2/2 + c1x + y2/2 + c2y + c3, which is the required integral, c1, c2, c3 being arbitrary constants. Ex. 8. Solve 2pr + 2qt – 4pq (rt – s2) = 1. Sol. Comparing the given equation with Rr + Ss + Tt + U(rt – s2) = V, we have R = 2p, S = 0, T = 2q, U = –4pq, V = 1. ...(1) 2 2 2 2 2 Then the =–quadratic = (UV + RT) + =SU + U = 0 becomes (0 × = )+ (0 × =; + 4p q = 0. Since the coefficients of both =2 and = are zero, so both roots of the =∃ quadratic are equal to >. or

So λ1 = λ 2 = >.

or or

Now the only intermediate integral is given by the equation Udy + =9Tdx + =9Udp = 0 and =9Rdy + Udx + =9Udq = 0 On dividing each term by =9 as =9 is infinite, the above equations become (9/=9) × Udy + Tdx + Udp = 0 and Rdy + (9/=9)×Udx + Udq = 0 2qdx – 4pqdp = 0 and 2pdy – 4pqdq = 0, using (1) 2pdp – dx = 0 and 2qdq – dy = 0.

Created with Print2PDF. To remove this line, buy a license at: http://www.software602.com/

Monge’s Methods

9.39 2

2

Integrating, p – x = c1 and q – y = c2. 1/2 Hence p = ± (c1 + x) and q = ± (c2 + y)1/2 Putting values of p and q in dz = pdx + qdy gives dz = ± (c1+ x)1/2dx ± (c2 + y)1/2dy. Integrating, z = ± (2/3) × (c1 + x)3/2 ± (2/3) × (c2 + y)3/2 + c3/2 or 3z = ± 2(c1 + x)3/2 ± 2(c2 + y)3/2 + c3, which is the complete integral, c1, c2, c3 being arbitrary constants. Ex. 9. Solve (1 + q2)r – 2pqs + (1 + p2)t + (1 + p2 + q2)–1/2 (rt – s2) = – (1 + p2 + q2)3/2. Sol. Comparing the given equation with Rr + Ss + Tt + U(rt – s2) = V, we get R = 1 + q2, S = –2pq, T = 1 + p2, U = (1 + p2 + q2)–1/2, V = – (1 + p2 + q2)3/2 ...(1) 2 Now, the =–quadratic = (UV + RT) + =SU + U2 = 0 becomes =2 {– (1 + p2 + q2) + (1 + q2)(1 + p2)}– 2pq (1 + p2 + q2)–1/2= + (1 + p2 + q2)–1 = 0 or p2q2(1 + p2 + q2)=2 – 2pq(1 + p2 + q2)1/2 = + 1 = 0 or {pq(1 + p2 + q2)1/2= – 1}2 = 0 so that =1 = =2 = 1/pq(1 + p2 + q2)1/2. Here there is only intermediate integral given by equations Udy + =1Tdx + =1Udp = 0 and Udx + =2Rdy + =2Udq = 0 or

1 # p2 dp 1 dy + dx + 2 2 = 0, by (1) pq (1 # p 2 # q 2 )1/ 2 (1 # p2 # q 2 )1/ 2 pq(1 # p # q )

and

1 # q2 dq 1 dx + = 0, by (1) 2 2 1/ 2 2 2 1/ 2 dy + (1 # p # q ) pq (1 # p # q ) pq(1 # p2 # q2 )

pqdy + (1 + p2)dx + [1/(1 + p2 + q2)1/2]dp = 0 pqdx + (1 + q2)dy + {1/(1 + p2 + q2)1/2}dq = 0.

or and

Eliminating dy between (2) and (3), {(1 + p2)(1 + q2) – p2q2}dx +

(1 + p2 + q2)dx +

or

or

dx +

(1 # q 2 )dp ∃ pqdq =0 (1 # p2 # q 2 )1/ 2

(1 # p 2 # q 2 )dp ∃ ( p 2 dp # pqdq ) (1 # p 2 # q 2 )1/ 2

dp p 2 pdp # 2qdq =0 2 1/ 2 – 2 2 3/ 2 2 (1 # p # q ) (1 # p # q )

=0

p &? &% dx # d ≅ )0 2 2 1/ 2 ∋ &Α (1 # p # q ) &(

or

2

...(2) ...(3)

Integrating, x + p(1 + p2 + q2)–1/2 = a, where a is an arbitrary constant. ...(4) Similarly, eliminating dx between (2) and (3), we have y + q(1 + p2 + q2)–1/2 = b, where b in an arbitrary constant. ...(5) 2 2 –1/2 2 2 –1/2 From (4) and (5), x – a = –p(1 + p + q ) , y – b = –q(1 + p + q ) . p x∃a = q y∃b Putting the above value of p in (4), we have

+

x+q or

RS T

2

x∃a 2 ( x ∃ a) 2 1# q #q y∃b ( y ∃ b )2

1+

UV W

∃1 / 2

= a or (x – a) +

( x ∃ a)2 # ( y ∃ b)2 2 q2 q = (y ∃ b)2 ( y ∃ b)2

or

p=

so that

LM N

x∃a q. ...(6) y∃b

x∃a ( x ∃ a)2 # ( y ∃ b)2 2 q 1# q y∃b (y ∃ b)2

OP Q

∃1/ 2

=0

(y – b)2 = q2[1 – {(x – a)2 + (y – b)2}].

Created with Print2PDF. To remove this line, buy a license at: http://www.software602.com/

9.40

Monge’s Methods

q = (y – b)/ [1 ∃{( x ∃ a)2 # ( y ∃ b)2 }]1/ 2 .

Thus, Now, (6) and (7) −

+ dz = pdx + qdy =

p=

x∃a x∃a q = . y ∃b [1 ∃ {( x ∃ a)2 # ( y ∃ b)2}]1/ 2

... (7) ... (8)

( x ∃ a) dx # ( y ∃ b) dy , by (7) and (8) [1 ∃ {( x ∃ a)2 # ( y ∃ b)2}]1/ 2

Integrating, z = [1 – {(x – a)2 + (y – b)2}]1/2 + c or (z – c)2 = 1 – {(x – a)2 + (y – b)2} 2 2 2 (x – a) + (y – b) + (z – c) = 1 is the complete integral, a, b, c being arbitrary constants. + 9.13 Type 2. When the roots of = –quadratic (8) of Art 9.11 are distinct. Solved Examples of Type –2 based on Rr + Ss + Tt + U(rt – s2) = V Ex. 1. Solve 3s + rt – s2 = 2. Sol. Given 3s + (rt – s2) = 2. ...(1) 2 Comparing (1) with Rr + Ss + Tt + U(rt – s ) = V, R = 0, S = 3, V = 0, U = 1, V = 2. ...(2) =–quadratic is =2(UV + RT) + =US + U2 = 0 ...(3) 2 Using (2), (3) reduces to 2= + 3= + 1 = 0 so =1 = –1, =2 = –(1/2). ... (4) Two integrals of (1) are given by the following sets Udy + =1Tdx + =1Udp = 0 % ... (5) Udx + =2Rdy + =2Udq = 0.∋( and Udy + =2Tdx + =2Udp = 0 % ... (6) ∋ Udx + =1Rdy + =1Udq = 0.( Using (2) and (4), (5) and (6) respectively gives dy – dp = 0 or dp – dy = 0 % ... (5A) ∋ dx – (1/2)dq = 0 or dq – 2dx = 0 ( and dy – (1/2)dp = 0 or dp – 2dy = 0 % ... (6A) dx – dq = 0 or dq – dx = 0. ∋( Integration of (5A) and (6A) respectively gives p – y = c1, q – 2x = c2 ...(5B) and p – 2y = c3, q – x = c4, ...(6B) where c1, c2, c3 and c4 are arbitrary constants. From (5B) and (6B), two intermediate integrals of (1) are given by p – y = f(q – 2x) and p – 2y = F(q – x), ...(7) where f and F are arbitrary functions. Let q – 2x = Β, ...(8) and q – x = Χ. ...(9) Then from (7) p – y = f(Β), ...(10) and p – 2y = F(Χ). ...(11) [If we treat Β and Χ as constants, then solution of four simultaneous equation (8), (9), (10) and (11) would show that x, y, p and q are all constants which is absurd. Hence Β and Χ will be regarded as variables (parameters) and we will get the general solution in parametric form involving Β and Χ as parameters]. Solving (8) and (9) for x and (10) and (11) for y, we have x=Χ–Β ...(12) and y = f(Β) – F(Χ). ...(13) From (10) p = y + f(Β). ...(14)

Created with Print2PDF. To remove this line, buy a license at: http://www.software602.com/

Monge’s Methods

or or

9.41

From (9) q = x + Χ. ...(15) From (12) and (13), dx = dΧ – dΒ, and dy = f 0(Β)dΒ – F 0 (Χ)dΧ. ...(16) + dz = pdx + qdy = [y + f(Β)]dx + (x + Χ)dy, using (14) and (15) dz = ydx + xdy + f(Β)dx + Χdy = d(xy) + f(Β)(dΧ – dΒ) + Χ[f 0(Β)dΒ – F 0(Χ)dΧ], by (16) Thus, dz = d(xy) + [f(Β)dΧ + Χf 0 (Β)dΒ] – f(Β)dΒ – ΧF0 (Χ)dΧ dz = d(xy) + d[Χf(Β)] – f(Β)dΒ – ΧF 0(Χ)dΧ. Integrating and using integration by parts in the last term on R.H.S. of the above equation,

8 f (Β )dΒ ∃ [ΧF (Χ) ∃ 8 1∆ F (Χ)dΧ]

we get

z = xy + Χf(Β) –

or

z = xy + Χ[f(Β) – F(Χ)] –

Let

z f (Β)dΒ = !(Β)

z f (Β)dΒ # z F(Χ)dΧ . and z F(Χ)dΧ = ∗(Χ)

...(17) ...(18)

so that f(Β) = !0(Β) and F(Χ) = ∗0(Χ) ...(19) Using (18) and (19), (12), (13) and (17) give x = Χ – Β, y = !0(Β) – ∗0(Χ) z =xy + Χ[!0(Β) – ∗0(Χ)] – !(Β) + ∗(Χ) which is the required solution in parametric form, ! and ∗ being arbitrary functions and Β and Χ being parameters. Ex. 2. Solve r + 4s + t + rt – s2 = 2. [I.A.S. 1979] 2 Sol. Given r + 4s + t + (rt – s ) = 2. ...(1) Comparing (1) with Rr + Ss + Tt + U(rt – s2) = V, R = 1, S = 4, T = 1, U = 1, V = 2. ...(2) =–quadratic is =2(UV + RT) + =US + U2 = 0. ...(3) 2 Using (2), (3) reduces to 3= + 4= + 1 = 0 so =1 = –1, =2 = –(1/3). Two integrals of (1) are given by the following sets Udy + =1Tdx + =2Udp = 0% ... (5) ∋ Udx + =2Rdy + =2Udq = 0( Udy + =2Tdx + =2Udp = 0% ∋ ... (6) Udx + =1Rdy + =1Udq = 0( Using (2) and (4), (5) and (6) respectively gives dy – dx – dp = 0 or dp + dx – dy = 0 % ... (5A) ∋ dx – (1/3) × dy – (1/3) × dq = 0 or dq + dy – 3dx = 0 ( dy – (1/3) × dx – (1/3) × dp = 0 or dp + dx – 3dy = 0 % ∋ ... (6A) dx – dy – dq = 0 or dq + dy – dx = 0 ( Integration of (5A) and (6A) respectively gives p + x – y = c1, q + y – 3x = c2 ...(5B) and p + x – 3y = c3, q + y – x = c4, ...(6B) where c1, c2, c3 and c4 are arbitrary constants. From (5B) and (6B), two intermediate integrals of (1) are given by p + x – y = f(q + y – 3x) and p + x – 3y = F(q + y – x). ...(7) Let q + y – 3x = Β, ...(8) and q + y – x = Χ. ...(9) Then from (7), p + x – y = f(Β), ...(10) and p + x – 3y = F(Χ). ...(11) Here Β and Χ are treated as parameters. Solving (8) and (9) for x and (10) and (11) for y gives

Created with Print2PDF. To remove this line, buy a license at: http://www.software602.com/

9.42

Monge’s Methods

x = (Χ – Β)/2 ...(12) and y = [f(Β) – F(Χ)]/2 ...(13) From (10), p = y – x + f(Β) ...(14) From (9), q=x–y+Χ ...(15) From (12) and (13), dx = (1/2) × (dΧ – dΒ), dy =(1/2) × [f 0(Β)dΒ – F 0(Χ)dΧ]. ...(16) + dz = pdx + qdy = [y – x + f(Β)]dx + (x – y + Χ)dy, by (14) and (15) = ydx + xdy – xdx – ydy + f(Β)dx + Χdy = d(xy) – xdx – ydy + f(Β) × (1/2)×(dΧ – dΒ) + Χ × (1/2)×[f 0(Β)dΒ – F 0(Χ)dΧ], by (16) = d(xy) – xdx – ydy + (1/2) × [f(Β)dΧ + Χf 0(Β)dΒ] – (1/2) × f(Β)dΒ – (1/2) × ΧF 0(Χ)dΧ or 2dz = 2d(xy) – 2xdx – 2ydy + d[Χf(Β)] – f(Β)dΒ – ΧF 0(Χ)dΧ. Integrating and using integration by parts in the last term on R.H.S. of the above equation,

8 f (Β )d Β ∃[Χ F (Χ) ∃ 8 1∆ F (Χ) dΧ]

we get

2z = 2xy – x2 – y2 + Χf(Β) –

or

2z = 2xy – x2 – y2 + Χ[f(Β) – F(Χ)] – Let

z f (Β)dΒ # z F(Χ) dΧ. and z F(Χ) dΧ = ∗(Χ)

z f (Β)dΒ = !(Β)

...(17) ...(18)

so that f(Β) = !0(Β) and F(Χ) = ∗0(Χ). ...(19) Using (18) and (19), (12), (13) and (17) give 2x = Χ – Β, 2y = !0(Β) – ∗0(Χ), 2z = 2xy – x2 – y2 + Χ[!0(a) – ∗0(Χ)] – !(Β) + ∗(Χ) which is the required solution in parametric form, Β and Χ being parameters and ! and ∗ being arbitrary functions. Ex. 3. Solve rt – s2 + 1 = 0 Sol. Given that 0.r + 0.s + 0.t + (rt – s2) = –1. ...(1) 2 Comparing (1) with Rr + Ss + Tt + U(rt – s ) = V, R = 0, S = 0, T = 0, U = 1 and V = –1. ...(2) Here =–quadratic =2(UV + RT) + =US + U2 = 0 ...(3) becomes =2 – 1 = 0 so that =1 = –1 and =2 = 1. ...(4) Since the two values of = are distinct, we shall get two intermediate integrals which are given by the following sets of equations Udy + =1Tdx + =1Udp = 0 % ... (5A) Udx + =2Rdy + =2Udq = 0 ∋( Udy + =2Tdx + =2Udp = 0 % ... (5B) Udx + =1Rdy + =1Udq = 0 ∋( Using (2) and (4), equations (5) and (6) reduces to dy – dp = 0 i.e., dp – dy = 0 % ∋ dx + dq = 0 i.e., dq + dx = 0 ( ... (5A) dy + dp = 0 i.e., dp + dy = 0 % dx – dq = 0 i.e., dq – dx = 0 ∋( ... (6A) Integrating of (5A) and (6A) respectively gives p – y = c1, q + x = c2. ...(5B) and p + y = c3, q – x = c4, ...(6B) where c1, c2, c3 are c4 are arbitrary constants From (5B) and (6B), two intermediate integrals are given by p – y = f(q + x)

and

p + y = F(q – x),

...(7)

Created with Print2PDF. To remove this line, buy a license at: http://www.software602.com/

Monge’s Methods

9.43

where f and F are arbitrary functions. Let q+x=Β ...(8) and q – x = Χ. ...(9) Then, from (7), p – y = f(Β) ...(10) and p + y = F(Χ). ...(11) In what follows Β and Χ will be regarded as parameters. Solving (8) and (9) for x and (10) and (11) for y, we have x = (Β – Χ)/2 ...(12) and y = [F(Χ) – f(Β)]/2 ...(13) From (10), p = y + f(Β) ...(14) From (9), q = x + Χ. ...(15) From (12) and (13), dx = (1/2) × (dΒ – dΧ), dy = (1/2) × [F 0(Χ)dΧ – f 0(Β)dΒ]. ...(16) + dz = pdx + qdy = [y + f(Β)]dx + (x + Χ)dy, using (14) and (15) = (ydx + xdy) + f(Β)dx + Χdy = d(xy) + f(Β) × (1/2) × (dΒ – dΧ) + Χ × (1/2) × [F 0(Χ)dΧ – f 0(Β)dΒ], by (16) = d(xy) + (1/2) × f(Β)dΒ – (1/2) × [f(Β)dΧ + Χf 0(Β)dΒ] + (1/2) × ΧF 0(Χ)dΧ or 2dz = 2d(xy) + f(Β)dΒ – d[Χf(Β)] + ΧF 0(Χ)dΧ. Integrating both sides and using integration by parts in the last term on the R.H.S., we obtain 2z = 2xy + Let

z f (Β) dΒ = !(Β)

z f (Β)dΒ # Χf (Β) # ΧF(Χ) ∃ z F(Χ) dΧ . and z F(Χ) dΧ = ∗(Χ)

so that f(Β) = !0(Β) and F(Χ) = ∗0(Χ). Using (18) and (19), (12), (13) and (17) may be re–written as 2x = (Β – Χ),

2y = ∗0(Χ) – !0(Β),

...(17) ...(18) ...(19)

2z = 2xy – !(Β) + Χ{!0(Β) + ∗0(Χ)} – ∗(Χ)

which is the required solution in parametric form, Β and Χ being parameters and ! and ∗ being arbitrary functions. Ex. 4. Solve r + 3s + t + (rt – s2) = 1.

[Rohilkhand 1995] 2

Sol. Given r + 3s + t + (rt + s ) = 1 ... (1) 2 Comparing (1) with Rr + Ss + Tt + U(rt – s ) = V, R = 1, S = 3, T = 1, U = 1, V = 1. ...(2) Now, or

=–quadratic is

2

2= + 3= + 1 = 0

=2(UV + RT) + =US + U2 = 0 so that = = –1,

–1/2.

Here =1 = –1,

Two intermediate integrals of (1) are giving by the following sets Udy + =1Tdx + =1Udp = 0 % ∋

Udx + =2Rdy + =2Udq = 0( Udy + =2Tdx + =2Udp = 0 % ∋ Udx + =1Rdy + =1Udq = 0( Using (2) and (4), equations (5) and (6) reduces to

and

dy – dx – dp = 0

i.e.,

dx – (1/2) × dy – (1/2) × dq = 0 dy – (1/2) × dx – (1/2) × dp = 0 dx – dy – dq = 0

i.e., i.e., i.e.,

...(3) =2 = –1/2. ...(4)

... (5) ... (6)

dp + dx – dy = 0 % ∋ ... 5(A) dq – 2dx + dy = 0 ( dp + dx – 2dy = 0 % ...(6A) ∋ dq – dx + dy =0 (

Created with Print2PDF. To remove this line, buy a license at: http://www.software602.com/

9.44

Monge’s Methods

Integrating of (5A) and (6A) respectively gives p + x – y = c1, q – 2x + y = c2 ...(5B) and p + x – 2y = c3, q – x + y = c4, ...(6B) where c1, c2, c3 and c4 are arbitrary constants From (5B) and (6B), two intermediate integrals are given by p + x – y = f(q – 2x + y) and p + x – 2y = F(q – x + y), ...(7) where f and F are arbitrary functions Let q – 2x + y = Β ...(8) and q – x + y = Χ. ...(9) Then, from (7) p + x – y = f(Β) ...(10) and p + x – 2y = F(Χ). ...(11) In what follows, Β and Χ will be regarded as parameters. Solving (8) and (9) for x and (10) and (11) for y, we have x=Χ–Β ...(12) and y = f(Β) – F(Χ). ...(13) From (10), p = y – x + f(Β) ...(14) From (9), q = x – y + Χ. ...(15) From (12) and (13), dx = dΧ – dΒ, dy = f 0(Β)dΒ – F 0(Χ)dΧ. ...(16) dz = pdx + qdy = [y – x + f(Β)]dx + [x – y + Χ]dy, using (14) and (15) + = –(x – y) (dx – dy) + f(Β)dx + Χdy = –(x – y) d(x – y) + f(Β) (dΧ – dΒ) + Χ[f 0(Β)dx – F 0(Χ)dΧ], by (16) = –(x – y) d(x – y) – f(Β) dΒ + {f(Β)dΧ + Χf 0(Β)dΒ} – ΧF 0(Χ)dΧ or dz = –(x – y) d(x – y) – f(Β) dΒ + d[Χf(Β)] – ΧF 0(Χ)dΧ. Integrating both sides and using integration by parts in the last term on the R.H.S., we obtain z = –(1/2) × (x – y)2 – Let

z

f (Β ) dΒ = !(Β)

z

LM N

z

OP Q

f ( Β ) d Β # Χ f (Β ) ∃ Χ F (Χ ) ∃ F (Χ ) d Χ .

and

...(17)

z F(Χ) dΧ = ∗(Χ) ...(18)

so that f(Β) = !0(Β) and F(Χ) = ∗0(Χ). ...(19) Using (18) and (19), (12), (13) and (17) may be written as x = Χ – Β, y = !0(Β) – ∗0(Χ), z = – (1/2)× (x – y)2 – !(Β) + ∗(Χ) + Χ[!0(Β) – ∗0(Χ)] which is the required solution in parametric form, Β and Χ being parameters, and ! and ∗ being arbitrary functions. Ex. 5. Solve rt – s2 + a2 = 0. [Rohilkhand 1993] 2 2 . . . Sol. Given that 0 r + 0 s + 0 t + (rt – s ) = –a . ...(1) Comparing (1) with Rr + Ss + Tt + U(rt – s2) = V, R = 0, S = 0, T = 0, U = 1, V = –a2. ...(2) Then, the =–quadratic =2(UV + RT) + =SU + U2 = 0 ...(3) 2 2 becomes –= a + 1 = 0 or = = ± 1/a. So =1 = 1/a, =2 = –1/a. ...(4) Two intermediate integrals of (1) are given by the following two sets Udy + =1Tdx + =1Udp = 0 % ... (5) ∋ Udx + =2Rdy + =2Udq = 0( and Udy + =2Tdx + =2Udp = 0 % ... (6) ∋ Udx + =1Rdy + =1Udq = 0(

Created with Print2PDF. To remove this line, buy a license at: http://www.software602.com/

Monge’s Methods

9.45

Using (2) and (4), equations (5) and (6) reduce to dy + (1/a) × dp = 0 i.e., dp + ady = 0 % ... (5A) dx – (1/a) × dq = 0 i.e., dq – adx = 0 ∋( and dy – (1/a) × dp = 0 i.e., dp – ady = 0 % ... (6A) dx + (1/a) × dq = 0 i.e., dq + adx = 0.∋( Integration of (5A) and (6A) respectively gives p + ay = c1, q – ax = c2 ...(5B) and p – ay = c3, q + ax = c4. ...(6B) where c1, c2, c3 and c4 are arbitrary constants From (5B) and (6B), two intermediate integrals are given by p + ay = f(q – ax) and p – ay = F(q + ax). ...(7) where f and F are arbitrary functions Let q – ax = Β ...(8) and q + ax = Χ. ...(9) Then, from (7) p + ay = f(Β) ...(10) and p – ay = F(Χ). ...(11) In what follows, Β and Χ will be regarded as parameters. Solving (8) and (9) for x and (10) and (11) for y, we have x = (1/2a) × (Χ – Β) ...(12) and y = (1/2a) × [f(Β) – F(Χ)]. ...(13) From (10), p = f(Β) – ay. ...(14) From (9), q = Χ – ax. ...(15) From (12) and (13), dx = (1/2a) × (dΧ – dΒ), dy = (1/2a) × [f 0(Β)dΒ – F 0(Χ)dΧ] ...(16) + dz = pdx + qdy = [f(Β) – ay]dx + (Χ – ay)dy, using (14) and (15) = f(Β)dx + Χdy – a(ydx + xdy) = f(Β) × (1/2a) × (dΧ – dΒ) + Χ × (1/2a) × [f 0(Β)dΒ – F 0(Χ)dΧ] – ad(xy), by (16) or 2adz = {f(Β)dΧ + Χf 0(Β)dΒ} – f(Β)dΒ – 2a2d(xy) – ΧF 0(Χ)dΧ. Integrating both sides and using the formula for integration by parts in the last term on R.H.S., we have 2az = Χf(Β) – Let

z f (Β)dΒ ∃ 2a xy ∃ ΧF(Χ) ∃ z F(Χ) dΧ .

z f (Β)dΒ = !(Β)

2

and

...(17)

z F(Χ) dΧ = ∗(Χ) ...(18)

so that f(Β) = !0(Β) and F(Χ) = ∗0(Χ). ...(19) Using (18) and (19), (12), (13) and (17) reduces to 2ax = Χ – Β, 2ay = !0(Β) – ∗0(Χ), 2az = Χ[!0(Β) – ∗0(Χ)] – !(Β) – 2a2xy + ∗(Χ). which is the required solution in parametric form, Β, Χ, being parameters and !(Β) and ∗(Χ) being arbitrary functions. Ex. 6. Solve 7r – 8s – 3t + (rt – s2) = 36. Sol. Given that 7r – 8s – 3t + (rt – s2) = 36. ...(1) 2 Comparing (1) with Rr + Ss + Tt + U(rt – s ) = V, R = 7, S = –8, T = –3, U = 1, V = 36. ...(2) The =–quadratic =2(UV + RT) + =US + U2 = 0 ...(3) 2 becomes 15= – 18= + 1 = 0 or (5= – 1)(3= – 1) = 0. So =1 = 1/5, =2 = 1/3. ...(4) Two intermediate integrals of (1) are given by the following sets

Created with Print2PDF. To remove this line, buy a license at: http://www.software602.com/

9.46

Monge’s Methods

Udy + =1Tdx + =1Udp = 0% ... (5) ∋ Udx + =2Rdy + =2Udq = 0( Udy + =2Tdx + =2Udp = 0% ∋ ... (6) Udx + =1Rdy + =1Udq = 0( Using (2) and (4), equations (5) and (6) reduce to dy + (1/5) × (–3)dx + (1/5) × dp = 0 i.e., dp – 3dx + 5dy = 0 % ... (5A) ∋ dx + (1/3) × 7 dy + (1/3) × dq = 0 i.e., dq + 7dy + 3dx = 0( dy + (1/3) × (–3)dx + (1/3) × dp = 0 i.e., dp – 3dx + 3dy = 0 % ... (6A) ∋ dx + (1/5) × 7dy + (1/5) × dq = 0 i.e., dq + 7dy + 5dx = 0 ( Integrating of (5A) and (6A) respectively, gives p – 3x + 5y = c1, q + 7y + 3x = c2 ...(5B) and p – 3x + 3y = c3, q + 7y + 5x = c4, ...(6B) where c1, c2 c3 and c4 are arbitrary constants From (5B) and (6B), two intermediatre integrals are given by p – 3x + 5y = f(q + 7y + 3x) and p – 3x + 3y = F(q + 7y + 5x) ...(7) where f and F are arbitrary functions Let q + 7y + 3x = Β ...(8) and q + 7y + 5x = Χ. ...(9) Then, from (7) p – 3x + 5y = f(Β) ...(10) and p – 3x + 3y = F(Χ). ...(11) In what follows, Β and Χ will be regarded as parameters. Solving (8) and (9) for x and (10) and (11) for y, we have x = (Χ – Β)/2 ...(12) and y = [f(Β) – F(Χ)]/2 ...(13) From (10), p = f(Β) + 3x – 5y. ...(14) From (9), q = Χ – 7y – 5x. ...(15) From (12) and (13), dx = (1/2) × (dΧ – dΒ), dy = (1/2) × {f 0(Β)dΒ – F 0:Χ; dΧ}. ...(16) dz = pdx + qdy = {f(Β) + 3x – 5y}dx + {Χ – 7y – 5x)dy, using (14) and (15) + = 3xdx – 7ydy – 5(ydx + xdy) + f(Β)dx + Χdy = 3xdx – 7ydy – 5d(xy) + f(Β) × (1/2) × (dΧ – dΒ) + Χ × (1/2) × {f 0(Β)dΒ – F 0(Χ)dΧ} or 2dz = 6xdx – 14ydy – 10d(xy) + {f(Β)dΧ + Χf 0(Β)dΒ} – f(Β)dΒ – ΧF 0(Χ)dΧ or 2dz = 6xdx – 14ydy – 10d(xy) + d{Χf(Β)} – f(Β)dΒ – ΧF 0(Χ)dΧ. Integrating both sides and using the formula for integrating by parts in the last term on R.H.S., we have

z

z

2z = 3x2 – 7y2 – 10xy + Χf(Β) – f (Β) dΒ – [ΧF(Χ) – F(Χ) dΧ ] 2z = 3x2 – 7y2 – 10xy + Χ[f(Β) – F(Χ)] –

or Let

z f (Β) dΒ = !(Β)

and

z

z

f (Β ) dΒ # F (Χ) dΧ .

...(17)

z F(Χ) dΧ = ∗(Χ)

...(18)

so that f(Β) = !0(Β) and F(Χ) = ∗0(Χ) ...(19) Using (18) and (19), relation (12), (13) and (17) become x = (1/2) × (Χ – Β), y = (1/2) × [!0(Β) – ∗0(Χ)], 2z = 3x2 – 7y2 – 10xy + Χ[!0(Β) – ∗0(Χ)] – !(Β) + ∗(Χ). which is required solution in parametric form, Β and Χ being parameters and !(Β) and ∗(Χ) being

Created with Print2PDF. To remove this line, buy a license at: http://www.software602.com/

Monge’s Methods

9.47

arbitrary functions. 9.14 Miscellaneous examples on Rr + Ss + Tt + U(rt – s2) = V. In some problems only one intermediate integral is possible. Sometimes even after getting two intermediate integrals, it may not be possible to get p and q from those intermediate integrals. In such problems, final solution is obtained by integrating only one intermediate integral by the methods of solution of first order equation, for example, Charpit’s method. Again, we can avoid Charpit’s method by taking u1 = !1(v1) and u2 = constant = = (say) to obtain final solution. [Here we have assumed that u1 = !1(v1) and u2 = !2(v2) are two intermediate integrals]. Since an arbitrary constant can be regarded as a particular case of an arbitrary function, the values of p and q derived from u1 = !1(v1) and u2 = = will make dz = pdx + qdy integrable. The complete integral so obtained will involve one arbitrary funtion !1 and two arbitrary constants, namely, = and the constant of integration. To obtain the general integral, express one of the arbitrary constant as an arbitrary function of the other and eliminate this remaining constant between the equation so obtained and that deduced from it by differentiation with respect to that constant. Ex. 1. Obtain the intermediate integral of 2yr + (px + qy)s + xt – xy(rt – s2) = 2 – pq. [Rohilkhand 1992] Sol. Given 2yr + (px + qy)s + xt – xy(rt – s2) = 2 – pq. ...(1) Comparing (1) with Rr + Ss + Tt + U(rt – s2) = V, we have R = 2y, S = px + qy, T = x, U = –xy, V = 2 – pq. ...(2) Now, =–quadratic =2(UV + RT) + =US + U2 = 0 ...(3) reduces to =2[2yx – xy(2 – pq)] + =[–xy(px + qy)} + x2y2 = 0 or =2pq – =(px + qy) + xy = 0 or (=p – y)(=q – x) = 0 or = = y/p, x/p so that =1 = y/p and =2 = x/q. ...(4) Two intermediate integrals are given by the following sets Udy + =1Tdx + =1Udp = 0 % ... (5) ∋ Udx + =2Rdy + =2Udq = 0( Udy + =2Tdx + =2Udp = 0 % ... (6) ∋ Udx + =1Rdy + =1Udq = 0.( Using (2) and (4), equations (5) and (6) reduce to –xydy + (y/p)xdx + (y/p)(–xy)dp = 0 i.e., (pdy + ydp) – dx = 0% ... (5A) and –xydx + (x/q)(2y)dy + (x/q)(–xy)dq = 0 i.e., (qdx + xdq) – 2dy = 0.∋( –xydy + (x/q)xdx + (x/q)(–xy)dp = 0 i.e., –qydy + xdx – xydp = 0 ...(6A) and –xydx + (y/p)(2y)dy + (y/p)(–xy)dq = 0 i.e., –pxdx + 2ydy – xydq = 0 %∋ Integrating (5A), py – x = c1 and qx – 2y = c2 ( Hence one intermediate integral is py – x = !(qx – 2y). ...(7) Note, the equation (6A) cannot be integrated. Hence in this problem we can obtain only one intermediate integral, i.e., (7). Here ! is an arbitrary function. Ex. 2. Solve qr + (p + x)s + yt + y(rt – s2) + q = 0. [Rohilkhand 1992] Sol. Given qr + (p + x)s + yt + y(rt – s2) = –q. ...(1) Comparing (1) with Rr + Ss + Tt + U(rt – s2) = V, we have R = q, S = p + x, T = y, U=y and T = –q. ...(2) Now, the =–quadratic =2(UV + RT) + =SU + U2 = 0 ...(3) 2 2 2 reduces to (0 × = ) + =y(p + x) + y = 0. Since coefficient of = is zero, it follows that its one root is >. The other root is –y/(p + x). Let =1 = –y/(p + x) and =2 = >. ...(4) One intermediate integral is given by the following sets

Created with Print2PDF. To remove this line, buy a license at: http://www.software602.com/

9.48

Monge’s Methods

Udy + =1Tdx + =1Udp = 0 ,i.e., Udy + =1Tdx + =1Udp = 0 Udx + =2Rdy + =2Udq = 0 (1/=2)×Udx + Rdy + Udq = 0. Using (2) and (4), equations of (5) reduce to

% ∋ (

... (5)

2 2 dp # dx dy y y ∃ dx ∃ dp = 0 i.e. =0 % p# x y p# x p# x ... (5A) ∋ ( and qdy + ydq =% 0 i.e., d(yq) = 0 Integrating (5A), log (p∋(+ x) – log y = log c1 or (p + x)/y = c1 ...(6) and yq = c2, c2 being an arbitrary constant ...(7)

ydy –

FG p # x IJ . H y K dy dp dx = = . y F F p # x I 1 f ' p # xI 1 ∃ f 'G G J J y H y K y H y K qy = f

From (6) and (7), an intermediate integral is given by Charpit’s auxiliary equations for (8) are

...(8) ...(9)

Taking the first and third fractions of (9), we have dx + dp = 0 so that x + p = c, where c is an arbitrary constant. ...(10) p=c–x

Solving (8) and (10) for p and q,

q = (1/y) × f(c/y).

and

Putting these values of p and q in dz = pdx + qdy,

dz = (c – x)dx + (1/y) × f(c/y)dy. 2

z = cx – x /2 + F(c/y) + G(=),

Integrating,

which is the complete integral, F and G being arbitrary functions. Ex. 3. Solve qxr + (x + y)s + pyt + xy (xt – s2) = 1 – pq. qxr + (x + y)s + pyt + xy(rt – s2) = 1 – pq.

Sol. Given

...(1)

2

Comparing (1) with Rs + Ss + Tt + U(rt – s ) = V, we have R = qx,

S = x+y,

T = py,

Now, the =–qaudratic reduces to or

2

= + (x + y)= + xy = 0

U = xy 2

V = 1– pq ...(2)

and 2

= (UV+RT) + =US + U = 0 2

...(3) 2 2

= [qxpy+xy (1 – pq)] + =xy (x + y) + x y = 0 or

(= + x) (= + y) = 0

so that

= = –x , – y.

Let =1 = –x and =2 = –y. ...(4) Two intermediate integrals are given by the following two sets : Udy + =1T dx + =1Udp = 0 % ...(5) ∋ Udx + =2 Rdy + =2Udq = 0 ( and Udy + =2Tdx + =2Udp = 0 % ...(6) ∋ Udx + =1 Rdy + =1Udq = 0. ( Using (2) and (4), equation (5) and (6) reduce to xydy – xpydx – x2 ydp = 0 i.e., (xdp + pdx) – dy = 0 % ...(5A) and xydx – yqxdy – yxydq = 0 i.e., (ydq + qdy) – dx = 0 ∋( xydy – ypydx – y xy dp = 0 i.e., xdy – pydx – xydp = 0 % ...(6A) 2 ∋ and xydx – xyqdy – x ydq = 0 i.e., ydx – qxdy – xydq ( =0 We observe that (5A) can be integrated whereas (6A) cannot the integrated. So we shall obtain only one intermediate integral with help of (5A) : Integrating (5A), px – y = c1 and qy – x = c2 Hence the only intermediate integral of (1) is given by px – y = f(qy – x), where f is an arbitrary function. ...(7)

Created with Print2PDF. To remove this line, buy a license at: http://www.software602.com/

Monge’s Methods

9.49

A general solution of (1) can be obtained by supposing the arbitrary function f occuring in the intermediate integral (7) to be linear, giving px – y = m(qy – x) + n, where m and n are arbitrary constants or xp – myq = y – mx + n, which is in Lagrange’s form dx = dy = dz ...(8) ∃my x y ∃ mx # n From first and second fractions of (8), m(1/x)dx + (1/y)dy = 0 Integrating, m log x + log y = log a or xmy = a. ...(9) Chossing m, 1/m, 1 as multipliers, each fraction of (8) mdx # (1 / m) dy # dz mdx # (1 / m) dy # dz = ...(10) ) mx # (1 / m)(∃my) # y ∃ mx # n n Taking first fraction of (8) and (10), we have 1 n dx ) mdx # (1 / m) dy # dz or dz # dy # mdx ∃ dx = 0. x n m x Integrating, z + (1/m)y + mx – n log x = b, b being an arbitrary constant ...(11) From (9) and (11) the required general solution in z + (1/m)y + mx – log xn = ∗(xmy), ∗ being an arbitrary function 2 Ex. 4. Solve (rt – s ) – s(sin x + sin y) = sin x sin y. [Meerut 1999] 2 . . Sol. Given 0 r – s(sin x + sin y) + 0 t + (rt – s ) = sin x sin y. ...(1) Comparing (1) with Rr + Ss + Tt + U(rt – s2)= V, we have R = 0, S = – (sin x + sin y), T=0, U = 1, V = sin x sin y. ...(2) Now, the =–quadratic =2(UV + RT) + =US + U2 = 0. ...(3) 2 reduces to sin x sin y = – (sin x + sin y) = + 1 = 0 or (=sin x ∃ 1)(= sin y – 1) = 0 so that = = cosec x or cosec y. Let =1 = cosec x and =2 = cosec y. ...(4) Two intermediate integral are given by the following two sets : Udy + =1Tdx + =1Udp = 0 % ...(5) Udx + =2 Rdy + =2Udq = 0 ∋( and Udy + =2 Tdx + =2Udp = 0 % ...(6) Udx + =1 Rdy + =1Udq = 0 ∋( Using (2) and (4), equations (5) and (6) reduce to dy + cosec x dp = 0 % ∋ ...(5A) dx + cosec y dq = 0 ( dy + cosec y dp = 0 i.e., dp +sin y dy = 0 % ...(6A) ∋ and dx + cosec x dq = 0 i.e., dq + sin x dx = 0 ( We observe that (5A) cannot be integrated where as (6A) can be integrated. So we shall obtain only one intermediate integral with help of (6A). Integrating (6A) , p – cos y = c1 and q – cos x = c2. Hence the only intermediate integral of (1) is given by p – cos y = f(q – cos x), f being an arbitrary function. ...(7) A general solution of (1) can be obtained by supposing the arbitrary function f occuring in the the intermediate integral (7) to be linear, giving p – cos y = m[q – cos x] + n, m, n being arbitrary constants or p – mq = cos y – m cos x + n, which is in Lagrange’s from.

Hence here Lagrange axuiliary equations are

Its Lagrange’s auxiliary equations are

dy dx dz = = . 1 ∃m cos y ∃ m cos x # n

...(8)

Created with Print2PDF. To remove this line, buy a license at: http://www.software602.com/

9.50

or

Monge’s Methods

From the first two fractions of (8), dy + mdx = 0 so that y + mx = a. ...(9) Again, taking the first and third fractions of (8), we have dz = (cos y – m cos x + n)dx = [cos (a – mx) – m cos x + n]dx, as from (9), y = a – mx Intergrating, z = – (1/m) × sin (a – mx) – m sin x + nx + (1/m) × b mz + sin y + m2 sin x – mnx = b, b being an arbitrary constants ...(10) From (9) and (10), the required general solution of (1) is mz + sin y + m2 sin x – mnx = ! (y + mx), ! being an arbitrary function. Ex. 5. Solve xqr + (p + q)s + ypt + (xy – 1) (rt – s2) + pq = 0. Ans. z – log (x – m)n = !{(x – m)n (1 – my) 2 Ex. 6. Solve 2yr + (px + qy)s + xt – xy(rt – s ) = 2 – pq.

Ans. z + (1/m) × (a2 – mx2)1/2 + ( x / m ) × sin–1 ( x m / a) + 2mx = !(mx2 + y2) Ex. 7. Solve ar + bs + ct + e(rt – s2) = h, where a, b, c, e and h are constants. Sol. Comparing with Rr + Ss + Tt + U(rt – s2) = V, here R = a, S = b, T = c, U = e, V = h. The =–quadratic =2 (UV + RT) + =SU + U2 = 0 gives (ac + eh)=2 + =be + e2 = 0. ...(1) Let = = –e/m. ...(2) (1) reduces to m2 – bm + (ac + eh) = 0. ...(3) + Let m1 and m2 be the roots of (3). The first intermediate integral is given by Udy + =1Tdx + =1Udp = 0 , where =1= –e/m1 and Udx + =2Rdy + =2Udq = 0 , where =2 = – e/m2 i.e., e dy – (e/m1)× c dx – (e/m1) × e dp and ex – (e/m2) × a dy – (e/m2) × e dq = 0 i.e., c dx + edp – m1 dy = 0 and ady + edq – m2dx = 0. Integrating, cx + ep – m1y = c1 and ay + eq – m2 x = c2. So the first intermediate integral is cx + ep – m1y = !1(ay + eq – m2x). ...(4) Proceding as before, the second intermediate integral is cx + cp – m2y = ! 2(ay + eq – m1x). ...(5) Notice that p and q cannot be determined from (4) and (5). Hence we proced as follows : We also have, cx + ep – m2y = c3 ...(6) From (4) and (6), (m2 – m1)y = !1(ay + eq – m2x) – c3 ay + eq – m2x = ∗ 1{(m2 – m1)y + c3}] ...(7) + where ∗1 is the inverse function of !1. From (7). q = (1/e) × [– ay + m2x + !1{(m2 – m1)y + c3}] and from (6) p = (– cx + m2y + c3)/e. Putting these values in dz = pdx + qdy, we get edz = – (xdx – aydy + m2(xdy + ydx) + c3dx + !1{(m2 – m1)y + c3}dy Integrating, ez = – (1/2) × cx2– (1/2) × ay2 + m2xy + c3x + F{(m2 – m1)y + c3} + k.

EXERCISE Solve the following partial differential equation: 1. 3r + s + t + (rt – s2) = –9 [K.U. Kurukshetra 2004] 2 2 2. 3s – 2(rt – s ) = 2 3. 2r – 6s + 2t + (rt – s ) = 4 Objective problems 1. The equations R dpdy + T dqdx – V dxdy = 0 and Rdy2 – S dxdy + T dx2 = 0 are called Sol. Ans. Monge’s subsidiary equations. Refer Art. 9.2. [Meerut 2003]. 2. Monge’s method is used to solve a partial differential equation of (a) nth order (b) first order (c) second order (d) none of these [Agra 2007] Sol. Ans. (c). Refer Art 9.2

Created with Print2PDF. To remove this line, buy a license at: http://www.software602.com/

10 Transport Equation 10.1 INTRODUCTION The hyperbolic character of a system of first order differential equations exhibits in the fact that it is possible to have solutions whose derivatives are discontinuous and these discontinuities propagate along the characteristic curves. In this chapter we propose to use the above fact and derive a system of linear homogeneous ordinary differential equations known as transport equation. 10.2 An IMPORTANT THEOREM If the first order partial derivatives of a continuous function U (x, t), satisfying a system of quasi-linear equations of first order on both sides of a curve C in xt-plane, are discontinuous across a curve C, then the curve C must be a characteristic curve of the system of equations. Proof : Suppose that the given first-order system of n quasi-linear partial differential differential equations be given by or

Aij (!u j / !t ) Bij (!u j / !x)

Ci # 0,

i # 1, 2, ..., n;

A(!U / !t ) B (!U / !t ) C # 0, where the n components u1, u2, ..., un of the column vector U are dependent variables, A and B are n × n matrices and C is a n × 1 column vector. Let D be a domain in the xt-plane and let D1 and D2 be two portions of D separated by a curve C such that D1 is on the left and D2 on the right of C as shown in the adjoining figure. Let U1 be the genuine solution of (1) in the domain D1 and U2 that in the domain D2. Suppose that the limiting value Ul of U1 as we approach a point P on C from the domain D1 and the limiting value Ur as we approach P from the domain D2 exist and are such that Ul = Ur at every point of the curve C. Let a function U be defined in the domain D such that (U in D1 U #) 1 ∗U 2 in D2

j # 1, 2, ..., n ∃% & %∋

...(1)

t Ul Ur

D Pl

Pr U = U2(x, t)

U = U1(x, t) Dl O

C D2

x

...(2)

The function U given by (2) is a genuine solution of (1) in D1 and D2 respectively. The function U is continuous in D but its derivatives may be discontinuous across the curve C. Suppose that the limiting values of the derivatives of U as we approach P on the curve C from the two domains D1 and D2 exist. Also assume that these derivative, if discontinuous across the curve C, have only a finite jump across the curve C. Let the equation of the curve C be + ( x, t ) = 0 and let η( x, t ) be any other function independent + of such that + and η are sufficiently smooth and the Jacobian !(+ , η)/!( x, t ) , 0 in the domain D. Therefore, if we can introduce a new set of independent variables (+ , η) in place of (x, t), then U+ represents an exterior derivative and U η is a tangential derivative along the curve C. 10.1

Created with Print2PDF. To remove this line, buy a license at: http://www.software602.com/

10.2

Origin of partial differential equations

We have and

!U !+ !U !− ∃ # U + + x U− −x % !+ !x !− !x % & !U !+ !U !− Ut # # U + +t U − −t % %∋ !+ !t !− !t Ux #

...(3)

Let us now assume that the first order partial derivatives Ux and Ut are discontinuous across the curve C. Since the function U is continuous across C, its tangential derivative U− is also continuous across C. Hence from the above two relations (3), it follows that the exterior derivative

U + must be discontinuous across C. Let (U + )r and (U + )l be the limiting values of U + across C, as we approach a point P on C from the domains D1 and D2 respectively. Then, the jump [U + ] in

U + across C is given by [U + ] # (U + )r – (U + )l

....(4)

Since U− is continuous across the curve C, hence from (3), the jumps in the first order derivatives U x and Ut are related to [U + ] by the relations

[U x ] # [U + ] + x

[U t ] # [U + ] +t and ...(5) The quasi-linear equation (1) is valid everywhere in D except at the points on the curve C. Since all the terms appearing in it other than the first order derivatives are continuous across the curve C, hence taking the limit of (1) as we move from the region D1 to P and again as we more from the region D2 to P and then subtracting the equations so obtained, we obtain ( A +t

B + x )at P [U + ] # 0

Since [U + ] is not a zero vector, the matrix A+t

...(6)

B+ x must be singular on the curve C, i.e.,

at every point of the curve C, we have or

det ( A +t det(–. A

B +x ) # 0

∃ & B) # 0, where . # – +t / + x ∋

...(6)

Hence, it follows that C is a characteristic curve. Note 1. In (6), det X stands for determinant of the matrix X. Note 2. If the derivatives of a solution U of system (1) upto order (r / 1) are continuous across a curve C and the (r + 1) th derivatives are discontinuous across C, then differentiating (1) r times and then proceeding as discussed above, we can show that C is necessarily a characteristic curve. 10.3 GENERALISED OR WEAK SOLUTION [Allahabad 2003; G.N.D.U. Amritsar 2004; Kanpur 2003, 05) Consider a general first order quasi-linear hyperbolic system of first order equations A( x, t )U t

B ( x, t ) U x

H ( x, t ) J ( x, t ) # 0,

...(1)

where the elements of matrices A, B and H (each of order n × n) and column vector J (of order n × 1) are functions of x and t only. Also note that U t # !U / !t and U x # !U / !x. Let D be a domain in the xt-plane and let D1 and D2 be two portions of D separated by a curve C.

Created with Print2PDF. To remove this line, buy a license at: http://www.software602.com/

Origin of partial differential equations

10.3

Suppose we have a solution U which satisfies (1) in D1 and D2 separately but is itself discontinuous across C. It has been established that discontinuities in the solution cannot be discussed for every function U satisfying (1) in D1 and D2. However, such discontinuities can be discussed for a ‘‘generalised’’ or ‘weak’’ solution. To end, we first reduce the system (1) to the characteristic canonical form and note that (! / ! t )

. M ( x, t ) (! / !x) represents the directional derivatives along the characteristics of the Mth

field. Integrating it from a point PM to a point (0, 1), both lying on a characteristic of the Mth field, we obtain. WM (0, 1) # –

2

P

PM

(WM ) at PM

[ H M i { xM (t , 0, 1), t} Wi { xM (t , 0, 1), t}]dt

2

P

PM

J M {xM (t , 0, 1), t}dt , for M = 1, 2, ... n

H M i # [l ( M ) A{!r ( i ) / !t . M ( !r ( i ) / !x )} (l ( M ) H r (i ) )] /(l ( M ) A r ( M ) )

where

J M # (l ( M ) J ) /(l ( M ) A r (M ) )

and

...(2) ...(3) ...(4)

with no sum over M in these expressions and x = xM (t, 0 , 1 ) is the characteristic of Mth field through the point P. We can rewrite in compact form the expression for H M i in terms of the operator 1 3 A(! / !t )

B (! / !x ) H . Thus, (3) takes the form H M i # [l ( M ) 1 r (i ) ] /[l ( M ) A r ( M ) ]

...(5)

We now define a generalised or weak solution of (1) to be a function U(x, t) obtained from W1 , W2 ,..., Wn which satisfy the system of equations (2). 10.4 TRANSPORT EQUATION FOR A LINEAR-HYPERBOLIC SYSTEM [Calicut 2003 G.N.D.U. (Amritsar 2005; Kanpur 2004; Meerut 2005, 06, 10, 11] Consider a general first order quasi-linear hyperbolic system of first order equations A( x, t ) U t

B ( x, t ) U x

H ( x, t ) J ( x, t ) # 0,

...(1)

where the elements of matrices A, B and H(each of order n × n) and column vector J (of order n × 1) are functions of x and t only. Also note that U t # !u / !t and U x # !U / !x. Let D be a domain in the xt-plane and let D1 and D2 be two portions of D separated by a curve C. Suppose U ( x, t ) is a weak solution of (1) which is continuous in the domain D except on the curve C and is a genuine solution of (1) in the domains D1 and D2. We also suppose that the function U has a jump discontinuity across C. We now reduce the system (1) to the characteristic canonical form and note that (! / dt )

. M ( x, t ) (! / !x ) represents the directional derivatives along the characteristics of the Mth

field. Integrating it from a point PM to a point (0, 1), both lying on a characteristic of the Mth field, we have

WM (0, 1) # –

2

P

PM

[ H Mi {xM (t, 0, 1), t} Wi {xM (t , 0, 1), t ] dt

(WM )at PM –

2

P

PM

[ J M {xM (t , 0, 1), t} dt , for M = 1, 2, ... n

...(2)

Created with Print2PDF. To remove this line, buy a license at: http://www.software602.com/

10.4

Origin of partial differential equations

where

H M i # [l ( M ) A{!r (i ) / !t . M (!r ( i ) / !x )} l ( M ) Hr (i ) ] /[l ( M ) A r ( M ) ]

J M # (l ( M ) J ) /[l ( M ) A r (M ) ]

and

...(3) ...(4)

with no sum over M in these expressions and x # xM (t , 0, 1) is the characteristic of Mth field through the point P. In the case when the function U has a jump discontinuity t D across the curve C, the integrands on the right hand side of (2) are continuous functions of t except for a finite jump across C. On performing the integration in (2) along a characteristic of the Mth family, we find that the characteristic variable WM Pl P Pr is given by a continuous function of this curve. If a curve C is C not tangential to a characteristic of the Mth family, WM must be continuous across the curve C. However, according to our Dl D2 assumption at least one of W1 , W2, ..., Wn must be discontinuous across the curve C. Therefore, it follows that C, the curve of discontinuity, must be a characteristic curve O x of jth family (say), and the jump in all characteristic variables Wi, i , j, must be zero across the carve C. Now, suppose that the curve of discontinuity C is a characteristic curve of the jth family then, the jump [Wi] in Wi satisfies [Wi ] # 0, for

i# j

[W j ] # 0

...(5)

[U ] # r ( j ) [W j ], on sum over j

...(6)

and

Again, we have Re-writing the equation, we have

(! WM / !t ) . M (!WM / !x) C1 M i Wi

J M # 0,

M = 1, 2, ..., n

...(7)

Consider two points Pl and Pr on the two sides of C in the regions D1 and D2 respectively as shown in the figure. Taking limit as both these points tend to P on the curve C and substracting the results so obtained we obtain d 4W j 5 # – H j j [W j ], no sum over j dt 6 7

where

d / dt 3 (! / !t ) (. j )(! / !x)

...(8) ...(9)

The above equation (8) is known as the transport equation. Along a given characteristic curve x = x j (t) of the j the family, the function

H j j ( x, t ) # H j j ( x j (t ), t ) is a function of t only.. Thus, the transport equation (8) is a linear homogeneous ordinary differential equation of first order and determines the vibration of [W j ], jump in W j along a characteristic curve of the jth family. From the properties of solutions of linear homogeneous ordinary differential equations, it follows that if there is a discontinuity in U at some point of a characteristic curve C, the discontinuity in U remains non-zero at every point on the curve. Note. In order to obtain the transport equation for the discontinuities in the derivatives of U of order n (n / 1), differentiate (7) n (n / 1) times and proceed in exactly same manner as above.

Created with Print2PDF. To remove this line, buy a license at: http://www.software602.com/

Origin of partial differential equations

10.5

EXERCISE 1. Write short note on the transport equation for a linear hyperbolic system of first order equations. (Calicut 2003; Meerut 2005; 06) 2. When all the characteristic velocities .i are different from zero, prove that the first order quasi-linear hyperbolic system A( x, t , U ) (!U / !t ) B( x, t , U ) (!U / !x) C ( x, t , U ) # 0 can be reduced to a diagonal canonical system of 2n equations (!U / !t ) – RW # 0 and (!U / !t )

A(!W / !x ) F # 0, where the coefficients A, R and F are functions of x, t, U and W

3. Consider the hyperbolic system

ut

( x t )vx # 0,

( x t )vt

ux # 0

Show that the variation in jump [ v] along the characteristic curve ( x – t ) # C (C = constant) is given by

[ v] # A /(2t c)1/ 2 , A being a constant. Derive also the transport equation of discontinuities in the first order partial derivatives of u and v.

Created with Print2PDF. To remove this line, buy a license at: http://www.software602.com/

MISCELLANEOUS PROBLEMS BASED ON THIS PART OF THE BOOK Ex. 1. The solution of xux + yuy = 0 is of the form (a) f(y/x) (b) f (y + x) (c) f (x – y) (d) f (xy) [GATE 2008] Sol. Ans. (a). Given xux + yuy = 0 …(1) which is in the form of Lagrange equation Pp + Qq = R, with u in place z. Hence, the Lagrange’s auxiliary equations for (1) are given by (dx)/x = (dy)/y = du/0 …(2) Taking the first two fractions of (2), (1/y) dy – (1/x) dx = 0 Integrating, log y – log x = log c1 or y/x = c1 …(3) Again, the last fraction of (2) yields du = 0 so that u = c2 …(4) From (3) and (4), the required solution is u = f (y/x). Ex. 2. Solve x(y2 + z)p + y(z + x2)q = z(x2 – y2) [Madurai Kamraj 2008] Sol. Do like Ex. 6, page 2.10. Here Lagrange’s auxiliary equations are dx

or

x( y

2

z)

!

dy 2

y( z x )

!

dz

… (1)

2

z (x # y2 )

Choosing 1/x, – (1/y), 1/z as multipliers, each fraction of (1) =

(1/ x ) dx # (1/ y )dy (1/ z )dz y

2

2

z # (z

2

x ) x #y

!

2

(1/ x )dx # (1/ y ) dy (1/ z )dz 0



or

(1/x) dx – (1/y) dy + (1/z) dz = 0 so that log (xz/y) = log c1 or Choosing x, – y, – 1 as multipliers each fraction of (1) =

xdx # ydy # dz 2

x (y

2

2

2

z) # y (z

2

2

x ) # z(x # y )

!

log x – log y + log z = log c1 (xz)/y = c1 …(2)

xdx # ydy # dz 0

xdx – ydy – dz = 0 or 2xdx – 2ydy – 2dz = 0 ∃ 2 2 Integrating, x – y – 2z = c2, c2 being an arbitrary constant …(3) From (2) and (3), the required solution is given by (xz)/y = % (x2 – y2 – 2z), % being an arbitrary function Ex. 3. If the partial differential (x – 1)2 uxx – (y – 2)2 uyy + 2xux + 2yuy + 2xyu = 0 is parabolic in S & R 2 but not in R 2 \ S , then S is

∋ (c) ∋( x, y ) ) R

(

2 (a) ( x, y ) ) R : x ! 0 or y ! 2 2

(

: x !1

∋ (d) ∋( x, y ) ) R

(

2 (b) ( x, y ) ) R : x ! 1 and y ! 2 2

(

:y!2

[GATE 2008]

Sol. Ans. (a). Refer Art. 8.1. Here R is the set of all real numbers Ex. 4. Find the complete integral of xp + 3yq = 2(z – x2q2) [Delhi Maths (H) 2008] Sol. Here given equation is Charpit’s auxiliary equations are

f (x, y, z, p, q) = xp + 3yq – 2z + 2x2q2 = 0 fx

… (1)

dp dq dz dx dy ! ! ! ! p fz f y q f z # p f p # q fq # f p # fq

M.1

Created with Print2PDF. To remove this line, buy a license at: http://www.software602.com/

M.2

i.e.,

Miscellaneous problems based on this part of the book

dp 2

p 4 xq # 2 p

!

dq dz dx dy ! ! ! , using (1) 3q # 2 q # px # q (3 y 4 x 2 q ) # x # (3 y 4 x 2 q )

…(2)

Taking the second and fourth fractions of (2), we have (1/q) dq + (1/x) dx = 0 so that log q + log x = log a, giving qx = a so that q = a/x, a being an arbitrary constant …(3) Substituting the value of q given by (3) in (1), we have xp + (3ya)/x – 2z + 2a2 = 0 or xp = 2(z – a2) – (3ya)/x Thus, p = 2(z – a2)/x – (3ya)/x2. …(4) Substituting values of q and p given by (3) and (4) in dz = pdx + qdy, we get dz = {2(z – a2)/x – (3ya)/x2}dx + (a/x) dy or x2dz = 2x (z – a2) dx – 3yadx + axdy or x2dz – 2x(z – a2) dx = axdy – 3yadx or [x2dz – 2x (z – a2)dx]/x4 = ax–3dy – 3ayx–4 dx or d{(z – a2)/x2} = d (ayx–3) Integrating, (z – a2)/x2 = (ay)/x3 + b, b being an arbitrary constant or z = a(a + y/x) + bx2, which is the required solution. Ex.5. Find the general integral of the partial differential equation px(z – 2y 2 ) = (z – qy) (z – y2 – 2x3). Also, find the particular integral which passes through the straight line x = –1, z = 1. (Delhi B.A. (Prog.) 2009) Sol. Re-writing the given partial differential equation, we have px (z – 2y2) + qy(z – y2 – 2x3) = z (z – y2 – 2x3) ... (1) Hence the usual Lagrange’s, subsidiary equations are given by dx 2

x( z # 2 y )

!

dy 2

3

y( z # y # 2 x )

!

dz

…(2)

z ( z # y 2 # 2 x3 )

Taking the last two fractions of (1), we get (1/y)dy = (1/z)dz Integrating, log y = log z + log c1, c1 being an arbitrary constant Thus, y = c1z. Next, taking the first and third fractions of (2) and using (3), we obtain dx x( z # 2c12 z 2 )

=

dz

dz z # c12 z 2 # 2 x 3 ! dx x (1 # 2c12 z )

or

z ( z # c12 z 2 # 2 x 3 )

...(3)

Re-writing it,

(1 # 2c12 z ) (dz / dx) – ( z # c12 z 2 ) ∗ (1/ x) ! #2 x 2

Putting z # c12 z 2 ! v

so that

…(4)

(1 # 2c12 z )(dz / dx) ! d v / dx, (4) reduces to

(d v / dx) # (1/ x)v ! #2 x2

…(5)

# log x whose integrating factor is e + ( #1/ x ) dx = e = x–1 and hence solution of (5) is

2 #1 v × x–1 = + {(#2 x ) ∗ x } dx c2 = –x2 + c2, c2 being an arbitrary constant

or

( z # c12 z 2 ) / x x2 ! c2

or

(z – y2)/x + x2 = c2, using (3)

…(6)

The required general integral is given by (3) and (6). We now find the required particular integral. To this end, replacing x by –1 and z by 1 in (3) and (6), we obtain y = c1 and – (1 – y2) + 1 = c2 ...(7)

Created with Print2PDF. To remove this line, buy a license at: http://www.software602.com/

Miscellaneous problems based on this part of the book

M.3

c12 ! c2

Eliminating y between two relations of (7), we have

…(8)

Substituting the values of c1 and c2 given by (3) and (6) in (8), the required particular integral is given by y2/z2 = (z – y2) / x + x2 or y2x = z2(z – y2 + x3) Ex.6. Solve the partial differential equation z = px + qy + 3p – 2q by Lagrange’s method as well as Charpit’s method. Hence or otherwise give two different solutions of the above partial differential equation passing through (–3, 2, 0). (Delhi B.A. (Prog). II 2009) Sol. Solution of the given equation by Lagrange’s method: Re-writing the given equation, (x + 3)p + (y – 2)q = z …(1) Here the usual Lagrange’s subsidiary equations are given by (dx)/(x + 3) = (dy)/(y – 2) = (dz) / z …(2) Taking the first two fractions of (2), (dx)/(x + 3) = (dy)/(y – 2) Integrating, log(x + 3) = log(y – 2) + log a or (x + 3) = a (y – 2) …(3) Next, taking the last two fractions of (2), (dy)/(y – 2) = (dz)/z Integrating, log (y – 2) + log b = log z or b (y – 2) = z ...(4) From (3) and (4), (x + 3)/a = (y – 2)/1 = (z – 0)/b, a and b being arbitrary constants …(5) which is the required solution of the given equation passing through (–3, 2, 0). Solution of the given equation by Charpit’s method: Let f(x, y, z, p, q) = (x + 3)p + (y – 2)q – z = 0 …(6) Here Charpit’s auxiliary equations f x

dq dz dx dy dp = = = = f qf # pf # qf # f # fq pf z y z p q p

dz dx dy dp dq ! = = = # p( x 3) # q( y # 2) #( x 3) #( y # 2) 0 0

yield

Hence, dp = 0 so that p = c1, c1 being an arbitrary constant From (6) and (7), (x + 3)c1 + (y – 2)q – z = 0 ∃ q = {z – (x + 3)c1}/(y – 2) Substituting the values of p and q given by (7) and (8), we have dz = pdx + qdy = c1dx + [{z – (x + 3)c1}/(y – 2)]dy or

or

dz #

z ( x 3)c1dy dy ! c1dx # y#2 y#2 , z − , c ( x 3) − d. !d. 1 / 0 y # 21 0 y # 2 /1

or

( y # 2) dz # z dy ( y # 2)

giving

2

!

…(7) …(8)

c1 ( y # 2) dx c1 ( x 3) dy ( y # 2)2

z c ( x 3) ! 1 c2 y#2 y#2

Thus, z = c1(x + 3) + c2 (y – 2), which is the required second solution of the given equation passing through (–3, 2, 0). Ex. 7. Define the singular integral of first order partial differential equation. Is it true that singular integral always exists? Justify your answer. [Delhi Math (Hons.) 2009] Hint: Refer Art. 3.1. Ex. 8. Write the form of the solution of the equation F (D, D’) = 0, where F (D, D’) is not reducible. [Delhi Math (Hons). 2009] Sol. Let z = ehx + ky be a trial solution of the given equation. Then, the required solution is

Created with Print2PDF. To remove this line, buy a license at: http://www.software602.com/

M.4

Miscellaneous problems based on this part of the book

z = 2 Ai ehi x i

ki y

, where Ai, hi and ki are arbitrary constants,

and hi, ki are connected by the relation F(hi, ki) = 0. Ex. 9. Solve z = px + qy + p2 + q2. [Kanpur 2009] 2 Sol. Refer Art 3.12. The complete integral is z = ax + by + a + b2 … (1) Singular integral. Differentiating (1) partially w.r.t. ‘a’ and ‘b’, we get 0 = x + 2a and 0 = y + 2b …(2) From (2), a = – (x/2) and b = –(y/2). Substituting these values of a and b in (1), we get z = – (x2/2) – (y2/2) + x2/4 + y2/4 or 4z + x2 + y2 = 0. General integral Take b ! %(a), where % is an arbitrary function Then, (1) yields

z = ax + y % (a) + a2 + [ % (a)]2

…(3)

0!x

…(4)

Differentiating (3) partially w.r.t. ‘a’,

y %3 (a) 2a 2% (a)% 3(a)

The general integral is obtained by eliminating a between (3) and (4). Ex. 10. Classify the following partial differential equation into elliptic, parabolic or hyperbolic and find its degree and order x (y – x) r – (y2 – x2)s + y(y – x) t + (y + x) (p – q) = 0. Hint. Use Art 1.3, 1.4 and 8.1. [Delhi BA (Prog.) II 2009] Ans. The given equation is hyperbolic, its degree is one and its order is two. Ex. 11. Find the characteristic strips of the equation xp + yq – pq = 0 and then find the equation of the integral surface through the curve z = x/2, y = 0 [Meerut 2011] Sol. Given equation is xp + yq – pq = 0 ...(1) We are to find its integral surface passing through the given curve, namely z = x/2, y=0 …(2) Re-writing (2) in parametric form, we have y = 0, z = 4 / 2; x!4, 4 being a parameter Let the initial values of x0, y0, z0, p0 and q0 of x, y, z, p and q be taken as

…(3)

x0 = x0 ( 4 ) = 4 y0 = y0 (4 ) ! 0, z0 ! z0 (4) ! 4 / 2 …(4 A) Let p0 and q0 be the initial values of p and q corresponding to the initial values of x0, y0, z0. Since the initial values x0, y0, z0, p0 and q0 satisfy (1), we have x0p0 + y0q0 – p0q0 = 0 Also, we have

4p0 # p0 q0 ! 0

or

q0 ! 4 , using (4A)

…(5)

z03 (4) = p0 x03 (4 ) q0 y03 (4) p0 = 1/2, using (4A)

…(6)

Thus, from (5) and (6), p0 = 1/2 and q0 = 4 Collecting relations (4 A) and (4 B) together, initial values are given by

…(4 B)

so that

1/2 = p0 + 0

or

giving

x0 = 4 , y0 = 0, z0 = 4 / 2, p0 = 1/2 and q0 = 4 , when t = t0 = 0 Re-writing (1), let f(x, y, z, p, q) = xp + yq – pq = 0 The usual characteristic equations of (8) are given by dx/dt = 5f / 5p = x – p dy / dt = 5f / 5q = y – p dz/dt = p(5f / 5p) q (5f / 5q) = p(x – q) + q (y – p) = –pq, using (1)

…(7) …(8) …(9) …(10) …(11)

Created with Print2PDF. To remove this line, buy a license at: http://www.software602.com/

Miscellaneous problems based on this part of the book

M.5

dp/dt = #(5f / 5x ) # p(5f / 5z ) = –p – (p × 0) = –p

…(12)

dq/dt = #(5f / 5y ) # q (5f / 5z ) = –q – (q × 0) = – q

…(13)

From (12), (1/p)dp = –dt so that log p – log c1 = – t –t Thus, p = c1e , c1 being an arbitrary constant Similarly, (13) yields q = c2e–t, c2 being an a arbitrary constant p0 = c1e # t0

Using initial values (7), (14) yields Hence, (14) reduces to

p = (1/2) × e

Using initial values (7), (15) yields

q0 = c2 e #t0

c1 = 1/2

giving –t

…(16) c2 = 4

giving

q = 4 e–t

Hence, (15) reduces to #t dx/dt = x # 4e

From (9) and (17),

…(14) …(15)

...(17) dx/dt – x = #4 e# t ,

or

whose integrating factor = e + ( #1) dt ! e #t and hence its solution is given by xe # t = c3

#t #t +{(#4e ) ∗ e } dt , c3 being an arbitrary constant

xe # t = c3

or

6 4 / 27 ∗ e#2t x0 e# t0 = c3 + (4 / 2) ∗ e#2 t0

Using initial values (7), (18) yields 4 = c3 + 4 / 2

or

…(18)

c3 = 4 / 2.

so that

xe#t ! (4 / 2) ∗ (1 e#2t )

Hence, (18) yields

x = (4 / 2) ∗ et (1 e#2t )

or

–t

dy/dt = y – (e )/2

Now, from (10) and (16),

…(19) or

–t

dy/dt – y = –(e )/2

+ ( #1) dt ! e # t and hence its solution is given by whose integrating factor = e

or

or

ye–t = c4 + +{(#e#t / 2) ∗ e#t }dt, c4 being an arbitarary constant ye–t = c4 + (1/4) × e–2t Using initial values (7), (20) yields 0 = c4 + 1/4 Hence, (20) reduces to Thus,

z = (4 / 4) ∗ e#2t

Using initial values (7), (22) yields 0 4 / 2 = ( 4 / 4 ) × e + c5

Hence, (22) reduces to

…(21)

(1/z)dz = #(4 / 2) ∗ e#2t dt

Thus,

or

y0 e #t0 = c + (1/4) × e #2t0 4 so that c4 = – (1/4) ye–t = (1/4) × (e–2t – 1) y = (1/4) × et ( e–2t – 1)

dz/dt = #(e# t / 2) ∗ (4e#t ) = – (4 / 2) ∗ e#2t

From (11), (16) and (17),

Integrating,

... (20)

c5 , c5 being an arbitrary constant z0 = (4 / 4) ∗ e #2t0

so that z = (4 / 4) ∗ (e#2t 1)

…(22)

c5

c5 = 4 / 4. …(23)

Created with Print2PDF. To remove this line, buy a license at: http://www.software602.com/

M.6

Miscellaneous problems based on this part of the book

The required characteristics of (1) are given by (19), (21) are (23) In order to obtain the desired integral surface of (1), we now proceed to eliminate two parameters t and 4 from (19), (21) and (23). From (19) and (23), we have x/z = 2et giving et = x/2z. …(24) t t From (21), y = (1/4) × (1/e – e ) = (1/4) × (2z/x – x/2z), using (24) or 8xyz = 4z2 – x2, which is the required integral surface of (1). Ex.12. Prove that for the equation z + px + qy – 1 – pq x2y2 = 0 the characteristic strips are given by x = (B + Ce– t)–1 , y = (A + De– t)–1, z = E – (AC + BD)e– t, p = A(B + Ce– t)2 , q = B(A+ De– t)2, where A, B, C, D and E are arbitrary constants. Hence, find the integral surface which passes through the line z = 0, x = y. [I.A.S 2001] Sol. The given equation is z + px + qy – 1 – pq x2y2 = 0 …(1) 2 2 Let f(x, y, z, p, q) = z + px + qy – 1 – pq x y …(2) Then, the characteristic equations of (1) are given by dx/dt = 5f / 5p ! x # qx2 y 2

…(3)

2 2

dy/dt = 5f / 5q ! y # px y

…(4)

2 2 2 2 dz/dt = p(5f / 5p ) q( 5f / 5q ) ! p( x # qx y ) q( y # px y ) = px + qy – 2pqx2y2 2

…(5)

2

dp/dt = #(5f / 5x ) # p(5f / 5z ) = –(p – 2 pqxy ) – p = –2p(1 – qxy )

…(6)

dq/dt = #(5f / 5y ) # q (5f / 5z ) = –(q – 2pqx2y) – q = –2q (1 – px2y)

…(7)

From (3) and (6), (1/x) (dx/dt) = –(1/2p) (dp/dt) or (2/x) dx + (1/p) dp = 0 Integrating, 2 log x + log p = log A or x2p = A, A being an arbitrary constant …(8) From (4) and (7), (1/y) (dy/dt) = (–1/2q) (dq/dt) or (2/y)dy + (1/q)dq = 0 Integrating as before, y2q = B, B being an arbitrary constant …(9) 2 –2 –1 From (3) and (9), dx/dt = x – Bx or x (dx/dt) – x = –B …(10) Putting x–1 = v and –x–2(dx/dt) = d v / dt, (10) reduces to #(d v / dt ) # v ! # B

or

d v / dt v ! B,

whose integrating factor is e + dt , i.e., et and hence its solution is given by vet = C + + Bet dt , C being an arbitrary constant et / x ! c Bet or 1/x = Ce–t + B or Similarly, (4) and (8) yield dy/dt = y – Ay2 or Putting y–1 = u and –y–2(dy/dt) = du/dt, (12) yields – (du/dt) – u = – A or

x = (B + Ce–t)–1 y–2(dy/dt) – y–1 = –A

…(11) …(12)

du/dt + u = A

+ dt whose integrating factor is e , i.e., et and hence its solution is given by

uet = D + Aet dt , D being an arbitrary constant or or or

or 1/y = De–t + Aor y = (A + De–t)–1 et / y ! D Aet Using (8) and (9), (5) yields dz/dt = A/x + B/y – 2AB –t –t dz/dt = A(B + Ce ) + B(A + De ) –2AB, using (11) and (13) dz/dt = (AC + BD)e–t or dz = (AC + BD) e–t dt –t Integrating, z = E – (AC + BD)e , E being an arbitrary constant

…(13)

…(14)

Created with Print2PDF. To remove this line, buy a license at: http://www.software602.com/

Miscellaneous problems based on this part of the book

M.7

–2

–t 2

From (8) and (11), p = Ax = A(B + C e ) …(15) –2 –t 2 From (9) and (13), q = By = B(A + De ) …(16) The required characteristics are given by (11), (13), (14), (15) and (16). We now proceed to find the required integral surface passing through the line given by z=0 and x=y …(17) x = 4, y = 4, z = 0, 4 being a parameter …(18) Let the initial values x0, y0, z0, p0, q0 of x, y, z, p, q be taken as Re-writing (17), x0 = x0 (4) ! 4,

y0 ! y0 (4 ) ! 4,

z0 ! z0 (4) ! 0

…(19)

Let p0, q0 be the initial values of p, q corresponding to the initial values x0, y0, z0. Since the initial values statisfy (1), we have

z0

p0 x0

q0 y0 # 1 # p0 q0 x02 y02 ! 0

or

4 ( p0

Thus,

q0 ) ! p0 q0 4 4 1

…(20)

z03 (4) = p0 x03 (4 ) q0 y03 (4)

Also, we have 0 = p0 + q0

so that,

p0 4 q0 4 # 1 # p0 q0 4 4 ! 0, using (19)

p0 q0 4 4 1 ! 0

Using (21), (20) yields

p0 = 1/ 4 2

Thus,

q0 = –p0, using (19)

giving

so that

…(21)

# p02 4 4 1 ! 0 , using (21)

giving

q0 = #(1/ 4 2 ) , using (21)

…(22)

Using initial values x = x0 = 4 , t = t0 = 0, (11) reduces to –1 4 = (B + C)

B + C = 1/ 4

so that

…(23)

Using initial values y = y0 = 4 , t = t0 = 0, (13) reduce to –1 4 = (A + D)

A + D = 1/ 4

so that

…(24)

Using initial values p = p0 = 1/ 4 2 , t = t0 = 0, (15) reduces to p0 = A(B + C)2

1/ 4 2 ! A ∗ (1/ 4) 2 , by (23)

or

so that

A = 1 …(25)

Using initial values q = q0 = #(1/ 4 2 ) , t = t0 = 0, (16) reduces to q0 = B (A + D)2

or

2 #(1/ 4 2 ) = B × (1/ 4 ) , by (24)

–1 + C = 1/ 4

From (23) and (26),

so that

so that

C = 1 + 1/ 4

From (24) and (25), 1 + D = 1/ 4 so that Using the initial values z = z0 = 0, t = t0 = 0, (14) reduces to 0 = E – (AC + BD)

or

E ! 1 1/ 4 # (1/ 4 # 1)

B = –1

or

…(26) …(27)

D = ( 1/ 4 ) – 1

…(28)

E=2

…(29)

Substituting the values of A, B, C, D and E given by (25), (26), (27), (28) and (29) in (11), (13) and (14), we obtain

x ! {#1 (1/ 4 1)e#t }#1

…(30)

y ! {1 (1/ 4 –1)e#t }#1

…(31)

#t z ! 2 # {1 1/ 4 # (1/ 4 # 1)}e# t = 2(1 # e )

…(32)

In order to obtain the required surface, we now eliminate 4 from (30), (31) and (32).

Created with Print2PDF. To remove this line, buy a license at: http://www.software602.com/

M.8

Miscellaneous problems based on this part of the book

From (30),

x–1 = #1 (1/ 4 1)e#t

so that

1/x + 1 = (1/ 4 1)e# t

…(33)

From (31),

y–1 = 1 + (1/ 4 # 1)e#t

so that

1/y – 1 = (1/ 4 # 1)e#t

…(34)

–t

–t

Subtracting (34) from (33), 1/x – 1/y + 2 = 2e or 1/x – 1/y = –2(1 – e ) From (32) and (35), we get 1/x – 1/y = –z which is the required integral surface.

…(35)

Ex. 13. The general solution of the partial differential equation 5 2 z / 5x5y ! x

y is of the

form (a) (1/ 2) ∗ xy ( x

(b) (1/ 2) ∗ xy ( x # y ) F ( x ) G ( y )

y ) F ( x) G ( y )

(c) (1/ 2) ∗ xy ( x # y ) F ( x ) G( y )

(d) (1/ 2) ∗ xy ( x

y ) F ( x) G ( y )

(GATE 2010)

Sol. Ans. (a). Integrating the give equation w.r.t ‘x’, we get

5z / 5y ! x 2 / 2 xy g ( y ), g ( y) being an arbitrary function of y. Integrating the above equation w.r.t. ‘y’, we get

z ! ( x 2 y) / 2 ( xy 2 ) / 2 G ( y) F ( x) ,

where

+

G ( y ) ! g ( y)dy

Ex. 14. Find whether the following is hyperbolic, parabolic or elliptic : (i) x2 r # y 2t # px # qy ! x 2 (ii) x2 r (5 / 2) ∗ xys (iii) 5 2 u / 5t 2 (iv) u xx

u yy

[Delhi B.A. (Prog) II 2010, 11]

y 2t xp

yq ! 0

[Delhi B.A. (Prog) II 2010]

5 2 u / 5x 5t 5 2 u / 5x 2 ! 0

[Meerut 2010]

u zz ! (1/ c 2 ) ∗ (5u / 5t )

(v) (1 # x 2 )r # 2 xys (1 # y 2 )t

[Meerut 2007, 10]

xp 3x 2 yq ! 0

[Ravishankar 2010]

(vi) 5 2 z / 5x 2 ! x 2 (5 2 z / 5y 2 )

[Bhopal 2010]

(vii) 5 2u / 5t 2

[Meerut 2011]

(5u / 5x) (5u / 5t ) 52u / 5t 2 ! 0

Hint. Use Articles 8.1, 8.2 and 8.2A. Ans. (i) Hyperbolic (ii) Hyperbolic (iii) Elliptic (iv) Parabolic (v) Hyperbolic if x2 + y2 > 1, parabolic if x2 + y2 = 1, elliptic if x2 + y2 < 1 (vi) Hyperbolic (vii) Elliptic Ex. 15. The

partial

differential

y ( y # 1)2 (5 2 z / 5y 2 ) x(5z 5x)

equation

x2 (5 2 z / 5x2 ) # ( y 2 # 1) x(5 2 z / 5x 5y)

y(5z / 5y) ! 0 is hyperbolic in a region in xy–plane if

(a) x 8 0 and y = 1 (b) x = 0 and y 8 1 (c) x 8 0, and y 8 1 (d) x = 0 and y = 1.[GATE 2011] Sol. Ans (c) The given can be re-written as

x2 r # x( y 2 # 1) s

y( y # 1) 2 t xp

yq ! 0

... (1)

Comparing (1) with Rr + Ss + Tt + f (x, y, z, p, q) = 0, we have R = x2, S ! # x( y 2 # 1) and T ! y( y # 1)2 . Now, in order that (1) may be hyperbolic we must have S 2 # rRT 9 0, i.e.,

x2 ( y 2 # 1)2 # 4 x 2 y( y # 1) 2 9 0

or

x2 ( y # 1)2 ( y 1)2 # 4 x2 y ( y # 1)2 9 0

Created with Print2PDF. To remove this line, buy a license at: http://www.software602.com/

Miscellaneous problems based on this part of the book

or

x2 ( y # 1)2 {( y 1)2 # 4 y} 9 0

M.9

x2 ( y # 1) 2 ( y # 1)2 9 0,

or

... (2)

Which is true when x 8 0 and y 8 1. Ex. 16. The integral surface for the Cauchy problem 5z / 5x 5z / 5y ! 1 which passes through the circle z = 0, x2 + y2 = 1 is (a) x2

y 2 2 z 2 2 zx # 2 yx # 2 yz # 1 ! 0

(c) x2

y2

2 z 2 # 2 zx # 2 yz # 1 ! 0

(b) x2

(d) x2

y2

y2

2 z 2 2 zx # 2 yz # 1 ! 0

2 z 2 2 zx 2 yz # 1 ! 0 [GATE 2011]

Sol. Ans. (c) In usual symbols, the given equation is

p+q=1

... (1)

(dx ) / 1 ! (dy ) / 1 ! (dz ) / 1

Lagrange’s auxiliary equation of (1) are

Taking the first two fractions of (2), we get dx – dy = 0 Integrating (3), x – y = c–1, c1 being an arbitrary constant Next, taking the first and third fractions of (2), we get dx – dz = 0 Integrating (5), x – z = c2, c2 being an arbitrary constant The given curve is defined by x2 + y2 = 1, z = 0 Putting z = 0 in (6), we have x = c2, Now, from (4) and (8), we have c2 – y = c1 so that y = c1 – c2 Substituting the values of x and y given by (8) and (9) in (7), we obtain

c22 (c2 # c1 )2 ! 1

( x # z )2

or

x2

or

y2

( y # z )2 ! 1,

... (2) ... (3) ... (4) ... (5) ... (6) ... (7) ... (8) ... (9)

using (4) and (6)

2 z 2 # 2 zx # 2 yz # 1 ! 0.

Ex. 17. The integral surfaces satisfying the partial differential equation

(5z / 5y) z 2 (5z / 5y ) ! 0 and spassing through the straight line x = 1, y = z is (a) ( x # 1) z z 2 ! y 2

(b) x2

y 2 # z2 ! 1

(c) ( y # z ) x x2 ! 1

(d) ( x # 1) z 2

z!y

[GATE 2012]

Sol. Ans. (d). Given p + z2q = 0, where

p ! 5z / 5x, q ! 5z / 5y

Lagrange’s auxiliary equation (1) are

(dx) /1 ! (dy) / z 2 ! (5z ) / 0

From third fraction of (1),

dz = 0

so that

... (1)

z=c

Using (3), from first and second fractions of (2), (dx) /1 ! (dy) / c12 or

c12 x # y ! c2

Integrating,

... (2) 1

c12 dx # dy ! 0

z 2 x # y ! c2 , as c1 ! z

or

... (3)

... (4)

In order to get the required integral surfaces, we shall use method II given on page 2.28. The given straight line is represented by x = 1, y=z ... (5) Using (5) in (3) and (4), we get

y ! c1

Eliminating y from the two equation of (6), we get

and

y 2 # y ! c2

c12 # c1 ! c2

... (6) ... (7)

Substituting the values of c1 and c2 given by (3) and (4) in (7), we get

z2 # z ! z2 x # y

or

( x # 1) z 2

z 2 ! y , which is the required integral surface

Created with Print2PDF. To remove this line, buy a license at: http://www.software602.com/

M.10

Miscellaneous problems based on this part of the book

1 Ex.18. The expression D 2 # D 2 sin( x # y ) is equal to x y (a) #( x / 2) ∗ cos( x # y )

(b) #( x / 2) ∗ sin( x # y ) cos( x # y )

(c) #( x / 2) ∗ cos( x # y ) sin( x # y )

(d) (3x / 2) ∗ sin( x # y )

[GATE 2012]

Sol. Ans. (a). Here note that Dx and Dy stand for D and D 3 respectively. For solution, processd as in Ex. 7(d). Here, we wish to find only P.I. Thus,

1 Dx2

# Dy2

sin( x # y ) !

1 2

D # D 32

sin( x # y) !

1 1 sin( x # y) D D3 D # D3

1 1 = D # D3 1 # (#1)) + sin v dv , where v ! x # y

[Using formula (i) page 4.9]

1 1 1 1 1 x cos ( x # y ) = 2 D D 3 [# cos( x # y )] ! 2 (#1) D # (1) ∗ D cos( x # y ) ! 2 ∗ (#1)1 ∗ 1!

[Using formula (ii), page 4.9] ! #( x / 2) ∗ cos( x # y )

Created with Print2PDF. To remove this line, buy a license at: http://www.software602.com/